머리말 KAIST 수학문제연구회는한국수학올림피아드가처음탄생할무렵인 988년부터지금까지다양한영재교육프로그램을운영하거나적극적으로기여해왔습니다. 한국수학올림피아드통신강좌와계절학교등의교육프로그램, 대전 충남지역중학생영재수학교실, KAIST Cyber영재교육등이그예들입니다. 이

Size: px
Start display at page:

Download "머리말 KAIST 수학문제연구회는한국수학올림피아드가처음탄생할무렵인 988년부터지금까지다양한영재교육프로그램을운영하거나적극적으로기여해왔습니다. 한국수학올림피아드통신강좌와계절학교등의교육프로그램, 대전 충남지역중학생영재수학교실, KAIST Cyber영재교육등이그예들입니다. 이"

Transcription

1 hkmo 차시험대비 i 엠제곱 (M ) 실전수학올림피아드 80 고등학생중 / 고급풀이편 고봉균 KAIST 수학문제연구회

2 머리말 KAIST 수학문제연구회는한국수학올림피아드가처음탄생할무렵인 988년부터지금까지다양한영재교육프로그램을운영하거나적극적으로기여해왔습니다. 한국수학올림피아드통신강좌와계절학교등의교육프로그램, 대전 충남지역중학생영재수학교실, KAIST Cyber영재교육등이그예들입니다. 이런활동의결과들을수학잡지 MathLetter를통해담아내어왔으며, 수학경시준비를위한교재를따로만들어달라는요청에부응하여 003년에엠제곱 (M ) 수학올림피아드셈본중학생초급 / 중급 / 고급을펴내었습니다. 그후셈이의문제해결기법, 셈본고등학생초급,BalticWay 팀경시대회등출판활동이꾸준히이어졌고, 수학올림피아드기본학습을마쳤으나보다다양한실전문제들로 KMO 차시험을준비하는학생들을위해몇달전에중학생실전수학올림피아드문제집을만들어낸데이어이번에는고등학생들을위한실전문제집을펴내게되었습니다. 이책에는수론 / 대수 / 기하 / 조합의네분야별로, 또중급 / 고급의난이도로나뉘어, 총 80문항에이르는매우방대한분량의문제가마련되어있습니다. 각분야의중급은 KMO 고등부 차시험에서장려상이나동상정도를, 그리고각분야의고급은은상이나금상을목표로하는난이도로볼수있습니다. 많은문제를스스로풀어보며접하여본학생일수록안정적인실력을갖추게되고또한응용력도성장하게됩니다. 끈기있게도전하여힘든여정을겪으면서자신의능력에차차자신감을갖게될것입니다. 그러나, 모든문제를다풀고넘어가겠다고하는것은시간이아까운면이있으니, 어떤수준의문제가 70% 정도가풀리고있다면그수준은그것으로끝내고다음단계의수준으로넘어갈것을권합니다. 모든문제에는최대한원래의출전을밝히려고노력하였습니다. 원래의출전대신에그것을인용한중간출전을적어둔경우도있습니다. 이책에는문제의풀이와해답이실려있지않습니다. 책이지나치게두꺼워질것을우려하여과감히생략하였고, 대신에 xmo 까페 ( 의 `실전MO 풀이 ' 게시판에문제의풀이를게시하고있으니, 거기서찾아보시기바랍니다. 기본학습단계가아닌실전단계에서는, 몇몇문제가잘풀리지않더라도, 가능하면풀이를참고하지않고스스로풀어내겠다고마음먹고승부욕을불태우는것이꼭필요하다고생각합니다. 게시판에올려진풀이보다더멋진여러분만의풀이를찾아내어까페에서함께논의하기를기대해봅니다. 008 년 0 월, 책임편집자고봉균

3 차례 수론 5. 수론중급문제 수론고급문제 대수 89. 대수중급문제 대수고급문제 기하 기하중급문제 기하고급문제 조합 조합중급문제 조합고급문제 종합문제 종합중급문제 종합고급문제...446

4 4 차례 문제의풀이는 xmo 까페 ( 의 ` 실전 MO 풀이 ' 게시판에게시하고있습니다. xmo 까페로오시면그밖에도많은학습자료를찾을수있고, 여러회원들과온라인에서만나수학문제에대한의견 교환및질의응답도할수있습니다.

5 제 장 수론. 수론중급문제 은정수가아님을보여라.(n ) n + 증명 3 k 중 k 가최대인것을찾아서분모를통분하면분자는 3 의배수가아니게됨.. 3 n+ j 3n + 임을보여라. 증명귀납법. n =0일때3j3 성립. n일때성립가정하자. 3 n+ +=( 3n ) 3 + = ( 3n + )(( 3n ) 3n +) 임에서 3 j ( 3n ) 3n + 임만보이면 n + 일때도성립하게된다. (mod 3) 이므로 홀 (mod3). 따라서,( 3n ) 3n + ( ) ( ) + 0(mod3). 3. 모든자연수 n 에대하여 n = X djn Á(d) 임을보여라. (Euler's totient) 풀이 n = X djn Á(d) 임을보이자. k n 이라하자. 우선 c j n 인 c 에대하여 gcd(k; n) =c 인것 들의개수를알아보자.gcd(k; n) =c 이면 k c 와 n c 이서로소이고, 역으로 k가 c의배수이고 k c 와 n c 이서로소이면 gcd(k; n) =c 이다. k c n c 이므로 gcd(k; n) =c 인 k의개수는정확히 Á( n c ) 이된다. 한편각각의k에대하여 gcd(k; n) =c 라고하면 c j n 이다. 그러므로 n은각각의약수 c에대하여 Á( n c ) 을모두합한것과같게된다. 따라서 n = X Á( n c ) 이고 n c 을 d로치환하면 n = X Á(d) 이다. } cjn djn 4. X n n= s = Y p 풀이 µ p s 임을보여라 ( 단, s> 인실수 ). Y p X n n= s = + s + + () 3s µ µ n s = + s + µ + + s 3 s + + µ+ 5 3s s + () () 과 () 는각각절대수렴하며, 자연수 n은유일하게소인수분해되므로 () 의우변의괄호에서각각하나씩뽑는순열을생각해볼때하나의순열이존재해서그곱이 이다. 반대로 () 의우변의서로다 ns

6 6 수론 른모든순열에대해서그곱은서로다르며 (* 곱이같다면소인수분해의유일성에의해, 두순열은같은순열이된다.) 꼴이므로결국 () = () 이다. } ns 5. d =gcd(a; m) j b 일때, 방정식 ax b (mod m) 은정확히 d 개의 (mod m 에대하여 ) 서로다른해를가짐을보여라. 증명 a = da, m = dm, b = db 라하면, 위의잉여방정식은 Ax B (mod M) 과동치. A와 M은서로소이므로 mod M에대한A의잉여역수A 0 이존재하고, 양변에이것을곱하면 x A 0 B (mod M) 과동치가됨. 따라서, x는 mod M으로유일한해를갖고, 그럼 mod m으로는정확히 d개의서로다른해를가짐. 6. a, b, n 은자연수들이고, b 는 a n 의배수이다.(a +) b 이 a n+ 의배수임을증명하여라. ( 헝가리 93-) 증명 (a +) b 은 (a +) an 의인수를가지므로 b = a n 일때만보이면충분하다. n에대한귀납법으로풀자. 우선 n =0일때는 (a +) =a는 a 의배수이므로성립. n일때성립함을가정하고 n +일때를보자.(a +) an+ =((a +) an ) a =(m )(m a + + m +) 로인수분해되는데 (m := (a +) an ), m 은 a n 의배수이고 m (moda) 임에서 m a + + m =a 0(moda) 이므로확임되었다. 7. a, b, c, d 는정수들이다. 임의의두정수 m, n 에대해, ax + by = m 과 cx + dy = n 을만족하는두정수 x, y 를항상찾을수있다고한다. ad bc = 임을증명하여라. ( 헝가리 94-) 증명 ( 신한솔 ) m =일때 (a; b) =임을알수있다. a = b =0이면 0=m 이므로모순. 따라서 a =0이면 b 6= 0이다. m = a, n = c +일때ax + by = a, cx + dy = c +을만족하는 x, y가존재해야한다. a 6= 0일때ax + by = a 에서 a j by 이고 (a; b) =이므로 a j y 이다. 따라서 y = ak 라하면 x = bk 이고 cx + dy = c + 에대입하면 c cbk + adk = c +, k(ad bc) =이므로증명끝. a =0일때b6= 0이므로 y =0이고 cx + dy = c + 에서 cx = c +이므로 c j 이다. m =을대입하면 b j 이므로 jbj ==jcj, 즉 jbcj =이다. 따라서증명끝 8. 서로다른정수 a, b, c, d 에대해, 방정식 (x a)(x b)(x c)(x d) 4=0 이정수해 r 을갖는다고한다.4r = a + b + c + d 임을보여라. (Putnam 947-B5) 증명 (r a)(r b)(r c)(r d) =4이고 x, a, b, c, d 모두정수이므로 (r a); (r b); (r c); (r d) 는모두정수이고, a, b, c, d가서로다르므로이넷도서로다른정수이다. 곱하는순서를제외하면 4개의서로다른정수의곱으로표현하는방법은 4= ( ) () ( ) 가유일하다. 따라서 r a =,r b =, r c =,r d = 이고이를모두더하면 4r = a + b + c + d로우리가원하는결과를얻는다. 9. T =,T n+ = Tn T n +(n>0) 으로주어진수열이있다. n 6= m 이면항상 T n 과 T m 이서로소임을증명하여라. (Putnam 956-B6) 증명 m>n 이라고가정하자. 이제먼저수학적귀납법에의해서다음명제를증명하자. m>n 이면항상어떤수 A 에대해서 T m = A T n + 이다. T n+ = T n T n +=(T n )Tn+ 이므로 m = n + 일때성립한다. m = n + k 일때명제가성립한다고가정하면 T n+k = A T n + 이고 T n+k+ = T n+k (T n+k ) + = (A T n + )T n+k + = (A T n+k ) T n + 이므로준명제는수학적귀납법에의해항상성립한다. 이때 m, n 이어떻게주어져도 T m = AT n + 이므로 (T n ;T m )=(T n ;T m AT n )=(T n ; ) = 이되어 T n, T m 은항상서로소이다.

7 . 수론중급문제 7 0. 자연수 m 에대해,3 k 이 m 의약수인 k 의최대값을 f(m) 으로나타내기로하자. 임의의 n> 개의자연수의집합 M, 그원소 a; b M 에대해, f(a b) 의값은많아야 n 종류임을증명하여라. ( 스웨덴 968-4) 증명 M 에 mod 3 m 으로서로다른두원소가존재하는최소의 m 0 을생각. 거기서귀납법.. 정수계수의다항식이서로다른세정수점에서 또는 의값을가지면, 이다항식은정수근을가질수없음을보여라. ( 폴란드 차 -) 증명 f(x) =0,jf(a)j = 일때 jx aj jjf(x) f(a)j = 이므로 jx aj = jx bj = jx cj =. 즉, x a; x b; x c f ; g 이므로비둘기집의원리에의해 a, b, c 중어느둘은같고그럼모순.. n! 이하의자연수는 n 개이하의서로다른 n! 의약수의합으로나타낼수있음을보여라. ( 소련 968-9) 증명귀납법. n =일때는자명하고, n 일때잘성립한다고가정하면, m n! 일때m을 n으로나눗셈을하여 m = nq + r 라하면q (n )! 이므로 (n )! 의서로다른약수의합q = d + + d k 로나타낼수있고 (k n ), 그럼 m = nd + + nd k + r 도 r(< n) nd i 이므로 n! 의서로다른약수의합으로나타낸것이된다 (k + n). 3. n 을자연수라하자. n + 이 4 의배수일때 n 의모든양의약수들의합이 4 로나누어짐을보여라. (Putnam 969-B) 풀이 ( 서울과학고 3 년박정근 ) n = ab 라할때 ab + 0 (mod 4); ab (mod 4); a 가 의배수이거나 3 의배수이면 ab + 이 4 로나누어질수없다 (). ) a (mod ); a =k +; a =4k (k +)+; ) a (mod 8); 또 3 으로나누어지지않으므로 a (mod3); ) a (mod 4): 또 () 에서, 3 이소수이므로 a 와 4 가서로소임을알수있다. a (a + b) a + ab 0 (mod 4): a 와 4 가서로소이므로 a + b 0 (mod 4); 여기에서 a 6= b 이다. 왜냐하면 a = b 이면 a a + a + + (mod4) 이므로모순이다. 따라서, ab = n 이되는서로다른두약수 a, b 에대해서 a + b 가 4 의배수이므로 n 의모든양의약수의합도 4 의배수이다. } 4. 정수계수다항식 f(x) =x n + a x n + a x n + + a n x + a n 이서로다른네정수a, b, c, d에대해 f(a) =f(b) =f(c) =f(d) =5 를만족한다. 이때, f(k) =8을만족시키는정수 k는존재하지않음을보여라. ( 캐나다 970-0)

8 8 수론 증명 g(x) =f(x) 5 라하자. 그럼 g(a) =g(b) =g(c) =g(d) =0 이고, 이때 g(k) =3 인정수 k 가없음을보이면된다. 인수정리에의해 g(x) =(x a)(x b)(x c)(x d)q(x) 이고, g(x) 가정계수다항식이므로 q(x) 도정계수다항식이다. 만일 g(k) =3 이라면 (k a)(k b)(k c)(k d)q(k) =3 3 은소수이므로좌변의다섯인수중에절대값이 3 인것이하나, 나머지넷은모두절대값이 이다. 즉, (k a), (k b), (k c), (k d) 중적어도셋은 이고, 그럼그중둘은함께 이거나함께 로같다. 이것은 a, b, c, d 가모두다르다는데에모순. 따라서, 이런 k 는없다. 5. 보다큰정수 n 이소수일필요충분조건은 k n 을만족하는모든 k 에대해이항계수 n k 가 n 으로나누어지는것임을증명하여라. ( 폴란드 차 -3) 증명 n이소수일때이것이성립함은쉬움. n이합성수일때는, n j n (n )! k 는 k!(n k)! 이정수라는의미임에서 k를 n의한소인수로택하로택하면끝. 6. 에서 까지의수들을임의의순서로이어썼다. 이렇게얻어진 자리의수가 k 꼴의수가될수없음을보여라. ( 소련 970-3) 증명 mod 9. 자릿수가 i인것을9 i로바꾼것과서로짝지으면 ( 단,9와0은그대로냅두고 ), 짝이없는수들은 9와 0만으로된수들이니까뭐. 7. 자연수 x, y, z, n 이방정식 x n + y n = z n 을만족할때,min(x; y) n 임을증명하여라. ( 폴란드 97 3 차 -4) 증명 WLOG x y(< z). x n = z n y n =(z y)(z n + +y n ) >x n + +x n = nx n 8. r +3 n 을만족하는임의의자연수 n, r에대하여이항계수 n r ; n r+ ; n r+ ; n r+3 은등차수열의연속된네항이될수없음을증명하여라. ( 폴란드 974 3차-5) 증명네항을 a, b, c, d 라할때 d c = c b = b a 를그냥열심히풀면됨. 9. (a) 다음부등식을증명하여라. [5x]+[5y] [3x + y]+[3y + x] 단, x; y 0 이고,[u] 는 u 이하의가장큰정수를나타낸다 ( 예 :[ p ]=). (b) 임의의자연수 m과 n에대해 (5m)!(5n)! m! n!(3m + n)! (3n + m)! 증명 a 5 이정수임을증명하여라.(a) 를이용해도좋고이용하지않아도된다. ( 미국 975-) () 0 x y<일때만확인해도된다.[5x] =a, [5y] =b 라하자 (0 a b<5). 그럼 b+ y< 5. x< a+ 5, b 5 3a + b +3 우변 + 5 a +3b +3 0 a b 4 일때 f(a; b) 와 a + b 의값을표로구해비교해보자. 5 [ 표 ] 좀노가다 = f(a; b)

9 . 수론중급문제 9 어느경우에나항상 a + b쪽이더크거나같음을확인할수있다. () 임의의소인수 p에대해, p가분자에같거나더많이들어있음을보이면된다. 즉, 다음을보이면된다. X 5m i= p i + 5n p i X 3m + n i= p i + m +3n 이것은 () 에의해각각의 i 에대해좌변이더크므로성립한다. 0. 다음을만족하는양수 a, b 가존재하는지각각답하여라. () a; b = Q 이고 a b Q. () a; b; a b = Q. (3) a Q 이고 b; a b = Q. ( 유고슬라비아 979 고4-) 풀이 p p 가유리수일때, 무리수일때로나누어각각조금씩흔들거나하면됨. () 의예로는 ( p ) logp 3 같은것도있음. } p i. 다음방정식이무한히많은자연수해 (x; y; z) 를가짐을보여라. ( 폴란드 차 -) x + y + z =3xyz 증명 ( 박경태 ) 위의방정식을 x 3yzx+y +z =0와같이이차방정식으로바꿀수있다. 그러면어떤자연수해 x에대해서이는이차방정식이므로다른근 x 0 를가진다. x + x 0 =3yz이므로 x 0 =3yz x. 따라서 x y z라고할때,3yz x>z이므로 (x; y; z) 가이방정식의자연수해이면 (y; z; 3yz x) 도자연수해이다 (Vieta's jumping). 이러한방식으로 (x; y; z) =(; ; ) 부터시작하면무한히많은자연수해를가진다.. f n 은 f = f =,f n = f n + f n (n>) 로정의된피보나치수열이다. 모든자연수 n에대해 f n anb n 이 m으로나누어떨어지고 0 <a<m,0<b<m인정수들a, b, m은유일하게존재함을보여라. ( 영국 983-) 풀이 ( 고봉균 ) 조건을만족하는 a; b; m이존재한다고가정하면 n =; ; 3일때 m j ab; m j ab ;mj 3ab 3 () 이고, 우선 ab (modm) 이므로 (a; m) =; (b; m) = 이다. 또, m j ( ab) ( ab )=ab(b ) 이므로 m j b 이다. 이제 m j ( ab)+(b ) = b( a) 이므로 m j a 이고,0<a<m 이므로 ) a =; 그리고 () 은다음과같이쓸수있다. m j b; m j 4b ;mj 6b 3 () m 은홀수 b 를나누므로역시홀수이고, m j 6b 3 에서 m j 3b 3 이된다. 이제 m j ( 4b ) ( 3b 3 )=b (3b 4), 즉 m j 3b 4 이고, 또 m j (3b 4) 3( b) =5 이다.0<a<m 이므로 m =5 이고,0<b<m 과 m j b 에서 b =3 이다. 그러면, a =;b=3;m=5 이면조건이만족되는지보자. n =; 일때성립하고, n 3 일때는 5 j f n (n ) 3 n (3) 5 j f n (n ) 3 n (4) 5 j 3 n (5n +5) (5) (3) + (4) (5) 에서 5 j f n n 3 n ) a =;b=3;m=5일때유일하게존재. }

10 0 수론 3. p(p +)+q(q +)=n(n +) 을만족하는자연수순서쌍 (p; q; n) 을모두찾아라. 단, p와 q는소수여야한다. ( 오폴 983-) 풀이먼저 n p + q 일때는우변이커서해없음. n p + q 일때만보자. q(q +) 을우변으로넘겨인수분해하면 p(p +) =(n q)(n + q +). n q<p이므로 p j n + q +. 마찬가지로 q j n + p +. 즉, p; q j n + p + q + 이므로 p 6= q 라면 pq j n + p + q + p +q. 즉, pq p +q 이고 (p )(q ) 4. WLOG p<q라하고가능한경우를좀따져주면 p =일때q =,n =3 과 p =3일때q =5,n =6이해의전부. p = q 일때는p(p +)=n(n +) 임에서 n과 n +중에 p의배수가있고 n>p이므로 n p 인데그럼우변이커서해없음. } 별해 WLOG p q 라하고p(p +) 을우변으로넘기면 q(q +)=(n p)(n + p +). q j n p 이면 q n p, q +<n+ p +이므로우변이커서모순. 즉 q j n + p +. n + p +=qk 라하면 k =일때는좌변이크고 k 4 이면우변이크게되어 k =또는 3. k =일때 :3q 3=4p 이되어 p =3,q =5,n =6. k =3일때 :4q =3p +가되어p =,q =,n =3. } ny 4. 모든자연수 n에대해,( n+ )! 은 ( n+ i ) i 의배수임을증명하여라. ( 오폴 983-8a) i=0 증명 n =일때는3j3! 로자명. 귀납법으로, n일때성립가정. n+ n+ 은정수이므로 ( n+ ny )! j ( n+ )!. 따라서, 가정에의해 ( n+ i ) i+ j ( n+ )!. j = i +로치환하면좌 i=0 n+ Y 변은 ( n+ j ) j 가되고여기에j =0일때인 n+ 을양변에곱해주면증명끝. j= 5. 임의의소수 p 에대해, n n 이 p 의배수가되는자연수 n 이무한히많음을보여라. ( 캐나다 983-4) 증명우선 p =일때는n이짝수이면된다. p가홀수인소수일때를생각하자. 페르마의작은정리에서 p (modp) 이므로 n 은 mod p로주기p 을갖고, n는 mod p로주기p를갖는다. 즉, 두주기의공배수인 T = p(p ) 에대해 n+t (n + T ) n n (mod p) 이므로 f(n) = n n 은 mod p 로주기 T 를갖는다. (p ) (p ) =0 (modp) 이므로 f(n) 0(modp) 인 n 이있다. 그럼 mod p 로주기함수이므로 f(n) 0(modp) 인 n 이무한히많다. 6. 다음을만족하는자연수쌍 (n; k) 를모두구하여라. ( 호주 983-4) (n +) k =n! 풀이 n =; ; 3; 4 일때각각체크해보면 (n; k) =(; ); (; ); (4; ) 일때가능. n 5 일때를살펴보자. 우변이짝수이므로좌변도짝수가되려면 n은짝수, 즉 n =m, m 3. n! 에는, m, m이모두곱해져있으므로 n j n!. 즉, 좌변에서도 n j (n +) k 이어야한다. 이항정리로전개하여 차이하의항만살펴보면 n j kn 임을알수있으므로njk. 그럼 (n +) k (n +) n >n n >n! 로문제의방정식이성립할수없다. 답 (,), (,), (4,) } 7. 두홀수인합성수의합으로나타낼수없는가장큰짝수는무엇인가? (AIME 984-4) 풀이 6n +=9+(6n +3), 6n +38=35+(6n +3), 6n +8=5+(6n +3). (n ). 답 38 }

11 . 수론중급문제 8. 어떤자연수가 () 0인자리수를갖지않고 () 자리수의합의배수가된다면 이수가 ` 자리수로나누어진다 ' 고한다. 예를들어,3 는자리수로나누어진다. 자리수로나누어지는자연수는무한히많이있음을보여라. ( 캐나다 984-3) 증명 {z } 가모두자리수로나누어지는양의정수들이된다. 이수의자리수의합이 3 n 이므로, 3 n 개 이수가3 n 의배수임을수학적귀납법으로보이자. 우선 n =일때는3j 로쉽다. n = k 일때성립한다고가정하면 = {z } {z } 3 k+ 개 3 k 개 로 3 j 이므로이것은 3 k+ 의배수, 즉 n = k + 일때도성립한다. 따라서, 임의의양의정수 n 에대해이런수들이하나씩존재하므로, 자리수로나누어지는양의정수는무한히많다. 9. n 이 k 꼴일때에만 n 이 n! 를나눔을증명하여라. ( 캐나다 985-4) 증명 n! 을소인수분해했을때 p 의지수는다음과같다. f(n; p) = ¹ º ¹ º ¹ º n n n + p p + p 3 + 이문제는 f(n; ) n 이될동치조건이 n = k 꼴이라는것을증명하라는것과같다. 우선 n = k 꼴이면 f(n; ) = k + ++= k =n 로성립한다. n = k 꼴이아니면 k <n< k 인양의정수k가존재하고, 그럼수가아니므로 j n k j n k j n k j n k f(n; ) = k < n + n + n n k = n n <n k n k 이모두정 이된다. 즉, 문제가성립하고, n = k 꼴일때특히 n k n! 임을알수있다. nx 30. 자연수 n과 k에대해, F (n; k) = r k 로둔다. F (n; ) 이 F (n; k) 를나눔을증명하여라. r= ( 캐나다 986-4) 증명 F (n; ) = n = n(n+) 이므로 n(n +)j F (n; k) 임을보이면된다. F (n; k) =( k + n k )+( k +(n ) k )+ +(n k + k ) 에서각항 i k +(n i +) k 은 k 이홀수이므로 i +(n i +)=n + 로나누어떨어진다. 따라서,(n +)j F (n; k). F (n; k) =(+(n ) k )+( k +(n ) k )+ +((n ) k +)+n k 으로정리할수도있으므로, 같은방법으로 n j i k +(n i) k, n j n k 로 n j F (n; k). n + 과 n 은서로소이므로 n(n +)j F (n; k) 임이확인되었다.

12 수론 3. 자연수 m, n, r 이다음식을만족하면 m 은완전제곱수임을증명하여라. ( 남미 987-3) ( + p 3) r =+m + n p 3 증명 b 0 =, b =,b n+ =4b n b n 이라할때, m r = b r. 3. 이차식 x + p x + q =0과 x + p x + q =0의계수가정수이다. 이두식은정수가아닌공통해를가진다. p = p 이고, q = q 임을보여라. ( 통신강좌 988-B3) 풀이 x + p x + q =0:::() x + p x + q =0:::() 일반적으로 x + ax + b =0 의해는 x = a p a 4b 만약여기서 p a 4b = m (m은유리수 ) 라놓으면 a 4b = m 에서 a가짝수면m도짝수, a가홀수면m도홀수이므로 x = a m 에서 x는정수가된다. 즉, 유리수근을가지려면꼭정수가되어야한다. 그런데이방정식 (), () 는정수가아닌즉유리수가아닌공통근을가지므로이공통근은무리수나허수밖에되지않는다. 이공통근을 라놓으면 (p p )+q q =0에서 p = p, q = q. } 33. x 3 y 3 =xy +8 의모든정수해 (x; y) 를구하여라. ( 통신강좌 988-B4) 풀이 x 3 y 3 =(x y)(x + xy + y )=xy +8인데문제로부터 x 6= y이고 x y (mod ) 임을알수있으므로 jx yj 이다. 즉 jx + xy + y j jxy +4j이된다. x + xy + y xy +4와 x + xy + y xy 4로부터얻을수있는해는 (; 0); (0; ) 이다. } 34. x, y, z 는유리수이고 t = 3p 일때 x + yt + zt 으로표현되는수들을생각하자. 만약 x + yt + zt 6=0 이면다음식을만족하는유리수 u, v, w 가존재함을증명하여라. (x + yt + zt )(u + vt + wt )= ( 남미 988-5) 증명유리수 a, b, c에대해a + bt + ct =0이면 a = b = c =0임을먼저증명하고. 그담엔준식을 t에대한이차식으로전개한후연립방정식. 35. x ;:::;x n 은 n 개의정수이고 p 는 n 보다작은자연수이다. S = x + x + + x p T = x p+ + x p+ + + x n S = x + x x p+ T = x p+ + x p x n + x. S n = x n + x + x + + x n T n = x p + x p+ + + x n 이라하자. a; b f0; ; ; 3g 에대해, S i a (mod 4), T i b (mod 4) 인 i ( i n) 의개수를 m(a; b) 라하자. m(; 3) m(3; ) (mod 4) 일때, 또그때만 m(; ) 가짝수임을보여라. ( 아일랜드 988-B, 호주 988-6)

13 . 수론중급문제 3 증명 S i + T i 는 x i 를전부합한것이므로일정하고, N = x + x + + x n 이라할때4-N 이면 m(; 3) = m(3; ) = m(; ) = 0 이되어서문제없음. 4 j N 이면 S + S + + S n = pn 0 m() m(3) + m() (mod 4) 이므로, m() m(3) (mod 4) 와 m() 0(mod4) 가동치임이금방확인된다. 36. 연속한네정수의곱에 988 을더한것이완전제곱수일수없음을증명하여라. ( 오스트리아 988) 증명 mod 5 로끝. 37. m 이자연수일때 (m) 을 m 의 ( 양의 ) 약수의개수라하자. 조건 ( n ) >n 을만족하는자연수 n 이무한히많이있음을보여라. ( 통신강좌 989-C3) 풀이 ( ) = 4 >. ( b ) >b이면 ( b ) = (( b + )( b )) ( b ) > b 이므로 ( b +과 b 은차가 인홀수이므로서로소이고, 그럼 b 의약수들이 ( b ) 개, 또그약수들에 b +을각각곱한것이 ( b ) 개있고이들은모두서로다르다 ) b가만족하면 b도만족. 따라서, 무한히많은 n이조건을만족한다. } 38. n 이자연수일때, 어떤자연수 m 이존재하여 ( p +) n = p m + p m 을만족시킴을보여라. (IMO-LL 989, 홍콩출제 ) 풀이다음을만족하는자연수 an ; b n 이존재한다. ( + p ) n = p a n+ + b n+ = p a n + b n p n p (+ ) (an + b n ) = (a n +b n)+(a n + b p n) ) a n+ = a n +b n ; b n+ = a n + b n a n+ b n+ = (a n +b n ) (a n + b n ) ) a n b n =( ) n = a n +b n n 이짝수이면 n 이홀수이면 q q a n + a n = q q b n + b n = q q a n + b n = a n + b n p =(+ p ) n q q b n + a n = b n p +an =(+ p ) n } 39. n 을 0 진법으로나타낸자연수라한다. n 의각자리수들의곱이 n 0n 인모든 n 을구하여라. ( 한국 989-5) 풀이 (i) n이한자리수라하면 n 0n = n ) n n = 0 n = p 33 이므로정수해는없다.

14 4 수론 (ii) n이세자리이상의수라하고 n = a 0 +0a +0 a +0 3 a k a k (k = ; 0 5 a k 5 9) 라하면 a 0 a a k = n 0n = n(n ) 좌변은 a 0 a a k 5 9 k+ < 0 k+ 이고우변은 k +자리수이상이므로이식을만족하는자연수는없다. (iii) n을두자리수라하고 n = a +0b 5 a; b 5 9 라하면 (a +0b) 0(a +0b) = ab 00b(b ) + 0ab a(0 a) =ab + 00b(b ) + 9ab = a(0 a)+ a(0 a) =5 (a +5) < 5 이므로 b = 이때 9a =0a a + ) a +9a = 0 (a )(a +)=0 ) a = 구하는수는 n = 답 n = } 40. P (0) =, P (3) = 8 이고모든근이정수인정수계수의다항식 P 를모두찾아라. ( 북유럽 989-) 4. 풀이 P (x) =a(x b )(x b ) (x b n) 으로둘수있고, P (0) = ( ) n ab b b n =. 따라서 jaj = jb i j =. P (3) = a(3 b )(3 b ) (3 b n )= 7 에서 3 b i =or4이므로모두양수이고 a =. b i 들중인것이u개, 인것이v개라하면 u +v =7을만족하는음이아닌정수u, v에대해 P (x) =(x ) u (x +) v. 답 (x )(x +) 3,(x ) 3 (x +),(x ) 5 (x +), (x ) 7 } p 3 n + p n ++ 3p n p n + 이자연수인것과적당한자연수 m에대해n = m(m +3) 꼴인것이동치임을보여라. ( 아일랜드 989-9) 증명 ()) a = 3p n + p n +, b = 3p n p n + 라하면 a 3 + b 3 =n, ab =. 따라서, n = a 3 + b 3 =(a + b)((a + b) 3ab) =(a + b)((a + b) +3) 이고, 문제에서 m := a + b 가자연수라고했으므로 n = m(m +3) 꼴. (() 역시위와같이 a, b 를정의하면 m(m +3)=(a + b)((a + b) +3) 이됨. m 3 +3m 은단조증가하므로 a + b 일수밖에없음. 4. x, y, z 가 x 3 +3y 3 +9z 3 9xyz =0 를만족시키는유리수일때, x = y = z =0 임을보여라. ( 통신강좌 990-E7) 풀이 x, y, z의분모의최소공배수를식의양변에곱해도상관없으므로우리는 x, y, z를모두정수라고생각해도된다. x, y, z 을, 모두가 0은아닌주어진식의정수해라면, x, y, z 의최대공약수 d로각각의수를나눈, x = x d, y = y d, z = z 도역시이식의해가되며, 이때이들의최대공약수 d 는 이된다. x 3 =9xyz 3y 3 9z 3 이므로 x는 3의배수이다. x =3u로치환하면, 식은, 다시 y 는 3 의배수이다. y =3v 로치환하면, 9u 3 + y 3 +3z 3 9uyz =0 3u 3 +9v 3 + z 3 9uvz =0 이때z는 3으로나뉜다. 이는 x, y, z의최대공약수가 이라는가정에모순이므로, x = y = z =0이유일한해이다. }

15 . 수론중급문제 n 이홀수이고 5 의배수가아닌자연수일때, n 의배수중에는모든자리수가 인것들이무한히많이있음을증명하여라. ( 통신강좌 ) 44. 풀이 n이홀수이고 5의배수가아니므로 gcd(0;n)=이다. 또한 gcd(0; 9) = 이므로 gcd(0; 9n) = 이다.Euler정리에의하면 0 Á(9n) (mod9n) 이다. Á(9n) =r 이라고놓자. r 6= 0임은자명하다. 모든양의정수 m에대하여 0 rm (mod9n) 이므로 0 rm 은 9n의배수들이고 0rm 9 는 n의배수들이다. 한편 0rm 은모든자리수가 이다. } 9 X k n gcd(k;n)= 풀이 k = n Á(n) 임을증명하여라. ( 통신강좌 990--) X k n gcd(k;n)= 을간단히 P 로표시하자. k n 인정수 k 에대하여 gcd(k; n) = 이면 gcd(n k; n) = 이고, 그역도성립함을쉽게알수있다. 그러므로 P fk +(n k)g = P n = ná(n) 이다. 그런데 P = P (n k) 임이자명하므로 P k = ná(n) 이다. } 45. ny k 가 nx k 로나누어지는 n을구하여라.( 단, Q 는곱의기호이다.) ( 한국 990-) k= k= 풀이 ny m = k ± X n k = n! n(n+) 가정수가되는자연수 n을구하면된다. k= k= (i) n가홀수일때 : n =k 이라하면 m = n! (n )! (n )! = 이고, k =일때m =; k > 이면 k<n 이므로은정수이다. nk k k (ii) n이짝수일때 : n =k라하면 (n )! m = n + : n +>n 이므로 n +이소수이면 m은정수가아니다. n +이합성수 이면이의약수는모두 n 보다작다. p를소수라하고 n +=ap m 이면 (n )! = (ap m )! 에는인수 p를 ap m 개포함하고, ap m m 이다. 따라서 (n )! 은 p m 로나누어진다. 이것은 n +의임의의소인수에대해 (n )! 서성립하므로 n +이합성수이면 n + 은정수이다. ( n 답이홀수일때 n이짝수이고 n +이합성수일때 } 46. k 90 인자연수 k 에대하여 f(k) = ( k ( k 45 일때 ) k 9 (46 k 90 일때 ) 으로정의하면 f : f; ;:::;90g!f; ;:::;90g 은전단사함수가된다. 이때 f 를 n 번합성한함수 f n 이항등함수가되는최소의자연수 n 을구하여라. ( 한국 990-4) 풀이 ¼ = µ 라하고, 단위원주를 (,0) 으로부터 9등분한점들을점 (,0) 을제외하고각각 µ; µ; ; 90µ 9 라한다. 5 k 5 90의각k에점kµ 를대응시키면 f(k) 는점kµ 에대응된다. 즉단위원주상에서각을 배로하는사상이다. 따라서 f n (k) 는 n kµ 에대응되고이것이항등사상이면 n kµ = kµ +9mµ(m은자연수 ) 인자연수 m이존재하면된다. ( n )k =9m 5 k 5 90이므로 n 이 9로나누어지는최소의자연수 n을구하면된다. 9 = 7 3이고 6 =63=7 9 = 4095 = 35 = 이므로최소의 n은 이다. 답 n = }

16 6 수론 주 p 가소수이면 p (modp) 이다. 이정리를이용하면 6 은 7 의배수이므로 = ( 6 )( 6 +) 도 7 의배수이고 은 3 의배수이므로 은 3 의최소공배수 9 의배수이다. 47. 음이아닌정수 n 에대해함수 f(n) 이다음과같이정의되어있다 : f(0) = 0, f() =, 그리고 m(m ) < n m(m+), m 인정수에대해 µ f(n) =f n µ m(m ) m(m +) f n f(n) =5 가되는최소의정수 n 을찾아라. ( 통신강좌 ) 풀이 f (n) =f (a) f (b) 라면, a + b = m (a ; b 0) 이고 n = m (m ) + a = (a + b)(a + b ) 이므로 a, b 가증가하면 n 도증가한다. 우선 f (0) = 0, f() =, f() = 0 이고 n 3 에서 n>a;b;m 이며, 또한 n 에대해 a, b, m 이유일하게결정되므로문제의정의는 f 를유일하게결정하는점화식이다. f (n) =i 를만족하는최소의 n 을 n i 라하자. 이 n i 를찾기위해서는최소의 a, b 를구해야하므로결국이 n i 들에대해서만생각하면된다. 먼저 n = 을쉽게얻고, f (n )= =0 =f() f() 이므로 n =5, 또 f (n )== ( ) = f (n ) f (n )=f() f (5), f (n )=f (n ) f() 으로 n =6,n =0 이다. 같이하여 n 3 =,n 3 =5,n 4 =646,n 4 = 650 을구할수있고, 5=4 ( ) = 3 ( ) = ( 3) = ( 4) 에서최소인것으로 n 5 = 658 이얻어진다. } + a 48. a, b, c 는서로다른자연수이고, a + b, a + c, b + c 는모두제곱수이다. a + b + c 의최소값을구하여라. (IMTS R4-) 풀이일반성을잃지않고 a<b<c라하고, a + b = x, a + c = y, b + c = z 이라하면 x<y<z 이고 x +y >z (). x +y +z =(a+b+c) 이므로 x, y, z는모두짝수이거나홀수가 개 (). () 과 () 를고려하며약간노동하여경우를따져보면 (x; y; z) =(5; 6; 7) 일때a + b + c =6+9+30=55 로최소. } 49. 각 A 는각 B 의두배이고각 C 는둔각이며, 세변의길이가모두정수인삼각형 ABC 가있다. 이삼각형의둘레의길이는최소얼마인가? ( 미국 99-) 풀이 sin 법칙과 cos 제법칙등을이용해 a = b(b + c) 임을먼저얻어내고,gcd(a; b; c) =일때만보면되는데그럼 b와 b + c가서로소이고각각제곱수 b = m, b + c = n, a = mn, c = n m. c >a + b 과 c<a+ b 에서 m p 3 <n<m. 여기서 m =; ; 3 일때는해가없고, m =4,n =7 일때a + b + c = mn + n =77이최소. 답 77 = } 50. 방정식 x pqx + p + q =0 이정수해를갖도록하는자연수 p, q 를모두구하여라. ( 러시아 99 4 차 -y0-) 풀이두근을a, b라하면a + b = pq, ab = p + q. 합과곱이둘다양이므로 a, b는둘다양수. 두식을빼면 (a )(b )+(p )(q ) =. 즉,(p )(q ). p, q가둘다3 이상이면부등식에위배되어해가없고, 둘중에하나가이나 일때를조사해보면 ( 좀노동 )(p; q) =(; ); (; 3); (3; ); (; 5); (5; ) 답 } 5. x 0 =0,x =, 그리고 n 0 에대해 x n+ =3x n+ x n 으로정의된수열에서, x n +n+ 은항상어떤홀수의완전제곱이됨을증명하여라. ( 오스트리아 ) 증명점화식을풀면 xn = n 임. 즉, x n +n+ =( n +). 5. a 0 =9,a k+ =3a 4 k +4a3 k (k 0) 으로주어진수열 a n이있다. a 0 을 0진법으로썼을때 000개보다많은 9를가짐을증명하여라. (Towns 99가을 JA4)

17 . 수론중급문제 7 증명 0 n j a n 임을보이자. 우선 n =0일때0 j a 0 + = 0 성립. n = k 일때성립하면, a k+ +=(a k +) (3a k ak +) 이므로 (0 k ) j a k+ + 로 k +일때도역시성립. 따라서, 0 04 j a 0 +이고 a 0 은마지막에 04개이상의9로끝난다. 53. 다음과같은조건을만족시키는자연수 p, q, n 을모두구하여라 :99 p 3 n+ 이고, q n + 은 p 의약수이다. ( 통신강좌 99-4-) 풀이 99 3 n+ 이고,3 6 =79,3 7 =87이므로 n 6이다. 또, q n + p 3 n+ 에서 3 n+ q n +>q n, 즉 (n +)log3> n log q이다. n 6에서 n + n > log q log 3 인데 q 이면우변이좌변보다커지게되어모순이므로 q =이다. q n + = j p에서 p =k꼴이고, 답은 (p; q; n) =(k; ;n) 이다. 단, n 6은임의의자연수이고, k는 996 k 3n+ 인자연수이다. } 54. 임의의자연수 n 에대해다음합을구하여라. ( 통신강좌 ) X n + k k+ = k=0 n + + n 풀이 자연수 n 의이진전개를 n = a m a a 0 () = mx a j j j=0 라하면 X n + k k+ k=0 = X k=0 n k+ + 3 mx a j j X j=0 = 6 k=0 4 k = k 3 X a j j X mx a 6 j j (k+) + a k + + j=0 k=0 4 k+ 7 j=k+ 5 = = 0 X a j j (k+) ak + + A k=0 j=k+ X mx X a j j (k+) + a k k=0 j=k+ k=0 = = = mx j X mx a j j (k+) + a j j= k=0 j=0 0 mx j X a k +A + a 0 j= k=0 mx mx a j ( j +)+a 0 = a j j = n j= j=0 이다. }

18 8 수론 55. 주어진 n 개의정수중임의의 n 개의곱과나머지한수의차가항상 n 으로나누어진다. 이때 n 개정수의제곱의합이 n 으로나누어짐을보여라. ( 통신강좌 ) 풀이 주어진 n 개의정수를 a,, a n 이라하자. 그러면 Q ai a j a j 0 (mod n) ) Y a i a j (mod n) 이므로 P a j n Q a i 0(modn) 이된다. } 56. x 4 + y 4 + z 4 =y z +x y +z x 3 을만족하는모든자연수근을구하여라. ( 통신강좌 ) 풀이 준식을변형하여 x 4 + y 4 + z 4 y z z x +x y = 4x y 3; (x + y z ) = 4x y 3; (x + y z ) (xy) = 3; (x + y + z)(x + y z)(x y + z)(x y z) = 3 를얻는데,(x + y + z) j 3이고 x + y + z 3이므로 x = y = z =이다. } 57. 정수수열 fu ng 이다음의조건을만족시킨다. u =; u 3n =0; u 3n = u n ; u 3n+ = u n + (n ) () u 99 를구하여라. () n이자연수이고 m =(3 n +) 3 일때, u m 을구하여라. (3) n 3 M (M은자연수 ) 인 n에대하여, u n =0인 n의개수를구하여라. ( 한국 99-3) 풀이 () u 99 = u = u 664 = u 3 + = u + = u =0+= () m =(3 n +) 3 =3 n+ f3 n +3 n +g + =3 n+ f3 n (3 n +)+g + 이므로 u m = u 3 n (3 n +)+ +=u 3 n + ++ = u 3n +++ 에서 n =이면 u 3 n + = u = u 3 =0이므로 u m = n = 이면 u 3 n = u =이므로 u m =4 즉 n =일때 u m =; n = 일때u m =4 (3) u 3n = u n ;u 3n =0이므로 u 3 k (3n ) =0 (k 0; n ) 인경우뿐이다. 3 k (3n ) 3 M 에서 k =0; ; ; ;M 이고이때 3n 3 M k ) 3n 3 M k + n 3 M k 이므로 k에대해서 n은 3 M k 개가있다. 따라서구하는가짓수는 M X k=0 M X 3 M k = k=0 3 k = 3M 3 = (3M ) } 주 u n 은 n 을 3 진법으로썼을때, 뒤에서부터찾아 가나타나기전까지의자릿수의합이다.

19 . 수론중급문제 ( + p 3) 5 를넘지않는최대의정수를구하여라. ( 한국 99-8) 풀이 ( + p 3) 5 +( p 3) 5 = 5X k=0 = f 5 + ³ 5 k 5 k ( p 3) k + 5X k=0 ³ 5 ³ g = ( ) = 74 ³ 5 k 5 k ( p 3) k 0 < p 3= + p 3 < 이므로 0 < ( p 3) 5 < 따라서 73 < ( + p h 3) 5 < 74 ) ( + p 3) 5i =73 } 59. (n 7 7) 이 9 의배수가되도록하는자연수 n 의최소값을구하여라. ( 한국 99-9) 풀이 n 7 7(mod9) 인최소의 n 을구하는문제이다.7 3 =343=9 8 + 이므로 7 3 (mod 9). 따라서 (n 7 ) 3 = n 7 3 (mod9).9 는소수이므로 n 9 = n 8 (mod9) ) n = n 3 n 8 n 3 (mod9) ) n 6 (mod9) 따라서 n 7 = n 6 n n 7 (mod 9). n =9m +7 인꼴이고최소값은 n =7 } 60. a ;a ;:::;a n 은서로다른자연수들이다 (n ). 다항식 f(x) =(x a )(x a ) (x a n) 이정수범위에서더이상인수분해되지않음을보여라. ( 북유럽 99-) 증명 f(x) =p(x)q(x) 로인수분해된다고하고 x = a i 들을대입하면 p(x i )q(x i )=. 즉 p(x i )+ q(x i )=0. 따라서, p(x)+q(x) 는 n 개의서로다른실근을갖는데, 이것은 n 차이하의식이므로모순. 6. 유리수좌표의두꼭지점을갖는삼각형이있다. 이삼각형의나머지한꼭지점도유리수좌표일동치조건은이삼각형의각내각의 tangent 값이유리수이거나직각일때임을증명하여라. ( 아일랜드 99-5) 증명 ()) 각변이 x- 축으로부터기울어진각은 tangent 값이유리수이거나직각. 그럼 tan(a b) = tan a tan b +tana tan b 의차각공식에의해두변이이루는각도 tangent 값이유리수이거나직각.(90± 가끼었을때조금조심스럽게서술해줄필요가있음 ) (() 두점 A, B 가유리수좌표이므로 AB 와 x- 축이이루는각의 tangent 값은유리수 ( 또는무한 ). 모든내각의 tangent 값이유리수 ( 또는무한 ) 이므로위에서처럼합각 / 차각공식에의해다른변도 x- 축과이루는각의 tangent 값은유리수 ( 또는무한 ). 유리수좌표의점으로부터유리수기울기를가지는두직선의교점이므로역시사칙연산에의해서만구할수있어서좌표는유리수.( 역시 90 ± 가끼었을때는조금조심스럽게서술해줄필요가있음 ) 6. n 3 이면 n! =d + d + + d n 을만족하는 n! 의서로다른 n 개의약수 d ;d ;:::;d n 이존재함을증명하여라. ( 이탈리아 99-3) 증명귀납법. 항상 d = 로하기로하고 (n +)d = d + nd 으로만분할해주면됨 a, b는자연수이다. p a + 3p b 가유리수면, a, b가모두세제곱수임을증명하여라. ( 이탈리아 99-6) 증명 3p a 3p ab + 3p b 도유리수이고, 그럼 3p 3p ab도유리수. a b, a + 3p ab + 3p b 이유리수이므로 3p a 3p b 도유리수. 그럼 3p 3p a, b 각각이유리수. 유리수의세제곱이정수이므로사실은정수의세제곱. 64. m +3 n 이완전제곱수가되는정수들의순서쌍 (m; n) 을모두찾아라. ( 인도 99-8)

20 0 수론 풀이 m, n이짝수임을 mod 3, mod 4로보이고그럼넘겨서인수분해하면두항의차이가 또는 가되어 또는 3 꼴이되는... } 65. n + 이홀수인소수가아니면, 또그때에만, 최초의 n 개의자연수의곱이최초의 n 개의자연수의합의배수가됨을증명하여라. ( 캐나다 99-) 증명 ( n) j n! 일필요충분조건이 n + 이홀수인소수가아님을보이라는문제이다. ( n) j n! () n(n +)j n! () n +j (n )! 이므로, n + 이홀수인소수이면이것은성립하지않는다. 이제 n + 이홀수인소수가아니면항상성립함을보이면된다. 우선 n +=k 로짝수이면 k n 이므로성립한다. 다음합성수중에서 보다큰서로다른두수의곱 n +=ab 로표현되는경우에는 a; b f; ;:::;n g 이므로역시성립한다. 합성수중에서이런곱으로표현할수없는것은 p 꼴 (p 는소수 ) 뿐이므로, 이제이런수들에대해서만보면된다. 홀수만보면되므로 p 3 이고, 그럼 p; p f; ;:::;n g 이므로역시성립한다. 그러므로, 모든양의정수 n 에대해문제가증명되었다. 66. 자연수 x, y, z 에대하여 f(x; y; z)=++3+ +(x + y ) z 라한다. f(a; b; c) =f(c; b; a) = 993 을만족시키는자연수 a, b, c, d 의모든짝을구하여라. ( 한국 993-7) (x + y )(x + y ) 풀이 f(x; y; z) = z f(a; b; c) =f(c; d; a) 에서 (a + b )(a + b ) c = (c + d )(c + d ) ) (a + b )(a + b ) + a = (c + d )(c + d ) + c ) (a + b ) +(a b +) = (c + d ) +(c d +) ) (a + b ) (c + d ) = (c d +) (a b +) ) (a + b + c + d )(a + b c d) = (c d a + b) (a c)(a + b + c + d ) = (d b)(a + b + c + d 3) M = a + b + c + d 라하면M = 4이고 (a c)(m ) = (d b)(m 3) b = d라가정하자.(b <d라가정해도되고, 이때 a>c) (M ) (M 3) = 이므로 M 과 M 3의최대공약수를 m라하면m =또는 m =이다. m =이면 d b 는 M 의배수이고, d b<m 이므로모순 m =이면 d b 는 M 의배수이고, a c 는 M 3 의배수이므로 4(a c)(d b) 는 (M )(M 3) 의배수이다. (M )(M 3) 4(a c)(d b) =(a + c b d) +4fa(b ) + b(a ) + c(d ) + d(c )g +3> 0 이므로모순따라서 b = d 이다. 이때 a = c 를얻는다. (a + b )(a + b ) a =993에서 (a + b )(a + b ) = a =3969< 3986 이므로 a + b =63이라하면 a = = 46 ) a =3;b=4 a + b =64이면 a = = 74 ) a =87;b<0 이므로버림 답 ½ a = c =3 b = d =4 a }

21 . 수론중급문제 67. 수열 x; x +; x+; x+3; ::: 에서등비수열을이루는서로다른세항을골라낼수있다는것과 x 가유리수라는것이필요충분임을보여라. ( 캐나다 993-) 증명 () 의증명 ) x + a, x + b, x + c 가등비수열을이룬다고하자. 그럼 (x + a)(x + c) =(x + b), 즉 (a + c b)x = b ac ( ) 산술-기하평균부등식에서 (x + a)+(x + c) (x + b) 이므로 a + c b 0 인데, 서로다른항들이므로등호가성립하지않고, 따라서 a + c b >0 이다. 즉,( ) 의계수가 0이아니므로 x = b ac a+c b 로유리수이다. (( 의증명 ) x가음수이면 x +, x +, ::: 로증가하다보면언젠가양수가되므로, x가양수일때에만증명해도충분하다. 유리수이므로 x = q p 라하자. ( + (p + p )) = ( + p) 의항등식에서양변에 x x를곱하면 x (x +(q + pq)) = (x + q) 즉, x, x + q, x +(p + pq) 가등비수열을이룬다. 68. 방정식 x n +(+x) n +( x) n =0 이정수해를가질수있는모든양의정수 n 을구하여라. (APMO 993-4) 풀이 (O±cial) y = x n +(+x) n +( x) n 이라하자. () n 이짝수일때, x 가정수이면 x n ; ( + x) n ; ( x) n 은모두음이아닌정수이므로 y =0 이되려면이들은모두동시에영이되어야한다. 이는불가능하므로이경우는정수해가없다. () n 이 n 3 인홀수일때, y 는 x n 의계수가 인 n 차다항식이고, 상수항은 n+ 이며모든항의계수는양수이므로 y =0 이정수해를갖는것은 x = t (0 t n +) 인경우뿐이다. t =0 일때 : x = 이고, y =( ) n + n +3 n = 3 n 6=0. t = 일때 : x = 이고, y =( ) n +0+4 n =4 n n 6=0. 따라서, t. t = p + 이라하면, p 이고 y =( p+ ) n +( p+ ) n +(+ p+ ) n = n f pn +( p ) n +(+ p ) n g = n [ pn +f+( n )p +( n 4 )4p + g] 이므로 n 3 일때위식의우변의 [] 안은 [] (mod4) 이므로 y 6= 0 이다. 따라서 n 이 n 3 인홀수일때도정수해는없다. 남은경우는 n = 일때뿐이다. 이때 x +(+x)+( x) =0 에서 x = 4. 이상을종합하면 n = 일때만정수해를갖는다. } 69. 방정식 x + y 3 = z 의자연수해는유한개인가아니면무한개인가? (Towns 993 가을 SO) 풀이 y 3 = z x =(z + x)(z x) 이므로 z + x = y, z x = y 인것만생각해보면, z = y(y+), x = y(y ) 로무한히많은해가있음. } 70. n 번째소수를 p n 이라할때다음을보여라. ( 통신강좌 ) p n < n

22 수론 풀이수학적귀납법을쓴다. n =일때, p =이므로자명하다. k n인모든자연수 k에대해성립한다가정하자. N = p p p n +이라하면, p,, p n 은 N을나누지않는다. 그러므로, m>n인자연수 m이존재하여, p m 이 N을나누어야한다. 즉, p m N 그런데, p n+ p m 이므로, p n+ N이다. 한편, 수학적귀납법에의해 N = p p p n +< + = + + n + = n+ + n+ ) p n+ < n+ 이다. } 7. 자연수의수열 fa n g 이다음과같이정의된다 : a n+3 = a n+ (a n+ + a n ); n =; ; 3;::: a 6 =44 일때 a 7 을구하여라. ( 통신강좌 ) 풀이 a = x, a = y, a 3 = z 라하면 a 4 = z(y + x), a 5 = z(y + x)(z + y), a 6 = z(y + x)(z + y)[z(y + x)+z] =z (z + y)(y + x)(y + x +) 이다. y + x 와 y + x + 이인접정수임에유의하면서 44 = 4 3 의약수들을쓰면,,,3,4,6,8,9,, 6, 8, 4, 36, 7, 44 이고 (,3), (3,4), (8,9) 가후보이다. (, 3) 인경우는 x = y = 이고 44 = z (z +) 3 즉,4=z (z +) 이나정수해가없다. (3, 4) 인경우는 44 = z (z +y)3 4 에서 = z (z +y) 이고 z =,y = 만이해이다. 그러므로 x = 이고 a 7 = a 6 (a 5 + a 4 ) = 44[ ] = 44 4 = 3456 이다. (8, 9) 의경우 44 = z (z + y)8 9 에서 =z (z + y) 이고 z =,y =,x =7(* x + y =8) 이고 a 7 = 44(6 + 8) = 3456 이다. [ 답 ] a 7 = 3456 } 7. a <a < <a n < 은음이아닌정수열로서모든자연수 n 에대해 a n = a n + n 을만족한다. 그리고 n 이소수일때만 a n 도소수가되는성질을갖고있다. a 994 를구하여라. ( 통신강좌 ) 풀이 a k <a k+ <a k+ <:::<a k = a k + k 이므로 a k+ = a k +;k 을얻을수있다. 즉, a n = a +(n );n 이다. a =0 이면 a =;a 3 =;a 4 =3 이므로모순. a 이라고하자. 이때 p 를 (a +)!+(a +) 보다큰소수라고하자. 그리고, n = p (a ) 로잡자. a n = p 이므로가정에의해 n 도소수이어야한다. 그런데, n>(a +)!+ 이고 (a +)!+k; k f;::: ;a +g 은모두소수가아니므로 n p 가되어 n = p (a ) 에모순. 즉, a = 이어야하고 a 994 = 994. } 73. 모든정수 x 에대해, 다음의수가정수임을증명하여라. ( 호주 994-) 5 x5 + 3 x x 증명 통분하고 mod 3, mod 5 에의한단순잉여 증명 x 5 + x 3 을빼고더하여페르마소정리

23 . 수론중급문제 3 증명 3 귀납법 74. 여섯정수의 6제곱의합빼기 하면이여섯수의곱의여섯배가된다. 이여섯수중에하나만 이고나머지는모두 0임을증명하여라. (Towns 994가을 JA6) 증명산술-기하로 6abcdef = a 6 + b 6 + c 6 + d 6 + e 6 + f 6 3a b c +3d e f, 즉 3(abc def) 얻을수있음. 즉, abc = def 일수밖에없고, 임의의세항의곱은나머지세항의곱과항상일치. 0이없다면둘씩교환하다보면모두같아야해서문제의조건이 만큼맞지않음. 즉, 0이있고, 그럼 6제곱의합이 이므로원하던결론이나옴. 75. x, m, n 이모두 보다클때다음식이해를가지지않음을증명하여라. n x m = ( 통신강좌 ) 증명 ( 부산과학고허석문 ) x 는홀수이므로 x =k + 이라고두자. n 이 보다크므로, n = x m +=(k +) m + 0(mod4). 이항전개에의해 (k +) m + ) m; k 는홀수 m 이홀수이므로 x m + 을인수분해하면 ³³ m k + + (mk +) 0 (mod 4) x m +=(x +)(x m x m + x m 3 +)= n x 는홀수이므로 x m x m + x m 3 + (mod) ) x += n, x m x m + x m 3 += n = x m +=x +) m =) m 이 보다크다는것에모순 ) n x m = 을만족하는해는없다. 76. 다음수열의항중최소인것을구하여라. ( 통신강좌 ) a = < a n+ = 3 a n (3 j a n 일때 ) : a n +3 (3- a n 일때 ) 풀이일단최소값은 4보다작음을증명한다. 만일 am 이최소값이고 4이상이라고가정해보자. (i) a m =3k +(k : 자연수 ) 라면 a m+ = k +4<a m 이므로모순. (ii) a m =3k + (k : 자연수 ) 라면 a m+ =3k +4 a m+ = k +8<a m 이므로모순. (iii) a m =3k +3(k : 자연수 ) 라면 a m+ =3k +6 a m+ =3k +39 a m+3 = k +3<a m 이므로모순. 따라서최소값은 3이하이다. 이제최소값을구하기위해 mod3으로관찰을해보자. ( 9a n (3 j a n 일때 ) a n+ = a n (3 - a n 일때 ) 한편, a = (mod 3) 오일러정리에의하면 6 (mod3).

24 4 수론 한편 (mod) 이므로, a 6 3 8(mod3). a n 이취할수있는값을다음그림으로살펴보자.? 9 8 7? 9? 9 따라서 a n 8, 7, 이다. 그러므로최소값은 8이나 7, 이다. 만일 a m =8이라면 a m+ = a m+ =7 만일 a m =이라면 a m+ =4 a m+ =8 a m+3 = a m+4 =7 즉, 최소값은 7이다. } 77. 방정식 x y = 의정수해가무한히많이존재함을보여라. ( 한국 995-) 증명 ( 과기원 95 학번허충길 ) ( + p ) n = a n + b n p (n 0 인정수 ) 이라정의하면명백히다음과같은성질을얻을수있다. ( p ) n = a n b n p (n 0 인정수 ) 그러면 a n b n =(a n + b n p )(an b n p ) = ( + p ) n ( p ) n =( ) n = 따라서, x = a n, y = b n 은 x y = 의해이고, a n 은단조증가하므로무한히많음도알수있다. 78. 복소수계수의방정식 x 3 +ax +bx+c =0 의모든복소근이절대값이 이면, 방정식 x 3 +jajx +jbjx+jcj = 0 의모든복소근도마찬가지로절대값이 임을증명하여라. ( 아일랜드 995-7) 증명세근과그켤레를이용하면 jaj = jbj 이고 jcj =임을알수있음. 그럼우선 을한근으로갖고, 나머지두근 ( 서로켤레 ) 의곱이이니까두근모두절대값이 임. 79. 서로다른네소수의곱으로임의의자연수 n 에대해,=d <d < <d 5 <d 6 = n 을 n 의약수들이라하자. n<995 이면 d 9 d 8 6= 임을증명하여라. ( 아일랜드 995-0) 증명약수들의대칭성에의해 n = d d 6 = d d 5 = = d 8 d 9. n을구성하는네소수를 p<q< r<s, 즉 n = pqrs 라하자. 귀류법으로, d 9 d 8 =라하자. d 9 와 d 8 은홀짝이같다. 둘다짝수이면 n = d 8 d 9 는 의배수이므로 n이서로다른네소수의곱임에모순. 따라서, d 9 와 d 8 은둘다홀수이고, n도홀수, 즉 p 3 이다.(p; q; r) =(3; 5; 7) 이아니면n > = 995 가되어모순. 즉,(p; q; r) =(3; 5; 7) 이고, s<9 이다. s =; 3; 7 일때각각d 9 d 8 을구해보면모두성립하지않음 ( 혹은 d 8 ;d 9 모두둘이상의소수의곱이므로딱두소수의곱이고대충약수들의차례를생각하면 fd 8 ;d 9 g = f3s; 5 7g뿐인듯.3p 35 = 를풀면p =9)( 또혹은, d +d n =0이정수해를가져야하므로판별식 + n 이제곱수라야하는데 s =; 3; 7 일때n을대입해보면제곱수인것이없음 ).

25 . 수론중급문제 n 은고정된자연수이다. k 가음이아닌정수이면, x i 와 y 에대한부정방정식 x 3 + x x 3 n = y 3k+ 이무한히많은자연수해를가짐을보여라. ( 캐나다 995-4) 증명 x = x = = x n = x 로두어보자. nx 3 = y 3k+ x = n a, y = n b 으로두면 3a + = 3kb +b 을만족하면된다.b (mod3) 이어야하므로 b =3t + 로하면되고, 이때 a =3kt +k +t + 이된다. 즉, 임의의음이아닌정수 t 에대해, (x; y) =(n 3kt+k+t+ ;n 3t+ ) 가모두해가된다. 8. a0 09 꼴 (0 은 개이상, a ) 의수는완전제곱수일수없음을보여라. (Towns 995 봄 SO4) 증명 a 0 n =(m +3)(m 3) 의두인수m +3 과 m 3 이둘다5의배수일수없으므로어느한쪽이 5 n 을다가짐. 따라서, m +3 5 n. 그럼 m 3 a n 9 n. 이둘에서 5 n 6 9 n 인데이것은 n 이면성립하지않음. 8. 자연수들로된무한수열 fa ng 과 fb ng 이있다. fa ng 은공차가 r (> ) 인등차수열이고 fb ng 은공비가 q (> ) 인등비수열이다. 그리고 r 과 q 는서로소이다. 이두수열에같은항이있다면, 같은항이무한히많이있음을증명하여라. ( 폴란드 995/996 차 -6) 증명 q c (modr) 인자연수c가존재. a i = b j 이면 a i <q kc b j b j = a i (mod r) 로같은항이또있음. 83. 어떤정수계수의다항식을 x x + 로나누었더니나머지가 990x 889 가나왔다. 이다항식은정수근을갖지않음을보여라. ( 폴란드 995/996 차 -9) 증명 P (x) =(x )((x )Q(x) + 990) + 0 = 0 이면 x = ; 0. x =0; ; 00; 0 를각각대입해보면각각모순. 84. 다음방정식의모든자연수해 (x; y) 를구하여라. ( 통신강좌 ) x 3 y =7 풀이 mod 3 으로생각하면 x =x 0. 다시 mod 4 로생각하면 y =y 0 이된다. 그럼준식은 ( x0 +3 y0 )( x0 3 y0 )=7 이고, 그럼 7 =7 이되는경우뿐이다. 즉,( x0 ; 3 y0 )=(4; 3) 뿐이고,(x 0 ;y 0 )=(; ), (x; y) =(4; ) 가유일한해가된다. } 85. p 는소수이고 a 와 n 은 p +3 p = a n 을만족하는자연수이다. n = 임을증명하여라. ( 아일랜드 996-8) 증명귀류법으로, n 라가정. p =일때 +3 =3로성립하지않음. p> 이면 5 j p +3 p, 즉 5 j p +3 p. p 3 p =( 3) p p (mod 5) 임에서 p (mod5). 따라서,5jp, 즉 p =5. 그런데 =75도성립하지않음. 86. 다음식의모든정수해를구하여라. ( 폴란드 996 차 -5) x (y ) + y (x ) = 풀이 x =또는 y =일때따로풀고, x; y 6= 일때는y (modx ) 이므로 jx j jy j, 마찬가지로 jy j jx j. 여기서좀따져보면아마... }

26 6 수론 별해 x 에대한내림차순으로정리하면 (y )x + y x (y +) =0. 판별식 D = y 4 +4(y )(y +)=y 4 +4y 3 4y +4y 4 가제곱수가되어야함을부등식법으로좀해보면... } 87. 정수 n 이합성수일때, 또그때만, 두관계식 a + b = n 과 x a + y =을만족시키는자연수 a, b, b x, y가존재함을증명하여라. ( 폴란드 996/997 차-4) 증명 ()) n = ad 일때b = a(d ), x =,y =(a )(d ) 로두면됨.(() n이소수이면 ( 귀류법 ) 정리하면 n(a x) =a(a x + y). 모든항이양이고 n과 a는서로소이므로 n j a x + y. a x<a 이므로우변이항상큼. 88. gcd((n +) m n; (n +) m+3 n) > 을만족하는자연수 m; n 의모든쌍을구하여라. ( 통신강좌 ) 풀이 ( 과기원수학과 96 학번허석문 ) n, m 을문제의조건을만족하는자연수라하고, 이라하면, d =gcd((n +) m n; (n +) m+3 n) (n +) m+3 (n +) m =(n +) m ((n +) 3 ) = n(n +) m (n +3n +3) 이 d 의배수이고 d 는 n, (n +) 과서로소이므로 (n +3) 3 (mod d); n +3n +3 0 (mod d) d 의정의에의해,(n +) m n (mod d) 임을상기하자. (i) m =3k + 일때, n (n +) m =(n +)((n +) 3 ) k (n +) (modd) 이므로모순. (ii) m =3k + 일때, n (n +) m =(n +) ((n +) 3 ) k (n +) (mod d) 에서 (n +) n = n + n + 0 (mod d): 또한 n +3n +3 0(modd) 이므로변변빼주면 (n +) 0(modd) 이고 d 와 (n +) 이서로소이므로 d =: 그러나이경우 n(n +) 이짝수이므로 n + n + (modd) 가되어모순. (i), (ii) 에의해 m =3k 이어야한다. 이때 n (n +) m =((n +) 3 ) k (modd) 이므로 n (mod d) 이다. 따라서 (n +) 3 ( + ) 3 =8 (modd) 이므로 이경우 (n +) =8 (mod7) 이므로 d =7; n (mod 7): (n +) m n =(n +) 3k n k 0 (mod 7) (n +) m+3 n =(n +) 3k+3 n k+ 0 (mod 7) 이므로문제의조건을만족한다. 따라서문제의일반해는 (n, m) =(7` 6, 3k)(`, k 는자연수 ) 이다. } 89. a 를 3 보다큰홀수라하자. 모든자연수 n 에대하여 a n 은적어도 n + 개의서로다른소인수를가지고있음을증명하여라. ( 통신강좌 ) 증명 ( 과기원 97학번이수인 ) 수학적귀납법으로증명하자. n =인경우u = a +, v = a 라하면a가5이상의홀수이므로 u, v는연속한자연수이고둘중하 나는큰홀수이다. 따라서 a =4uv는홀수인소인수를적어도하나가지고있다. 또한 도 a 의소인수이므로이는적어도 개의서로다른소인수를가지고있다. n 인 n에대해a n 이 (n +) 개이상의소인수를가지고있다고하자. a n+ =xy; x = a n +; y = a n a n (mod4) 이므로 x (mod4) 이다. 따라서 x 는홀수인소인수 p 를갖는다. x 와 y 는연속한 자연수이므로서로소이고 p 는 x 의소인수이므로 y 의약수가아니다. 귀납법가정에의해 a n 은적어도 (n +) 개의소인수를가지며 p 가그들중하나가아니므로 xy = a n+ 은적어도 (n +) 개의소인수를가진다. 따라서임의의자연수 n 에대해 a n 은적어도 (n +) 개의소인수를가진다.

27 . 수론중급문제 어떤정수 A, B 에대해서는다음두집합 M = fx + Ax + B j x Zg; M = fx +x + C j x Zg 가교집합을갖지않도록하는정수 C 가있음을보여라. ( 통신강좌 ) 증명 ( 과기원 97학번김병두 ) i) A가홀수일때 M : x(x + A)+B B (mod ) 즉 C 6 B (mod ) 로 C를정하면됨. ii) A가짝수일때 µ M : x + A + B A 4 B A A 또는 B + (mod4) 4 4 M : x(x +)+C C (mod 4) 역시 C B A A +또는 B +3으로하면됨. 4 4 i), ii) 에서증명가능. 9. (7 + p 50 ) 997 을십진법으로썼을때의소수점이하처음 997 자리를구하여라. ( 몰도바 997 최종 -y-) 풀이 t =7 p 50, a n = t n + + tn 으로두면 a n 은정수. 0: <t < 0 이므로 n이홀수이면 t n + = a n t n (a n;a n +0: n ). 따라서, 소수점이하 n자리가모두 0. } 9. 주어진자연수 n 에대해 ¾(n) 을 n 의약수의합이라하자.( 예 : ¾(3) = +3 = 4, ¾(6) = =, ¾()= =8) 이때 ¾(n) > n 이면 n 을초과수라부르자.( 따라서, 예를들어, 는초과수이다.) a, b 가자연수이고 a 가초과수이면 ab 도초과수임을보여라. ( 아일랜드 997-6) 증명 a의약수들을d ;d ;:::;d k 라하면d b; d b;:::;d k b는 ab의서로다른약수들. 이들만모두합해도 ab를넘으므로 ab는초과수. 93. 다음성질을만족하는세자연수의쌍을모두찾아라 : 세수중임의의두수의곱을나머지한수로나눈나머지는항상 이다. ( 폴란드 997 차 -4) 풀이 abc j (ab )(bc )(ca ) 이므로 abc j ab + bc + ca. 즉 a + b + c = k( abc 정수 ). k =or이고, 경우를나눠풀면됨. } 94. a, b 는자연수이고 ab 가 a + b 의약수라고한다. a = b 임을증명하여라. (Towns 997 봄 SO) 증명 a 6= b 라면 p m k a, p n k b 이고 m 6= n 인경우가존재. 그러나 ab j a + b 임에서 m + n min(m; n) 라야하는데이것은 > 이기도해서모순. 증명 a kab + b =0에서 a 6= b 인자연수해가존재한다면 WLOG a>b인최소의해가존 재. b를고정시키면 a의이차방정식. a와짝이되는해를a 0 이라하자. 그럼 0 <a 0 = b a <b이므로 (b; a 0 ) 은 (a; b) 보다더작은해가되어최소성에모순. 95. p가홀수인소수이고 k가 k p 인정수라할때다음을증명하여라. ³ p ( ) k (mod p) ( 통신강좌 ) k 증명 ( 서울과학고 학년박영훈 ) (p )(p ) (p k) ( )( ) ( k) k! ( ) k (mod p) gcd(p; k!) = 이므로 p k ( ) k (mod p).

28 8 수론 96. A는정수들의집합으로, 모든 x; y A 에대해서 x y 또한 A에속하는성질을갖고있다.998A 이고, 구간 [ 00; 00] 가 A의원소를 33개이상,66개이하로포함한다고하자. 구간 [ 998; 998] 에는 A의원소가몇개포함되겠는가? ( 몰도바 998 최종-y0-7) 풀이 A = dz꼴 ( 셈본고등중급최대공약수편참조 ). 33 b 00 c + 66 d 에서 3 <d<7 이고, 그럼 d j 998 에서 d =6. } 97. P (x) 는정계수다항식이고 P (x) =0을만족하는정수해가있다. P (995)P (000) 이 7의거듭제곱이될수없음을증명하여라. ( 몰도바 998 최종-y-8) 증명 P (x) = 0 의정수해를 a. P (x) = (x a)q(x). P (995)P (000) = (995 a)(000 a)q(995)q(000) 이 7 의거듭제곱이려면 995 a, 000 a 도 7 의거듭제곱. 그런데두 7 의거듭제곱수의차이는 5 가될수없다 ( 가장작아도 7 =6 이상 ). 98. n 은 3 이상의정수이다. x x 와 x n x 가둘다정수가되는실수 x 는정수뿐임을증명하여라. ( 아일랜드 998-5) 증명 x x = a, x n x = b 라하자.0<x< 이면 a는정수가아님. 그외엔 a>0. x = x + a 를이용해 x n 의차수를떨어뜨리면식 x n x = b 는 Ax = B꼴이됨. 특히귀납적으로 A>0 임을확인할수있음. 따라서, x는유리수이고, 이제유리해정리를이용하거나직접결론내리면됨. 99. 다음을만족하는 a b c 이고 x y z 인자연수 a, b, c, x, y, z 를모두구하여라. a + b + c = xyz; x + y + z = abc ( 폴란드 차 -) 풀이 (a; b; c) 와 (x; y; z) 의대칭성에서 WLOG a + b + c abc. c 이면모순이므로 c =. 또 b 3 이면모순이므로 b =or. 이런식으로경우나눠서막노동. } 00. x, y 는실수이고 x + y, x + y, x 3 + y 3, x 4 + y 4 가모두정수일때, 모든정수 n 에대하여 x n + y n 이정수임을보여라. ( 폴란드 998/999 차 -4) 증명 xy, x y 이정수가되고이로부터 xy 는정수. 그럼점화식 P n = x n + y n =(x + y)p n xyp n 에의해 P n 은모두정수. 0. y x = x 50 의자연수해를모두찾아라. ( 폴란드 998/999 차 -5) 풀이 x를소인수분해했을때지수들의최대공약수를 a, 즉 x = n a 이고 n은더이상거듭제곱으로나타낼수없는수라고할때, n 50a=na = y 가자연수임으로부터 n a j 50a. 다시이로부터 n j 50 임을알아낼수있다 ( ). 그럼 n =; ; 5; 0; 50 일때각각풀면됨.( ) 이틀림없음을확인해보자.gcd(n; a) 의임의의소인수 p에대해, e p(a) e p(n a ) 임을확인할수있다 (e p(m) 은 m의소인수분해에서의 p의지수 ). n a n j 50a 에서 e p (n a )+e p (n) e p (50) + e p (a) 이므로그럼 e p (n) e p (50) 이라는것이고, 따라서 n에서 50을나누다남아서 a만나누는부분은없다. } 풀이 x가, 5 이외의소인수 p를갖는다면 y도 p를소인수로갖는다. 양변의 p의지수를비교하면 50e p (x) =xe p (y), 즉 p ep(x) j e p (x) 인데, 이것은일반적으로 e 일때p e >e임에서모순. 즉, x( 와 y) 의소인수는, 5뿐이다. x = a 5 b, y = c 5 d 이라하고대입하여비교하면,50a = a 5 b c, 50b = a 5 b d. a =0일때, b =0일때각각따로풀고, a; b > 0 일때는a a,5b 5 b 임을이용해풀면됨. } 0. n =(d(n)) 을만족하는모든자연수 n 을구하여라. 단, d(n) 은 n 의양의약수의개수를나타낸다. ( 캐나다 999-3)

29 . 수론중급문제 9 풀이 n 은완전제곱수이므로 n = p e p e k k 꼴로소인수분해된다. d(n) =(+e ) ( + e k ) 이므로 n = d(n) 은다시쓰면 가된다. 우변은홀수이므로 p i 3. 이항정리에의해 p e pe k k =(+e ) ( + e k ) ( ) p e i i ( + ) ei +e i 이성립하고, 등호는 p i =3,e i =일때만성립하므로,( ) 의등식이성립하기위해서는 k =,n =3 일수밖에없다. } 03. a 3 +6ab + 과 b 3 +6ab + 이동시에세제곱수가되도록하는자연수쌍 (a; b) 를모두구하여라. ( 폴란드 999/000 차 -5) 풀이부등식법 a 3 +6ab + (a +) 3, b 3 +6ab + (b +) 3 으로범위줄여놓고풀면금방. 아마 a b 로하면b a 이되고a 3 +6ab +< (a +4) 3 이었나.. } 04. m, n 은자연수이고 mn 은 m + n + m 을나눈다. m 이완전제곱수임을증명하여라. ( 폴란드 999/000 차 -9) 증명 gcd(m; n) =d 라하면d j m. m = d M, n = dn 으로두면 M, N은서로소. 대입하면 d 3 MN j d 4 M + d N + d M, 즉 dmn j d M + N + M. 여기서 M j N 이고서로소임에서 M j. 끝. 05. n 8 n 이 7 의배수가되지않는음아닌정수 n 을모두구하여라. ( 몰도바 000 최종 -y9-6) 풀이 n 3 0; (mod9) 이므로준식은 9의배수. 8의배수여부만검토하면됨. n이홀수이면연속한두짝수의곱n 을인수로가지므로 8의배수. n이 4의배수이면당연히8의배수. n (mod 4) 일때만8의배수가아님. 답 n =4k +꼴. } 06. f(x) =5x 3 +3x 5 +9ax 라하자. 모든정수 x 에대해 f(x) 가 65 로나누어떨어지게되는최소의자연수 a 를구하여라. ( 아일랜드 000-3) 풀이 0 f(x) x 5 ax ( a)x (mod 5). 모든 x에대해성립해야하므로 a (mod 5). 0 f(x) 8x 3 4ax ( 8 4a)x (mod 3). 모든 x에대해성립해야하므로4a 8 (mod 3), 즉 a (mod3). 따라서, a (mod65) 이고, 이것이면충분. a의최소값은 63 답 } 07. 임의의정수 n 과임의의소수 p 에대하여, n pp + p p 이항상합성수임을증명하여라. ( 폴란드 000/00 차 -5) 증명 p =일때소피제르망인수분해. p 3 일때 (n pp ) p + p p =(n pp + p)q꼴로인수분해 됨. 08. 모든자연수 k 를 k = mn + 꼴로쓸수있음을증명하여라. 단, m, n은자연수이다. m + n ( 몰도바 00 최종-y7-8) 증명 (m; n) =(k+; k ) 으로끝.(m; n) =(k+;k +k ) 로할수도. 둘다 (m k)(n k) = k 에서적당히두어구한것임. 09. 임의의 n> 에대해, n 과 (n +) 사이에는 a + b 이 c의배수가되는서로다른세정수 a, b, c가존재함을증명하여라. ( 몰도바 00 최종-y9-6)

30 30 수론 증명 (n + n +) +(n + n) (n +) +(n ) =(n +) 0(modn +). 0. a, b, c 는실수이고, ax + bx + c =0 의두근을 p, p 라하고 cx + bx + a =0 의두근을 q, q 라할때, p ;q ;p ;q 는, 이순서로, 서로다른항으로된등차수열을이룬다고한다. a + c =0 임을보여라. ( 중미 00-5) 증명 p D jaj = p + p =q, q + q =p 임에서변변더하여정리하면 p p = q q. 즉, 두근의차 p D. a = c jcj 이면두식이같으므로서로다른등차수열일수없음. 따라서, a = c. ³ n ³ k. k; n > 은정수이고 p =k 은소수이다. 가 p의배수이면 p 의배수이기도함을증명하여라. ( 폴란드 00 차-) 증명준식은 (n k)(n + k ) 이고, 이것이 p의배수이면 (n k) 와 (n + k ) 중하나가 p의배수인데,(n + k ) (n k) =p 이므로둘다 p의배수.. 다음명제가성립하도록하는정수 n 3 을모두찾아라 : a +a + + na n 이유리수인등차수열 a ;:::;a n 은항상유리수인항을적어도하나갖는다. ( 폴란드 00 차 -4) 풀이 a i = a+id 로두면합S = n(n+) a+ n(n+)(n+) d = n(n+) (a+ n+ d). 6 3 즉, a+ n+ d 3 가유리수. 따라서,3jn +, 즉 n =3k +꼴일때는명제가성립. 이꼴이아닐때는 a = p = n+ d 3 로두면성립하지않음을확인할수있음. } 3. 모든자연수 n 에대하여 n a + b 가항상완전제곱수가되도록하는정수 a, b 가있다. a =0 임을증명하여라. ( 폴란드 00 3 차 -4) 증명 n+ a + b 와 4( n a + b) 가둘다제곱수이고그차인 3b가제곱수의차. a 6= 0이면 n을키워두수를얼마든지키울수있으므로, 상수 3b가계속큰두제곱수의차가될수는없음. 4. 자연수 n 을십진법으로나타낸각자리수의합을 S(n) 이라하자. n 을십진법으로나타내었을때오른쪽으로부터몇개 ( 적어도 개이상 ) 의자리수를지워얻은자연수를 n 의도막이라고부른다. n 의모든도막의합을 T (n) 이라하자. n = S(n)+9T (n) 임을보여라. (APMO 00-) 증명 (Kalva) Let the digits of n be a d ;a d ;:::;a 0,sothatn = a d 0 d + +a 0.Andletn(k) be the number formed by deleting the last k digits of n. Thenn(k) =a d 0 d k +a d 0 d k + +a k and T (n) = P d k= n(k). Obviously S(n) =a d + + a 0. Hence S(n)+9T (n) =a d (9 0 d +9 0 d + +9+)+a d (9 0 d + +9+)+ +a d k (9 0 d k + +9+)+a (9+)+a 0 = a d 0 d + + a 0 = n. 5. n 의자릿수의총합을 S(n) 으로나타내자. 자연수 n 에대하여 S(n +3) 은완전제곱수가될수없음을증명하여라. ( 폴란드 00/00 차 -8) 증명 3 - n 이면 S(n +3) n +3 (mod3) 으로안되고,3j n 이면 S(n +3) n +3 3(mod9) 로불가능. 6. a = a = a 3 = 이고 a n+ a n a na n =(n 3) 으로정의된수열 (a n) 이있다. 모든 n 에대해 a n 은자연수임을증명하여라. ( 아일랜드 00-4) 증명 준식과 ana n 3 a n a n = 를변변빼어정리하면 a n a n + a n+ = a n a n 3 + a n = a = = a + a 3 또는 a 3 = 가된다 ( 전자는 n이짝수일때, 후자는 n이홀수일때 ). 즉, a + a 4 4 a n+ =a n a n 또는 4a n a n 의점화식으로얻어지게되므로, a n 은항상자연수이다.

31 . 수론중급문제 3 7. 다음의수들이모두소수가되도록하는자연수 p q r 을모두찾아라. pq + r; pq + r ; qr + p; qr + p ; rp+ q; rp + q ( 폴란드 00 차 -4) 풀이 p, q, r이모두홀수이면모든항이짝수, 즉모두로동일, 그럼 p = q = r =뿐. p가짝수이면 이상이므로모든항이 3 이상, 즉홀수인소수, 그럼 pq + r, pr + q 가홀수임에서 q, r은홀수라야하고그럼 qr + p 가짝수라서모순. q나 r이짝수일때도대칭적으로마찬가지. } 8. 방정식 (x + y) (xy) = 을만족하는모든자연수해 (x; y) 를구하여라. ( 폴란드 00/003 차 -) 풀이 ( 서울과학고 학년김성구 ) y = 을넣으면 (x +) x =, 즉 x +x =0,x =. 대칭성에따라일단해 (x; y) =(; ) 와 (; ) 을얻을수있다. 이제 x>, y> 일때만살피면되고, 그럼 (x )(y ) > 0, 즉 xy +>x+ y. 이것을준식에대입하면 (xy +) (xy) > 정리하면 x y +xy > 0. xy > 0 이므로나눠주면 xy +> 0, 따라서 >xy. 따라서, x; y > 일때더이상의해는없다. } 9. n 을 333 자리의자연수라고하자. n 의각자리수는 0 이아닌수들로이루어져있다. i =; ;:::;33 에대해, n i+ 은 n i 의맨마지막자리수를맨앞으로옮겨만들어진수라고하자. 이때, n ;n ;:::;n 333 은모두 333 의배수이거나, 모두 333 의배수가아님을보여라. ( 폴란드 00/003 차 -5) 증명 ( 여주홍지현 ) a i 를 n i 에서의맨마지막자리수라고하자. 그러면, 그런데, 0n i+ (0 333 )a i = n i = {z } 333 개 = {z } 3 이 개 이므로 은 333의배수. 따라서 n 0n 00n 3 (000n 4 )n 4 00n 333 (mod 333) 0, 00 모두 333 과서로소이므로, n i 중어느하나가 333 의배수이면나머지도모두 333 의배수이다. 즉, n ;n ;:::;n 333 은모두 333 의배수이거나, 모두배수가아니다. 0. 3a +4b +9c +d =4 의정수해를모두구하여라. ( 셈본중등고급도전문제..) 풀이 a +3c +4d = u 라하면 3a +4b +9c +d =3u +4b =4 (b; u) =(; 0) 은방정식의특수해이므로 (b; u) =( 3k 0 +; 4k 0 ) a +3c +4d = u에서 3c +4d = v라하면 a +3c +4d = a + v = u (a; v) =(u; 0) 은위방정식의특수해이므로 (a; v) =( k + u; k )=( k +4k 0 k; k ) 3c +4d = r 에서 (c; d) =(4k v; 3k 3 + v) =(4k 3 k ; 3k 3 + k ) ) (a; b; c; d) =( k +4k 0 ; 3k 0 +; 4k 3 k ; 3k 3 + k ) (k ;k ;k 3 Z) }

32 3 수론. a, b, c 가둘씩서로소일때 xbc + yca + zab = n 의일반해를구하여라. ( 셈본중등고급도전문제..) 풀이 xbc + yac + zab = u u = yc + zb로치환xbc + ua = n (bc; a) =이므로 xbc + ua =은해를갖는다. (x; u) =(x 0 ;u 0 ) 를 xbc + ua =의특수해라고하자. 그럼 xbc + ua = n의해는 (x; u) =( ak 0 + nx 0 ;bck 0 + nu 0 ) u = yc + zb 에서 (b; c) = 이므로 =yc + zb 는해를갖는다.(y; z) =(y 0 ;z 0 ) 가 yc + zb = 의특수해라고하면 u = yc + zb 의해는 (y; z) =( bk + uy 0 ;ck + uz 0 ) =( bk + bck 0 y 0 + nu 0 y 0 ;ck + bck 0 z 0 + nu 0 z 0 ) ) (x; y; z) =( ak 0 + nx 0 ; bk + bck 0 y 0 + nu 0 y 0 ;ck + bck 0 z 0 + nu 0 z 0 ) }. 조건 x 4 y = 을만족시키는정수의순서쌍 (x; y) 의개수는모두몇개인가? ( 한국 003 차 -S4) 풀이 x 는홀수임을알수있고, 그럼 y =(x )(x +)(x +) 에서 y 는짝수이다. y =Y, x =X + 을대입하고양변을 8 로나누면 Y = X(X + )(X(X +)+) 인데, 우변의세항이모두서로소이므로모두 완전제곱수여야한다. X 와 X + 의차는 인데 n 꼴들의수중에서차가 이되는경우는 과 0, 그리고 0 과 뿐이다. 즉, X(X +)=0 으로어느경우나 Y =0, 즉 y =0 이다. 따라서, 해는 ( ; 0) 의 개뿐이다. } 3. 음이아닌정수 n 에대하여 ³h n i h n i f(0) = 0; f(n) =f + n 을만족시키는함수 f(n) 이있다.05n5003 일때, f(n) 의최대값은얼마인가? ( 실수 x에대하여 [x] 는 x를넘지않는가장큰정수이다.) ( 한국 003 차-S7) 풀이의개요 f(n) =f(n), f(n +)=f(n) 이므로 f(n) 은 n을 진법으로썼을때자리수의합을구하는함수이다.003는 진법으로 자리의수이므로자리수의합은 을넘을수없고,인것은 047이므로역시불가능하고,0인것은여러개있다. 답 0 } 4. 조건 n = m 4 +m 3 +m +m + 를만족시키는정수의순서쌍 (m; n) 의집합을A = f (m i ;n i ) j kx i =; ;:::;kg 라할때 (m i + n i ) 의값은얼마인가? ( 한국 003 차-S6) i= 풀이 ( 대덕중 3 학년이태희학생의풀이를수정 )(m +) +> 0 이므로 n = m 4 +m 3 +m +m + >m 4 +m 3 + m =(m + m) 또, m =0이면 n =여서곤란하므로 m 0 이고, 그럼 n = m 4 +m 3 +m +m + m 4 +m 3 +3m +m +=(m + m +)

33 . 수론중급문제 33 또한, 이식의등호는 m = 일때에만성립한다. 즉, m > 이면 (m + m) <n < (m + m +) 이되어해가없다. 따라서,(m; n) 의해는m = 일때가전부이고, A = f(; 3); (; 3); ( ; ); ( ; )g kx 이다. 따라서, 구하는식의값은 (m i + n i )=( +3 )+( + )=4. } i= 5. 다음방정식을만족하는정수해 (x; y; z) 를모두구하여라. ( 북유럽 003-) x 3 + y 3 + z 3 3xyz =003 풀이좌변을 (x + y + z)((x + y + z) 3(xy + yz + zx)) 로인수분해. x + y + z =일때는mod 3으로맞지않음. 그렇지않은경우엔 003이소수이고 (x + y + z) 3(xy + yz + zx) 0 이므로 (x + y + z) 3(xy + yz + zx) =밖에없는데이건 (x y) +(y z) +(z x) =와같고, 이건 ++0꼴뿐.WLOGx y z 라하면가능한것은 (x; y; z) =(x; x; x +) 또는 (x; x +;x+) 뿐임을얻을수있고, 그중 (667; 668; 668) 만성립. } 6. m +m = n 4 +0n 3 +04n +40n 의모든정수해 (m; n) 을구하여라. ( 아일랜드 003-8) 풀이 ( 장영실과학고 학년김윤섭 ) (m +) = m +m + (n +0n +) = n 4 +0n 3 +04n +40n +4 이므로 M = m +; N = n +0n + 으로치환하면문제의조건은다음의식이된다. M N =(M + N)(M N) = 000 M + N 과 M N 은홀짝이같은데, 우변이짝수이므로둘모두짝수이다. 그러므로 fm + N; M Ng = f; 000g; f4; 500g; f8; 50g; f000g; f0; 00g; f40; 50g 혹은여기에부호를모두음으로바꾼것들이어야한다. N =(n +5) 3 이므로우선 N 3 을만족하는것만고르면 N = (M + N) (M N) =499; 48; ; 95; 40; 5; 5 즉, (n +5) =5; 7; 44; 8; 63; 8; 8 이중 (n +5) 로가능한유일한해는 44 뿐이고, 따라서 N =, 그리고 n = 7 또는 7. 즉, (M + N; M N) =(50; 8) 또는 ( 8; 50) 이고, M = 9. m = 8 또는 30. (m; n) =(8; 7); (8; 7); ( 30; 7); ( 30; 7) } 의마지막세자리를구하여라. ( 캐나다 003-)

34 34 수론 풀이 우선 x y (mod 000) 이다. Á(000) = Á( )=( 3 )(5 3 5 )=400에서페르마-오일러정리의의해 (mod 000) 이므로, y y 0 (mod 400) 즉 y z (mod 5) 이다. 다시, Á(5) = 0 에서 0 (mod5) 이므로 z (mod 0) 이제거꾸로대입해가면 y (mod 5) y 48 (mod 400) x 3 48 (mod 000) 이된다 (mod 000) (mod 000) (mod 000) (mod 000) 이과정을계속하면 (mod 000). 따라서답은 4이다. } 풀이 이항전개를이용하면 3 4n ( 0) n (n)0 n(n ) + 0 0n +00n(n ) (mod 000) 이므로,3 00 (mod 000) 임을알수있다. 이제 (mod 000) 이고, 00? (mod00) 을구해보자. 페르마 - 오일러정리에서 0 = Á(5) (mod 5) 이고 3 (mod 5) 이므로, (mod 5), 00 5 (mod 00) 이다. 그럼 (mod 000) 이된다. 그러므로, 답은 4. } 8. 수열 a k = n k (k =0; ;:::;003) 이다음성질을갖도록하는자연수 n>003 이존재함을증명하여라 : 각각의 0 m; k 003 에대해 m>k 이면 a k 는 a m 을나눈다. ( 폴란드 003 차 -) 증명 n k j n k+ = n k n k+ k 이므로 n k n+ k+ 이항상정수이면, 즉 k+ 이항상정수이면됨. n = 003! 이면충분함 <x<y<z<p 를만족하는소수 p 와정수 x, y, z 가주어져있다. x 3, y 3, z 3 을 p 로나눈나머지가모두같다면, x + y + z 은 x + y + z 로나누어떨어짐을보여라. ( 폴란드 차 -4)

35 . 수론중급문제 35 증명우선 p 5. () p j x + xy + y ;y + yz + z ;z + zx+ x.() 에서둘을빼면 () p j x + y + z 이고,() 의셋을모두더한후 () 를적당히빼면 (3) p j x +y +z. x+y +z <3p 임에서 x+y+z = p 또는 p 인데, p일때는자명하고,p일때도x + y + z 와 x + y + z 의홀짝이같으므로역시자명함. 30. 소수 p, q, r 이있다. p 는 qr 의약수이고, q 는 rp 의약수이고, r 은 pq 의약수라고한다. pqr 의값으로가능한것을모두구하여라. ( 영국 003/004 차 -5) 풀이 ( 경북과학고 학년심영준 ) p q r 이라고하자. 우선아래의세식이문제의조건에서성립해야한다 (k, m, n은정수 ). kp = qr mq = pr nr = pq 3 이제여러가지경우를살펴보자. (i) p 가짝수일때 ( 즉, p = 일때 ), 3 에서, nr =q r < r 이므로, n =. 즉, r =q ᄀ. ᄀ을 에대입하면 mq =(q ), 즉 3=(4 m)q. 여기서 q j 3 이므로 q =3 뿐이다. 그럼, r =5. 따라서, (ii) p 가홀수일때,,, 3 에서 qr p (p; q; r) =(; 3; 5) 이고 pqr =30, pr, pq q r 이모두정수이므로, 셋의곱역시정수이다. (qr )(pr )(pq ) P = pqr = (pqr) pqr(p + q + r)+(pq + qr + pr) pqr 이정수이려면, pq + qr + pr 이정수이면된다. 그런데, p:q:r 3 에서 pqr r ; p ; q 3 이므로 pq + qr + pr pqr = r + p + q pqr < 로이것은정수일수없어서모순. 따라서, p가홀수일때는만족하는 (p; q; r) 이없다. 그러므로가능한 pqr의값은30뿐이다. 답 } 3. (x + 5)(y +7)=p z 을만족하는음이아닌정수 x, y, z 와소수 p 가존재하는지결정하여라. ( 폴란드 003/004 차 -) 풀이 p 6= ; 3 이므로 p 5. p (mod3; 4) 이므로 p (mod). 그럼 x +5 나 y +7 모두 p 의홀수승이어야함. 즉, z 는짝수. 그럼준식은 (mod) 로모순. 즉, 해는없음. } 3. 자연수 m, n 이식 m +n = 37(m n) 을만족시킬때, m + n 의값은얼마인가? ( 한국 004 차 -S3) 풀이좌변이짝수이므로우변도짝수. 즉, m과 n은홀짝이같다. 그럼 m 과 n 도홀짝이같으므로좌변은다시 4의배수이고, 따라서우변의 m n도 4의배수. m = n +4k (4는자연수) 를준식에대입하면 (n +4k) + n = 37 k (n +k) = k(37 4k)

36 36 수론 로정리된다. 좌변이양수이므로우변도양수, 따라서 0 < 4k <37. 그럼 gcd(k; 37 4k) =gcd(k; 37) 은 37 일수없으므로 일수밖에없다. 서로소인두수의곱이완전제곱수이므로둘다각각완전제곱수 k = a,37 4k = b 꼴이어야한다. 4a + b =37 을만족하는경우는 (a ;b )=( ; ) 뿐이다. 따라서, k =4,n =4,m =30 이되고, m + n = 44. } 33. 피보나치수열은 f =0,f =, 그리고 n 에대해 f n+ = f n+ + f n 으로정의된다. 단조증가하는정수들의등차수열중에, 피보나치수를하나도갖지않는것이있음을증명하여라. ( 북유럽 004-) 증명 f n 은순환잉여수열. 적당한 m에대해mod m으로나타나지않는것이있으면충분. 즉,k+4, 6, 7 등. 34. n 0 + n 5 + 이소수인자연수 n 을모두구하여라. ( 오폴 004-4) 풀이 n 0 + n 5 +=(n + n +)(n 8 n 7 + n 5 n 4 + n 3 n +) 인수분해이용. } 35. 정확히 p n 개의양의약수를갖는자연수 n 을모두찾아라. ( 폴란드 004 차 -4) 풀이 n은제곱수, 또홀수. p n = m =k + 으로두자. m보다작은약수와 m보다큰약수가쌍을이루므로각각 k개씩인데, m보다작은홀수도 k개이므로그것들이모두약수. 특히 k j (k +) 에주목하면 k j 4, 즉 k =0;. 고로 n =; 9뿐. } 36. 방정식 n(m + n ) m(m + n +47) = 0 을만족시키는자연수쌍 (m; n) 의집합을 A = f (m i ;n i ) j i =; ;:::;kg 라할때m + m + + m k 의값은? ( 한국 005 차-S7) 풀이 ( 대전전민중 3학년정명진, 수정됨 ) n(m + n ) = m(m + n +47) 에서모든인수가양수이고 m + n <m + n +47 이므로 n>m이어야한다. n = m + k 라두자 (k ). (m + k)(m + n ) = m(m + n +47) m + km + kn k =47m km +k m + k 3 k =48m km +(k 4)m + k(k ) =0 k 4 이면 ( 좌변 ) 4m 8m +30> 8(m ) 0 으로성립하지않는다. 따라서, k =; ; 3 만가능. ² k =이면 ; m 3m =0. ) m =3. ² k =이면 ; 좌변이홀수이므로모순. ² k =3이면 ;3m 5m +=0. (m )(m 4) = 0. ) m =; 4. 따라서, 가능한 m은, 4, 3뿐이고,+4+3=8 답 } 주이외에몇몇학생이제출한풀이에서는, km +(k 4)m + k(k ) = 0 에서이식의정수해를가져야하므로근과계수와의관계를이용, k 4 k 와 (k ) 이정수여야한다고주장하며풀이를쓴사례가있었다. 그러나, 정수해를갖는것과두근이모두정수라는것은다른얘기이므로이런풀이는직접성립하지않는다. 37. 이차방정식 (bc )x +(a b + c abc)x + ab =0 이정수근을갖도록하는한자리의자연수 a, b, c 에대하여 00a +0b + c 의최대값을구하여라. ( 한국 005 차 -S7)

37 . 수론중급문제 37 풀이 ( 제주과학고 학년강진호, 수정됨 ) 정수계수이차방정식이므로정수근을갖는다면판별식이완전제곱수라야한다. D =(a b + c abc) 4(bc )(ab ) = a b c abc(a + b + c)+(a + b + c )+(ab + bc + ca) 4 =(abc) (abc)(a + b + c)+(a + b + c) 4 =(a + b + c abc) 4=m 즉,(a + b + c abc) m =4 이다. 완전제곱수들 0,, 4, 9, 6, ::: 의분포를관찰하면두완전제곱수의차가 4 가되는경우는 4 0 일때뿐이다. 따라서, m =0 이고 a + b + c abc = ( ) 이다. 그럼 a(bc ) = b + c, 즉 bc j b + c 이다. b + c =0 일때는 bc =0 이기도하므로, bc b + c 이항상성립한다. 즉, 이다.( ) 은대칭식이므로, 마찬가지로 (b )(c ) 4 (a )(b ) 4; (a )(c ) 4 이기도하다. a>5 이면 b = c = 이어야하므로문제의식에서최고차항이 0 이되어이차방정식이아니므로곤란하다. a =5 일때는 b ;c 이고, 이경우 (a; b; c) =(5; ; ) 또는 (5; ; ) 일때 ( ) 이만족된다.(a; b; c) =(5; ; ) 일때문제의방정식은 x 6x +9=0 으로 x =3 의정수근을갖는다. 따라서, 00a +0b + c 의최대값은 5 이다. } 38. 어떤적당한서로소인자연수 a, b 에대하여, a + b 와 a005 + b 005 모두구하여라. a + b 의최대공약수가될수있는정수를 ( 한국 005-S) 풀이 a b 005 a + b = a 004 a 003 b + a 00 b ab b 004 =(a + b)(a 003 a 00 b +3a 00 b 004b 003 ) + 005b 004 이다. 이것과 gcd(a + b; b 004 )= 임으로부터 d =gcd µa + b; a005 + b 005 =gcd(a + b; 005b 004 )=gcd(a + b; 005) a + b 따라서, d =, 5, 40, 005 만가능하다. 그리고, 이각각의경우에대해 (a; b; d) =(5; ; ); (3; ; 5); (00; 0; 40); (; 003; 005) 의실제예가존재한다. 답, 5, 40, 005 } 39. 다음을만족하는함수 f : Z Z! R 을모두구하여라. (i) 임의의정수 x; y; z에대해f(x; y) f(y; z) f(z;x) =. (ii) 임의의정수 x에대해f(x +;x)=. ( 루마니아 005 지역예선 y0-)

38 38 수론 풀이 ( 대전과학고 학년송지용 ) (i) 에서 y; z 에모두 x 를대입하면 f(x; x) 3 =, 즉 다시 (i) 에서 z 에 x 를대입하고 f(x; x) 를소거하면 f(x; x) = f(x; y)f(y; x) = 이로부터,(i) 의양변에 f(x; z) 를곱하면다음의관계식을얻을수있다. 이로부터, x>y 일때 f(x; y)f(y; z) =f(x; z) f(x; y) =f(x; x )f(x ;y) = = f(x; x )f(x ;x ) f(y +;y) = = {z } x y x y 개 와같이분해하여계산할수있다. 또한, 앞의성질들에의해이것은 x = y, x<y 일때도성립함을확인할수있다. 따라서, 답은 f(x; y) = x y. } 40. x 는정수이고 y, z, w 는양의홀수들이다. x yzw x yz 가 7 로나누어떨어짐을증명하여라. ( 아일랜드 005-8) 증명 x 7 x (mod 7) 이므로 y zw y z (mod 6) 임을보이면충분. 이것은또 z w z (mod 8) 이면충분. z (mod) 이므로이것은당연. 4. 다음을만족하는최소의자연수 a 를구하여라 : a p a p 을만족하는소수 p 와자연수 b 가존재한다. ( 오폴 005-4) 풀이 a p a = a(a p ). gcd(a; a p ) =. gcd(a; p) =일때와그렇지않을때로나눠서풀자. () gcd(a; p) =일때 : 페르마소정리에의해 p j a p 이므로 a = A, a p =pb 꼴. A (p ) =(A p +)(A p ) = pb. p j A p 이므로 (A p +;A p ) = (x ;py ) 꼴 (i) 또는 (x ; py ) 꼴 (ii) 이다.(i) 의경우 : p가홀수이면 A p +=x 이양의제곱수에 을더했더니다시제곱수가되는꼴이라성립할수없고, 따라서 p =. A =B 의가장작은해는 (A; B) =(3; ) 이므로 a =9가이경우의최소이고, p =,b =6의해가실제로존재한다.(ii) 의경우 : 이경우는 A가홀수일때만생기므로, A 3 이라서어차피 a 9 가되어 (i) 에서얻은것보다 a가더작은경우는없다.()gcd(a; p) =a 일때 : a = pk 라하자. k((pk) p ) = b 에서두인수가서로소이므로 k = A,(pk) p =B 꼴. 여기서 p가홀수이면뒷식이제곱수에서 을뺐더니양의제곱수가되는꼴이라곤란하고, 따라서 p =. A =B 의가장작은해는 (A; B) =(; ) 인데이것은 b =이되어문제의조건에위배되고, 그다음해는 (A; B) =(5; 7) 인데이것은 (i) 에서구한것보다훨씬해가커진다. 답 9 } = b 4. n n +과 (n) n +이둘다소수가되게하는자연수 n을모두구하여라. ( 폴란드 005 차-) 풀이 n이홀수인소인수를가지면 n n + 이인수분해되어곤란. 즉, n = m 꼴. 그럼 n n + = m m + 임에서 m도홀수인소인수를가지면곤란. 즉, n =또는 n = k 꼴. 그럼 (n) n += (k +) k + +에서 k 이면또끝. 즉 n =; 일때만가능하고둘다 ok. }

39 . 수론중급문제 n +05 가완전제곱수가되는음이아닌정수 n 을모두구하여라. ( 폴란드 005/006 차 -) 풀이 mod 3으로보면 n은짝수. n =m 이라하고 m +05=x 이라하면 (x + m )(x m )= 05 = 경우나눠풀면 답 n =4; 6. } 보다큰소수 p 와자연수 a, b, c 가 a + b + c = p + 을만족하고, a 3 + b 3 + c 3 은 p 로나누어떨어진다. a, b, c 중에 인것이있음을증명하여라. ( 폴란드 005/006 차 -7) 증명 a 3 +b 3 +c 3 (a+b+c)((a+b+c) 3(ab+bc+ca)) +3abc 3abc 3(ab+bc+ac) 3(abc (ab + bc + ca)+(a + b + c) ) = 3(a )(b )(c ) (mod p). 45. 세자연수 a, b, c 에대하여,gcd(a; b; c) = 이고 a + b + c =(ab + bc + ca) 일때, a, b, c 가모두완전제곱수임을보여라. ( 한국 006-3) 증명 ( 서울중산고 학년최서준 ) a + b + c =(ab + bc + ca) () a (b + c)a +(b c) =0 a 는자연수이므로판별식은제곱수여야한다. D 4 =(b + c) (b c) =4bc = k (k 는정수 ) 여기서 bc 도제곱수임을알수있다. 이제 b, c 가서로소임을보이면충분하다. b 와 c 의공통소인수 p 가존재한다고하자. 원식에대입하면 p 는 a 을나눈다. p 가소수이므로 p 는 a 를나눈다. 이것은 gcd(a; b; c) = 에모순. 따라서 b, c 의공통소인수는없고, 즉 b 와 c 는서로소이며, bc 가제곱수이므로 b, c 모두각각제곱수이다. 문제는 a, b, c 에대해대칭적이므로, 같은방법으로 a, c 도모두제곱수라는결론을내릴수있고, 따라서 a, b, c 는모두제곱수이다. 46. n = 006 일때다음방정식의정수해 x, y, z 가존재하는가? z =(x +)(y ) + n n = 007 일때는어떤가? ( 아일랜드 006-) 풀이 n = 006 일때는 (x; y; z) =(4; 6; 5) 등의해가존재한다. n =007일때는mod 8로해보면해가없음을확인할수있다. } 47. 자연수 a, b, c, x, y, z 가다음을만족한다. a + b = c ; x + y = z ; jx aj ; jy bj 집합 fa; bg 와 fx; yg 가같음을보여라. ( 폴란드 006 차 -) 증명 (a; b) 6= (x; y) 라하자.WLOGjx aj =,WLOGx = a+. y = b 이면 z = a +a++b = c +a + 인데, c>a 이므로이것은 c <z < (c +) 임을의미하여모순. y = b + 일때도비슷하게좀해보면 (c +) <z < (c +) 로모순. y = b 일때만보면됨. c z =(a + b )(x + y ) (ax + by) 이므로 cz ax + by. (c z) = c cz + z (a ax + x )+(b by + y ). 즉, jc zj. 막노동좀하면확인은되는데막노동피하고깔끔하게끝내는방법이없나 k +4m 과 m +5k 가모두완전제곱수가되는자연수쌍 (k; m) 을모두구하여라. ( 폴란드 006/007 차 -) 풀이 A := k +4m>k 이고 A 6= (k +) 이니까 (4m과 k +을홀짝이달라서 ), A (k +), 즉 m k +. 그럼 m <B:= m +5k<m +5m<(m +3) 이니까 B =(m +) 또는 (m +). 즉,5k =m + 또는 5k =4m +4. 전자일때 A = k +0k < (k +5) 이고, A =(k +) ; (k +3) ; (k +4) 일때각각풀어보면됨. 후자일때 A = k +5k 4 < (k +3) 이고, A =(k +) 일때만풀어보면됨. 답 (,), (8,9), (9,) }

40 40 수론 49. 두정계수다항식 p 와 q 가비슷하다는것은차수가같고계수가순서만다를수있음을의미한다. (a) p와 q가비슷한다항식이면 p(007) q(007) 은짝수임을보여라. (b) 비슷한임의의두다항식 p, q에대해p(007) q(007) 이항상k의배수가되는정수 k> 가존재하는가? ( 이탈리아 007-) 풀이 (a) p(007) q(007) p() q() = 0 (mod ). (b) 존재한다. p(007) q(007) p() q() = 0 (mod 006). 006 대신 006 의약수로해도다된다. } 50. 짝수개의 ( 서로다를필요는없는 ) 소수들의곱으로나타나는자연수를하얗다고말한다. 도하얗다. 그밖의자연수들은까맣다고말한다. 하얀약수들의합과까만약수들의합이같은자연수가존재하는가? ( 폴란드 차 -) 풀이 ( 하얀약수들의합 ) ( 까만약수들의합 )=( p + p +( p ) e ) ( p r + + ( p r) er )=0 을의미하는데, 이것은 p i 라서어느항도 0 이될수없어불가능. }. 수론고급문제. 다음조건을만족시키는방정식 x y + z = 의해가무한히많음을보여라 : x, y, z 는서로다른자연수로, 어느두수를택해도그곱이남은수의배수이다. ( 소련 ) 증명 x =(c )(c c ), y = c(c c ), z = c(c ), c는 이상의자연수로두변 x y+z =이고위의조건을모두만족한다. 따라서이런 x, y, z는무한히많이있다.. 자연수 n 에대해 D(n) 을다음과같이정의하자 : D() = 0 이고, p 가소수이면 D(p) =, 그리고임의의두자연수 u, v 에대해 D(uv) =ud(v) +vd(u). () 이들성질에의해 D(n) 이모순없이잘정의되고, 또유일하게결정됨을보여라. () D(n) =n 이되는n은어떤값들인가? (3) D (n) =D(D(n)) 등으로정의할때, m!일때의d m (63) 의극한값을구하여라. (Putnam 950-A5) 풀이 ( 김대성 )() 주장 n = p p p m (p ;p ; ;p m 은서로다를필요가없고, 모두소수이고이러한표현은순서를무시하면, 유일하다 ) 이라면, D(n) = P m n k= 이다. p k 증명 m 에대하여, 즉 n 의소인수의개수 ( 중복한것을세어서 ) 에수학적귀납법을적용하겠다. ) D(n) == p p 이므로성립 ) m 이하에서된다고하자. n = p p p m p m+ 하자. D(n) =D(p p p m) p m+ ) = p p p md(p m+ )+p m+ D(p p p m) X m p p p m = p p p m + p m+ p k= k = p p p m p m+ mx p p p m p m+ + p m+ p k= k m+ X n = p k= k 이므로 m + 에서도성립한다. ( 잘정의됨에대한보충 : 이증명과정에서 u = p p p m;v = p m+ 라고했는데, p ;p ;p 3 ; ;p m+ 의재배열을 q ;q ; ;q k ;r ;r ; ;r l (k + l = m +;k ;l ) 라고하고, u = q q q k ;v = r r r l 라고해도, 위와비슷하게전개하여같은결과에이르게된다.)

41 . 수론고급문제 4 () 주장 n = p p (p 는소수 )$ D(n) =n 증명 n이두개이상의서로다른소인수를가진다고하자. n = p a pa pa m m (p ;p ; ;p n 은서로다른소수, a i ) 라고하자.() 의내용을적용시켜보면, D(n) =a p a p a pam m + a p a pa p am m + + a m p a pa pam m 이고, D(n) =n 을적용해보면, mx k= a k p a pa pa k k 양변을 p a p a p a m m 로나누면, p a m m = p a pa pa m m mx a k p p p k p k+ p m = p p p m k= 이고 a k p k 이면바로모순이되니까, a k <p k 그런데, a k p p p k p k+ p m 항만제외하고모두 p k 의배수이고, p p p k p k+ p m 은 p k 와서로소이므로, a k 는 p k 의배수이것은모순. 그러므로 n 은하나의소인수만가져야하고, p p 꼴만됨을쉽게확인할수있다. (!) 너무쉽기때문에생략 (3) 위의결과를가지고, 몇차례계산을해보면, D(63) = 5;D(5) = 0;D(0) = 4;D(4) = 44;D(44) = 48 = 4 3=D 5 (63) 임을확인할수있다. n = m a(m ;a 은홀수 ) 라고하면, D(n) =D( m a) m a m + m = m (ma +)> m a 이다. 즉, D(n) >n 이고, 또 D(n) 은다시 m 으로나누어진다. n = m+ a(m ;a 은홀수 ) 라고하면, D(n) =D( m+ a) m a (m +)+ m+ = m (ma + a +)> m+ a 이다. 즉, D(n) >n 이고, 또 D(n) 은다시 m 으로나누어진다., 에서 k 5 이면, D k+ (63) >D k (63) 임을알수있다. 즉, 로발산한다. } 3. 실계수다항식 f(x) =a 0 + a x + + a nx n 은모든정수i에대해f(i) 가정수가된다고한다. n!f(x) 는정계수다항식임을증명하여라. (Putnam 953-B) 증명 f(x) =b x x b + + x bn n 꼴로변형시켰을때, 모든정수 i에대해f(i) 가정수가될필요충분조건은 b 0 ;:::;b n 이모두정수인것. k!a k = b k + 정수꼴이되므로이것이면문제를확인하는데에는충분. 4. n 이자연수이고 + n +4 n 이소수이면, n 은 3 의거듭제곱임을보여라. ( 불가리아 98 3 차 -4) 증명 ( 김범수 )+y +y =0의한근을w라하면m이 3의배수가아닐때 +y m +y m 은 y = w 를근으로가지므로 +y +y 을인수로갖는다. n이 3의거듭제곱이아니라면 n =3 k m 이라할때 (3 - m), m> 이다. 그럼 x 3k = y 로치환하여생각하면 +x n + x n =+y m + y m 은 +y + y 을인수로갖는다. 따라서,+ n +4 n 은 + 3k +4 3k 의배수이고, 그래서소수가될수없다.

42 4 수론 5. 다음방정식의모든정수해 (x; y) 를구하여라. ( 폴란드 98 3 차 -5) x 3 + x y + xy + y 3 =8(x + xy + y +) 풀이이식의정수해가존재하지않음을보이고자한다. 좌변을인수분해하여방정식을다음과같이고쳐쓸수있다. (x + y)(x + y )=8(x + xy + y +) 우변은 8의배수, 즉짝수이므로, 좌변의 x, y는동시에홀수이거나동시에짝수이다. 그리고이식은대칭식이므로일반성을잃지않고 jxj jyj라고할수있다. 따라서 x + y < 8 일때는 (x; y) 가 (0; 0), ( ; ), ( ; ), (0; ) 등일경우만살펴주면되고, 이때방정식이만족되는경우가없음을확인할수있다. 이제 x + y 8 인경우만을살펴보면된다. 준식은 x + y 8 와같이변형할수있는데, x + y jxyj 임에서 = x + xy + y + x + y =+ xy x + y + x + y xy x + y 이므로, < x + y 즉 4 <x+ y 3 이다. x + y는짝수이므로 6, 8, 0, 만을취할수있고, 이제준식을기본대칭식 x + y와 xy로만변형하여구성한식 (x + y) 3 =8(x + y) +xy(x + y 4) + 8 에이수들을대입하여각각풀면, x + y =0,xy =6 일때만정수해 (x; y) =(; 8) 을가진다는것을확인할수있다. 결국구하고자하는정수해 (x; y) 는 (; 8) 과 (8; ) 뿐이다. } (x + y) 별해간단히홀짝에관한조사로 x + y가짝수여야함을알수있다. = T, xy = S로치환하 자. T, S는모두정수이고준식은 T (4T S) =8(4T S +) 가되고이를정리하면 8T 3 4ST 3T +8S 8=0 S(8 4T )= 8T 3 +3T +8 S = T 3 +8T + =T 4T 8+ 8 이게되고 S, T 모두정수이므로 T 는 8의약수여 T T 야한다. 따라서 T =; 3; 4; 5; 8; ; 0중하나이고이때각각의S값을구해보면 T =5,S =6이유일하게 x, y가정수인해라는것을알수있다. x + y =5,xy =6을풀면 (x; y) =(; 8), (8; ) 가유일한 해임을알수있다. } 6. 모든자연수 n 에대해서 k n + 이항상합성수가되는자연수 k 가존재함을보여라. ( 미국 98-4) 증명 k m+ + 0(mod3) 이되려면k (mod3). 이제 k 4 m + 꼴만합성수가되도록하면된다. k 4 m+ + 0(mod5) 가되려면k (mod5). 이제 k 6 m + 꼴만합성수가되도록하면된다. k 6 m+ + 0(mod7) 이되려면 k (mod7). 이제 k 56 m + 꼴만합성수가되도록하면된다. 57은소수. k 56 m+ + 0 (mod 57) 이되려면 k (mod 57). 이제 k m +꼴만합성수가되도록하면된다 은소수. k m+ + 0 (mod 65537) 이되려면k (mod 65537). 이제 k ( 3 ) m + 꼴만합성수가되도록하면된다. 페르마소수의내용에서 3 +=pq 로두소수의곱으로나타내어짐이알려져있다 (p =64,q = ). 따라서, k ( 3 ) m+ + 0(modp) 이고 k ( 3 ) m + 0(modq) 가되도록하면된다. 즉, k (mod 3; 5; 7; 57; 65537;p) 이고 k (modq) 인 k는중국의나머지정리에의해존재하므로, 그런 k를잡으면항상합성수.

43 . 수론고급문제 43 증명 k a im+b i + 0(modp i ) 를만족하는 (a i ;b i ;p i ) 들의집합 X를잘잡아서 (p i 들은모두서로다른소수 ) a i m + b i 꼴의수들을모두모은것이자연수전체를덮도록할수있으면된다 ( 그후는중국의나머지정리로완성 ). 이런덮기의예는다음과같은것들이있다 : a i b i p i a i b i p i a i b i p i j m 3n + 인자연수 n 이존재하는자연수 m 을모두구하여라. ( 오스트리아일반화 ) 보조정리 7 j k 3 +이면 7 j k +이다. 증명은 k k +이 7의배수가될수없음을막노동으로확인하여... 즉, 위의보조정리로부터 7 j m 3n +() () 7 j m +이됨. } 8. u ;u ;u 3 ;::: 는점화식 u n+ = u n+ un 을만족하는정수들의수열이다. u =39이고 u =45라하자. 이수열의무한히많은항이 986의배수가됨을증명하여라. ( 캐나다 986-5) 증명 mod 986 으로 (u n+ ;u n ) 은유한개밖에없으므로 (u m+ ;u m ) (u m+t+ ;u m+t ) (mod 986) 인양의정수 m, t 가존재한다. mod 986 으로 (u n+ ;u n) 이주어지면 u n+ 도 mod 986 으로결정되므로, u n mod 986 은 n m 에서순환한다. 또한, mod 986 으로 (u n+ ;u n+ ) 이주어지면 u n 도마찬가지로결정되므로, u n mod 986 은 n m 에서도순환하고, 따라서, u n+t u n (mod 986) 이모든 n 에대해성립한다. u 3 =45 39 = (mod 986) 인항이있으므로, 이것이순환하여 u n 0 (mod 986) 이무한히많게된다. 그러므로,986 의배수가되는항이무한히많다. 9. n 은자연수, p 는 p>3 인소수라하자. xyz = p n (x + y + z) 을만족하는자연수 (x; y; z) 의쌍 ( 순서에관계없이 ) 을최소 3(n +) 개찾아라. (IMO-LL 986, 체코슬로바키아출제 ) 풀이 ( 울산학성고변명광 ) z = p k (0 k n, k 는정수 ) 라면 xy = p n k (x + y + p k )=p n k (x + y)+p n (x p n k )(y p n k )=p n + p n k ) 0 k n 일때, n k >0이므로 (x p n k )(y p n k )=p n ( + p n k ) i) x p n k = p t (0 t n) 라하면 y p n k = p n t (p n k +) x = p n k + p t ; y = p n k + p n t (p n k +) (x; y; z) =(p a + p b ; p c + p d + p e ; p f ) 꼴이고 p>3 인솟수이므로 k, t 하나에서로다른근이결정된다. ii) p n k + 은짝수 (p >3 인솟수 ) x p n k =p t 라면 y p n k = p n t pn k + x = p n k +p t ; y = p n k + p n t pn k +

44 44 수론 ³ (x; y; z) = p a + p b ; p c + p d pe + ; p f 꼴이므로 k, t하나에서로다른근이결정된다. ) n =m 이면 0 k m ³ ; 0 t m 0 k n i), ii) 에의해가능한근의갯수 V =m m =4m V 3(n +)=4m 3 m = m(4m 6) (m ) m = 일때는 V =4 의근은 k =0 에서구한근이다.(z =) z = p 라면 (x )(y ) = p + (x; y) = (; p+); (x; y; z) = (; p+; p); µ 3; p +3 µ 3; p +3 ; p 그러므로 m =( 즉 n =) 일때근의갯수는 6 개이상이므로 V =6 3(n +) n =m (m ) 인 n 에대해성립한다. 3) n =m 이면 (m ) 0 k m 0 t m i), ii) 에의해가능한근의갯수는 V = (m +)(m +)=4m +6m + V 3(n +) = 4m +6m + 3(m +)=4m 0 (m ) 그러므로 n =m 인 n 에대해성립한다. ), 3) 에의해모든자연수 n 에대해성립한다. } 0. 다음을만족하는모든정수 (x; y; z) 를구하여라. ( 오스트리아 TST 987) xz = y x + z =987 풀이 987은소수이고y는짝수임은자명하다. x 6= z이고 x =987일수없다. 그리고 x와 z는서로소이다. 그러므로 xz = y 에서 x, z중한수는제곱수이고한수는제곱수의두배이다. 이로부터 x =a ; z = b ; gcd(a; b) =; 그러므로, a + b = 을일반소수 p 로바꾸면 a + b = p (6) () 의식의해는 ( 약간의정수론을사용하면 ) p 를 8 로나눈나머지가, 3 일때만유일하게존재한다. 그리고 a, b 는홀수이다. b =987 a 에서 a<3 이고, a =3; 9 일때는 987 a 는제곱수가아니다. 그러나 a = 7, b = 59, b =3 일때 987 a 은제곱수이다. 그러므로유일한해는 x =a = 3 6 = 458, z = b =3 = 59, y = p xz = = 4 이다. }. 정계수다항식 w(x) 에서 jw(n)j 의자릿수의합을 p n 으로정의하자. p ;p ;p 3 ;::: 에는무한히많이나타나는수가있음을보여라. ( 폴란드 차 -3)

45 . 수론고급문제 45 증명 w(x) = a k x k + + a x + a 0 이라하자. n 의자릿수의합은 S(n) 으로쓰기로하자. 즉, p n = S(jw(n)j). 만일모든계수가양이라면, 계수의최대값보다더큰임의의 0 m 에대해 w(0 m ) 에는각각의계수가 m 자리씩그대로드러나므로, p 0 m = S(w(0 m )) = S(a k )+ + S(a )+S(a 0 ) 으로원래계수들의자릿수의합을모두합한값 ( 상수 ) 이된다. 이런 m 은무한히많으므로이값도무한히많이나타난다. 이제계수중에음수가있을때를보자. 일단최고차항의계수가양수라고가정할수있다. u(x) :=w(x + N) 으로두어서 u 의모든계수를양수로만들수있다면 ( ) 앞에서본것처럼 u 에대해증명해줄수있다 (p N ;p N+ ;p N+ ;::: 만을살펴보는것이됨 ). ( ) 을증명하자. k 차항을양수로만들수있음만보이면충분하다 ( 왜냐하면, x + N 을다시반복했을때이미계수가양수인높은차수의계수들은계속양수임이유지되므로, k 차항을이용해 k 차항의계수를양수로만들고, 이런식으로귀납적으로바로다음항을정복해나가면된다 ). u(x) 의 k 차항의계수는 u k = kna k + a k, 이것은 N>ja k j 로하면양수임이충분하다.. x, y, z가정수일때다음방정식을풀어라. (IMO-LL 987, 그리스출제 ) 8 x =9 y +87 z 풀이하면 ( 대전한밭고김복기 ) x 0 이면 y<0;z < 0 이어야한다. 이때, x = u; y = v z = w 라 8 u = 9 v +87 w 8 = 9 v + 87 w = 9v +87 w 9 v 87 w ) 9 v 87 w = 8 u (9 v +87 w ) 0 (mod ) ) 모순 x>0 이면 : y =0 일때 8 x =+87 z z 가홀수인경우 6 z +( ) z 0(mod) gcd(6; ) = 이므로 6 z 0 인 x 가존재하지않는다. z 가짝수인경우 z =k 라놓으면 ) 모순 z =0 일때 8 x =9 y + 0 +( ) z +( ) k + (mod 4) + (mod9) ) 모순 x; y; z가모두양의정수이어야한다. 8 x =9 y +87 z 이므로 x, 따라서 8 x =(4 7) x 는 6의배수. 0 3 y +7 z (mod 6): (3 ; 3 ; 3 3 ; 3 4 ) (3; 9; 5; ) (mod 6); (7 ; 7 ) (7; ) (mod 6) 이므로 y (mod 4);z (mod ): +(3 9) z (mod 9) 이므로 z. 그런데 z (mod) 이므로 z 3: 따라서 87 z =(3 9) z 은 7 의배수이다. ( 8) y 8 y (mod 7) (y 는짝수 ) 8 의 order (mod 7) 은 6 이므로 y 0(mod6) 위에서 y (mod4) 이므로 y 6 (mod ). 0 ( ) y +3 z (mod 7) ( ) y ( ) k+6 ( 6 ) k+ (mod 7) 이므로 3 z (mod 7)z 3 (mod 6) (mod 3) 으로다시쓰면 x 6 y +( 4) z 6 y 4 z. 따라서 6 y x +4 z (mod 3) 6 y 6 k+6 6 k 6 6 ( ) (mod 3) 4 z 4 6m+3 4 6m 4 3 ( ) (mod 3)

46 46 수론 이므로 x 0(mod3): 그런데,gcd(; 3) = 이므로만족하는 x 는존재하지않는다. 따라서만족하는정수해는없다. } 3. p 가자연수일때다음방정식의정수해를모두구하여라. ( 루마니아 TST 988-C4 변형 ) jx pxy y j = 풀이 ( 대전과학고 년이효상학생의아이디어를정리 ) 준식은 x pxy y = 과동치이다. i) x>0, y 0일때, y =0이면 x =이다. y>0이면 x(x py) =y 0이므로 a = x py 0, x py y이다. x = py + a 를준식에대입하여 y>0, a 0, y pya a = 을얻는데이식은준식을꼭닮았다. 따라서이과정을반복하면 x = a 0, y = a 이라할때귀납적으로 9n 0; a i = pa i + a i+ ( i n); a n =; a n+ =0 의감소하는수열을얻는다. 이제과정을거꾸로생각하여 b 0 =0,b =,b i+ = pb i + b i 의점화식을생각하면일반항 b i = i (p ) i (p ) ; 단,, 는 x px =0의두실근 : 이구해지고,(x; y) =(b n+ ;b n )(n 0) 이이경우의일반해이다. ii) 원점 O(0; 0) 은해가될수없으므로 R fog 을 (x > 0;y 0); (x 0;y > 0), (x <0;y 0); (x 0;y < 0) 의네영역으로나눌수있는데그첫영역에해당하는경우는 i) 에서풀었다. x 0, y>0 의두번째경우는 x 0 = y, y 0 = x 라두면 x 0 > 0, y 0 0, x 0 px 0 y 0 y 0 = 이되어 i) 에서 (x 0 ;y 0 )=(b n+ ;b n )(n 0), 즉 (x; y) =( b n ;b n+ )(n 0) 이다. 같이하여 (x; y) =( b n+ ; b n ), 또 (x; y) =(b n ; b n+ )(n 0) 을구하면모든경우가구해졌다. 답은 (x; y) =(b n+ ;b n ); ( b n ;b n+ ); ( b n+ ; b n ), 또는 (b n ; b n+ )(n 0) 이다. } 4. 어떤 n 자리의자연수는자리수의순서를순환적으로바꾸었을때나타나는수들이모두 989 의배수가된다고한다. 이런 n 의최소값은얼마인가? 또, 이런자연수중에서최소인것은무엇인가? [370 의자리수를순환적으로바꾸면 703, 37, 370 을얻는다.] ( 아일랜드 989-5) 풀이 989 = 순환적으로늘 989 의배수가되는수를착한수라고하자. N = a na n a 이착한수이면 N 0 = a n a a n 도착한수이므로 (N) =0N N 0 = a n (0 n ) 도 989 의배수. 0 n 이 9 의배수임은자명하므로, (N) 이 3 과 7 의배수이면충분한데, a n 은 0 이아닌한자리의수이므로 3 이나 7 의배수가될수없어서 0 n 이 3 과 7 의배수일때를찾아야한다.0 3 (mod 3), 0 6 (mod3) 이므로우선 6 j n. 또,0 8 ( ) 4 =6 (mod 7) 이고 0 6 (mod 7) 이므로 6 j n. lcm(6; 6) = 48 이므로, 989 j (N) () 3; 7 j 0 n () 48 j n 따라서, 가장작은 n 의후보는 48 이다. 또한 989 j 0 n 이기만하면 N 을순환시킨임의의 N 0 에대해서항상 989 j (N 0 ) 이므로,989j N 이기만하면된다는것이고, n =48 일때의 989 의배수가되는최소의 N 을찾으면된다 ( 그런 N 은틀림없이존재할테니우선 n 의최소값은 48). N = k 라하자. k 이면 9 의배수인 48 자리의수들이차례로모두나열되므로,3; 7 j N 이되는최소의 k 를구하면된다 k 0 4k 0 4k 0(mod3) 을풀면 k 5 8, 즉 k 4(mod3). 또, k 0 7 8k ( ) 3 0 8k 6 8k 0(mod7) 을풀면 4k 3 4, 즉 k 7 0, 즉 k 5(mod7) 이다. k 4; 5 (mod3; 7) 이면되므로, k (mod 3; 7), 즉 k (mod 3 7). 이런최소의 k 는 3 7 = 99 이다. 즉, 최소의 N 은 } 5. f : N 0! N 0 이고모든 n N 0 에대해서 f(f(n)) + f(n) =n +6 을만족하는함수 f 를모두구하여라.(N 0 = f0; ; ;:::g) ( 오스트리아 989-6)

47 . 수론고급문제 47 풀이 f(f(0)) + f(0) = 6 f(0) 에서 f(0) = 0; ; 3; 4; 5; 6 일때각각좀따져보면모순을얻을수있음. 즉, f(0) = 일수밖에없음. 귀납적으로 f(n) =n +. f() 8 임을이용해비슷하게따져보면 f() = 3 일수밖에없음을얻을수있고, 귀납적으로 f(n +)=n +3. } 6. 0이아닌완전제곱수들만으로이루어진수열 (a n) nn 이각각의 n에대하여 a n+ a n 이소수이거나소수의제곱이라고한다. 이를만족하는수열은모두유한수열임을보이고그중에서가장긴수열을찾아라. ( 오폴 989-6) 풀이 a n = b n 으로두자. a n+ a n = b n+ b n =(b n+ + b n)(b n+ b n) 이고이것이 p 또는 p 이므로,0<b n+ b n <b n+ +b n 임에서 b n+ b n 은무조건 일수밖에없다. 따라서, b n = n+c꼴. 그럼 b n은 mod 3으로 0 한번과두번이반복되므로,두번을빼는경우에는 3의배수가됨. 즉, a n+ a n 은세번에한번꼴로3의배수.3의배수중에소수이거나소수의제곱인것은 3과 9뿐이므로그이상의 3의배수를만드는경우는길이가 3 이하밖에안되게되어,,4,9,6,5,36,49가가장긴수열이됨. } 7. 방정식 6(6a +3b + c )=5n 은 a = b = c = n =0 를제외하고는정수해를갖지않음을보여라. (APMO 989-) 증명 (a; b; c; n) 6= (0; 0; 0; 0) 인정수해가존재한다면최대공약수 g도존재하고, 그럼 ( a g ; b g ; c g ; n g ) 도해가되므로 g =이라고가정해도된다. 좌변이 3의배수이므로우변도 3의배수, 그럼 n도 3의배수이다. 그럼 36a,8b,5n 이모두9의배수이므로 6c 도 9의배수, 그럼 c도 3의배수이다. 또, 처음에했던것처럼좌변이짝수임에서 n도짝수임을알수있다. n =6m, c =3d 를대입하고양변을 8로나누어주면 a + b +3d =0m 양변의홀짝을검토하면 b 와 d 는홀짝이같아야한다. 만일 b 와 d 가둘다홀수이면홀수의제곱은 mod 8 로 만가능하므로 a +4 m (mod 8) 이된다. 정리하면 a + m (mod 4) 제곱수는 mod 4 로 0, 뿐이므로좌변은 mod 4 로 or3, 우변은 0or 이므로서로맞지않다. 따라서, b 와 d 는둘다짝수이다. b =b, d =d 을대입하고양변을 로나누자. a +b +6d =5m b, c, n 이이미찍수이므로 g = 이려면 a 는홀수여야한다. 그럼양변의홀짝을검토하면 m 도홀수이다. 역시 mod 8 로보면 +b +6d 5(mod8) 이고, 정리하면 b d (mod4).d 을우변으로이항하면앞에서와같은꼴로역시곤란하다. 따라서,(a; b; c; n) 6= (0; 0; 0; 0) 인정수해는없다. 8. f(x) =Ax 7 =7+Bx 5 =5+9x=35, g(x) =Bx 7 =7+Ax 5 =5 x=35 라하자. x가정수일때 f(x), g(x) 도항상정수라면, A와 B가가질수있는최소값을구하여라. ( 통신강좌 990-E7) 풀이먼저 A 와 B 를만족하는값으로음이아닌가능한최소의정수 ( 솟수일필요는없다 ) 를찾자. x = 이라고놓으면, ½ 5A +7B 6 (mod 35) () 5B +7A (mod 35) () () 식과 () 식을더하고 3 을곱하면 A + B 6 (mod 35) (7) () 식에서 () 식을빼고 7 을곱하면 A B 0 (mod 35) (8) 끝으로 (3), (4) 로부터가능한음아닌최소정수는 A =3,B =3( 모두우연히소수 ).

48 48 수론 역으로 A, B 값이위와같을때 f(x) = 3 x7 x 7 g(x) = 3 x7 x 7 +3 x5 x 5 +3 x5 x 5 +3x; +3x: 또, 이들은 Fermat's Theorem 에의하여어떠한 x 에대해서정수값을갖는다. 이같은방법은 5 나 7 이외의소수에대해서도쉽게일반화된다. } 9. a, b 는임의로주어진정수이다. 이때, 모든정수 n 에대하여 p + na 와 q + nb 가서로소가되도록하는 p, q 가존재할까? ( 통신강좌 990-E3) 풀이 gcd(a; b) =d, a = d, b = d 라두자.gcd( ; ) =이므로, p( )+q( ) =인 p, q를잡을수있다. 그럼모든정수 n에대하여 (p + na)( )+(q + nb)( ) = 즉, p + na 와 q + nb 는항상서로소이다. 따라서, 문제에서제기된 p, q 가존재한다. } 0. 모든자연수 n 에대하여 f(n) 이항상소수가되는, 정수계수를가지는다항식 f(x) =a k x k + a k x k + + a x + a 0 가존재하는가? 존재한다면모두구하고, 구한것외에는그러한다항식이없음을증명하여라. ( 통신강좌 ) 풀이문제에서요구하는다항식은존재하지않는다. 증명은다음과같다. f(x) =ak x k + +a x+a 0 가구하고자하는다항식이라고가정하자. f() = p 라고하자 (p 는소수 ). 그러면모든자연수 n 에대하여 f( + np) =a k ( + np) k + + a ( + np)+a 0 =(a k + + a + a 0 )+p g(n) = f() + p g(n) =p + p g(n) =p( + g(n)) 을만족시키는정수 g(n) 이존재하게된다. 한편모든자연수 n 에대하여 f( + np) 가언제나소수이어야하므로 f( + np) =p 일수밖에없다. 그러나 f 는차수 k 인다항식으로서 f(x) =p 를만족시키는 x 가많아야 k 개뿐이므로모순이다. }. P (x) 를정수계수의다항식이라하자. 주어진정수 x ;x ;:::;x n (n 3) 이다음을만족한다. P (x i )=x i+ ; i n ; P(x n )=x 이때 x = x 3 임을보여라. ( 오폴 990-6) 풀이 x i x i+ jp (x i ) P (x i+ ) 이므로 ( i n, x n+ = x ), 모두곱하면, x x 3 = P (x ) P (x )=(x x )Q(x ;x ) x 3 x 4 = P (x ) P (x 3 )=(x x 3 )Q(x ;x 3 ). x n x = P (x n ) P (x n )=(x n x n )Q(x n ;x n ) x x = P (x n) P (x )=(x n x )Q(x n;x ) (x x 3 ) (x n x )(x x )=(x x )(x x 3 ) (x n x )Q(x ;x ) Q(x n ;x )

49 . 수론고급문제 49 I) X i = x i+ 인 i 가없을때, 나누어버리면, Q(x ;x ) Q(x n ;x )=. 그런데 Q(x i ;x i+ ) 은정수값만가지므로모두 혹은 이다. i) 9i; Q(x i ;x i+ )= x i+ x i+ = (x i x i+ ) ) x i = x i+ ) x i+ = P (x i )=P (x i+ )=x i+3 반복하면, n 이짝수인경우는 x = x 3 = = x n, x = x 4 = = x n n 이홀수인경우는 x = x = = x n ( 사실상은전제에모순된다.) 어느경우에나 x = x 3 ii) 8i, Q(x i ;x i+ )= x x 3 = x x ;x 3 x 4 = x x 3 ; ;x x = x n x 위의식에서 fx g 는등차수열이고, 초항과말항이같으므로공차는 0 이다. ) x = x = = x n( 사실상은전제에모순된다 :) 따라서이경우에도 x = x 3 II) 9i, x i = x i+ x i+ = P (x i )=P (x i+ )=x i+ 반복하면, x = x = = x n. 역시이때도 x i = x i+ }. 자연수에서정의되는함수 f 가다음을만족한다 : f() =, f() =, f(n +)=f(n + f(n +))+f(n + f(n)) (n ) (a) (i) 0 f(n +) f(n) 임을보여라. (ii) f(n) 이홀수이면, f(n +) =f(n) +임을보여라. (b) f(n) = 0 +을만족하는 n을모두구하고, 그것을증명하여라. ( 캐나다 990-5) (a) 의증명 (i) n =일때는f() f() = 이므로성립한다. f(3) = f(3 f()) + f( f()) = f() + f() = 이므로, n =일때도f(3) f() = 0 으로성립한다. 이제 n k + 일때성립한다고가정하자 (k ). (n + f(n +)) (n + f(n)) = (f(n +) f(n)) = 0; 이므로 임을기억해두자. 점화식에의해 f(n + f(n +)) f(n + f(n)) = 0; ( ) f(k +3) f(k +)=f(k +3 f(k +)) f(k + f(k)) = 0; ; ( ) 이므로, 이것이 일때모순이라는것만확인하면된다. f(k+) f(k) =0; ; 중에서 f(k+) f(k) =0 이라야이게 가될수있다. 이때는 f(k +)=f(k +)=f(k) 이고, k = 이면이게만족되지않으므로 k. ( ) 에의해 f(k +)=f(k + f(k +))+f(k + f(k) f(k + f(k)+f(k f(k )) = f(k +) 이고, 여기서등호가성립해야하므로 f(k + f(k +))=f(k + f(k)) = f(k f(k )) 이라야한다. 따라서,( ) 은 f(k +3) f(k +)=f(k +3 f(k +)) f(k + f(k)) = f(k +3 f(k + )) f(k + f(k +)) 로역시0 또는, 즉 가될수없다. 그러므로, 수학적귀납법에의해임의의양의정수 n에대해f(n +) f(n) =0또는 임이증명되었다. (ii) f() = f() + 이므로 n =일때는잘만족하고 f() 는짝수이므로 n 3 이라해도된다. f(n) =f(n f(n )) + f(n f(n )) 가홀수이면두항이같을수없으므로,( ) 에의해 f(n f(n )) = f(n f(n )) + 이다. 그럼 f(n +)=f(n + f(n)) + f(n f(n )) f(n f(n )) + [f(n f(n )) + ] = f(n)+ 또,(i) 에서 f(n +) f(n)+ 이므로 f(n +)=f(n)+ 이다.

50 50 수론 (b) 의풀이 (a) 에서홀수인 f(n) 은한번나타나면그다음엔반드시 이증가하고f(n) 은감소하지않으므로, 홀수인 f(n) 은한번밖에나타나지않는다. 만일 f(n) 이언젠가부터증가하지않고 n N 일때항상f(n) =c 로상수함수가된다면, c = f(n + c +)=f(n + c + f(n + c +))+f(n + c + f(n + c)) = f(n +)+f(n +)=c 가되어모순. 따라서, f(n) 은무한히증가하고, 또 f(n) 은많아야 씩증가하므로어떤양의정수도건너뛰는법이없어서임의의양의정수 m에대해f(n) =m 인 n이항상존재한다. 특히 m이홀수이면이런 n은유일하게존재함을알수있다. f( n+ )= n + (n ) 임을수학적귀납법으로보이자. f(4) = f(4 f(3))+f(3 f()) = f()+f() = + 이므로 n = 일때성립. n = k 일때 f( k+ )= k + 이성립함을가정하자. k+ + 은홀수이므로 f(m) = k+ + 인 m 은유일하게존재한다. 즉, f(m ) = k+. k+ +=f(m) =f(m f(m )) + f(m f(m )) 이홀수이고 ( ) 에의해두항의차이가 이하이므로, f(m f(m )) = k +, f(m f(m )) = k 이다. k + 도홀수이므로, 그럼귀납법의가정에의해 m f(m ) = k+ 이된다. 따라서, m = f(m ) + k+ = k+ + k+ = k+ 로 n = k + 일때도성립함을알수있다. 이로부터, f(n) = 0 + 이되는 n 은 뿐이다. } 3. a는자연수이고, A와 B는실수이다. 다음연립방정식 x (Ax + By )+y (Ay + Bz )+z (Az + Bx (A + B)(3a)4 )= 4 x + y + z =(3a) 이자연수해 (x; y; z) 를가질동치조건을 A 와 B 에대한식으로구하여라. ( 중국 990-4) 풀이아래의식을윗식에대입하여정리하면 (A B)(x+y+z)( x+y+z)(x y+z)(x+y z) =0 뭐대충이런식이나올거임.A B =0 이면주어진두식은동치이고, 그래서 (x; y; z) =(3a; 4a; a) 등의해가존재하고,A B 6= 0 일때는적당히해를변형하면무한내림 ( 또는최소원의원리 ) 으로논리를풀어갈수있어서해가없게됨. } 4. 혼자하는게임이다음처럼진행된다. 각게임이끝난후의결과에의해서게임하는사람은 a 나 b 의점수를받는다 (a 와 b 는자연수이고 a>b). 이사람의점수가각게임마다더해지는데이사람이받을수없는점수의합계가 35 가지있다. 그중의하나가 58 일때, a 와 b 를구하여라. ( 통신강좌 99-3-) 풀이우선 gcd (a; b) =d>이면 d j (ax + by) 이므로, ax + by (x 0;y 0) 의꼴로나타낼수없는, d의배수가아닌수들이무한히많아서안된다. 따라서,gcd(a; b) =이다. 모든자연수 (0은포함하거나말거나풀이에관계없으므로따지지않기로하자 ) 들을 b로나눈나머지 i에따라서로소인b개의집합 M i (i =0; ;:::;b ) 들로나누어담을수있다. M 0 의모든원소는a의배수이므로 ax + by꼴로표현된다. 이제 ax + by (x; y 0) 꼴로표현될수없는자연수의집합을 T 라 (a ) (b ) 하고 jt j = 임을보이려한다. gcd (a; b) =에서 ka i (mod b) 인 k ( k<b ) 가항상하나존재하며 (), 이 k를 m i 라하자. 이때, M i T (i =; ;:::;b ) 의최소원소가m i a가된다 (). 만일더작은원소mb + i (< m i a) 가있다면 ax + by = mb + i에서 ax =(m y) b + i<m i a로 ax M i 가되는데, 이는 () 의유일성에모순이다.() 에서 m i a보다 M i 의원소들은 m i a + nb = T 가되므로결국 i =;:::;b 에대해 h mi a i jm i \ T j = = m ia i b b 이다. 여기서 jt j = b X b X jt \ M i j = i=0 i=0 Ã! = b X bx a i i b i=0 i=0 = (a ) (b ) m i a i b = a b à =! b X b X a m i i b i=0 i=0 b X i=0 i = a b b (b )

51 . 수론고급문제 5 이구해진다. (a ) (b ) 이제문제에서 = 35, 곧 (a ) (b ) = 70의부정방정식을풀어 a > b > 0, gcd (a; b) =을만족하는해를구하면,(a; b) =(7; ) ; (; 8) 이다. 여기서 58 T 인것은a =, b =8뿐이다. } 5. p를 보다큰연속된두정수의곱이라고하자. 그러면다음의방정식을만족하는정수근 x ;x ;:::;x p 가존재하지않음을보여라. Ã px px! x i 4 x i = 4p + i= i= 또는이방정식을만족하는정수근 x ;x ;:::;x p 가존재하기위한 p 의값이단지 개임을증명하여라. ( 통신강좌 ) 풀이 ( 과기대수학과고봉균 ) px m을 x i 들의평균, 즉 pm = x i 라하자.(m은정수일필요가없음을주의하자.) 먼저 i= px (x i m) = i= = = px (x i mx i + m ) i= px px px x i m x i + m i= i= i= px px x i m pm + pm = x i pm i= i= 의관계로부터준식이다음과같이변형된다. 두항이모두음이아니므로 0 = = px µ x i pm + p 4 (pm) 4p + i= px (x i m) + i= p(4p +) (pm) px (x i m) ; 0 p(4p +) (pm) (9) i= 의부등식을얻는다. 첫째식을가만히살펴보면 x i 들중가장작은것과가장큰것의차가 이상일수없음을알수있다.(x i m) 의꼴이 보다큰것이있거나, 인것이둘이상있어오른쪽부등호에모순이되기때문이다. 따라서 x i 들중가장큰것과가장작은것의차는 또는 0 이다. 이제 x i 들중 t 개가 x + 이고 p t 개가 x 라하자 (0 t p). 그러면 px pm = x i = px + t; i= px x i = px +(x +)t i= 가된다. 이를준식에대입하여 (4p +)[px +(x +)t] 4(px + t) =4p + 을얻고, 전개하여정리하면다음의식을구할수있다. p(4t + x 4) = 4t (x +)t + (0) 잠깐 x i 가모두같을수없음을먼저짚고넘어가자. x i 가모두 m 이면준식은양변에 4p + 을곱해 p(4p +)m 4p m =4p +

52 5 수론 로변형되는데, 좌변은 p의배수이나우변은 p로나누어이남으므로이식의정수근은없다. 곧 t p 이다. () 의둘째부등식을생각하자.(px + t) p(4p +)< (p +) 이므로 jpx + tj p가된다. 이를만족시킬수있는 x는,, 0, 의네가지뿐이다. x와 x +은부호가같으므로, 문제의식으로부터 x = 와 x =,x = 과 x =0은부호만다른동일한해를구하게할것임을알수있다. 따라서 x = ; 의두가지경우에대해서만 () 를풀면된다. (i) x = 일때;() 식은 4tp =4t +3t +=t(4t)+(3t +) 이므로 4t j (3t +) 이다. 따라서 t =이며 p == 가되는데, 이는문제에서 p의조건에맞지않는무연근이다. (ii) x = 일때;(4t 3)p =4t + t +=(t + )(4t 3) + 4이므로 (4t 3) j 4이다. 따라서 t =이며 p =6= 3이된다. 이도역시무연근이다. (i), (ii) 에서 p =일때 ( ; ), (; ), 또 p =6일때 ( ; ; ; ; ; 0), (0; ; ; ; ; ) 의해를가짐을알수있다. 그러나 p = n(n +), n>의조건으로부터모두적절한해가될수없으므로, 방정식의해는없다. } 6. S 는유리수집합의부분집합으로다음을만족한다 : () 0 = S. () a; b S 이면 a b S. (3) 0 이아닌어떤 q= S 가있어,0 이아닌임의의 s= S 에대해 s q S 이다. S 의모든원소는 S 의어떤두원소의합임을보여라. ( 아일랜드 99-0, 한국 99-) 증명 s S 라하자. s = s = S라하면 (ii) 에의하여 s =S이다. s s ;s = s로잡으면 (ii) 에의하여 s S t S이면 s = t; s = 로잡으면st S이다. s 0이아닌유리수u가 0 6= u 6 S이면 (iii) 에의하여 0 6= q 6 S가존재하여 u = qs인꼴이다. u = q s 에서 q 6 S이면 (iii) 에서 q = qt; t S인꼴이고q = t S 0 가되어모순. 따라서 q S 즉 u S이다. 따라서 0이아닌임의의유리수의제곱은 S의원소이다. 따라서 x S이면 이고 x = x µ 3 + x 5 µ 4 5 µ 3 µ 4 y = x S; z = x S 5 5 = y + z 이다. 7. 임의로주어진자연수 n 에대해, 다음수열은 mod n 으로결국상수수열이됨을보여라. ( 미국 99-3) ; ; ; ; ::: 증명 n에대한귀납법. n 미만에서항상성립한다고할때 n이 의거듭제곱이면그냥끝이고, 아니면 k m (m은홀수) 꼴로놓을때 Á(m) 갖구어찌놀아보면 n, z 5 n, 그리고 x n+ y n+ = xyz + n+ 을만족하는자연수해 (n; x; y; z) 를모두구하여라. ( 중국 99-4) 풀이 x>y. x와 y의홀짝이다르면좌변을홀수이고우변은짝수라서모순. 고로 j x y, 즉 x y. x n+ xyz x((y+) n y 5 n )=:A라잠시두고,() n y n + n y n 5y n 이대부분성립함을확인하자. 이것은 n(n )y +4n 0 과동치이고, y 이거나 n 3 이면무조건성립함을알수있음. () 에의해A의이항전개에서 n차,차,차,0차항만생각해보면 A>x(y n + n ) y n+ + n+ 이되어문제의방정식이성립하지않음. 따라서,() 이거짓일수있는 y =이고 n =일때만살피면됨. 그럼문제의방정식은 x 5 = xz +33 이됨. x j 33 임에서 x =3; ; 33 만가능성이있고, 문제에서준범위 z 80 에의해x 4 = z = 9. x 즉, x =3이고 z =70뿐. 답 (,3,,70) }

53 . 수론고급문제 S(n) 을자연수 n 의자릿수의합이라고하자. S(n) =S(n) =S(3n) = = S(n ) 인모든 n 을찾아라. ( 체코슬로바키아 99-3) 풀이우선 은그냥되고, n = 0 k 꼴들도다됨 ( ). 왜냐하면 a < 0 k 일때, (a +)n = a 0 k +((0 k ) a) 이라서 0 k 이상의자리에서 a가추가된만큼아랫자리에서도로빼어짐. 그밖의 n은불가능함을보이자. 우선 n> 이므로 n S(n) =S(n) n (mod 9) 에서 n =9t꼴. 0 k 꼴이아니므로 0 k <n<0 k+ 인 k가존재. () 0k+ n<0 9 k+ 일때 : 이범위에서항상 k +자리의수인데모든자리가 9일수는없으므로 S(n) < 9(k +) (n은 9의배수이므로사실상 S(n) 9k 로쓸수도있음 ). 그런데 t < 0 k+ 이므로, 문제의조건에서 S(n) =S( 0k+ n) =S(t (0 9 k+ )) = 9(k +) 이다 ( 마지막등호는 ( ) 의성질에의해 ). 이것은모순. () 0 k <n< 0k+ 9 일때 : k +자리의수인데처음두자리가둘다 이하이므로 S(n) < 9k. 또, t<0 k 이므로 S(n) =S( 0k n) =S(t (0 9 k )) = 9k. 이것역시모순. } 이상의정수 n 에대해서, 다음두가지조건이동치임을보여라. () n이소수 () n = n +n +n 3 +n 4 (n i N, i 4) 를만족하는모든 (n ;n ;n 3 ;n 4 ) 에대해서 n n 6= n 3 n 4 이다. ( 통신강좌 ) 풀이먼저 ii))i) 을보이자. 만일 n이 n 4인짝수이면 n =m =(m ) + + (m ) + 의꼴로나타낼수있으므로부적당하다. 또 n이 n 5이고홀수인합성수라면, n =(k + )(l +)= k +l ++4kl의꼴로나타낼수있으므로역시부적당하다. 따라서 n은소수이다. 이제 i))ii) 임을보이자. n 5이고 n = n +n +n 3 +n 4, n n = n 3 n 4 라고가정하자. n j n 3 n 4 이므로 n 3 = pr, n 4 = qs, pq = n 으로쓸수있다. 이제 n n = pqrs에서 n = rs이고, n = pq+rs+pr+qs = (p + s)(q + r) 로나타나므로 n은소수가아니다. 따라서대우명제가증명되었다. } 3. p 는소수이고 a 와 k 는자연수이다. p k j (a ) 이면모든자연수 n 에대해 p n+k j (a pn ) 임을보여라. ( 통신강좌 ) 풀이 n 에대한수학적귀납법으로증명한다. i) n =0 일때 ; 가정과같으므로자명하다. ii) p k+n j (a pn ) 이라고가정하자. x = a pn 이라하면 a (modp) 이므로 x (modp) 이고 x p + x p + + x p 0 (mod p) 이다. 따라서 p j (x p + x p + + x +) 이고가정에의해 p k+n j (x ) 이므로 p k+n+ j (x )(x p + x p + + x +)=x p =a pn+ 도성립한다. i), ii) 의귀납법으로증명이되었다. } 3. 어떤함수 f : N N! N 가다음을만족한다. Y \ 모든 m; n N 에대해 f(m; d) = mn " kjn 이때, 만일 p a pa pas s 가 n 을소인수분해한결과라면, µ n f(m; n) =f m ;p p p s p p p s 임을보여라. ( 통신강좌 )

54 54 수론 풀이 ( 서울과학고 3 년이은수 ) f(m; n) 의값을알아내기위해실험을해보자. f(m; ) = Y dj f(m; d) = m ; p 가소수이면 Y f(m; d) djp f(m; p) = Y f(m; d) f(m; p k )= dj Y djp k f(m; d) Y djp k f(m; d) = mp m ; = mpk mpk : Y f(m; pq), f(m; p r q s ) 등에대해서도실험을해보면아래와같은식을추측할수있다. hm; ni = f(m; d) = mn 이라정의하고, p a pa pa s s 가 n을소인수분해한결과라하면, djn Y À n hm; ni m; p i<j s i p j f(m; n) = Y À m; npi Y À n m; p i s i<j<k s i p j p k 즉, 분자와분모는각각 * m; + n ; ( 서로다른짝수 (0포함) 개의소인수의곱 ) * + n m; ( 서로다른홀수개의소인수의곱 ) 들을모두곱한것이다. 이것을증명하겠다. d j n=( 서로다른몇개의소인수의곱 ) 이면 d j n이므로, d j n인 f(m; d) 에대해서만생각하면된다. 먼저, f(m; n) 은 hm; ni에만포함되어있으므로,( ) 에서분자에만한번나타난다 (i). d = p b p b p bs s (0 b i a i, i s) 라할때, b i = a i 인 i의개수를 g(d) 라하자. d<n인경우, g(d) <s이다. f(m; d) 는분자에서 µ µ µ s g(d) s g(d) s g(d) {z } {z } {z } hm;ni Q m; n Q m; n p i p j À p i p j p k p l À ( ) 개, 분모에서 µ µ s g(d) s g(d) {z } {z } Q D E m; p n Q i m; n p i p j p k À 개나타난다. 이항정리에서 s g(d) X i=0 µ s g(d) i ( ) i =( ) s g(d) =0

55 . 수론고급문제 55 이므로 f(m; d) 는모두약분된다 (ii). (i), (ii) 에서 ( ) 가성립함을알수있다. 한편, hm; ni = mn 이므로 uv = mn이면 hm; ni = hu; vi인데, mn = µ m n p p p s (p p p s ) 이고, n과 p p p s 는소인수의구성이같으므로, µ n f(m; n) =f m ;p p p s p p p s 이다. } 33. 자연수의수열 a 0 ;a ;a ;::: 에대하여두수열 fp ng 과 fq ng 을다음과같이정의하자. () 임의의실수 x 에대하여 p =0; p =; p i = a i p i + p i (i 0) q =; q =0; q i = a i q i + q i (i 0) a 0 + a + a + + a n + x = xp n + p n xq n + q n 임을증명하여라. 좌변의연분수는 [a 0 ; a ;a ;:::;a n ;x] 로표기하기로한다. () lim [a p n 0; a ;a ;:::;a n! n]=x 라면 lim = x 임을증명하여라. ( 통신강좌 ) n! q n 풀이 () n에대한수학적귀납법을이용하자. p 0 = a 0, q 0 =이므로, n =일때양변이모두 a 0 + x 로성립한다. n = k 일때,[a 0 ; a ;a ; ;a k ;x]= xp k + p k 가성립한다고가정하면 (x ), a k 이양의 xq k + q k 정수임에서 [a k ; x] =a k + x 이므로 [a 0 ; a ;a ; ;a k ;a k ;x]=[a 0 ; a ;a ; ;a k ; [a k ; x]] = [a k; x]p k + p k = a kp k + p k + p k =x [a k ; x]q k + q k a k q k + q k + q k =x = p k + p k =x q k + q k =x = xp k + p k xq k + q k 로 n = k 일때도성립한다. () [a 0 ; a ;a ; ;a n ;a n ]= anp n + p n a nq n + q n = pn q n 이므로, lim n! [a 0 ; a ;a ; ;a n ;a n ]= x 이면, lim n! p n q n = x 임은자명하다. } 34. x 4 y 4 = z 은자연수해를가지지않음을보여라. ( 통신강좌 ) 풀이 x가최소인해를 (x 0 ;y 0 ;z 0 ) 라하자. x 0 ;y 0 ;z 0 중어느두수가소수 p로나누어지면나머지한수도 p로나누어지게되고, 따라서 ( x 0 p ; y 0 p ; z 0 p ) 가 x가더작은해가되므로모순이다. 따라서 x 0;y 0 ;z 0 은둘씩서로소여야한다. 준식을 (x 0 ) = (y0 ) +(x 0 ) 으로변형한후, 피타고라스의정리의일반해 (a + b ;a b ; ab) (a; b는서로소이고, 합이홀수 ) 에서 x 0 = a + b ; y0 = a b ; z 0 =ab () 혹은

56 56 수론 x 0 = a + b ; y 0 =ab; z 0 = a b () 이다. () 의경우는 (x 0 y 0 ) =(a + b )(a b )=a 4 b 4 이므로,(a; b; x 0 y 0 ) 이준식의해가되고, 특히 x 0 = a + b 로부터 x 0 >a 이므로 x 가더작은해가되어모순이다. () 의경우는 x 0 = a + b 을다시피타고라스의정리의일반해 (c + d ;c d ; cd) 로풀면, y 0 = ab =4cd(c d ) 이된다. c; d; c d 이각각서로소이므로, 모두완전제곱수이어야한다. c = c 0 ;d = d 0 ;c 0 d 0 = e 0 로두면, c 04 d 04 = e 04 이므로,(c 0 ;d 0 ;e) 가준식의해가되고, 특히 x 0 = c + d 으로부터 x 0 >c 0 = c 04 c 0 이므로역시모순이다. (), () 로부터, x 4 y 4 = z 은양의정수해를가지지않음이밝혀졌다. } 35. n +5 가소수가될모든음이아닌정수 n 을구하여라. ( 한국 993-3) 풀이 (i) n이짝수일때 =4; 4 =6 (mod7) ) n =( 4 ) n n (mod 7) 이를반복하면 n이짝수일때 n 0 (mod7) ) x n (mod7) 따라서 n이 0아닌짝수이면 n +5는 7의배수이고 n =0일때 0 +5=7은소수 (ii) n이홀수일때 n 은짝수이고, 4 (mod3) 이므로 n =k라하면 n k 4 k (mod3) ) n (mod3) 따라서 n +5는 3의배수이고 n +56= 3이므로항상합성수이다. n +5가소수가되는경우는 (i) (ii) 에서 n =0뿐이다. 답 n =0 } 36. 음이아닌임의의정수 m, n 에대하여, 다음두조건을만족시키는함수 f(m; n) 을구하여라. (i) f(m; n) =+f(m +;n ) + f(m ;n+)(m, n ) (ii) f(m; 0) = f(0;n)=0(m 0, n 0) ( 한국 993-4) 풀이 (i) 에서 f(m; n) f(m ;n+)=f(m +;n ) f(m; n)+ () m; n 대신에 m k; n + k (k m) 을대입하고 m X ff(m k; n + k) f(m k ; n+ k +)g k=0 m X = ff(m k +;n+ k ) f(m k; n + k)+g k=0 을계산하면 f(m; n) f(0;n+ m) =f(m +;n ) f(;n+ m ) + m f(0;n+ m) =0이므로 f(m +;n ) f(m; n) =f(;m+ n ) m () () 에서 m; n 대신 m + k; n k(k <n) 을대입하고 n X ff(m + k +;n k ) f(m + k; n k)g k=0 n X = ff(;m+ n ) (m + k)g k=0

57 . 수론고급문제 57 을계산하면 f(m + n; 0) f(m; n) =nf(;m+ n ) (m + n )n ) f(m; n) =(m + n )n nf(;m+ n ) (3) 다시 () 에서 m; n 대신에 m k; n + k(k m) 을대입하고 mx ff(m k +;n+ k ) f(m k; n + k)g k= = mx ff(;m+ n ) (m k)g k= 을계산하면 f(m; n) f(0;n+ m) =mf(;m+ n ) m(m ) (3) m +(4) n 을계산하면 ) f(m; n) =mf(;m+ n ) m(m ) (4) (m + n)f(m; n) = mn(m + n ) mn(m ) = mn(m + n) 답 f(m; n) =mn 주 : (3), (,4) 에서 f(;m+ n ) = m + n 참고 : 우선, 다음을보인다. 0 i n 일때 f(m; n) =i +(m n)i + f(m + i; n i) () 이성립한다. 증명 : i =0일때, 당연히성립한다. 지금 0 a<n이고, i a 인 i에대하여성립한다고가정하고 i = a +일때성립함을보이면된다. (i) 에서 f(m + a; n a) =+f(m + a +;m a ) + f(m + a ;n a +) () 귀납법의가정에의하여 (i = a 일때와 i = a 인경우 ) f(m + a ;n a +)=f(m; n) (a ) (m n)(a ) (3) (), (3), (4) 에서 즉, i = a + 일때성립한다. 같이하여,0 j m 인 j 에대하여 을증명할수있다. (), (5) 에서 여기서 i = n; j = m 라하면 f(m; n) =a +(m n)a + f(m + a; n a) (4) f(m; n) =(a +) +(m n)(a +)+f(m + a +;n a ) f(m; n) =j +(n m)j + f(m j; n + j) (5) f(m; n) =i + j +(m n)(i j)+f(m + i; n i)+f(m j; n + j) f(m; n) = n + m (m n) =mn ) f(m; n) = mn 이경우a =때 (4) 에서 f(m; n) =+m n + f(m +;n ) 을증명하여야한다. [ 본증명에서 f(;m+n ) = m+n 을이용하여 () 에대입하면 f(m; n) =m n++f(m+;n ) 을얻는다.] }

58 58 수론 37. 수열 y ;y ;y 3 ;::: 을정의하는데, y =이고, k>0 에대해서는다음과같다 : ( y k k가짝수일때 y k = y k + k가홀수일때 ( y k k가홀수일때 y k+ = y k + k가짝수일때 수열 y ;y ;y 3 ;::: 는모든자연수를정확히한번씩표시함을증명하여라. ( 캐나다 993-5) 증명 k =n, n +일때로나누고,진법으로나타내며, y n = y(n) 으로쓰기로하자. 그럼점화식은다음과같이옮겨쓸수있다 : f() = 이고 y a a k 00 = y a a k 0 0 y a a k 0 = y a a k 0 y a a k 0 = y a a k y a a k = y a a k 0 이것은마지막두자리수가서로같을때는마지막자리를 0 으로하고그렇지않으면마지막자리를 로한후에,0 의자리이상에대해다시반복해주는점화식이다. 즉, y(n) 은각각의 i =0; ; ;::: 에대해 n 의 i 자리와 i+ 자리를비교하여, 둘이같으면 0 을그렇지않으면 을 y(n) 의 i 자리로하는함수이다.(y n 이이것이된다는증명은, 위의점화식에의해수열 y n 이유일하게결정되므로, 우리가추측한 y n 이이점화식을만족한다는것만확인하면된다. 그것은또바로자명하게확인된다.) 예를들어, 0000 에대해서는다음의왼쪽처럼변환이된다. n = 000_0_ y(n) = 00_00 n = 00_00 z(n) = 000_0_ 이때, y 의역변환 z 가존재한다. z(n) 은 n 의최고자리가 k 자리라면우선 z(n) 의 k 자리를 로하고, i = k ;:::;0 에대해차례로, n 의 i 자리와 z(n) 의 i+ 자리를비교하여, 둘이같으면 0 을그렇지않으면 을 z(n) 의 i 로하는함수이다. 즉, 앞의예에서숫자위에점 (_) 을찍어표시한그런이웃한세위치의 의개수가항상짝수개가되도록하는함수이다. 그럼 z 는 y 의역함수이고, 둘다양의정수들을입력과출력으로하므로, y 는양의정수들의집합에서의일대일대응이다. 38. 다음연립방정식의유리수해 x; y; z; t; w 를모두구하여라. ( 폴란드 차 -) xy = t w + z ; xz = t y + w ; yz = t w + x 풀이 () () + (3) 하면 (x y + z) + t =3w. 분모의최소공배수를적당히곱하면정수부정방정식 X + Y =3W.mod3에대한무한내림으로해는 X = Y = Z =0뿐. } 39. 다음식을만족시키는모든자연수 n 을구하여라. ( 통신강좌 ) 3 n +4 n + +(n +) n =(n +3) n 풀이 n =; 3 일때는각각3 +4 =5, =6 3 으로성립한다. n =; 4; 5 일때3 =4, =7 4, =8 5 등은좌우변의홀짝이맞지않아성립하지않는다. n =6일때도성립하지않음을 mod 3으로간단히확인할수있다. n 7 일때항상우변이더커서성립하지않음을보이자. µ n +3 k n < n +3 k () 임을보일것이다. 이것이성립하면문제의양변을 (n+3) n 으로나눠준식은 < n + n + + < 로성립하지않음이확인된다. 우선 k =일때의 ( n+ n+3 )n < 을보이자 (). 양변의역수를취하면 ( + n+ )n > 와동치이다. 이항전개하면 µ + n > + n n(n ) n + n + + (n +)

59 . 수론고급문제 59 이므로 n n+ + n(n ) (n+) > 만보이면되고, 이것은 n(n ) (n+) >, n+ 즉 n(n ) > 4(n +) 와동치이다. 이것은 n 7 일때n(n 5) > 8 임에서확인되므로 () 는증명되었다. 그럼임의의 k n 에대해, µ n +3 k n µ Ã n +3 k n +n + n µ! n + k n = < n +3 n +4 k n +n +3 n +3 k 이되어 () 이증명된다. } 40. 자연수 n 이다음성질을만족할때성질 P 를가졌다고한다 : n 이 a n 을나눌때마다 n 도 a n 을나눈다. (a) 모든소수 n은성질p를가짐을보여라. (b) 성질 P 를갖는합성수가무한히많음을보여라. ( 통신강좌 ) 풀이 (a) p는소수이고, p j a p 이라고하자. 그러면 gcd(a; p) =이므로 p j a(a p )+(a ) 인데페르마의작은정리에의해 p j a 임을알수있다. 즉, a i (modp), 0 i p 이다. 그러므로 +a + a + + a p p 0(modp) 이다. 즉, a p =(a )( + a + + a p ) 0(modp ) (b) n을성질p를갖는자연수라하자. 만일 nja n 이면 gcd(a; n) =이고 a Á(n) (modn) 이다. 그런데 a n (modn) 이므로이두사실을합하면a gcd(á(n);n) (modn) 이된다. 만일 gcd(á(n);n)=이면 a (modn) 이므로 (a) 에서와마찬가지로 n ja n 이성립한다. 이제 n이무한히많고합성수임을보이자. p 5를소수이고 g는 p 를나누는소수로선택하자. 그리고 n = pq로놓으면 Á(n) =(p )(q ) 이고 q j p 이므로 Á(n) 과 n은서로소이다. p를무한히많은방법으로택할수있으므로증명은끝난다. } 4. 수열 fa n g이다음과같이정의된다. a =; a n+ =a n + q 3a n ; (n ) 모든자연수 n 에대하여 a n 이정수임을증명하여라. ( 통신강좌 ) 증명 (a n+ a n ) =3a n 에서 a n+ 4a n+ a n + a n = 이고, n 대신 n 을대입하면 a n 4a na n+ + a n = 이다. 따라서, a n+ 4a n+ a n + a n = a n 4a na n+ + a n 이므로 (a n+ a n) =(a n a n ) 이다. 한편 a n+ a n 이므로 a n+ a n 0이고, a n > 0이므로 a n a n a n a n 에서 a n a n 0이다. 그러므로 a n+ a n =a n a n 이고 a n+ =4a n a n 이다. 이제수학적귀납법을사용하여 a n 이정수임을증명한다 (a =4이다.). 사실상 a n =(+ p 3) n +( p 3) n 이다. 4. c 가유리수일때방정식 x 3 3cx 3x + c =0 은많아야하나의유리수근을가짐을증명하여라. ( 통신강좌 ) 증명 k 를 x 3 3cx 3x + c =0 의유리수근이라하자. 그러면 c =(k 3 3k)=(3k ) 이고방정식은 x 3 (3k ) 3x (k 3 3k) 3x(3k ) + k 3 3k =0 즉,(x k)[(3k )x + qkx (k 3)] = 0 이다.(3k )x + qkx (k 3) = 0 의판별식은 6k +(3k )(k 3) = 3(k +) 으로유리수의제곱이될수없다. 따라서유리수근의갯수는 0 또는 이다. 43. 정수열 fa ng 이 a =,a =7, <a n+ a n a n 로정의될때, 모든자연수 n 에대하여 an 은홀수임을보여라. ( 통신강좌 )

60 60 수론 a 증명 n 구간 a n ; a n + 은길이을가지고있으므로단하나의정수만을포함한다. 따 a n 라서수열은조건에의해유일하게결정된다. a =;a =7;a 3 =5=3 7+ ;a 4 =89= ;a 5 = 37 = ; 이므로수열 fb ng을 b =;b =7;b n+ =3b n +b n (n ) 으로정의하고 fb ng도동일한부등식을만족시킴을보이면모든자연수 n에대하여a n = b n 이된다. b n+ b n = b n+b n b n 이고 b n+ b n b n b n b =3(b n +b n )b n b n (3b n +b n )= n (b nb n b n )(n >) 이므로수학적귀납법에의해 b n+b n b n =( ) n (n ) 을증명할수있다. 역시 b n n ( 등호는 n = 일때만 ) 을증명할수있으므로 n 에대하여 <b n+ b n b n 이다. 따라서모든자연수 n에대하여 a n = b n 이된다. a n+ =3a n +a n a n (mod ) 이므로 n 이면 a n a (mod) 가성립한다. 증명 ( 서울언주중공유식 ) 처음몇항을계산해보면a =,a =7,a 3 = 5, a 4 =89가나와 a n =3a n +a n 임을쉽게짐작할수있다. 또한 a n, a n 이유일하면a n+ 도유일하게결정되므로 a n 이 a n =3a n +a n 를만족시킴을보이면이두수열은같은수열이된다. 그러기위해수학적귀납법을쓰겠다. i) n =,일때성립한다. ii) n 6 m일때성립함을가정하자. 즉 a n =3a m +a m ; <am a m 6 a m, a m <a ma m a m 6 a m a m =3a m +a m 를넣으면 a m < 3a m a m +a m a m 6 a m n = m +일때성립함을보이자. 앞에서말했듯 a m+ 은 a m 과 a m 이결정되면유일하다. 따라서 a m+ =3a m +a m 을넣어서만족하면 a m+ =3a m + m 이되게된다. a m+ 에 3a m +a m 을넣어계산해보면 <a m+ a m =3a n +a m a m 6 a m a m, a m < 3a ma m +a m + a m 6 a m, a m < 3(3a m +a m )a m +a m (3a m +a m ) 6 a m, a m < a m 6a m a m 4a m 6 a m, 4 a m <a m 3a m a m a m 6 4 a m 이제 식이성립함을보이면 n = m +에도성립하게되어 a n =3a n +a n 이증명된다. 4 a m = 4 (3a m +a m 3 ) > 3 4 a m > a m ) a m < 4 a m ; 4 a m < a m 이다. 에서 4 a m < a m < 3a m a m +a m a m 6 a m < 4 a m 양변에 을곱하면 4 a m <a m 3a m a m a m < 4 a m 되어 식이성립한다.

61 . 수론고급문제 6 이제 a =,a =7,a 3 =5 이므로역시귀납적으로생각해서 a n, a n 가홀수이면 a n =3a n +a n 3a n (mod ) 이므로 n > 에대해 a n 이항상홀수임을알수있다. 44. 주어진자연수 m 에대하여 m, n 이서로소이고 (x + y ) m =(xy) n 을만족시키는자연수들의쌍 (n; x; y) 를모두구하여라.( 단, n, x, y 를 m 의함수로나타내어라.) (Putnam 99-A3) 풀이우선 AM-GM 부등식에서 (xy) n =(x + y ) m (xy) m 이므로 n>m. x, y가둘다홀수이거나하나만홀수이면준식의양변의홀짝이맞지않음. 따라서, x, y는둘다짝수. 임의의소수 p에대해 p a k x, p b k y, 즉 x = p a X, y = p b Y 라하자.WLOGa b 라하면p ma (X + p (b a) Y ) m = p (a+b)n (XY ) n. a<b이면양변의 p의지수를비교했을때 ma =(a + b)n >(a + b)m 이므로모순. 따라서, a = b. p가임의였으므로 x = y. 그럼준식은 m x m = x n. 즉 m = x (n m). 따라서, x는 의거듭제곱이며 (n m) j m. 그런데 m, n이서로소이므로 n m도 m과서로소이고, 따라서 n>m 에서는 n = m +만가능. 그리고 m은짝수. 그리고, 이경우x = y = m= 의해가항상존재. 답 (n; x; y) =(m +; m= ; m= ) 주이문제는 994 년 KMO 번에도중복출제되었었다. } 45. a, b 가서로소인정수일때, n 은 a + b 인꼴의정수라하자. 그리고, 만일 p 가 p pn 인소수이면 ab 는 p 로나누어진다고한다. 이러한 n 을모두구하여라. 임의의자연수 m 에대해 m<p m 인소수 p 가항상존재한다는사실을이용해도좋다. (APMO 994-3) 풀이 (O±cial) a; b는서로소이고 ab가 p n이하의모든소수로나누어지므로 n = a + b 는 p n이하의모든소수로나누어지지않는다. 따라서, n은소수이다. 그러므로 a = b =일때n =이거나, n 5일때는a; b 중하나는짝수이므로 n은 4k +꼴인수이다. 이제 n 5 =5임을밝히고자한다. n>5이라고하자. ab는 5이하의소수즉,,3,5로나누어져야한다. c; d를각각a; b의서로다른양의소인수의곱이라하면 n = a + b c + d cd ( 3 5) = 60 > 49 = 7 P k 를 k번째소수라하자. 여기서 P k 5이면 ( 3 P k ) >Pk+ 임을밝히고자한다. 귀납법을이용한다. 문제에서이용을허락한베르트랑의공준에의해 P k+ >P k+ 이므로, 귀납법에의하여 ( 3 P k P k+ ) P 3 k+ >P k+ µ Pk+ = P k+ 4 P k+ >P k+ 이성립한다 (* P k+ 5). 이결과와가정 (n >5) 으로부터임의의소수 p에관하여 n>p 이다. 이는명백히불합리하다. 따라서 n 5이다. 위의결과로부터 n은 ; 5; 3; 7중의하나이다. n =7=4 +일때3 p7 이고 4 은 3으로나누어지지않는다. 그러나 = + ; 5= + ; 3 = +3 은조건을만족시킨다. ) n =; 5; 3 답 } 주 = + 일때 p 이고 p 이하의소수는없다. 문제는 p가 p n이하의소수이면 ab를나눈다고되어있으므로 n =도답이된다. 46. x + y + z = x 3 + y 3 + z 3 의정수해가무한히많음을증명하여라. (Towns 994 봄 SA) 증명 z = y 인것만생각하면 x +y = x 3. y = mx꼴인것만생각하면 +m = x. 이런것만도무한히많음.

62 6 수론 47. 다음과같이정의된정수들의수열이있다고하자. a n =a n + a n a 0 =0 a = (n>) k 이 n을나누는것이 k 이 a n 을나누는것과동치임을보여라. ( 통신강좌 ) 증명 ( 서울과학고신석우 ) a = a + a 0 = a n = a n + a n = a a n + a a n 또 a n = a i+ a n i + a i a n i 이라면 ( i n ) a n = a i+ (a n i + a n i )+a i a n i = (a i+ + a i )a n i + a i+ a n i = a i+ a n i + a i+ a n i 즉수학적귀납법에의하여, i n 인정수 i 에대하여 a n = a i+ a n i + a i a n i () 한편 k jn () k j a n 은 k kn =) k ka n 과같다. 이것은 n =0,n = 에서당연히만족한다. 이제 0 j m 인임의의정수 j 에대해 k kj =) k ka j 이라고하자.(m ) ³(i) m 이홀수라면, a m =a m + a m - (m ) 이므로귀납법가정에서 - a m, 즉 - a m ³(ii) m 이짝수라면, m = c k 라하자.(c 는홀수, b 는자연수 ) () 에서 n 대신 m, i 대신 m 을넣으면 m = c k 이므로, 귀납법가정에서 k ka m 또, 점화식에서 n m a m = a m +a m + a m a m = a m (a m + a m +) a n + a n = (a n + a n )+a n =(a n + a n ) a n + a n (n ) + (n ) (mod ) ) k (a n + a n ) 여기서 n = m 이되면,k(am + a m +) 임을알수있다. ³(i)(ii) 에서귀납적으로 k kn =) k ka n 임을알수있다. 48. p, q 가서로다른소수이고, n 은 3 이상의자연수이다. 다항식 f(x) =x n + ax n + pq 가 차이상인두정계수다항식의곱으로나타내어질 a 의값을구하여라. ( 통신강좌 ) n X n X 풀이일단두다항식을각각 a i x i, b i x i 라하자.(a i ;b i : 정수 ) 즉, 다음처럼나타낼수있다. i=0 i=0 Ã n!ã X n! X a i x i b i x i = x n + ax n + pq i=0 i=0 상수항의계수를비교하면 a 0 b 0 = pq

63 . 수론고급문제 63 pja 0 또는 pjb 0 이므로, 편의상 pja 0 라고가정하자. 그렇게되면 p - b 0 가된다. k n 일때, x k 의계수를비교해보면 k X a k b 0 + a i b k i =0 i=0 만일 pja 0, pja ; ;pja k 이면 pja k 가된다. 따라서수학적귀납법에의해 pja i (0 i n ) 가성립한다. 한편두다항식의최고차항의계수는둘다 이던가 인데,로가정할수있다. p - 이므로 n X a n =. 따라서 b i x i 는일차식이되어야한다. 따라서 b = b 3 = = b n =0,b =이다. i=0 만일 qjb 0 라면앞에서와마찬가지로하여 qjb 이되어모순이다. 따라서 qja 0 이다. 그러면 pqja 0 b 0 이다. a 0 b 0 = pq에서 b 0 = 이다. 따라서 f(x) =0은이나 을근으로가지게된다. 따라서 f() = +a + pq =0, 즉 a = pq 또는 f( ) = ( ) n + a( ) n + pq =0, 즉 a =( ) n pq + 한편, a값이위와같으면 f(x) 가정수계수다항식 개로인수분해됨을알수있다. } 49. p 가홀수인소수이고 b 의약수가아니며 ( 즉 p - b) a p b p (mod p n+ ) 이면 a b (mod p n ) 임을증명하여라. ( 통신강좌 995--) 증명 ( 부산과학고 학년허석문 ) a p b p (mod p n+ ) 이므로 a p b p (mod p), 즉 a b (mod p). p k 0(modp)( k p ) 이고 ; a p b p = fa b + bg p b p ³ p = (a b) p + (a b) p b + ³ p ³ p + (a b) b p + (a b)b p p p n ³ p = (a b) (a b) p + (a b) p b + ³ p ³ ¾ p + (a b)b p + b p p p a b 0 (mod p) 이므로, ³ p (a b) p (k+) b k 0 (mod p )( k p ) k 그리고 p 이므로,(a b) p 0(modp ). 그런데 ³ p pk b p p 이므로, ³ p ³ p ³ p pk(a b) p + (a b) p b + + (a b)b p + b p p p 이다. a p b p 0(modp n+ ) 이므로 a b 0 (mod p n ) 이다 m n 인정수 m, n 에대해수열 d(n; m) 이다음과같이정의된다 : 모든정수 m 0 에대해 d(n; 0) = d(n; n) = 이고,0<m<n 에대해 md(n; m) =md(n ;m)+(n m)d(n ;m ) 이다. d(n; m) 은항상정수임을보여라. ( 통신강좌 )

64 64 수론 증명 ( 서울과학고 학년우지철 )d(n; m) = n m 임을보이자. 계산에의해 ³ d(; 0) = = 0 ³ d(; 0) = = 0 ³ 3 d(3; 0) = = 0 ³ 3 d(3; 3) = = 3 ³ d(; ) = = ³ d(; ) = 4 = ³ 3 d(3; ) = 9 = ³ d(; ) = = ³ 3 d(3; ) = = 이므로 n =; ; 3 일때까지는성립한다. 이제 n =; ; ;k 일때까지성립한다고가정하면 md(k +;m) = md(k; m)+(k + m)d(k; m ) ³ k ³ k = m +(k + m) m m ½ ³k + ³ k ¾ ³ k = m +(k + m) m m m = ³ k + ³ k + ³ k ³ k m m +(k +) m m m m = ³ k + ³ k ½ ³ k ³ k + ¾ m + (k +) m m m m m = ³ k + ³ k ½ ³ k m + (k +) m k + ³ k ¾ m m m m m = ³ k + m m ³ k + 따라서 d(k +;m)= 이므로수학적귀납법에의해성립한다. ³ m n 따라서 d(n; m) = = 정수 m 5. x, y, n 이자연수일때, x 3 + y 3 =3 n 을만족하는순서쌍 (x; y; n) 을모두구하여라. ( 통신강좌 ) 풀이 ( 언주중공유식 ) x 3 + y 3 =3 n 을만족하는 (x; y; n) 이있다고하면, x =3x 일때 y 3 3 n 3x 0 (mod 3) 따라서 y =3y 이라할수있다.(x ;y ;n 3) 도근이된다. 이러한조작을유한번하면, x; y 6 0(mod3) 인 x, y 를얻을수있다. x, y 를위와같은과정을통해만들어낸수라가정한다. 이때, n 6= 0 임은자명하다. x 3 + y 3 =(x + y)(x xy + y )=3 n x + y =3 k, x xy + y =3` 이라하면,(k>0;`>0) x xy + y = (x + y) 3xy =3 k 3xy = 3(3 k xy) =3` ) 3 k xy =3 p 꼴이된다. xy > 이고,k 이므로,3 k xy =3 p 에서, xy 6 0(mod3) 이므로 p =0 이된다. ) 3 k xy = ) xy =3 k

65 . 수론고급문제 65 ) x xy + y = 3 k 3 k +3=3 x + y = 3 k xy = 3 k ) 3 4 y 3; y 4 ) y =또는 (i) y = x xy + y = ³ x y y =3 x x +=3 ) x(x ) = ) x = (ii) y =일때x = x =,y =이라하면, =9=3 n 에서 n = 이런원시근은 (; ; ) 밖에존재하지않는다. (x; y; n) 이해이면 (3x; 3y; n +3) 도해가되므로모든해는 ( 3 k ; 3 k ; +3k) 이된다.( 단, k N [f0g) } 5. a 0 =,a =,a n = a n + a n (n ) 로정의되는수열이있다. 모든항이정수임을증명하여라. ( 오스트리아 ) 증명 a n = F n 임을귀납법으로증명. 증명 a n, a n 가서로소라면, a n+ = ( a n + a n ) + 을정리하여 (a n+)a n = a n (a 3 n + a n + a n 3 ) 임에서 a n j a n + 이됨. 그럼 a n+ 도정수가되고 a n, a n 도서로소가됨 ( 귀납법 ). a n+ +a n a n 증명3 이상수임을증명하면됨 (n일때의점화식과 n +일때의점화식을빼어서얻을 수있음 ). 53. 자연수 n (n 3) 이주어져있다. 두자연수 s, n 의최대공약수를 gcd(s; n), 세자연수 s, t, n 의최대공약수를 gcd(s; t; n) 이라할때, n = pq ( 단, p, q 는서로다른소수 ) 에대하여다음등식을증명하여라. ( 한국 995-4) nx nx nx fgcd(s; n)g = gcd(s; t; n) s= s= t= 증명 ( 과기원수학과 94 학번이정훈 ) nx s= t= nx q X fgcd(s; n)g = gcd(n; n) + s= nx gcd(s; t; n) p X fgcd(ip; n)g + fgcd(iq; n)g i= i= +(n (p ) (q ) ) = p q +(q )p +(p )q +(n p q +) = (q + q )(p + p )

66 66 수론 = = nx q X p X 4gcd(s; n)+ gcd(s; ip; n)+ gcd(s; iq; n) s= i= i= +(n (p ) (q ) nx [gcd(s; n)+(q ) gcd(s; p)+(p ) gcd(s; q)+(n p q +)] s= = pq +(p )q +(q )p +(n p q +)+(q )fqp +(n q)pg+ (p )fpq +(n p)qg +(n p q +)n = p (q + q ) + p(q + q ) (q + q ) = (p + p )(q + q ) 54. 자연수 n 에대하여 p =4 n + 이라한다. p 가 3 4n + 의약수이면 p 는소수임을보여라. ( 한국 995-6) 증명 ( 과기원수학과 94 학번엄상일 )3 4n + 이 p 의배수이면 3 4n 3 n 3 p (mod p) 이다. 양변을제곱하면 3 n 3 p (mod p) 이다. 위식에서 ord p(3) 은 k 꼴임을알수있다. 만약여기서 k n 이라면 3 n (modp) 에모순이므로 k =n 이다. 그러므로 ord p (3) = n = p 이다. p =ord p (3) '(p) p 이므로 '(p) =p 이다. 그러므로 p 는소수이다. 55. 유리수좌표의점을딱 개만지나는구면이존재하는가? (Towns 995 봄 SA) 풀이존재함. 임의의무리수 m 에대해 x + y +(z m) = m 은원점만지남. } 56. n개의실수 x ;:::;x n 의곱을p라하자. k =;:::;n 에대해p x k 가항상홀수이면, x ;:::;x n 은모두무리수임을보여라. (Towns 995가을 SA) 증명우선 n. p가무리수임을보이면됨. p xk = a k 로두자. p =(p a )(p a ) (p a n ) 가성립하는상황인데, 이것은정계수다항식. 유리해정리에의해 p가유리수라면최고차항이 이므로 p는정수. 그럼 x k 도모두정수이고홀짝도모두같다. p도홀짝이같아서모순. 57. 임의의자연수 n 에대하여 x x (mod n) 을만족하는근 x 의개수를구하여라 ( 단,0 x<n 이다 ). ( 통신강좌 996--) 풀이 ( 대전과학고 3학년박형우 ) (i) n =일때, 0 x<이므로 x =0만이근이다. 따라서근이 개 (ii) n 일때, n = p e pe pe k k 이라하자.( 단, p i 는서로다른소수 ) x x (mod n), x(x ) = 0 (mod n) 에서 x =0, 은근이된다. x 이라하면 x +( ) (x ) = 이므로 gcd(x; x ) = 이다. 따라서 x 0 (mod a) x 0 (mod b) ab = n; gcd(a; b) =; a > ; b >

67 . 수론고급문제 67 이되어야한다. 그런데중국인의나머지정리에의해위연립방정식은 mod (ab), 즉 mod n에대해단하나의근을가진다. 또한그근을x 라하면gcd(x ;b)=이므로 gcd(n; x )=gcd(ab; x )= gcd(a; x )=a이다. 따라서 gcd(a; b) =,ab = n, a>, b>이되는a; b를잡을때마다근이하나씩나오고그근들은모두다르다.gcd(a; b) =,ab = n, a>, b>이되게a; b를잡는가지수는 a가 n = p e pe pe k k 에서어떤소수를취하느냐에따라결정되므로 ³ k ³ k ³ k = k k 이다. 여기에 x =0,x =의근을더하면전체근의개수는 k 개이다. 따라서 (i), (ii) 로부터근의개수는 ( n =일때 :개 n 일때 : k 개 ( 단, n = p e pe pe k k 이다.) 이다. } 58. pq 가 (5 p p )(5 q q ) 의약수가되는소수 p, q 를모두구하여라. ( 불가리아 차 -) 풀이페르마소정리에의해 5 p p 5 (modp) 이므로 p; q 6= 3일때는pj5 q q, q j 5 p p 라야함. p; q 6= ; 5. a 5(modp) 인 a가존재하고, 그럼 p j q (a q ), 즉 p j a q. 고로 ord p a j gcd(q; p ). 즉 WLOG q>p라하면pja 이되어모순. p, q 중에 3인것이있을때는, WLOG p =3이라하면 3q j (5 3 3 )(5 q q )=9 3(5 q q ). q =3; 3 만가능함. } 주 a; b가모두p의배수가아닐때 p j a m b m 을만족하는최소의 m이다른m의약수가된다는것을간단히보조정리로증명해두고논리를끌어가면이론적인느낌을풍겨서더예쁜풀이가될수있음. 59. 다음의다항식에서 T (n) (x) x 가모든정수 x 에대해 0 의배수가되도록하는자연수 n> 이존재함을보여라. T (x) =x 3 +4x x + 단, T (n) (x) =T (T ( T(x) )) 이다. ( 브라질 996-6) {z } n 번 증명 ( 신한솔 ) f(x) =f(y) (mod0) 이면 x = y (mod 0) 임을보이자. f(x) =x 3 +4x x +=x x +3 7 x =(x +7) (mod 0) (x +7) 3 =(y +7) 3 (mod 0) (x +7) 99 =(y +7) 99 (mod 0) x +7 가 0 의배수이면 y +7 도 0 의배수이므로 x = y (mod 0) 이다. x +7 가 0 의배수가아니라고하면 (x + 7)(x +7) 99 =(x + 7) 00 ==(y +7) 00 =(y +7)(y + 7) 99 (mod 0) x +7=y +7,x = y (mod 0). 따라서증명되었다. 이제각각의 x 에대해인정수 n x 가존재함을보이겠다. 0 개의수 T (x);t (x);t 3 (x);:::;t 0 (x);t 0 (x) 중에서비둘기집원리에 0 로나눈나머지가같은두수 T i (x), T j (x) (i j) 가존재한다. 그럼 (mod 0) (* T (x) 가일대일함수임 ) 이므로 n x = i j 로하면된다. n 0 ;n ;n ;:::;n 00 의최소공배수를 k 라하면이므로 T k (x) x = 0 (mod 0). 따라서증명되었다.

68 68 수론 별해 ( 이상훈 ) T ();T();T(3);:::;T(0) 이법0에대한완전잉여계임을증명하는데에제가쓴방법입니다. T (a) =T (b) (mod0) 이고 a 6= b (mod 0) 이면 (a b)(a +ab+b +4(a+b) ) = 0 (mod 0). 따라서 a + ab + b +4(a + b) =0 (mod0). 4a +4ab +4b +56(a + b) 8=0 (mod 0). (a + b +4) =(7b ) (mod 0). 만약 7b =0(mod0) 이면 a = b =9 (mod 0) 이므로모순. 아니라면 7b 의법0에대한잉여역수를 b 0 라했을때f(a + b +4)b 0 g = (mod 0). 그러나가우스정리에의해 ( 0 )[ 르장드르부호 ]= ( )(0 ) =. 8 따라서모순이다. 즉 T (a) =T (b) (mod0) 이면 a = b (mod 0). 따라서 T ();T();T(3);:::;T(0) 이법0에대한완전잉여계가되고 T 는 (mod 0) 에대하여전단사성을가진다. 이하풀이는신한솔님과동일. 60. P (x) 는 P (x) 가유리수이면 x 도항상유리수가되는유리계수다항식이다. P 가일차식임을보여라. ( 이란 차 -B3) 증명우선 P 는상수함수는아니다. 즉, n을이다항식의차수라하면 n. 임의의 0 아닌정수를다항식에곱해줘도상관없으므로, P 가정계수다항식일때만봐도된다. 또한임의의 0 아닌정수 a에대해 P ( x a ) 를고려해도상관없으므로, 최고차항의계수가 a일때an P ( x a ) 를대신고려하는것으로하면, P 가최고차항의계수가 인정계수다항식일때만봐도된다. 그럼충분히큰 M에대해x>M 의범위에서 P 는기울기가양수이고일대일함수이다. 따라서, 충분히큰소수p에대해P (x) =p + c 는해를딱하나만갖는다 (c는 P 의상수항 ). 그런데, 유리해정리에의해다항식 P (x) c p =0의해는, p 만가능하다. P (), P ( ) 의값은고정되어있고, p는무한히많이선택할수있으므로, p 또는 p를해로갖는경우가무한히많다. 즉, P (x) =x + c 또는 P (x) = x + c 는무한히많은소수들을근으로가지므로항등적으로성립해야한다 ( 결론에는무관하지만최고차항의계수가 이므로전자만가능하다 ). 따라서, P 는일차식이다. 6. m, n 은서로소인자연수이다.5 m +7 m 과 5 n +7 n 의최대공약수를구하여라. ( 일본 996-) 풀이 a n := 5 n +7 n 으로두자. n m 이면 a n = a ma n m 5 m 7 m a n m 이므로 gcd(a m;a n)= gcd(a m ;a n m ). 비슷하게 m<n<m이면 a n = a m a n m 5 n m 7 n m a m n 이므로 gcd(a m ;a n )= gcd(a m ;a m n ). 이런알고리즘에의해 m<n이면 m+n 을계속줄여갈수있으므로 ( 그동안첨자 m, n이서로소라는것도불변 ), 언젠가 m = n 이되거나m =0이되어야종료된다. m과 n이서로소임에서각각 (m; n) =(; ), (0; ) 의경우이고, 첨자의홀짝도불변임을관찰할수있으므로, m, n의홀짝이다르면 gcd(a m ;a n )=gcd(a ;a 0 )=가되고, m, n이둘다홀수이면gcd(a m ;a n )=gcd(a ;a )= 가된다. } 6. x> 는정수가아닌실수이다. 각각의자연수 n 에대해, a n = bx n+ c xbx n c 으로정의하자. 수열 (a n ) 이주기함수가될수없음을증명하여라. ( 일본 996-3) 증명귀류법으로, 주기 T 가존재한다고하자. 즉, an+t = a n. n! 일때 x n! 이므로 bx n+t c bx n c > 0 인 n 이존재한다. 그 n 에대해 a n+t = a n 을식으로풀면 x = bxn+t + c bx n+ c bx n+t c bx n c 이므로 x 는유리수이다. y = x T 으로놓으면 y 도정수가아닌유리수이고 telescoping 으로 T X b m := x T i a mt +i = bx mt +T c x T bx mt c = by m+ c yby m c i=0 이된다. a n+t = a n 임에서 b m+ = b m 이고이것은 by m+ c = by m c + d (d 는상수 ) 를의미한다. 따라서, c m := by m+ c by m c 으로놓으면 c m+ = yc m = = y m c 이되는데, 이로부터 m 을충분히큰수로택하면 c m 이정수가될수없어서모순. 63. 다음조건을만족하는함수 f : N! Z 가존재하기위한 k 의조건을찾아라. (i) f(995) = 996 (ii) 임의의자연수 x, y에대해, f(xy) =f(x) +f(y) +kf(gcd(x; y)). ( 체코슬로바키아 996-5)

69 . 수론고급문제 69 풀이 ( 과기원기계과 96 학번이원철 ) ii) 에서 x = y 라두면 f(x )=f(x x) =f(x)+f(x)+kf(x) =(k +)f(x) 두번적용하면 f(x 4 )=f(x x )=(k +)f(x )=(k +) f(x) 다른방식으로 x 4 를전개하면 f(x 4 )=f(x x 3 ) = f(x)+f(x 3 )+kf(x) =(k +)f(x)+f(x x ) =(k +)f(x)+f(x)+f(x )+kf(x) =(k +)f(x)+f(x ) =(3k +4)f(x) f(x) 6= 0 인 x 를잡으면 (ex. x = 995) (k +) f(995) = f(995 4 )=(3k +4)f(995) ) (k +) =3k +4 ) k =0, ) k = 일때 x = p p n n 이라두면 f(x) =f(p )+ + f(p n n ) (n )f() 이된다. f(5) = 996 으로두고나머지소수에대해서는 f(p i )=0 f() = 0이라두면위식을만족하는함수f가된다. ) k =0일때 x = p p n n 이라두면 f(x) = f(p )+ + n f(p n ) f(5) = 996 나머지소수에대해 f(p i )=0 라두면조건을성립. h 성립 i 답 : k =0, } 64. m 개의주어진양의유리수 r ;r ;:::;r m 이 P m k= r k = 을만족한다. 각각의자연수 n 에대해 f(n) = n P m k= [r k n] 으로함수 f 를정의하자. f(n) 의최대값과최소값을구하여라. 단,[x] 는 x 보다작거나같은가장큰정수를나타낸다. ( 캐나다 996-5) 풀이 0 x [x] < 이므로 mx mx mx mx f(n) =n [r k n]=n r k [r k n]= (r k n [r k n]) [0;m) k= k= k= k= 특히 f(n) 은정수이므로 0 f(n) m 임을알수있다. L이 r k 들의모든분모의공배수이면 r k L이모두정수가되어f(L) =0이되고, mx mx f(l ) = (r k (L ) [r k (L )]) = ( r k [ r k ]) = m k= k= 이된다. 따라서,0과 m 이 f(n) 의정확한최소값과최대값이다. } 65. n ;n;n+ 이모두두자연수의제곱의합인 n 이무한히많음을증명하여라. (Towns 996 봄 SA5b)

70 70 수론 증명 n =(k +). 증명 n =(m(m +)) +. 증명 3 a +=b 의해를찾으면되겠음.(a 0 ;b 0 )=(4; 3), (a k+ ;b k+ )=(a k b k ; b k ). 66. A; A; 3A;:::; A 중어느것도마지막여섯자리가모두같은숫자로끝나는경우가없는 6자리의수 A가존재하는가? (Towns 996가을 JA5) 풀이존재함. A = = 로하면됨. } 67. p + q 가 pq 의배수가되는모든소수 p, q 를구하여라. ( 통신강좌 ) 풀이개요 p, q 중에 가있을때먼저따로풀고, 그후3 p q 라하자. p j p + q 이므로페르마소정리에서 p j q +, 즉 q (modp), q (modp). t (modp) 인최소의자연수를 t라하자. t =또는 q가되는데 (이나 q는안된다는것이자명 ), q일때는역시페르마소정리에서 q j p, 이것은 p q 에모순. 따라서 t =이고그럼 (modp) 이므로 p =3. 이걸대입하면 q j q 3 +, q 3 (modq), q 3 (modq). 그런데페르마소정리에서 q (mod q). 따라서 (modq) 로이것은모순. 답은 (; 3), (3; ) 뿐. } 68. n 인자연수 n 이있다.5 r (mod n ) 이되는최소의자연수 r( 위수 ) 은 n 임을보여라. ( 통신강좌 ) 증명수학적귀납법으로,5 n =+kn n 인홀수 k n 이존재함을보이자. n = 일때 5=+ ;k = n = m 일때 5 m =+km m 이었다고하자. 5 m =(5 m ) = +k m m + k m m = +(k m + k m m ) m+ 따라서 k m+ = k m + k m m 이라두면이것역시홀수이다. 따라서 5 n =+kn n 이다. 즉,5 n (mod n ) 만일 5 r (mod n ) 이라하면, 그런 r 중의최소의자연수를택하자. 그러면 r j n 따라서, r =` 꼴이다.(` n ) 그런데 5 r =5 ` =+k`+ `+ 5 r (mod n ) 이므로, n j k`+ `+ 그런데 k`+ 는홀수이므로, n j `+ : ) n ` + 따라서 n = ` + 즉,5 r (mod n ) 인최소의자연수 r 은 n 이다. 69. a 0 과 a 은정수이다. 수열 fa n g 은다음과같이정의된다. a =a a 0 + a n+ =3a n 3a n + a n (n ) 임의의자연수 m 에대하여, 수열안에어떤연속된 m 개의항이모두제곱수가된다면, 모든항이제곱수임을증명하여라. ( 통신강좌 ) 증명 ( 과기원수학과 96 학번신희성 ) a n = n + ¹n + a 0 (n 0), ¹ = a a 0 ³ a n = n + ¹ + a0 ¹ ³ 4 = n + ¹ + (¹ +) n + a0 ³ 4 = n + ¹ (¹ ) + n + a0 4

71 . 수론고급문제 7 n j¹j + (j¹j o +) n 0 > max, + ja 0 j 라하자. 4 n>n 0 라할때, a n 은단조증가 (strictly increasing) 하고, ³ n + ¹ ³ <an < n + ¹ + 이다. 가정에의해, 적당한 t>n 0 와 b가존재해서 a t = b 이된다. 그결과, t + ¹ <b<t+ ¹ + 이다. 따라서, ¹ 는짝수이고, b = t + ¹ 이다. 그러므로 ³ a t = t + ¹ ; 따라서 a 0 ¹ 4 =0 ³ 결론적으로 a n = n + ¹ 이된다. 70. n 차 (n 5) 의정계수다항식 P (x) 가서로다른 n 개의정수해 =0; ;:::; n 을갖는다. P (P (x)) 의모든정수해를구하여라.( 다항식의해란다항식의값을 0 으로하는것을말한다.) ( 몰도바 997 최종 -y-7) 풀이 P (x) =cx(x a ) (x a n ) 이라둘때, P (P (x)) = 0 의근은P (x) =0;a ;:::;a n 이되는것을말함. P (x) =a i (i ) 의정수해는없음을보이자. b가정수근이라면 cb(b a ) (b a n)=a i. b j a i. a i = bk 로치환하면 c(b a ) (b bk) (b a n)=k. 즉,(k )m = k 꼴의식이된다. k 과 k는서로소이므로이것이가능하려면 jk j =, 그런데 k 6= 0이므로 k =뿐. 그럼 bc(b a ) (b a n)=( 단, 곱중에 (b a i ) 만은제외됨 ) 라야하는데, n 5 라서좌변에 (b a j ) 인수는 3개이상. 이런인수들은서로다른수들이고, 그럼절대값이 인것이하나있고나머지둘은 과 이어야함. 또, jcj = jbj =이어야함. 즉, b a p =,b a q = 인것이있는데jbj =이므로 a p 와 a q 중에 0인것이있어야해서모순. } 7. 다음방정식은유리근을갖지않음을증명하여라. ( 불가리아 차 -3) x + y + z +3(x + y + z)+5=0 증명 (u; v; w) =(x +3; y +3; z +3) 으로치환하면 u + v + w =7. 즉, U + V + W =7K 의 gcd(u; V; W; K) = 인정수해가없음을보이는문제와동치이다. U +V +W +K 0(mod8) 이고, 홀수가적어도하나있으므로이것은성립불가능. 7. m, n 은자연수이고, i =; ;:::;n 에대해 m + i = a i b i 이고 b i 가최대가되도록자연수 a i, b i 를잡자. a + a + + a n = 가되도록하는 m 이존재하는 n 의값을모두구하여라. ( 불가리아 차 -3) 풀이 n 임은당연. 연속한 개이하의자연수에는제곱수가많아야 3개이므로, a i 중 인것은많아야 3개. 따라서, n 7. a i 들이모두서로다름을보이자. 만일 m + i = ab i 이고 m + j = ab j 이라면 ajb i b j j = ji jj n 6. 따라서,(a; fb i;b j g)=(; f; g), (; f; 3g), (; f; g) 만가능하고, 각각확인하면어느것도해가될수없다.( 첫경우는 m + i( ) 가 부터시작해야해서해당없고, 두번째경우는4부터 9까지, 마지막경우는 부터 8까지를포함하는데둘다 5와 7만으로도 a i 의합이 를채우므로곤란함.) 이제 a i 들은 f; ; 3; 5; 6; 7; 0; g에서고른서로다른수들이고, 여기서다섯개이상더하면를넘고하나로가되는경우는없으므로 n 4. n =4인경우는f; ; 3; 6g뿐인데,(6b b b 3 b 4 ) =(m +)(m +)(m +3)(m +4)=(m +5m +5) 이되어모순. n =3인경우는 4, 5, 6 (m =3, +5+6=) 의예가있고, m =인경우는 99, 00 (m = 98, + = ) 의예가있다. 답, 3 } 73. (a) 과 (b) 은연속한몇개의정수의세제곱의합으로나타낼수없음을증명하여라. (Towns 997 봄 JA5) 증명 (a) mod 9 (b) mod 7 로확인.

72 7 수론 증명 p k = m 3 이려면 3 j k 여야함을좀더일반적으로대수적으로증명할수있음. O±cial 참조. 74. p 가 5 보다큰소수일때다음과같은집합을생각하자. X = fp n j n N; n <pg 집합 X 안에 x 6=,x j y 인서로다른원소 x; y 가있음을보여라. ( 통신강좌 ) 증명 ( 충북과학고서준규 ) 실수 x에대해 [x]: 가우스기호 p =[ p p] + k라하자. p>5이므로 [ p p] 이다. p p p p>[ p] > 0 ) p>[ p] ) k = p [ p p] > 0 그리고 k = p p [ p p] = ( p p +[ p p])( p p [ p p]) < ( p p +[ p p]) < f([ p p]+)+[ p p]g =[ p p]+ 곧 k [ p p] i) k= 일때 [ p p] (mod) 이면 p =[ p p] + 0(mod) 가되어 p 가 5 보다큰소수임에모순 ) [ p p] (mod) [ p p]= n (n: 자연수 ) 라하면 p =4n + 이되고 p (n ) =4n X, p =4n X 이고 4nj4n 이니까성립. (p>5 이므로 n> ) 4n <4n ) ii) k =[ p p], [ p p] 인경우 p =[ p p]([ p p]+) 또는 p =[ p p([ p p]+) 가되어 p 가소수임에모순 iii) 이제 <k<[ p p]; [ p p] <k<[ p p] 인경우적당한i(i =; ; ; [ p p] ) 에대해kjp i 임을보이면 <k= p [ p p] X p i X [ p p] >i 이므로성립한다. 그런데 (p i ) = (p [ p p] )+([ p p] i ) = k +([ p p] 에서 kjp i () kj([ p p] i ) 그런데 [ p p] i =([ p p]+i)([ p p] i) 이므로 <k<[ p p] 이면 i =[ p p] k; [ p p] <k<[ p p] 이면 i = k [ p p] 로두면 ( i<[ p p] 성립 ) 두 factor 중하나가 k 가되어성립한다. ) i), ii), iii) 에의해모든소수 p>5 에대해주어진주장이참이다.

73 . 수론고급문제 73 풀이 ) p = n +일때 p>5이므로 n은짝수이다. n = p =p 이므로 n X 또 p (n ) = p n +n =n X n은짝수이므로 njn ) x =n; y = n 으로잡으면된다. ) n +<p<(n +) 일때 x = p n 이라하자. p는소수이므로 p<n n이고 p 6= n + n이다. ) O<p n < n; p n 6= n ) O<j(p n) nj <n ) O<jx nj <n ) p (x n) = p x +xn n = x x +xn = x( x +n) ) x = p n ; y = p (x n) 으로잡으면 xjy이다. } ³ n + k ³ n ³ n + k ³ n 75. k 에대하여 k k 임을보여라. 즉가 + k k 의약수임을보여 + 라. ( 통신강좌 ) µ n 풀이우선 = (n a)(n n )(n n 4) (n n a +) a µ a a! k 여기서 a =; ; 4; 7; ; +임을알수있다. 또한 n n = n(n ) n n =(n +)(n ). n n. µ n 따라서 n k+ µ k 은 =(n + k)(n k ) ky j= k 76. 두자연수 a, b 에대해 a + b µ n + k (n + j) =(k)! k + b + a 를인수로가지고있다. } 이정수이다.gcd(a; b) pa + b 임을증명하여라. ( 통신강좌 ) 풀이 a + + b + = a + b + a + b (a + b)(a + b +) (a + b)(a + b +) = 이므로이정수 b a ab ab ab 이다.(a; b) =r이라면 a = ra 0 ; b = rb 0 (a 0 ; b 0 )=이다. (a + b)(a + b +) (a + b)(a + b +) = ab r a 0 b 0 인데 a 0 와 b 0 는 a + b와서로소이므로 r j(a + b) 이다. 즉 r pa + b이다. } 77. 방정식 y = x 5 4 은정수해를갖지않음을보여라. ( 발칸 998-4) 증명 mod 로생각. 78. 임의의양의정수 a 와 b 에대하여 (36a + b)(a +36b) 는 의거듭제곱으로나타낼수없음을보여라. (APMO 998-) 증명 (O±cial) (36a + b)(a +36b) 가어떤양의정수a; b에대하여 의거듭제곱으로나타내어졌다고하자. 36a + b = m = r; a +36b = n = s

74 74 수론 라하면 36r s =35 37a; 36s r =35 37b 준식은 a; b 에대한대칭식이므로 a = b ( 즉 m = n) 라가정하여도일반성을잃지않는다. 이때 m n = r s = 36a + b < 36 즉 0 5 m n<6 () a +36b 한편 이므로 4 n (4 m n ) = 4 m 4 n = r s =35 37(a b ) 4 m n (mod 37) 관찰에의하여 4 t (mod37) 이되는최소의 t 는 9 이므로 m n 은 9 의배수이다.() 에서 m = n. 이때준식은 a = b 에서 37 a 이므로 의거듭제곱이될수없다. 따라서가정에모순이다. 증명 (O±cial) 36a + b = m ; a+36b = n () 라하자. 여기서 m = n ( 혹은 m 5 n) 라하여도일반성을잃지않는다.() 에서 a; b 는모두짝수이어야하므로 a =a, b =b 이라하면 36a + b = m ; a +36b = n 이와같은방법을계속하면 a = n a n, b = n b n 이라할때 36a n + b n = m n () a n +36b n = 0 = (3) m>n 일때 () 에서 b n =b n+,(3) 에서 a n 는홀수여야하므로 a n =a n+ +(a n+ 0) 이라하자. 8(a n+ +)+b n+ = m n (4) a n+ +36b n+ = 0 (5) a; b 는양수이므로 (5) 에서 a n+ =0,b n+ =0 이때, a = n ; b =0 이므로모순. m = n 이면 () 에서 a = b 이고 (36a + b)(a +36b) =37 a 은 의거듭제곱이되지않는다. 증명 3 (O±cial) 36a + b = m ; a+36b = n (m = n라하여도일반성을잃지않는다.) 이라하면 a; b는양의정수이므로 m 6; n 6이다. R7 m n = 36a + b < 36이므로 m n 5이 a +36b 다. R5준식에서 (36 )b = n 36 m 이므로 35 37b = n+ (8 m n ) 이다. m n = 0,,, 3, 4에대해서 8 m n =7; 6; 4; 0; 이들은 35 37로나누어지지않으므로준식을만족하는양의정수 m, n은존재하지않는다.

75 . 수론고급문제 임의의정수 x에대하여 f(x)+f(x +)=f(x +)f(x +3) 를만족하는함수 f : Z! Z 를모두구하여라. 단, Z는정수의집합이다. ( 통신강좌 ) 풀이 x 일때의식과 x + 일때의식을변변빼면 f(x) f(x +)=f(x +3)(f(x +) f(x +4)) (). 즉, f(x) f(x +)=f(x +3)f(x +5)f(x +7) f(x +k +)(f(x +k) f(x +k +)) 가항상성립하고, f(x) f(x +) 6= 0 인 x 가있다면절대값이 이아닌인수는유한하므로 f( 홀 ) 과 f( 짝 ) 은각각유한개를제외하면절대값이모두, 즉, f(x) 는유한개를제외하면모두절대값이 임을알수있다. 그런예외적인 x 들중최대의절대값을 M 이라하면, jxj >M 일때항상 f(x) =. 그럼 x>m 에서문제의식의좌변은짝수, 우변은홀수가되므로모순. 따라서, f(x) =f(x +) 가항상성립한다. 즉, 두수 a, b 가번갈아나오는함수이다. 문제의식에대입하면늘 a + b = ab, 즉 (a )(b ) = 3 이므로, 와 4 가번갈아나오는함수와 0 과 가번갈아나오는함수뿐이다. } 80. 다음연립방정식이정수해 x ;x ;:::;x n 을갖는 n 을모두구하여라. x + x +50=6x +x x + x 3 +50=6x +x 3 x 3 + x 4 +50=6x 3 +x 4. ( 폴란드 차 -4) x n + x n +50=6x n +x n x n + x +50=6x n +x 풀이첨자는모두 mod n으로보기로하고, 모든 i에대해 (xi 8) +(x i+ 6) =50. 그럼 jx i 6j, jx i 8j은항상, 5, 7 중의하나. 따라서, x i =; 7; 3 중의하나. x i =! x i+ =7! x i+ =3! x i+3 =로순환하게되므로3jn 이고이때항상가능. } 8. S(n) 은자연수 n 의자릿수들의합을나타낸다. 다음을만족하는자연수 n <n < <n 50 이존재함을증명하여라. ( 폴란드 차 -3) n + S(n )=n + S(n )= = n 50 + S(n 50 ) 증명더일반화하여 50개대신임의의 m개일때도존재함을증명하자. 귀납법으로한다. m =일때 : 자명. m =일때 : 두수를95, 04 로하면성립. m 일때n <n < <n m 이조건을만족한다고가정하고이로부터 m + 일때의조건을만족하는예를만들어보자. L을 n m 의자릿수보다크게잡는다면 (n i +0 L )+S(n i +0 L )=n i + S(n i )+0 L + 이고, 이것은 i m 에서일정하므로이것이 N + S(N) 과같도록새로운 N 을하나맞춰주기만하면된다. N =0 L 0 + a = 999 9a 꼴로잡아보자 (L 자리수, L 은나중에결정, a 9). N + S(N) =0 L 0 + a +9(L ) + a =0 L +9L 9 + a 이게 n m + S(n m)+0 L + 이되도록하면된다. 즉, 9L = n m + S(n m ) 0 a 이고 a 를 ;:::;9 중에잘골라주면우변이 9 의배수가되도록할수있다. 이렇게잡은 L 이충분히크다 (n m 의자릿수보다크다 ) 는것만마지막으로확인하자. n m 이 k 자리의수라하자 (m = 일때 04 로잡았고, 그후계속증가하므로 k 3. 그럼 0 k > 0k +50 쯤은항상성립 ). 그럼 0L >9L 0 k > 0k. 따라서, L>k 증명됨. 아, N 이 0 L + n 보다작다는것도언급할필요가있음. 8. 수열 (u n) 은 u 0 =0,u =, 그리고각각의 n 에대해 u n+ 은다음을만족하는, u n 보다큰최소의정수로정의된다 : fu 0 ;u ;:::;u n+ g 의어떤세항도등차수열을이루지않는다. u 00 을구하여라. ( 아일랜드 999-5)

76 76 수론 풀이 3진법으로자릿수에 가없는수열, 즉 진법으로모든자연수를써놓고 3진법으로읽는수열. 답은 u 00 = 0000 (3) =98. 이것을수학적귀납법으로증명하자. 초항들은자명하고, u 0 ;:::;u n 까지잘성립한다고가정하자. 기대하는 u n+ 보다작은 ( 그리고 u n 보다는큰 ) 임의의수 z는 3진법으로썼을때자릿수에 가나타난다. 그런자릿수들을모두 로바꾼수를y, 또모두0으로바꾼수를 x라하면, x; y; z가 x; y fu 0 ;u ;:::;u ng 인등차수열이되어곤란하다. 또역으로, x; y fu 0 ;u ;:::;u ng 인등차수열 x<y<z는 x와 y의자릿수가다른가장낮은자리를생각하면그자리에서 z에 가나타나므로 (0 또는 0... 혹은일반적으로확장하려면그자리에서완전잉여계가되어야함을생각하면됨 ), 기대하는 u n+ 은못만들어낸다. 따라서, u n+ 도잘성립한다. 이상의내용은임의의소수 p에대해 `어떤연속한 p개의항도등차수열을이루지않는다 ' 고문제를수정하면 `p 진법으로쓰고 p진법으로읽는 ' 수열이된다 (p가소수가아닐땐성립하지않는다 ). } 83. 연속한 개의정수중에는 0 개의네제곱수의합으로나타낼수없는수가항상적어도하나있음을증명하여라. ( 폴란드 000/00 차 -9) 증명 n 4 0; (mod6) 이므로 0개의네제곱수의합은 mod 6으로 0에서 0 사이. 그런데연속한 개의정수중에는이범위밖의수가있음. 84. 임의의자연수 k 에대하여, m; m; 3m;:::;m 을이진법으로나타내었을때 0 이아닌자릿수를각각정확히 k 개씩갖도록하는자연수 m 이존재함을증명하여라. ( 폴란드 00/00 차 -5) 증명 m = k 로두면됨.(n +)m = nm + m =( k n)+(m n) 으로분석하면 n<m이므로 k 이상의자리에서 의개수가늘어난만큼 m에서 의개수가줄어듦. 85. 각각의자연수 k 를 k 개씩나열하여만든, 감소하지않는수열 a ;a ;a 3 ;::: 을생각하자. 다음의꼴의수중에서소수인것을모두찾아라. ( 폴란드 00/00 차 -) a + a + + a n 풀이 a n = k 이면 + + +(k ) 도 k를인수로갖고, 그후에a n 까지나타나는 k항들도 k를인수로가지므로, k 6 의배수가됨. 즉, k =4; 5; 7; 8; 9;::: 일때는분자에남는게있으므로합성수임이확실하고, k가 6의약수, 즉 k =; ; 3; 6 일때는직접확인해보면됨. } 86. 적당한자연수 a 에대하여다음의두조건을모두만족시키는음이아닌정수들의순서쌍 (x ;x ;:::;x 00 ) 를모두구하여라. () 0 <x + x + + x 00 a () x + x + + x = a ( 한국 00 차 -S7) 풀이 S = x + + x 00, S = x + + x 00 이라하고 x 이 x i 중가장크다고하자.() 에서 x i 들이모두 0일수는없으므로,() 에서 a>4. 또 x a 이므로 정리하면 a 8, 즉 5 a 9. 한편, a 6 = x + x + + x n x +(x + + x n) x +(a x ) (a ) + x S = x + x + + x n x (x + x + + x n) x a 따라서, S a x ps 이다 (3). 5 a 9 일때 (), (), (3) 을정리하면, a =5 a =6 a =7 a =8 a =9 S 5 S 6 S 7 S 8 S 9 S =9 S =0 S =33 S =48 S =65 x 3 x =4 x =5 x =6 x =8 각경우를풀면 (x ;x ; )=(; ; ;O); (3;O); (4; ;O); (8; ;O) 뿐이다. } 87. a =0,a =30,a n+ =3a n+ a n (n ) 로정의된수열 fa n g n 이있다.+5a n a n+ 이완전제곱수가되는자연수 n 을모두구하여라. ( 발칸 00-)

77 증명귀류법으로 - f(n ); 7 - f(n 7 ); - f(n ); 3 - f(n 3). 수론고급문제 77 풀이이웃한두항을 a, b 라할때귀납적으로 a 3ab + b = 500 임을보일수있고, 그럼 (+5ab) (a+b) =50. 이로부터 +5a n a n+ = k 이라면, k>56 이면 k m k (k ) = k > 50 이되어해없음. 즉, n 7 쯤이면해없고, 그이하를체크하면답이하나뿐이었던듯. } 88. c 이 a + b + 의약수가되는음이아닌정수해 (a; b; c) 를모두찾아라. ( 오폴 00-) 풀이 우선 c =0 이면해가없고, c = 이면모두해가된다. c = 일때는 a + b + ( ) a +( ) b + 0 (mod 3) 이므로, a, b 모두짝수일때가해가된다. 이제 c 3 일때를보자. a (mod c) b (mod c) (, 는 0 ; < c 인정수 ) 라하자. 그러면 a (mod c ) b (mod c ) 이므로 c j + + 여기서, 가최대일때, 즉 = = c 일때 + += c + c += c + 로좌변보다겨우크다는것을관찰할수있다. c 3 이므로이때 c j c + 일수없다. 그밖의경우에서는 + + c + c += c ( c ) c 이된다. 등호는 f ; g = fc ;c g 이고 c =3 일때만성립하고, 그이외에는해가존재할수없다. 즉, 이때의해는 c =3,a (mod3),b (mod3) 이다 ( 복호동순 ). 이상을모두종합하면 (m; n; ); (m; n; ); (3m +; 3n +; 3); (3m +; 3n +; 3) 들이구하는모든정수해 (a; b; c) 이다 (m, n 은음이아닌정수 ). } 89. 집합 A = f; 7; ; 3g 이라하자. 어떤정수계수의다항식 f 가다음의성질을갖는다 :` 각각의정수 n 에대해, p j f(n) 인 p A 가존재한다.' ` 모든정수 n 에대해 p j f(n)' 인 p A 가있음을증명하여라. ( 오폴 00-5) 이라하자. 여기서 f(n) 은다항식이므로 n i m i (mod i) 이면 i - f(m i ) 이다. 중국의나머지정리에의하여모든 i에대하여 n n i (mod i) 인 n이존재한다. 그러면 - f(n); 7 - f(n); - f(n); 3 - f(n) 이므로모순. 따라서문제는성립한다. 90. a +,b + 이모두소수이고 (a +)(b +)=c + 을만족하는모든자연수 a, b, c 를구하여라. ( 폴란드 00 3 차 -)

78 78 수론 풀이 (KAIST 수학과 03 학번김린기 ) a b + a + b = c a (b +)=(c b)(c + b) a b 라하자. () b +j c b 인경우 : c b b + 이므로 c b + b +. 그런데 a b 로부터원래의식을살피면 c + (b +), 즉 c<b + 이어야하고, 이것은모순이다. () b +j c + b 인경우 : 마찬가지로 c b b +. (a) b a + 일때 : 원래의식으로부터 c < ((b ) +)(b +) < (b b + b)(b b ++b) < (b b +) 이므로모순. (b) b = a 일때 : (b +) = c + 로연속한두제곱수의차가 인경우는 (0 ; ) 뿐이어서 b += 이어야하는데, 이것은소수가아니므로모순. (c) b = a + 일때 : a 와 b 의홀짝이다르므로 a + 과 b + 의홀짝도다르다. 이둘이모두소수이므로 a + =, 즉 a = 이고, 이때 (a; b; c) =(; ; 3) 이해가됨을확인할수있다. a>b 일때도마찬가지이므로, 답은 (a; b; c) =(; ; 3), (; ; 3). } 9. 자연수 k가주어져있고, 수열 (a n ) 은다음과같이정의된다. a = k +; a n+ = a n ka n + k (n ) m 6= n 이면 a m 과 a n 은서로소임을보여라. ( 폴란드 00 3차-6) 증명 (KAIST 04학번윤혜원 ) k의임의의소인수를 p라하자. a (modp) 이고, a n+ = a n ka n + k k + k = (modp) 에의해귀납적으로모두 (modp) 가된다. 따라서, k 는모든 a n 과서로소이다 ( ). a n 의임의의소인수 p 를잡자. 그럼 a n+ k (mod p) 이다. 그리고, a n+ = a n+ ka n+ + k k k k + k = k (mod p) 이므로, 이후부터쭉 k (mod p) 이다.( ) 에의해 k 는 a n 과서로소이므로 p 와도서로소이고, 따라서 a m 은 (m >n) p 의배수가될수없다. 즉, 임의의 m>n 에대해 a m 은 a n 과서로소이다. 증명 ( 이매고 학년우승찬 ) a = a ka + k =k + 이므로, a + k = a 가된다. 이성립한다고가정하자. 그럼 a a n + k = a n+ ( ) a a na n+ + k =(a n+ k)a n+ + k = a n+ 가되고, 따라서수학적귀납법으로 ( ) 이모든자연수 n 에대해성립한다. 그럼유클리드호제법의원리에의해임의의 m>n 에대해 gcd(a m ;a n )=gcd(a n X + k; a n )=gcd(k; a n )= 이된다.(gcd(k; a n)= 임은앞의풀이에있으므로생략 )

79 . 수론고급문제 번부터 000 번까지 000 명의직원이있는어떤공장에, 부터 000 까지각직원의번호가붙어있는사물함이있다. 어느날아침모든사물함은닫혀있었는데, 처음출근한직원이자기번호와서로소인모든사물함들을다열어놓았다. 두번째출근한직원이그것을보고따라하여, 자기번호와서로소인사물함들을, 열려있는것은닫고닫혀있는것은열어놓았다. 이런식으로계속해서모든직원이출근하면서자기번호와서로소인사물함들을열려있는것은닫고닫혀있는것은열어놓았다. 이날모든직원이출근했을때, 000 의약수인사물함중에서열려있는것은모두몇개일까? (00 KAIST Cyber 영재교육겨울캠프평가시험 5) 풀이각자가어느사물함을열고닫을것이냐는관점에서각사물함이누구에의해열리고닫힐것이냐는관점으로이동하자. 그럼각사물함은자신의번호와서로소인직원들에의해열리고닫힌다. 즉, 자신의번호와서로소인직원이짝수명이면닫혀있게되고, 홀수명이면열려있게된다. [ 보조정리 ] 임의의양의정수 n> 에대해, n보다작거나같은양의정수중에서 n과서로소인것은짝수개있다. 이보조정리에의하면, 000의임의의약수 d> 에대해구간 (0;d] 의정수중d와서로소인것은짝수개이다. 또한 d + k와 d가서로소인것은k와 d가서로소인것과동치가되므로, 구간 (0;d]; (d; d]; (d; 3d]; ::: ; (000 d; 000] 들의정수중 d와서로소인것들의개수는각각다똑같고모두짝수이다. 따라서 000 이하의양의정수중에서 d와서로소인것은짝수개이므로사물함d는닫혀있다. 이제살펴보야야할남은 000의약수는 과 뿐이다.과서로소인수는모든수로000개이고,와서로소인수는모든홀수로500개이다. 즉, 이들도짝수이므로모두닫혀있다. 따라서, 답은 0개. [ 보조정리의증명 ] n> 이면우선 n 6= 이므로 n은스스로와서로소가아니다. k [;n=) 이면 n과 k가서로소인것은 n과 n k(= k 0 ) 가서로소인것과동치이다. 즉, 이런 k들은 k 0 (n=;n ] 들과서로둘씩짝을이뤄, 둘이동시에서로소이거나둘이동시에서로소가아니다. 또한 n> 이므로, n이짝수여서 n=이정수가되어도 n과 n=(> ) 은서로소가아니다. 따라서, n 이하의양의정수중에서n과서로소인것은모두짝을이루고있으므로짝수개있다. } 93. (m n) j (m + n ) 과 (n m) j (m + n) 을모두만족하는모든양의정수쌍 (m; n) 을구하여라. (APMO 00-) 풀이우변은양수이므로 m n m + n 과 n m m + n 이성립한다. 즉, m m n + n 과 n n m + m, 혹은 m(m ) n(n +); n(n ) m(m +) m>n+ 이면왼쪽부등식이, n>m+ 이면오른쪽부등식이성립하지않는다. 따라서 jm nj. () m = n 일때 ; 두식모두m m j m + m, 즉 m j m +. m =또는. () m = n + 일때 ; n + n +j n + n + 은당연하고, n n j n +3n + 만살피면되겠다. n n j 4n +. n n 은홀수이므로 n n j n +. n 4 이면 n n > n + 이므로 n =; ; 3 일때만살피면된다. 그중만족하는해는 n =;. (3) n = m +일때 ;() 와대칭적. 따라서, 모든양의정수해는 (m; n) =(; ), (; ), (; ), (; 3), (; ), (3; ). } 94. 다음방정식의자연수해 a, b, c 를모두구하여라. ( 영국 00/003 차 -5) a! b! =a! +b! +c! 풀이일반성을잃지않고 a b 라하자. b = 이면우변이더크므로 a; b. () c a 일때 : a! b! a!+b!+c! a!+a!+a! 로 b! 3 이므로 b = 여야한다. 이때 a! =b!+c! b; c < a 여야하고, 이때 a! =b!+c! (a )! 로 a = 일때만가능한데 a = b = 라서모순. 따라서, 이때에는해가없고 c>a 여야한다.

80 80 수론 () c>a>b 일때 : 양변을 a! 로나누면 b! =+ b! a! + c! a! 나머지항이모두정수인데b!=a! 은정수가아니므로모순. (3) c>a= b 일때 : (a!) =a!+c!, 즉 a! =+(a +) c (3a) a가짝수일경우 : c = a + 이면좌우변의홀짝이안맞아모순이발생하므로 c a +. 이때 a+ a+ 가 a 이하의정수이고 a! 과 (a +) c를모두나누므로 j 여야한다. 그럼 a =인데이것은대입하면해가안된다. a+ a+ (3b) a가홀수일경우 : 가 a 이하의정수이고 a! 과 (a +) c를모두나누므로 j 여야한다. 그럼 a =3인데이때 (a; b; c) =(3; 3; 4) 가해가된다. 이상에서 (a; b; c) =(3; 3; 4) 이며, 이해는유일하다. } 95. 다음의조건을만족하는실수계수다항식 W 를모두구하여라 : x + y 가유리수이면, W (x) +W (y) 도유리수이다. ( 폴란드 00/003 차 -9) 풀이 ( 부산건국고 학년신승현, 수정됨 ) x +( x) =0은유리수임이자명하므로 F (x) =W (x)+w( x) 은임의의실수 x에대해항상유리수값만을취하는다항식이된다. 다항식은연속함수이므로서로다른두유리수값을갖는다면중간값정리에의해그사이의무리수값도가져야해서모순. 따라서, F 는유리값의상수함수, 즉 ² W (x)+w( x) c 이다. W (x) c = (W ( x) c ) 이므로 W (x) c 는홀함수 ( 기함수 ) 이고, 따라서, ² W 는유리값의상수항을제외하면홀수차수의항만을갖는다. W 가유리값의상수함수일때는문제의명제가항상성립한다. 이제 W 가 (k +) 차다항식일때를보자 (k 0). 0이아닌적당한유리수 r에대해x+(r x) 는항상유리수이므로 G(x) =W (x)+w(r x) 도항상유리값을갖는다항식, 즉상수함수이다. k 일때G(x) 를이항정리로전개하면 G(x) =(ax k+ + bx k + )+(a(r x) k+ + b(r x) k + ) =(a a)x k+ +(k +)arx k + 로 k차의항이살아있어상수함수일수없으므로모순. 따라서 k =0, 즉 ² W (x) =ax + b 꼴이다. 앞에서 b가유리수임은구했고, a = (W () + W ()) b 이므로 a도역시유리수여야한다. 그리고, a와 b 모두유리수이면이 차식은문제의명제를항상잘만족한다. 그러므로, 답 차이하의유리계수다항식들 } 96. 다항식 W (x) =x 4 3x 3 +5x 9x 가있다. W (a) =W (b) 를만족하는서로다른정수 a, b 의쌍을모두구하여라. ( 폴란드 003 차 -3) 풀이그래프를상상해볼때대충 a + b ; 0 쯤일것임에착안하면됨. a + b = p, ab = q 라하자. W (a) W (b) =(a + b)(a a b + b ) 3(a + ab + b )+5(a + b) 9=p 3 3p pq +5p +3q 9=0. 즉,(p 3)q = p 3 3p +5p 9. p 일때, 절대부등식 p 4q 를이용해q를소거한부등식을구하면 p(p +0) 9(p +4). 여기서 p <9 가되고, p =; 3; 4 일때확인해보면 p =,q = 3 만가능. p 일때도비슷하게확인해볼수있고, 그때해는없음. p =0; 일때만따로확인해보면역시해가없음. } 보다큰각각의소수 p 에대하여, kp +3=x + y 과 0 < k <p 를만족시키는정수 x, y, k 가존재함을증명하여라. ( 폴란드 003 차 -4) 증명 0 x p 에서 x 들은 mod p로모두서로다름. x mod p와 (3 y )modp는둘다종류이므로, 겹치는게있음. 즉, kp +3대신 kp + r 로해도다성립할듯. p+

81 . 수론고급문제 음이아닌모든정수쌍에대해정의되는실함수 f 가음이아닌모든정수 x, y 에대하여다음을만족한다. f(0; 0) = 0; f(x; y) =f(x +; y +)=f(x; y); f(x +; y) =f(x; y +)=f(x; y)+ n, a, b 는 f(a; b) =n 을만족하는음이아닌세정수라고하자. 방정식 f(a; x)+f(b; x) =n 의음이아닌정수해 x 의개수를구하여라. ( 폴란드 003 차 -6) 풀이 f(x; y) 는 x와 y를각각이진법으로써서비교했을때, 서로다른자릿수의개수임. 그럼 f(a; x)+ f(b; x) n 일수밖에없고, 등호가성립할때는 x의자릿수중 a나 b와다른숫자는원래 a와 b가다른곳만가능함. 그런 n개의자릿수만마음대로 x에서결정할수있으므로 n 개. } 99. 좌표평면에서각좌표가정수인모든격자점들을생각하자. 반지름 R 을잘택하면, 원점을중심으로하고반지름 R 인원이몇개의격자점을지나도록할수있다. 예를들어, 반지름 인원은 4 개의격자점을지나고, 반지름 p 인원도 4 개의격자점을, 반지름 5 인원은 개의격자점을지난다. 임의의 n N 에대해, 반지름 R 을잘택하면원점을중심으로하는원이 n 개이상의격자점을지나도록할수있음을증명하여라. ( 플란더즈 003-4) 풀이 ( 부산중앙고 3 학년김규현 ) a + b = c 을만족하는 a; b; c 중자연수이고서로소인 a; b; c 를 (a; b; c) 라하고그것을원시피타고라스세쌍이라하면원시피타고라스세쌍은무한히많다.(* 예를들어 a =n +, b =n +n, c =n +n +. 단, n 은자연수 ) 이제원점이중심인단위원을생각해보면그위에유리수점이무한하다는것을알수있다. 그리고 n 개이상의점을지나게할때, n 개이상의유리수점의분모의최대공약수를반지름으로잡으면 n 개이상의격자점을지나고원점이중심인원을찾을수있다. } 00. 임의의자연수 x > 에대해, x 이 t 꼴이아니면다음을만족하는자연수수열 x ;x ;::: ;x r 이존재함을보여라. (a) x i + x i+ 은모두완전제곱수이다. (i =; ;::: ;r ) (b) p(x ) p(x ) p(x r)=. 단, p(n) 은 n 의소인수중에서가장큰것이다. (ML 프로포절 3-) 증명 n이 의거듭제곱이아니므로 p(n) =p(x ) 3 이다. p(x )=m +, 그리고 n = x = b(m +) a (p(b) < m +) 으로두자. () a =일때 : (m +; m +m; m +m +) 이피타고라스삼중쌍임을이용하자. x = b(m +m) 으로두면, x + x = b (m +m +) 로완전제곱수가되므로 (a) 가만족된다. 또한, x =bm(m +) 이므로 p(x ) < m +=p(x ) 이기도하여 (b) 도만족된다. () a> 일때 :() 을다음과같이확장하자. x = b(m +) a x = b(m +) a (m +m) x 3 = b(m +) a (m +m). x a+ = b(m +) 0 (m +m) a = b a m a (m +) a 그럼 i a 에대해x i + x i+ 은항상완전제곱수가되고, p(x a+) < m +=p(x ) 이다. x a+ 이아직의거듭제곱이아니면위와같은과정을다시반복하여 p(x i ) 을계속줄일수있고, 그럼언젠가는유한번안에 p(x r )=가될수밖에없다. 0. 임의의자연수 n에대해, 모든자리수가홀수이고 5 n 으로나누어떨어지는 n자리수가존재함을증명하여라. ( 미국 003-)

82 8 수론 증명 5 n 의배수이고 0 n 의약수인다음 n 개의수를생각하자. 0 5 n ; 5 n ; 5 n ; :::; n 5 n i 5 n 은 0 i ( 홀수 ) 이므로뒤에서 i번째자리까지는계속 0이고 i +번째자리에서홀수 ( 정확히는 5) 가나타난다. 또한, 이들은모두합해도 µ 0 n n < 0 n 로 0 n 보다작으므로 n 자리수를넘지않는다. 따라서, 이들중에다음과같은절차로적절히몇개를택해서더하면문제에서원하는모둔자리수가홀수인 n 자리수를얻을수있다 : 먼저 0 5 n 을 S =0 에더하면 의자리는홀수가된다. 현재의 S 에뒤에서 i 번째자리에처음짝수 ( 모든자리가홀수이면최고자리위의 0 을처음나타나는짝수로간주 ) 가나타난다면 i 5 n 을 S 에더해 i 번째자리도홀수로만든다. 이런식으로 n 번째자리까지모두홀수가되도록계속하면된다. 증명 수학적귀납법을사용한다. 먼저 n =일때는5로자명. 이제문제의조건을만족시키는 n자리수를a n 이라하자. a n 의앞에, 3, 5, 7, 9 중하나의숫자 k를추가하여 a n+ 을만들수있음을보이자. 이것은이미 5 n 의배수이므로, k 0 n + a n 5 n = k n + M (k =; 3; 5; 7; 9) 중에 5 의배수가있으면된다. 그리고, 이 5 개의수는 mod 5 로완전잉여계를구성하므로 ( 즉,5 로나눈나머지가모두서로다른 5 개의수이므로 )5 의배수가꼭하나있다.( 만일 5 로나눈나머지가같은두수가있다면, 그두수의차 (k k ) n 이 5 의배수여야하고그럼 5 j k k, 즉 k k (mod 5) 인데, 3, 5, 7, 9 는 5 로나눈나머지가모두달라서모순이다.) 따라서, 그경우를 a n+ 으로택하면된다. 0. n n 크기의정사각형꼴의방을 p p, q q 등두가지크기의타일을이용해겹치지않게가득깔수있다고한다. p 와 q 가서로다른소수이면 n 은 p 또는 q 의배수임을보여라. (ML 프로포절 36-) 증명귀류법으로, n이 p의배수도q의배수도아닌데두가지크기의타일로잘깔수있다고가정하고풀자. 일반성을잃지않고p<q라하고, n과 q를 p로나눈나머지를각각a, b라하자. 이제각칸에다음과같이표시를하자. 3 p p a 3 p p a 3 p p a p p a p p a 이렇게번호를달면 ;:::;a 의칸들이 a +;:::;p 의칸들보다 n 개씩더많다 (). p p 타일은방의어디에놓이든 ;:::;p 의칸을각각 p 개씩차지하고, q q 타일은방에놓이는위치에따라어떤종류의칸들을나머지칸들보다 q 개더많이차지한다. 즉, p p 타일과 q q 타일에의해깔리는각종류의칸들의개수차이는 0 또는 q 이므로, 이들에의해완전히깔린방에서각종류의칸들의개수차이는 q 의배수가되어야한다 (). () 과 () 를비교해보면 n 이 q 의배수여야한다는것이고, 이것은귀류법의가정에모순이다. 따라서, 문제는맞는것이다. 03. k 4 는정수이고, p k 를 k 보다작은가장큰소수라하자. p k 3k=4 임을아는것으로해도좋다. n 이합성수일때다음을증명하여라. (a) n =p k 이면, n은 (n k)! 을나누지않는다. (b) n>p k 이면, n은 (n k)! 을나눈다. (APMO 003-3)

83 . 수론고급문제 83 증명 (KAIST 03학번옥성민 ) (a) p k <k에서 p k k<p k. n k<p k 이고, p k 는소수이므로 p k - (n k)!. (b) k 4 이므로 p k 3, n 7 이다. p k 3k=4 임을이용해도되므로, n>p k 3k= 에서 n 3 <n k 따라서, 소인수분해 n = p e pe r r 에서 p e i i 꼴들이모두 n 3 이하라면 n j (n k)! 이된다. 이렇지않은예외적인경우는 p e = n 이나 n이되는경우뿐이다. 즉, n = p e 혹은 n =p e 꼴일때만보면된다. n = e 꼴이면 n 7 이므로 e 5. 이때, n = e 로분해하면 ; e < n <n k 3 이므로 n j (n k)!. 이제 p 3 이라해도된다. n = p e 꼴일때 ; n은합성수라했으므로 e. p 3 이므로 n = p p e 로분해하면 p; p e n 3 가된다. 즉, e 3 이면 n j (n k)!. n = p 꼴이면 n 7 이었으므로 p>6. 따라서 p; p < n 3 으로역시된다. n =p e 꼴일때 ; e 이면 n = p p e 로분해하면된다. n =p꼴이면 n>p k 에서 p>p k 이고, p k 의최대성에서 p k, 즉 n k 가된다. 그럼 n n k 이므로,;p n 여서 n =pj (n k)!. 따라서, 모든경우에 n j (n k)! 이다. 04. a + b = c 의자연수해중에서 a, c 가소수이고 b 는네개이하의소수의곱인것을모두찾아라. ( 폴란드 003/004 차 -7) 풀이 c+a a =일때는해가없으므로a<c는모두홀소수. 그럼 b는짝수. c a =( b ) 에서 c+a = m, c a = n 꼴. 그럼 a = m n =(m + n)(m n) 이소수라야하므로 m n =. (a; b; c) =(n+; n(n+); (n+) +n b ). 4 = n(n+) = 가두개이하의소수의곱. =;p;pq 일때로나눠풀자. =일때n =,(a; b; c) =(3; 4; 5). = p 일때n =,p =3,(a; b; c) =(5; ; 3). 이제 n 3 이고 = pq꼴일때, a =n + 이소수이려면 n은 3k +꼴일수없으므로 는 3의배수, 즉 =3p꼴. fn; n +g = f3; pg or fp; 6g만가능하고, 즉 p =; 5; 7. 각각풀면 n =3; 5; 6 일때이고, c가 5의배수가아닌 n =5일때의 (a; b; c) =(; 60; 6) 만남은해가됨. } 05. 임의의소수 p에대하여 p의배수가운데 x + y + x + y + 여라. ( 단, x, y 는자연수 ) 꼴의배수가무수히많음을보 ( 한국 004-J3) 증명 ( 과천중 3 학년함태준 ) x = t, y = t 로치환하면준식은다음과같이쉬운꼴이된다. ( 준식 )= t 4 + t t +(t ) + = t(t +)(t t +) t(t +) (t = t +) p = 이면 : t 를 4 의배수로택하면준식이항상 p 의배수가되고, p> 이면 : p 는홀수이므로 t 를 p 의홀수인배수로택하면역시준식은항상 p 의배수가된다. 06. 서로소인자연수 x, y 와정수 z 가 (5z 4x)(5z 4y) =5xy 를만족시킨다고한다. 그러면 0z + x + y 와이수를3으로나눈몫가운데적어도하나는완전제곱수임을보여라. ( 한국 004-S)

84 84 수론 증명 ( 서울대수학과 03 학번서인석 ) 준식을다시정리하여다음과같이변형할수있다. (0z + x + y)(9x +9y 0z) =9(x y) ( ) A =0z + x + y, B =9x +9y 0z 로쓰기로하자. A 의소인수분해에서 3 이아닌소인수의지수가모두짝수임을보이면문제가증명된다. p j A, p - B 이면 ( ) 의우변이완전제곱임에서 p 의지수는짝수이다. 이제 p j A; p j B 인소수 p(6= 3) 에대해서만살피면된다. p j (3(x y)) 에서 p j x y 이고, p j A + B =0(x + y) gcd(x + y; x y) =or이므로, p =or5이다. 다시 p j A =0z + x + y 에서 p j x + y 이므로 p =5는불가능하다. 즉, p =. 이때x, y는모두홀수라야한다. 또, 원래의식에서우변이홀수이므로 z는홀수임을알수있다. x + y가짝수이므로원래식에서 z xy (mod 8) 을얻고, 이로부터 x y ; 3; 5; 7(mod8) 을얻는다. 따라서 6 j 9(x y) 이다. 그럼 A, B 둘중적어도하나는 8 의배수이다. 한편, A B 4z 4 (mod 8) 이므로, A 와 B 둘중하나는 mod 8 로 4, 즉 의지수가짝수이다. 따라서,( ) 의우변이완전제곱임에서, A 와 B 모두 의지수는짝수이다. 07. p 를홀수인소수라하자. p X k p k= p(p +) (mod p ) 임을증명하여라. 단, a b (mod m) 은 a b 가 m 의배수임을뜻한다. ( 캐나다 004-4) 증명 이항정리에의해 ³ p r p +(p r) p = r p r p + p r p p A + + p p (p )pr p pr (p ) (mod p ) 따라서, r =;:::; p 에대해이것을모두합하면, p X k p = k= = p X r= p X r= p X r= r p +(p r) p p X r= pr (p ) p(pa r +) (* Fermat 의작은정리 ) p = p ( p) (mod p ) 이므로, 마지막식에 p 을더하여정리하면문제가성립함을알수있다.

85 . 수론고급문제 W 는정계수다항식이고, 어떤서로다른두정수에서의 W 의값이서로소라고한다. 어떤서로다른무한개의정수들이있어서, 그정수에서의 W 의값들이모두둘씩서로소임을증명하여라. ( 폴란드 차 -) 증명다음을보이자 :[ 보조정리 ] 임의의자연수 m에대해, W (n) 이 m과서로소인정수 n이항상존재한다. W (n + p) W (n) (modp), 즉 W 는 mod p로주기가 p이다. 임의의소수 p에대해, W 의값이항상 p의배수일수는없으므로 ( 항상 p의배수라면, 서로다른두정수에서 W 의값이서로소라는문제의조건이성립하지않음 ), n r p (mod p) 이면 n이항상p와서로소인상수 r p 가존재한다. m의소인수를 p ;:::;p k 라하면, 모든 i =;:::;k 에대해n r pi (mod p i ) 를만족하는 n이존재하고 ( 중국의나머지정리 ), 그 n에대해서는w (n) 이어떤p i 의배수도아니므로 W (n) 과 m이서로소이다. 보조정리증명끝. 이제문제에서원하는무한개의정수열 a ;a ;::: 이존재함을보이자. 먼저 a 은 ( 근이아닌것으로 ) 아무렇게나잡고, a ;:::;a n 을잡았을때보조정리에의해이들의최소공배수를 m과서로소인정수 a n+ 을잡을수있으므로귀납적으로이런수열을잡을수있다. 09. 모든자연수 a 에대해다음을만족하는자연수 n> 을모두찾아라. a Á(n)+ a (mod n) (ML 프로포절 43-) 풀이 (I) p j n 일때 (p는소수): a = p 라놓으면 a(a Á(n) ) 은 p 의배수가될수없다. (II) n이 square-free일때, 즉 n의소인수의지수가모두 일때 : n = p p p m 으로소인수분해된다고하자. (a) n j a 이면 ) trivial하게성립. (b) (a; n) =이면 ) 페르마의소정리에의해성립. (c) (a; n) =p p p l ( 당연히 l m) 이면 : a(a Á(n) ) 은 p ;p ;:::;p l 로나뉘어진다. 그리고 a Á(n) = a (p )(p ) (p m ) (mod p k ) (k>l) ( 왜냐하면 (a; p k )=) [ 답 ] 소인수가 multiple이 인모든자연수 (square-free인자연수 ) } 0. 임의의실수 x에대해, x보다작거나같은가장큰정수를 bxc로쓰기로하자. 모든양의정수 n에대해 ¹ º (n )! n(n +) 이짝수임을증명하여라. (APMO 004-4) 증명 (KAIST 과학영재센터연구원고봉균 ) 먼저 n 5 일때는분모가분자보다커서준식이 0임을각각금방확인할수있다. n 6 일때만보자. 먼저다음의사실을확인하자. () n>4 인합성수이면 (n )! n 은짝수. () n>4 인합성수이면 (n )! n+ 은짝수. (3) (Wilson의정리) 임의의소수 p에대해 (p )! (modp). () 의증명 n이홀수이면 n = ab (a; b 3 인홀수 ) 꼴로나타난다. 그럼 a; b f;:::;n g 이므로 n j (n )!. n이짝수이면 ( 가 ) n = k a (a는 3 이상의홀수, k ) 꼴이면 4; k a f;:::;n g 이므로 n j (n )!. ( 나 ) n = k (k 3) 꼴이면 ; 6; k f;:::;n g 이므로 6n j (n )!. () 의증명 () 에의해 n! n+ 은짝수이다. 분자의 n 과분모의 n + 이서로소이므로 (n )! n+ 은정수이고, 특히 n 이홀수일때는사라진 n 이홀수여서, 또 n 이짝수일때는분모의 n + 이홀수이고분자에 가있어서 (n )! n+ 은언제나짝수이다.

86 86 수론 이제문제를풀어보자. (i) n과 n +이모두합성수일때 : (), () 에의해 (n )! 로, n (n )! n(n+) 은짝수이다. 과 (n )! n+ 은짝수이고, 또 n 과 n + 은서로소이며둘중하나만짝수이므 (ii) n이소수일때 : n +은합성수이므로 () 에의해 (n )! n+ 은짝수이다. 또,(3) 에의해 (n )!+ n 은정수, 특히분모분자가모두홀수이므로홀수이다. 그럼부분분수를이용해 ¹ (n )! n(n +) º = ¹ º (n )! (n )! n n + ¹ = n = + 홀 짝 = 짝수 º (n )! + (n )! + n n + (iii) n +이소수일때 : n은합성수이므로 () 에의해 (n )! n 은짝수이다. 또,(3) 에의해 (p )! (p )(p )! (mod p) 이므로 (n )! n+ 은정수이고, 특히분모분자가모두홀수이므로홀수이다. 그럼부분분수를이용해 ¹ º ¹ (n )! (n )! = n(n +) n = (n )! n + º (n )! n + ¹ n + = 짝 +( ) 홀 = 짝수 º (n )! n + (i){(iii) 에의해준식은언제나짝수이다.. 서로다른소수 p, q 에대하여 S p;q = fp; q; pqg 라정의하자. 집합 S p;q 의임의의두원소가 x +005y ( 단, x, y 는정수 ) 꼴이면나머지한원소도그러한꼴로나타낼수있음을보여라. ( 한국 005-J6) 증명 x +005y 꼴로나타내어지는수의집합을 S라하자. (i) p; q S 일때 ; p = a + 005b, q = c +005d 이라하면 pq =(a +005b )(c +005d )=(ac 005bd) + 005(ad bc) 으로 pq S 이다 ( 복호동순 ). (ii) p; pq S 일때 ; p = a + 005b, pq = x +005y 이라하자. 이때, ac 005bd = x; ad bc = y 를만족하는정수 c, d가존재함을보이면충분하다 ( 복호동순 ). 이연립방정식을풀면 ( 그리고 a + 005b = p 임에서 ) ax 005by bx ay c = ; d = p p 이다. d가정수이면 c +005d = q 에서 c도정수이므로 (c 이정수이면 c는정수가아니면무리수 bx + ay bx ay 인데, c는유리수이므로 ), 결국나가정수임을보이면충분하다. a +005b = p, p p x +005y = pq 에서 이므로, x 005y (mod p); a 005b (mod p) (bx + ay)(bx ay) =b x a y 005(b y b y )=0 (modp) 가된다. 따라서, p j bx + ay 또는 p j bx ay 가되어성립한다. (iii) q; pq S 일때 ;(ii) 의경우와대칭적이므로역시증명된다. (i), (ii), (iii) 에의해어느경우에나결론은증명된다.

87 . 수론고급문제 87. 자연수 n에대하여, n + n + 의약수의개수를 f(n) 이라할때, f(n) f(n +) 인자연수 n이무한히많이존재함을보여라. ( 한국 005-S7) 증명귀류법으로, f(n) f(n +) 인자연수 n이유한개밖에없다고가정하자. 즉, 어떤자연수 N이존재하여, n N 이면 f(n) <f(n +) 이성립하게된다. 그로인하여 n N 이면 f(n) >n N 이성립한다. 이제 n k 를다음의세조건을만족하도록잡자 : () n k = p p p k ( 소수를크기순서로 p =,p =3,p 3 =5,::: 로둠 ) () p p p k > N (3) p p p k > 4 k gcd(n k + n k +;n k +)= 이므로 n k + n k +=q q q r (q ;q ;::: 들은소수 ) 라고하면 q ;q ;:::>p k 이다. 만약 r k 라면 n k + n k +=q q q r >p r k (p k) k > (p p k ) =(n k +) 이되어모순. 따라서, r<k 이고, n k + n k + 의약수의개수는 r 개미만이다 ( 약수는 q i 들의곱인데, 이곱이모두다를때최대인 r 개가된다 ). 그러나, n k N<f(n k ) < 4 k < p p p k <p p p k N = n k N 이되어역시모순. 따라서, n 은무한히많다. 3. 다음을만족하는자연수순서쌍 (x; y; n) 을모두구하여라. ( 폴란드 차 -) (x y) n = xy 풀이 d =gcd(x; y) 라하자. x = da, y = db 라두면d n (a b) n = ab. a b 는 ab와서로소이므로 ja bj =. 그다음은싸바싸바하면됨. } 4. 자릿수사이에 0 을몇개적당히끼워넣어 7 의배수로만들수있는자연수와그렇지못한자연수를모두분류하여라. (005 KAIST Cyber 영재교육가을과제 ) 풀이모든자릿수가 7과 0으로만이루어져있는수는당연히 7의배수이고 0을어디에끼워넣어도 7의배수이다.7이나 0이아닌자릿수가딱하나뿐인수는 7의배수가아니므로,0을어디에끼워넣어도마찬가지로 7의배수가될수없다. 이제 7이나 0이아닌자릿수가둘이상있는수는항상자릿수사이에 0을몇개적당히끼워넣어 7의배수로만들수있음을보이자. 그런자릿수가양쪽에하나이상씩포함되도록전체자릿수를둘로나누자. 즉, n =0 m a + b 의꼴로분리한다. 만일 a나 b 중에 7의배수인것이있다면미리적당히 0을끼워넣어 7의배수가아니도록만든다 ( 예를들어, b가 7의배수라면 b의 7이나 0이아닌가장높은자릿수를택해그다음에 0을하나끼워넣으면 7로나눈나머지가바뀌게된다 ). 이제 a와 b가둘다7의배수가아닌수가되었으면, a와 b 사이에 0을차례로끼워넣다보면 6번이내에반드시 7의배수가된다 ( 왜냐하면,0 m a 를 0배할때마다 mod 7로! 3!! 6! 4! 5! 로주기 6으로순환하므로그중에 b와합동이될때멈추면된다). 답그렇지못한수 : 7이나 0이아닌자리수가딱하나있는수 } 5. 3 k +5 k 가거듭제곱수가되는자연수 k 를모두찾아라. 단, 거듭제곱의지수는 보다커야한다. ( 폴란드 차 -) 풀이홀수 + 홀수 = 짝수이므로 3 k +5 k =(a) b 꼴. b 이므로이것은 4 의배수. 그런데 3 k +5 k ( ) k + (mod 4) 이므로 k 는홀수, k =l l +5 5 l 8 9 l 8(mod6) 이므로 b =3 이고 a 는홀수. 다음 mod 9 로확인해보면 k = 또는 k =6m +3 꼴. 다음 mod 7 로확인해보면 k = 일때빼고는해없음. k = 일때는해가됨. 답 k =. }

88 88 수론 6. p 5 는소수이다. p p 크기의사각판에 p 개의 ( 똑같이생긴 ) 말을한칸에많아야하나씩놓는데, 모든말이한행에놓이지는않도록 ( 그러나모든말이한열에놓일수는있다 ) 하는방법의수를 r 이라하자. r 은 p 5 의배수임을증명하여라. (APMO 006-3) 증명 ( 서울경희고 학년이승후 ) 가능한경우의수는 p n p p p 가지가있다. 이식을정리하면 p o p 이다. 따라서 p 4 ³ p p 임을보이면된다. 또 ³ p = (p )(p ) (p (p )) p (p )! 은정수이고, 기약분수로만들때, 분모는 p와서로소이므로 p 4 j (p )(p ) (p (p )) (p )! () 임을보이면된다. 위의식을전개했을때, (mod p 4 ) 으로관찰하면 µ p (p )! p 이남는다. p 5 일때,+ + + p 를기약분수로나타내면분자가 p 의배수가된다는것은 Wolstenholme 의정리로잘알려져있다. 따라서, 위의식은 p 4 의배수이고, 문제가증명이되었다. 7. p 는소수이다. x ; x x ; x x x 3 ; :::; x x x p 들이 mod p 로모두다르게되는, f; ;:::;p g 의재배열 (x ;x ;:::;x p ) 이존재함을증명하여라. ( 폴란드 006/007 차 -8) 증명우선재배열임은잊어버리고, i p 에대해늘x x i i (mod p) 가되도록 x i 들을결정하자. 이것이재배열이됨을보이자. x i x j (= a) (modp) 인 i<j p 이존재한다면,(i )a i 이고 (j )a j (mod p). 즉,(i )(a ) (j )(a ) (mod p). 이로부터 a 은 p와서로소이므로소거하면 i j (mod p) 가됨. 8. 자연수 k 에대하여 k 의모든양의약수의곱을 F (k) 로나타내자. F (m) =F (n) 을만족하는서로다른두자연수 m, n 이존재하는지증명또는반증하여라. ( 폴란드 006/007 차 -9) 증명 F (k) =k d(k)= 임이꽤알려져있다. F (m) =F (n) 이면, 우선 m과 n은똑같은소인수들을갖는다. 또한, 각소인수의지수의비가모두일정하다. 즉, m = a x, n = a y 꼴이고, x와 y의대소관계가 d(m) 과 d(n) 의대소관계와같으므로둘은같을수밖에. 즉, 존재하지않는다. 9. n 은자연수이다. a = n 이고, 각각의 k > 에대해 a k 는 a + a + + a k 가 k 의배수가되는 0 a k k 범위의유일한정수로정의하자. 예를들어, n =9 이면수열은 9; ; ; 0; 3; 3; 3;::: 이된다. 임의의 n 에대해수열 a ;a ;::: 는항상결국상수가됨을증명하여라. ( 미국 007-) 증명 k x k := a + + a k n +(++ +(k )) = n + k(k ). 즉, x k n k + k. 그럼 k n 일때, x k +(k ) = k. a k+ x k =(k +)(x k+ x k ) 로 k + 의배수인데 ja k+ x k j k 이므로 a k+ = x k = x k+ (k n). 즉 a n 부터상수수열. 0. 임의의음아닌정수 n 에대해,7 7n + 은 n +3 개이상의 ( 서로다를필요는없는 ) 소수들의곱임을증명하여라. ( 미국 007-5) 증명 n =0 일땐성립. n 일때성립가정하고 n + 일때를보자. x =7 7n 로두면 7 7n+ += x 7 +=(x +)(x 6 x 5 + x +). x + 은 n 일때의가정에의해 n +3 개이상의소수들의곱이므로, x 6 x 5 + x + 이둘이상의소수의곱임을보이면충분함. x 6 x 5 + x += A 7x B =(A + p 7xB)(A p 7xB) 이므로 ok. 단, A = x 3 +3x +3x + 이고 B = x + x +.

89 제 장 대수. 대수중급문제. A, B, C 를임의의세실수라고하자. 모든자연수 n>º 에대해다음부등식이성립되도록하는수 º 가존재함을증명하여라. ( 헝가리 95-) An + Bn + C<n! 힌트적당한 º 에대해An + Bn + C<n 3 n! 임을보인다. º>jAj; jbj; jcj; 3 이되도록잡으면쉽게확인된다. 예를들어 º = jaj + jbj + jcj +3. }. k 와 n 은 k n 인자연수들이고 x 는 0 x< n 인실수이다. ³ n ³ n ³ n x + x 0 +( ) k³ n x k > 0 k 이성립함을증명하여라. ( 헝가리 99-) 증명 ETS: n k x k > n k+ x k+. 동치변형하면 k +> (n k)x. 이것은 k + >nx (n k)x 로성립. 3. 네실수 a<b<c<d 를임의의순서로다시쓴것을 x; y; z; t 라하자.(x y) +(y z) +(z t) +(t x) 의최대값과최소값은얼마인가? ( 헝가리 943-3) 풀이 ( 육형빈 ) 주어진식을전개하면 P =(a + b + c + d ) (xy + yz + zt + tx) 이다. a, b, c, d 에서두개를뽑는방법은 6 가지이다. 뽑은값을모두더하면, Q = ab + ac + ad + bc + bd + cd 이식에서적절하게두항을뺀값이 xy + yz + zt + tx 이다. 즉, Q 에서 ab + cd, ac + bd, ad + bc 의최대값을빼면 P 는최소가되고최소값을빼면 P 는최대가된다. a>b>c>d 이므로최대값은 ab + cd 이고, 최소값은 ad + bc 이다. 따라서, 준식의최소값은 (a + b + c + d ) (ab + ac + bd + cd), 최대값은 (a + b + c + d ) (ac + ad + bc + bd). } 4. 실계수 3차방정식 x 3 + ax + bx + c =0이세실근을갖는다. a 3b 0 이고 p a 3b 은최대근과최소근의차보다크지않음을증명하여라. (Putnam 95-B6a)

90 90 대수 증명세근을 라하면, a 3b 는 ( + + ) 3( + + ) 와동치이고이것은재배열 또는두배하면 ( ) +( ) +( ) 0 과동치. 또, 최대근과최소근의차는제곱하면 ( ) =(( )+( )) ( ) +( ) 임에서 ( ) (( ) +( ) +( ) ) 이되고이우변은앞에서보았듯이 p a 3b와같은식임. 주미국 988- 에도재출제된문제. 5. a j (j =; ;:::;n) 들은 이아닌임의의실수들이다. 다음을증명하여라. 0 nx i Y ny a i ( a j ) A = ( a j ) (Putnam 95-A5) i= j= j= 증명귀납법으로간단. 6. 실수 x ;x ;x 3 은어느둘을더해도나머지한수보다크다. 다음을보여라. Ã 3X 3X! Ã 3X! x i!ã x i > x 3 i + x x x 3 (Putnam 953-A3) 3 i= i= i= 증명 우선세수는모두양수.3 곱하고전개하여정리하면 (x x + ) >x3 +x3 +x3 3 +3x x x 3 과동치. 이것은 (x + x 3 )x >x3 과 x x (x + x ) >x x x 3 등을모두합하면이부등식이됨. 증명 위부등식은 ( x + y + z)(x y + z)(x + y z) > 0 을전개한부등식에의해일부소거한후남은것을간단히비교해도됨. 증명3 x = b + c, x = c + a, x 3 = a + b 로 Ravi 치환하여그냥아무생각없이전개해서풀어도되긴하지만막노동. 7. A는예각이다. 다음을보여라. ( 헝가리 963-) µ + µ + > 5 sin A cos A 증명 L ( + p ) =(+ q sin A cos A sin A ) ( + p ) > 5 8. f(x) =+ x 에대해, f (x) =f(x), f (x) =f(f (x)), f 3 (x) =f(f (x)), ::: 로정의하자. 임의의자연수 n 에대해, f n (x) =x 의해를모두구하여라. ( 스웨덴 966-4) 풀이 f n(x) 은일차함수 / 일차함수꼴의유리함수임을금방확인. 그럼 f n(x) =x 는이차함수이고, 기껏해야해는두개. 근데 f(x) =x 의해 x =, 는항상해가되니까이게전부. } 9. 서로다른 n 개의양수들 0 <a <a < <a n 이주어져있을때, 이들을둘씩짝지어곱해합한값은최소 a a n + a a n + + a n a n+ 임을보여라. 증명재배열부등식에서알고리즘적으로 sorting하듯이귀납적으로 an 과는 a 과짝이아니면a 과짝이되도록바꾸어주고... 하면됨. 혹은 a ;:::;a n 중에서로짝인 a i <a j 가있다면 a n+ ;:::;a n 중에도서로짝인 a k <a l 이반드시있게되고, 그럼 a i a j + a k a l >a i a l + a j a k 임을말한후이런작업의반복으로더이상 a ;:::;a n 안에서짝을이루는것이없게되면재배열부등식으로마무리.

91 . 대수중급문제 9 0. 함수 f(x) 는, 모든실수 x, y 에대해다음관계식을만족한다고한다. 이때, f(x) 가다음성질을가짐을증명하여라. f(x + y)+ f(x y) = f(x)+f(y) () f(0) = 0 () f( x) =f(x) (3) 모든자연수 n에대해f(nx) =n f(x) ³ m x ³ m (4) 모든자연수 m, n에대해f = f(x) n n 풀이 () x = y =0 대입.()x =0 대입. (3) n = 일때자명하고, n = 일때는 y 에 x 를대입하면성립.(x; y) =((n )x; x) 를대입하면 f(nx) =f((n )x)+f(x) f((n )x) =((n ) + (n ) )f(x) =n f(x) 가되므로귀납적으로성립. (4) 위의식에 x 대신 x n 를대입하면 f(x) =n f( x n ), 즉 f( x n )= n f(x). 이로부터 f( m n x)=f(m x n )= m f( x m )= n n f(x). }. 임의의양수 a, b, c 에대해다음부등식을증명하여라. +a + +b + +c a + b + c 증명음의부호의항을우변으로넘겨주고우변을통분하자. +a + +b + +c 양변의분모를서로넘겨주고 9 로나누어주면 a + b + c 3 a + b + c +a + +b + +c = 3+ a + b + c 3 a + b + c + a + + b c 와동치가된다. 여기서 +x 와 + x 는둘다양수x가증가함에따라감소하는함수이므로 +a, +b, +c 와 + a,+ b,+ c 는크기순서가같다. 그리고 +x + x = +x x+ x = x 이므로, 위의부등식은체비셰프부등식으로성립한다. 주통분하고모두전개해서풀어도된다. 기본대칭식은같이묶어다니게하면서열심히전개하고소거하면 (a + b + c)(ab + bc + ca)+(ab + bc + ca) 9abc +6abc(a + b + c) 의식이남게되는데, 차수가같은항끼리비교하여 (a + b + c)(ab + bc + ca) 9abc (ab + bc + ca) 6abc(a + b + c) 이됨을확인하면된다. 윗식은산술-기하로바로되고, 아랫식은양변을 (abc) 으로나누어주면 µ a + b + µ 3 c ab + bc + ca a + b + c ab + bc + ca 와동치인데, 이것은잘알려진부등식 x + y + z xy + yz + zx 꼴이다.

92 9 대수. 임의의양의실수 a ;:::;a k 에대해서다음부등식이성립함을증명하여라. 단, s = a + a + + a k 이다. kx i= a i k s a i k 증명양변에 k 를더하면 ( 좌변의각항에 을더하면 ) kx a i ) kx i= i= s kx k s a i k 과동치. 즉, (s i= s a i k. 이것은 AM-HM 부등식혹은코시부등식으로확인하면끝. 3. r 이자연수라고하자. 절대값이 r 보다작은정수들을계수로가지는모든다항식은 x rx 을인수로갖지않음을보여라. ( 폴란드 차 -3) 증명 r보다절대값이작은정수들만을계수로갖는다항식은 r보다큰근을갖지않음. 이부등식은 r보다큰근 에대한 진법으로상상하면쉬움. 4. fx i g, fy i g (i =;:::;n) 들을같은순서로정렬된수열이라하고, fz i g를 fy i g의재배열이라하자. 다음 을증명하여라. nx (x i y i ) nx (x i z i ) i= i= nx 단, a i = a + a + + a n 을의미한다. (IMO 975-) i= 증명 fx i g 와 fy i g 는같은순서로정렬되어있고, fz i g 는 fy i g 의재배열이므로 nx nx x i y i x i z i () i= i= () nx nx x i y i x i z i i= i= nx nx (x i y i ) (x i z i ) i= i= 5. 다음연립방정식을만족하는모든실수해 x, y, z, w 를구하여라. (Putnam 977-A) x + y + z = w; x + y + z = w 풀이 x + y + z = w, x + y + z = w 일때 x, y, z 는 t3 wt + kt wk =0 의해이다. 이는비에타 의정리 (= 근과계수와의관계 ) 에의해서쉽게알수있다.( 두식을각각정리해보면쉽게알수있다.) t 3 wt + kt wk =(t w)(t + k) 임을쉽게알수있고따라서x, y, z 중적어도하나는 w여야한다. 또한, 남은두개는더하면 0인관계에있다는것을알수있다.( 이모든게실수여야하므로 k는음수이다.) 따라서실수해 x, y, z, w는 (w; s; s; w), (s; w; s; w), (s; s; w; w) (s; w는임의의 0이아닌실수 ) 이다. } 6. 다음을증명하여라. ( 오폴 978-3) 44 p tan ± tan ± tan 44 ± < p < tan ± + +tan44 ± 44

93 . 대수중급문제 93 증명 ( 양희빈 ) 왼쪽 tan tan(45 ± ) =tan tan +tan (p ) 임을증명하자. (3 p )(tan +) tan ( tan ) tan ( p ) tan +( p ) =(tan ( p )) 0 ) tan tan +tan (p ) ) 44p tan ± tan ± tan 44 ± < p 오른쪽 P 44 ± k= ± tan k tan ± tan ± = = tan 44± 젠센부등식 44 tan ± tan ± tan 44± =tan( ) = tan( 45 ) > tan( ) cos =cos! cos 45 p + = 4 sin =sin cos p sin cos = 4 ) tan( 45 p )= p =( p p ) + = p ) 성립 7. 임의의자연수 k 에대해다음과같이 n 개의제곱근기호로된수를정의하자. a n = r k + q k + + p k (a) 각각의고정된 k에대해이수열이수렴함을보여라. (b) 이극한값이정수가되는 k를모두찾아라. 더불어, k가홀수이면이극한값은무리수임을증명하여라. ( 오폴 978-8) 풀이 a n 은증가수열. a n k = a n <a n 이므로 a n a n <k로상한이존재. 따라서수렴. 극한값은 a a = k 를만족. 따라서, k = a(a ) 꼴이고당연히짝수. } 8. 다음의꼴을한정계수다항식들중에서 n 개의근 x ;:::;x n 이 k =;:::;n 에대해 k x k k + 이되는것을모두찾아라. ( 오폴 979-) P n(x) =n! x n + a n x n + + a x +( ) n n(n +)

94 94 대수 증명 ( 장도한 ) n 차정수계수방정식 P n (x) =n!x n + a n x n + a n x n + + a x + a x +( ) n n(n +)=0 의 n개의근이 x k (k x k k +) 이므로, n! n k= x k (n +)! 이고 n k= x n(n +) k = 이다. n! ) n =또는 () n =일때, P (x) =x 이고 x (= ) 이다. () n =일때, P (x) =x + a x +6이고 x, x 3이므로, P () = a +8 0, P () = a +4 0이고 P (3) = 3a +4 0이다. ) 8 a 7, a 은정수이다. ) a = 7 또는 8 ) P (x) =x 7x +6(x = 3 ;x =) 또는 P (x) =x 8x +6(x =;x =3) 9. 임의의 n개의양수 a ;:::;a n 에대해다음부등식이성립하는자연수 n을모두찾아라. ( 오폴 979-3) Ã X n!ã n! X nx a i a i a 3 i 6 Y n a i i= i= i= i= 풀이 ( 김규완 ) a i = n 인경우에도부등식이성립해야한다. ) n 5 n 4 6n n n 5 이면 6n n >n 5 >n 5 n 4 이므로식이성립하지않는다. n =; ; 3; 4 에대해대입해보면 n =3 만이가능함을알수있다. n =3 인경우는재배열부등식에의해자명하다. } 풀이 ( 민원준 ) n =인경우a a a 3 6a 이성립하지않는것은자명 n =인경우 (a + a )(a + a ) (a 3 + a 3 ) 6a a, a + a 6 그러므로언제나성립하지는않는다. n =3 인경우 (a + a + a 3)(a + a + a 3 ) (a 3 + a 3 + a 3 3)= X sym a a 6a a a 3 ( 왜냐하면산술 - 기하평균부등식 ) n>3 인경우 a =a =a3 = = an = k 으로잡으면 nk nk nk 3 6k n 을만족해야한다. 고로 n n 6k n 3 이성립해야하고 k = n n 으로놓으면 n 3 > 0 이고 n n 이 이상이므로 (n n) (n 3) n n 이므로성립하지않는다. 고로 n =3 으로유일 } 0. 다음의두복소계수다항식에서, P (x) =0 의해는 x ;x ;:::;x n 이고, Q(x) =0 의해는 x ;x ;:::;x n 이다. P (x) =x n + a x n + + a n Q(x) =x n + b x n + + b n a + a 3 + a 5 + 과 a + a 4 + a 6 + 이모두실수라면, b + b + + b n 도실수임을보여라. ( 유고슬라비아 979- 고 3-) 증명 P (x) =(x x ) (x x n ), Q(x) =(x x ) (x x n). P () + P ( ) 과 P () P ( ) 이실수이므로 P (), P ( ) 은실수이고, P ()P ( ) = ( ) n Q() 도실수.

95 . 대수중급문제 95. a; b > 0 이고 a; A ;A ;b 가등차수열, a; G ;G ;b 가등비수열을이룬다. 다음을보여라. ( 캐나다 979-) A A G G 증명 A + A = a + b, G G = ab 이다. µ A + A µ A A µ a + b µ a b A A = = ab = G G 따라서, 성립한다. 주이것은 G, G 와는상관없이 A A ab 를증명하라는문제와같으며, a; A ;A ;b가등차수열이된다는것보다일반적으로 A + A = a + b; ja A j ja bj 이기만하면위의증명이그대로성립한다. 비슷하게 A + A G + G 를증명하라는문제도생각할수있고, 이것은 A, A 와는상관없이 a + b G + G 임을증명하라는문제이기도하고, 다음과같이증명할수있다. (a + b) =(a b) +ab (G G ) +G G =(G + G ) 사실은 A G, A G 이기도하다. 이것은가중치산술 - 기하평균부등식그자체이다.. 모든세변수다항식 W 에대하여, 다음을만족하는두다항식 U, V 가존재함을보여라. ( 폴란드 차 -4) W (x; y; z) =U(x; y; z)+v (x; y; z); U(x; y; z) =U(y; x; z); V (x; y; z) = V (x; z; y) 증명 U(x; y; z) =W (x; y; z)+v (x; z; y) =W (x; y; z)+w (x; z; y) U(z; x; y) = = W (x; y; z)+ W (x; z; y) W (z;x; y) W (z; y; x)+w (y; x; z) U(x; y; z) 로풀림. V 도비슷하게하거나아니면그냥위에서구한 U에의해V = W U 로얻거나. 물론이렇게구한 V 가문제의조건을만족함은확인해야함. 3. P (x) 와 Q(x) 는실수 x 에대해항등식 P (Q(x)) Q(P (x)) 를만족하는다항식이다. P (x) =Q(x) 가실근을갖지않는다면 P (P (x)) = Q(Q(x)) 도실근을갖지않음을보여라. ( 캐나다 98-4) 증명 P (x) Q(x) =0이될수없으므로, 중간값정리에의해 P (x) Q(x) 는항상양수이거나항상음수이어야한다. 일반성을잃지않고항상 P (x) >Q(x) 라고하자. P (P (x)) >Q(P (x)) = P (Q(x)) >Q(Q(x)) 도항상성립하므로, P (P (x)) = Q(Q(x)) 도실근을갖지않는다. 4. 수열 (x n ) nn 은다음과같이정의된다 : x =,x =3 이고, 임의의 m 에대해 x m+ = x m + x m x m+ = x m+ +x m 수열 x n 을 n 에대한함수로나타내어라. ( 오스트리아 983-4) 풀이첨자가모두홀수 (or 짝수 ) 만있는식으로바꿔놓고홀수번째항들의일반항을먼저구하면. }

96 96 대수 5. a <a <a 3 <a 4 는주어진자연수들이다. 다음의연립방정식이음이아닌해 (x ;x ;x 3 ;x 4 ) 를갖도록하는실수상수 c 를모두찾아라. ( 오폴 983-5) x + x + x 3 + x 4 = a x + a x + a 3 x 3 + a 4 x 4 = c a x + a x + a 3 x 3 + a 4 x 4 = c 풀이세식의좌변을차례로 A, B, C 라하면코시부등식에서 AC B. 그런데이것은 c (c) 으로등호가성립할때임. x i 6=0,x j 6=0인서로다른두첨자i, j가존재한다면코시부등식의등호조건에서 a i x i=x i = a j x j=x j, 즉 a i = a j 가되어모순. 따라서, x i 들중에적어도셋은 0이고, 그럼첫번째식에의해나머지하나는 이다. 즉,(x ;x ;x 3 ;x 4 )=(; 0; 0; 0), (0,,0,0), (0,0,,0), (0,0,0,) 등만가능하고각각 c = a ;a ;a 3 ;a 4 답 } 6. a 5 + b 5 과 x 5 + y 5 을만족하는음이아닌실수 a, b, x, y에대해, a x 3 + b y 3 임을증명하여라. ( 오폴 983-) 증명 a x 3 + b y 3 a5 +a 5 +x 5 +x 5 +x 5 + b5 +b 5 +y 5 +y 5 +y =. AM-GM 5 7. a < 5b 이면다음방정식의근이모두실근일수없음을증명하여라. ( 미국 983-) x 5 + ax 4 + bx 3 + cx + dx + e = 개의실수가주어져있다. 0 x y +xy p 3 을만족하는두수 x, y 가그들중에있음을보여라. ( 캐나다 984-5) 증명 7개의실수를 tan i ( ¼ < ;:::; 7 < ¼ ) 라하자. 구간 ( ¼ ; ¼ ) 를 6등분하면각구간은길이가 ¼ 6 가되고, 비둘기집의원리에의해같은구간에속하는두 i, j 가있다. i > j 로순서를잡으면, tangent는증가함수이므로, 0=tan0 tan( i j )= tan i tan j +tan i tan j tan ¼ 6 = p 3 이성립한다. 9. 다항식 x 4 +(a +)x 3 +(a ) x + bx +4 가다음성질을만족하는최고차항의계수가 인두이차다항식 Á(x) 와 Ã(x) 의곱으로인수분해되도록하는 a, b 의값을모두구하여라 : 방정식 Ã(x) =0 은서로다른두근, 를가지며 ( 실근일필요는없다 ), Á( ) =, Á( ) = 를만족한다. ( 호주 984-3) 풀이 Á(x) =(x )(x )+ + x + = p, = q 로치환하고대입하여계수비교로풀면그냥막노동. } 30. 방정식 x 4 8x 3 + kx +00x 984 = 0 의네근중어느두근의곱이 3 이다. k 의값을구하여라. ( 미국 984-) 풀이네근을a; b; c; d라하고ab = 3 라하면, 근과계수와의관계를정리하여 (a+b)+(c+d) = 8, (a + b)(c + d) 3 + cd = k, 3(c + d)+cd(a + b) = 00, 3cd = 984. 여기서 cd =6이고이제첫번째식과세번째식에서 a + b 와 c + d 에대해연립하여풀면됨. a + b =4,c + d =4가되고 k =86 답 }

97 . 대수중급문제 a + b + c =0를만족하는,0이아닌서로다른수a, b, c에대해다음을증명하여라. µ b c + c a + a b µ a a b c b c + b c a + c =9 ( 오폴 985-) a b 증명그냥전개. 왼쪽항은통분하면 (a 대신 b를대입하면다소거되므로 ) 분자에 (a b)(b c)(c a) 가 a(a b)(a c)+b(b c)(b a)+c(c a)(c b) 나와서오른쪽항과곱해주면가되고, a 3 + b 3 + c 3 abc 공식과 a + b + c =0등이용해서정리하면분자에 9abc만남음. 3. <x < 이고, n =; ;::: 일때 x n+ =+x n x n 이라정의하자. n 3 일때 jx n p j < n 임을증명하여라. ( 캐나다 985-5) 증명 f(x) =+x x = (x ) + 3 이 <x 에서감소함수임을이용하면, <x < =) <x < 3 =) 8 <x 3 < 3 따라서, jx 3 p j < 8 이성립한다. yn = xn p 로치환하자. 그럼 y n+ =+y n (y n + p ) = y n( p y n ) n = k( 3) 일때 jy k j < k 이성립한다고가정하면, j p y k j < j p 8 j < 이므로, jy k+ j = jy kjj p y k j < k+ 로 n = k + 일때도성립함을알수있다. 따라서, 수학적귀납법으로문제가증명되었다. 33. 다음연립방정식을만족하는실수해를모두찾아라. ( 오폴 986-5) x + y + u + v =4 xu + yv + xv + yu =0 xyu + yuv + uvx + vxy = xyuv = 풀이 x + y = p, xy = q, u + v = a, uv = b 로치환. 그럼두번째식은pa =0. p, q와 a, b는서로대칭적이므로 WLOG p =0. 이제준식은 q + a b =4,qa =, qb = 이됨. 뒷두식에서 a =b, q = b. 이둘을첫식에대입하면, b +4b b =4, 즉 b 3 b b +=0 이고인수분해하면 (b )(b ) = 0, 즉 b = ;. 각각풀면됨. } 34. 자연수 n> 과실수 a>0 가주어져있다. 합이 a 인 n 개의음의아닌수 x i 들의모든가능한집합에대해, 다음식의최대값을구하여라. ( 호주 986-) n X x i x i+ i= 풀이 (x ;x ;x 3 ;:::;x n )=( a ; a a ; 0;:::;0) 일때이식의값은 4 이고, 이것보다클수없음을보이자. x x +x x 3 + +x n x n (x +x 3 +x 5 + )(x +x 4 +x 6 + ) ( (x +x 3 +x 5 + )+(x +x 4 +x 6 + ) ) =.( 마지막부등식은 GM-AM) } a 4 풀이 최대인항을 x i 라하자. x x ;x x 3 ;:::;x i x i 는각각x x i ;x x i ;:::;x i x i 이하이고, x i x i+ ;x i+ x i+ ;:::;x n x n 은각각 x i x i+ ;x i x i+ ;:::;x i x n 이하이므로,( 준식 ) x i (a x i ) ( x i+(a x i ) ) 이된다. }

98 98 대수 35. a; b; a ;a 3 ;:::;a n 는실수들이고 ab 6= 0이다. 그리고, 방정식 ax n ax n + a x n + a 3 x n a n x n bx + b =0 의해는모두실수이고또한양수이다. 이방정식의모든해가서로같음을증명하여라. ( 호주 986-6) 증명근과계수와의관계를이용하면 x + + x n =, + + x x n = n 임을알아낼수있음. 코시부등식 or AM-HM 부등식의등호조건. 36. 함수 f : R! R ( 단, R은실수전체의집합 ) 이임의의두실수x, y에대하여 f(x + y)f(x y) ff(x)g ff(y)g 을만족시킬때, 다음을증명하여라. () f( x) = f(x) () f(x + y)f(x y) =ff(x)g ff(y)g ( 한국 987-9) 증명 () x = y =0이라하면 ff(0)g 5 ff(0)g ff(0)g =0 f(0) 은실수이므로 ff(0)g = 0 ) f(0) = 0 () x =0 이라하면 f(y)f( y) 5 ff(y)g ) f(y)ff(y)+f( y)g 5 0 () () 가임의의 y에대해서성립하므로 y대신에 y를대입하면 f( y)ff( y)+f(y)g 5 0 (3) ()+(3) 에서 ff(y)+f( y)g 5 0 f(y)+f( y) 는실수이므로 f(y)+f( y) =0 ) f( y) = f(y) 따라서임의의 x에대해서 f( x) = f(x) () f(x + y)f(x y) 5 ff(x)g ff(y)g 에서 x; y 를바꾸면 f(y + x)f(y x) 5 ff(y)g ff(x)g () 에서 f(x + y)f(x y) 5 ff(y)g ff(x)g ) f(x + y)f(x y) = ff(x)g ff(y)g 따라서 f(x + y)f(x y) =ff(x)g ff(y)g 37. n이 n 3 인자연수일때, 다음부등식을증명하여라. ( 한국 987-0) n n+ > (n +) n 증명 (i) n =3 일때 : n n+ =3 4 = 8, (n +) n =4 3 =64 ) n n+ > (n +) n 이성립한다. (ii) n = k 일때 k k+ > (k +) k 이성립한다고가정하면 (iii) (k +) k+ = = = (k +)k+ k k+ k k+ (k +)k+ > k k+ (k +) k = µ (k +) k+ µ k k+ +k + = k k µ k ++ k+ > (k +) k+ k (k +)k+ k k+ ) (k +) k+ > (k +) k+ 따라서 n = k +때도성립한다. (i) (ii) (iii) 에서수학적귀납법에의하여 n = 3인모든 n에대하여 n n+ > (n +) n 이성립한다.

99 . 대수중급문제 99 별해 (n +) n n n+ = µ + n n n = µ +n n(n ) n(n ) (n k +) n n! n k! n! + + nk n! n n ( 이항전개 ) à = ++ n + + ( k ) ( n n ) n! k! + ( n ) ( n n )! n! < µ ++ n! + + k! + n! < µ ++ n k n à = +! Ã! n n < + = 3 n n 따라서 n = 3 이면 (n +)n n n+ < 3 n 5 ) n n+ > (n +) n 별해 n n+ > (n +) n, (n +)logn>nlog(n +) 이므로 f(x) =(x +)logx x log(x +) 이라놓고 x = 3 인실수 x 에대해서 f(x) > 0 을증명한다. ( 단,log 는자연로그이다.) f 0 (x) =logx log(x +)+ x + x x x + =logx log(x +)+ x + x+ f 00 (x) = x x + x (x +) = x + x + x (x +) < 0 따라서 f 0 (x) 는단조감소함수이고 이므로 x = 3 일때 f 0 (x) > 0 따라서 f(x) 는증가함수이다. µ (* x + x += x > 0) f 0 (3) = log > 0 ( 이것은안밝혀도된다.) µ log lim x! f 0 (x) = lim x! x x + + x + =0 x + f(3) = 4 log 3 3log4=log > log = 0 따라서 x = 3 일때 f(x) > 0 특히 x = n = 3 인정수일때 (n +)logn n log(n +)> 0

100 00 대수 ) log n (n+) > log(n +) n ) n n+ > (n +) n log n n > log(n +) n + ) (n +)logn>nlog(n +) 로하여f(x) = log x 가 x = 3에서단조감소함수임을밝혀도된다. x 38. 방정식 x(x )(3x 7) = 의세근을 r, s, t 라하자. r, s, t 가양의실수임을보이고 arctan r+arctans+ arctan t 를구하여라. ( 남미 987-5) 풀이 x =0; ; ; 3 에서부호를비교해보면세근은각각 0 <r<, <s<, <t<3의 범위에있다. 근과계수에의해 r + s + t = 3 4, rs + st + tr = 3 3, rst =. 3 모든실수범위에서 tan a+tan b+tan c tan a tan b tan c tan arctan x = x 이고 tan 합각공식에서 tan(a + b + c) = tan a tan b tan b tan c tan c tan a 이므로, tan(arctan r +arctans +arctant) = =. 그런데 0 < arctan r +arctans +arctant< 3 ¼ 이 므로 arctan r +arctans +arctant = 4 3 ¼. } 39. 임의의실수 ;:::; n 에대해다음을증명하여라. ( 루마니아TST 987-C) nx nx ij cos( i j ) 0 i= j= 증명 ( 서준영 ) 코사인합공식을이용하면 XX ij cos(ai a j )= XX (ij cos a i cos a j + ij sin a i sin a j ) =( X i cos a i )( X j cos a j )+( X i sin a i )( X j sin a j ) =( X i cos a i ) +( X i sin a i ) a 는실수이고, 함수 f : R! R 은 f(0) = 과임의의실수 x, y 에대해 f(x + y) =f(x)f(a y)+f(y)f(a x) 를만족한다. f는상수함수임을증명하여라. ( 발칸 987-) 증명 (x; y) =(a; 0) 대입하면 f(a) = = f(0). y =0 대입하면 f(a x) =f(x). y = a x 대입하면 f(x) = 4. y = x 대입하면 f(x) 0. 따라서, f(x) = 의상수함수. 4. 다음연립방정식을풀어라. ( 오스트리아 ) x (x ) = x x (x ) = x 3. x n (x n ) = x 풀이다더하면 P (x i x i +)=0, 즉 P (x i ) =0. 따라서 x = = x n =. }

101 . 대수중급문제 0 4. 다음연립방정식을풀어라. ( 오스트리아 ) x + y + xy x + y =; p x + y = x y 풀이우선 x + y>0. x + y> 이어도첫식에서모순이생기고, x + y< 이어도첫식에서모순이생김. 즉, x + y =뿐. 그담엔풀면됨. } 43. 자연수 n에대해다음을보여라. (Towns 987봄 SA4) v s u r t q 3 4 (n ) p n<3 증명 좌변을지수꼴로나타낸후 GM-AM 부등식으로풀어냄. v s u r q 증명 t L< (n ) p +n(n +)=3 44. n 은자연수이다. z n+ z n =0 이 jzj = 인복소근을가질필요충분조건은 n + 가 6 의배수일때임을증명하여라. ( 중국 987-) 증명 ( 김대현 ) [] jzj = 일때, z =cosµ + i sin µ 라할수있겠다. 따라서 n + 두번째식으로부터 첫번째식에대입하면 z n+ z n =[cos(n +)µ cos nµ]+i[sin(n +)µ sin nµ] = ( cos(n +)µ cos nµ = sin(n +)µ sin bµ =0 sin 8 n + >< sin µ sin, µ = n + >: cos µ sin µ =0 µ =(p + )¼(* sin µ 는공통인수이므로 0이아니다.) sin(p + p + )¼ sin 4n + ¼ = p + 4n + ¼ = p +, 4n + ¼ =(m + 6 ; 5 6 )¼, 6(p 4nm m +)=4n +or5(4n +) p 와 m 은기우성이다르므로 ( 하나가홀수면다른하나는짝수이므로 ) (p 4nm m +) 3( (mod 4)) 따라서우변이 6의배수가되게하는 n =6t +; 4 중 n =6t +4만성립한다. [] n =6k +4일때, z =cos 3 ¼ + i sin ¼라고하면 [] 의과정에의해방정식이성립한다. 따라서 3 jzj =인근을갖는다. 증명 ( 장도한 ) n =6k +4,k는음이아닌정수일때, z = e ¼ 3 i 라두면jzj =이고, z 3 =cos¼ + i sin ¼ = 이므로, z n+ z n =z 6k+5 z 6k+4 =(z 3 ) k+ z (z 3 ) k+ z = z + z =0

102 0 대수 이다. 역으로, 복소방정식 z n+ z n =0,jzj = 의근을 z 0 = e iµ ; ¼ <µ ¼ 라두면, =jz0 n (z 0 )j = je inµj je iµ j = je iµ j q = (cos µ ) +sin µ = p cosµ 이다. ) cos µ = 즉, µ = ¼ 3 이다. ) z 0 =e ¼i 3 = i p 3 ¼i n¼i ¼i (n +)¼i 3 이므로,=z0 n(z 0 )e 3 e 3 = e 3 이 = e 다. (n +)¼i ) =m¼; m은정수이다. 3 ) n +는 6의배수이다. 45. 다음을증명하여라. ( 통신강좌 988-B4) < p + p + p 3+3 p p p 987 < 풀이위식은다음과같이나타낼수있다. 그런데, 987 X < X k p k ++(k +) p k < k= k p k ++(k +) p k k= 987 X = k p k ++(k +) p k p k + (k +) p k k k p k + (k +) p k k= = = = 987 X k p k + (k +) p k k (k +) (k +) k k= 987 X (k +) p k k p k + k(k +) = 987 X k= k= µ p µ p + p = p 988 µ pk p k + µ p p =936; 45 =05 이므로 44 < p 988 < 45, 45 < p < , < p < ) < p + p + p 3+3 p p p 987 < } 46. 임의의자연수 0 <a <a < <a n 에대해 (a + a + + a n) a 3 + a a 3 n 임을증명하여라. ( 루마니아 TST 988-A4)

103 . 대수중급문제 03 증명 ( 이종민 ) n = 일때 a a3 성립. 단, 등호는 a =. (a + a + + a n) a 3 + a3 + + a3 n 이성립한다고가정. a + a + + a n = h 라하자. h a 3 + a3 + + a3 n 성립. a(a + a + + a na n+ ) =(h + a n+ ) = h + a n+ (h + a n+ ) nx a 3 k + a n+(h + a n+ ) 이므로 h + a n+ a n+ 이성립함을보이면끝난다. 식을정리하면 h an+(a n+ ). 우변은 부터 a n+ 의총합이고좌변은 a + a + :::+ a n 이고 a n a n+ 이므로 h an+(a n+ ) ( 등호는 a =;a =;a 3 =3;::: ;a n = n) 는성립. 따라서 h + a n+ a n+ 이성립. nx a 3 k + a n+(h + a n+ ) k= nx k= k= n+ a 3 k + X a3 n+ = a 3 k k= (a +a +:::+a n +a n+ ) a 3 +a3 +:::+a3 n+ 이성립.(a +a +:::+a n +a n ) a 3 +a3 +:::+a3 n 성립.( 등호는 a =;a =3;a 3 =3;::: ;a n = n) 이아닌실수들의수열 x ;x ;x 3 ;::: 은다음의점화식을만족한다. x n = x n x n x n x n (n 3) 이수열의무한히많은항이정수가되도록하는 (x ;x ) 를모두구하여라. ( 아일랜드 988-8) 풀이점화식의양변에역수를취하고 yn := x n 로치환하면y n =y n y n, 즉 y n y n = y n y n = = y y =: c 의상수가된다. 그럼 y n 은공차가c인등차수열이므로, c 6= 0이라면 y n 은 n에대한일차함수가되어충분히큰 M에대해n>M 이면 jy n j > 이된다. 그럼 0 6= jx n j < 이므로정수인항은 n M 일때의유한개밖에없어서모순. 따라서, c =0이어야한다. 즉, y = y, 다시말해 x = x 이다. 그리고이때, y n = y, 즉 x n = x 의상수함수가되므로문제의조건에부합한다. } 48. k는주어진양의실수이다. 수열 x 0 ;x ;x ;::: 이다음의점화식을만족한다. x n+ = x n( kx n) (n 0) 이수열이 k 로수렴하도록하는 x 0의값을모두구하여라. ( 아일랜드 988-) 풀이 y n := x n k 로치환하자 (0으로수렴하는경우가더다루기쉬울테니까 ). 대입하면 y n++ k = (y n + k )( k(y n + k )) 이고정리하면 y n+ = kyn. 따라서, ky n =( ky n ) = =( ky 0 ) n. 이식으로부터, y n! 0 () ky n! 0 () j ky 0 j < 임을알수있다. 즉, jy 0 j < k 이고다시말하면 0 <x 0 < 답 } k 49. 모든 x; y R 에대해다음을만족하는단조증가함수 f : R! R 을모두구하여라. f(f(x)+y) =f(x + y)+f(0) ( 오폴 988-4) 풀이 f(x) =f(x + a) 인 x와 a(> 0) 가있다면 x를대입했을때와 x + a를대입했을때좌변이같으므로우변도 f(x + y) =f(x + y + a). 이것이임의의 y에대해성립하고 f가단조증가이므로그럼 f는상수함수. 그경우는영함수 f(x) =0밖에없음을바로알수있다. 이제 f가단사일때만보면되므로, f는강증가. x에 0을대입한식 f(f(0) + y) =f(y)+f(0) 과 y에 0을대입한식 f(f(x)) = f(x)+f(0) 을비교하면 f(f(x)) = f(f(0) + x). 단사이므로 f(x) =x + f(0). 그리고, 이런꼴의함수 f(x) =x + c 는문제의조건식을항상만족. 답 0, x + c }

104 04 대수 50. 다음을만족하는최고차항의계수가 인정계수다항식 P (x), Q(x) 를모두구하여라. Q(0) = 0; P(Q(x)) = (x )(x ) (x 5) ( 루마니아 TST 989-A) 풀이 ( 임준혁 ) P 차수 Q차수 =5 Q차수가,3,5,5일때를따지면 일때, Q(x) =x이고 P (x) =(x ) (x 5) 3일때, Q(x) =x 3 + ax + bx로놓자. 5,4,3차항은 (x 3 + ax + bx) 5 항에만연관되어있다. 이항정리를이용하여연립하면 5a = ( ) 4차항여기서 a = 4 0a +5b =3차항계수 = ( ) 5 =6580이고, 이둘을풀면b를알수있고, a = 4;b =64 그리고 P (x) =0의해는Q(); ;Q(5) 가전부이므로 Q를알수있으니P (x) =0의해를알수있다. 그런데서로다른해가6개이상이므로모순이다. Q() = 4;Q() = 40;Q(3) = 303;Q(4) = 336;Q(5) = 345;Q() = 3 Q의차수가5일때는 (x 5 + ax 4 + bx 3 + cx + dx) 3 + 에서 3차항 :3a +3b = ( ) 5 =6580인데저것은 3의배수가아니므 로모순. 따라서없다. Q의차수가5일때는Q(x) =(x ) (x 5) + 5! P (x) =x 5! } 풀이 ( 민원준 ) Q 의상수항은 0 이고 deg Q = m; deg P = n 이라고하면 nm =5 가된다. Q(n) = a n 이라고하자. ) m =;n=5 그러면 P (x) =(x a )(x a ) (x a 5 ) 가될것이다. (P (x) =0 의근이 Q();Q(); ;Q(5) 이므로 ) Q(x) =x 라고하면성립함을알수있다. 그러므로 P (x) =(x )(x ) (x 5);Q(x) =x ) m =3;n=5 인경우 P (x) 는 a» a 5 를근으로갖기때문에대수학의기본정리인 n 차식의근은 n 개이다에의해 a» a 5 중서로같은수들이존재한다. 그런데 3 차식의그래프는이런모양이므로같을수있는수의최대개수는 3 개이다. 그런데 개만같다거나한다면 fa ; ;a 5 g 의원소의개수가 6 개이상이되므로 n =5 라는조건에모순이다. 즉 3 개원소씩이루어진 5 쌍이존재하여각쌍안의세수는모두같다. 그림에서세모로표현해놓은 ` 미분하면 0 이되는점 ' 이 개나오는데그 점을기준으로왼쪽을 번영역, 가운데를 번영역, 오른쪽을 3 번영역이라고하자. a ;a ; ;a 5 에서나오는 5 개의값을 b ;b ; ;b 5 라고하면 b 의값을갖는세개의 a i 가,, 3 번영역에각각하나씩있어야한다. b ;b 3 ;b 4 ;b 5 도마찬가지. 그러므로 Q();Q(); ;Q(5) 는 번영역, Q(6); ;Q(0) 은 번영역, Q(); ;Q(5) 는 3 번영역에있어야한다. Q()» Q(5) 값중 Q() 값이가장작고, Q(6)» Q(0) 의값중 Q(0) 의값이가장작으며, Q()» Q(5) 의값중 Q() 의값이가장작으므로 Q() = Q(0) = Q() 이된다. Q(x) =x 3 + bx + cx 라고하면 b; c 값을구할수있다. b =;c= 3 이되므로 Q() = Q(9) = Q() 가되어야한다는조건에모순. 즉만족하는 Q 없다. 3) m =5;n=3 인경우 번경우와마찬가지방법으로 5 차식의그래프는아래와같은모양이므로아무리많아야 a» a 5 중같은수의개수는 5 개이고,4 개이하인것이있다면 n =3 에모순이므로 5 개수씩 3 쌍이존재하여각쌍안의수는모두같아야한다. 이세수를 b ;b ;b 3 라고한다면 b 인값을갖는 5 개 a i 들은,, 3, 4, 5 번각영역안에하나씩존재해야한다. 고로 Q()» Q(3) 은 번영역,, Q(3)» Q(5) 는 5 번영역이

105 . 대수중급문제 05 다. Q(x) =x 5 + bx 4 + cx 3 + dx + ex 라고하면위의식에서 Q() = Q(6) = Q(7) = Q() = Q(3) 이다. 그러므로 b; c; d; e 를구할수있다. b = 39;c=56;d = 360;a=9630 이된다. 그러면 Q() = Q(5) 라는조건에모순이므로만족하는 P; Q 없다. 4) m =5;n= 인경우 P (x) =x + c 고로 P (Q(x)) = Q(x)+c 즉 Q(x) =(x )(x ) (x 5) c 가된다. Q(0) = 0 이므로 c = 5! 이되고 P (x) = x 5!;Q(x) = (x )(x ) (x 5) + 5! 이된다. } 5. x 0 = t, x n+ =x n 로정의된수열이 n N 일때항상x n =이되는자연수 N을갖는다고한다. 이런 t가 t 의범위에무한히많음을증명하여라. ( 북유럽 989-3) q 증명 x 0 = t k 가 x k 에서처음으로 이나타나는 t 6= 라고하면, t k+ = tk + ( ; ). 증명 cos 배각공식 5. 실수 가주어져있고, 함수 f 가 f0; ; ;:::g f0; ; ;:::g 에서모든 m; n > 0 에대해다음과같이정의되어있다. f(0; 0) = ; f(m; 0) = f(0;m)=0; f(m; n) = f(m; n ) + ( )f(m ;n ) jf(m; n)j < 989 가모든자연수 m; n 0 에대해성립하도록하는 를모두찾아라. ( 이탈리아 989-6) 풀이 f(;n)= n ( ), f(n; n) =( ) n 이므로, 발산하지않으려면 j j; j j. 즉, 0. 이범위에서는항상 0 f 로성립. } 53. 각각의자연수 n 에대해, 다음과같이두자. S n = n T n = S + S + S S n U n = T + T 3 + T T n n + T 988 = as 989 b 와 U 988 = cs 989 d 를만족하는정수 0 <a;b;c;d< 를구하고, 그것을증명하여라. ( 미국 989-) T 풀이 n 귀납적으로다음을확인할수있음. Tn = (n +)S n+ (n +), n + = S n+, U n =(n +)S n+ (n +) n =(n +)S n+ (n +). } 54. x ;x ;:::;x n 은양의실수들이고, S = x + x + + x n 이라하자. ( + x )( + x ) ( + x n ) +S + S! + S3 Sn + + 3! n! 임을보여라. (APMO 989-)

106 06 대수 증명 양변에서차수가같은항만비교해보자. 양변에서 r차식만골라쓰면 X x i x i x ir Sr r! i <i < <i r n 이것만보이면충분하다. 양변에 r! 을곱하면 r! X x i x i x ir (x + x + + x n) r 이고, i ;:::;i r 의순서를섞는경우의수가 r! 이므로, 좌변은우변을전개할때나오는 x i x ir 중에서 i ;:::;i r 이모두다를때만을모은것이된다. 우변에남는항은모두양수이므로부등식이성립함을알수있다. 별해 G.M A.M 부등식으로 µ ( + x )+ +(+x n) 좌변 n nx ³ n µ S i nx S i = = i n i! i=0 i=0 nx S i = 우변 i! i=0 n µ = + S n n n! (n i)! n i 이된다. 마지막부등식은 n! (n i)! n i = (n i+) (n )n n n n 임에서나온것이다. S>0이면마지막부등식은 n =일때만성립하므로, 등호가성립할조건은 S =0또는 n =일때이다. 별해 n에대한수학적귀납법을이용하기로하고명확히하기위해 S 대신 S n 으로표현하자. n = 일때는등호가성립하므로자명. n일때성립한다고가정하면 µ ( + x ) ( + x n )( + x n+ ) +S n + S n + S3 n + + Sn n ( + x n+ )! 3! n! 이므로 n +일때도성립함을보이려면 µ +S n + S n + + Sn n ( + x n+ ) +S n+ + S n+ + + S n+! n!! (n +)! 을보이면충분하다. 양변에서차수 r =; ;:::;n+ 에대해 r 차항끼리비교하는것을생각하자. 그럼 S r n r! + Sr n (r )! x n+ Sr n+ r! 임을보이면충분하다. 이것은양변에 r! 을곱하면, S n+ = S n + x n+ 이므로 Sn r + rsn r x n+ (S n + x n+ ) r rx ³ r 이된다. 우변을이항정리로전개한식 Sn i r i x i n+ 에서좌변은 i =0; 일때만쓴것이고, 모든 i=0 항이다양수이고 r 이므로이부등식은성립한다. 따라서, n + 일때도성립하고, 수학적귀납법으로증명이되었다. 55. a + b =4인음아닌실수 a, b에대하여부등식찾아라. 증명조건식에서 AM-GM 으로 ab. 즉 a + b + ab ab a + b + p 이성립함을보이고등호조건을 p ab + ab ( 오스트리아 ) = p ab + ab p+.

107 . 대수중급문제 실수 a ;a ;:::;a n 에대해다음을증명하여라. ( 북유럽 990-) q 3 a 3 + a3 + + a3 n q a + a + + a n 증명 n =일땐자명. n =일때따로증명해주고, n =일때를이용하여수학적귀납법. 등호조건은많아야하나만제외하고모두 0일때. à 57. a> 3 4 일때, a + + a +3 r! à 4a +3 3 a + + a +3 r! 4a 은 a의값에상관없이일 정함을보여라. ( 스페인 990-4) 증명준식을 x+y = t 라하면x 3 +y 3 = a+, xy = a 3 임을약간의계산을통해확인할수있다. 그럼 (x+y) 3 =(x 3 +y 3 )+3xy(x+y) 임에서 t 3 +at a =0. 인수분해하면 (t )(t +t+(a+)) = 0. a> 3 4 이면 t + t +(a +)> 0 이므로 t =. 58. 다음연립방정식의자연수해를모두구하여라. ( 오폴 990-4) x 4 +4x x +=y 4 x 4 +4x x 3 +=y 4. x 4 n +4x n x +=y 4 n 풀이좌변을살폈을때 y 4 i >x 4 i 이므로 y i x i +, 즉 yi 4 (x i +) 4. 이부등식을변변모두더하면 4 P x i x i+ 4 P x 3 i +6P x i +4P x i. 그런데,AM-GM에서 4x 3 i +4x i 8x i 이므로 4 P x i x i+ 4 P x i. 이것은 P (x i x i+ ) 0 으로정리되므로 x = x = = x n = a 일때뿐이고, 등호성립조건에서 a 3 = a 이므로 a =. } 59. a ;a ;:::;a n 을양의실수라하고, a ;a ;:::;a n 에서서로다른 k개를골라곱한것들의합을 S k 라한다. k =; ;:::;n 에대하여다음을증명하여라. (APMO 990-) ³ n a S k S n k a a n k 증명 A A = a a a n 이라할때, S k 의한항a i a i a ik 는 S n k 의한항 a i a i a ik 와짝을이룬다 ( 일대일대응 ). 즉, S k = x + x + + x m 의항으로되어있다고하면 S n k = A + A + + A x x x m 이다. 그럼코시-슈바르츠부등식으로 µ A S k S n k =(x + x + + x m ) + A + + A x x x m à s s s p A x + p A x + + p! A x m x x x m =(m p A ) = m A 가된다. S k 의항의개수는 m = n k 이므로증명이되었다. 주 A.M G.M 으로도바로풀수있다. 60. 모든자연수 n에대해다음이성립함을증명하여라. (Towns 990봄 JO) µ µ + n + + µ + + n n + µ + n n µ =n n

108 08 대수 증명 귀납법. n = 일때 = 로자명. n 일때 Ln = R n 이성립하면 n + 일때 L n+ L n =(n +) (n +) + n + = n + + n + ½µ µ + n µ n n = +n n + = n + = R n+ R n + + n µ ¾ + + n 이므로 L n+ = R n+ 이성립. 6. a = 이고 b = 일때 ja bj < 임을증명하여라. 99! 00! (Towns 990 가을 JA) 증명 a n = n +, b n = n + (n +)+ + (n +) ja bj = a b = ja b j a b = a b a 3 b = ja 3 b 3 j 3 a b a 3 b 3 = = ja 99 b 99 j a b a 3 b 3 a 99 b 99 < (99!) j 00 j 로정의하자. 그럼 임을확인할수있다. 마지막부등식에서 a n ;b n >n 임을이용했다 ( 단, a 99 =99 만등호성립 ). 6. 음이아닌실수 x, y, z 가 xyz x y z = 를만족할때 x + y + z 의최소값을구하여라. ( 통신강좌 99--) 풀이 AM-GM에서 xyz =x + y + z 3(xyz) 3 이므로 (xyz +) (xyz 8) 0. x; y; z는모두 음이아닌실수이므로 (xyz +) > 0이되어xyz 8이된다. 따라서,AM-GM으로 x + y + z 3(xyz) 3. x = y = z =일때실제로등호가성립하므로, x + y + z 의최소값은 답 } 63. 임의의양수 x, y, z 에대해다음을증명하여라. ( 통신강좌 99--3) x + y + z (x + y)(y + z)(z + x) (xyz) 풀이 8 (x + y)(y + z)(z + x) = f(x + y + z)(xy + yz + zx) xyzg 8

109 . 대수중급문제 09 이므로 8 (x + y)(y + z)(z + x) 3 (x + y + z)(xyz) 3 8 (x + y + z) 3(xyz) 3 8 xyz 3 (x + y + z)(xyz) 3 (By Arithmetic-Mean inequality) = 4 (x + y + z)(xyz) 3 8 xyz = 8 0 (By Arithmetic-Mean inequality) ½ x + y + z (xy) 3 3 ¾ (xyz) 3 이성립한다. 즉 이성립한다. (x + y)(y + z)(z + x) 8 3 (x + y + z)(xyz) 3 } 64. Y n= n 3 n 3 의값을구하여라. ( 통신강좌 99--7) + 풀이 Y n= n 3 n 3 + = Y n= µ = lim m! (n )(n + n +) (n +)(n n +) = lim m! m(m +) m + m + 3 my n= n n + (n +) (n +)+ n n + = lim m! 3 + m + m + m = 3 } 65. T =,T n+ = T n T n + 일때다음을보여라. X n= T n =. ( 통신강좌 99--9) 풀이 이므로, T n+ = T n (T n ) = + T n T n µ T n+ = T n T n T T T 이다. ) nx k= = T k T n+ 그리고, T n+ T n = T n T n +=(T n ) 0. 즉, T n+ T n 이므로, 이다. 따라서, ) T n T =; T n+ = T n T n T T + n +: 0 T n n ) lim n! T n =0 ) X n= T n = lim n! T n+ = }

110 0 대수 66. a, b, c, d 를 0 이아닌임의의실수라하자. 다항식 f(x) =x 6 + ax 3 + bx + cx + d 의모든해가실수가될수는없음을증명하여라. ( 통신강좌 99--3) 풀이 ( 서울과학고박종원 ) f(x) =x 6 + ax 3 + bx + cx + d =0 의근들을 x ;x ; ;x 6 라고하자. 근과계수와의관계에서, 이다. 따라서, X 6Y x + x + + x 6 =0; x i x j =0; x i = d à i6=j i= 6X! 6X 6X 0= x i = x i +X x i x j = x i i= i= i6=j i= 이성립한다. 그러므로, x i ( i 6) 가모두실수라면 x = x = = x 6 =0이어야하는데이것은 6Y x i = d 6= 0에모순이다. } i= 67. 0이아닌실수x 에대하여 x n+ = µ x n + ; n x n 로정의된수열 fx n g의수렴여부를판정하고, 수렴한다면그극한을구하여라. ( 통신강좌 99--8) 풀이 x < 0 이면 y n = x n 으로치환했을때상황이똑같고부호만바뀐다. x > 0 일때를풀자. AM-GM에서 x + x 이므로귀납적으로 x n (n ) 이다. 또한 x n+ x n = µ x n + 0(* x n ) x n 따라서, x n 은 ( 초항을제외하면 )[;x ] 에의해bounded이고단조감소한다. 즉, 수열 fx ng은수렴한다. lim x n = 라고하면, = µ + 이므로 =이다. 따라서, 초항이양이면 로수렴, 음 n! 이면 로수렴한다. } 68. 구간 [0; ] 에서정의된함수 f 가다음을만족시킨다고한다 : f(0) = f(), 그리고 0 x ;x 인임의의 x 6= x 에대해 jf(x ) f(x )j < jx x j: 임의의 0 x ;x 에대해 jf(x ) f(x )j < 임을증명하여라. ( 통신강좌 ) 풀이 jx x j 이면볼게없으므로, x >x + 일때만보자. 일반성을잃지않고f (0) = f() = 0이라둘수있다. x = 0; 을대입하여각각 jf (x ) j < x (x 6=0) 과 jf (x ) j < x (x 6=) 을얻는다. 따라서 y = f (x) (0 x ) 의그래프는아래그림의정사각형내부에놓이게된다. f(x) (/, /) P P' 0 P' P (,0) (/, - /)

111 . 대수중급문제 결론은 x = x 일때자명하므로, x 6= x 라가정하자. 그러면문제의가정 f (x ) f(x ) x x < 은, f의그래프위의임의의서로다른두점 P (x ;f(x )), P (x ;f(x )) 을지나는직선의기울기가 보다작다는것을의미한다. 이직선이P 0 x 0 ;f x 0 과 P 0 x 0 ;f x 0 에서이정사각형의경계와만난다고하면, x 0 >x0 + 인상황이므로 P 0 은 P 00 = P +( ; ) 의지점 ( 마름모의오른쪽두변위에있거나외곽으로벗어난다 ) 보다더 x-축에가깝게놓인다. 즉, jf (x ) f (x ) j jf x 0 f x 0 j < 이성립한다. } 69. n =; ; 3;::: 에대해 a n = n + p n 4 +4 이고, bn = a a a n 이다. b n = 이로부터 (n +) 3 < b n p p (n +) p n 임을보이고, +n + n n + < 임을보여라. ( 아일랜드 99-7) n3 70. 임의의실수 x 에대해 p(x )=p(x) 을만족하는다항식 p(x) 를모두구하여라. ( 아일랜드 99-) 풀이상수함수중에는 a = a 에서 P (x) =0과 P (x) =만가능. 이제일차이상일때보자. 우선최고차항의계수는. 최고차가 n차, 다음차가 m차라면 m+n 차항이존재하게되어모순. 따라서, p(x) =x n 꼴뿐. 답 0,, x n } 7. 모든실수 p, q, r, ' 에대해서, 다음부등식이성립함을보여라. ( 체코슬로바키아 99-) p cos ' + q sin ' cos ' + r sin ' µ p + r q(p r) + q 증명양변에 를곱하고배각공식으로고쳐서쓰면다음과같다. q p( + cos ')+qsin ' + r( cos ') p + r (p r) + q q (p r)cos' + q sin ' (p r) + q 이것은 a = p r, b = q, x =cos', y =sin' 라할때다음의코시부등식으로부터확인된다 : (ax + by) (a + b )(x + y )=a + b 7. 자연수 n에대해u k (n) (k =0; ; ;:::) 이 jxj < 인임의의 x에대해다음을만족하도록정의되어있다. x n ( x)( x n ) = X u k (n)x k k=0 u k (n) 의간단한식을찾아라. ( 통신강좌 99-4-) 풀이 계수비교법으로풀수있겠다.() 의좌변은 0 Ã x n X! ( x)( x n ) = x j A x in j=0 i= (4) 으로나타낼수있다. 한편, k = pn + q (0 q<p 이고 p, q 는정수 ) 로놓고,() 의우변과 () 의좌변에서 x pn+q 의계수를비교하면, 모든 k 에대해 u k (n) =u pn+q (n) =p (* i =,,:::, p 에대해 번씩, 모두 p 번세어진다.) 이된다. 이값은 q 에무관하며, p = k 이므로 u k (n) = n k 이다. } n

112 대수 73. 초항 r 0 = a>0 가주어졌을때, r n = r n+ + r n+ 를만족하는양의실수열 fr n g 은유일함을증명하여라. ( 통신강좌 ) 증명특성방정식을이용하면 rn = c n + d n 꼴인데, < 이고 0 < < 이라서 d 6= 0이면 n이충분히커지면음양을크게진동함. 따라서, d =0이어야하고, 그래서등비수열뿐임. 74. 다음을만족하는함수 f :[0; ]! R 가있다. (i) f() = (ii) 모든 x [0; ] 에대해f(x) 0 (iii) x; y; x + y [0; ] 이면항상 f(x + y) f(x)+f(y) 모든 x [0; ] 에대해 f(x) x 임을증명하여라. ( 미국 993-3) 증명 f(0) = 0, f( ), f 는증가함수. 귀납적으로 f( n ) n. k+ <x k 라하면 f(x) f( k ) k < x. 미국 993-3에서는 f(x) cx 가항상성립하는c의최소값을구하라고했 ( 0 (x 는데, c< 일때는다음의반례로해결됨 : f(x) = ) (x> ) 75. 다음조건을만족하는연속함수 f :[0; ]! R 를모두찾아라 : 임의의자연수 n 3 에대해, ( 통신강좌 ) x ;x ;:::;x n [0; ]; x + + x n = =) f(x )+f(x )+ + f(x n)= 풀이 =f() = f() + f(0) 임에서 f() =, f(0) = 0. x = x = = x n = n 을대입하면 nf( n )=, 즉 f( n )= n. x = k n, x = x 3 = = x n k+ = n 을대입하면 f( k n )+(n k)f( n )=, 즉 f( k n )= k n. 따라서, 모든유리수 x [0; ] 에대해서 f(x) =x. 연속함수이므로모든실수 x [0; ] 로바로확장된다 ( 각각의무리수에대해그무리수로수렴하는유리수열을잡으면됨 ). } 76. 다음연립방정식의양의실수해를모두구하여라. ( 러시아 993 4차-y0-3) x + =4; x + =; :::; x 99 + =4; x x 3 x 00 x 00 + = x q 풀이 각식을AM-GM 으로 4 q x, x x x 3 등으로정리한후변변다곱하면등호조건이 성립할경우가되어끝. } 풀이 코시부등식에의해 (50 4)(50 )=(x + + x x x x 99 + )( + x 4 x 00 x + + x x x 3 x 00 ) 등호가성립할때이므로 x 홀 = a = x 인상황이고, 대입하면짝 a =. 답 (; ; ; ;:::;; ) } 풀이 3 WLOG x 홀중에서 x 이최대라할수있고, 그럼 x = 4 x, x 3 = x = 4 x 3 x x 이므로 (x ) 0. 등호가성립할때이므로 x = x 3 = = x 99 =. } 77. 다음을보여라. ( 러시아 993 4차-y0-6) r + q p 993 < 증명차근차근제곱근을벗기면준식은 ( ((( ) 3 3) 4 4) 99) 993 > 993 과동치. n 일때 n n = 이므로 n n 을계속다시 로대체할수있음. 즉, 마지막에 993 > 993 이되고이것은참.

113 . 대수중급문제 다항식 x 3 + ax + bx + c 가서로다른세실근을가질때, 다음다항식도마찬가지로서로다른세실근을가짐을증명하여라. ( 폴란드 993/994 차 -5) x 3 + ax + 4 (a + b)x + (ab c) 8 증명처음식의세근을,, 라하면나중식의세근은 ( + ), ( + ), ( + ). 79. N 0 는음이아닌모든정수들의집합이다. 함수 f : N 0! N 0 는모든 n N 0 에대해다음을만족한다. f(f(n)) + f(n) =n +3 f(994) 을구하여라. ( 통신강좌 ) 풀이 f(m) =f(n) 이면좌변이같으므로우변도같아야해서 m = n. 즉, 단사함수. f(0) = x 라하면 f(x)+x =3. x =0이면 0=3모순. x =3이면 f(3) = 0 으로단사임에모순. x =이면 f() = 이고 n =대입하면 f() = 6, 다시 n =대입하면 f(6) = 로모순. 즉, x =이다. 그럼 f() =, f() = 3 등이연이어얻어진다. f(n) =n + 이면 f(f(n)) + f(n) =n +3 에서 f(n +)=n + 가되므로귀납적으로 f(n) =n +이항상참임을확인할수있다. 즉, f(994) = 995. } 80. P (x +)=P (x)+x + 을만족하는모든다항식을구하여라. ( 통신강좌 ) 풀이 P (0) = a 로두자. P (n) =n + a 이면 P (n +)=n + a +n +=(n +) + a 이므로귀납적으로 P (n) =n + a 는무한히많은근 ( 모든자연수 ) 을갖고따라서다항식이므로항등적으로 P (x) =x + a 답 } 8. 모든정수 n> 에대해다음을증명하여라. ( 아일랜드 994-8) n((n +) =n ) < nx i= i + i <n( n =(n ) )+4 P ³ 증명왼쪽은양변에 n을더한후산술-기하. 오른쪽은양변에 n 더하고 ni= i i 을뺀후산술-기하하면 P n 에관한부등식만남음 8. q는임의의양의실수이고, 실수수열 a n (n =; ;:::) 은 a 0 =,a =+q 이며모든자연수 k에대해다음식을만족한다. (i) a k = a k a k a k (ii) a k a k = a k+ a k q 가얼마로주어져도, 다음을만족하는자연수 N 을항상찾을수있음을보여라 : n>n 을만족하는모든 n 에대해서항상 a n > 994 이다. ( 호주 994-5) 증명 a n+ a n a n a n,am-gm 83. a 3 + b 3 + c 3 + d 3 = a + b + c + d =0 인실수 a, b, c, d 가있다. 어떤둘을합하면 0 이됨을증명하여라. (Towns 994 가을 SO3) 증명 b 3 + c 3 + d 3 =(b + c + d) 3 이므로이것을정리하면 (b + c)(b + d)(c + d) =0. 또는 (a + b) 3 + (c + d) 3 =0 을정리해도비슷하게할수있음. 84. 임의의양수 a ;a ;:::;a n 에대해다음부등식을증명하여라. (Towns 994가을 SA4) µ µ µ + a + a + a n ( + a )( + a ) ( + a n) a a 3 a 증명 양변에 a a a n 곱하고, 윤환식이므로 WLOG a n 이최대라하여귀납법.

114 4 대수 증명 양변에우변을다시한번곱해주고코시부등식. 85. 임의의실수 x> 에대하여다음을보여라. ( 통신강좌 ) (x )x x x (x )x > 증명 ( 상문고이경용 ) (i) (x )x x x µ x x µ () x x (x ) x x x (ii) f(x) =x log x 는아래로볼록 Jensen 부등식에의해 µ x x µ = x (x ) µ x µ = x x + x µ x 4 µ > (x x) x x > f(x )xg x x = (x ) x x x µf 0 (x) =logx +;f 00 (x) = x > 0 f(x ) + f(x) > f(x ) µ x (x ) log(x ) + x log x > (x ) x x x > log µ x x µ x (i), (ii) 에서 > (x )x x x µ x x > 86. 함수 f : Q! Q 가다음조건을만족한다고한다 (Q는유리수의집합). µ x + y f = f(x)+f(y) ; f(xy) =f(x)f(y) f(0) =, f() = 일때, 가능한 f 를모두구하여라. ( 통신강좌 ) 풀이 왼쪽식의 y에 0, 을각각대입하면 ³ x f = f(x)+ µ x + = f 따라서, 여기에 x 대신 x를대입하면 f(x) = f(x + ) 이항상성립한다. 그럼모든정수 n에대해 f(n) =. 이번에는오른쪽식에서 f() = f(n)f( n ) 이므로 f( ) =. n 다시오른쪽식에서 f( n m )=f(n)f( )=. m 따라서모든유리수 x에대해f(x) = 답 }

115 . 대수중급문제 다음의두조건을만족하는함수 f : N! N 을모두구하여라. () f(xf(y)) = yf(x). () x>y이면 f(x) >f(y). ( 통신강좌 ) 풀이 f(f()) = f() 에서 x>y이면 f(x) >f(y) 라하였으므로, f() > 이나 f() < 이될수없다. 즉, f() = 이다. 한편 x =을대입하면 f(f(x)) = x 가되고, f(x) =k 라하면 f(k) =x 가된다. 여기서만약 x>k 라면 x = f(k) >f(x) =k 가되어모순이생기고 x<k 라면 x = f(k) <f(x) =k 가되어역시모순이다. 따라서 x = k 가되어야한다. 즉, f(x) =x 이다. } 88. 모든실수 x, y 에대해다음조건을만족하는함수 f : R! R 를모두구하여라. xf(x) yf(y) =(x y)f(x + y) ( 아일랜드 995-5) 풀이 f(x) 대신 f(x)+c 를대입해도성립하므로상수부는 f(0) = 0 일때만풀고나중에상수를더해줘도된다. y에 x를대입하면 xf(x)+xf( x) =0, 즉 f( x) = f(x) 가성립 (x =0일때는따로성립함을확인 ). 이번에는 y에 y를대입하면좌변은불변이므로 (x y)f(x + y) =(x + y)f(x y) 가성립. 여기에 x = X+Y, y = X Y 를대입하면Yf(X) =Xf(Y ), 즉 f(x)=x = f(y )=Y = a 의상수이다. 따라서, f(x) =ax + b 꼴만가능하고, 이꼴은대입하면항상성립. } 이아닌모든실수 x 에대해다음을만족시키는실계수다항식 P (x) 를모두구하여라. ( 오폴 995-4) P (x) + P µ µ = P (x )P x x 풀이우선상수함수 P (x) a 일때는a = a 이므로 a =0뿐. P (x) =a nx n + a n x n + + a x + a 0 라하자 (n, a n 6=0). 양변에 x n 을곱하면양변모두다항식이므로, 무한히많은 x에대해등식이성립한다면양변은다항식으로서완전히같아야한다 ( 항등정리 ). 양변의최고차항 x n 의계수를비교하면 a n = a na 0, 즉 a n = a 0. 한편양변의상수항을비교하면 a 0 = a n + a n + + a + a 0, 즉 a + + a n =0이므로 a = = a n =0. 따라서, P (x) =a(x n +) 꼴이다. 이것을준식에대입하면 a (x n +x n ++x n + x n )=a (x n ++x n ) 에서 x n + x n =0으로해가없다. 즉, P (x) 0 이유일한해이다. } nx nx 90. x ;x ;:::;x n 은 x i x i 을만족하는양의실수들이다.보다큰모든실수 t에대해다음부등 i= i= 식이성립함을증명하여라. ( 폴란드 995 차-4) nx nx x t i x t+ i i= i= 증명 0 <x i < 일때와 x i 일때각각 x i (x i ) x t i (x i ) 임을확인. P 별증 x t+ 체비셰프응용으로얻어지는부등식 P x t 를증명하여이용해도됨. 하지만해머로파리를잡는격인듯. P x P x

116 6 대수 9. n 은주어진자연수이다 = n x x x n 을만족하는임의의양수 x ;:::;x n 들에대해, 합 x + x + x xn n n 의가능한최소값을구하여라. ( 폴란드 차 -4) P 풀이가중치산술-기하평균부등식으로 L ( k )(x= x = x n=n n ) = P k ( P )( k 조화평균 n ) = P k = P. k 등호는모두 일때성립. } 별해 [ 보조정리 ] 임의의 x>0와자연수k에대해 xk k + x + k. 증명은 xk + k x = xk + x + + x k + 의 AM-GM. 위의보조정리에의해싹다더하면 S + n n + H n, 즉 S H n := n. 등호는 x = = x n =일때실제성립. } 9. f, g, h 가이차다항식일때방정식 f(g(h(x))) = 0 의해가,, 3, 4, 5, 6, 7, 8 이될수있는가? ( 러시아 995 최종 -y9-3) 풀이 g(h());:::;g(h(8)) 은모두f(z) =0의근. 만일이들중어느 5개가같으면 ( 그값을k, 그 5개의 x를 a ;:::;a 5 라하면 ) h(a );:::;h(a 5 ) 는모두g(y) =k 의근.차방정식이므로그럼이들중어느 3개가같고, 그럼그 3개의 a i 가 h(x) =m 의서로다른해가되어모순. 즉, g(h());:::;g(h(8)) 은 4:4로같고, h();:::;h(8) 은 :::로같음. 이차식의그래프개형을생각할때 h() = h(8), h() = h(7), h(3) = h(6), h(4) = h(5) 이고, 또이순서대로증가혹은감소이므로, g(h()) = g(h(4)), g(h()) = g(h(3)). 대칭축을생각하면 h() + h(4) = h() + h(3) 인데, 이것을 h(x) =b(x 4:5) + c 에대입하여풀면b =0이나와서모순. } a<¼=4, 0 b<¼=4 일때, 다음을증명하여라. ( 통신강좌 ) µ a + b (tan a tan b) = tan (tan a +tanb) 증명왼쪽부등식은제곱하고아마반각공식으로전개해서. 오른쪽은옌센. 94. 다음식을만족하는양의실수해 (a; b; c; d) 를모두구하여라. ( 영국 996 차 -4) a + b + c + d = abcd =7+ab + ac + ad + bc + bd + cd 풀이두식에서각각 8 abcd 8 의부등식한쪽씩나옴. 등호성립조건 a = b = c = d =3이유일한해. } 95. 다음연립방정식의모든실근을구하여라. ( 캐나다 996-) 8 4x +4x >< = y 4y +4y = z >: 4z +4z = x 풀이 좌변을관찰하면 x, y, z 는모두음이아닌수임을알수있다. 산술 - 기하평균관계에의해 y = 4x +4x 4x 4x = x 마찬가지로하여 y x z y 이고, 따라서 x = y = z 일수밖에없다. 이제세식은모두같고, 4x +4x = x 의식을풀어주면 4x = x +4x 3, 즉 x(x ) =0이므로, 실근은 (x; y; z) =(0; 0; 0) 과 ( ; ; ) 뿐이다. }

117 . 대수중급문제 다항식 (x +) n r 이 x +x + 을인수로갖도록하는자연수 n 과실수 r 을모두구하여라. ( 폴란드 차 -) 풀이 x +x+ 의근은 i 이고, 이것을 (x +) n r 에대입하면 r =( p e ¼ 4 i ) n = 4 i. 이것이실수가될조건은 4 j n 이고, n =4k를대입하면 r =( 4 )k. } n= e n¼ 97. 양수 a, b, c 가 a + b ab = c 을만족하면 (a c)(b c) 0 도만족함을증명하여라. (Towns 996 봄 JA) 증명 WLOG a b 라하면 c = b + a(a b) b, c = a + b(b a) a 이므로 b c a. 증명 cos 제 법칙으로보면 cos C = 이므로 \C =60±, 즉중간각. 즉 c 는중간변. 98. x ;x ;:::;x n ;x n+ 이 x + x + + x n = x n+ 을만족하는양의실수들이라할때, 다음을증명하여라. ( 통신강좌 ) v nx p ux xi (x n+ x i ) t n x n+ (x n+ x i ) i= i= 증명 시각적편의상 xn+ = s 로대체해서쓰기로하자. 코시부등식에의해 Ã n! X ( 좌변 ) p = xi (s x i ) n X x i (s x i ) i= ³ = n s X x i ns ( X x i ) ( 이것도코시부등식 ) = X s(s x i )=( 우변 ) 99. 함수 f;g : R! R 는다음성질을만족한다. () f ± g ± f ± f ± g(x) =g(x) () f ± f ± f ± g ± g(x) =f(x) 임의의실수 x에대하여 f(x) =g(x) 임을보여라. ( 통신강좌 ) 증명 ( 과기원수학과 96학번신희성 ) g = fgffg = f(fgffg)ffg = ffgf(fgffg) = ffgfg = ff(fgffg)fg = fffg(ffgfg) = fffgg = f 00. x; y; z > 0 이고 x + y + z = p xyz 일때다음을보여라. ( 통신강좌 ) xy + yz + zx 9(x + y + z) 증명코시부등식에의해 (x + y + z)(yz + zx + xy) 9xyz =9(x + y + z). 이것으로끝 cos ± +cos ± + +cos44 ± sin ± +sin ± + +sin44 ± 의값을구하여라. (AIME 997-)

118 8 대수 풀이 cos n sin(/) = sin(n + /) - sin(n - /), so the numerator is 50(sin sin 0.5)/sin 0.5. Similarly the denominator is (cos cos 44.5)/( sin 0.5), so we get 00(sin sin 0.5)/(cos cos 44.5). Evidently this is roughly 00 (/ )( - / ) = 00( + ) = 4.4. Using sin 44.5 = sin 45 cos sin 0.5 cos 45 etc we get 00(cos ( + )sin 0.5)/( ( - )cos sin 0.5) which is exactly 00( + ). 답 4 } 0. 보다큰모든홀수의집합을 S 라하자. 각각의 x S 에대해부등식 ±(x) <x< ±(x)+ 을만족하는유일한정수를 ±(x) 라한다. 임의의 a; b S 에대해 a b = ±(a) (b 3) + a 라고정의한다.[ 예를들어,5 7 을계산해보자. < 5 < 3 이므로 ±(5) = 이다. 따라서,5 7= (7 3) + 5 = 3 이다. 또 < 7 < 3 이므로 ±(7) = 가되고 7 5= (5 3) + 7 = 이다.] a; b; c S 일때다음을증명하여라. () a b S () (a b) c = a (b c) ( 아일랜드 997-5) 증명 ( 과기원수학과 96학번허석문 ) (i) S의원소는 3이상이므로 (ii) a b = ±(a) (b 3) + a ±(a) (3 3) + a = a 고로 a b는 보다크고만일 a 4이면 ±(a) 이므로 ±(a) (b 3) 은짝수, 고로 a b는홀수 a =3이면 ±(a) = b 3+a = b =) 홀수고로 a b S a b = ±(a) (b 3) + a = ±(a) (b ) ±(a) + a = ±(a) (b +) ±(a)+ + a ±(a) <a< ±(a)+ 이므로 0 < ±(a) + a ±(a)+ + a<0 ±(a) (b ) <a b< ±(a) (b +) ±(b) <b< ±(b)+ (b가홀수이므로 ) ±(b) b b + ±(b)+ 고로 ±(a b) =±(a)+±(b) (a b) c = ±(a b) (c 3) + a b = ±(a)+±(b) (c 3) + ±(a) (b 3) + a a (b c) = ±(a) ((b c) 3) + a = ±(a) (( ±(b) (c 3) + b) 3) + a = ±(a)+±(b) (c 3) + ±(a) (b 3) + a ) (a b) c = a (b c)

119 . 대수중급문제 임의의실수 a 에대하여, 다음을만족하는세실수의순서쌍 (x; y; z) 의개수를구하여라. ( 폴란드 997 차 -) x + y + z = x + y + z = x + y + z = a 풀이 (x y)(x + y ) = 0 등을얻을수있고, x = y = z 일때, x = y 6= z 일때, x 6= y 6= z 6= x 일때등으로경우를나눠서풀면됨. } 04. 다음을만족하는세실수쌍 (x; y; z) 를모두구하여라 : ( 폴란드 차 -) 3(x + y + z )=; x y + y z + z x = xyz(x + y + z) 3 풀이 x + y + z = a, xy + yz + zx = b, xyz = c 로치환. 첫식은 a b = 3. 첫식에서코시부등식으로 =3(x + y + z ) (x + y + z) 이므로 a. 둘째식에서재배열부등식으로 xyz(x + y + z) 3 =(xy)(xy)+(yz)(yz)+(zx)(zx) xyz(x + y + z) 이므로 ca 3 ca. a =일때는 x = y = z 로따로풀면 ( 3 ; 3 ; 3 ). a < 일땐ca 0, 그런데 ca 3 0 이므로 ca =0. 그럼 (xy) +(yz) +(zx) =0이므로 x; y; z 중둘이상이 0이고답은 f p ; 0; 0g. } 각항의분모가삼각수 ( 즉, k = n(n +)=꼴의수. n =,,:::, 996) 의역수의합으로주어진다음수 S = 에대하여 S>00 임을보여라. (APMO 997-) 풀이 (O±cial) nx k= 준식의값을 S라하면 n(n +) = ) n + n = (n +) = = = 3993 k(k +) = 996 X S = n= ) n + = 3993 ) n = 996 X n µ k µ = k + n + k= 996 n + n = X n+ = X n n+ 여기서 996 X µ n = n= µ µ > + + µ µ + + = n n + µ + = X n n n µ µ µ µ µ µ = =6 {z } 0개 µ µ

120 0 대수 참고 996 X µ n =+ n= ) S> = 00 µ µ < +++ += {z } 0개 에서 S<998 + = 이다 µ µ } 06. 양수 a 가주어져있다. 임의의양수 x, y 에대하여다음성질을만족하는모든실수 c 를구하여라. ( 폴란드 997/998 차 -4) (c )x a+ (cy x)y a 풀이준식은 c (ct )t a 과동치 (t = y x > 0). 즉, c(ta+ ) t a. t > 이면 c ta+ t a = t + t t a =: R(t) 라하면t! + 일때R(t)! +... a 대충미분과극한써서확인하면뭐든나올듯. t =+e 로치환한다거나. } 07. 수열 fa n g의각항은서로다른자연수이다. S n = a + +a ( + a )( + a ) + + a a a n ( + a )( + a ) ( + a n ) 이라할때, <S n < 을만족하는최소의 n과이때의수열 a n 을구하여라. ( 통신강좌 ) 풀이 ( 이익재 ) 먼저 을 a 으로변형하자. 그러면, +a a + a + +a ( + a )( + a ) = a + a + ( + a )( + a ) = a a ( + a )( + a ) N = a + +a ( + a )( + a ) + + a a a k ( + a ) ( + a k ) a a a k = 로변형된다고가정하면 ( + a ) ( + a k ) a a a k N + ( + a ) ( + a k+ ) = a a a k ( + a ) ( + a k ) + a a a k ( + a ) ( + a k+ ) +a k+ = a a a k ( = < a a k+ ( + a ) ( + ak +) < = 998 > a a n ( + a ) ( + a n) > 0 0 < a +a a a n < +a n 998 ( + a ) ( + a k+ ) ) a a k+ ( + a ) ( + a k+ ) = S n이성립한다. +a 각각의 an 을살펴보면 a n 이커질수록식이커지므로 a n 은작을수록좋다. +a n ) a n = k로차례대로값을취할때식이최소 n +n = +n < 998 ; n > 998 이때수열 a n = f; ; 3; ; 998g }

121 . 대수중급문제 08. 다음점화식을만족시키는양의실수로이루어진수열 fa n g, fb n g 을구하여라. a = ; a n = a n + b n; (a n + b n) + a n = (n =; 3;:::) ( 통신강좌 ) 풀이 ( 서울과학고 학년김홍식 ) (a n + b n ) + a n =(a n + b n ) + a n b n =a n(a n + b n )= ) a n + b n = a n ) (a n + b n ) + a n = 4a + a n = n 위식에서a n = c n 이라하자. + c n =(n = );c = 4c n 4 수학적귀납법을이용하여수열의성질을보이자. i) c = 4 = ii) c k = + k k + 라하고,c k+ = = c k = 4c k+ k + 4c k+ = k + ) c k+ = k + k k + k + 임을보이자. (k +)+ = k + k + k + 4(k +) = k + k +4 = k + (k +)+ ) 수학적귀납법에의하여 c n = n r n +. 즉, a n = } n n +, b n = a n a n = r r n + n n n 다음부등식을만족하는자연수 n을모두구하여라. ( 호주 Senior Contest 998-) s + n < n 풀이 q 좌변은 보다크므로 p < n =) n < p =) n> p p =4+ p =6:8 따라서 n 7. n =7일때준식의양변을제곱하면 + µ 64 <, 즉 < 5 이고이것은 < 300 으로성립. n이더커지면좌변은점점감소하고우변은점점증가하므로당연히성립한다. 따라서, 문제의부등식은 n 7 일때에만성립한다. } 0. f :[0; ]! R 는 n =; ;::: 일때 f( n )=( )n 을만족하는함수이다. f = g h 를만족하는두증가함수 g; h :[0; ]! R 는존재하지않음을증명하여라. ( 폴란드 999 차 -) 증명존재한다고하면, g( n+ )=h( n+ ) h( n ) =g( n ) g( n ) 이므로 g( ) g() n. n+ 즉 g(0) g() n 이모든자연수n에대해성립해야하므로모순.. y( + x )=x( p 4y ) 을만족하는실수 x, y 에대해, xy 의최대값을구하여라. ( 몰도바 000 최종 -y8-6) 풀이 x =0 이면 xy =0. x 6= 0 이면 y x = p 4y +x 0 이므로 xy 0. 답 0 }

122 대수. 방정식 (x 3x ) 3(x 3x ) x =0 의실수해를모두구하여라. ( 몰도바 000 최종 -y9-5) 풀이 F (x) =x 3x 라하면준식은 F (F (x)) = x 의해를묻는것. F (x) x j F (F (x)) x 이므로준식은 x 4x 로인수분해됨. 즉, 실제로인수분해해보면준식은 (F (x) x)(x x 4) = 0. } 3. 모든실수 x, y에대해f(x + y) f(x y) =y(3x + y ) 을만족하는함수f : R! R 을모두구하여라. ( 몰도바 000 최종-y0-5) 풀이 x + y = X, x y = Y 로치환하면 f(x) f(y )=(X Y )((X + Y ) XY )=X 3 Y 3. 즉 f(x) =X 3 + c 꼴 (c = f(0)). 그리고, 이꼴은모두다성립. } 4. 모든양수 x ;x ;:::;x n 에대해부등식 x x + x x x n x n n n (x + + x n) 을만족하는정수 n 을모두찾아라. ( 폴란드 000/00 차 -3) 풀이 n = 일때는성립. n 3 일때는다음의반례가있음 : x = x n =,x = = x n =3. } 5. 모든자연수 n 과 k 에대해다음부등식을증명하여라. (Towns 000 봄 SO3) k + k + + n k nk (n ) k n k (n ) k 6. 모든자연수 n 에대해다음을증명하여라. ( 아일랜드 00-4) n 3n + nx k=n+ 3n + k 4(n +) 증명왼쪽은가우스합의방식으로둘씩묶어 AM-HM 쓰면끝. 오른쪽은적분법이나귀납법으로하면풀림. 혹은적분법보다조금약하게, 넓이를하나의사다리꼴로부풀리면우변대신 3 4 을금방끌어낼수있는데, 이것으로는조금부족하니까사다리꼴을조금만더정밀하게축소하면될듯. 7. x, y, z 가삼각형의세변일때, 다음을증명하여라 : ( 인도지역예선 00-6) jx (y z)+y (z x)+z (x y)j <xyz 증명준식의좌변을정리하면 j(x y)(y z)(z x)j 이고이것은삼각부등식 jx yj <z, jy zj <x, jz xj <y에의해우변xyz보다작다. 8. 임의의음이아닌실수들 x ;x ;:::;x n (n ) 에대해다음부등식이성립함을증명하여라. ( 폴란드 00 3 차 -) nx ³ n nx ix i + x i i i= i= 증명 산술-기하로 x i i ++ + ix {z } i 이고, 이것을모두변변합하면됨. i 개

123 . 대수중급문제 3 9. 다음방정식의실수해를모두구하여라. ( 폴란드 00/00 차 -) jxj jx +j + jx +4j jx +6j + jx +998j = jx +j jx +3j + jx +5j jx +7j + jx +999j 풀이좌우변모두 500부터 500까지감소하는비스듬-계단형그래프. 답은 x 0, x 999, 그리고 4n x 4n +꼴의범위들의합집합. } 0. 임의의실수 x에대하여 f(x) =f(x) =f( x) 을만족하는함수 f : R! R 는모두주기함수임을증명하여라. ( 폴란드 00 차-) 증명 f(x +)=f( +x x )=f( )=f( x) =f(x). 임의의실수 x ;x ;:::;x n, y ;y ;:::;y n 에대해다음부등식이성립한다. q x x x n + y y y n qx qx + y + y x n + y n 이런자연수 n 을모두찾아라. ( 폴란드 00 차 -6) 풀이모든항이양수일때만확인해보면충분. n =일때는성립하지않고, n 일때는양변전개해서제곱해보면 (x x n ) 과 (y y n ) 은양변에서소거되고, 좌변에 (x x n )(y y n ) 이남는데, 우변에최소한 x y y n + y x x n 은있으므로 AM-GM 으로충분함. } 별해귀납법으로풀수있음. n =일때는코시로성립하고, n일때성립함을가정하면 n +일때는 x 0 n = x nx n+, yn 0 = y ny n+ 로한항으로묶어보면, n일때를한번이용한후우변에서 p (x 0 n ) +(yn) 0 px q n + yn x n+ + y n+ 만보이면충분함. 이건전개해보면자명. }. 다음부등식을증명하여라. ( 플란더즈 00-3) 5 < < 0 증명 A = n 00 라하자. n < n n+ 이므로다음부등식이성립한다. A< A> = X = Y 그럼 A <AX= = 0 A >AY = = 00 즉, 0 <A < 5 이므로문제의부등식은성립한다. 별증 (n +)(n ) = n <n 임을이용하자. A = < 00 A = > 00 역시 0 <A < 5 이므로문제의부등식은성립한다.

124 4 대수 3. R은실수전체의집합이다. 임의의실수 x, y에대해다음을만족하는모든함수 f : R! R 를결정하여라. ( 미국 00-4) f(x y )=xf(x) yf(y) 풀이 준식에 (x; y) =(t; 0), (0;t) 를각각대입하면 f(t )=tf(t); f( t )= tf(t) 를얻는다. 이로부터 f( t) = f(t) 가되고, 또 f(x y )=f(x )+f( y ), 즉 x 0, y 0 일때다음이성립한다. f(x + y) = f(x)+ f(y) ( ) 이식의 (x; y) 대신 (x + y; y) (x; y 0) 를대입하면 f(x) =f(x + y) f(y), 즉 ( ) 은 x, y의부호와상관없이성립함을확인할수있다. 또, 이로부터귀납적으로임의의자연수 n에대해f(nt) =nf(t) 도성립한다. 문제의식에 (x; y) =(t +;t ) 을대입하면 f((t +) (t ) )=(t +)f(t +) (t )f(t ) f(4t) =(t +)(f(t)+f()) (t )(f(t) f()) 4f(t) =f(t)+tf() 따라서, f(t) =tf() 이다. f() = a 라하면 f(x) =ax 가되고, 또이런꼴의함수는문제의식을항상만족함을금방확인할수있다. 따라서, 구하는함수는 f(x) =ax (a 는임의의실수 ) 들이전부이다. } 4. x, y, z 는 x + y + z = 을만족하는양의실수들이다. 다음을증명하여라. x yz + xy z + xyz 3 ( 영국 00/003 차 -3) 증명 준식은다음과같다. 코시 - 슈바르츠부등식으로 (x + y + z)xyz 3 ( ) ( + + )(x + y + z ) (x + y + z) 따라서, x + y + z p3 ( 등호는 x = y = z 일때 ) () 또, 산술 - 기하평균부등식으로 x + y + z 3 q 3 (xyz) 따라서, xyz 3 p 3 ( 등호는 x = y = z 일때 ) () (), () 를변변곱하여 ( ) 이증명되고, 등호는 x = y = z = p 3 일때성립한다.

125 . 대수중급문제 5 증명 x, y, z 과 yz, zx, xy는그크기순서가서로역순이되므로, 체비셰프부등식에서최소값을주는경우에의해 (x )(yz)+(y )(zx)+(z )(xy) 3 x + y + z 3 yz + zx + xy 3 이다. 완전제곱식혹은재배열부등식에의해 x + y + z xy + yz + zx 이므로준식이성립한다. 5. 양수 x, y, z 가 을만족시킬때 x + y + z =3 xy z + yz x + zx y 의최소값을구하여라. ( 셈본중등고급도전문제 3..) 풀이 xy yz = a, z x = b, zx y = c로치환하면 x = ca, y = ab, z = bc이므로 x + y + z =3에서 ab + bc + ca =3 a + b + c의최소값을구하기위해 (a + b + c) 의최소값을구하자. (a + b + c) = a + b + c +ab +bc +ca 3ab +3bc +3ca =3 따라서, p 3 을최소값으로갖는다 ( 등호는 x = y = z = 일때성립 ). } 6. 실수 x, y, z 가조건 4(x + y + z) =x + y + z 을만족시킨다. 식 xy + yz + zx 의최대값을 a, 최소값을 b 라할때,0a + b 의값은얼마인가? ( 한국 003 차 -S5) 풀이 A = x + y + z, B = x + y + z, C = xy + yz + zx 라하면문제에서 4A = B 이다. 이로부터 A = B +C =4A +C, 즉 C = (A ) 가된다. 또, 잘알려진부등식 C B 에서 A = B +C 3B =A, 즉 0 A 이범위에서 C 는 C 48 이되고,(x; y; z) =(; p ; p ), (4; 4; 4) 등으로등호가성립하는경우도각각존재한다. 답 478 } 7. 조건 (a ) + b 4 와 (x +) + y 를각각만족시키는실수의순서쌍 (a; b), (x; y) 에대하여 ax + by 의최대값을 M, 최소값을 m이라할때m + M 의값은얼마인가? ( 한국 003 차-S8) 풀이 () A =(a )(x +) 라하면코시부등식에서 jaj ((a ) + b )((x +) + y ) 4 즉 A 이므로 ax + by =(a )(x +) a +x = 8

126 6 대수 실제로 (a; b; x; y) =(4; 0; ; 0) 일때등호가성립하므로최소값은 8 이다. () 이제최대값을구해보자. 역시코시부등식에서 따라서,(a )x + by p x, 즉 ((a )x + by) ((a ) + b )(x + y ) 4 ( x) ax + by x + p x = ( p x ) + 이다. 실제로 (a; b; x; y) =(; p 3; p ; 3 ) 일때등호가성립하므로최대값은 이다. (), () 에서답은 m + M =65. } 8. 정수계수를가지는다항식 f(x) 에대하여 f(a) =,f(b) =,f(7) = 3, a<b<7 을만족시키는두정수 a, b 가존재한다고하자. 이때, () 방정식 f(x) =5의정수해는기껏해야한개뿐임을보이고, () 방정식 f(x) =5의정수해가꼭한개일때, 이러한다항식 f(x) 를모두구하여라. ( 한국 003-S) 풀이 () f(7) = 3 이므로 f(x) =(x 7)G(x)+3 으로나타낼수있다 (G(x) 는몫,3 은나머지 ). x = b 대입 f(b) ==(b 7)G(b)+3; (b 7)G(b) = 이때 G(x) 는정수계수로이루어진 f(x) 를 (x 7) 로나눈몫이므로정수계수로이루어져있고 b 도정수이므로 G(b) 는정수. b 7 도정수. ) b 7 = 또는 인데 b<7 이므로 b 7 =. ) b = 6. 로나타낼수있다. x = a 대입 f(x) =(x 6)H(x)+ =(a 6)H(a)+; (a 6)H(a) = 위와마찬가지로 H(a) 는정수, a 6 도정수이므로 a 6 = (* a<6). ) a =5. f(x) =5 를만족하는 x 를 라할때, f(x) =(x 5)I(a)+ 3 에서 5=( 7)G( )+3 ) ( 7)G( ) = 4 에서 5=( 6)H( )+ ) ( 6)H( ) =3 5 3에서 5=( 5)I( )+ ) ( 5)I( ) =4 6 ) 7 = ; 중하나, 6 = ; 3 중하나, 5 = ; ; 4 중하나. 세조건을만족시키려면 =9 여야한다. ) G( ) =H( ) =I( ) = 일때에기껏해야한개의해가존재하고, 그이외에는존재하지않는다. () f(x) =(x 5)(x 6)(x 7)(x 9)G(x)+ax 3 + bx 3 + cx + d 의꼴. x =5; 6; 7; 9 를각각대입해서연립하면 a = b =0,c =,d = 4. ) f(x) =(x 5)(x 6)(x 7)Q(x)+x 4. ( 단, Q(x) 는정수계수인다항식또는정수인상수 ) } 9. a + b + c =0 이라할때 a n + b n + c n n abc =0 이되는자연수 n 을모두구하여라. (ML 프로포절 33-4)

127 . 대수중급문제 7 풀이 a, b, c에대해성립한다면, 임의의양의실수 k에대해, ka, kb, kc에대해서도성립해야합니다. 고로,() 에의해 k n (a n + b n + c n ) n k 3 abc = k n nabc k 3 nabc = nabc(k n k 3 )=0 a, b, c, k 는모두 0 이아닌실수로만들수있으므로 n =3. } 30. A 가 0 을제외한모든실수들의집합일때, x + y 6= 0 인모든 x; y 6= 0 에대해다음을만족하는함수 f : A! A 를모두구하여라. ( 북유럽 003-4) f(x)+ f(y) = f(xyf(x + y)) 풀이만일 xf(x + y) =이되는경우가있다면준식에서 f(x) =0이되어모순. 즉, xf(x + y) 6=. x + y 를 x로치환하면 (x y)f(x) 6=, 혹은 y 6= x. y f(x) 가가지지않을수있는값은0과 x뿐인데, 우변은 x가될수없으므로 0이어야함. 즉, f(x) = x. } 3. 둘레의길이의합이 인삼각형의세변의길이를 a, b, c 라할때, 다음부등식을증명하여라. ( 아일랜드 003-6) ab + bc + ca abc + 7 증명 ( 서울과학고 학년김성구 ) a + b + c = 이므로 ab + bc + ca abc = a + b + c +( a)( b)( c) =+( a)( b)( c) 따라서, 준식을증명하는것은 0 ( a)( b)( c) () 7 을증명하는것과같다. a 이라면 a, b, c가삼각형의세변이라는조건에따라 b + c>a이고, 따라서 b + c>, 그러면 a + b + c> 인데이것은모순이다. 이로부터대칭적으로고려하면 a, b, c 이다. 그럼 () 의왼쪽부등식은당연히성립한다 ( 등호는성립하지않는다 ). () 의오른쪽부등식은산술-기하평균부등식에의해 p 3 ( a)+( b)+( c) ( a)( b)( c) = 3 3 임에서성립한다. 등호는 a = b = c = 3 일때성립한다. 3. 양의실수 a, b가주어져있다. 임의의양의실수 x에대해, c는두수ax + ax 와 bx + 보다동시에크 bx 지는않다고한다. 실수 c의최대값을구하여라. ( 아일랜드 003-9) 풀이 ( 장영실과학고 학년김윤섭 ) ax + ax 은 x = 에서최소값을갖는다 ( 미분혹은산술-기하평 a 균부등식에의해알수있다 ). 따라서, a = b 라면, 명백히 c 최대 = a 이다. a 6= b 라면, c 최대는두곡선 의교점이다. 즉 ax + ax = bx + bx 일때. 이것을풀면 (a b)x = bx (a b) = ax abx 그러므로 x = abx, x = ab, 즉 x = p 에서 c ab 최대가발생하며, 그값은 c 최대 = ax + ax = p a p p p ab a b + ab a = p + p = a p + b b a ab }

128 8 대수 33. 주어진정수 a>, b> 에대해, a b + b(a +) 임을보이고등호가성립할조건을구하여라. ( 중미 003-3) 증명 a를고정시키고 b에대한귀납법. b =3일때a 3 + 3(a +) 은 (a +) (a ) 0 으로성립. b일때성립하면 a b+ + = a(a b +) a+ >ab(a+) (a+) (b )(a+) > (b+)(a+) 로 b +일때도성립. b +로올라가면서등호가깨지므로등호는 b =3일때,(a; b) =(; 3) 뿐. 34. p(x) 는최고차항의계수가 인 003 차의정수계수다항식이다. 방정식 p(x) =5 는 003 보다많은개수의정수해를가질수없음을보여라. ( 호주 003-4) 증명 p(x) =5이므로 p(x) =5이거나 p(x) = 5 이다. I) p(x) =5의정수해와 p(x) = 5 의정수해가모두존재할때. 일단 A를 p(x) =5의정수해라고가정하고, B를 p(x) = 5 의정수해라고가정하자. 그렇다면 p(a) p(b) =0이다. 이때, p(a) p(b) =0은 A B 를인수로가진다 (A n B n 은모두 A B 를인수로가짐 ). 따라서 A B 는다음중하나이다. +; +; +5; +0; ; ; 5; 0 그러므로 p(x) =5의가능한정수해 (A) 는기껏해야 8개뿐이고, p(x) = 5 의가능한정수해 (B) 역시 8개뿐이다. 따라서 p(x) =5는기껏해야 6개의해를가진다. 따라서 003개이하의정수해를가진다. II) p(x) =5나 p(x) = 5 중하나만정수해를가질때. 이경우의정수해는기껏해야 003개나올수있다. 따라서 003개이하의정수해를가진다. III) 둘다모두정수해를가지지않을때정수해는 0개이다. 따라서모든경우에정수해는 003개이하이다. 35. 모든정수 m, n 에대해 f(m + f(n)) = f(m)+n 을만족하는함수 f : Z! Z 를모두구하여라. (ML 프로포절 39-) 풀이 m을고정시키고 n을변화시키면 f는모든함수값을갖는다. ) 전사함수. f(a) =f(b) 라하고 n에 a, b를각각대입하면 f(m + f(a)) = f(m + f(b)) =) f(m)+a = f(m)+b =) a = b =) 단사함수 m = n =0 을대입하면 f(f(0)) = f(0) 가되는데, 단사함수이므로 f(0) = 0 m 에 0 을대입하면 n 에 f(n) 을대입하면 f(f(n)) = f(0) + n = n f(m + n) = f(m)+ f(n) 이로부터 f() = a 라하면 f(n) =an. 이것을문제의식에대입하면 am + a n = am + n, 즉 a = or. 그러므로 f(n) =n or f(n) = n 대입해보면성립하므로이것이답이다. } 36. a, b, c, d, e, f 는실수이고, 다음의다항식 p(x) =x 8 4x 7 +7x 6 + ax 5 + bx 4 + cx 3 + dx + ex + f 이여덟개의일차식 x x i (x i > 0, i =; ;:::;8) 들의곱으로인수분해된다. f 의값으로가능한수를모두구하여라. (APMO 003-)

129 . 대수중급문제 9 풀이 (KAIST 수학과 0 학번황진석 ) 근과계수와의관계에서 8X p = x i =4; q = X 8Y x i x j =7; f = x i i= i<j i= 여기서 8X r = x i = p q = i= 그런데코시 - 슈바르츠부등식에서 (x + + x 8 ) (x + + x 8)( + + ) p 8r = p 딱등호가성립하는경우임을알수있다. 등호조건은 x = x = = x 8 = 일때이므로 f ==8. } 풀이 (KAIST 과학영재센터연구원고봉균 ) 앞의풀이에서처럼 p, q, r을두면 X (x i x j ) =7r q =7p 6q =0 i<j 따라서, x = x = = x 8 = 8 p = 이고 f =( )8. } 37. 다음식을만족시키는 a, b에대하여 sin (a + b) 의값을기약분수 n 으로나타낼때,0m + n의값을구 m 하여라. ( 한국 004 차-S4) p p 6 sin a +sinb = ; cos a +cosb = 풀이 삼각함수의합차공식에서 sin a +sinb =sin a + b cos a +cosb =cos a + b cos a b cos a b p = p 6 = 따라서,tan a+b = p 3 이된다. t =tan x t 라할때sin x = +t 이되는공식을이용하면 sin(a + b) = =p p 3 3 +(=3) = 따라서,sin (a + b) = 3 4 이고,0m + n =43. } 38. 부등식 p a + p b c 를만족시키는모든실수 a, b ( 단,0 a; b; ) 에대하여, 다음연립방정식이실수해를갖도록하는실수 c의최대값은얼마인가? ( 한국 004 차-S5) ( a =( x)y b =( y)x 풀이개요에서 x<0, y<0 이면 b> 로곤란. 즉 x; y; y; x > 0. 그럼산술 - 기하평균부등식 p a + p b ( x)+y + ( y)+x 따라서, c 이고, 등호가성립하는경우가있으므로 이구하는최대값. } =

130 30 대수 별해 a b = y x, 즉 y = x + a b 이고, 이것을첫번째식에대입하면 이이차식이근을가져야하므로판별식 (x )(x + a b)+a =0 x +(a b )x + b =0 D =(a b ) 4b 0 이어야한다.0 a; b; 이므로 b 0, a b 0 이고, 따라서동치변형하면 이된다. 실제로 p a + p b = 일때, D =0 이고, D 0 () ja b j p b () a + b + p b () ( p b ) a () p b pa () p a + p b x = a + b + ; y = a b + 혹은 x = p b, y = p a 가연립방정식의해가된다. 따라서, c 의최대값은. } 39. 다음조건을만족시키는수열 fa n g 에대하여 a 00 + a a 004 의값을구하여라. ( 가 ) a = a = a 3 = a 4 =,a 5 =, ( 나 ) a n+5 = an + a n+ + a n+ + a n+3 + a n+4 a na n+ a n+ a n+3 a n+4 (n ) ( 한국 004 차 -S6) 풀이조건에따라처음몇항을구해보면다음과같다.,,,,, 6,,,,,, ::: 즉, 수열의시작부와같은,,,, 가다시나타나고, 이로부터수열은 6 개항을주기로하여계속같은꼴로반복된다. 따라서, a 00 + a a 004 = a 4 + a 5 + a 6 =++6=9. } 40. () a + b + c =0을만족하는임의의실수 a, b, c에대해, a 3 + b 3 + c 3 > 0 과 a 5 + b 5 + c 5 > 0 이동치임을증명하여라. () a+b+c+d =0을만족하는임의의실수 a, b, c, d에대해, a 3 +b 3 +c 3 +d 3 > 0 과 a 5 +b 5 +c 5 +d 5 > 0 이동치임을증명하여라. ( 영국 004 차-3) 증명 ( 대원외국어고 3학년최일규, 수정됨 ) () a = b + c 이므로 또, a 3 + b 3 + c 3 > 0 () b 3 + c 3 > a 3 =(b + c) 3 = b 3 + c 3 +3bc(b + c) () 0 >bc(b + c) = abc () abc > 0 a 5 + b 5 + c 5 > 0 () b 5 + c 5 > a 5 =(b + c) 5 () b 5 + c 5 >b 5 + c 5 +5bc(b 3 + c 3 )+0b c (b + c) () 0 > 5bc(b + c)(b bc + c +bc) = 5abc(b + bc + c ) () abc > 0 와같이둘다 abc > 0 과동치임을알수있다.

131 . 대수중급문제 3 () (a + b) = (c + d) 이므로 a 3 + b 3 + c 3 + d 3 > 0 () a 3 + b 3 > (c 3 + d 3 ) () (a + b)(a ab + b ) > (c + d)(c cd + d ) () (a + b)((a + b) 3ab) > (a + b)((c + d) 3cd) () (a + b)(ab cd) < 0 또, a 5 + b 5 + c 5 + d 5 > 0 () a 5 + b 5 > (c 5 + d 5 ) () (a + b)(a 4 + b 4 ab(a + b )+a b ) > (c + d)(c 4 + d 4 cd(c + d )+c d ) () (a + b)((a + b) 4 5ab(a + b + ab)) > (a + b)((c + d) 4 5cd(c + d + cd)) () (a + b)ab(a + ab + b ) < (a + b)cd(c + cd + d ) () (a + b)ab((a + b) ab) < (a + b)cd((c + d) cd) () (a + b)(ab cd)((a + b) (ab + cd)) < 0 () (a + b)(ab cd)((a + b) +(c + d) (ab + cd)) < 0 () (a + b)(ab cd)(a + b + c + d ) < 0 () (a + b)(ab cd) < 0 와같이둘다 (a + b)(ab cd) < 0 와동치임을알수있다. 4. 양수 a, b, c, d 가다음등식을만족한다. a 3 + b 3 + c 3 =3d 3 b 4 + c 4 + d 4 =3a 4 c 5 + d 5 + a 5 =3b 5 a = b = c = d 임을증명하여라. ( 폴란드 004 차 -) 증명 d가가장크면첫째식에서3d 3 = a 3 + b 3 + c 3 d 3 + d 3 + d 3 이고등호가성립해야하므로 a = b = c = d. d가가장작을때도마찬가지. a가가장크거나가장작을때도둘째식에서비슷하게하면같은결론. b가가장크거나가장작을때도셋째식에서마찬가지. 즉, c가가장크고동시에가장작을때만보면되는데, 그때는 a = b = c = d 일때니까볼거없음. 별해적당히 AM-GM 에의해,3d3 3abc, 4a 4 4abcd, 5b 5 5ab cd. 그럼 a 4 ;b 4 ;d 4 abcd c 4 이됨. a 가가장크다면두번째식에서 a = b = c = d, 다른경우도비슷비슷하게같은결론이나옴. 4. 자연수 n 과양의실수 a i 들에대해다음부등식이성립함을보여라. (ML 프로포절 4-) a n + an + + an n a n + a n + + a n n n a a a n 증명 a n ;an ;:::;an n 과 a ;a ;:::;a n 은서로크기순서가역순으로배열된수열이다. 따라서, 체비셰프부등식에서 a n + an + + an n n a + a + + a n an + a n + + a n n n n 이것으로문제의부등식이확인된다. 등호는 a = = a n 일때성립.

132 3 대수 43. p(x), q(x) 는최고차항의계수가 인임의의두정계수다항식이다. 무한히많은정수 x 에대해 p(x) 가 q(x) 의약수가된다면 p(x) 가 q(x) 의인수가됨을보여라. (ML 프로포절 45-) 증명 (KAIST 수학과 98학번김선해 ) p(x) 의차수가0이라면 p(x) =로당연. p(x) 의차수가 이상이라면, q(x) 를 p(x) 로나누자. 그러면, q(x) =f(x)p(x)+r(x) 이때, r(x) 의차수는 p(x) 보다작음. 따라서, r(x) lim jxj! p(x) =0 따라서, 충분히큰 jxj 에대해항상 jp(x)j > jr(x)j. 문제의조건에의해서, 무한히많은 x 에대하여, p(x) j r(x). 즉만일 r(x) 가 0 다항식이아니라면무한히많은 x 에대하여 jp(x)j jr(x)j. 이것은모순이므로 r(x) 는영다항식. 따라서, q(x) =f(x)p(x) 가되어문제가성립. 44. a; b; c 일때 ab + bc + ca 3 임을보여라. (ML 프로포절 47-) 증명 b, c를고정시켜두고 a만을변수로보면, f(a) =(b + c)a + bc 는 a에대한일차식이므로그최대최소는양끝점에서만확인하면된다. +f( ) = (b + c)+bc =( b)( c) 이고이것은 0 +f( ) 4 의범위에있다. 따라서,0 +f(a) 4, 즉 ab + bc + ca 3 이성립한다. 45. a>0, b>0, c>0, a + b>c, b + c>a, c + a>b 일때 a 3 + b 3 + c 3 +4abc a (b + c) b (a + c) c (b + a) > 0 을증명하여라. (ML 프로포절 59-) 증명 (KAIST 과학영재센터연구원고봉균 ) 일반성을잃지않고 a b c 라하자. 즉, 이제나머지항 a 3 + b 3 =(a + b)(a ab + b ) >c(a ab + b ): a 3 + b 3 + abc > ca + cb : c 3 +3abc ab + ac + ba + bc 만확인되면변변더해서끝난다. 그리고이것은두개의부등식 c 3 + abc ac + bc () abc ab + ba () 의합으로보면간단히확인된다.() 는 c a+b 라서쉽고,() 은 c +ab > ac+bc, 즉 (c a)(c b) 0 과동치라서참이다. 46. 서로다른세개의복소수 a와 b와 c에대하여, a 3 =0a +b +c a 340, b 3 =0b +c +a b 340, c 3 =0c + a + b c 340 일때, abc의값을구하여라. ( 한국 005 차-S) 풀이 ( 과천고 학년홍성준 ) a, b, c 의값이주어지면 a + b + c = K 의값도정해진다. 이때, a, b, c 는 3 차방정식 x 3 =9x x + K 340 의세근이된다. 근과계수와의관계에의해 a + b + c =9; ab+ bc + ca =; abc = K 340 여기서 K =(a + b + c) (ab + bc + ca) =359 이므로 abc = = 9 답 }

133 . 대수중급문제 a; b > 는실수이다. 다음을만족하는함수 f :(0; )! R 가존재할필요충분조건은 a>b 임을증명하여라 : (i) g(x) =f(a x ) x 로주어진함수 g : R! R 은증가함수이다. (ii) h(x) =f(b x ) x 로주어진함수 h : R! R 은감소함수이다. ( 루마니아 005 지역예선 y0-) 증명 ( 부산건국고 학년신승현 ) (() a>b일때 : a>c>b인 c를잡아서 f(x) =log c x 라두면 g(x) =f(a x ) x = x(log c a ) h(x) =f(b x ) x = x(log c b ) 은증가함수 은감소함수 따라서만족. ()) (i), (ii) 를만족하는 f 가존재한다면 : 임의의양수 x 에대해 g(x) >g(0) = h(0) >h(x) 이므로, f(a x ) >f(b x ) ( ) g(x) =f(a x ) x 가증가함수이므로 f(a x ) 도증가하고, 즉 f도증가함수. 그럼 ( ) 에서 a x >b x, 즉 a>b임을알수있다. 따라서증명끝. 48. 모두같지는않은음이아닌세실수 a, b, c 에대해다음을증명하여라. 3 a + b + c 3 3p a b c (a b) +(b c) ( 아일랜드 005-5) +(c a) 증명 3 3p a b c ab + bc + ca 이용하면 a + b + c 3 3p a b c a + b + c (ab + bc + ca) = ((a b) +(b c) +(c a) ) 이므로왼쪽부등식은 로강화할수있음. 별해로, 왼쪽부등식은 a + b + c +(ab + bc + ca) 9 3p a b c 의통째AM-GM 과동치로볼수도있음. 오른쪽부등식은 a + b + c +3 3p a b c (ab + bc + ca) 와동치인데, 이것은적당한치환으로슈르부등식 x 3 + y 3 + z 3 +3xyz xy(x + y)+yz(y + z)+zx(z + x) xy p xy + 로확인할수있음. 49. 실수 x, y, z 는 3x +y + z = 을만족시킨다고한다. +jxj + +jyj + +jzj 의최대값을서로소인자연수 p, q에대하여 q p 로표현할때 p + q의값을구하여라. ( 한국 006 차-S7) 풀이준식을잘관찰해보면분모가최대한작아야최대값이얻어짐을알수있고, x = 3, y = z =0 일때최대가된다는것을추측할수있다. 이렇게추측된부등식 +jxj + +jyj + +jzj 4 를증명해보자. 양변에 4( + jxj)( + jyj)( + jzj) 를곱하여전개하면 4(3 + jxj +jyj +jzj + jxyj + jyzj + jzxj) (jxyzj + jxyj + jyzj + jzxj + jxj + jyj + jzj +) 이된다. 우변으로넘겨서정리하면 3jxj +3jyj +3jzj +7jxyj +7jyzj +7jzxj +jxyzj 가된다. 그런데, =3x +y + z 3jxj +jyj + jzj 이므로위의부등식은성립하게된다. 이때의등호조건은 x =, y = z = 0 3 일때이다. 따라서, p + q =4+=5 답 }

134 34 대수 50. 실수 a ;:::;a 004 에대하여, 방정식 x x a 004 x a x + a x +=0 이 006 개의양의실근을가진다고할때, a 이취할수있는최대값을구하여라. ( 한국 006-) 풀이 ( 고양저동고 학년박은구 ) n =006 이라하고 n 개의양의실근을 c ;c ;:::;c n 이라하자. 근과계수와의관계에의해 c + c + + c n = n c c c n = 실근이양수들이므로산술 - 기하평균부등식에의하여 c + c + + c n n np c c c n 여기서양변이모두 이어서등호가성립하므로, 등호가성립할조건에의해 그럼근과계수와의관계에의해 c = c = = c n = a = A c + A c + + A c n = n (A = c c c n =) 따라서, a = n = 006 으로유일하다. } 주 ( 부산국제고 학년장진한 ) 단지최대값에만관심이있다면산술-기하평균부등식으로 s a = n n = n c c c n c c c n 즉, a n 이됨을확인하는것으로충분하다. 물론등호조건도확인해야한다. 이와같이풀때 c + c + + c n = n 의조건은불필요하다.. 대수고급문제. P (z) =z + az + b 는복소수 a, b를계수로갖고복소수 z에대해정의되는이차다항식이다. jzj =인모든복소수 z에대해jp (z)j =이된다고한다. a = b =0임을증명하여라. (Putnam 947-B4) 증명 ( 임준혁 ) P (z) =z + az + b 에서 z = x + yi, a = m + ni, b = k + li(x + y =) 라고놓자.( 단, x; y; m; n; k; l 은실수 ) P (z) =x y +xy i + mx ny +(nx + my)i + k + li =(x y + mx ny + k)+(xy + nx + my + l)i ) (x y + mx ny + k) +(xy + nx + my + l) = y! y 를하면 ( 단, y 6= 0 이라고하자.) (x y + mx + ny + k) +(xy + nx my + l) =

135 . 대수고급문제 35 두식을빼면 ( ) (x y + mx ny + k) +(xy + nx + my + l) (x y + mx + ny + k) ( xy + nx my + l) =0 이것은 ny(x y +mx +k)+(4xy +my)(nx +l) =0 로되고 (* a b =(a + b)(a b)) 4y 로나누면 n(x y + mx + k) =(x + m)(nx + l) 이되고, 전개하면 nx ny + nmx + nk =nx +xl + mnx + ml 이항하면 nx ny + nk ml =lx인데 x + y =이므로 n + nk + ml =lx 그런데좌변은상수이고우변에는변수 x가있으므로이것이항상성립하게하려면 l =0인수밖에없다. ) l =0 그러면 ml =0이므로 n + nk =0이다. 그러면 k =혹은 n =0이된다. () k = x! ;y! 0 에서 ( + m) + n =이고, x! ;y! 0 에서 ( m) + n =이다. 그러면 j+mj = j mj인데,+m = ( m) 일수없으므로+m = m이고 m =0 그러면 n = 3인데실수라는데에서모순. 따라서전체가모순이다. () m =0 x! 0;y! 에서 ( +k) + m = 이고, x! ;y! 0 에서 ( + m + k) = 이고, x! ;y! 0 에서 ( m + k) =이다. 그러면 +m + k; m + k가 일때의모든해를찾아보면(m; k) =(; ); (0; ); (0; 0) 그리고 ( ; ) 이다. 이것을 ( +k) + m =에대입해보면맞는해는오직 (0, 0) 밖에없다. 따라서 m = n = k = l =0이고 a = b =0이다. 증명 ( 장도한 ) ` 대수학의기본정리 ' 에의하여, P (z) =(z )(z ) 인복소수 ; 가존재하므로, jzj = 인모든복소수 z 에의하여,=jP (z)j = jz j jz j 이다. 이제, a = b =0 임을보이기위해서 =0; =0 임을보이면된다. 6= 0 또는 6= 0 이라가정하여모순됨을보이자. 일반성을잃지않고 6= 0 이라가정하고, 복소평면상의두점을 A( ), B( ) 이라하자. () 6= 0 인경우, \AOB 의이등분선과폐윤곽선 C = fz C : jzj =g 과의교점중에서 \AOB ¼ 인 AB 와의교점을 D(z 0 ) 이라하자. 그러면, jz 0 j = j j + jz 0 j j jjz 0 j cos(\aod) j j + jz 0 j > jz 0 j =(* 6= 0 이므로, j > 0 이다.) 즉, jz 0 j > 이다. 그리고, jz 0 j = j j + jz 0 j j jjz 0 j cos(\bod) j j + jz 0 j > jz 0 j =(* 6= 0 이므로, j j > 0 이다.) 즉, jz 0 j > 이다. 그러나이는 =jp (z 0 )j = jz 0 j jz 0 j > 가되어모순이다. () =0 인경우, AB 의연장선과폐윤곽선 C = fz C : jzj =g 과의교점을 E(z ) 이라하면, jz j > 이고, jz j = 이된다. 그러나이는 =jp (z )j = jz j jz j > 가되어모순이다. 주 ( 고봉균 ) 직선 AB와평행하고원점을지나는직선을 PQ(P; Q는단위원위의점 ), 그리고 AB와만나지않는호 PQ의중점을 D라하면A나 B는 D를중심으로하는단위원안에들어오지않는다고말해도되겠다.. a d = c 6=0일때, x 4 + ax 3 + bx + cx + d =0의네근은순서를적당히잘정의하면 r =r = r 3 =r 4 를만족함을보여라. (Putnam 953-B5) 증명네근을 p, q, r, s 라하고, 근과계수와의관계를 a d = c 에대입하면 pqrs(p + q + r + s) = (pqr +pqs+prs +qrs). 양변전개하고소거하여정리하면 pqrs(p +q +r +s )=(pqr) +(pqs) + (prs) +(qrs). 인수분해하면 (pq rs)(pr qs)(ps qr) =0. 3. 모든양의실수 w, x, y, z 에대해다음부등식이성립함을증명하여라. ( 헝가리 964-3) µ w + x + y + z µ wxy + wxz + wyz + xyz 3 4 4

136 36 대수 증명 LHS 6 P x =( ) 4 3 =( P x P P xyz x xyzw. 4 4 따라서, P x ) 4 P x xyzw P 4 x 4 4 P x. RHS 4 6 P xyz =( ) 4 P P xyz xyz = = 4 4 P xyz 4, 즉 4 P x P x P xyz 만보이면충분함. 우변은 다시 P xy+ P yzw x 와같고,3P x P xy ( 완전제곱들의합으로 ) 임과 P x P xyz x P yzw x ( 차례로완전제곱과재배열로 ) 임에서증명됨. 4. 임의의양수 a, b, c, d 에대해, 다음부등식이모두성립할수는없음을보여라 : a+b <c+d, (a+b)(c+d) < ab + cd, (a + b)cd < ab(c + d). ( 소련 969-4) 증명귀류법으로, 그런양수들이존재한다고하자. a + b = p, ab = q, c + d = r, cd = s 로치환하면문제의부등식들은 p<r, pr < q + s, ps < qr 이되고, 추가적으로양수해임에서 p; q; r; s > 0 임과 p 4q, r 4s 가성립함. p<r이므로 np < r (n +)p 인자연수n이존재함. 그럼 q + s>pr>np 4nq 로부터 s>(4n )q 이고, 다시 qr > ps > (4n )pq 로부터 r>(4n )p 가됨. 이것은 r (n +)p 와모순되므로끝. 5. n 3 은임의의정수이고, 합이 0 이고모두 0 은아닌세실수 a, b, c 가주어져있다. 다음연립방정식을만족하는순서쌍 (x ;x ;:::;x n) 을모두구하여라 : ax i + bx i + cx i+ 0 (i =; ;:::;n) 단, x n = x 0, x n+ = x 이다. ( 오스트리아 ) 풀이다더하면등호가성립. 따라서각각의식모두등호가성립해야함. axi + bx i =(a + b)x i+ 의식들을제곱하여변변다더하면 P abx i x i = P abx i+ 재배열부등식만남음. ab 6= 0이라면 x i들이모두같을때뿐. ac 6= 0일때나bc 6= 0일때도비슷하게됨 (a, b, c 중둘이상이 0일순없음 ). } 6. 수열 x 0 ;x ;x ;::: 와 y 0 ;y ;y ;::: 가다음과같이정의되어있다 : x 0 = y 0 =, 그리고 n =0; ; ;::: 에대해 xn + x n+ = x n + ; y n+ = y n + y n 모든 n =0; ; ;::: 에대해 y n = x n 임을보여라. ( 폴란드 ) 풀이 a n = x n p x p, b n = yn p n+ y p n+ 로치환하면 a 0 = b 0 = p + p = c 라할때a n+ = ca n, b n+ = b n 임을확인할수있어서 a n = c n+, b n = c n 이됨. } 풀이 x k+ n = ynx k+ x k +y n 임을추측할수있고 n 에대한귀납법으로증명할수있음. } 7. P (x; y) 는모든 x, y 에대해 P (x; y) =P (y; x) 를만족시키는 x, y 두변수에대한다항식이라하자.( 예를들어, 다항식 x xy + y 은이조건을만족시킨다.) (x y) 가 P (x; y) 의인수라할때,(x y) 이 P (x; y) 의인수가됨을보여라. ( 캐나다 976-7) 증명 P (x; y) =(x y)q(x; y) 라하자. P (x; y) =P (y; x) 라했으므로 (x y)q(x; y) =(y x)q(y; x) x = y 인경우를제외한거의모든 x, y에대해 Q(x; y) = Q(y; x) 가성립하므로, 항등정리에의해이둘은같은다항식이다. 그럼특히 Q(x; x) = Q(x; x), 즉 Q(x; x) =0 이다. Q(x; y) 를 y에대한식으로보고 (y x) 로나누어 Q(x; y) =(y x)r(x; y)+s(x) 라하자. 임의의 x에대해s(x) =Q(x; x) =0이므로 S(x) 는영다항식이다. 따라서, P (x; y) =(x y)q(x; y) = (x y) R(x; y) 로 (x y) 이 P (x; y) 의인수가된다.

137 . 대수고급문제 다음을만족시키는자연수쌍 (m; n) 을모두구하여라 : ( 미국 977-) ( + x n + x n + + x mn ) 이 ( + x + x + + x m ) 로나누어떨어진다. 풀이 xm+ j xn(m+) x x n, 즉 (x m+ )(x n ) j (x n(m+) )(x ) 과동치이다. m + 과 n이서로소이면 x m+ 과 x n 도최대공약식이 x 이되고x n(m+) 은 x m+ 과 x n 로모두나누어떨어지므로식이성립한다. m +과 n이서로소가아니면 d j m +;n 인 d 가존재하고, 그럼좌변에는 x d 이두번곱해져있어서근 e ¼i=d 를두번갖는데 ( 중근 ) 우변은한번만가지므로성립하지않는다. 답 m +과 n이서로소인모든 (m; n) } 9. 자연수 n 과실수 a 가주어져있다. 연립방정식 nx x k i = ak i= (k =; ;:::;n) 을만족하는모든실수해 (x ;:::;x n) 을구하여라. ( 오폴 979-6) 풀이 ( 김규완 )(x ;x ; ;x n) 는 x i = a; x j =0(j 6= i)(i =; ; ;n) 임을보이자 (*) ) n =일때본조건에의해 x = a: (*) 성립 ) n =일때 x + x = a; x + x = a = x + x +x x : 이를풀면 (*) 성립. 3) n>일때 (x + x + + x n )(x 3 + x3 + + x3 n ) (x + x + + x n ) (* 코시슈바르츠부등식 ) LHS = a 4 = RHS이므로등호조건이성립한다. ) x i 는 0이거나동일한값 b이다. a =0이면 b =0이어야하므로 (*) 성립. a 6= 0인경우를보자. x i = b인개수를 t라하자. x + x + + x n = bt = a에서 b = a t : x + x + + x n = b t = a 에서 t =을얻는다. ) b = a; (*) 가성립한다. ) (x ;x ; ;x n ) 의실수해는 x i = a; x j =0(j 6= i)(i =; ; ;n) } 주 x i 들중에음수가있으면위의 3) 과같이풀기곤란함. n 4 일때는코시부등식 (x + x + + x n)(x 4 + x x 4 n) (x 3 + x x 3 n) 의등호조건에서비슷하게해결할수있겠고, n =3 일때는따로풀어줘야겠음. 0. 수열 a ;a ;a 3 ;::: 의각항은모든 k, m 에대해서다음의다음의특성을갖는다. ja k+m a k a m j 모든자연수 k, m에대해서다음을보여라. ( 오폴 980-6) ak k a m < m k + m 증명 ( 신한솔 ) a n = f(n) 이라하자. 귀납법으로 (x ) f(xy) xf(y) x 임을증명하겠다. x =일때0 0 0이므로성립. x = a일때성립한다고하자. f(k + m) f(k) f(m) 에서 k = ay, m = y라하면 f((a +)y) f(ay) f(y). (a ) f(ay) af(y) a. 변변더하면 a f((a +)y) (a +)f(y) a. 따라서 x = a +일때에도성립한다. 따라서 (x ) f(xy) xf(y), f(xy) xf(y) x 이증명되었다.

138 38 대수 왼쪽부등식에 x = m, y = k 을대입하면 (m ) f(mk) mf(k). 오른쪽부등식에 x = k, y = m 을대입하면 f(km) kf(m) k. 따라서 m ++mf(k) f(mk) k +kf(m). mf(k) kf(m) m + k <m+ k. 양변을 mk 로나누면 k 에 m 을, m 에 k 를대입하면 따라서 ( m + k ) < f(k) k f(m) m f(k) k f(m) m f(k) k < f(k) k < m + k m < m + k ( m + k ) f(m) m f(m) m < m + k 이므로증명끝.. x, y, z, A, B, C 는실수이고, A + B + C 는 ¼ 의정수배일때 F r = x r sin ra + y r sin rb + z r sin rc 라하자. F = F =0 이면모든자연수 r 에대해 F r =0 임을증명하여라. ( 미국 980-3) 증명 a = xe ia = x(cos A + i sin A), b = ye ib, c = ze ic 라두자. 그럼 F r 은 a r + b r + c r =: X r 의허수부를의미한다. 즉, F r =0임을보이는것은 X r 이실수임을보이는것과동치이다. X r 은기본대칭식 p = a + b + c, q = ab + bc + ca, r = abc 에대한다항식으로나타나므로 p, q, r이모두실수임만보이면충분하다. 문제에서 X, X 는실수라고했으므로 a + b + c = X,(ab + bc + ca) =X X 는모두실수이고, abc = xyze i(a+b+c) 에서 A + B + C가 ¼의정수배이므로이것도역시실수이다.. 함수 f :[0; ]! [0; ] 에대해 f = f, f n+ (x) =f(f n (x)) 로정의하자 (0 x, n 은자연수 ). 적당한자연수 n 이존재하여, 서로다른모든 x; y [0; ] 에대해다음을만족한다고한다. jf n (x) f n (y)j < jx yj f(x 0 )=x 0 을만족하는 x 0 [0; ] 이유일하게존재함을증명하여라. ( 오폴 98-9) 증명먼저, 저런 x0 는두개있을수없음 (). 왜냐하면식에대입하면 ja bj < ja bj 가되니까. f n 은연속이므로 f n (x 0 )=x 0 인 x 0 가존재 ( 중간값정리 ). f(x 0 )=x 이라하면 f n (x )=x 이성립하게되고,() 에의해 x = x 0 일수밖에. 3. n은자연수이고 와 x는실수이다. 또 0 < < 이고 n+ x 이라하자. 다음을증명하여라. ( 호주 98-) (x ) (x ) (x 3 ) P (x) = (x + ) (x + ) (x + 3 ) (x n ) (x + n ) P () 증명 ( 김두형 ) a m >x a m+ 인 m은유일하게존재한다. 이때각각의단항에대하여음수일수도 있고, 양수일수도있으므로두그룹A; B로나누어생각하자. A : m x a k k= ( 이그룹은각각의단항이음수인단항들의곱 ) x + ak B : n x a k k=m+ ( 이그룹은각각의단항이음수가아닌단항들의곱 ) p(x) =AB이다. x + ak 일단 k m +인경우를보면 x ak x + a k ak +a k 이자명하다.( 계산해보면 ( x)ak 0이므로성립 한다.)

139 . 대수고급문제 39 이제 m x a k k= x + a k a k m k= 임을보이는것으로충분하다. +ak m이홀수라면좌변이음수이고우변은양수이므로자명 m이짝수일때를보자. 이때는좌변의각단항에` ' 를붙여주자 (m이짝수이므로부호는변동이없다.) 만약 am k x a m k + x ak+ 임을보일수있다면위식은자명해진다. +ak+ 그런데이식의양변에 (a m k + x)( + a k+ ) 을곱하여전개후정리하면 x a m+ 이나와성립하게된다. 즉, m이짝수일때와홀수일때를모두증명하였으므로준식은성립한다. 4. x 를양의실수, n 을자연수라할때, [nx] [x] + [x] + [3x] [nx] n 임을보여라. 단,[t] 는 t 를넘지않는가장큰큰정수를나타낸다. 예를들어,[¼] =3,[ p ]= 이다. ( 미국 98-5) 증명 x 대신 +x 를대입하면양변에서 n을빼내소거시켰을때다시원래의식이된다. 즉, 이식은 을주기로가지므로,0 x< 일때만보이면충분하다. x =0일때는0=0으로성립하고, x를연속적으로증가시키면서생각하면좌변과우변은모두증가함수인데우변은 x = a k 꼴 (a <k n 인서로소인자연수들 ) 일때만값이증가하므로그때만살펴보면충분하다. 소수부 fxg := x [x] 로정의하면 nx = x + x nx + + n 임에서다음의부등식이문제와동치가된다. fnxg fxg + fxg + + fnxg n ( ) a = q k + r,a = q k + r, :::; na = q nk + r n (0 r i <k) 라하자. 그럼 fmxg m = r m m 이므로 r n r + r + + rn n 을보이는것과같다. 그런데, a와 k가서로소임에서 r ;:::;r k 은 ;:::;k 의재배열이다. 왜냐하면우선 0인것은없고, r i = r j 라하면ia ja (mod k) 가되어kj (i j)a, 즉 k jji jj <k로모순이므로, r i 들은다다르기때문이다. 따라서, 재배열부등식의최소의경우에의해 r + r + + r k k + k + + k = k 이고, 이것만해도벌써 fnxg = r n k 이상이므로뒷부분의항을추가해도당연히 ( ) 은성립한다. 5. 이상의정수 n 에대해다음을증명하여라. ( 오폴 98-3) ny tan ¼ ny µ+ 3k 3 3 n = cot ¼ µ 3k 3 3 n k= k= ³ 증명 a k := tan ³+ ¼ 3k 3 3 n = tan ¼ 3 + 3k 3 n = ³ tan ³ ¼ 3k 3 3 n = tan ¼ 3 3k 3 n = p 3+xk p 3x k, 단 x k := tan 3k 3 n. b k := p 3 xk + p 3x k 라하면 Q n k= a k b k = 임을보이면충분함. tan 3배각공식에의해 x k+ = 3x k x 3 k 3x k 이므로 Q n k= a k b k = Q n x k+ k= tan( ¼ 3 n ) =의 telescoping이됨. = tan(¼+ 3 n ¼ ) x k nx 6. S n = p j;k= j + k 이라하자. 부등식 n S n Cn 이모든n 3에대해서만족하는자연수상수 C를찾아라.[ 주 ] C가작을수록더좋은답안이다. ( 오폴 98-9) 풀이 a j;k := p 으로나타내고, b j +k i := a ;i +a ;i + +a i ;i +a i;i +a i;i + +a i; +a i;, 즉, j와 k 중 i인것이있는항들의합이라고하자. 그럼 b i < (i ) p = i <. i i 따라서, S n = P n i= b i < n. 즉, 최소의자연수상수 C는 이다. }

140 40 대수 7. () 임의의자연수 x ;x ;:::;x k (k 4) 들에대해다음부등식을증명하여라. x + x x k + + x k + x x + x 3 x k + x () 우변의값 를그보다더큰어떤값으로바꾸어도이부등식은더이상성립하지않게됨을보여라. (Towns 98 봄 S) 증명 () k =4일때는 x +x 3 + x +x 4 x +x 4 x +x 3 로성립 (AM-GM). k일때성립한다고하고 k +일때증명하자. 윤환식이므로일반성을잃지않고 x k+ 이최소항이라할수있다. L k+ L k = x x k+ + x x x k + x + x k x k + x k+ x k x k + x + x k+ x k + x =: a b + c d + e 라하면 a>b, c>d, e>0 이므로 L k+ >L k 로성립한다. 등호는 k =4 일때만 (x + x 3 = x + x 4 일때 ) 성립할수있다. () 좌변의값을얼마든지 에가깝게할수있음을보이자.(() 의과정에서마지막항이늘앞선항들보다많이작아야덜늘어날것같다는점등에착안하여 (;a;a ; :::; a ;a 등의다른예도가능할수있다 )... 편의상순서는뒤집어서 ) 충분히큰상수 a 에대해 로잡아보자. 그럼 (x ;x ;:::;x k )=(a; a ;a 3 ;:::;a k 3 ;a k ;a k ;a k ) a L k = a k + a + a a + a a k a k 3 + a k + a k a k + a k + ak a k + a = a a k 3 +(k 3) + a +a + a +a + ak a k + 인데, a! 이면앞선두항은 0 으로가고뒤의두항은 로가므로 L k!. 즉, 얼마든지 에가깝게만들수있다. 8. 실수 x, y, z에대해s r = x r + y r + z r 이라하자. S =0이면 (m; n) =(; 3), (3; ), (; 5), (5; ) 에대해 S m+n m + n = Sm S n ( ) m n 임이알려져있다. x + y + z =0인모든실수x, y, z에대해 ( ) 이성립하는다른정수쌍 (m; n) 이더있다면모두구하여라. ( 미국 98-) 9. 모든양수 a; x ;x ;:::;x n (n ) 에대해서다음의부등식이성립함을증명하고, 등호가성립할조건을구하여라. ( 오폴 984-3) a x x + ax x3 + + axn x n x + x x + x 3 x n + x (x + + x n) 증명우변의분모를양변에곱하고코시로풀면 LHS (a (x x )= +a (x x 3 )= + +a (xn x)= ) (n(a ((x x )+(x x 3 )+ +(x n x )) )). 마지막은 AM-GM. 0. n> 개의음이아닌서로다른정수 a ;:::;a n 이주어졌을때, 다음을만족하는음이아닌정수들 y 와 x ;:::;x n 을모두찾아라 :gcd(x ;:::;x n )= 이고 a x + a x + + a n x n = yx a x + a 3 x + + a x n = yx a n x + a x + + a n x n = yx n ( 오폴 984-5)

141 . 대수고급문제 4 풀이 x i 들이모두 0일순없으므로 (* gcd = ) x + + x n > 0. 준식을모두다변변더하고 x + + x n 을소거하면 a + + a n = y. x i 들중에x m 이최소라면 m번째식에서 yx m a m x + + a m x n (a + + a n )x m = yx m. 등호가성립하는경우이므로앞의계수 a i 가 0인항을제외하고는모든 x j 들이 x m 과같아야함. a i 들은서로다르다고했으므로 0인것은많아야하나. 즉, 최소가아닌항은많아야하나뿐임. 마찬가지로최대가아닌항도많아야하나뿐임. n 3 이므로이로부터 ( 최소가아닌항이하나있다면그항이최대인데최대가아닌 ( 최소인 ) 항이둘이상있으므로모순 ) 모든항이같아야함을알수있다.gcd=이므로 x = = x n = 답 }. 모든 x; y Q 에대해다음을만족하는함수 f : Q! R 를모두찾아라. ( 오폴 984-9) f(x + y) =f(x)f(y) f(xy)+ 풀이 x = y =0대입해보면 f(0) =. x = y =대입해보면 f()f( ) = f( ). f() = 이면 y =대입하면 f(x +)=이므로항상 인상수함수 ( 그리고, 이것은문제를만족 ). 이제 f( ) = 0 일때만보면된다. g(x) :=f(x) 로치환하자. 그럼 g(0) = 0, g( ) = 이고, 문제의식은다음과같이바뀐다. g(x + y) = g(x)+ g(y)+ g(x)g(y) g(xy) (x; y) Ã (x; ) 과 (x ; ) 을각각대입하면 g(x ) = g( x), g(x) =g()( + g(x )) 이되고 ( ), 따라서 g(x) = g()g( x) ( ). 이식의x 대신 x를대입하면 g( x) = g()g(x) 이므로두식을변변곱하면 g(x)g( x) =g() g(x)g( x). x =을대입하면 g() 3 = g(). () g() = 0 일때 : ( ) 에서 g 0 인데이것은 g( ) = 에모순. () g() = 일때 : ( ) 에서 g(x) +g(x ) = 이므로 g( 홀 )=, g( 짝 )=0이됨. 또,( ) 에서 g( x) =g(x). 이두식에서 g( )=g( ), g( )+g( )= 이므로 g( )=. 다시 g(x)+g(x ) = 임에서 g( 정수 + )=. (x; y) =(; ) 을대입하면 =0 +0 로모순. (3) g() = 일때 : ( ) 과 ( ) 에서 g(x) =+g(x ), g( x) = g(x). 즉, 우선 n이정수일때 g(n) =n. y = n 을대입하면 g(nx) =ng(x). 여기에 x = n 을대입하면g( n )= n. 다시 x = m 을대입하면 g( n m )= n m 이된다. 따라서, 모든유리수 r에대해g(r) =r 이고이것은조건을항상성립시킴. 답, x + }. P (x) 는다음을만족하는 3n차다항식이다. 이때, n을구하여라. ( 미국 984-5) P (0) = P (3) = = P (3n) = P () = P (4) = = P (3n ) = P () = P (5) = = P (3n ) = 0 P (3n +)= 다음을만족하는실계수다항식 f(x) 를모두구하여라. ( 호주 985-6) f(x)f(x +)=f(x + x +) 4. f() = 이고모든 x R 에대해 f(f(x)) = f(x) 인연속인단조함수 f : R! R 를모두찾아라. ( 오폴 986-9) 풀이치역을 A 라하자. y A 이면 f(y) =y 이성립 ( 그리고, 이것은문제의조건 f(f(x)) = f(x) 과동치 ) 하고, 다시 y A. 이미 A 임을알고있으므로, f의연속성에서 a A 이면 [;a]or [a; ] ½ A 가된다 ( ). (a) 만일 A 에 보다큰원소가있다면 :+e A 라하면,+e<( + e) A 이므로 +e A. 따라서, 얼마든지큰수 + k e 가 A 의원소가되고, 그럼 A 의연속성에의해 이상의모든실수 가 A 에속한다. (b) 만일 A 에 0 과 사이의원소가있다면 : +e A 라하면, +e > (+e) A 이므로 +e A. 위와마찬가지로얼마든지작은수 + k 가 A의원소이므로 (0; ] ½ A. 그럼또연속성에의해 e 0도 A의원소가됨. (c) 만일 A에음수가있다면 : ( ) 에의해 e; 0;e A 인 e>0 가존재. 그럼 f( e) =e >f(0) = 0 <f(e) =e 으로단조성에모순. 따라서, A에음수는포함될수없다.

142 4 대수 ( 0 (x<0) 이제 (a) 와 (b) 의성립여부에따라서다음과같이함수가결정된다 :(ab)f(x) = x (x 0). (a¹ b) 8 ( (x<) >< 0 (x<0) f(x) = x.(¹ab) f(x) = x (x ) >: (0 x ).(¹a b) ¹ f(x) =. 답 } (x>) 5. x ;x ;:::;x n 은 0 <x <x < <x n < 이고 x 0 =0,x n+ =인실수라하자. 이수들이연립방정식 n+ X j=0 j6=i x i x j =0 (i =;:::;n) 을만족한다고가정하자. x n+ i = x i (i =;:::;n) 임을보여라. (IMO-LL 986, 프랑스출제 ) 풀이 ( 서울경복고김범석 ) 주어진연립방정식을풀면 x + x + x n + x x + x x +. x n x + x x x x 의결과를이용하여양변을모두더해정리하면 nx i=. x n x + + x x n+ =0 x x n +. x i x j = x j x i nx i= x i + nx i= x n x n + x i =0 을역순으로이항하고 x i x i = 을이용하여정리하면 x i nx i= x i = x x n+ =0 x n x n+ =0 nx µ n X nx = = x i= i x i=0 n i x i= n i+ ) x i = x n i+ (i =; ::: ; n) ) x n i+ = x n+ i = x i (i =; ::: ; n)( 증명끝 ) } 6. n 개의실수 a a a n 에대해 M = nx a n i= X M = a i a j n(n ) i<j n q Q = M M 라정의한다. a M Q M + Q a n 이성립함과, 등호는 a = a = = a n 일때또그때에만성립함을보여라. (IMO-LL 986)

143 . 대수고급문제 43 주아래의풀이는서울대수학과김영훈이제출하였다. 단계 M M 임을증명하자. By Schwartz's inequality n X (a i ) (X a i ) 0: M M = = = = ³X X ai ai n a j n(n ) ½ ³ X X ¾ ai a i ³X ai n n(n ) µ n ³ X X ai + a n(n ) n(n ) i ½n X ³X a ai ¾ i 0: n (n ) } X 단계 Let M ;k = (ai a k )(a j a k )Then n(n ) q Q = M M q = (M a k ) M ;k 이것을증명하자. M ;k = X (ai a k )(a j a k ) n(n ) = X ai a j a X k X (ai + a j )+ a n(n ) n(n ) n(n ) k = M a X k ai + a k n = M M a k + a k q Q = M M q = M a km + a k (M a k M + a k ) q = (M a k ) M ;k } 단계3 M a Q 임을증명하자. Let k =intheabove. Sincefa ng is increasing, a i a, a j a arepositiveandthereforesoare M a and M ;.Hence q Q = (M a ) M ; M a } 단계 4 M + Q a n 임을증명하자. Repeat the proof of step 3 with k = n. ByStep»4, we are done. } 7. a ;a ;:::;a n 이임의의실수들일때, 다음두문장이동치임을증명하여라. (A) 서로다른임의의 i, j 에대해 a i + a j 0. (B) x ;x ;:::;x n 이 x + x + + x n = 을만족하는임의의음아닌실수들일때, a x + a x + + a nx n a x + a x + + anx n 이항상성립한다. ( 중국 986-)

144 44 대수 증명 ( 임준혁 ) A가아니면B가아님을증명하겠다. 그러면 a i + a j < 0인 i; j가존재하고, 그때x i = x j =0:5로놓으면 B가성립하지않는다. A면 B을증명하겠다. a i 들중에서음수인것은딱하나이다.( 둘이상이면, 그둘의합이음수이므로모순 ) 그러면 a i 들중에음수가없을때는 x k x k을이용하면, 문제가자명해지고, 그렇지않을때일반성을잃지않고, a < 0 이라놓자. 좌변 우변 = a (x x )+a (x x )+ + a n(x n x n) 에서 a a ;a 3 ;:::;a n 을이용하면 a (x x )+a (x x )+ + a n(x n x n) a (x x ) a (x + + x n x x n) 이성립하고, x + + x n = x, x + + x n ( x ) 을이용하면, a (x x ) a (x + + x n x x n) = a (x x + x + + x n) a (x x +( x ) )(* a < 0) =0 8. z ;z ;:::;z n 은 jz j + jz j + + jz nj =을만족하는임의의복소수들이다. z i 들중몇개를골라합한것을 S라하자. jsj 6 이되도록고를수있음을증명하여라. ( 중국 986-3) 증명 ( 장도한 ) (k )¼ T k = fz i j 3 U 3 =이므로, U ;U ;U 3 중적어도하나는 3 이상이다. 이제일반성을잃지않고 U 3 = P j arg z< k¼ 3 g, k =; ; 3 이라두고, U k = P z i T k jz i j( 0) 라두면, U + U + 이라하자. k= z i k 이라하 P ij P ij 사이의각을 µ j 라두고, ¾ j = jz ij j cos µ j = 그리고 T 의원소들을재배열하여 T = fz ij j j mg 이라두고! OP ij 자. 또한, 복소평면의편각이 ¼! 3 인동경과 jj P! ij P ij jj cos µ j 이라두면,0 µ ¼ 3 이므로, ¾ j jz i j j이다. 따라서, S = jj! OP im = j mx mx z ij j j= j= ¾ j mx j= jz ij j U 6 이다. 아래그림은위증명의직관적인이해를돕기위해그린것입니다. 증명 ( 박민재 )

145 . 대수고급문제 45 실수 x k ;y k 에대해서 z k = x k + iy k 라고두자. 삼각부등식에의해 = X jz k j X jx k j + jy k j = X jx k 중양수인것 j + X jx k 중음수인것 j + X jy k 중양수인것 j + X jy k 중음수인것 j 비둘기집의원리에의해 4 개중하나는 4 보다크거나같음. 그것들을 a k 라고하면 j X q z k j = ( X x k ) +( X y k ) j X a k j = X ja k j 4 > 6 여기서 j P a k j = P ja k j 인이유는모두음수이거나모두양수이기때문에. 9. x ;:::;x n 이모두양수일때, 다음을증명하여라. ( 오스트리아 TST 987) 풀이 s p = nx k= v nx ux x k + t n x k n + p n n k= k= x p k이라고하자. 그러면주어진부등식은 Ã X n!ã n! X x k x k= k k= s + p µ p n + n s n s s 가된다.,, 는 > 0 와 >0 를만족하는상수라하자. 지수평균부등식에의해서 ³ s n ³ s n (5) ³ s n ³ s n (6) 이다. 결국 (), () 에의해 ³ s ³ s n n ³ s ³ s n n 가성립한다.(3) 식에 n 를곱해서두식을더하면 ³ s ; n (7) ³ s n (8) (s ) +(s ) n + n (s ) n + (s ) 를얻을수있다. 이문제는 =, = = 인특별한경우이다. } 30. 모든실수 x에대해f(x +)=f(x)+ 을만족하는함수 f : R! R 이있다. 주어진 a R 에대해, 수열 (x n) 은 x 0 = a, x n+ = f(x n)(n 0) 으로정의된다. x m x 0 이정수가되는자연수 m이존재한 x n 다면, 극한 lim 이존재함을증명하고그값을구하여라. ( 오폴 987-3) n! n 풀이 f(x + k) =f(x) +k임은쉽게알수있다. k := x m x 0 으로두었을때 x n+m = x n + k (n =0; ; ;:::) 가성립함을수학적귀납법에의하여증명하자. n =0인경우는 m과 k의가정, 그리고 x (n+)+m = f(x n+m )=f(x n + k) =f(x n )+k = x n+ + k

146 46 대수 에의해성립한다. µ n t 만약 n t (mod m), 0 t m 이면 x = x n + k임을증명하자. m 만약 n t (mod m) 이면 n = n t m + t이다. 따라서 m x n = x t+m n t m = x ³ t+m n t + k m = x ³ t+m n t +k m = µ n k = x t + k m 그러므로 n t (mod m), 0 t m 일때, x n n = x t n + µ n k n k 이다. 따라서, m x n lim n! n = k m } 3. S 는다항식들의집합으로다음의규칙에의해귀납적으로정의되는것이다. (i) x S (ii) f(x) S 이면 x f(x) S, x +( x)f(x) S S 의서로다른어떤두다항식도그그래프가구간 0 <x< 에서만나지않음을증명하여라. ( 미국 987-3) 증명 (ii) 의과정을 k번적용한함수를 `레벨 k의함수 ' 라부르기로하자. 즉, x는레벨0의함수. 우선모든함수f(x) S 는 0 <x< 에서 0 <f(x) < 임을금방알수있다 ( 귀납법. 레벨 0에서성립하고, 레벨 k에서성립한다면레벨 k +에서도성립 )( ). f(x)! xf(x) 로만드는과정을변환 A, 또 f(x)! x +( x)f(x) 로만드는과정을변환B라하면,( ) 에의해항상xf(x) <x<x+( x)f(x) 이성립하므로변환 A를마지막으로적용한함수와변환 B를마지막으로적용한함수는서로만날수가없고, 변환이한번이라도이루어진함수는레벨0의함수인x와도만날수가없다. 또, 마지막으로적용한변환이같은함수끼리만나는경우가있다면그직전의레벨에서도만나야하므로 (af (a) =af (a) 이면 f (a) =f (a). a +( a)f (a) =a +( a)f (a) 일때도f (a) =f (a)), 귀납적으로둘의레벨을계속거슬러추적하다보면언젠가서로다른변환이이루어졌거나하나가레벨 0에이르게되어서로만날수가없음을알수있다. 3. a [0; ] 인모든 a 와임의의자연수 n 에대하여 a k ( a) < (n +) 3 이되는최소의자연수 k 를구하여라. (IMO-LL 987, 유고출제 ) 풀이 f(a) =a k ( a) n 이라하고미분하면, (n +) 3 f 0 (a) =ka k ( a) n na k ( a) n = a k ( a) n fk (n + k)ag

147 . 대수고급문제 47 이므로 f 는 a = k µ k 에서최대값을갖는다. 따라서문제의조건과 f < 0 은동치이다. n + k n + k µ k f < 0 $ n + k µ k k µ n n n + k n + k (n +) 3 < 0 $ (n + k) n+k k k n n (n +) 3 > 0 (n + k)n+k $ k k n n (n +) 3 > ( ) (n + k) n+k $ log k k n n (n +) 3 > 0 ½ (n + k) n+k ¾ g k (n) = log k k n n (n +) 3 이라하자. 그런데 g () < 0;g () < 0;g 3 (3) < 0이므로 k 4이어야지문제의조건에맞는다. [ g 4 (n) > 0의증명 µ ] g4 0 (n) =log n n + 에서 n 이면 g0 4 (n) > 0이므로 n =일때g 4(n) 은최소값을가지며 µ 5 5 g 4 () = log > 0 [ 답 ] k =4 } 33. x>0 일때, 다음을증명하여라. ( 통신강좌 988-B5) µ + µ x + x + x < x+ 풀이면 ( 물리전공 년이현우 ) 위식을 () 이라하고동치인식을구해보자.() 의양변에 x+ 를곱하 () 의양변에 x 승을하고 x+ 를곱하면, µ x+ + µ x+ x+ x x+ x < x+ < x+ () ( x+ ) x+ < ( x+ ) x x+ µ ( x+ ) x ( x+ ) < x+ x x+ Ã! x x+ µ x+ x+ < x+ (3) 을얻게된다. 여기서 Ã x+ x+! x µ < ; x+ > x+ 이므로식 (3) 이성립하게된다. 위의결론으로부터 µ + µ x + x + x < x+ 이성립함을알수있다. }

148 48 대수 34. 함수 F 는 F :[0; ]! R 인연속함수이며다음조건을만족한다. (i) F (x) =bf (x) (0 x ) (ii) F (x) =b +( b)f (x ) ( x ) 여기서 b = +c, c>0 이라하면모든 x (0; ) 에대하여0 <F(x) x<c임을증명하여라. +c ( 통신강좌 988-B7) 증명 ( 경기과학고 년김태희 ) ㄱ ) x =0일때 i) 에서 F (0) = bf (0) (b )F (0) = 0: b = +c +c ;c>0이므로 <b< 이다. ) b 6= 0: ) F (0) = 0 (4) ㄴ ) x = 일때 ii) 에서 F () = b +( b)f ():bf () = b; b 6= 0 이므로 F () = (5) ㄷ ) x = 일때 i) 에서 F ( )=bf () = b =b: F ( )=b (6) 이제함수 y = F (x) 와 y = x를생각하자. i) 에서 0 x 일때y ;y 의교점을알아보면 F (x) =x: ) bf (x) =bx: F (x) =bf (x) 이므로 x =bx:(b ) = 0: <b<이므로 b 6= 0이고따라서 x =0: 0 x< 일때y 과 y 는 x =0에서교점을가진다. ii) 에서 x 일때 y ;y 의교점을알아보면 F (x) =x: F (x) =b +( b)f (x ) 이므로 b +( b)f (x ) = b +( b)(x ) x = b +( b)(x ): (b )x = b : b 6= 0이므로x =: x 일때, y 과 y 은 x =에서교점을가진다. 0 x 에서 F (x) 과 x는 x =0;x =에서교점을가지며 0 <x<에서는교점을가지지않고 x = 에서 F ( )=b> 이고 F (x) 가연속이므로부등식 F (x) >x (a) 가성립한다. 이번에는함수 y 와 y 3 의교점을생각해보자. F (x) =bf (x); ) bf (x) =b(x + c) F (x) =bf (x) 이므로x + c = b(x + c): ) (b )x = c(b ): ) x = b = +c +c 이므로 c(b ) b

149 . 대수고급문제 49 x = c(b ) b = ii) 에서 x 일때, F (x) =x + c, µ +c c +c µ = : +c +c ) 0 x< 에서 y 과 y 3 의교점은없다. ) b +( b)f (x ) = b +( b)(x +c) ) x + c = b +( b)(x +c): (b )x = b bc µ µ +c +c c b bc +c +c x = = µ b +c +c = c +c c +c = c <0 ) x 에서 y 과 y 3 의교점은없다. f(0) = 0 < 0+C이고 0 <x<에서 F (x) 와 x + c는교점이없고연속이므로부등식 F (x) <x+ c (b) 이성립한다. (a), (b) 에서 0 <F(x) x<c임을알수있다. 35. x ;x ;:::;x n 은 x x x n > 0 인수이다.0 a 일때, ( + x + x + + x n ) a +x a + a x a + + n a x a n 이성립함을보여라. ( 통신강좌 988-B) 증명 ( 환일고 년이상엽 ) F n(a) = +xa +a x a + + na x a ( + x + + x n ) a µ a µ x a = + +x + + x n +x + + x n + µ x a + + µ nx a n +x + + x n n + + x n 이라놓자. F n (a) 을 a에관해서미분하면 µ Fn 0 (a) = a log +x + + x n +x + + x n µ x a x + log +x + + x n +x + + x n + µ x a x log + +x + + x n +x + + x n + µ nx a n nx n log < 0 n +x + + x n +x + + x n ix i ( x x x n > 0이므로 < 이고 µ +x + + x n ix a i > 0(i =; ;::: ;n) 이므로 i +x + + x n

150 50 대수 µ ix i i + + x n a log ix i + + x n < 0 이다.) ) F 0 n(a) < 0: 그러므로 F n (a) 는단조감소함수이다. F n () = 이므로 F n (a) 는 0 a 에서 F n (a). 곧 ( + x + x + + x n) a > 0 이므로 +x a +a x a + + na x a n ( + x + x + + x n ) a ( + x + x + + x n ) a +x a + a x a + + n a x a n 36. F (x) =ax + bx + c 와 G(x) =cx + bx + a 는 jf (0)j ; jf ()j ; jf ( )j 을만족하는함수이다. jxj 에대하여다음식이성립함을보여라.(i)jF(x)j 5. (ii) jg(x)j. 4 ( 통신강좌 988-B3) 증명 F (x) =Ax(x +)+Bx(x ) + C(x ) 의꼴로써보자. 그러면 A = F ();B = F ( );C = F (0) 이다. 따라서 jf (x) j j Ajjx(x +)j + j Bjjx(x ) j + j Cjjx j j x(x +)j + j x(x +)j + j x j = 8 >< 이므로 j F (x) j 5 4 이다. G(x) =x F ( x ) 이므로 >: µ 5 4 x + x 0 µ 5 4 x 0 <x j G(x) j j +x j + j x j + j x j= x 이되고 jg(x)j 가된다 37. 어떤항을택해도그수가나머지항들의역수의합이되는길이 n 의실수순서쌍을모두찾아라. ( 루마니아 TST 988-B) 풀이 ( 강성경 ) 우선전체역수의합을 s라하고임의의항을 a라하면a +( )=s a 이므로 a의부호는 s의부호와같다. 즉, 모든항의부호가같다. 또, 모든항에 을곱해도문제는동치가되므로모든항이양수라고가정해도일반성을잃지않는다. i) n =: 모든서로역수관계인두실수. ii) n>: 그 n개의실수를각각 a ;::: ;a n 이라고할때, 다음이성립한다 : a = a + + a n. a n = a + + a n

151 . 대수고급문제 5 모두더하면 a + + a n =(n )( a + + a n ) 그런데, a =. a n = + + a a n a + + a n (n ) (* 산술 조화 ) + + a + + a n (n ) a a (* 산술 조화 ) n 모두더하면 + + (n )(a + + a n a a n (n ) = a + + a n (n ) 즉, a + + a n (n )( + + ). ( a a n 등호조건 : a = = a n) 이때, 식에의해등호조건이성립해야하므로, a = = a n = a이다. 그런데, a = + + a a n 여야하므로, a = n a 다. 즉, a = p n. 따라서, 구하는수열은 p n ; p n ;:::; p n 이다. } 풀이 ( 김규완 ) 역수들의합을 s라하자. 임의의항 a에대해s = a a 가성립한다. 이를풀면 p p p a = s s 4. ( 두근이서로역수관계 ) s+ s 4 가 x개있다고하자. 그럼 s s 4 는 n x개. (jsj =라서중근일때도이렇게두어도상관없다 ) case p x 6= 0,n x p 6= 0 p p s+ s 4 에대해 s+ s 4 = s s 4 (x ) + s+ s 4 (n x) 이성립. 정리하면 s p s 4 (x ) + +p s 4 (n x ) = 0 p p p x, n x 0, s+ s 4 와 s s 4 의부호가같으므로, x =n x =0(* s+ s 4 = p s s 4 6= 0). 이를풀면 n =4,x =. case x =0 또는 n x =0 a (n ) = a 가성립하므로 a = p n. ) 모든항이 p n 인실수쌍.( 단, n = 이면모든서로역수관계에있는실수쌍이답이될수있다.) } 38. 임의의자연수 n 과임의의 0 x 에대해다음부등식이성립함을증명하여라. ( + x) n ( x) n +nx( x ) (n )= ( 아일랜드 988-0) 증명 ( x) n 이항하여좌변 a n b n =(a b)(a n + + b n ) 꼴로인수분해. 양변에서 x 소거하고 (x =0일때는당연 ), n항짜리산술-기하평균부등식으로확인하면됨. 별증 ( x) n 이항하여양변제곱하고, 좌변에서나오는 ( x ) n 은우변으로다시넘겨주고, 양변을 ( x) n ( + x) n 으로나눠준후, 좌변을 ( + ) n +( ) n 꼴로변형하여이항정리로 차항까지전개해준다. 39. P (z) =z n +c z n +c z n + +c n 은복소변수 z 와실계수 c k 들로이루어진다항식이다. jp (i)j < 이라하자. P (a + bi) =0 이고 (a + b +) < 4b + 인실수 a, b 가존재함을증명하여라. ( 미국 989-3)

152 5 대수 증명근들을 r ;:::;r n 이라하면, P (z) =(z r ) (z r n ) 이므로 jp (i)j = ji r j ji r n j <. 임의의실근 r에대해ji rj = p Y +r 이므로, 모든근이실근이면 jp (i)j 이되어모순. 따라서, 허근들이존재하고, ji rj < 이다. 실계수다항식이므로허근들은켤레를이루며, 따라 r: 허근서 ji rjji ¹rj < 인허근r이존재한다. 이허근을 r = a + bi 로두면P (r) =0임은당연하고, > j(i r)(i ¹r)j = j(r¹r ) i(r +¹r)j =(r¹r ) +(r +¹r) =(a + b ) +4a. 이것은 (a + b +) < 4b +과동치. 40. f 는실수에서실수로의함수이고 g 는 f 의역함수이다. () f(x) 는단조증가. () f(x)+g(x) =x 가모든실수x에대해성립. 를만족하는 f 를모두구하여라. (APMO 989-5) 풀이적당한상수 c에대해f(x) x + c 이면 g(x) x c 이고조건을잘만족한다. 이것말고는 f가더없음을보이자. 만일그런 f가존재한다면 f(a) =a + c; f(b) =b + d 인 a, b, c, d (c 6= d) 가존재한다. f(a + c) =(a + c) g(a + c) =(a + c) a = a +c, 귀납적으로 f(a + nc) =a +(n +)c; 마찬가지로 f(b + nd) =b +(n +)d; (n Z) 가된다. 일반성을잃지않고 c>0 이라하자 (c; d 중에 0 이아닌것이적어도하나있고, c<0 일때는 f 대신대칭적으로 g 에대해생각하면된다 ). a + mc<b<a+(m +)c 인 m Z 을찾을수있고, f 가단조증가이므로각변에 f 를취하면 a +(m +)c<b+ d<a+(m +)c, 귀납적으로 a +(m + k)c <b+ kd < a +(m + k +)c (k =0; ; ;:::) 가된다. 즉,< b+kd a+(m+k)c < + c a+(m+k)c 이고, k!을취하면 d = c 임을알수있다. 이것은 c 6= d 임에모순. 따라서, f(x) x + c 뿐이다. } 4. 복소수집합 C 에서정의되는함수 g : C! C 와복소수 a 가주어져있고,! 3 = 인 이아닌복소수를! 라하자. 임의의복소수 z 에대하여 f(z)+f(!z + a) =g(z) 를만족시키는함수 f : C! C 가유일하게존재함을보여라. (IMO-SL 989-0, 그리스출제 ) 증명 z 에!z + a 를대입하면 (! +=! 임을이용하여정리 ) 여기에같은대입을한번더반복하면 f(!z + a)+f(! (z a)) = g(!z + a) f(! (z a)) + f(z) =g(! (z a)) 따라서, 이세식을연립하면 f(z) = (g(z)+g(! (z a)) g(!z + a)) 로유일하게존재. 4. x + x + + x n = 인임의의양수 x ;x ;:::;x n 에대해다음을증명하여라. x p x + x p x + + x n p xn p x + p x + + p x n p n ( 중국 989-) P p 증명코시로 ( xi ) P P q x i = n 이므로좌변 n n 임을보이면충분. x i = ( x i ) X xi 임을이용. p = X p X p xi =: A B 라하자. 역시코시로 AB n 이 xi xi 고, 또 B P P ( x i )=n(n ). 따라서, B pn(n ) 이고이것을 AB n q q 에대입하면 A n n n. 따라서, A B n n.

153 . 대수고급문제 53 별증 x ;x ;::: 와 p x ; p x ;::: 는크기순서가같은수열이므로재배열부등식을생각할 x 수있음. p xi i 의분자x i 대신 x i+ ;x i+ ;:::;x i 을한번씩바꿔넣은부등식들을모두다합하면 (n )L P p xi x i = P p x i 가됨 (L은문제의좌변 ). 이식의양변에L을곱하면코시부등식으로원하는결과가나옴. 43. S 는 보다큰모든실수들의집합이다. 모든실수 x; y > 과모든실수 m; n > 0 에대해 f(x m y n ) f(x) 4m f(y) 4n 을만족하는함수 f : S! S 를모두구하여라. ( 중국 989-6) 풀이 x m = y n = x, 즉 m =, n = log y x 를대입하면, f(x) f(x) f(y) log y x, 즉 f(x) ln x f(y) ln y. 대칭적으로반대방향의부등호도얻을수있고, 따라서 f(x) ln x = c 의상수함수. 즉,보다큰적당한상수 c에대해f(x) =c ln x 가되고, 이런함수는문제의조건을항상성림. ( 대입해보면 ln x, lny들이등장하는 AM-HM 부등식이되어성립함 ). } 44. 다음을만족하는함수 f : R +! R + (R + = f x j x>0 g) 를모두구하여라. i) f(xf(y)) = yf(x). ii) lim f(x) =0. ( 통신강좌 ) x! 풀이임의의 x0 > 0 에대하여y 0 = f(x 0 ) 라두면f(x 0f(y 0 )) = y 0 f(x 0 )=이므로 은치역에있다. 그러므로적당한 y>0가존재하여 f(y) =이된다. f() = f( ) = f( f(y)) = yf(), f() > 0 에서 y =이되어f() = 을얻는다. 임의의 x>0 에대하여 f(xf(x)) = xf(x) 이므로 F = f z>0jf(z) =z g 라두면xf(x) F 이다. a; b F 이면, ab = ba = bf(a) =f(af(b)) = f(ab) 에서 ab F 이다. 따라서 F 는곱에의하여닫혀있다. 특히 a F 이면 a n F. 그럼조건 ii) 에의하여 <af 인 a는존재할수없다. xf(x) F 이므로 xf(x), 즉 f(x) x. a = xf(x) F 에서 f( a f(a)) = f() = = af( a ) 이므로 f( a )= a, f( xf(x) )= xf(x) 이므로 xf(x) 도 F 에속한다. xf(x) F 에서 xf(x), x f(x). 따라서 f(x) = x. } 주 f(a) =a 인원소a를함수f의부동점이라한다. 따라서위의집합 F 는함수f의부동점들의집합이다. 45. 수열 a 0 ;a ;a ;::: 와 b 0 ;b ;b ;::: 가다음과같이정의되어있다. a 0 = p ; an = b 0 =; b n = p r q +b n q a n n =; ; 3;::: b n n =; ; 3;::: 모든자연수 n =; ; 3;::: 에대해서다음부등식이성립함을증명하여라. n+ a n <¼< n+ b n ( 통신강좌 ) 풀이 a n =sin n 이라하면 a 0 =sin 0 = p r q a n = sin n = p p. 고로 0 = ¼ 4. p cos n =sin n =sin n 즉 n = n 이고 n =( )n 0 = ¼ n+. b n =tan n 이라하면 b 0 =tan 0 =. 즉 0 = ¼ 4. bn = sec n = tan n cos n sin n =tan n =tan n

154 54 대수 ¼ n+. 즉 n = n 이고 n =( )n 0 = a n =sin n =sin ¼ n+ < ¼ n+ b n =tan n =tan ¼ n+ > ¼ n+ 이므로 n+ a n <¼< n+ b n. } 46. a, b, c 는서로다른정수들이고, P 는정수계수의다항식이다. P (a) =b, P (b) =c, P (c) =a 일수없음을보여라. ( 미국 974-) 풀이 ( 서울과학고 3년박종원학생의풀이를약간수정 ) 준식 () 을동치변형하면 fp (a) bg + fp (b) cg + fp (c) ag =0() P (a) =b; P (b) =c; and P (c) =a 이고, a 6= b 에서 a b 0(modja bj) 이므로 a b (mod ja bj) 이다. 또 P (x) 가정수계수다항식이므로 P (a) P (b) (mod ja bj) =) b c (mod ja bj) ( ) =) ja bj (b c) =) ja bj jb cj 이다. a, b, c 에대해위의과정은순환적이므로 ja bj jb cj jc aj ja bj 이고, 따라서 ja bj = jb cj = jc aj 이어야한다. 여기서일반성을잃지않고 c 를세수중최대라고하면 jc aj = jb cj 에서 c a = c b, 곧 a = b 가되는데이는 a 6= b 6= c 6= a 에모순이된다. 따라서 P (x) 는 () 을만족시킬수없다. } 보조정리 ( ) 의증명 [ 보조정리 ( )] f(x) 가정수계수의다항식이고 p q (mod m) 이면 f(p) f(q) (modm) 이다. f(x) = P n a k x k 이라하자. 그러면, k=0 nx nx f(p) = a k p k ; f(q) = a k q k k=0 k=0 이다. p q (mod m) 에서 p k q k (mod m) =) a k p k a k q k (mod m) nx nx =) a k p k a k q k (mod m) k=0 k=0 이므로 f(p) f(q) (modm) 이되어보조정리가증명되었다. } 47. 음이아닌임의의실수 x ;x ;:::;x n 에대하여 (x + x + + x n ) (x x + x x x n x ) k(x x + x x x nx ) 이항상성립되게하는상수 k 의최대값을구하여라. ( 통신강좌 99--) 풀이 ) x x + + x nx =0일때 : 모든실수 k에대해준식이성립. ) x x + + x nx 6=0일때 : x ;x ; ;x n 중적어도두개는양수이므로, 이중가장큰수를 x p, 그다음으로큰수를x q( p; q n) 라하면, x px q 6= 0이며다음이성립한다. (x + x + + x n) (x x + x x x nx ) (xp + xq) (x x + x x x nx ) x px q (A)

155 . 대수고급문제 55 이것을증명하자. (x + x + + x n) (x x + x x x nx )x px q (x p + x q) (x x + x x x nx )x px q (x p + x q ) (x x + x x x n x ) (* i 6= j n인모든i; j에대하여 x px q x i x j ) 위식의양변을 x p x q (x +x + +x n ) 으로나누면증명이성립한다. 이때등호는 x k =0( k n, k 6= p; q) 일때성립한다. 또한 (x p + x q) x px q 4 ( 등호는 x p = x q 6=0 일때성립.) (B) 이므로 (A), (B) 로부터, 음이아닌임의의실수 x ;x ; ;x n 에대하여 (x + x + + x n ) (x x + x x x n x ) (x x + x x x n x ) 4 ( 등호는 x p = x q = (6= 0);x k =0( k n; k 6= p; q) 일때성립.) ) maxfkg =4 } 48. a; b; c 0, a + b + c =일때 a +b + b +c + c +a 가취하는값의범위를구하여라. ( 통신강좌 99--4) 풀이 () a +b + b +c + c +a a + b + c = 등호는 (a; b; c) =(; 0; 0); (0; ; 0); (0; 0; ) 일때성립 () f(x) = 이라하면, f(x) 는아래로볼록한함수이므로 Jensen 부등식에의해서다음이성립한다. x a +b + b +c + c +a = af( + b)+bf( + c)+cf( + a) a( + b)+b( + c)+c( + a) 3 4 마지막식은다음에서나온다. a( + b)+b( + c)+c( + a) = +ab + bc + ca + 3 (a + b + c) = 4 3 등호는 a = b = c = 일때성립.(),() 에의해 a +b + b +c + c +a 최소값의등호는 a = b = c = 일때, 최대값의등호는 (a; b; c) =(; 0; 0); (0; ; 0); (0; 0; ) 일때성 3 립. } 49. x; y; z 0, x + y + z = 일때, 다음을보여라. ( 통신강좌 99--5) yz + zx + xy 4(y z + z x + x y )+5xyz

156 56 대수 풀이 ( 서울과학고이은수 ) yz + zx + xy 4(y z + z x + x y ) 5xyz = (yz + zx + xy)(x + y + z) 4(y z + z x + x y ) 5xyz = (yz + zx + xy)(x + y + z )+(yz + zx + xy) 4(y z + z x + x y ) 5xyz = y f(x + z)y x z g + z f(x + y)z x y g + x f(y + z)x y z g = y f( y)y x z g + z f( z)z x y g + x f( x)x y z g = (x + y + z)(x 3 + y 3 + z 3 ) (x + y + z ) 0 (By Cauchy-Schwartz's inequality) 8 단, 등호는 x = y = z = 9 >< 3 >= x = y =0;z = >: x =0;y = z = >; 일때성립.(x; y; z 는대칭이므로서로바꾸어도됨.) } 50. f(f(x)) = kx 9 을만족시키는연속함수 f : R! R 가존재할필요충분조건을 k 에관한내용으로서술하여라. ( 통신강좌 99--6) 풀이먼저 f 가전단사함수임을보이자. f 가전사함수임은너무도당연하다.(f(f(x)) 가모든실수를나타내므로 ) 이제 f 가단사함수임을보이자. m 6= n R 일때 f(m) =f(n) 이라고가정하면 f(f(m)) = f(f(n)) = km 3 = kn 3 이되고 k 6= 0 이면모순이된다. 즉 k 6= 0 일때는단사함수임을알수있다. f 가전단사함수이므로단조증가또는단조감소함수이다. 먼저단조감소함수라고가정하면, m n 일때 f(m) f(n). km 3 = f(f(m)) kn 3 즉 k 0 이면 f 는단조감소함수가된다. f 가단조증가함수라고가정하면, m n 일때 f(m) f(n), km 3 = f(f(m)) kn 3 = f(f(n)) 이므로 k 0 이면된다. 즉 f(f(x)) = kx 9 에서 k 0 이어야한다. ) k 0 은 f 가존재할필요조건. 그런데 k 0 일때 f = k 4 x 3 으로잡으면 f(f(x)) = kx 9 이성립한다. 즉 k 0 은 f 가존재할충분조건이기도하다. 결국 f 가존재할필요충분조건은 k 0 이다. } µ 5. 0 <x < 인 x 에대하여 x n+ = x n x n + (n ) 로수열x ;x ;::: 을정의할때, x 의값에 n 따른수열의수렴여부를판정하고, 수렴한다면극한을구하여라. ( 통신강좌 99--6) 풀이 ( 서울과학고박지웅 ) 통신강좌에서증명된사실을이용하기로 µ 한다. 0 <x < 인 x 에대해x n+ = x n x n + (n ) 인수열x ;x ; ;x n 을만들면, 모든 n에대 n 해 0 <x n <x n+ < 을만족하는 x 은오직하나뿐이다.( 이 x 을 x 0 이라하자.) (i) x = x 0 일때위증명에의해 0 <x <x < <x n <x n+ < < 이고또한 x n <x n+ 을만족하기위해서는모든 n에대해x n > 을만족한다. 그러므로모든 n에 n 대해 <xn < 이다. n µ = lim lim xn lim n! n = n! n!

157 . 대수고급문제 57 이므로 lim x n =. n! (ii) >x >x 0 일때우선 µ f n(x) =x x + ;g n(x) =(f nf n f )(x) n 로정의하자. 이때g n (x) 는단조증가함수가된다. 모든 n에대해g n (x 0 ) > 임이이미증명되어 n 있다.( 증명과정에서 ) g n (x) 는단조증가하므로모든 n에대해서 g n (x ) >g n (x 0 ) > n 이고모든 n 에대해서 x n+ = g n (x ) > (n ) n 이므로 x n+ >x n(n ), n =일때x x = x + x x = x > 0 ) x >x. 만약모든 n에대해x n+ = g n (x ) < 이라면결국모든 n에대해서 0 <x n <x n+ < (n ) 을만족하므로 x = x 0 가된다.( 모순 ) 그러므로어떤 m이존재하여 x m+ = g m (x ) 이어야한다. 따라서, x m+ = x m+ µ x m+ + m + x m+3 = x m+ µ x m+ + m + µ x m+4 = x m+3 x m+3 + m +3. x m+k = x m+k µx m+k + > >x m+ >x m+3 >x m+ > >x k m+ (k 3) m + k (iii) x 0 >x > 0일때모든 n에대해서 g n(x 0 ) < +p 4n + n 임은이미증명되었다. g n (x) 은단조증가함수이므로모든 n에대해서 x n+ = g n (x ) <g n (x 0 ) < +p 4n + n x < (n ) 이므로, 모든 n에대하여 x n < 이다. 만약모든 n에대해x n <x n+ 이성립하면 x = x 0 이므로모순이다. 따라서어떤 m에대하여 µ x m x m+, x n m µ ) x m+ = x m x m + µ >x m+ x m+ + = x m+ m m + 수학적귀납법에의해 k m +에대해x k >x k+ > 0이다. 그러므로 x n 은 0이상 미만의어떤값으 로수렴한다.( 수렴하는경우에는극한의연산이가능하다.) 따라서, lim n n! = µ lim n x n + = lim n! n n! x n + lim x n n! n = lim n! x n = lim n lim n n! n! 그런데 ) lim n! x n =; 0 lim xn < 이므로 lim xn =0. } n! n!

158 58 대수 5. 영함수가아닌연속함수 f 가 를만족시킬때 f( p x + y )=f(x) f(y) f(x) =ff()g x 임을증명하여라. ( 통신강좌 ) 풀이 f(x)f(x) =f(x)f( x) =f( x)f( x) 에서 f(x) =f( x) 또는 f(x) =f( x) =0이므로어느경우나 f(x) =f( x) 가된다. 이제 x 0일때만증명하면 f( x) =f(x) =[f()] x =[f()] ( x) 으로모든증명이끝난다 (). 이제특별한언급이없는한모든문자는양수라가정한다. f p 0+0 ³ px = f(0)f(0) 에서 f(0) = 0또는 f(0) = 인데, 만일 f(0) = 0이면 f(x) =f +0 = f(x) f(0) = 0이되어f 0이므로 f가항등적으로는 µ 0 µ 이아님에 모순이 µ 되므로, f(0) = 이다 µ (). t t t t 만일 f(t) =0인 t(> 0) 가존재한다면 f(t) =f p f p 에서 f p =0이고, 다시 f = µ t 0, :::, f n =0,::: 가된다. n!일때 t n! 0+ 이므로, f가연속함수이기위해서는 f(0) = 0이어야하는데, 이는 () 에모순이다. 따라서 f(t) =0인 t가존재하지않는다 (3). (3) 에서 f가연속함수이므로항상양이거나음이며, () 에서 f는항상양이다 (4). ( 이것은 f(t) = f( p t ) 0 임에서알수도있음.) 자연수 m, n에대하여 0s 0 ³ n µ f = µ s µ n A = f µ (n ) A n = = f m m m m m 이되는데, 다시 (4) 에의해 ³ m µ m µ f() = f = f =) f =[f()] =m m m m 이므로, ³ n µ n f = f =[f()] (n=m) m m 이되어,(),() 와함께 x 가유리수일경우 ( ) 의증명이끝났다. 잘알려진것처럼임의의무리수 에대해 n! 일때 a n! 인유리수의수열 fa n g 을구할수있으므로, f 가연속함수임을이용하면 f ( ) = lim f (a n) = lim n! n! [f()]a n =[f()] 이된다. 따라서, 임의의실수 x 에대하여 ( ) 을얻는다. } 53. m, n은자연수이고 a =(m m+ + n n+ )=(m m + n n ) 이다. a m + a n m m + n n 임을증명하여라. [ 실수 a 0 와자연수 N에대해, 비 (a N N N )=(a N) 을분석해보는것이도움이될지도모른다.] ( 미국 99-4) 증명힌트는 a N N N (a N)N N 를의미함. 54. k는주어진양수이다. 모든 x; y; z [0; ] 에대해 f(x;) = x; f(f(x; y); z)=f(x; f(y;z)) f(;y)=y; f(zx; zy) =z k f(x; y) 을만족하는함수 f :[0; ] [0; ]! [0; ] 을모두구하여라. ( 중국 99-)

159 . 대수고급문제 59 풀이우하의식에 z =0대입하면 f(0; 0) = 0. x y 일때우하의식에서f(x; y) =f(y x y ;y)= y k f( x y ; ) = xyk. 비슷하게 x y 일때는f(x; y) =x k y. x y 이고 z가충분히작을때우상의식과연계하면 f(f(x; y);z)=f(x k y; z) =x (k ) y k z f(x; f(y; z)) = f(x; y k z)=x k y k z 이므로이둘을비교하면 (k ) = k, 즉 k = 또는. k = 일때는 f(x; y) =minfx; yg 이고, k = 일때는 f(x; y) =xy. } 55. a ;a ;::: ;a n, b ;b ;::: ;b n 을양의실수라하고 nx a k = k= nx b k k= 라한다. 다음부등식을증명하여라. (APMO 99-3) 증명 ( 서울중산고 학년최서준 ) 라하면, nx k= (a k ) a k + b k nx a k k= P = a a + b + a a + b + + Q = b a + b + b a + b + + a n a n + b n b n a n + b n P Q = a b + a b + + a n b n a + b a + b a n + b n =(a b )+(a b )+ +(a n b n ) =(a + a + + a n ) (b + b + + b n )=0 한편 ) P = Q P = P + Q = a + b a + b + a + b a + b + + a n + b n a n + b n a + b + a + b an + bn + + = a + a + + a n 등호조건은위식에서 a i = b i 일때이다. 별증코시부등식을이용하자. µ a a + b + a a + b + + ((a + b )+(a + b )+ +(a n + b n)) a n a n + b n (a + a + + a n) 이므로 a + a + + a n (a + a + + a n) a + b a + b a n + b n (a + a + + a n) = (a + a + + a n ) 로문제의증명이끝난다.

160 60 대수 56. 실수들의수열 fa i g와 fb i g (i =; ;:::;n) 에서 a i 는모두양수라한다. 다음을증명하여라. ( 통신강좌 99-4-) X a i b j =0 =) X b i b j 0 i6=j i6=j 풀이 조건을 ai > 0 보다약한 a i 0 and X ai 0 에서먼저생각하자. 우선 X ³X a i b j =0=) ai ³X bj = X a j b j =) X X aj b j b j = X i6=j ai 임을얻어두면 X i6=j b i b j + X b j = ³X bj = ³X aj b j ³X ai ³X ³X a i b j ³X X bj (Cauchy-Schwarz 부등식 ) ai 이쉽게구해진다. 따라서 () 의결론이얻어진다. 여기서등호는 X b j =0 이거나 a i = kb i and X a i = ³X ai (7) 일때, 또그때만성립한다. 그러나다시문제의조건 a i > 0 으로의 b j =0 일때만성립한다. } 57. 양수 a 0, b 0, c 0 에대하여 a n, b n, c n (n =0; ; ;:::) 를다음과같이정의한다. a n+ = a n + b n + c n ; b n+ = a nb n + b n c n + c n a n ; c n+ = 3 a n + b n + c n 3a n b n c n a n b n + b n c n + c n a n () 각각의 n 에대하여 a n, b n, c n 의대소관계를밝혀라. () 수열 fa ng, fb ng, fc ng은같은극한값으로수렴함을보이고그값을구하여라. ( 한국 99-7) 풀이 () a n+ b n+ = (a n + b n + c n ) 3(a n b n + b n c n + c n a n ) 3(a n + b n + c n ) = (an bn) +(b n c n) +(c n a n) = 0 6(a n + b n + c n) ) a n+ b n+ (n = 0) ) a n = b n (n = ) b n+ c n+ = (a nb n + b n c n + c n a n ) 3a n b n c n (a n + b n + c n ) (a n + b n + c n )(a n b n + b n c n + c n a n ) = (a nb n b n c n ) +(b n c n c n a n ) +(c n a n a n b n ) (a n + b n + c n )(a n b n + b n c n + c n a n ) ) b n+ = c n+ (n = 0) ) b n = c n (n = ) = 0 따라서 a n = b n = c n 이다. 등호는 a 0 = b 0 = c 0 일때성립한다. () a n = b n = c n 이므로

161 . 대수고급문제 6 a n+ = a n + b n + c n 5 a n + a n + a n = a n 3 3 즉 fa n g은단조감소하고 a n > 0 이므로수렴한다. cn[an(bn cn)+bn(an cn)] 또 c n+ c n = = 0 a nb n + b nc n + c na n 이므로 fc ng은단조증가하고 c n 5 a n( 유계 ) 이므로수렴한다. 한편 a n+ b n+ c n+ = a n b n c n 이므로 a n b n c n = a 0 b 0 c 0 ( 일정 ) 따라서 fb n g도수렴한다. lim a n = A; lim b n = B; lim c n = C 라하면 n! n! n! A = A + B + C ;B= 3 에서 A = B + C; B = AB + BC + CA A + B + C ;ABC= a 0b 0 c 0 AB + BC + CA 3A ) AB = C(A + B) ) (B + C)B = C(A + B) ) B = CA = C B + C ) (B C)(B + C) =0 B + C>0 이므로 B = C ) A = B = C 이때 A = B = C = 3p a 0 b 0 c 0 } 58. a ;:::;a n; b ;:::;b n 이모두양의실수일때다음을증명하여라. (a a a n ) n +(b b b n ) n ((a + b )(a + b ) (a n + b n )) n 또, 등호가성립할조건은 a = a = = a n b b bn 임을보여라. ( 아일랜드 99-0) 증명우변을좌변으로이항하고산술 - 조화. 59. 임의의실수 a, b, c에대하여다음부등식이성립함을보여라. ( 폴란드 99/993 차-9) (a + b c )(b + c a )(c + a b ) (a + b c) (b + c a) (c + a b) 증명우변은항상 0 이상이므로좌변이음수일경우는항상성립한다. 좌변이양수이기위해서는 a + b c, a b + c, a + b + c 중두개가음수이거나모두양수이어야한다. 그러나서로다른두개씩더해보면항상양수가나오므로두개가음수일수는없다. 셋다양수라는조건 a + b c ; c + a b ; b + c a 을추가하여문제를다시풀어보도록하자. 같은방법으로 (a + b c) (b + c a) =(b (c a) ) = b 4 b (c a) +(c a) 4 b 4 (c + a )(c a) +(c a) 4 = b 4 (c a) (c + a) = b 4 (c a ) =(a + b c )( a + b + c ) (b + c a) (c + a b) (b + c a )(c + a b ) (c + a b) (a + b c) (c + a b )(a + b c ) 이므로주어진부등식이성립한다. 60. a, b, c, d 는음이아닌실수이고 ab + bc + cd + da = 을만족한다. a 3 b + c + d + b 3 c + d + a + c 3 d + a + b + d 3 a + b + c 3 임을보여라. ( 통신강좌 )

162 6 대수 풀이 s = a + b + c + d라면, 함수 x3 는 0 x<s에서 x에대한볼록함수이다. 따라서젠센의부 s x 등식을이용하여, a 3 s a + b3 s b + c3 s c + d3 s d 4 ( s 4 )3 s s = s 4 또, s 4(ab + bc + cd + da) =4 ) a 3 b + c + d + b 3 c + d + a + c 3 d + a + b + d 3 a + b + c 3 등호는 a = b = c = d 일때성립한다. } 6. 양의실수 x, y, z 에대하여다음을보여라. ( 통신강좌 ) xyz(x + y + z + p x + y + z ) (x + y + z )(xy + yz + zx) 3+p 3 9 풀이코시 - 슈바르츠부등식에의하여, 또, 산술 - 기하평균간의부등식에의하여, x + y + z p3 p x + y + z x + y + z 3(xyz) 3 ; xy + yz + zx 3(xyz) 3 ) xyz(z + y + z + p x + y + z (x + y + z )(xy + yz + zx) xyz( + p 3) p x + y + z (x + y + z )3(xyz) 3 = +p 3 3 +p 3 3 (xyz) 3 p x + y + z p = 3+p } 6. 3 j x + y 인임의의 0 아닌정수 x, y 에대해다음을만족하는함수 f : Z nf0g!q 를모두구하여라. ( 통신강좌 ) µ x + y f = f(x)+f(y) 3 풀이준식에서 y 에 x 를대입하면, 준식에서 x 와 y 에 3x 를대입하면, ³ x +x f(x) =f = f(x)+f(x) ) f(x) =f(x) 3 ³ 3x +3x f(x) =f = f(3x)+f(3x) ) f(x) =f(3x) 3 ) f() = f() = f(3)

163 . 대수고급문제 63 이제귀납법을써서 f(3n) =f(3n +)=f(3n +)=f() 임을보이자.(n =; ; 3; ) ³ (n ) + 3 f(3(n )) + f(3) f(3n) =f(n) =f = 3 f(n ) + f(3) = = f() ³ 9n +3 f(3 3n)+f(3) f(3n +)=f = = f() 3 ³ 9n +6 f(3 3n)+f(6) f(3n +)=f = = f(3n)+f() = f() 3 ) f(n) =f(); n =; ; 3 f(4) + f( ) 이제준식에 x =, y =4를대입하면, f() = 이고, f(4) = f() 이므로, f( ) = f() 이다. 이제음의정수에대해서도마찬가지방법으로하여다시귀납법을쓰면 f( n) =f() 임을알수있다. ) f(n) =f() Q; 8n Z nf0g; 즉 f 는상수함수이다. 63. a, b, c, d는자연수이고 r = a b c 라할때, a + c 993 이고 r>0 이면 r> 임을보여라. d 9933 ( 한국 993-) 증명 r = a b c bd ad bc = > 0 이므로 a<b; c<d이고, bd ad bc는자연수이므로 d bd bd ad bc = 따라서 r bd 이다. b d라하여도일반성을잃지않는다. (i) b d 993 이면 r bd 993 > (ii) 993 b d 일때 a + c =993이면, b = d =993이라하면 bd ad bc = b(d a c) =0이므로모순따라서 a + c 99이다. 이때 r = a b c d a 993 c 993 = a + c = 993 > (iii) b<993 <d일때 bd ad bc r = = b a µ bc > 0 bd b d b a 이므로, 여기서 a; b; c를고정시키면 r을최소로하는 d 의값은 bc + ( 여기서 [x] 는 x를넘지않는정수 ) 이다. b a r>0이므로 b a 이고 c 99이므로, d는최대993b이다. 이때, r bd 993b > (i) (ii) (iii) 에서 r> 이다. 주 : r>0 이면서 r을최소로하는경우는 일때 a =38;b=39;c= 665; d= [a + c = 993] r = 39( ) 이다. }

164 64 대수 64. 임의의양수 a, b, c에대하여다음부등식을증명하여라. ( 플란더즈 993-3) µ a b 993 µ b c 993 µ c a 993 < + + < a + b b + c c + a 증명 ( 이상훈 ) a, b, c 중어느두수를바꾸어도부호만바뀔뿐절대값은바뀌지않으므로 WLOG a b c라할수있다. e x = a b, ey = b c 이고 f(x) =(ex e x + )993 라두자. X j ( a b a + b )993 j < $ <f(x)+f(y) f(x + y) < cyclic a b, b 에서 x; y 0이다. 또, f가강증가함수이며 <f(x) < ( 모든실수 x에대해 ) 임은자 c 명하다. 따라서 f(x + y) < 이므로 f(x + y) < 0 f(x)+f(y) 가된다. 그리고 f(x) f(x + y), f(y) < 이므로 f(x)+f(y) < +f(x + y) 이성립한다. 증명끝 이아닌실수계수다항식 f(x) =a nx n + a n x n + + a 0 g(x) =c n+ x n+ + c n x n + + c 0 이어떤실수r에대하여g(x) =(x+r)f(x) 를만족하고있다. a =max(ja nj;:::;ja 0 j), c =max(jc n+ j;:::;jc 0 j) 이면 a n + c 임을증명하여라. (APMO 993-3) 풀이 따라서, (O±cial) g(x) =(x + r)f(x) 이므로 n+ X nx nx nx c k x k =(x + r) a k x k = a k x k + ra k x k k=0 k= k=0 k=0 nx = ra 0 + (a k + ra k )x k + a n x n+ k= c n+ = a n ;c k = ra k + a k (k =; ; ;n) c 0 = ra 0 ) a n = c n+ ; a n = c n rc n+ ; a n = c n rc n + r c n. a 0 = c rc + r c 3 +( ) n r n c n+ 일반적으로 a n i = c n i+ rc n i+ + +( ) i r i c n+ i =0; ; ; ;n 인관계식을얻는다. () jrj 일때, ja n i j jc n i+ j + jrc n i+ j + + j( ) i r i c n+ j (i +)c 가모든i =0; ; ; ;n에대해서성립하므로 a (n +)c 즉 a n + c 이성립한다. () jrj > 일때, y = x 이라하면 g( y )=( y + r)f( y ) 에서 c 0 r y n+ + c r y n + + c n+ =(y + r r )(a 0y n + a y n + + a n) 이다. 여기서 j r j < 이고 max(j c n+ j; j c n r r j; ; j c 0 r j)= c jrj 이므로 () 에의하여 a (n +) c jrj < (n +)c ) a < (n +) c (), () 에의하여 a n + c 이성립한다. }

165 . 대수고급문제 x ;:::;x n 은음이아닌실수들이고 a 는이수들중가장작은항이다. +x + +x + + +x n + +x n n + (x a) + +(x n a) +x +x 3 +x n +x ( + a) 임을보여라. 그리고등호가성립할때를밝혀라. ( 통신강좌 ) 증명귀납법으로보이자. n = 일때는자명하다. n 일때위부등식이성립한다고하자. a 를 x ;::: ;x n+ 중의최소값이라고하자. x n+ 이최대라고가정하여도일반성을잃지않는다. 그러므로귀납적가정에의해 n X k= +x k +x k+ + +xn +x n + ( + a) nx (x k a) k= 을얻을수있다. 이제 +x n + +x n+ +x n + (x n+ a) +x n+ +x +x ( + a) 를보이면된다. 그런데, 이것은 (x n+ x n )(x n+ x ) (x n+ a) ( + x n+ )( + x ) ( + a) 와동치이고, a x ;x n x n+ 이므로성립한다. 그리고등호가성립하기위해서는 x = x n = x n+ = a. 그러므로 x = x = ::: = x n+ 이다. 67. a, b는임의의실수이다. 방정식 x 4 + ax 3 +x + bx +=0 이실근을가지면 a + b 8 임을증명하여라. (Towns 993가을 SA6) 증명 x 6= 0 이므로 x 로나눠주고완전제곱식의합 :(x + a ) +( x + b ) = a +b 4 0 으로정리하면끝. 68. f, g 는모든정수들의집합에서정의되고정수값을취하는함수들로서 (a) 모든정수 m, n에대해f(m + f(f(n))) = f(f(m +)) n 이성립하고 (b) g는정계수다항식으로, 모든정수 n에대해g(n) =g(f(n)) 이성립한다. f(994) 를구하고 g 의가장일반적인꼴을구하여라. ( 통신강좌 ) 풀이 f (n) 은 f(f(n)) 을풀이한다. 그러면 (a) 에서 f(m + f (n)) = f (m +) n 이되고, m대신 f (m) 을대입하면f(f (m)+f (n)) = f (f (m)+) n이된다. m과 n을바꾸면 f(f (n)+f (m)) = f (f (n)+) m이된다. 따라서 f (f (m)+) f (f (n)+)=m n이된다. 다시 (a) 에서 f (f (m)+)=f(f(f (m)+))=f( m f ()) f () = k라두면f( m k) f( n k) =m n이모든정수m; n에대해성립한다. m 대신 m k를, n대신 k를대입하면 f(m) f(0) = m k + k = m 즉, f(m) = m + f(0) 이모든정수m에대해성립한다. 이제 f(f(m)) = f(m)+f(0) = m f(0) + f(0) = m이므로 f (m) =m이모든정수m에대해성립한다.(a) 에의해f(m + n) = m n이되고 m =0이라두면 f(n) = n 이모든정수n에대해성립한다. f(994) = 995를얻는다.(b) 로부터 g(n) = g( n ) 이모든정수n에대해성립한다. g는다항식이므로 g(x) =g( x ) 이모든실수x에대해성립한다. 이제 g(x) =p(x + ) 꼴로표현하면 g( x ) = p( x ) 이므로 p(x + )=p( x ) 이된다. 그러므로 g는 (x + ) = x + x + 4 의다항식이고, 따라서 x + x의다항식이다. g(n) =a 0 + a n(n +)+a n (n +) + + a sn s (n +) s } 69. 함수방정식 f(f(x)) + f(x) =ax + b 가딱한개의연속인실수함수 f 를해로가질수있도록하는음이아닌실수의순서쌍 (a; b) 를모두구하여라. ( 통신강좌 )

166 66 대수 풀이 ( 광주과학고이준성 ) f(x) =cx + d 인 f(x) 가 개이상존재하면안된다. c(cx + d)+d + cx + d = ax + b 계수비교하면 c + c = a, (c +)d = b 이다. c + c a =0 에서 D =+4a>0 이므로 c 의한해가 일때 (d 가불능일수있을때 ) 만성립한다 ( 그렇지않으면 c 의해가둘있고그때마다 d 가결정되므로해가유일하지않다 ). 즉 a =,b>0(* b =0 이면 d 가여러개 ) 이어야한다. 우선 f(x) =x + b 3 가한해가된다. f(x) =g(x)+x + b 3 라하자. g(g(x)+x + b )+g(x) =0 3 여기서, g(x 0 ) 6= 0인 x 0 가존재하면 꼴에서함수값이계속 배되기때문에 g(x ) b 3 g(x ) 인 x ;x 가존재한다. g(x) 는연속함수이므로중간값정리에의해 g(y) = b 인 y가존재하게된다. 3 에의해g(g(y)+y + b 3 )+g(y) =0이고 3g(y) =0,g(y) =0으로 g(y) = b < 0 에모순이다. 따라 3 서, g(x) 0 이고, 더이상의해는없어서늘해가유일함을알수있다. 답 a =,b>0 } 70. 다음세조건을만족시키는함수 f : R! R 을모두구하여라. (i) 모든 x; y R 에대하여f(x)+f(y)+ f(x + y) f(x)+f(y) (ii) 모든 x [0; ) 에대하여f(0) f(x) (iii) f( ) = f() = (APMO 994-) 풀이 (O±cial) (ii) 에서 f(0) f( ) 이므로이것과 (i), (iii) 에의하여 f(0) + f(0) + f( )+f( )+ f( + )= f() =, 즉 f(0) 0. 또한, f( + 0) f() + f(0) 임에서 f(0) 0. ) f(0) = 0 (ii) 에서 x (0; ) 일때 0 f(x);f( x). 이로부터 f(x)+f( x)+ f(x+ x) =f() = 인데등호가성립해야하므로 ) x [0; ) 이면 f(x) =0 () (i) 에서 f(x +) f(x) +f() = f(x) + 이므로 f(x +) f(x) +. 또한 (i), (iii) 에서 f(x) = f(x + ) f(x +)+f( ) = f(x +) 이므로 f(x +) f(x)+. 즉, f(x +)=f(x)+ () (), () 로부터 답 f(x) =[x] } nx 7. n 이고, W ;W ;:::;W n 은 =을만족하는양의실수이다. 다음을보여라. +W i= i ( 통신강좌 ) nx nx W i (n ) i= i= W i

167 . 대수고급문제 67 증명 W j = +W j +W j nx W j R0 = n +W j= j 0 Ã nx nx nx n! W i (n ) W i X A W i +W i= i= j= j i= 0 Ã nx W j X A W i +W j= j i= nx nx W i W j = i= j= W i ( + W j ) = X Ã W i W j + W! j W i i<j W i ( + W j ) W j ( + W i ) = X i<j = X i<j (W i W j )(W j ( + W i ) W i ( + W j )) W i W j ( + W i )( + W j ) (W i W j )(W j W j )(W i W j ) W i W j ( + W i )( + W j ) = X (W i W j ) (W i + W j ) (W i W j ) i<j W i W j ( + W i )( + W j ) 한편, + +W i +W j () +W j ++W i +W i + W j + W i W j () W i W j 따라서 X (W i W j ) (W i + W j ) (W i W j ) 0 i<j W i W j ( + W i )( + W j ) nx nx W i (n ) W i 0 i<j i= 한편등호는 n = 또는 W = W = = W n 인경우에만성립한다. 7. a i > 0(i =; ;:::;n) 이고 a n+ = a 일때 nx i= a i a i+ (a i + a i+ ) nx a i= i + a i+ 임을보이고등호조건을밝혀라. ( 통신강좌 )

168 68 대수 증명 ( 대전대덕중임재한 ) nx a i a i= i+ (a i + a i+ ) nx µ = a i= i a i + a i+ µ = nx a a a n a i= i + a i+ = µ + + µ µ + nx a a a a 3 a n a a i= i + a i+ nx a + a a + a 3 a n + a a i= i + a i+ nx nx = a i= i + a i+ a i= i + a i+ nx = a i= i + a i+ 등호조건은산술조화평균부등식의등호조건에의하여 a = a = = a n 이다. 73. 모든실수 x, y 에대해다음을만족하는함수 f : R! R 를모두찾아라. f(f(x + y)) = f(x + y)+f(x)f(y) xy ( 벨로루시 995-A3) 증명 ( 한석원 ) 원식에y =0대입 : f(f(x)) = (f(0) + )f(x) () 이것을원식에대입하고정리하면, f(0)f(x + y) =f(x)f(y) xy () f(0) 6= 0이라가정하자. () 에 x = f(0) 을대입하면, f(0)f(f(0) + y) =f(f(0))f(y) f(0)y =(f(0) + )f(0)f(y) f(0)y ) f(f(0) + y) =(f(0) + )f(y) y () 에서 x에 y + f(0) 을대입하면, f(0)f(y + f(0)) = f(y + f(0))f(y) (y + f(0))y f(y + f(0))f(y) (y + f(0))y =(f(0) + )f(y)² yf(y) y² f(0)y f(0)f(y + f(0)) = f(0)(f(0) + )f(y) yf(0) =(f(0) + )f(y) (f(0) + )y yf(0)(* () 에서x = y대입 ) ) (f(0) + )f(y) yf(y) y f(0)y =(f(0) + )f(y) (f(0) + )y yf(0) 정리하면, yf(y) =f(0)y + f(0)y 여기서 y 6= 0이면 f(y) =f(0)y + f(0) (3) 이것을 () 에대입하면, f(0)(f(x)+) =(f(0) + )f(0)(x + ) ) f(x) =(f(0) + )x + f(0) (4) (3),(4) 를비교하면 f(0) = f(0) + 이되어모순 ) f(0) = 0 그러면 () 에서 f(x)f(y) =xy 이다. 이때 f(x) = x 인데 a 6= 0 이면서 f(a) = a 인 a 가존재한다면, () 에 x = a 를대입하면 f( a) = a 그런데 () x = a; y = a 를대입하면 a = a 이되어모순.

169 . 대수고급문제 u는 0 <u< 인실수이다. 8 < 0 if 0 x u f(x) = ³ pux p : + ( u)( x) if u x 으로정의하고, 수열 fu n g 을점화적으로다음과같이정의하자 : u = f(); u n = f(u n ) (n>) u k =0 이되는자연수 k 가존재함을보여라. ( 캐나다 995-5) 증명 우선 0 x 이면산술 - 기하평균부등식에의해 0 pux + p ( u)( x) u + x 이므로 0 f(x), 따라서수열 u k 는잘정의된다. x u 일때, f(x) = ( p ux + p ( u)( x)) + ( u)+( x) = ux ( u)( x) p ux( u)( x) x + u ux p uu( x)( x) = x + u ux u( x) =x u = 로 u 이상줄어든다. 따라서, u k 는 u 이상씩계속작아져언젠가 u 보다작아지고, 그럼그다음에 0 이될수밖에없다. 75. 다음을만족시키는실수열 a ;a ;::: ;a 995 를모두구하여라 : p a n (n ) a n+ (n ); (n =; ;::: ;994) 이고 p a a + (APMO 995-) 풀이 T n = a n (n ) 이라놓으면, 994 X n= p 994 X T n 여기서 p T 995 T + 이므로위식은, 995 X 0 (T n p T n +)= n= 995 X 995 X (T n+ +)= (T n +) p 995 X T n n= (T n +) 이된다. n= n= 995 X ( p T n ) 이되므로, p T n =0이된다. n= n= T n =에서 a n n += ) a n = n이된다. } 76. 임의의양의실수 a ;:::;a k 에대해서다음부등식이성립함을증명하여라. X cyclic a k a +a + +(k )a k k 단, k 는 이상의자연수이다. ( 통신강좌 )

170 70 대수 증명양변에 k를곱해준후좌변의각항에씩더해주고우변에도 k를더해주면다음과같이동치변형된다. X a +a + +(k )a k + ka k k a +a + +(k )a k k + k cyclic 여기서좌변은다음과같이분해되는데 X a + a + + a k + X a +a 3 + +(k )a k a cyclic +a + +(k )a k a cyclic +a + +(k )a k 왼쪽항은코시-슈바르츠부등식혹은산술-조화평균부등식에의해 k 이상이되고, 오른쪽항은 k 산술-기하평균부등식혹은재배열부등식에의해 k 이상이된다. 그러므로준식은성립한다. 등호는 a = = a k 일때성립. 77. a i, b i, c i 는양의실수이다 (i =; ;:::;n). 다음부등식을증명하고등호조건을밝혀라. ( 통신강좌 ) nx nx (a i + b i + c i ) i= i= 증명 ( 대전과학고 학년윤재문 ) 보조정리 우선다음을보이자. a i b i + b i c i + c i a i a i + b i + c i a + b + c + d nx i= a i b i c i a i b i + b i c i + c i a i a+c + ( 단, a; b; c; d > 0) b+d nx X n X n a i b i c i i= i= i= 증명 (a + b + c + d)(a + b)(c + d)( 우변 좌변 ) = (a + b)(c + d)(a + c)(b + d) (a + b + c + d)fab(c + d)+cd(a + b)g = (a + b)(c + d)(ab + cd + ad + bc) (a + b + c + d)(ab(c + d)+cd(a + b)) = (a + b)(c + d)(ad + bc) (a + b) cd (c + d) ab = (a + b)(bc + ad ) (c + d) ab = a d + b c abcd = (ad bc) 0 보조정리 a + b + + c d + e + f a+d + b+e + c+f 증명정리 을이용한다. a + b + + c ab a+b + d+e de à * d + e + f ab a + b + + c+f = ) ab a+b + de d+e ab + c a+b + de + f d+e a+d + b+e + c+f de d + e = a + b a + d + b + e + d + e a+d + b+e! 등호는 a d = b e = c f 이다.

171 . 대수고급문제 7 이제문제로돌아가서귀납법을사용하자. n = 일때는성립함을알수있다. n 일때성립한다고가정하고 n + 인경우를보자. a 0 n = a n + a n+ b 0 n = b n + b n+ c 0 n = c n + c n+ 로두고, a 0 k = a k; b 0 k = b k; c 0 k = c k(k =; ; ;n ) 라하자. a 0 ;a0 ; ;a0 n 에대하여는부등식을적용할수있다. n+ X nx (a i + b i + c i )= (a 0 i + b0 i + c0 i ) i= i= a nb n + b nc n + c na n + a n+b n+ + b n+ c n+ + c n+ a n+ a n + b n + c n a n+ + b n+ + c n+ = ½ ¾ an(bn + c n) bn(cn + an) cn(an + bn) a n + b n + c n a n + b n + c n a n + b n + c n ½ an+ (b n+ + c n+ ) + b n+(c n+ + a n+ ) + a n+ + b n+ + c n+ a n+ + b n+ + c n+ ¾ c n+ (a n+ + b n+ ) a n+ + b n+ + c n+ 80 = : + + A + + a n b n +c n a n+ b n+ +c n+ + + A + + b n c n+a n b n+ c n+ +a n+ 0 = + + A + ; c n a n +b n c n+ a n+ +b n+ 8 < = : + a n+a n+ + b n+b n+ +c n+c n+ 9 = + ; c n+c n+ a n+a n+ +b n+b n+ ½ (an + a n+ )(b n + b n+ + c n + c n+ ) a n + a n+ + b n + b n+ + c n + c n+ + (b n + b n+ )(c n + c n+ + a n + a n+ ) + b n + b n+ + c n + c n+ + a n + a n+ ¾ (c n + c n+ )(a n + a n+ + b n + b n+ ) c n + c n+ + a n + a n+ + b n + b n+ = a0 nb 0 n + b 0 nc 0 n + c 0 na 0 n a 0 n + b 0 n + c 0 n b n+c n+ + + c n+c n+ +a n+a n+

172 7 대수 a n b n c n a n+ b n+ c n+ + a n b n + b n c n + c n a n a n+ b n+ + b n+ c n+ + c n+ a n+ = a n b n c n a n+ b n+ c n+ + a n +a n+ a 0 = nb 0 nc 0 n a 0 nb 0 n + b 0 nc 0 n + c 0 na 0 n b n +b n+ + = c n +c n+ a 0 + n b 0 + n c 0 n n+ X n+ X n+ a i b i + b i c i + c i a i X a i b i c i (a i + b i + c i ) a i= i= i + b i + ci a i= i b i + b i c i + c i a i nx (a 0 i + b0 i + c0 i ) X n a 0 i b0 i + b0 i c0 i + nx c0 i a0 i a 0 i b0 i c0 i a 0 i= i= i + b0 i + ci0 a 0 i= i b0 i + b0 i c0 i + c0 i a0 i nx nx nx n+ a 0 i b 0 i c 0 i = X n+ X n+ X a i b i c i i= i= i= i= i= i= 따라서모든자연수 n 에대하여성립하고, 등호가성립할조건은 a i+ a i = b i+ b i = c i+ c i (i =; ; ;n ) 이다. 즉, a i a = b i b = c i c (i =; ; ;n) 이다. 78. a i (i =; ;:::;n) 이고 n 은자연수이다. 다음을증명하여라. ( 통신강좌 ) ( + a )( + a ) ( + a n ) n n + ( + a + a + + a n ) 증명귀납법. n =일때는좌변 = 우변으로자명. n일때성립한다고가정하면,(+a )( + a ) ( + a n )( + a n+ ) n n+ ( + a + + a n )( + a n+ ) 이므로 n n + ( + a + + a n )( + a n+ ) n+ n + ( + a + + a n+ ) 만보이면 n + 일때가증명된다. n 으로나눠주고양변에분모를넘기면 (n +)(+a + + a n+ +(a + + a n )a n+ ) (n +)(+a + + a n+ ) (n +)(a + + a n )a n+ n( + a + + a n+ ) 과동치가된다. 그리고, 이식은다음의세부등식을변변합한것이다. n(a + + a n )a n+ n(a + + a n ) (a + + a n)a n+ na n+ (a + + a n)a n+ n 따라서, 성립. 별증 좌변을완전히전개한식에서나타나는각항을 a i a i a ik k (a i + a i + + a ik )

173 . 대수고급문제 73 꼴의부등식으로분해하기로하자 ( 이부등식은k를곱한후좌변을 k개의식으로분할하여변수를차례로하나씩만남기면자명하다 ). 그럼하나의 a i 가나타나는항들을생각해보면, a i 이외에 r개의변수가더곱해진항은 n r 개이고그때ai 에는 r+ 의계수가남으므로, a i는총 의계수를갖게된다. 즉, n X ³ n r r + = n X ³ n = n n r + n r=0 r=0 + n (a + + a n ) n n n + ( + a + + a n ) 을증명하면충분하다. 양변에 n(n +) 을곱하면 n(n +)+( n )(n +)(a + + a n ) n n( + a + + a n ) n(n +)+( n n )(a + + a n ) n n 와동치이다. n n 0 이고 a i 이므로 ( 좌변 ) n(n +)+( n n )n =( 우변 ) 으로성립. 79. a =이고 n 에대해a n+ = an n + n 으로정의되는수열 fa n g n 이있다. n 4 에대해, 항상 a n ba nc = n 임을증명하여라. ( 불가리아 996 4차-4) 관찰제감은귀납법으로증명하는것인데, 귀납법으로해보려고하면왼쪽부등식은쉽게되는데오른쪽부등식이잘안됨을알수있다. 즉, 오른쪽부등식은꽤빡빡한데반해왼쪽부등식이느슨해서생기는현상이므로, 양쪽의빡빡한정도를비슷하게맞춰줘야겠다는생각이든다. n + <a n <n+ 을귀납적으로증명할수있는 를찾으면되고, 오른쪽부등식을증명할때딱필요한정도의 을찾아보면 3 정도면적당할거란것을발견하게되고, 그것으로왼쪽부등식도되는지다시확인하면충분함을역시알수있음. 증명 n + 3 <a n <n+ 을귀납적으로증명하면충분함. 별증 ( p n<) p n <a n < p n n +(< p ) 을귀납적으로증명하면충분함. f n (x) := x n n n + n x n 라하면이함수는x = n 일때극소이고, f n ( p n )= p n n 등에유의하면됨. 80. a, b, c 가한삼각형의세변의길이일때, 다음을증명하여라. p a + b c + p b + c a + p c + a b p a + p b + p c 또, 등호가성립할때는어떤경우인가? (APMO 996-5) 증명 ( 과기원수학과 94학번이정훈 ) A = b + c a, B = c + a b, C = a + b c라두자. a, b, c가삼각형의세변의길이이므로 A>0; B > 0; C > 0 이다. 코시슈바르츠부등식에의해 p p p p A + B A + B p p p p B + C B + C p p p p C + A C + A 이다. 모두더하면 p p p p ( A + B + B + C + C + A) p p p ( A + B + C) ( p a + p b + p c) p p p a + b + c ( p a + b c + p b + c a + p c + a b) p p p a + b c + b + c a + c + a b 이다. 한편등호는 a = b = c, 즉정삼각형일때만성립한다.

174 74 대수 8. a; b, c 0 는실수이고 n 은정수이다. 다음을증명하여라. (ab + c) n c a n ((b + c) n c) ( 폴란드 996/997 차 -3) 증명 c(a n ) (ab + ac) n (ab + c) n 과동치. x = ab + ac, y = ab + c 로두면a> 일때이것은또 a n + + a + x n + x n y + + y n 과동치. x; y a 이므로이것으로끝. 8. 실수에서실수로의함수들인 f ;f ;:::;f n 이모든 x; y R 에대해다음조건을만족시킨다. () 모든 i f; ; 3;:::;ng 에대해f i (x + y) =f i (x)+f i (y). () f (x)f (x)f 3 (x) f n (x) =ax n.( 단, a 6= 0) 모든 i f; ;:::;ng 에대해 f i (x) =b i x 를만족시키는실수 b i 들이존재함을보여라. ( 통신강좌 ) 증명 첫번째성질에서정수 m, 임의의실수 x 에대해 f i ( + mx) =f i () + mf i (x) 임이성립함은당연하다. 이제, f i () = c i 라놓고생각하자. 임의의실수 x 에대하여 ny ny f i ( + mx) = fc i + mf i (x)g = a( + mx) n (* 두번째준식 ) i= i= 이제실계수다항식두개를가정해보자. P x(t ) = ny fc i + f i (x)t g i= Q x(t ) = a( + xt ) n a 6= 0 이므로일단두함수는영함수가아니다. 그리고 8m z 에대해 P x(m) =Q x(m) 이므로, P x(t ) Q x(t ) 결론적으로다항식의인수분해의유일성에대해, 실수 b i 가존재하여, c i + f i (x)t = b i ( + xt ) 를만족시킴을알수있다. 83. r = +p 5 일때, 주어진정수 n 에대해 [nr ]=[kr]+ 을만족하는정수 k를구하여라. 단,[x] 는 x보다작거나같은정수이다. ( 통신강좌 ) 증명 k =[nr] 로잡으면된다. 이때문제의식이성립함을보이자. [nr ] kr < [nr ] 임을보이면된다. r = r +,r =+ r 이므로이식은 n +[nr] [nr]+ [nr] r <n+[nr], 즉 n [nr] r 과동치이다. 오른쪽부등식은 nr이정수가아니므로 [nr] <nr임에서바로나오고, 왼쪽부등식도 r> 임에서 [nr] > nr = n >n r r r 에의해금방확인된다. 답 [nr] <n

175 . 대수고급문제 모든실수 x에대해f(x) =f(x + ) 4 을만족하는연속함수 f : R! R 을모두찾아라. ( 불가리아 997 3차-4) 풀이 g(x) = x + 4 이라하면 0 x 의범위에서 x g(x). 따라서, 이범위에서 x; g(x);g(g(x)); 은위로유계인증가수열이므로수렴하고, 그수렴값을 a라하면연속함수이므로 g(a) =a. 즉, a =. x 의범위에서도 g(x) x 로마찬가지로수렴하고그수렴값은역시. 따라서, x 0 을어떻게잡아도문제의조건과연속함수임에서 f(x) =f(g(x)) = f(g(g(x))) = = f( ), 즉 f는 x 0 에서상수함수. f( x) =f(x) 이므로 f는전범위에서상수함수. } 85. abc = 을만족하는양의실수 a, b, c 에대해다음부등식을증명하여라. +a + b + +b + c + +c + a +a + +b + +c ( 불가리아 차 -4) 증명 p = a + b + c, q = ab + bc + ca 라하고양변통분하면 3+4p + p + q + 4p + q p + q + p + pq 9+4p +q. 분모넘겨서정리하면 3p q + pq +6pq 5p + q +4p +3q +7. AM-GM에서 p; q 3 이므로, 3p q 5p +p, pq q +3q +9, 6pq p +8 등으로나눠합하면확인됨. 86. x ;x ;:::;x 997 은실수들로,(a) 모든 i =; ;:::;997에대해 p x 3 i p3 이고 (b) x + x + + x 997 = 38 p 3 을만족한다. x + x + + x 997 의최대값을구하여라. ( 중국 997-) 풀이 f(x) =x 이라하면 f는볼록이므로, 즉 x y 이면임의의 a>0 에대해f(x) +f(y) < f(x a)+f(y + a) 가되므로, 최대값은 x i 들이딱하나만제외하고모두범위의경계값을가질때발생한다. x i 들중n개가 p 3 이고 996 n개가 p 3이라고하면남은한항은 38 p 3+ p n (996 n) p 3 3 이된다. 이것이 (a) 의범위에있어야하므로 p 3을곱해서생각하면 38 3+n 3(996 n) 3, 정리하면 4n 풀면자연수해는 n = 736뿐이다. 즉,736항이 p 3 이고 60항은 p 3이며남은한항은 p3 일때최대값은 답 736= (4=3) 6 } 87. 양수 a, b, c에대하여다음부등식을증명하여라. (APMO 998-3) ³ + a µ + b ³ + c µ + a + b + c b c a 3p abc 증명 (O±cial) x = a 3p abc ; y = b 3p abc ; z = c 3p abc 라하면주어진부등식은 µ + x ³ + y ³ + z = ( + x + y + z) y z x 와동치이다. 여기서 xyz =이므로 (x + y)(y + z)(z + x) = +(x + y + z) () 을밝히면된다. s = x + y + z라하면 (x + y)(y + z)(z + x) = (s z)(s x)(s y) = s 3 s (x + y + z)+s(xy + yz + zx) xyz = (x + y + z)(xy + yz + zx) xyz 이므로 () 은 (x + y + z)(xy + yz + zx ) xyz = ()

176 76 대수 와동치이다. xyz = 이고산술평균과기하평균의부등식에의하여 x + y + z = 3 3p xyz =3 xy + yz + zx = 3 3 q(xyz) =3 즉, xy + yz + zx = 이므로 () 가성립한다. 88. a, b, c 가양의실수일때다음부등식을증명하여라. (Towns 998 봄 SA) a 3 a + ab + b + b 3 b + bc + c + c 3 c + ca + a a + b + c 3 증명좌변이대칭식이아닌것같지만사실은대칭식. 89. 수열 a 0 ;a ;a ;a 3 ;::: 이다음과같은성질을가지는경우이를볼록이라한다. a n a n + a n+ (n ) 임의의양의실수 c 에대해서 b 0 ;b c; b c ;:::;b n c n ;::: 의수열이항상볼록이된다고할때, 아래의수열도볼록임을보여라. ( 통신강좌 ) log b 0 ; log b ; log b ; ::: 90. a; b; c Q, ac 6= 0 이고, 다음의방정식 ax + bxy + cy =0 에다음과같은꼴의자명하지않은 ( 모든변수가 0 은아닌 ) 해가있다고한다. (x; y) =(a 0 + a 3 p +a 3 p 4;b 0 + b 3 p +b 3 p 4) 단, i =0; ; 에대해 a i ;b i Q 이다. 이방정식이자명하지않은유리수해도가짐을증명하여라. ( 벨로루시 TST 999-) 증명 ( 신승현 ) x y = t )at + bt + c =0 )t = b p D = a 0 + a 3 p p +a 3 4 a b 0 + b 3 p p +b 3 4 ) p D = c 0 + c 3 p p +c 3 4 b 0 + b 3 p p (c +b 3 0 ;c ;c Q) 4 D 가 (P Q) 꼴임을보이자. 만약 D 가 P 꼴이아니라면 p p p p p ) (b 0 D + b 3 D + b 3 4 D)=C0 + c 3 p p +c 3 4 p p ) (b 0 D + b 6 4D 3 + bp 6 6D 3 )=C 0 + C 6 p p 4+C 6 6 4D 3 ; 6D 3 은 4 q 6 or 6 q 6 (q Q) 꼴이될수없다. 왜냐하면, 만약된다고가정하면 (i) 4D 3 =4q 6 ) D = q (ii) 4D 3 =6 q 6 ) D 3 =4q 6 =( 3p 4 q ) 3 ) D = 3 p 4 q = Q

177 . 대수고급문제 77 (iii) (iv) 6D 3 =4q 6 ) D 3 = 4 q6 ) D = q 3 p 4 = Q 6D 3 =6q 6 ) D = q ) bp 6 D 3 ;b 6 p p p p 6D; c 6 4;c 6 6;b0 D; c0 중동류항이하나도없으므로무리수의상등에의해 b 0 = b = b = c 0 = c = c =0 ) p D = p = c 0 + c 3 p p +c 3 4 b 0 + b 3 p p +b 3 4 무리수의상등에의해 c i = p! c i = p 0 ( 유리상수 ) b i b i ) t = p 0 = x y 이므로자명하지않은유리해존재 9. 실수항으로이루어진수열 a ;a ;::: 은모든자연수 i, j 에대하여 a i+j a i + a j 를만족한다. 다음부등식이모든자연수 n 에대해성립함을보여라. (APMO 999-) a + a + a a n n a n 증명 b i = i a i(i =; ; 3; ) 라하고 b + b + + b n = a n (n =; ; ) 임을 n 에관한귀납법으로증명한다. n = 일때, b = a = a 이므로성립한다. k =; ; ; n 일때 b + b + + b k = a k 가성립한다고가정하고 b + b + + b n = a n 임을밝히면된다. 이식은다음식과동치이다. 즉 nb + nb + + nb n + nb n = na n 에서 nb n = a n 이므로 nb + nb + + nb n = (n )a n 이식은a n 5 a i + a n i 을이용하여다음과같이증명한다. nb + nb + + nb n = (n )b +(n )b + + b n +b +b + +(n )b n = b +(b + b )+ +(b + b + + b n ) +(a + a + + a n ) = (a + a + + a n ) n X n X = (a i + a n i ) = a n =(n )a n i= i=

178 78 대수 9. 모든자연수 x, y 에대해다음을만족하는함수 f : N! N 을모두구하여라. f(x + f(y)) = f(x)+ y ( 아일랜드 00-0) 풀이 y 대신 y + f() 을대입하면좌변 = f(x ++f(y)) = f(x +)+y = 우변 = f(x)+y + f() 이므로 f(x +) f(x) =f(). 따라서, f(n) =an 의등차수열 (a = f()). 문제의식에대입하면 ax + a y = ax + y 임에서 a =. 끝. } 93. 모든자연수 n 3 과임의의양의실수 x ;x ;:::;x n 에대해서 nx i= x i (x i+ + x i+ ) n ; nx i= x i (x i + x i ) n 중적어도하나는성립함을증명하여라. 단, x n+ = x, x n+ = x, x 0 = x n, x = x n 이다. ( 폴란드 00 3 차 -4) 증명 ( 민족사관고 학년남율희 ) X = Y n 임을보이면충분하다. A + n = nx i= nx = i= nx i= x i x i+ + x i+, Y = nx i= x i + x i + x i+ + x i+ x i + x i+ x i + x i x i + x i+ + nx i= n + n ( 산술 - 기하 ) x i+ + x i+ x i + x i+ x i x i + x i 라두자. A = X + 따라서, A n 이고, 명제는참이다. 94. a, b, c가양의실수이며 a + b + c = 을만족한다. 다음을증명하여라. (APMO 00-4) p p p p p p p a + bc + b + ca + c + ab abc + a + b + c 증명 (KAIST 0학번김민규 ) a = x, b = y, c = z 로치환하고양변에 p xyz를곱하면, 문제는 x + y + z =에대해 p p p p p p yz + x + zx + y + xy + z + xy + yz + zx 임을증명하는문제로바뀐다. 그리고이것은다음세식을확인하여합하면충분하다. p p p p p p yz + x x + yz; xy + z z + xy; zx + y y + zx 세식은모두대칭적이므로하나만보이면되겠다. p p p q xy + z = xy + z(x + y + z) = (z + y)(z + x) (z + p yx ) = z + p xy 이와같이잘확인된다. 95. 양의실수 a, b, c, d 에대해 A = a 3 + b 3 + c 3 + d 3, B = bcd + cda + dab + abc 라할때, (a + b + c + d) 3 4A +4B 임을보여라. ( 폴란드 00/003 차 -)

179 . 대수고급문제 79 증명 (KAIST 04 학번윤혜원 ) S = a + b + c + d, C = X sym. ab = ab + ba + ac + ca + 이라하자. S 3 =(a + b + c + d) 3 = A +3C +6B 이므로문제의부등식 S 3 4A +4B 은 S 3 4(S 3 3C), 즉 4C S 3 과동치이다. 이것을증명하자. C = a (b + c + d)+ = a a(s a)+ µ a +(S a) = a + =(a + b + c + d) S 4 = S3 4 으로증명이되었다. 96. a, b, c 가주어졌을때, x, y, z 에대한다음연립방정식을풀어라. x yz = a y zx = b z xy = c ( 셈본중등중급도전문제 3..) 풀이 A = a, B = b, C = c 으로바꿔쓰자. A; B; C 0 임에만유의하면된다. 우선문제의식들로부터 () ya + zb + xc =0,()zA + xb + yc =0임을알수있다 (A, B, C 대신문제의식을대입해서확인해보면됨 ). () A =() B 으로 z를소거하고,() C =() A 으로 y를소거하면 x : A BC = y : B AC = z : C AB (3) 를얻는다. 이것을문제의첫번째식 x yz = A 에 (A BC) 을곱한후 y, z 대신대입하여 x 만남기면 ((A BC) (B AC)(C AB))x = A(A BC), 이것을정리하면 A =0 또는 x : A BC =: p K (K = A 3 + B 3 + C 3 3ABC) (4) 이된다. 조건은대칭적이므로, y, z 에대해서도같은종류의식이성립한다.AM-GM 으로 K 0. (i) K>0 일때 :AM-GM 의등호조건에의해 A, B, C 가모두같지는않으므로 0 이아닌것이있고, (3), (4) 에의해 A, B, C 중 0 이아닌것이하나만있어도 x : A BC = y : B CA = z : C AB =: p K 이된다. K>0이므로이식으로 x, y, z가모두결정된다. (ii) K =0일때 : 등호조건에서 A = B = C. 만일 A = B = C =0이면문제의세식을더하여 x + y + z = yz + zx + xy 가되고이것은 (x y) +(y z) +(z x) =0과동치이므로 x = y = z 뿐이다. A = B = C 6= 0일때는 () 에서 x + y + z =0. 이때문제의세식은모두 x + xy + y = A 가되고, 이것을만족하는모든 x, y에대해각각의해가결정된다. } 97. a, b, c, d, e, f가 a + b + c + d + e + f = 0 a 3 + b 3 + c 3 + d 3 + e 3 + f 3 = 0 을만족할때, (a + c)(a + d)(a + e)(a + f) =(b + c)(b + d)(b + e)(b + f) 임을보여라. ( 셈본중등중급도전문제 3..3)

180 80 대수 증명 K = ( c d e f) 3 + c 3 + d 3 + e 3 + f 3 =(a + b) 3 a 3 b 3 = 3ab +3a b =3ab(a + b) T = (cd + ce + cf + de + df + ef)(c + d + e + f) 로두자. (a + c)(a + d)(a + e)(a + f) (b + c)(b + d)(b + e)(b + f) =(a 4 b 4 )+(c + d + e + f)(a 3 b 3 )+(cd + ce + cf + de + df + ef)(a b ) +(cde + cdf + cef + def)(a b) 여기에 c + d + e + f = a b를대입하면 = ab 3 ba 3 +(cd + ce + cf + de + df + ef)(a b )+(cde + cdf + cef + def)(a b) = (a b)[ ab(a + b)+(cd + + ef)(a + b)+(cde + + def)] = (a b)[ K 3 = (a b)[ K 3 +(cd + + ef)( c d e f)+(cde + + def)] T +(cde + + def)] 이때 K 3 따라서 + T 를계산해보면 cde + cdf + cef + def 임을알수있다. (a b)[ K 3 T + K 3 + T ]=0 임을알수있고따라서좌변과우변은같다. 98. 양수 a, b, c 에대하여식 a 7a + b + b 7b + c + c 7c + a 이취할수있는값의범위를구하여라. ( 한국 003 차 -S7) 풀이 (KAIST 03학번김린기 ) b a = x, c b = y, a = z 로치환해주면식이간단해진다. xyz =, c x; y; z > 0 의제한식에서범위가다음과같음을보이자. 7 < 7+x + 7+y + 7+z 3 8 ( 오른쪽부등식 ) 양변에 8(7 + x)(7 + y)(7 + z) 를곱하고정리하면 (x + y + z)+8(yz + zx + xy) (x + y + z)+3 7(yz + zx + xy)+3xyz(= ) 다시정리하면다음의식과동치가된다. 산술 - 기하평균부등식에서 44 35(x + y + z)+3(yz + zx + xy) x + y + z 3 3p xyz =3; yz + zx + xy 3 3p x y z =3 이므로이로부터위부등식이성립함을확인할수있고, 등호는 x = y = z(= ) 일때, 즉 a = b = c 일때성립한다.

181 . 대수고급문제 8 ( 왼쪽부등식 ) 일반성을잃지않고 x y z 라하자. 그럼 x, z 임은당연하다. 만일 y 이면 7+y + 7+z = 4 이므로 OK. y> 이면 x>, zx < 이기도하므로 7+x + 7+z > 7+x + x 7x + zx > +7x + x 7x + = +x +7x > 7 로항상부등식은성립한다. 또한, x; y!이면 z! 0 이되고, 이때준식은 로수렴한다는것도알 7 수있다. } 풀이 (KAIST 과학영재센터연구원고봉균 ) ( 오른쪽부등식 ) 준식을 A 라하고, x = e X, y = e Y, z = e Z 로다시치환하자. xyz = 이므로 X + Y + Z =0 이된다. 이제 F (T )= 7+e T 라하면오른쪽부등식은 A 3 = F (X)+F (Y )+F (Z) 3 µ X + Y + Z F 3 ( ) 의 Jensen 부등식꼴이됨을볼수있다. F 를미분하면 F 0 (T )= (7 + et ) 0 (7 + e T ) = et (7 + e T ) F 00 (T )= ( et ) 0 (7 + e T ) ( e T )[(7 + e T ) ] 0 (7 + e T ) 4 = et (7 + e T )+e T e T (7 + e T ) 3 = et (e T 7) (7 + e T ) 3 따라서, e T < 7 이면 F 00 (T ) < 0 이고, 그범위에서위로볼록한함수이다. 즉, x; y; z < 7 이면 ( ) 의 Jensen 부등식에의해오른쪽부등식이성립한다. 그렇지않은경우라면, 일반성을잃지않고 x 7 이라하면, A = 7+x + 7+y + 7+z = 5 4 < 3 8 으로역시성립한다. ( 왼쪽부등식 ) 먼저 x; y!이면 A! 7 임을알수있다. 귀류법으로 A 7 인경우가있다고가정해보자. 일반성을잃지않고 z가제일작다고하자. 그럼 z x + 7+y = A 7+z 7 7+z 7 7+ = 7+7 이므로, x도 y도 7 보다작아서는안된다. 그럼 z = xy 이어야한다. 이방법을확장하자. 74 <z ( ) 7n+ 7 n 인자연수 n 이존재한다 a = 7+7 a 를이용하면 7+x ; 7+y 7 7+z 7 7+ = n n + 따라서, x; y 7 n + 7 n 이어야하고, 그럼 z = xy 7n+ 이어야한다. 이것은 ( ) 에모순. 따 라서, 가정 A 7 이틀렸고, 항상 A> 7 이다. }

182 8 대수 풀이3 ( 대덕중 3학년이태희 ) 오른쪽부등식만확인하자. A< = 3 7 은쉬우므로 B = 3 A(> 0) 7 로두자. µ 7B =3 7A = 7a + = b 7a + b 7a + b + c 7b + c + a 7c + a 이된다. 이제 B 3 56 을보이면되겠다.Cauchy부등식으로( 분모를이항하여생각 ) (a + b + c) 7B (7a + b)b +(7b + c)c +(7c + a)a = a + b + c +bc +ca +ab x + y + z +7yz +7zx +7xy = a + b + c bc ca ab a + b + c +7bc +7ca +7ab 3 8 따라서, A = 3 7 B = 3 8 이성립한다. } 99. R 을실수전체의집합, R + 를양의실수전체의집합이라하자. y>x 인임의의양의실수 x; y 에대해 f(y) > (y x)f(x) 을만족시키는함수 f : R +! R 은존재하지않음을보여라. ( 아일랜드 003-5) 증명 ( 과천중 학년함태준 ) 먼저이함수가존재한다고가정하자. 그때y =,x =을대입하면 f() > ( )f() > 0 이란식을얻을수있다. f() = k>0 이라고하자. 이번엔 y =+m, x = 를대입해보자. 그러면 f( + m) >mk 이다. m을 m> k 인것으로잡는것을생각하면, f(h) > 인 h를찾을수있다. µ f h n > n+ ( ) 이성립함을증명해보자. n =0일때f(h +)>f(h) > 이므로성립한다. n일때성립한다고가정하자. y = h n+ x = h n 을대입하면 µ f h n+ > µ n+ f h n > n+ (n+ ) = n+3 도성립하므로, 수학적귀납법으로 ( ) 이증명되었다. 이제 y = h +3,x = h n 이라하자. y x =+ n > 이되므로, f(y) >f(x) > n+4 이성립해야하는데, 이것은모든 n 에대해항상성립하는것은불가능하다. 따라서, 문제의조건을만족하는함수 f : R +! R 은존재하지않는다. 00. a, b, c 가양의실수일때다음을증명하여라. ( 미국 003-5) (a + b + c) (b + c + a) (c + a + b) a + +(b + c) b + +(c + a) c +(a + b) 8

183 . 대수고급문제 83 증명 a + b + c = s 로치환하자. 그러면부등식은 f(x) = (s + x) x 이라하자. +(s x) (s + a) a +(s a) + (s + b) b +(s b) + (s + c) c +(s c) 8 f(x) = x +sx + s 3(x 3 sx + 3 s ) = 3 x +sx + s x 3 sx + 3 Ã s = sx +! Ã 3 s 3 x 3 sx + = sx +! 3 s 3 s 3 (x 3 s) + 9 Ã s x + 3 s! 3 9 s = ³ +4 3 s x 이것을이용하면 f(a)+f(b)+f(c) ³ +4 3 s a + ³ +4 3 s b + ³ +4 3 s c =4+4=8 0. x ;x ;y ;y ;z ;z 0 일때다음부등식을증명하여라. (ML 프로포절 3-) (x 3 + x 3 )(y 3 + y 3 )(z 3 + z 3 ) (x y z + x y z ) 3 증명우선 (x y z + x y z )=0이면자명하다.(x y z + x y z ) 6= 0이라고가정하고양변에 (x y z + x y z ) 를곱하면 (x y z + x y z )(x 3 + x 3 )(y 3 + y 3 )(z 3 + z 3 ) (x y z + x y z ) 4 ( ) 이고, 이것은문제의부등식과동치이다. Cauchy-Schwarz 부등식에의해 (x y z + x y z )(x 3 + x 3 ) (x p y z + x p y z ) (y 3 + y 3 )(z 3 + z 3 ) ( p y z 3 + p y z 3 ) 또 (x p y z + x p y z )( p y z 3 + p y z 3 ) (x y z + x y z ) 따라서,( ) 이참이고원래부등식이성립한다. 주이것은 Cauchy-Schwarz 부등식의확장에해당한다. 이와관련한글이 MathLetter 제60호 ( 혹은모음집제4권 ) 에실렸던적이있다. 0. ab + bc + ca = 이고 a; b; c > 0 일때, a + b + ab + b + c + bc + c + a + ca > 임을증명하여라. (ML 프로포절 3-5)

184 84 대수 증명다음과같이하면된다. a + b + ab + b + c + bc + c + a + ca > (a + b) + (b + c) + (c + a) a + b b + c + b + c c + a + c + a a + b (a + b + c) = (a + b)(b + c)(c + a) (ab + bc + ca)(a + b + c) = (a + b)(b + c)(c + a) = 3abc + a b + a c + b a + b c + c a + c b abc + a b + a c + b a + b c + c a + c b > 03. x + + y + + =을만족하는양수 x, y, z에대하여 z + x x + + y y + + z z 임을보여라. (ML 프로포절 38-) 증명 (KAIST 과학영재센터연구원고봉균 ) A := x+ + y+ + = z+ 그리고이식의양변을 3에서빼면B := x x+ + y y+ + z = z+ 이다. 양수 t에대해 t t + 5 t t + () 5t +5t t 3 +8t + t +8 ( ) () t 3 +3t 4t +8 0 () (t ) (t +7) 0 이므로이것은항상성립하는식임을알수있다. 따라서,( ) 의부등식의 t 에 x, y, z 를대입하여얻은세식을모두더하면 x x + + y y + + z z + 5 x x + 5 y y + = 5 B A = z z + 으로문제의부등식을얻는다. 주위의증명에서 ( ) 의식을찾는요령은다음과같다 : 우선 A 와 B 의식에서 t t + at+b t+ ( ) 꼴의부등식을찾을수있으면좋겠다는힌트를얻는다. 문제의부등식의등호성립조건을실험적으로검사해보면 x = y = z = 일때라는것을추측할수있고, 그럼 ( ) 에서 t = 일때등호가성립해야하므로두식이 t = 일때접하도록하면된다. 이로부터간단한미분을사용하면 a 와 b 를금방구할수있다. 04. 임의의실수 x, y 에대해 f(f(x + y)) = f(x + y)+f(x)+f(y)+xy 를만족하는함수 f 를모두찾아라. (ML 프로포절 38-3)

185 . 대수고급문제 85 풀이 () 식에 y =0 을대입 f(f(x)) = f(x)+f(0) () 다시 () 식에서 () 에의해 f(f(x + y)) = f(x + y)+f(0) = f(x + y)+f(x)+f(y)+xy 정리하면 f(x + y) =f(x)+f(y)+xy f(0) (3) g(x) =f(x) f(0) 이라하면 g(0) = 0 이고 (3) 식에의해 g(x + y) = g(x)+ g(y)+ xy (4) 이제 n 이자연수이면 g(nx) =ng(x)+ n(n ) x (5) 이성립함을수학적귀납법을이용해증명하자. (i) n =이면자명 (ii) n = k 일때성립한다고가정하면 n = k + 일때 g((k +)x) =g(kx + x) = g(kx)+g(x)+kx ((4) 식에의해 ) k(k ) = kg(x)+ x + g(x)+kx ( 귀납법의가정에서 ) (k +)k =(k +)g(x)+ x 따라서성립. 이제 (5) 식에두자연수 x; y(= n) 를대입하면 g(xy) =yg(x)+ y(y ) x (6) x; y 교환하여대입하면 (6) = (7) 이므로 yg(x)+ g(xy) =xg(y)+ y(y ) x = xg(y)+ x(x ) y (7) x(x ) y 양변을 xy 로나누면 (x; y 둘다자연수이므로 xy 는 0 이아니다 ) g(x) x x = g(y) y y F (x) = g(x) x x 라하자. 라하고위식에대입하면 F (x) =F (y), 즉 F ( 자연수 ) 는상수함수. F ( 자연수 ) c c = g(x) x x 따라서 g(x) = x + cx (x 는자연수 ) f(x) =g(x)+f(0) = x + cx + f(0)

186 86 대수 그러나이식을 () 에대입하면 f(f(x)) = f(x)+f(0) µ x µ x () f + cx + f(0) = + cx + f(0) + f(0) 다시한번전개해서 f(x) 를없애면 x 를변수로하는최대계수가 =8 인 4 차다항식이나온다. 그런데 x 는임의의자연수이므로위식은무한개의해를갖으나대수학의기본정리에의해서 4 차방정식의해는최대 4 개이므로모순. 따라서위의조건을만족하는방정식은존재하지않는다. } 05. a, b, c 는어떤삼각형의세변이고, a + b + c = 을만족한다. n 은정수일때, 다음을보여라. (APMO 003-4) np a n + b n + np b n + c n + np c n + a n < + np 증명 ( 유신고 학년김제민 ) lemma a b 일때 np a n + b n < a + b lemma 의증명 np a n + b n < a + b () an + b n < ³ a + b n. 이항정리로 ³ a + b n ³ n = b n + b n a + >bn +a n a = bn + a n 문제를증명하자. 일반성을잃지않고 a b c 라하자.lemma 에따라 np a n + b n + np b n + c n + np c n + a n <b+ a + c + a + np b n + c n r <a+ b + c + n n + np n =+ 위에서 b; c < 은삼각형의어느변도둘레의길이의절반에못미친다는것에서나온것이다. 따라서, 성립. 06. 식 f(x + y) f(x)f(y) = sin x sin y 를만족시키는함수 f : R! R 를모두구하여라. 단, R은실수전체의집합이다. (004 충남대경시 ) 풀이 ( 미국 Cornell 대학교수학과김현규 ) f 가해를가진다고가정하자. f(x + y) f(x)f(y) = sin x sin y ( ) ( ) 에 y =0을대입하면 f(x) f(x)f(0) = 0, 즉 f(x)( f(0)) = 0 만약 f(0) 6= 이라면, 모든 x에대해서f(x) =0이성립해야한다. 그런데 ( ) 에적용해보면 0= sin x sin y for all x; y R 이나오므로모순. 따라서 f(0) =

187 . 대수고급문제 87 ( ) 에 x = ¼, y = ¼ 를대입하면 f(0) f( ¼ )f( ¼ )=, 즉 ³ ¼ ³ f f ¼ =0 따라서, f(s ¼ )=0 이다 (s = 또는 s = ). ( ) 에 x = t s ¼, y = s ¼ 를대입하면 f(t) f(t s ¼ )f(s ¼ )= s cos t s, 즉 f(t) = cos t for all t R 즉, 답이있다면이식을반드시만족해야만한다. 이제이것을원식 ( ) 에대입해서성립하는지살펴보자. ( 좌변 )= cos(x + y) cos x cos y = sin x sin y =( 우변 ) 무리없이성립함을확인할수있다. 따라서이게유일한답이다. } 07. 상수함수가아닌함수 f(x) 가다음조건을만족시킨다. ( 가 ) 실수 x 에대하여 (x 6 x 5 + x 3 x +)f(x 4 )=f(x) 이다. ( 나 ) 실수 x 와 y 에대하여 f(x + y) = f(x)f(y) (xy )f(x)f(y)+f(x)+f(y) 이다. 함수 f(x) 의최대값을기약분수 n 으로나타낼때 0m + n 의값을구하여라. m ( 한국 004 차-S5) f(0) 풀이 ( 나 ) 에 x = y =0을대입하면 f(0) = f(0). 분모가 0일수없으므로 f(0) 6= 0이 +f(0) 고, 따라서약분하고정리하면 f(0) = 임을얻을수있다.( 나 ) 에서 x = y 일때분모가 0일수없으므로항상f(x) 6= 0이다.( 가 ) 의 x에 x를대입하면 가되고, 이것을원래의 ( 가 ) 의식과비교하면 (x 6 + x 5 x 3 + x +)f(x 4 )=f( x) (x 6 + x 5 x 3 + x +)f(x) =(x 6 x 5 + x 3 x +)f( x) () 이다. 또,( 나 ) 의 y 에 x 를대입하고분모를이항해정리하면 () 과 () 에서 f( x) 를소거하자. (x +)f(x)f( x) =f(x)+f( x) () (x +)f(x) = x6 x 5 + x 3 x + x 6 + x 5 x 3 + x + + (x 6 +) = x 6 + x 5 x 3 + x + x 4 x + f(x) = x 6 + x 5 x 3 + x + = x + x + 따라서, f(x) 가최대일때는분모 x + x +=(x + ) 이최소인 3 4 일때이고, 그럼 f(x) 의최대값은 4 3 이다. } 08. 자연수 n 과양의실수 a i 들에대해다음부등식이성립함을보여라. (ML 프로포절 4-3) a + a + + a n a a a n a n + a n + + a n n

188 88 대수 증명산술 - 기하평균부등식에의해다음식들이성립한다. 변변더하면 n a a a n a n n a a a n a n a n. n a a 3 a n a n µ a (n ) + + a a a n a n + a n + a n + a n 3 a a a n + + a n n + + a n n + + a n n à (n ) a n + a n n! + + a n n 따라서성립. 등호는 a = = a n 일때. 09. a, b, c 는양의실수들이다. 다음부등식을증명하여라. ( 미국 004-5) (a 5 a +3)(b 5 b +3)(c 5 c +3) (a + b + c) 3 증명 ( 대전전민중 3학년정명진 ) 문제의부등식을직접증명하는대신 (a 5 a +3)(b 5 b +3)(c 5 c +3) (a 3 +)(b 3 +)(c 3 +) (a + b + c) 3 를증명하겠다. 먼저왼쪽부등식은 x 와 y 가 에대한대소가같을때 (x )(y ) 0 이성립한다는것을이용하면된다. x = a 3, y = a 등으로하면 (a 3 )(a ) 0, 전개하면 a 5 a +3 a 3 + 가성립함을알수있다. 다음오른쪽부등식은다음의코시부등식을이용한다. (a 3 +)(b 3 +)(c 3 +)= (a )( 3 + b )( c 3 )(a + b + c) a + b + c f(p a 3 + p b 3 +)( p a + p b + c )g a + b + c fa + b + cg =(a + b + c) 3 a + b + c 따라서, 문제의부등식이증명되었다. 0. a ;a ;:::;a n 은서로다른양의실수이고 b ;b ;:::;b n 도서로다른양의실수이다. A = X a i a j ; B = X b i b j i<j n i<j n 일때 nx a i b i + p à n!ã X n! X AB a i b i i= i= i= 임을증명하여라. (ML 프로포절 45-3)

189 . 대수고급문제 89 증명 ( 경기도여주김병수 ) S = P a i, T = P b i, L = P a i b i 로쓰자. S =(a + a + + a n)+a T =(b + b + + b n)+b 이때증명할식은 L ST p AB 좌우변모두음이아니므로, 양변을제곱하여성립함을보이겠다. L (a + a + + a n)(b + b + + b n) =(S A)(T B) by 코시부등식 = S T AT BS +4AB S T 4 p AB ST +4AB by 산술-기하 =(ST p AB ) 양변을제곱근하면문제가성립한다.. 임의의실수 a; b; c > 0 에대해 (a +)(b +)(c +) 9(ab + bc + ca) 의부등식이성립함을증명하여라. (APMO 004-5) 증명 ( 대전과학고 학년김진현 ) 준식을전개해보자. a b c +(a b + b c + c a )+4(a + b + c )+8 9(ab + bc + ca) 이식을다음과같이구분해보자. (i) 3(a + b + c ) 3(ab + bc + ca) (ii) (a b +)+(b c +)+(c a +) 4(ab + bc + ca) (iii) a + b + c + a b c + (ab + bc + ca) 이세식이성립한다면이들을모두변변더하여문제의식이증명된다.(i) 의식은 (a b) +(b c) + (c a) 0 의전개식으로잘알려져있다.(ii) 는산술-기하평균부등식으로금방확인된다. 따라서, (iii) 만확인하면문제의증명이끝난다. (iii) 을 c에관한함수로보자. (a b +)c (a + b)c +(a + b ab +) 0 아래로볼록한그래프를갖는이차함수이므로, 판별식이 0 이하이면이부등식은항상성립한다. D 4 =(a + b) (a b +)(a + b ab +) = a + b +ab a 4 b a b 4 +a 3 b 3 a b a b +ab = (a b + ab) a b (a + b ab) = (ab ) a b (a b) 0 등호는우선 (i) 에서 a = b = c 여야하고,(ii) 를고려하면 a = b = c = 일때만성립함을알수있다.. x 0 = p 3 이고, n =0; ; ;::: 에대해x n+ = +ax n a x n 으로정의되는수열 (x n) 이 8을주기로갖는다. 가능한실수 a의값을모두찾아라. ( 폴란드 004/005 차-9) 풀이 cot 차각공식임. x n = cot(30 ± nµ). 단,cotµ = a. }

190 90 대수 3. 양수 x ;x ;:::;x n 이 x + x + + x n = 을만족할때, 부등식 +x + +x + x + + s < +x + + x n 3 µ x x x n 이성립함을보여라. ( 한국 005-J7) 증명 a 0 =,a =+x, a =+x + x, :::; a n =+x + + x n = 이라잡으면, y = x 의그래프에서다음과같이나타낼수있다. 여기에서작은직사각형들의넓이의합은사다리꼴 Aa 0 a nb 의넓이보다작으므로 x a + x a + + x n a < 5 n 8 이다. 코시-슈바르츠부등식을사용하면 µ µ x a a a n a + x a + + x µ n a n x x x n < 5 µ x x x n < µ x x x n 이므로문제의부등식의성립한다. 4. 실수 x ;x ;:::;x n 이 x + x + + x n = 을만족할때, 부등식 x x x n +x + +x x +x + + x n r n < 이성립함을보여라. ( 한국 005-S6) 증명 코시 - 슈바르츠부등식에의해 이므로 nx k= Ã X n! Ã n!ã x k X n! X +x k= + + x k x k ( + x k= k= + + x k ) x k ( + x + + < 임을보이면된다. x k ) y k = +x + + ; =y 0 y y y n = x k

191 . 대수고급문제 9 이라하자. nx x k ( + x k= + + = X n y x k ) k k= nx = µ y k y k y k (y k y k ) y k= k nx (y k y k )=y 0 y n = k= 그런데, 등호가성립하려면 y 0 = y = = y n 이되어야하므로곤란하다. 따라서, 문제의부등식이성립한다. 5. 임의의양의실수 a, b, c 에대해다음부등식을증명하고, 등호가성립할조건을밝혀라. ( 발칸 005-3) a b + b c + c a a + b + c + 4(a b) a + b + c 증명 ( 대전과학고 학년김진현 ) 이부등식을다음과같이변형할수있다. (a b) + b µ (a b) (a + b + c) + b (b c) c (b c) c + (c a) + a (c a) a 4(a b) a + b + c 4(a b) 이것은코시부등식과삼각부등식에의해다음과같이확인된다. µ (a b) (b c) (c a) (b + c + a) + + (ja bj + jb cj + jc aj) b c a (ja bj + jb aj) =4(a b) ja bj jb cj jc aj 등호가성립하려면, 코시부등식에서 = = 여야하고삼각부등식에서 c가 a와 b b c a 사이에있어야한다. 이것을열심히풀면, 등호조건은다음과같이나온다 : a = b = c 또는 c b = a p 5 c = 6. 실수 c> 가주어져있다. 양수 x ;x ;:::;x n 이 nx q x i + cx ix i+ + x i+ = p nx c + x i i= i= 을만족하면, c = 또는 x = x = = x n 임을증명하여라. 단, x n+ = x 이다. ( 폴란드 차 -4) 증명 c< 이면 (c +)( x i+x i+ ) x i + cx ix i+ + x i+ 이므로 ( 전개하면완전제곱꼴로정리됨 ) 끝. c> 일때도위의식이부등호의방향만바뀌게되어역시끝. 7. a, b, c 는 abc =8 인양의실수들이다. 다음을증명하여라. (APMO 005-) a p ( + a 3 )( + b 3 ) + b p ( + b 3 )( + c 3 ) + c p ( + c 3 )( + a 3 ) 4 3

192 9 대수 증명 ( 북서울중 3 학년류종하 ) +a>0, a + a > 0 임에서기하 - 산술평균부등식에의해 p +a 3 = q( + a)( a + a ) +a 가된다. 등호조건은 +a = a + a, 즉 a(a ) = 0 일때이므로 a = 일때. b, c 에대한식도마찬가지로얻어지므로, ( 좌변 ) 따라서, 다음부등식을증명하면충분하다. 4a (a +)(b +) + 4b (b +)(c +) + 4c (c +)(a +) 4a (a +)(b +) + 4b (b +)(c +) + 4c (c +)(a +) 4 3 이부등식은양변에 3 4 (a +)(b +)(c +) 를곱해모두전개하여정리하면다음과동치이다 (a b c 에는 abc =8 을대입 ). (a + b + c )+(a b + b c + c a ) 7 이부등식은산술 - 기하평균부등식 a + b + c 3 3p a b c =; a b + b c + c a 3 3p a 4 b 4 c 4 =48 에의해바로확인된다. 등호가성립할조건은 a = b = c = 이다. 8. 다음연립방정식의실수해 a, b, c, d, e 를구하여라. ( 폴란드 차 -) a = b 3 + c 3 b = c 3 + d 3 c = d 3 + e 3 d = e 3 + a 3 e = a 3 + b 3 풀이첫식에서 a ;b bc + c 0 이므로 b + c 0. 즉, a + b; b + c; c + d; d + e; e + a 0. 첫두식을빼면a + b; b + bd + d 0 이므로 a b 와 b d 는같은부호. 즉,(a b)(b d) 0. 이런식다섯개를모두더하여전개하면 (a + b + c + d + e )+(ab + bc + cd + de + ea) 0. 이것은 배하면 (a b) +(b c) +(c d) +(d e) +(e a) 0 과동치. 즉, a = b = c = d = e =0또는. } 9. n은양의정수이다. 다음을만족하는 (n에따라정해지는) 가장큰음아닌실수 f(n) 을찾아라 : a + a + + a n 이정수인임의의실수열 a ;a ;:::;a n 에대해, ja i j f(n) 인 i가존재한다. (APMO 006-) 풀이 ( 서울중앙중 학년이수홍 ) n이짝수인경우 : a i = 로하면0 f(n) 이어야한다. 그리고 ja i j 은항상0 이상이므로 f(n) =0 이최대. n이홀수인경우 : f(n) = n 임을증명하겠다. 먼저 a i = n+ n 로하면a + + a n = n+ Z 이므로 n f(n). 만약모든 i에대해ja i j < n 인수열fa ig가존재한다면 n <a i < n 즉, n + <a i < n +

193 . 대수고급문제 93 모든 i 에대해더하면 n <a + + a n < n + n, n+ 는연속한정수이므로 a + + a n 은정수일수없어서모순. 따라서, ja i j n 인 i가존재하면 f(n) = n. 그러므로 0 if n is even f(n) =( if n is odd n } 0. a 0 = 이고 n 에대해 a n + a n + a n a n + a 0 n + =0 을만족하도록정의된실수들의수열 a 0 ;a ;a ;::: 이있다. 초항을제외하면모두양수만나타남을보여라. (IMO-SL 006-A, 폴란드출제 ) 증명귀납법. n+ a n+ = a n + a n + + a 3 n+ n n+ (a n + a n + + a n )= n n+ n+ = n++ n < n+ 이므로 a n+ > 0.

194 94 대수

195 제 3 장 기하 3. 기하중급문제. 세방접원의반지름 r A, r B, r C 가주어졌을때, 삼각형의세변의길이를구하여라. 풀이 S = rs = r A x = r B y = r C z 이고, 헤론의공식 S = p xyzs 임에서 S = p rr A r B r C 도헤론의공식의한가지형태임. r = + + r A r B r 임에서 r을구하면이렇게 S도구할수있고, 그럼다시 C 처음에쓴공식으로 x, y, z를다구할수있어서완료. }. 세수선의발 ( 끝점 ) 이주어져있을때, 이에맞는삼각형을하나작도하여라. 그리고이삼각형 Y 의각변의길이를, Y 의주어진세수선의발을꼭지점으로하는삼각형 X 의세변의길이를이용해나타내어라. ( 헝가리 896-3) 풀이개요 ( 예각 ) 삼각형 Y 의수심삼각형이 X라면, X의방심삼각형이 Y 가된다. Y 가둔각삼각형이면? 내심과방심의연관성을볼수있을것이다. 내접원과방접원에의해나뉘는각변의길이는 s a, s b, s c 등으로표현된다는것과각의크기를알수있으므로, Y 의세변의길이도표현할수있다. } 3. 두삼각형이공통각을가지면, 삼각형의세각의 sine 값의합은나머지두각의차가작은삼각형쪽이더큼을증명하여라. 또이정리로부터, 세각의 sine 값의합이최대인삼각형의모양을결정하여라. ( 헝가리 898-) 증명공통각을 라하고, 나머지두각을,, 그리고이보다차가큰두각을 0, 0 이라하자. 그리고다음식과동치인식들을구해보자. sin +sin +sin <sin +sin 0 +sin 0 공통부분인 sin 를빼주고 sin 의합차공식으로정리하면, sin + cos < sin cos 0 0 이되고, + = = 80 ± 로일정하며또한그 sin 값은양수이므로, 결국 cos 0 0 < cos 이식을보이는것과같다. 그리고이것은 0 0 의절대값이 의절대값보다클때성립한다. 그것은주어진조건이므로, 증명이되었다. [ 주 ] 이문제는 sin, cos 함수들의좋은공식들을이용해서보였지만, 사실이문제는일반적인볼록함수들에대해모두얘기할수있는것이다. 그경우에는어떤풀이를생각할수있을까? }

196 96 기하 4. 어떤삼각형의넓이 T 와한각 가주어져있다. 각 의대변의길이 c 가가능한한짧아지도록다른두변의길이 a 와 b 를결정하여라. ( 헝가리 90-) 힌트 () ab 의값이고정되어있으므로, c 에대한 cosine 제 법칙이좋은식을만들어준다. () 이등변삼각형일때 c 가최소임을추측할수있으므로, 이등변삼각형이아닐때 c 가더커진다는것을보이면된다. (3) 고정된 와 c 에대해, 넓이 T 를최대화하는문제로변형하여생각한다. } 5. A, B, C 을각각삼각형 ABC 의꼭지점 A, B, C 에서내린수선의발이라하고, M 을이수선들의교점, 즉삼각형 ABC 의수심이라하자. A B C 이삼각형을이룬다고하자. 각점 M, A, B, C 는삼각형 A B C 의세변에 ( 혹은그의연장선에 ) 접하는어떤원의중심임을증명하여라. ABC 가예각삼각형일때와둔각삼각형일때의작용은어떻게다른가? ( 헝가리 909-3) 힌트수심삼각형의방심삼각형이자기자신이됨을알고있는가? } 6. Q 를원위의임의의점이라하고, P P P 3 P 8 을이원에내접하는정팔각형이라하자. Q 에서네지름 P P 5, P P 6, P 3 P 7, P 4 P 8 에이르는거리들의네제곱의합은 Q 의위치에관계없이일정함을증명하여라. ( 헝가리 9-) 증명원위의점Q에서 P P 5, P 3 P 7 에내린두수선은서로직교하며, 원의중심 O까지포함하여대각선의길이가원의반지름 r이되는직사각형을이룬다. 따라서, 피타고라스의정리에의해두수선의길이의제곱의합은 r 과같다. P P 6, P 4 P 8 에대해서도마찬가지이며, 따라서네거리들의제곱의합은 r 으로일정하다. 7. 삼각형 ABC 가어떤다각형의내부에완전히포함되어있다. 삼각형 ABC 의둘레가이다각형의둘레보다크지않음을증명하여라. ( 헝가리 95-) 증명반직선! AB,! BC,! CA 의연장이외부다각형과만나는점을각각 X, Y, Z 라하자. 그럼삼각부등식에의해 AB + BX AZ + ZX, BC + CY BX + XY, CA + AZ CY + YZ. 이세식을모두변변곱하고 BX + CY + AZ 를소거하면 AB + BC + CA XY + YZ+ ZX 이고이것은외부다각형의둘레의길이를넘지않는다. 8. 삼각형 ABC 의세변을연장한세직선에모두접하는네원을생각하자. 이중에서 A 와 B 의사이에서변 AB 와접하는두원을 k 와 k C 라하자. k 와 k C 의반지름들의기하평균은 AB 의길이의절반을넘지않음을증명하여라. ( 헝가리 97-3) 9. 임의의예각삼각형에대해,3 차원공간에서다음을만족하는점이존재함을증명하여라 : 이점에서이삼각형의한꼭지점과대변 ( 혹은그연장선 ) 위의한점을바라보는각이항상직각이다. ( 헝가리 938-3) 증명문제의예각삼각형을 ABC라하자. AB; BC; CA를각각지름으로하는세구면을그려그두교점중하나를 X라하자. 그럼 XA;XB;XC는서로수직이므로 XA;XB;XC는직교좌표계의세축을이룬다 (A축, B축, C축이라함 ). 직선 BC 위의임의의점을 D라하면XA랑 XD는항상수직이다 (A축과 BC 평면은수직이라서 ). 따라서 X가원하는점이고증명끝. 0. 예각삼각형 ABC 의변 BC, CA, AB 에서바깥쪽으로반원을각각하나씩그렸다. 이반원위에서 AB 0 = AC 0, BC 0 = BA 0, CA 0 = CB 0 이되는점 A 0, B 0, C 0 을각각작도하여라. ( 헝가리 939-3)

197 3. 기하중급문제 97 증명 ( 박민재 ) 위의그림과같이 x; y; z 를잡자. 삼각형 ABC 0 ;BCA 0 ;CAB 0 은직각삼각형이기때문에피타고라스의정리에의해서 x + y = c ; y + z = a ; z + x = b 이성립한다. 따라서 x = b + c a 이고, 제코사인법칙에의해 a = b + c bc cos A이므로 x = bc cos A () 가된다. 이제 B 에서 CA 에내린수선의발을 E, 수선과반원과의교점을 B" 라고하자. AE = c cos A; CA = b 이고직각삼각형 CAB" 에서 AE AC = AB" 이성립하므로 AB" = bc cos A () 이다.() 과 () 에서 B 0 = B" 임을얻는다. 즉 B 0 은 B 에서 CA 에내린수선과반원과의교점이다. 마찬가지로 A 에서 BC 에내린수선과반원과의교점, C 에서 AB 에내린수선과반원과의교점은차례로 A 0 ;C 0 이된다. 수선의작도방법은생략한다. 증명 ( 최재연 ) AB 0 = AH AC = AH AH! AB 0 = AC 0 = AH 3 AB = AC 0

198 98 기하 마찬가지방법으로 BC 0 = A 0 B; A 0 C = B 0 C ) 문제조건을만족한다.. C 는직선 OZ 위의고정된점이고, U 와 V 는각각직선 OX 와 OY 위를움직이는점이다. OX, OY, OZ 는서로수직이다 (3 차원공간 ). PU, PV, PC 가서로수직이되는점 P 의자취를구하여라. (Putnam 947-B6) 증명 ( 장도한 ) 직선 OX, OY, OZ를각각유클리드 3차원공간 E 3 상의 x, y, z-축이라하고, P (x; y; z), C(0; 0;c), U(u; 0; 0), V (0;v;0), c는상수, u; v ( ; ) 이라하면,! CP =(x; y; z c),! UP =(x u; y; z),! VP =(x; y v; z) 이다. 그런데,! CP?! UP 이므로,0=! CP! UP = x(x u) +y + z(z c). 즉, ux = x + y + z cz 이다. 비슷하게,! CP?! VP이므로,0=! CP! VP = x + y(y v)+z(z c). 즉, vy = x + y + z cz 이다. 또한,! UP?! VP이므로,0=! UP! VP = x(x u)+y(y v)+z 이다. ) 0=x(x u)+y(y v)+z = x (x + y + z cz)+y (x + y + z cz)+z ) x + y +(z c) = c ) P 의자취는 c =0 일때, 한점 P (0; 0; 0) 이고, c 6= 0 일때, 중심이 C(0; 0; 0) 이고반지름이 c 인구면상의점이다.. 정삼각형 ABC 가있고점 P 가 AP =,BP =3 을만족한다. CP 의최대값을구하여라. ( 소련 96-6b) 힌트톨레미부등식 or 60 도회전 } 3. 네점 A, B, C, D 가있는데, A 와 B 를지나는임의의원과 C 와 D 를지나는임의의원이항상만난다고한다. 그럼이네점이모두한직선위에있거나아니면모두한원위에있음을증명하여라. (Putnam 965-B6) 증명 (Kalva) The trick is to take concentric circles. If AA is not parallel to BB then their perpendicular bisectors must intersect at some point O. Take circles centre O through A, A and through B, B. These must either coincide, in which case the 4 points lie on a circle, or have no points in common. In the case where AA is parallel to BB. Assume they do not coincide (otherwise the 4 points would be collinear). Then take a point O on the perpendicular bisector of AA on the opposite side of AA to BB and sufficiently distant from AA that the circle through A, A centre O only extends less than halfway towards the line BB. Similarly, take a circle through B, B which extends less than halfway towards AA. Then these circles will not meet. 4. 평면위에네점A, B, C, D가평면상의임의의점P에대해서 PA+ PD PB + PC 를만족하도록주어져있다. B와 C가선분AD 위의 AB = CD 를만족하는두점임을증명하여라. ( 소련 966-b) 증명 ( 박민재 ) 삼각부등식에의해 AD AC + DC AD AB + DB 두식을더하면 AD (AB + AC)+(DB + DC) ()

199 3. 기하중급문제 99 ( 등호는 A, B, C, D 모두한직선위에있을때.) 조건에서 P = A; D 일때 AD AB + AC AD DB + DC 두식을더하면 AD (AB + AC)+(DB + DC) () () 과 () 에서 AD =(AB + AC)+(DB + DC) 이므로 A, B, C, D 모두한직선위에있고, AD = AC + DC, AD = AB + DB 임을알수있다. 즉 A, B, C, D 순서대로한직선위에 AB = CD 를만족. 이때 P 가어디에있든지 PA+ PD PB + PC 임은쉽게확인할수있다. 5. 단위정사각형 ABCD 가있다. 한마름모의세꼭지점이변 AB, 선분 BC, 선분 AD 위에하나씩있다. 이마름모의네번째꼭지점이놓일수있는자취의넓이를구하여라. ( 소련 967-9) 풀이 AB의중점을원점O, 마름모의중심 M의좌표를 ( a; 0) 이라할때, 마름모의네번째꼭지점은 a< 일때두점 (a; a ) 사이를움직이고, a> 일때도대충그것과상보되는영역을움직여서합체하면자취의넓이는. } 6. 한직선이삼각형 ABC의세변BC, CA, AB와각각점A, B, C 에서만난다. 선분 AB의중점을중심으로한 C 의대칭점을 C 라하고, 마찬가지로 AC의중점을중심으로한 B 의대칭점을 B 라하자. 직선 B C 와 BC가 A 에서만난다고할때, sin B A C sin C A B = B C 임을증명하여라. ( 헝가리 97-) B C 증명 B C B X가평행사변형이되는점 X를잡으면C X k BC ( 중점연결선에서떨어진거리를좌표적으로생각해보면당연함 ). 이제 4B C X에서 sine 법칙. 7. 서로다른네직선 L, L, L 3, L 4 가한평면위에주어져있다. L 과 L 는각각 L 3 와 L 4 에평행하다. 네직선에이르는거리의합이항상일정하도록움직이는점의자취를구하여라. ( 캐나다 97-9) 풀이거리 r만큼떨어진평행한임의의두직선이있을때, 임의의한점P에서두직선사이의영역 A에이르는거리를 d라하면 (P 가 A 안에있으면 d =0), 이점에서두직선에이르는거리의합은 X = r +d 가된다. L 과 L 3 사이의거리, L 와 L 4 사이의거리를각각 m, n 이라하고, 두직선사이의영역을각각 A, B 라하자. 그리고, 임의의한점 P 에서영역 A, B 에이르는거리를각각 d, e 라하자. 그럼네직선에이르는거리의합은 X = m + n +d +e 이다. 이값이변하지않는자취를찾는것이므로, d + e(= s) 가항상일정한자취를찾는것과같다. 우선 s =0 인자취는 A \ B 의평행사변형영역이된다. s>0 인경우는영역 A \ B c, A c \ B 에서는각각 d = s, e = s 인평행선분들이되고, 영역 A c \ B c 에서는앞에서구한선분들의끝점을잇는선분이되어전체자취는 A \ B 를적당한간격으로둘러싼팔각형꼴이된다.

200 00 기하 마지막으로매우특수한경우인, 네직선 L, L, L 3, L 4 가모두평행한경우를살펴보자. 네직선의이름을재정렬하면 A \ B 가공집합이아니라고보아도된다. 그럼 s =0 일때는영역 A \ B 가자취가되고, s>0 이면 A \ B 바깥의평행한두직선이된다. } 8. 주어진두원이두점 P, Q 에서만난다. 점 P 를지나는선분 AB 는양끝점이각원의원주위를움직인다고한다. AP PB 가최대가되도록하는선분 AB 의위치를작도하여라. ( 미국 975-4) 풀이두원의중심과반지름을각각 O, O, r, r 라고하자. \O PO = (fixed) 라고하면, AP = r cos \O PA; BP = r cos \O PB ) AP BP =r r (cos(\o PA+ \O PB)+cos(\O PA \O PB)) =r r (cos(¼ )+cos(\o PA \O PB)) 따라서 AP BP 값이최대가되려면, \O PA = \O PB 이면된다. A P B O O Q 작도법 : \O PO 의이등분선을그린후그것과수직이되도록 AB 를그리면된다. } 9. 원형의투기장의중심에서있는개가벽의토끼를본다. 토끼는벽주위를한방향으로돌고개는투기장의중심에서토끼를잇는반지름위를유지하며토끼와같은속력으로토끼를쫓는다. 토끼가투기장의 4 을돌때개가토끼를잡음을보여라. ( 캐나다 979-4) 증명토끼가 A지점에서출발해벽를따라투기장의 4 지점인 B지점를향하여도망간다면개는 O에서출발하여 OB를지름으로하는원M의A쪽반원을따라간다는것을증명하자. 문제의조건에의하면토끼의움직임이정해지면개가움직이는경로는유일하게결정되므로, 이반원의경로가문제의조건에부합한다는것만보이면된다. 토끼의위치 P 에대해 OP 와반원 M 의교점 Q 에항상개가위치한다면개의속력이늘토끼의속력과일치하게된다는것을확인하면되고, 이것은또개가움직인거리가토끼가움직인거리와같다는것과동치이다. 따라서호 OQ 의길이와호 AP 의길이가같음을보이면된다. \OMQ =\OBQ =\AOP 이고, 원 O 의반지름이원 M 의반지름의 배이므로두호의길이는같다. 따라서, 개는이반원 M 의원주를따라가게되고 B 에서토끼를잡는다.

201 3. 기하중급문제 0 0. 반지름이 인원에내접한사각형의네변의길이의곱은 4 를넘지않음을증명하여라. ( 폴란드 차 -5) 증명톨레미한방. p abcd ac + bd = xy (R)(R) =4.. 고정된각 A 와고정된반지름 r 의내접원을갖는삼각형중에둘레가최소인것을구하여라. ( 캐나다 980-3) 풀이이삼각형의둘레의길이는다음과같다. p =r cot A +r cot B +r cot C 여기서첫째항은상수이므로, 둘째항과셋째항의합을최소로하면된다.cot는구간(0; ¼ ) 에서아래로볼록이므로,Jensen부등식에의해 cot B +cot C cot B + C =cot ¼ A 가성립하고등호는 B = C } 일때성립한다. 즉, 둘레가최소인것은 \B = \C 인이등변삼각형이다.. 삼각형 ABC 의꼭지점 C 에서의내각의이등분선과외각의이등분선의길이가같으면 AC +BC =4R 임을증명하여라. 단, R 은외접원의반지름의길이이다. ( 불가리아 98 3 차 -5) 증명 ( 김두형 ) 사인정리에의해 BC= sin A = AC= sin B =R, AC + BC =4R (sin A +sin B) 즉, sin A +sin B = 임을보이는것과동치이다. C 에서 AB( 또는 AB 의연장선 ) 에내린수선의발을 K 라하고점 C 의내각의이등분선, 외각의이등분선과 AB 와의교점을각각 D, E 라하자. 이때 \BCK = \DCK \DCB =45 ± \DCB \A = \CDB \ACD =45 ± \ACD 또 \ACD = \DCB 이므로 \BCK = \A 이다. 즉 4ACK 와 4CBK 는닮음이다. 즉, \CBK = \ACK. sinb =sin(80 ± B) 이므로 sin B =sincbk =sinack =cosa. 따라서이식은자명하다. 3. 반지름이 r 인원이있고, 원위에주어진점 P 에접하는직선 ` 이있다. 원위를움직이는점 R 에서 ` 위에내린수선의발을 Q 라하자. 삼각형 PQR 의넓이의최대값을구하여라. ( 캐나다 98-)

202 0 기하 풀이직선 OP 에대한 R 의대칭점을 S 라하자. 그럼 j4prsj =j4pqrj 이므로, 4PRS의넓이가최대인경우를구하는것과같다. 원에내접하는삼각형의넓이는정삼각형일때최대이고 4PRS가정삼각형일수있으므로, 4PQR의최대넓이는그정삼각형의넓이 3p 3 4 r 의절반인 3p 3 8 r 답 } 별해 O 에서 RQ 에내린수선의발을 M 이라하고, x = RM, S = j4pqrj 이라하자. 그럼 S = PQ QR = p r x (r + x) S가최대일때는S 이최대일때와같다. f(x) =4S =(r x )(r + x) =(r x)(r + x) 3 f 0 (x) = (r + x) 3 +3(r x)(r + x) =(r + x) (r x) 와 q 같이미분을이용하면 x = r= 일때f 0 (x) = 0 으로 f(x) 가최대임을알수있다. 이때S = r r 4 (r + r )= 3p 3 8 r 답 } 4. 이등변삼각형 ABC의밑변BC의중점을 O라하자. O를중심으로하고두변 AB, AC에접하는원을그렸다. P 와 Q는각각AB와 AC 위에잡은점들이다. PQ가이원에접한다고할때, 다음을증명하여라. µ PB CQ = BC 그리고, 역도성립하는지논의하여라. ( 호주 98-3) 증명 ( 임준혁 ) 원과 AB; AC와의접점을각각 X; Y 라고하자.( 그림참조 ) 그리고 PQ와원과의접점을 Z라하자.

203 3. 기하중급문제 03 4XPO 4ZPO 이므로 \XOP = \ZOP 이고, 같은식으로 \YOQ = \ZOQ 이다. 그리고 \B = \C = a 라할때, \XOB = \YOC = 90 ± a 이다. 그러면 \XOY = a 임을알수있고, 3 번째줄에의해서 \POQ = a 이다. 여기서세삼각형 PBO;POQ;OCQ 가 AA 닮음을알수있다. 그럼 BP CQ = BO CO 이고, O 는 BC 의중점이므로명제가성립한다. 역은어떤점 P 를잡을때조건을만족시키는점 Q 은유일하므로, 역도성립한다 ± =n 의크기를갖는각이주어져있다. n 이 3 으로나누어떨어지지않으면, 이각을유클리드작도 ( 자와컴퍼스 ) 로 3 등분할수있음을증명하여라. ( 미국 98-) 증명 3과 n이서로소이므로3x + ny = 을만족하는정수 x, y가존재. 양변에 60 n 을곱하면 x +60y = n n. 6. P 는한변의길이가 인정사면체 ABCD 안의한점이다. 다음의식이성립함을보여라. d(p; AB)+d(P; AC)+d(P; AD)+d(P; BC)+d(P; BD)+d(P; CD) 3 p 또, 등호는 P 가 ABCD 의무게중심일때만성립함을보여라. 단, d(p; XY ) 는점 P 에서직선 XY 에이르는거리이다. ( 오폴 98-8) 증명정사면체가정육면체에 /3만큼들어차게내접한그림에서,P에서각모서리에이르는거리는 P에서정육면체의각면에이르는거리보다크거나같음. 등호는 P 가정육면체의중심일때. 7. 삼각형의넓이는그것의변의길이들로부터구해진다. 그럼사면체의부피는그것의면의넓이들로부터구할수있는가? ( 캐나다 983-3) 풀이명된다. 구할수없다. 대응하는각면의넓이가같은두사면체의부피가다른경우가있음을보이면증 네면이모두합동이어서네면의넓이가모두같은사면체를생각해보자. 정삼각형으로만들면정사면체가되어고정된부피를갖고, 꼭지각이 90 ± 에가까워지는같은넓이의이등변삼각형을이용하면매우납작하여정사각형이나다름없고부피가점점 0 에가까워지는사면체를얻는다. 따라서, 두사면체의부피가다름을알수있다. } 8. 네변의길이와두대각선의중점을연결하는선분의길이가주어졌을때이에맞는사각형을작도하여라. (Towns 983 가을 J3) 풀이 ( 김대성 ) 일단, 사각형의네변의길이가순서대로 ( 반시계방향으로 ) 주어졌다고생각하겠다. 그길이를 a; b; c; d라고하고, 두대각선의중점을연결한선의길이를 m이라고하면 ( 이문제풀이에서변의길이를연속해서적을때는반시계방향으로있는것으로생각하겠다.) () a ;m; c 을세변의길이로하는삼각형 ( 평행사변형 ) 을작도한다. 그평행사변형을 PMRN이라고하자. () b ; d ;m 을세변의길이로하는삼각형 ( 평행사변형 ) 을작도한다. 그평행사변형을 QMSN이라고

204 04 기하 하자. (3) M 을지나고, M 을중점으로하며, QP 와평행하고길이가 배가되는선분 BD 을작도하고, N 을지나고, N 을중점으로하며, PS 와평행하고길이가 배가되는선분 AC 을작도하자. 증명개요 () 작도과정에서사각형 QBMP 가평행사변형임을쉽게알수있다. 그러므로 BQ==MP;BQ = MP = a () 또, 4PNS 4RMQ 이고 QR==PS 을알수있다. 그러므로, 사각형 QRNA 도평행사변형이다. 즉, RN==QA; RN = QA 이다. RN==MP;RN = MP 이므로, A; Q; B 는일직선상에있고 AB = a 이다. (3) 비슷하게다른변에도적용하면된다. } 9. 단위면적을갖는예각삼각형이있다. 이삼각형의각꼭지점으로부터 4p 이상떨어진거리에있는점이 7 삼각형의내부에존재함을보여라. ( 캐나다 984-4) 증명 예각삼각형이므로외심이삼각형의내부에있다. 외접원의반지름을 R이라하면 R p 4 임 7 을보이면된다. 주어진원에내접하는삼각형중에서가장넓이가큰것은정삼각형임은잘알려진사 실이다 ( ). 이것을역으로이해하면, 고정된넓이를갖는삼각형중에서외접원의반지름의길이가가 p 장짧은것은정삼각형일때가된다. 한변의길이가a인정삼각형의넓이는 3 4 a 이므로, 단위면적의 정삼각형의한변의길이는 a = p 4 가되고, 외접원의반지름의길이는 3 R0 = p a = 3 4p. 따라서, 7 R R 0 으로확인이된다. 주 ( ) 의사실을증명해보자. 원에내접하는삼각형중에서둔각삼각형의경우는그보다넓이가큰

205 3. 기하중급문제 05 예각삼각형을바로찾을수있으므로제외해도된다. 세변의중심각을각각,, 라하자. 그럼삼각형의넓이는다음과같다. S = R (sin +sin +sin ) R 3sin + + = 3p R sine 함수는구간 [0;¼] 에서위로볼록이므로 Jensen 부등식에의해위와같은최대값을갖고, 이최대의경우는 = = =0 ± 로정삼각형의경우이다. 30. 세변의길이가각각 6, 8, 0 인삼각형이있다. 이삼각형의둘레와넓이를동시에이등분하는직선이유일하게존재함을보여라. ( 캐나다 985-) 증명우선이삼각형은직각삼각형이고, 둘레의길이와넓이가모두 4이다. 넓이를이등분하는직선이이삼각형의한꼭지점을지난다면대변의중점도지나야하고, 이삼각형은어느두변의길이도같지않으므로그럼이직선은둘레를이등분할수없다. 따라서, 구하는직선은꼭지점을지나지않고, 두변을지나는경우만살피면된다. 위와같이 3 가지경우로나누자. 각각전체삼각형의넓이에대한작은삼각형영역의넓이의비를구하면다음과같다. R (x) = x 8 x 0 R (x) = x 6 x 0 R 3 (x) = x 8 x 6 ( <x<8) ( <x<6) (6 <x<8) 이중에서 이되는것이있는지찾으면된다. x( x) R (x) = 80 x( x) R 3 (x) = 48 < < > > 이므로 R, R 3 은 이될수없고, R (x) = 이되는경우는 x( x) =30, 즉 x x +30=0, <x<6 에서 x =6 p 6 의해가유일하게존재한다. 주일반적으로, 임의의볼록다각형혹은볼록폐곡선에대해이도형의둘레와넓이를동시에이등분하는직선은항상존재한다. 증명은다음과같이하면된다. 이도형의둘레위의임의의점 P 에대해, P 로부터둘레를따라정확히 을움직인점을 P 0 으로쓰기로하자. 둘레를이등분하는직선 XX 0 을하나택하자. X에서 X 0 을바라볼때직선의왼쪽영역의도형

206 06 기하 의넓이와직선의오른쪽영역의넓이를 X 를움직여가면서비교하자. 직선이 XX 0 일때두넓이중어느한쪽이컸다면, 직선이 X 0 (X 0 ) 0 일때는 X 00 = X 가되어영역이서로뒤바뀌므로반대쪽넓이가더크다. 그리고, X 에서둘레를따라 X 0 까지옮겨가는동안이직선양쪽의넓이는연속적으로변한다. 따라서, 중간값정리에의해양쪽넓이가같아지는때가있다. 단, 이런직선이항상유일하게존재하지는않는다. 그리고, 넓이가쪼개어져도된다면굳이볼록일필요도없다. 3. P 과 P 는둘레의길이가각각 x, y이고 985개의변을가진정다각형이다. P 의각각의변은원주의길이가 c인한주어진원에접하고, P 의각각의점은이원의원주위에있다. x + y c 임을증명하여라. (0 µ< ¼ 일때tan µ µ 임을이용해도좋다.) ( 캐나다 985-3) 증명 P, P 의한변의길이를각각 a, b라하고, 원의반지름의길이를 r이라하자. 그리고, 보다일반적으로, n = 985, µ = ¼ n 라하자. 그림에서 a =r tan µ, b =r sin µ 임을알수있다. x = na, y = nb, c =¼r =nrµ 이므로, x+y c 는 tan µ +sinµ µ 와같은식이다. t =tan µ 의치환과문제에서제시된성질을이용하면 tan µ +sinµ = t t + t +t = 4t t 4 4t 4 µ 로성립함을알수있다. 3. a, b, c 가한삼각형의세변의길이이고 c 의대각의크기가 이다. 다음을증명하여라. (Towns 985 봄 JA) c (a + b)sin 증명우변은 A, B 에서각 C 의이등분선에내린수선의길이의합. 최단거리부등식. 33. 어떤사면체에서마주보는대변의중점끼리이은세선분의길이가모두같고서로수직이라고한다. 이사면체는정사면체임을증명하여라. ( 이탈리아 987-) 증명 AB, BC, CD, DA의중점을각각 M, N, P, Q라하고, 대변의중점연결선의길이를 a라하자. MNPQ는원래평행사변형인데, 대각선이 a로길이가같고서로수직이므로정사각형이다. 그럼 MN = a= p, BD =MN = a p 이다. 즉, 이사면체의모든변의길이가 a p 로같으므로정사면체. 별증벡터로도풀수는있지만...; 34. 이등변삼각형 GKA 의밑변 GK 는길이가 b 이고, 변 GA 와 AK 의길이는각각 a 이다. 변 AK 의중점을 C, 삼각형 GCK 의외접원을 z 라하자. AK 의연장선상에있는점 Y 는다음과같은성질을만족한다 : YG 와 z 의교점을 E 라고할때, EY 의길이는 a= 이다. EC 의길이를 x, KY 의길이를 y 라할때, ay = x, xb = y 임을증명하여라. ( 호주 987-) 증명삼각형 CEY 와 GKY 가닮음에서xy = ab 는쉽게나오지만... 피타고라스와톨레미등으로막계산해버릴순있긴하지만... 좀현명한방법없으려나.

207 3. 기하중급문제 M을예각삼각형 ABC의수심이라하자. M과꼭지점들과의거리를각각 x; y; z로표시하고, 대응하는꼭지점의대변의길이를각각 a; b; c로표시한다. 다음을증명하여라. ( 통신강좌 988-B6) abc = ayz + xbz + xyc 풀이 D, ( 수학전공 년김용덕 ) 꼭지점 A; B; C에서내린수선이변 BC; CA; AB와만나는점을각각 A a F E b y x B D C c F; G 라하고수심을 M 이라하자. 사각형 CDME 에서 마찬가지로 \DME + \C = ¼ \DME = ¼ \C \AMB = \DME ( 맞꼭지각 ) )\AMB = ¼ \C () )\CMA = ¼ \B () )\BMC = ¼ \C (3) 예각삼각형이므로수심은내부에존재한다. 따라서 4ABC = 4AMB + 4BMC + 4CMA( 여기서각각은면적을나타낸다.) 이다. 즉 ab sin(\c) = xy sin(\amb)+ yz sin(\bmc)+ zx sin(\cma) (), (), (3) 에의해 = xy sin(¼ \C)+ yz sin(¼ \B)+ zx sin(¼ \A) = xy sin(\c)+ yz sin(\b)+ zx sin(\a) sin 법칙에의해 sin(\c) = c R ; sin(\b) = b R ; sin(\a) = a R ( 단 R 은외접원의반지름 ) 이므로 abc R = cxy R + ayz R + bzx R ) abc = cxy + ayz + czx: } 별해 ( 풍생고 년김민균 ) c y Y A(0, r) x b z B(- q, 0) a 0 C(p, 0) X

208 08 기하 삼각형 ABC를 XY 축위로이동시켜생각해보자. A를 Y 축의양의방향으로잡고, BC를 X축위에잡은다음A(0;r);B( q; 0);C(p; 0) 이라잡자.(p; q; r > 0): BM의직선의방정식은 y = p ³ r (x + q) 이고 M의좌표는 0; pq 이다. r 4ABC은예각삼각형이므로 0 < pq <r이다. r a = p + q; b = p p + r ;c = p r + q ; x = r pq r r r ;y = q + q p r ;z = p + q p r 이되므로 abc =(p + q) p p + r p r + q ayz + xbz + xyc s s = (p + q) q + q p r p + q p r ³ + r pq p p + r sp + q p ³ r r + r pq sq + q p p r r r + q = (p + q) q p r + q p p p + r + r r r (r pq) p p + r p p r + q r + r (r pq) q p p + r p r + q r = p µ p + r p qp r + q + q p r + pr qp r + qr pq r = p µ p + r p pr r + q + qr r = p p + r p r + q (p + q) =abc ) abc = ayz + xbz + xyc ( 증명끝 ) } 36. 볼록사각형 ABCD 에서, 삼각형 BCD, CDA, DAB, ABC 의외심을각각 A, B, C, D 이라하자. (i) A, B, C, D 은모두같은점이거나아니면모두서로다른점임을보여라. 후자의경우라고가정하고 A 과 C 은선분B D 에대하여서로반대쪽에있고, B 와 D 는선분A C 에대하여서로반대쪽에있음을보여라.( 이때볼록사각형 A B C D 이생긴다.) (ii) 삼각형 B C D, C D A, D A B, A B C 의외심을각각 A, B, C, D 라하자. 사각형 A B C D 는사각형 ABCD와닮음꼴임을보여라. ( 통신강좌 988-B) 증명 ( 한성고 년한두진 ) () i) 볼록사각형 ABCD 가원에내접할때삼각형 ABC, BCD, CDA, DAB 는같은원에내접하게되므로외심 A, B, C, D 은한점에일치하게된다. 사각형 ABCD 가원에내접하지않으면삼각형 ABC; BCD; CDA; DAB 는각기다른원에내접하게되어외심들은일치하지않게된다. ii) 사각형 B 0 A C 0 C 에서 \A B 0 C = \A C 0 C = \R 이고 \B 0 A C 0 + \B 0 CC 0 =80 ± 이므로 마찬가지로사각형 BB 0 B A 0 에서 \D A B = \B 0 A C 0 < 80 ± \D 0 D C 0 < 80 ± : \A 0 BB 0 < 80 ± 사각형 A B C D 은볼록사각형이되어 A 과 C 은선분 B D 을기준으로서로반대쪽에, B 과 D 은선분 A C 을기준으로서로반대쪽에있게된다. () i) 사각형 AA 0 C D 0 에서 \A 0 AD 0 + \A 0 C D 0 =80 ± 사각형 A 00 C D 00 C 에서 \D C B + \A 00 C D 00 =80 ± \A 0 C D 0 과 \D C B 이맞꼭지각으로같으므로 \AA 0 D 0 = \A 00 C D 00 ()

209 3. 기하중급문제 09 마찬가지로사각형 B 0 A C 0 C와사각형B 00 A C 00 A 에서 \C 00 A B 00 = \B 0 CC 0 () ii) 사각형 BB 0 B A 0 에서 \A 0 BB 0 + \B 0 B A 0 =80 ± 사각형 B C 00 A D 00 에서 \B 0 B A 0 + \D 00 D C 00 =80 ± \A 0 BB 0 = \D 00 D C 00 = \A D C ( 맞꼭지각 ) (3) 마찬가지로사각형 D 0 D C 0 D와 D A 00 B B 00 에서 \D 0 DC 0 = \A 00 B B 00 = \A B C ( 맞꼭지각 ) (4) (),(),(3),(4) 에서사각형 ABCD와사각형 C D A B 의대응하는각의크기가같으므로 ABCD C D A B 이다. A' B A D' C C" D B A B" D" D C C" A B B' D C' C 37. n과 d는 보다큰자연수이다. 다항식 f(x) =(x )(x ) (x n ) ( 단, i 는복소수 ) 가! d =인모든복소수! 에대하여 f(!x) =f(x) 를만족시킨다. 이때, n 의값을구하여라. ( 한국 988-8) 풀이 f(x) =a nx n + a n x n + + a x + a 0 이라하면 f(wx) =f(x) 이므로양변의계수를비 교하면 a m w m = a m (0 m n) 이어야한다. m이 d의배수가아니면 w 6= 인 w가존재하므로 a m =0이다. 즉, f(x) 는상수항 x d ; x d ; ::: 인꼴 의항만남는다. 따라서 f(x) 는 f(x) =(x d )(x d ) (x d r) 인꼴로나타낼수있다. 이때n = dr이다. w 4 =의한허근을 z =cos ¼ d + i sin ¼ d 라하면 dx z k =0: () k= 이고 (x d j) =(x z j )(x z j ) :::(x z d j )( 단, j d = j; z d =) 로나타낼수있으므로 f(x) = Y (x z k j ) k d j 로나타낼수있다. 따라서 () 에의하여 rx dx n = j= k= 0 Ã rx dx! z k j j A z k =0 j= k= 답 : + + :::+ n =0 }

210 0 기하 풀이 f(x) =a n x n + a n x n + + a x + a n 이라하면, f(wx) =f(x) 이므로양변의계수를비교하며 a m w m = a m (0 m n) 을얻는다. w d =이므로 m이 d의배수가아니면 w 6= 인 w가존재하므로 a m =0이다. 즉 f(x) 는상수항, x d ;x d ; 인꼴의항만남는다. 따라서 f(x) 는다음과같은꼴로나타낼수있다. f(x) =(x d )(x d ) (x d r) ( 단, n = rd) w d = 의한허근을 z =cos ¼ d + i sin ¼ d 라하면 이성립한다. r d = j 의한근을 r j = x j 라하면 x d j 는 dx z k =0 () k= x d j =(x zr j )(x z r j ) (x z d r j ) ( 여기서 z d = z 0 =) 와같이인수분해할수있으므로 f(x) = Y k d j r (x z k r j ) 로나타낼수있다. 따라서 () 에의하여 0 Ã rx dx rx dx! n = z k r j r j A z k =0 j= k= j= k= 답 n =0 } 38. 반지름이각각 r, R 이고같은중심을갖는두구면이있다 (r <R). 큰구면위에세점 A, B, C 를잡는데, 4ABC 의모든변이작은구면과만나도록하려고한다. 이것이가능할필요충분조건은 R r 임을증명하여라. ( 북유럽 988-3) 증명먼저이문제보다작은평면버전의문제를다음과같이풀어놓고, 이것을다시구면에응용하면됨. ² R r 이면정삼각형그려서자명. ² R>r 이면각변이정삼각형일때보다더중심에가까워야해서각내각이 60보다크므로내각의합이80도를넘어삼각형이형성될수없음. 구면에서도 R r 이면자명하고, R>r 이면어떤단면을잡아도 R 0 > r 0 이라서항상불가능. 39. P 는정사각형 ABCD 의외접원의호 CD 위의점이다. 다음을증명하여라. PA PB = PB PD PA PC ( 아일랜드 988-) 증명사각형 ABCD를 P 와함께복제한후 A를기준으로 90 ± 회전시켜 A 0 B 0 이 AD와일치하도록하고또 B를기준으로 90 ± 회전시켜 B 00 A 00 이 BC와일치하도록하면, 등적변형에의해 j4bpp 0 j = j4p 00 PP 0 j = j4p 00 PAj 가된다. 별증원에내접하는두사각형 ABCP 와 ABP D에서톨레미의정리를각각쓴후두식을잘정리하면된다. 40. 삼각형 ABC의세변의길이를BC = a, CA = b, AB = c 라하자. a +b =989c cot C 일때, cot A +cotb 의값을구하여라. (AIME 989-0)

211 3. 기하중급문제 풀이 wlog c =. Obviously a, b must be about 3.5 (remember a - b < ). Suppose angle B = 90o. Then a = b + c, so a = 995/, b = 994/. Hence cot B = 0, cot C = /cot A = 994/. Hence expr = 994. So if cot C/(cot A + cot B) is independent of a, then it must have value 994. Proving that it is independent is harder, but not strictly necessary. 답 994 } 4. 삼각형 ABC의외심을 O라하고, AB의중점을D라하자. 삼각형 ADC의무게중심을 E라할때, 두직선 OE와 CD가수직일필요충분조건이 AB = AC 임을보여라. ( 통신강좌 990-E4) 풀이 ( 제주오현고고봉균 ) 임의의점 X에대해X의좌표나벡터! OX를그냥X로쓰기로하자. 그리고, jaj = jbj = jcj = r 이라하자. 이제 E! CD = E (D C) =0이되는필요충분조건이 AB = AC 임을보이면된다. AC의중점을 F 라하면E는DF 를 :로내분하는점이다. E = 3 D + 3 F = A + 6 B + 3 C 그럼, µ E (D C) = A + 6 B + µ 3 C A + B C = 4 jaj + jbj 3 jcj + 3 A B 3 A C µ = 4 + r jajjbj cos \AOB jajjcj cos \AOC 3 =0+ 3 r (cos \AOB cos \AOC) 따라서, E (D C) =0() cos \AOB =cos\aoc () \AOB = \AOC () AB = AC 임이확인된다. } 4. 삼각형 ABC 의내심을 K, 변 AB, AC 의중점을각각 C, B 이라하고직선 C K 와직선 AC 의교점을 B, 직선 B K 와직선 AB 의교점을 C 라한다. 삼각형 ABC 와삼각형 AB C 의넓이가같을때 \CAB 의크기를구하여라. ( 통신강좌 990--) 풀이 BC = a, CA = b, AB = c, C A = b 0, AB = c 0, s =(a + b + c)= 라고놓자. 삼각형 ABC의내접원의반지름을 r이라고하면, 4ABC = rs 이다. 4AC B = AC AB AB AC 4ABC = c0 rs b 4AKB = c0 r 4AC K = cr 4 이므로, c0 rs b = c0 r + cr 4. 정리하면,(s b)c0 = bc. 즉,(a b+c)c0 = bc. 같은방법으로 (a+b c)b 0 = bc 를얻는다. 4ABC = 4AB C 이므로 bc = b 0 c 0. 두식을곱하면b c =(a b + c)(a + b c)b 0 c 0 이므로 a (b c) = bc 가된다. 그러므로 b + c a = bc =cos\cab. 따라서 \CAB =60±. }

212 기하 43. 삼각형 ABC 의각 A 의이등분선과 BC 와의교점을 D, BC 의중점을 M 이라한다. AD 의연장선위에 B 에서내린수선의발을 P, AM 의연장과 BP 와의교점을 E 라할때 DE k AB 임을증명하여라. ( 통신강좌 ) 풀이 BP 와 AC의연장선이만나는점을 F 라하면 \BAP = \FAP 이고 AP? BF 이므로 BP = FP. PM의연장선이 AB와만나는점을 G라하면BM = CM, BP = FP 이므로 PM k CF, 즉 GP k AF. 따라서, AG = BG. 4ABP 에서 Ceva의정리에의하여 BE EP DP AD AG GB = 이고, 따라서 BE EP = AD. 고로 DE k AB 이다. } DP 44. 직선 AB 의한쪽에서로다른 6 개의점 P ;:::;P 6 이있다. 여섯개의삼각형 ABP i ( i 6) 들이모두닮은꼴이면 P ;:::;P 6 은모두한원위에있음을보여라. ( 오폴 990-) 풀이주어진선분 AB의양끝점A; B에, 닮은삼각형을결정할세각 ; ; (0 < ; + + = ¼) 의방향으로사선을각각셋씩아래의그림과같이취하자. l i 와 m j 는서로마주보는각 \BAl i 와 \ABm j 의합i + j가 ¼보다작으므로한점에서만나게되며, 그교각은 k가된다.( 단, i; j; k는 ; ; 의순서를바꾼것 ). 여기서 i; j; k에따른순열로3!=6개의점이생기게되며이들이 P ; ;P 6 에각각대응된다. 만일 i = j라면 l i 와 l j ;m i 와 m j 가각각일치하여 6개의점중일치하는점들이생기므로, 6= 6= 6= 이어야하며이때서로다른 6개의점이생긴다. P ; ;P 6 대신 l i 와 m j 가만나는교점을 P i j라하고풀이를전개하자. 삼각형의닮음비에서모든순서쌍 (i; j; k) 에대하여 P ij A P ik A = P ji B P ki B = AB (= 일정 ) 이므로네점P ij ;P ik ;P i 0 j 0;P i 0 k 0으로이루어지사각형과네점 P ji; P ki ;P j 0 i 0;P k 0 i0으로이루어진사각형은각각어떤원에내접한다. 이런모든사각형들을고려하면사각형 P ij ;P ik ;P jk ;P ji 와사각형 P kj ;P ij ;P ik ;P jk 는세점을공유하므로삼각형의외접원이유일하게결정되는성질을이용하여공통외접원을갖는다. m g l g l g m g m b l b m a m b l b l a m a l a j P ik P ij A B A B j A B 이런방법으로하여 P i j 들이모두한원위에있음을모두보일수있다. } 45. ABCD 는원에내접하는볼록사각형이고, 대각선 AC 와 BD 가 X 에서만난다. X 에서네변 AB, BC, CD, DA 에내린수선의발을각각 A 0, B 0, C 0, D 0 이라하자. ja 0 B 0 j + jc 0 D 0 j = ja 0 D 0 j + jb 0 C 0 j 임을증명하여라. 단, jpqj 는선분 PQ 의길이를나타낸다. ( 캐나다 990-3)

213 3. 기하중급문제 3 증명 A 0, B 0, C 0, D 0 을아래그림과같이정하자. \AA 0 X + \XD 0 A =90 ± +90 ± =80 ± 이므로 AA 0 XD 0 는원에내접하는사각형이다. 특히 \D 0 XA 0 =80 ± \DAB = \DCB 로원주각의크기가같으므로 A 0 D 0 :r = BD :R 이성립한다. 단, r과 R은각각 AD 0 XA 0 과 ADCB의외접원의반지름이다.r = AX 이므로 가된다. 그럼이로부터 A 0 D 0 = AX BD ; 마찬가지로, B 0 C 0 CX BD = R R A 0 D 0 + B 0 C 0 = BD(AX + CX) R = BD AC R 이것은대칭적인값이므로, 마찬가지로 A 0 B 0 + C 0 D 0 도같은값이되고, 따라서두합은같다. 별증 X에서 A 0 B 0 C 0 D 0 의네변에수선의발을내려위그림과같이 P, Q, R, S라하자. AA 0 XD 0, DD 0 XC 0, ABCD 등이원에내접하는사각형이므로, 원주각에의해 \XD 0 A 0 = \XAA 0 = \CAB = \CDB = \XDC 0 = \XD 0 C 0 그럼, 두직각삼각형 D 0 XP 와 D 0 XS 는빗변 D 0 X 를공유하고같은크기의각이한쌍더있으므로 RHA 합동. 그러므로, XP = XS. 마찬가지로, XP = XQ = XR = XS 가되고, 그럼 A 0 B 0 C 0 D 0 는 X 를중심으로하는원에외접하고, 따라서, 문제가성립한다. 46. 어떤입자가 x- 축위를지나갈때에는초속 미터까지속도를낼수있고, 다른방향으로움직일때에는초속 미터까지낼수있다. 이입자가원점에서출발하여 초동안도달할수있는영역을좌표로그려라. ( 캐나다 990-4, 한국 990-8) 풀이입자가움직인경로중에서 x-축위에있는마지막점을 T (t; 0) 이라하고, 입자가최종적으로도착하는점을 P 라하자. 그럼원점 O에서 T 까지는쭉 x-축위로이동하는것이가장빠르고, T 에서 P 까지는더이상x-축과만나지않기로한다면직선경로로가는것이가장빠르다. 또한, P 까지가는경로중간의점들은 P 까지가다가되돌아오면도달할수있으므로, 가장멀리도달하는점 P 들에의해생기는이영역의외곽선만구하면된다. 그럼구하는영역은 [ f (x; y) j (x t) + y ( t) g jtj t>0 인경우에 F (; 0) 에서보면 FT : FO = t : 는두원 T 와 O 의반지름의비와같으므로, 원 T 는점 F 를닮음의중심으로하여원 O 를축소한것이된다. t<0 인경우에도대칭적이고, 답

214 4 기하 따라서, 구하는영역은위와같다. } 47. 볼록사각형내부에네원이있는데각각의원은사각형의연속한두변과접하고또다른두원과접한다. 주어진사각형이원에외접하는사각형이라면, 사각형내부의네원중에서적어도둘은합동임을보여라. ( 러시아 990 4차-y0-3) 증명볼록사각형 ABCD 에서 A, B, C, D 쪽에서접하는원의반지름을각각 a, b, c, d 이라하고, 변과만나는접점들을 A 로부터돌아가며 E; F; G; H; I; J; K; L 이라하자. 그럼 EF = p (a + b ) (a b ) = ab. GH, IJ, KL 도마찬가지. 외접사각형이므로 AB + CD = AD + BC 인데, AE = AL 등은서로상쇄되므로 EF + IJ = GH + KL 만남고, 이것은 ab + cd = bc + ad, 즉 (a c)(b c) =0 이다. 48. 삼각형의세변의길이가 a, b, c이고, 그에대응하는각이 A, B, C, 또둘레의길이가 p라할때, 다음의부등식이성립함을보여라. ( 러시아 990 4차-y-) a cos A + b cos B + c cos C p 증명코싸인제 법칙대입하고 p = a + b + c 대입하고양변에 abc 곱하면 (a b + b c + c a ) (a 4 + b 4 + c 4 ) abc(a + b + c) 로동치변형됨. 이좌변은헤론공식의근호안을전개했을때나오는것이므로인수분해하여 (a + b + c)( a + b + c)(a b + c)(a + b c) abc(a + b + c) 가되고, 이제 a + b + c 소거한후 Ravi 치환을이용하면 AM-GM 으로끝. 49. 삼각형 ABC 에서변 BC, AC, AB 의중점을각각 D, E, F 라하고, 삼각형의무게중심을 G 라한다. 사각형 AEGF 를원에내접하게하는, 서로닮음꼴이아닌삼각형 ABC 의개수를각 BAC 의크기에따라구하여라. (APMO 990-) 풀이 AEGF 가원에내접하므로 BA = BF BA = BG BE = 3 BE 이것을 Pappus 의정리 AB + BC =BE + AC 에대입하면 BC = AB + AC 이된다.cosine 제 법칙에서 cos A = AB + AC BC AB AC = AB + AC 4 AB AC AB AC 4 AB AC = ( ) 이므로 \A 60 ± 이다. 따라서, 다음의결과를얻는다. () \A >60 ± 일때 : 조건을만족하는삼각형은없다. () \A =60 ± 일때 : 등호가성립할경우이므로 AB = AC 이고, 그럼정삼각형한가지뿐이다. (3) \A <60 ± AC 일때 : AB = x 라하면 ( ) 에서 x + =4cosA x 의이차방정식을만족하고, 그럼두근은서로역수관계이므로이때에도 AB, AC를바꿔놓으면닮은꼴로는한가지뿐이다. } 50. 두원c, d가평면위에서로의외부에놓여있다. C, D는각각원c, d 위의점으로, 최대한멀리떨어진두점이다. c와 d 안에좀더작은원을하나씩그렸다. 이중첫번째원은c와접하고, C에서 d에그은두접선에접한다. 또두번째원은d와접하고, D에서 c에그은두접선에접한다. 이두원의크기가같음을증명하여라. (Towns 990봄 JO)

215 3. 기하중급문제 5 풀이작은원의중심을각각 a, b 라하고큰원과작은원의접점을각각 E, F 라하자. 또, 원 A, B, a, b 의반지름을각각 r A, r B, r a, r b 라하자. 원 b 와 A 는점 D 에서두접선을따른확대변환이므로 r b r A = DF DC = r B DC ; ) r b = r Ar B DC 이다. 마찬가지로 가된다. r a = r Ar B DC = r b } 5. P 가 G를무게중심으로가지는 4ABC의내부의점일때, 다음식이성립함을보여라. ( 통신강좌 99--4) AP + BP + CP = AG + BG + CG +3PG 풀이 좌표평면상에서생각해보자. A(a; b), B( c; 0), C(c; 0), P (p; q) 라고하면, G(a=3;b=3) 이므로 AP + BP + CP = (a p) +(b q) +(p + c) + q +(p c) + q = a + b +c +3p pa +3q qb µ µ ³ µ a AG + BG + CG +3PG = 3 a + 3 b c b + 3 ³ µ a + 3 c b + +3³ p a µ +3 q b = a + b +c +3p pa +3q qb ) AP + BP + CP = AG + BG + CG +3PG } 5. ABCD 에서 AC? BD 일필요충분조건은 AB + CD = AD + BC 임을보여라. ( 통신강좌 99--5) 풀이 A a d D B b O c C ()) AC? BD일때 \AOB = \R이므로 AB + CD = (a + b )+(c + d ) = (b + c )+(a + d )=AD + BC (() AB + CD = AD + BC 일때 \AOB = µ 라하면, AB = a + b ab cos µ CD = c + d cd cos µ BC = b + c +bc cos µ AD = a + d +ad cos µ AB + CD = AD + BC 에대입해서정리하면 (ab + cd + bc + ad)cosµ =0; ) cos µ =0 ) µ = ¼ ) AC? BD }

216 6 기하 53. ABCD 는볼록사각형이고, DA 와 CB 는 F 에서, AB 와 DC 는 E 에서만나고각 F 와각 E 의이등분선은서로직교한다. 이두각의이등분선은대각선 AC 와 BD 가이루는각의이등분선들과평행임을보여라. ( 통신강좌 99--6) 풀이 E A O Q R P D F B C AC 와 BD 의교점을 O, 각 F 의이등분선과 BD, AC, 각 E 의이등분선의교점을각각 P; Q; R 이라고하자. 이제, \PQO = \QP O 임을보이기만하면된다. \ABC = \ERF (\BER + \BFR) = ¼ (\DER + \DFR) = \DEF + \DFE = \CDE ) ABCD는원에내접한다. ) \CAF = \DBF ) \PQO = \QF A + \CAF = \QF B + \DBF = \QP O } 54. 반전원의중심을지나는원 C 가반전원과만날때그교점을 A, B 라하면원 C 의반전은직선 AB 이고직선 AB 의반전은원 C 임을밝혀라. ( 통신강좌 99--0) 풀이 4OLB»4OBR 이므로 OL : OB = OB : OR ) OL OR = OB A Q P L R O C P' B Q' 따라서 R 은 L 의반점이다.( 또는 L 은 R 의반점.) _ AB 위의임의의점 P 를잡았을때 4OP R»4OLQ ) OQ : OL = OR : OP ) OP OQ = OL OR = OB

217 3. 기하중급문제 7 따라서 P ( 또는 Q) 는 Q( 또는 P ) 의반점이다. 원 O 의내부에 P 가있을때도마찬가지이다. 4OP 0 R»4OLQ 0 ) OP 0 : OR = OL : OQ 0 ) OP 0 OQ 0 = OL OR = OB 따라서 P 0 ( 또는 Q 0 ) 는 Q 0 ( 또는 P 0 ) 의반점이다. 위의결과에서원 C의반전은직선 AB이고직선 AB의반전은원 C이다. } 55. 반전원의중심을지나는원 C 가반전원과점 P 에서외접할때원 C 의반전은점 P 를지나는반전원의접선임을밝혀라. 이의역은어떻게되는가? ( 통신강좌 99--) 풀이하면 원 C 위의 O 아닌임의의점 R 을잡고 R 의반점을 Q 라고하자. 원 C 의지름을 r, \ROP = µ 라 이다. OR OQ = r 이므로, OQ = r cos µ OR = r cos µ Q O R θ C r ) OP OQ = r r cos µ =cosµ ) \OPQ = \R 또, P 의반점은 P 자신이다. 따라서원 C 의반전은직선 PQ 이다. 또한역도당연히성립한다. } 56. 두원 C, C 가 () 만나지않을때와 () 두점에서만날때, 두원 C, C 의근축위의임의의점 P 를지나고두원 C, C 에접하는원을작도하여라. ( 통신강좌 99--) 풀이 ( 서울과학고김민식 ) P 가근축위의점이므로 P 에서원 C, C 에그은 4 개의접선은길이가모두같다. 따라서 P 를중심으로하고각접점을지나는원을생각할수있다. 이원을 C 3 이라고하자. 원 C 3 은접점을지나는원이므로 C, C 와직교한다. 이때, C, C 의 C 3 에대한반전은자기자신이고, P 를지나고 C, C 에접하는원의반전은 C, C 의반전에접하는직선이되므로 (* 반전원의중심을지나는원이므로 ) C, C 의공통접선이된다. 그러므로공통접선을작도하여접점과 P 를잇는직선이 C, C 와만나는점을구한뒤그두점과 P 를지나는원을그리면 C, C 에접한다. 단, C, C 가만나지않을경우에는공통접선이 4 개이므로원이 4 개가생기되, P 가공통접선위에있을때에는 3 개, 공통내접선의교점과일치할때에는 개가생긴다. C, C 가만나는경우에는 개가생기고 P 가공통접선위에있으면 개가생긴다. } 57. 삼각형 ABC 는각 C 를직각으로가지는직각삼각형이다. E 와 F 는 CA 의삼등분점, 즉 CF = FE = EA 이고, G 는 CB 의중점이다. H 는 C 에서 AB 위에내린수선의발이라하고, 삼각형 AEH 의외심을 I 라하자. 삼각형 IGF 와 ABC 는닮은꼴임을보여라. ( 통신강좌 99-3-)

218 8 기하 풀이 I 에서선분 AC 에내린수선의발을 D 라고하면, AD = ED;EF = FC 이므로 DF = AC 가 된다. AB = c; AC = b; BC = a 라고하면,sinA = a c ;AI = R = EH sina = c EH에서 a EH = CE + CH CE CH cos \ECH = 4 9 b + a b c 3 b ab c a c = 4 9 b + a b c 4 a b 3 c = 4 9 b a b 3c ID = AI AD = c 4a EH b 36 = c µ 4 4a 9 b a b 3c b 36 = b c 9a b b 36 = b c 9a b 9 = b4 9a 이므로 ID = b3 b3 이되고, 다시 ID : DF = 3a 3a : b =b :3a = FC : CG이므로 4IDF»4FCG이다. 여기서 \IFG = \R이고 IF : FG = DF : CG = AC : CB이므로 4IFG»4ABC가성립한다. } 58. 육각형 ABCDEF 는반지름 r 인원에내접한다. AB = CD = EF = r 일때 BC, DE, FA 의세중점을연결하여얻은삼각형이정삼각형임을보여라. ( 통신강좌 ) 풀이복소평면상에서이원의중심을원점에두고, A, B, C, D, E, F 를절대값이 r인복소수라두되,! =cos ¼ 3 + i sin ¼ 에대해B =!A, D =!C, F =!E라두자. BC, DE, FA의중점은각각 3 p = (!A + C), Q = (!C + E), R = (!E + A) 이다. R을 Q를중심으로 ¼ 만큼회전시킨복소 3 수 Q +! (R Q) 는!! +=0을이용하여정리하면 P 와같음을보일수있다. 여기서삼각형 PQR은이등변삼각형이며, 더정확히말하면정삼각형이다. } 59. 평면위의서로다른네점 A, B, C, D 가있어 A 와 B 를지나는모든원과, C 와 D 를지나는모든원이서로만나거나일치한다고하자. 그러면, A, B, C, D 는한직선위에있거나, 한원위에있음을증명하여라. ( 통신강좌 ) 풀이두점X, Y 가직선`의한쪽에모두있으면, X, Y 를지나고 `과만나지않는원을그릴수있다 ( ). 만일선분 AB와선분CD가만나지않으면두선분사이에두선분과만나지않는직선을그릴수있고, 이때 ( ) 에의해문제의가정에위배된다. C B A A C B D D

219 3. 기하중급문제 9 두선분이한점에서만나면두선분의수직이등분선도한점에서만난다. 이교점을중심으로하고두선분을현으로하는원을각각그리면, 두원은일치하거나하나가다른것을포함하게되며, 문제에따라두원이일치하면 A, B, C, D 는동일원주상에있다. 또이때, 문제의가정이항상만족됨을보일수있다. 두선분이겹치는경우, A, B, C, D 는일직선상에있으며, 한선분위에다른선분의한끝점만있을경우문제의가정이만족됨을보일수있다. 하나가다른것을완전히포함하면두선분을지름으로하는원을각각그릴때문제의가정에위배된다. } ( ) 의증명 ` k XY 이면 `과 XY 사이에 ` k `0 인직선`0 을그려, X, Y 를지나고`0 에접하는원을그릴수있고, 평행하지않으면 ` k `0 이고 X, Y 중 `에가까운점을지나는 `0 을그려, X, Y 를지나고 `0 에접하는원을그릴수있다. } 60. 예각삼각형 ABC의세수선AD, BE, CF가한점H에서만난다. 사각형 AEHF 와 HECD의넓이가같다면, ABC는이등변삼각형임을보여라. ( 러시아 99 4차-y-) 증명 AE > CE 이면 j4ahej > j4chej 이고 AH > CH. 이닮음비로 j4ahf j > j4chdj 가되어j AEHF j > j CEHDj 가됨. AE < CE 일때도대칭적으로 j AEHF j < j CEHDj. 즉, AE = CE 일수밖에. 6. 주어진삼각형 ABC 에서무게중심을 G, 변 BC 의중점을 M 이라하고, G 를지나고변 BC 에평행인직선이변 AB, AC 와만나는점을각각 X, Y 라한다. 선분 GC, YB 의교점을 P, GB, XC 의교점을 Q 라하면 4MPQ 는 4ABC 와닮은꼴임을밝혀라. (APMO 99-) 증명 AG : AM = XY : BC =:3 이므로, XG : BC = YG: BC =:3 이다. 즉, 4QGX 와 4QBC 의닮음비나 4PGY 와 4PCB 의닮음비가모두 :3 이다. BG : GN =4: 이므로 BQ : QN =3:3, 즉 Q 는 BN 의중점이된다. 마찬가지로 P 도 CR 의중점이다. 평행사변형의두대각선이서로이등분함에서 Q 는 MR 의중점, P 는 MN 의중점이기도하고, 따라서, MP==AB, MQ==CA, PQ==BC 이므로 4MPQ 는 4ABC 와닮음이다. 특히닮음비는 PQ : BC =:4 이다. 6. 중심 O 의원 에대해, O 가아닌임의의점 P 의반전을 P 0 이라하자. 점 P 0 을지나고직선 OP 에수직인직선을원 에대한점 P 의극선이라하고, p(p ) 로나타낸다. 다음명제를증명하여라 : 점 B 가 p(a) 위에있을때, 또그때만점 A 가 p(b) 위에있다. ( 통신강좌 ) 풀이 ( 서일중 년신석우 ) A' B A B' a O g

220 0 기하 (i) p(a)? OA 이므로 p(a) 의 에대한반전은 OA 를지름으로하는원 가된다. 그러므로점 B 가 p(a) 위에있음은 B 의 에대한반전 B 0 이원 위에있음과동치이다. 이때 B 0 은 OB 와원 와의교점이되고, \OB 0 A =90 ±, 즉 OB 0? AB 0 이므로 p(b) 는직선 AB 0 이고, 점 A 를포함한다. 따라서, hb 가 p(a) 위에있다 ) A 가 p(b) 위에있다 i 이다. (ii) 한편, B 가 p(a) 위에없으면 B 0 도원 위에없다. 그런데만약 p(b) 위에 A 가있다면, OB 0? AB 0 이므로 B 0 이원 위에있어야하는데, 이것은명백히모순이된다. 따라서, hb 가 p(a) 위에없다 ) A 가 p(b) 위에없다 i 이다. (i), (ii) 에서 B 가 p(a) 위에있을때에만 A 가 p(b) 위에있다. } 63. 세원,, 가주어져있다. 이세원에모두직교하는원 ± 가존재하는가? 만일존재한다면 ± 가유일한가? ( 통신강좌 ) 풀이원 ± 가존재할필요충분조건은세원,, 가공통의내부를갖지않고, 이세원의중심이일직선위에있지않을경우이다. 이것을증명하자. ()) 세원,, 의중심을각각 A, B, C라하자. A, B, C는일직선위에있지않고또 와 의근축, 와 의근축은각각AB, BC에수직하므로이두근축은한점O에서만난다. 만일 O가 의내부에있다고하면, O는 와 의근축, 또 와 의근축위에있으므로 O는 와 의내부에있어야한다. 즉, 와, 와 가겹치는경우이다. 그러나, 이것은,, 가공통의내부를갖지않는다는가정에어긋나므로 O는 의외부, 같이하여, 의외부에있다. 따라서 O에서세원에접선을그릴수있고그때그접선들의길이는모두같다. O를중심으로, 그접선의길이를반지름으로하는원 ± 를그리면이것이,, 에직교함은쉽게확인할수있다. (() 원 ± 가존재한다고가정하면, ± 의중심에서세원,, 에그은접선의길이는모두 ± 의반지름으로같고, 따라서 ± 의중심O는,, 가이루는세근축의교점이다. 세근축이 ± 의중심에서만났으므로세근축은평행하지않다. 따라서,, 의중심은일직선위에있지않게된다. 만일,, 가공통의내부를갖는다면, 와 의근축과 와 의근축은 의내부에서만난다. 그러나그점은 O일수밖에없고, O에서 에접선을그을수있으므로모순이다. 결국세원은공통의내부를갖지않는다. 위의결과에서 ± 는세근축의교점임을알수있고, 따라서 ± 가존재한다면유일하다. } 64. 삼각형 ABC 에서다음을증명하여라. ( 통신강좌 ) 3(cot A +cotb +cotc) cot A +cotb +cotc 풀이 ( 경문고 년박준홍 ) 일단다음사실을이용하자. cot A = cos A sin A cos a = sin A cos A = +cos A sin A =cot A +csc A: 이를이용하면 X A;B;C cot A X A;B;C csc A, X A;B;C cos A sin A 0 으로이동치식을증명하자. X cos A sin A = X b + c a bc a = X R abc (b + c a bc) R = R abc (a + b + c ab bc ca) R = (a b) +(b c) +(c a) 0: abc 으로증명되었으며, 등호는 a = b = c 의정삼각형일때이다. }

221 3. 기하중급문제 65. 한변의길이가 인정육각형의각변위에한점씩을잡아서이들을꼭지점으로하는새로운육각형의각변의길이를 x ;x ;:::;x 6 이라정의할때 9 x + x + + x 6 9 가성립함을증명하여라. ( 통신강좌 ) 풀이 ( 서울이호웅 ) 그림과같이각길이에문자를부여하면 cosine 제 법칙에따라 x = a + b 6 a b 6 cos ¼ 3 = a + b 6 + a b 6 이고, 같은방법으로 x = a + b + a b ;x 3 = a 3 + b + a 3b ; x 4 = a 4 + b 3 + a 4 b 3 ;x 5 = a 5 + b 4 + a 5 b 4 ; x 6 = a 6 + b 5 + a 6 b 5 6X 이다. 그리고, a + b = a + b = = a 6 + b 6 =이므로 (a k + b k )=6이다. 따라서 k= 6X x k = a + + a 6 + b + + b 6 + a b 6 + a b + + a 6 b 5 k= 이된다. a b 6 a 6 b x a x x 6 b 5 b x 3 x 5 a 5 a 3 b3 x4 a 4 b 4 i) 오른쪽부등식의증명 ; 산술-기하평균부등식에의해 a b 6 a + b 6 ;:::; a 6 b 5 a 6 + b 5 이므로 6X k= x k 3 (a + + a 6 + b + + b 6) < 3 f(a + b ) + +(a 6 + b 6 ) g =9 가성립한다. 등호는성립하지않지만, 새육각형의꼭지점들이정육각형의정삼각형을이루는세꼭지점에가까이다가갈때그값이 9 에가까워져간다. ii) 왼쪽부등식의증명 ; 6X x k =(a + b 6 ) +(a + b ) + +(a 6 + b 5 ) + a + + a 6 + b + + b 6 k=

222 기하 에서,Cauchy-Schwartz 부등식 ( + + )f(a {z } + b 6 ) + +(a 6 + b 5 ) g (a + + a 6 + b + + b 6 ) =6 ; 6 ( + + )(a {z } + + a 6 + b + + b 6) (a + + a 6 + b + + b 6 ) =6 을이용하면 } 6X x k 6+3=9를얻는다. 등호는 a = = a 6 = b = = b 6 = 일때성립한다. k= 증명 ( 서울과학고 학년최서현 ) 정육각형 A A A 3 A 4 A 5 A 6 의각변에점B ;B ;B 3 ;B 4 ;B 5 ;B 6 를잡는다. 이때 A B = a, A B = b, A 3 B 3 = c, A 4 B 4 = d, A 5 B 5 = e, A 6 B 6 = f라한다. (0 5 a; b; ;f 5 ) A b -a B x a A x -f B 6 x 6 f A 6 -e B -b A 3 c x 3 x 4 x 5 -d B 3 B 4 -c d A 4 B 5 e A 5 그러면 B A = a; B A 3 = b; B 3 A 4 = c; B 4 A 5 = d; B 5 A 6 = e; B 6 A = f x = B 6 B, x = B B, x 3 = B B 3, x 4 = B 3 B 4, x 5 = B 4 B 5, x 6 = B 5 B 6 이라하면코사인제법칙 에서 x = a +( f) + a( f); x = b +( a) + b( a); x 3 = c +( b) + c( b); x 4 = d +( c) + d( c); ) x 5 = e +( d) + e( d); x + x + + x 6 x 6 = f +( e) + f( e) = (a a +)+(b b +)+ +(f f +)+(a + b + c + d + e + f) (ab + bc + cd + de + ef + fa) = (a a +)+(b b +)+ +(f f +) (ab + bc + cd + de + ef + fa) = (a a +)+(b b +)+ +(f f +) (a + b + c + d + e + f ) ( 코시 슈바르츠부등식에서 (ab + bc + + fa) 5 (a + b + + f ) ab + bc + + fa = 0; a + b + + f = 0 등호는 a = b = c = d = e = f 일때성립 ) = (a a +)+(b b +)+ +(f f +) = (a ) (b ) (f ) = (a ) +(b ) + +(f ) + 9 = 9 ( 등호는 a = b = c = d = e = f = 일때성립 )

223 3. 기하중급문제 3 이제, b = b 0, d = d 0, f = f 0 라놓자. 그러면, b = b 0, d = d 0, f = f 0 ; 0 5 b 0 ; d 0 ; f 0 5 x + x + + x 6 = (a a +)+(b 0 b 0 +)+ +(f 0 f 0 +)+(a + b 0 + c + d 0 +e + f 0 ) fa( b 0 )+( b 0 )c + c( d 0 )+( d 0 )e +e( f 0 )+( f 0 )ag = (a 3a +)+(b 0 3b 0 +)+ +(f 0 3f 0 +)+3 +ab 0 + b 0 c + cd 0 + d 0 e + ef 0 + f 0 a 5 (a 3a +)+(b 0 3b 0 +)+ +(f 0 3f 0 +)+3 +(a + b 0 + c + d 0 + e + f 0 ) ( 등호는 a = b 0 = c = d 0 = e = f 0 일때성립 ) = (3a 3a +)+(3b 0 3b 0 +)+ +(3f 0 3f 0 +) a; b 0 ;c;d 0 ;e;f 0 5 ) 4 5 3a 3a +5 ; 4 5 3b0 3b 0 +5 ; ; 4 5 3f 0 3f 0 +5 ) (3a 3a +)+(3b 0 3b 0 +)+ +(3f 0 3f 0 +) =9 ( 등호는 a; b 0 ;c;d 0 ;e;f 0 이 0 또는 일때성립 ) ) x + x + + x ( 등호는 a = b 0 = c = d 0 = e = f 0 이 0 또는 일때성립 ), 에서 9 5 x + x + + x 임의의점 P 에대하여정삼각형 ABC 의각꼭지점과의거리, 즉 PA, PB, PC 를세변의길이로갖는삼각형이항상존재함을보여라. 단, P 는 ABC 의외접원위에있는점은아니다. ( 통신강좌 99-5-) 풀이 A, B, C가시계반대방향의순서로있다고하고, P, B, C를 A를중심으로하여시계반대방향으로 60 ± 회전시킨점들을각각 P 0, B 0 = C, C 0 이라하자. 이때삼각형 AP P 0 이정삼각형이므로 AP = PP 0 이며, 회전합동에의해 BP = CP 0 이다. 이제삼각부등식 CP + CP 0 PP 0 에서 AP BP + CP () 이구해진다. 만일 () 의등호가성립한다면 C 가 PP 0 위에있으므로 \AP C = \AP P 0 =60 ± 가된다. 이는정삼각형 ABC 의 \ABC 와같으므로, 네점 A, B, C, P 가한원위에, 곧 P 가삼각형 ABC 의외접원위에있다는결론이되어문제에모순이된다. 따라서, AP < BP + CP 가되고, 다시처음으로돌아가회전을 B, C 를중심으로하여똑같이하면 AP < BP + CP; BP < CP + AP; CP < AP + BP 를모두얻는다. 따라서 AP, BP, CP 는한삼각형의세변의길이가된다. } 67. 삼각형 ABC 의각변에한점을잡아각각 D, E, F 라하고, BD, DC, CE, EA, AF, FB 의중점을각각 U, X, V, Y, W, Z 라하자. 이때삼각형 (4UV W의넓이 )+(4XY Z의넓이 ) (4DEF의넓이) 이전체삼각형 ABC 의넓이에비례함을보여라. 단, D 는 BC 위에, E 는 CA 위에, F 는 AB 위에존재한다. ( 통신강좌 99-5-)

224 4 기하 풀이먼저 BD = pbc, CE = qca, AF = rab라하자. 삼각형 ABC의넓이를 이라해도면적비를구하는데에는일반성을잃지않으며, 이때삼각형 AF C의넓이는 r이된다 ( 높이가같고밑변의비가 : r인삼각형 ). 다시삼각형 AF E의넓이는 ( q)j4af Cj =( q)r가된다. 같은방법으로하여삼각형DEF의넓이는 j4defj = j4abcj j4af Ej j4bdfj j4cedj = ( q)r ( r)p ( p)q 임을알수있다. 마찬가지로하여삼각형 UV W와 XY Z의넓이는 j4uv Wj = r ³ q p ³ r q ³ p ; j4xy Zj = q µ r r µ p 와같이얻는다. 이제구하는식을정리해보면 p µ q j4uv Wj + j4xy Zj j4defj = 3 4 으로상수가되어, D, E, F 의위치에관계없이전체삼각형 ABC의넓이에일정하게비례한다. } 주 D, E, F 가각변의연장선위에있을때도문제가그대로성립하는가? 아니라면어떻게문제를바꾸어야할까? 68. 삼각형 ABC 의각 B 안의방접원을 O, 원 O 와변 BA 의연장선과의접점을 P, 점 P 를지나는원 O 의지름의다른끝점을 Q 라고한다. 변 AB 의중점을 M, 점 M 에관한점 P 의대칭점을 R ( 즉, PM = MR) 이라할때, 세점 Q, C, R 은같은직선위에있음을보여라. ( 한국 99-0) 증명 BC = a; CA = b; AB = c, p = a + b + c 라하면BP = AR = p 이므로 PA = BR = p c; MP = MR = p c ; PR =MP =p c = a + b 방접원 O의반지름을 r b ; 4ABC의내접원의반지름을 r라하면 r b r = p p b ) r b = pr p b 따라서 PQ =r b = pr p b 4ABC의넓이를S 라하면S = pr C에서변 AB에내린수선의발을H라하면 CH = a sin B = ³ ca sin B = c c S = pr c RH = AR AH = p b cos A = p b + b( cos A) = p b + b a (b c) 4(p b)(p c) = p b + bc c = p b (a + b) c

225 3. 기하중급문제 5 따라서 CH RH = pr (a + b)(a b) = PQ PR 따라서 Q; C; R은동일직선상에있다. 69. 아래그림에서 ABCD 는정사각형이고, U 와 V 는꼭지점이아닌점으로각각변 AB 와 CD 위에있는점이다. 사각형 PUQV 의넓이를최대로할수있는 U, V 의위치를모두구하여라. ( 캐나다 99-3) 풀이다. AD, BC 의중점을각각 M, N 이라하자. U, V 가 MN 에대해대칭일때가답임을보일것이 U 0 을 MN 에대한 U 의대칭점이라하고, V 는임의의위치라하자. 그리고 AU 0, BU 0, AV, BV 들이 MN 과만나는점을각각 P 0, Q 0, P 00, Q 00 이라하자. 삼각형의중점연결정리에의해 P 0 Q 0 = P 00 Q 00 = AB 이고, 4P 0 Q 0 U, 4P 0 Q 0 U 0, 4P 00 Q 00 V 의넓이는모두같고 U, V 의위치에상관없이일정하다. 즉, j PUQVj j4p 0 Q 0 Uj + j4p 00 Q 00 V j = j P 0 UQ 0 U 0 j =( 일정 ) 따라서, 증명이되었다. 등호가성립할때는 P 0 ;Q 0 이 P 00 ;Q 00 과각각일치할때, 즉 V = U 0 일때이다. } 70. 각모서리의길이가 a, b, c, d, e, f 인사면체의겉넓이를 S 라고할때, 다음을보여라. S p 3 6 (a + b + + f ) ( 체코슬로바키아 99-) 증명삼각형에서의다음의보조정리를이용하면너무쉬워짐 : p 3 4 (a + b + c ) 3S. 7. 삼각형 ABC에서변 BC 위에점 D를잡자. 그리고점 O, O, O 를각각삼각형 ABC, ABD, ADC의외심이라하자. 네점O, O, O, A가한원위에있음을보여라. ( 러시아 99 4차-y0-6) 증명 AB, AC의중점을각각 M, N. \AO M = \AO B =80 ± \ADB = \ADC = \AO N 이므로내대각으로끝. 7. R 은 6 8 크기의직사각형이다. R 의각변에서한점씩잡아또다른직사각형 R 0 을만들었다. R 0 을조금회전시켜서여전히 R 의내부에포함되도록할수있다. R 0 의둘레의길이의최소값을구하여라. (AIME 993-4)

226 6 기하 풀이 Let R be ABCD, center O. Let R' be PQRS, as shown. Let ASP = θ. Then BRS = CQR = DPQ = θ. So ASP and CQR are similar, but PS = QR, so AP = CR. Hence PR passes through O. But PQR = 90^o, so PR is a diameter of the circle PQRS. Similarly for QS. Hence O is the center of the circle PQRS. [ 그림 ] We have AB + BC = (PS cos θ + RS sin θ) + (RS cos θ + QR sin θ), so perimeter R' = 8/(sin θ + cos θ) = 4 /sin(θ+45^o), which decreases as θ increases. Evidently the rectangle shown can be rotated, but the other rectangle with the same circle (and the same Q, S say) cannot. So the largest θ with a rotating rectangle occurs when P is the midpoint of AD. That implies area R' = ½ area R = 4, so PQ QR = 4, PQ^ + QR^ = 64. Hence (PQ + QR)^ = =. 답 p 448 } 73. 예각삼각형 ABC의변AC 위에점 D가있다. 중선 AM이수선CH, 선분 BD와만나는점을각각 N, K라하자. AK = BK 이면 AN =KM 임을보여라. ( 러시아 993 4차-y0-) 증명 AB 의중점을 L. 4GNC»4GKL 이고닮음비는 : 이므로 GN : GK =:=GA : GM. 가비의리. 증명 C를지나AM에평행한직선이 BD와만나는점을 E라하면좀따져보면 AE k CH 가되어 ANCE는평행사변형. AN = CE =KM. 74. 사각형 ABCD 는네변의길이가같고 \ABC =60 ± 이다. 점 D 를지나고이사각형과다시만나지않는직선 ` 이있다. 직선 ` 과직선 AB, BC 와의교점을각각 E, F 라한다. 선분 CE 와 AF 의교점을 M 이라고할때 CA = CM CE 임을증명하여라. (APMO 993-) 풀이 (O±cial) 네변의길이가같은사각형은평행사변형 ( 마름모 ) 이고 \ABC =60 ± 이므로 AC = AB = BC = CD = AD이다. 그림에서 \EAD = \DCF =60 ±, \EDA = \DFC (* 평행 ) 이므로 4ADE» \CFD. ) AE AD = CD. AD = CD = AC이므로 CF AE AC = AC CF () 또 \EAC = \ACF =0 ± 이므로 () 에의하여 4ACE»4CFA. 따라서 \FAC = \CEA () 4CAM와 \CEA에서 4ACE = 4ACM( 공통 ) 이므로 () 에의하여 4CAM» 4CEA. 따라서 CM CA = CA CE, ) CA = CE CM }

227 3. 기하중급문제 7 참고이문제는직선 l이변ab; BC중의어느하나와만나고다른한변의연장과만날때도성립한다. 직선 l이 BC와 F 에서만나고, AB의연장과 E에서만나면위의증명과정에서 \EAD = \DCF =0 ± ; \EAC = \ACF =60 ± 만바꾸면그대로증명된다. 여기서는직선 CE 와 AF 의교점이 M 이다. 75. ABCD는원에내접하는사각형이고두직선 AB와 CD의교점을M, 또 BC와 AD의교점을 N이라하자. BM = DN 이면 CM = CN 임을증명하여라. (Towns 993봄 SO) 증명 4MBC와 4MDA의닮음비를 :a, 4NCD와 4NAB의닮음비를 :b라하자. 삼각형 MAD와직선NCB에서메넬라우스정리로 MB AN DC = BA ND CM, MB = ND 이고 AB = b CD 이므로 AN = b CM. 그런데, AN = b CN 이므로 CM = CN. 76. 한삼각형의둘레의길이를 p, 외접원의반지름을 R, 내접원의반지름을 r 이라하자. p<(r + r) 임을보여라. ( 폴란드 993/994 차 -) 증명 WLOG \A 60 ±. 그럼 x<ef <r 이고 y + z = a R. 77. 원 O 에내접하는 4ABC 가있다. \A 의이등분선이변 BC 와만나는점을 M 이라하고, M 에서직선 AO 에내린수선이 AC 와만나는점을 P 하자. AB = AP 임을증명하여라. ( 서울시경시 994) 증명그냥각계산. \B >\C 라고할때, \MAO = (B C) 임으로부터 \AMB = A + C = 90 ± + C B = \AMP. \B <\C 일때도비슷하게하면되고, \B = \C 일때는자명. 78. AB 는원 의한지름이고직선 AB 위에있지않은임의의한점을 P 라하자. 직선 PA 가원 와다시만나는점을 U, 직선 PB 가원 와다시만나는점을 V 라하자.( 접선일경우에는 U = A 나 V = B 일수있다. 또한, P 가 위에있으면 P = U = V 이다.) 적당한음아닌실수 s, t 에대해 jpuj = sjpaj, jpvj = tjpbj 라하자. 각 AP B 의코사인을 s 와 t 에대한식으로구하여라. ( 캐나다 994-4) 풀이 P 가원의바깥에있을때는 위와같은경우들이있고, 어느경우나 U, V 는수선의발로서 cos \AP B = PU PB = PV r PU PA = PA PV PB = p st 를만족한다. P 가원주위에있을때는 cos \AP B =cos90 ± =0 이되고,

228 8 기하 P 가원의내부에있을때는 cos(80 ± \AP B) = PU PB = PV PA = r PU PA PV PB = p st 가된다. 정리하면, P 가원의바깥, 원주위, 원의내부에있음에따라 k =; 0; 로둘때,cos\AP B = k p st 이다. } 79. 한변의길이가 a 인정삼각형 ABC 의내부에점 O 가있다. AO, BO, CO 를연장하여대변과만나는점을각각 A, B, C 이라하자. joa j + job j + joc j <a 임을증명하여라. ( 북유럽 994-) 증명 ( 양준호 ) OH ==AB; OH ==AC 인점 H ;H 를 BC 위에잡는다. ) 4OH H 는정삼각형 H L ==OC ;H L ==OB 인점 L ;L 를각각 AB; AC 위에잡는다. ) OH L C ; OH L B 는평행사변형 ) OC = H L ;OB = H L 한편 4L H B 에서 \L H B = \C CB < 60 이므로 BH >L H (sin 법칙생각 ) ) BH >L H = OC 마찬가지로 CH >OB 또한 OA 는 4OH H 의한변의길이보다작으므로 OA + OB + OC <H H + CH + BH = BC = a ) 증명끝 80. 점 O 를중심으로하는어떤원에서두현 AC 와 BD 가원내부의점 M 에서직교한다. 현 AB 와 CD 의중점을각각 K, L 이라하자. OKML 이평행사변형임을증명하여라. ( 인도지역예선 994-6) 증명 직각삼각형의빗변의중점은외심이다. 또, O 에서현의중점을연결한선은그현과수직이다.

229 3. 기하중급문제 9 따라서, \KAM = 라하면, \KMA =, \AKM = ¼ 가되고, \OKM = ¼ ( 혹은 ¼ ) 가된다. 원주각으로 \LDM = \KAM = 이므로, 마찬가지로 \OLM = ¼ ( 혹은 ¼ ) 가되고, 또 \KML = ¼ + ( 혹은 3 ¼ ) 임이직접관찰된다. 그럼 OKML의내각의합이 ¼가되기위해마지막하나의각은 \KOL = ¼ + ( 혹은 3 ¼ ) 가될수밖에없고, 마주보는각의크기가같으므로평행사변형임을알수있다. 증명 KM의연장선이 CD와만나는점을 H라하면KH? CD 임이잘알려진기출문제중하나이다. 즉, \KMA =, \KMB = 라하면 + =90 ± 이고, \MDH = \KAM = \KMA = \HMD = \KMB = 이므로 4MDH 의나머지한각은 \MHD =90 ± 이다. 이로부터 CD 에대한수선으로서 OL k KM 임을알수있다. 마찬가지로, OK k LM 이될것이고, 마주보는대변이서로평행하므로 OKML 은평행사변형이다. 8. 정육면체의중심을지나는평면이이육면체와만나는단면은원에내접하는육각형을이룬다. 이육각형은정육각형임을증명하여라. ( 폴란드 994 차 -3) 증명중심에대해대칭이므로마주보는변끼리길이가같고, 중심으로부터각교점에이르는거리가같으므로이웃한두교점은정육면체의한꼭지점과이등변삼각형을이루고, 싸바싸바하면하나건너의변끼리길이가같아서그럼모든변의길이가같고, 그럼중심각이다같으므로정육각형. 8. 두점A, B에서만나는두원이있다. A에서의두접선이두원과각각 M, N에서다시만난다. 직선 BM과 BN은두원과각각P, Q에서다시만난다. MP = NQ 임을증명하여라. (Towns 994봄 JA) 증명 4AMP»4AQN (AA). \AP N = \ABN = \AMQ = \AMB + \BMQ = \BAN + \BAQ = \QAN = \MAP = \ANP. 뒤에서두번째만앞의닮음에의한것이고나머지는모두원주각, 내대각, 접현각등. 83. 삼각형 ABC에서 AA 은외접원의지름, A 0 은 BC의중점이고, A 는 A 0 에대한A 의대칭점이다. B 와 C 도비슷하게정의한다. A, B, C 는모두같은점임을증명하여라. (Towns 994봄 SO) 증명 A 0 B 0 는 AB 와평행이며길이가절반. 따라서 A B 과도평행이며길이가절반. 즉, A = B 는확대의중심. 증명 AO = OA, A A 0 = A 0 A 이므로 AA 는 A 을중심으로 OA 0 을 배확대한것. 따라서, AA? BC 이고 A 는수심. 84. 한원의원주위의한점을지나서로직교하는두직선 l 과 m 이원주를 3 개의호로분할한다. 각호에서점을하나씩잡아 M i (i =; ; 3) 라하는데, M i 에서의원의접선이 l, m 과만나는두교점까지의거리가같아야한다 ( 즉, M i 가두교점의중점 ). M M M 3 이정삼각형임을증명하여라. (Towns 994 가을 JA3) 증명 C를직각, 나머지두점을 A, B. 접점은표준적순서대로 D, E, F 라하자. 각계산좀해보면 E, F 는각각두호AC의 3등분점들로 EF는원주의 =3 등...

230 30 기하 85. 세변의길이가각각 a, b, c인삼각형 ABC의내접원의중심이I이고반지름은 r이다. s = a + b + c 두면 IA + IB + IC r 7 s 라 임을증명하고등호조건을밝혀라. ( 통신강좌 ) 증명 ( 대구달성고이충엽 ) AI = (s a) + r BI = (s b) + r CI = (s c) + r IA + IB + IC r = 3s 4s +(a + b + c )+r = s +(a + b + c )+r = s +4s s abc s = s +(a + b + c ) s s(a + b + c)+(ab + bc + ca) abc s = 4s 3s abc s = s abc s s 6 7 s = 7 s A B c r I a b C 여기서, rs = p s(s a)(s b)(s c) 이므로, r (s a)(s b)(s c) = 임을이용하였으며, 산술기하 s 평균부등식을사용하였으므로등호조건은 a = b = c 즉정삼각형일때이다. 86. n 차원공간의두벡터 ~x, ~y 에대하여다음을증명하여라. 단, ~x; ~y 6= ~0. ( 통신강좌 ) k~x ~yk (k~xk + k~yk) ~x 4 k~xk ~y k~yk 증명 k!! x k x! y k! x k k! y k k!! x k x! y k! x k k! x k + k!! x k y! y k! x k k! y k k! x! y k + k! y k k! x k k!! y y k k! x! y k+ jk! y k k! x kj k! x! y k

231 3. 기하중급문제 3 마찬가지로 k!! y k x! y k! x k k! y k k! x! y k 따라서위두식을합하면 (k! x k + k!! y k) x! y k! x k k! y k 4k! x! y k 이므로증명되었다. 87. AB와 CD는반지름이 r인원o의수직인두현이다. 그러면이원은두현에의해네부분으로나뉜다. X + Z X, Y, Z, W 를시계방향으로각부분의넓이라하자. 의최대값과최소값을구하여라. Y + W ( 통신강좌 ) 풀이 ( 대전과학고김인태 ) X + Y + Z + W = ¼r ) X + Z Y + W = ¼r Y + W S = Y + W 라두자. X + Y W + Z라해도일반성을잃지않는다. CD가 C 0 D 0 로옮겨갈때, 이므로, S가증가하게된다. ) S가최대일때는 B와 C가일치할때다. \BAD = µ 라하면 4ABD = r sin µ(0 <µ<90 ± ) ) µ =45 ± 일때 4ABD 는 r 으로최대가된다. 따라서 S 의최대값은 ¼r + r 이다. X, Z 와 Y, W 를바꿔생각하면 X + Z 의최대값도 S 의최대값과같다. Y + W = ¼r (X + Z) 이므로 S 의최소값은 ¼r r 이다. X + Z ¼ + 의최대값은 Y + W ¼ ; 최소값은 ¼ ¼ + Α Χ Χ ' Ξ Ψ Β Α Β = Χ Ω Ο Ζ Ο ' } 88. 삼각형 ABC 의외접원과각 A, B, C 의이등분선이만나는점을각각 A 0, B 0, C 0 이라하자. 삼각형 A 0 B 0 C 0 의넓이는삼각형 ABC 의넓이보다크거나같음을증명하여라. ( 통신강좌 ) 증명 ( 서울과학고김종민 ) 원주각의성질을이용하면 \A 0 B 0 C 0 = C + A \B 0 C 0 A 0 = A + B \C 0 A 0 B 0 = B + C

232 3 기하 Χ ' Α Β ' Β Χ 외접원의반지름을 R 이라고하자. Α' 4A 0 B 0 C 0 = R sin C + A sin A + B = R cos A cos B cos C sin B + C R cos A cos B cos C 8sin A sin B sin C (3) () 부분은다음과같다. = R sin A sin B sin C = 4ABC 8sin A sin B sin C 8 ½ 3 µ sin A +sinb +sinc ¾ 3 = ( 단, 등호는 () 에서 A = B = C 즉 4ABC 가정삼각형일때성립 ) 89. 직사각형 ABCD 의각변위에점 O, P, Q, R 이있다. 점 O, P, Q, R 로만들어지는직사각형의둘레의길이는직사각형 ABCD 의대각선의길이의두배보다작을수없음을증명하여라. ( 통신강좌 995--) 증명 4AOR 4CQP, 4BPO 4DRQ 에서, a + d = AB 대각선의길이 L = p (a + d) +(b + c) OP QR 의둘레 ` =( p a + b + p c + d ) ) p (a + d) +(b + c) pa + b + p c + d 을증명하면된다. (ac bd) 0 에서, a c + b d abcd a d + b c + a c + b d a d +abcd + b c (a + b )(c + d ) q (ad + bc) (a + b )(c + d ) (ad + bc) 양변에 a + b + c + d 을더하면, q a + b + c + d + (a + b )(c + d ) a + d +ad + b + c +bc ( p a + b + p c + d ) (a + d) +(b + c)

233 3. 기하중급문제 33 ) p a + d + p c + d p(a + d) +(b + c) ) OPQR 의둘레의길이는 ABCD 의대각선의길이의두배보다작지않다. 90. 어떤수학수업이끝난후, 칠판에 y = x 의그래프와 x-축은남아있었지만 y-축과좌표단위는지워져서남아있지않았다. 자와컴퍼스로 y-축과단위좌표점을작도하여라. ( 벨로루시 995-A) 증명 ( 임준혁 ) 일단 y 좌표가 a 인점 A 를잡자. 그러면 x 좌표가같고, y 좌표가 a 인점 B 를잡을수있다. A; B 를지나고 x 축에평행한직선을긋자. 그게그래프 y = x 와만나는점을 A 0 ;B 0 이라고놓자. 그두점의 x 좌표차이는 이다.(a = b 면 a = b+ 이므로 ) 그러면 A 0 ;B 0 에서 x 축으로수선의발을내리면, 만나는두점사이의길이는 이다. 그러면 x 축위로거리가 인 x 축과평행한직선 l 을그리면, 그것과의교점은 y = 이므로 x =0 이다. 그러면그점 (0,) 을지나고, x 축과수직한직선이 y 축이되고, 그것을이용하면 의길이를알고있으므로격자점도그릴수있다. 9. O 를중심으로하고 AB 를지름으로하는반원이있다. OC? AB 인반원위의점을 C 라하고, 호 BC 위의임의의점을 P 라하자. CP 와 AB 의교점을 Q 라하고, QR? AB 가되는 AP 위의점 R 을잡자. BQ = QR 임을보여라. ( 북유럽 995-) 증명 BQRP cyclic. \RBQ = \RP Q = \AP C =45 ±. 9. X = fa ;A ;A 3 ;A 4 g 를평면위의서로다른네점을모은집합이라하자. 다음성질을만족하는 X 의부분집합 Y 가존재함을보여라 : K \ X = Y 를만족하는원판 K 가존재하지않는다. ( 오폴 995-) 풀이어느세점이상이한직선상에있으면 ( 편의상 A ; A ; A 3 순서로있다고하자.) Y = fa ; A 3 g 로하면 K 가 Y 를포함하면 fa g 도반드시포함하므로조건을만족하는 K 는존재하지않고따라서 Y 가존재한다. 또네점이오목사각형을이루면 ( 편의상 < 그림 > 와같다면 ) Y = fa ; A ; A 3 g 로하면 K 가 Y 를포함하면 fa 4 g 도반드시포함하므로조건을만족하는 K 는존재하지않고따라서 Y 가존재한다. 네점이볼록사각형을이룰때 ( 편의상 < 그림 3> 과같다면 ) A A 3 와 A A 4 의교점을 P 라하자. Y = fa ; A ; A 3 g 로잡을때이 Y 가문제의조건을만족하는 Y 가되지않으려면 Y 를포함하면서 fa 4 g 를포함하지않는 K 가존재해야한다. 그러면원의성질로부터 PA PA 3 < PA PA 4 이다. 마찬가지로 Y = fa ; A ; A 4 g 로잦ㅂ고생각하면 PA PA 4 < PA PA 3 이되어모순이다. 따라서 fa ; A ;A 3 g 와 fa ;A ; A 4 g 중한집합은문제의조건을만족하는집합 Y 가된다.( 증명끝 ) } 93. 삼각형 ABC의외심을O라하자. 4AOB의외접원q가두직선CA, CB와다시만나는점을각각 D, E라하자. CO? DE 임을증명하여라. (Towns 995봄 SO3) 증명 \CED = \BAD = \BOC =90± \OCB. 94. 삼각형 ABC의세내각의이등분선을각각 AK, BL, CM이라하자 (K, L, M은대변위의점 ). 직선 BL, CM 위에각각 AP = PK, AQ = QK 인점P, Q를잡으면, \PAQ =90 ± \BAC 가됨을증명하여라. (Towns 995가을 JA3)

234 34 기하 증명원 ABK와직선BL의교점은 AK의수직이등분선과 BL의교점이므로이점이 P. 즉 ABKP, ACKQ가 cyclic. 그럼 \PAQ = \PAK + \QAK = \PBK + \QCK = (\B + \C). 95. 삼각형 ABC 의 A 에서내린수선을지름으로하는원이 AB, AC 와각각 D, E 에서만난다. A 를지나고 DE 에수직한직선이 4ABC 의외심도지남을증명하여라. ( 북유럽 996-3) 증명 A에서 BC, DE에내린수선의발을각각 H, K라하면, \KAE =90 ± \DEA =90 ± \DHA = \DAH =90 ± \B =90 ± \AOC = \OAE. 96. A ;A ;:::;A 8 은평행육면체의꼭지점들이고 O 는그것의중심이다. 다음을증명하여라. ( 통신강좌 ) 4(OA + OA + + OA 8) (OA + OA + + OA 8 ) 증명 그림에서 A A 4 A A 7 O A 3 A 6 A 8 A 5 OA = OA 5 = a; OA = OA 6 = b OA 3 = OA 7 = c; OA 4 = OA 8 = d ( 단, a b c d 라하자.) ) 4(a + b + c + d + a + b + c + d ) (a + b + c + d + a + b + c + d), (a + b + c + d ) (a + b + c + d) 사각뿔 OA A A 3 A 4 만생각해보면, A A = A 3 A 4 이다. O a d A A 4 b c A A 3 또한 a + b>a A ; A 3 A 4 + c>d ) a + b>a A = A 3 A 4 >d c ) a + b + c>d ( ) (a + b + c + d ) (a + b + c + d), a + b + c + d ab ac ad bc bd cd 0 이므로이를증명하자. a + b + c + d ab ac ad bc bd cd = a + b + c + d (a + b + c)d ab bc ca a + b + c + d d ad bc ca (by ) = a + b + c d ab bc ca a + b + c c a b ca (by a b c d) = a b ca 0

235 3. 기하중급문제 35 ) (a + b + c + d ) (a + b + c + d) 97. 어떤한변의길이가 l 이고넓이가 S 인삼각형에서세높이의곱의최대값을구하여라. ( 이탈리아 996-) 풀이 l에대한높이를 h라하고, 나머지두높이를 k, k, 나머지두변의길이를 a, b라하자. hk k = 8S3 = S abl R 이므로 R이최소일때를구하는것과같다. h l 이면 l을지름으로할때가 R = l 로최소. h> l 이면 a = b 인이등변삼각형일때가 R이최소. } 98. A 는원 C 의밖의한점이다. C 위의점 P 에대해정사각형 AP QR 을만들자 ( 단, A; P; Q; R 은반시계방향순서이다 ). P 가 C 위를움직일때 Q 의자취를찾아라. ( 이탈리아 996-5) 풀이원 C를 x + y = r 으로놓고, A(p; q), P (x; y) 라하면Q(X; Y )=(x +(q y);y (p x)). 즉 (X q) +(Y + p) =r. 이결과를역으로관찰하여논증기하로설명할수도있을듯. } 99. O, O 를중심으로하고서로만나지않는두원의한공통외접선의접점을각각 A, A 라하자. 선분 O O 는두원과각각B, B 에서만난다. 두직선A B 과 A B 의교점을 C라하고, C에서 B B 에그은수선이 A A 와만나는점을 D라하자. A D = DA 임을증명하여라. (Towns 996봄 JA) 증명 D가직각삼각형 A CA 의외심. \CA D =90 ± \O A B =90 ± \CB B = \A CD 등이용. 00. ABCDEF 임의의볼록육각형이다. 각변AB, BC, CD, DE, EF, FA의중점을각각 A 0, B 0, C 0, D 0, E 0, F 0 이라하자. 이육각형의넓이를삼각형 ABC 0, BCD 0, CDE 0, DEF 0, EFA 0, FAB 0 의넓이를이용한식으로나타내어라. (Towns 996가을 SA3) 풀이 jefa 0 j = jefaj + jefbj 등의식을모두다더하면육각형이세번완성됨. } 0. 삼각형 ABC 에서각 B 와각 C 의이등분선이 AC, AB 와만나는점을각각 D, E 라하자. \BDE =4 ±, \CED =8 ± 라할때, 삼각형 ABC 의세각의크기를구하여라. ( 통신강좌 ) 풀이선 BD에대한E의대칭점을 M이라하고선 CE에대한D의대칭점을 N이라하자. BD와 EC의교점을 I라하고, BD와 NE의교점을 S라하자. 4IDE와 4IBC에서 \DIE = \BIC이므로 \IBC + \ICB = \IDE + \IED =4 ±. IC, IB가 \B, \C의이등분선이므로 \B + \C =(\IDE + \IED)=84 ±. 따라서 이제 M 이 4DNS 의방심임을보이자. \A =96 ± \BSE = \SED + \SDE =60 ± (\BSE의 4SDE의외각이므로 ) \BSM = \BSE =60 ± (E와 M이 BD에대해대칭이므로 ) \NSB = \DSE =80 ± (\SDE + \SED) =0 ± \NSM = \NSB \BSM =0 ± 60 ± =60 ± 따라서 \NSM = \BSM 이므로 SM 은각 \NSB 의이등분선이다. \NDE = 90 ± \DEC =90 ± 8 ± =7 ± (DN?CE이므로) \NDM = \NDE \MDE =7 ± 48 ± =4 ±

236 Μ Π Λ 36 기하 따라서 \NDM =4 ± = \MDS이므로 DM은 \NDS의이등분선이다. SM과 DM이각각 \NSB, \NDS의이등분선이므로 M은 4DNS의방심이다. \DNE = \NDE =7 ± (D와 N이 CE에대해대칭이므로 ) \BNE = (80 ± \DNS) =54± (M이방심이므로 \BME는외각의 ) \DNC = 80 ± (\DNE + \BNE) = 80 ± (7 ± +54 ± )=54 ± \NCE = 90 ± \DNC =90 ± 54 ± =36 ± \BCA = \NCE =7 ± \CBA = 80 ± (\BAC + \BCA) =80 ± (96 ± +7 ± )= ± 따라서 \A =96 ±, \B = ±, \C =7 ± 이다. } 풀이 ( 과기원수학과 96 학번신희성 ) A D 4 4 I S 8 8 E C N M B 선 BD에대한E의대응점을 M이라하고, 선 CE에대한D의대응점을 N이라하자. BD와 EC의교 점을 I라하자. BD와 NE의교점을 S라하자. \A =80 ± (\ABC + \ACB) =80 ± (\DBC + \BCE) =80 ± (\CED + \BDE) =80 ± (8 ± +4 ± )=96 ± \BSE = \DEN + \EDB =\DEC + \BDE = 8 ± +4 ± =60 ± \MSB = \BSE =60 ± \NSM =80 ± \MSE =80 ± 0 ± =60 ± ) \MSB = \NSM DN?CE 이므로 \NDM =80 ± (90 ± +8 ± +4 ± +4 ± )=4 ± ) \NDM = \MDS 과 에의해 M 은 4DNS 의방심이다. \DNE = \NDE =3 4 ± =7 ± \BNE =(80 ± 7 ± =54± (* M이방심이므로 ) \BCA =\BCE =(\DNB 90 ± ) =(7 ± +54 ± 90 ± )=36 ± =7 ± \CBA = 80 ± \CAB \BCA =80 ± 96 ± 7 ± = ± 0. 볼록사각형 ABCD 의내부에 4PAB = 4PBC = 4PCD = 4PDA 인점 P 가존재한다. 사각형 ABCD 의넓이는그대각선에의해이등분됨을증명하여라. ( 통신강좌 ) 증명 ( 과기원 97 학번이수인 ) } Κ Χ Α Β

237 3. 기하중급문제 37 M, N 을각각대각선 AC, BD 의중점이라하자. 만약 M 과 N 이일치하면사각형 ABCD 는평행사변형이되고따라서두대각선각각에의해면적이이등분된다. 따라서 M 과 N 이다르다고가정하자. P 는이두점중적어도하나와는달라야하므로 P 와 M 이다르다고하자. 이제 M, P 가대각선 BD 위의점들임을보이자. A 와 C 에서 BP 의연장선에내린수선의발을각각 K; L 이라하면 4PAB = 4PBC 이므로이두삼각형의 BP 를밑변으로했을때의높이인 AK 와 CL 은같아야한다. 따라서사각형 AKCL 은평행사변형이며두대각선의교점은 M 이된다. 따라서 M 이직선 KL 위에있다. 직선 KL 위에 B, M, P 가있으므로직선 MP 위에 B 가있다. 같은방법으로직선 MP 위에 D 가있어야하며따라서 M, P 는대각선 BD 위의점들이다. 따라서 K, L 은 A, C 에서대각선 BD 위로내린수선의발이다. AK = CL 이므로두삼각형 4ABD 와 4CBD 는밑변의길이와높이가같으므로면적이같다. 즉사각형 ABCD 는대각선 BD 에의해면적이이등분된다. 03. 예각삼각형 XY Z 의외접원을 C 라고하자. C 내부의한점 P 에대해, 직선 PX, PY, PZ 가 C 와다시만나는점을각각 L, M, N 이라하자. LMN 이정삼각형이되게하는 P 의자취를구하여라. ( 몰도바 997 최종 -y0-3) 풀이 \MY L = \MNL =60 ± 등이고, \YLZ =80 ± \YXZ 이므로, PYLZ에서각을살펴보면 \YPZ =360 ± \YLZ \PYL \PZL = \YXZ+60 ±. 따라서, P 는 YZ를현으로하여 \YXZ+60 ± 의원주각을갖는원호위에있다. 현 XZ에서도그런원호를하나그리면앞의원호와만나한점P를만들게되고, 각계산좀더하면그P가모든조건을만족함을금방확인할수있다. } 04. P 는볼록사각형 ABCD 내부의점으로, 네삼각형 ABP, BCP, CDP, DAP 의넓이가모두같다. ABCD 의두대각선중하나가다른것을이등분함을증명하여라. ( 북유럽 997-) 증명 BD 의중심을 M. j4bcpj = j4dcp j 이므로 CPM 은일직선. 마찬가지로 AP M 도일직선. AP MC 가일직선이면 AC 가 BD 를이등분하니문제성립. 일직선이아니면 P = AM \ CM = M 일수밖에. 그럼 j4abdj = j4cbdj 이므로 BD 가 AC 를이등분. 05. 삼각형 ABC 의내부의점 P 가다음을만족한다. 이때, \PBA = \PCA = (\ABC + \ACB) 3 AC AB + PC = AB 임을증명하여라. ( 폴란드 997 차-) AC + PB 증명평행사변형 PBP 0 C를생각. AB \ CP 0 = Q, AC \ BP 0 = R 이라하자. 그럼 \AQC = \ARB = \BP 0 Q = B+C. QB = QP 6 0 = PC, CR = CP 0 = PB. 4ACQ» 4ABR 이므로 AC AQ = AB AR. 06. 한변AB의길이가나머지두변의길이의합의 3 인임의의삼각형에서, \C가항상가장작은내각임을증명하여라. (Towns 997봄 JA) 증명 3c = a + b. 귀류법. a c 이면 b =3c a a + c 모순. 07. 점 O 를중심으로가지는원에내접하는삼각형의수심을 H 라하자. AO = AH 라할때, \CAB 의크기를구하여라. ( 폴란드 997/998 차 -) 풀이 BC 의중점을 M 이라하면 OM = AH = R 이므로 \BOC = 0±. } 08. 삼각형 ABC 의중선 AD, BE, CF 가점 G 에서만난다. AF GE, BDGF 가원에내접하는사각형들이면, ABC 는정삼각형임을증명하여라. ( 폴란드 997/998 차 -0) 증명내대각을고려하면 CEGD도 cyclic. 방멱에의해 CE CA = CG CF = CD CB 이므로 ABDE도 cyclic이고 4CDE»4CAB»4CED.

238 38 기하 09. \A = 90 ± 인직각삼각형 ABC가있다. D를 A에서 BC 위에내린수선의발이라하자. \ABD와 \ADB의이등분선이 I 에서만나고 \ACD와 \ADC의이등분선이 I 에서만난다. I, I 에서각각 AD에이르는거리의합이 BC의 4 일때 \B와 \C를구하여라. ( 통신강좌 ) 풀이 AB = AP + BP AP = AM; BP = BN ) AB = AM + BN I N = I M = r; I E = r 라두자. 4ABC 4DBA ) AD : AC = AB : BC AC AB AD = = bc BC a BD : AB = AB : BC ) BD = c a AB = AM + BN = AD r + BD r = bc a + c a r = c 같은식으로 4ADC 에서생각해주면 bc a = b a r = b + : bc a + b + c (r + r )=b + c a r + r = 4 a; b + c = a 3 따라서 bc a + a 4 a = b + c bc a + a = b + c 4bc + a =ab +ac a a(b + c)+4bc =0 (a b)(a c) =0 ) a =b 또는 a =c a =b일때a : b =: 따라서 \B =30 ± ; \C =60 ± a =c일때a : c =: 따라서 \B =60 ± ; \C =30 ± ) (\B; \C) =(30 ± ; 60 ± ); (60 ± ; 30 ± ) } 0. 둘레길이가 p 인모든삼각형 ABC 에대해생각하자. 이삼각형의내접원의접선중에서 BC 와평행한선이두변과 M, N 에서만난다. (a) MN의길이는AC + AB =3BC 일때최대임을보여라. (b) 이경우에, 비 AI=IA 가이삼각형의모양에상관없이일정함을보여라. 단, I는내심이고 A 은 AI와 BC의교점이다. ( 몰도바 998 최종-y9-3) 증명 4AMN»4ABC 에서 A로부터방접원까지이르는접선의닮음비를생각하면 x : MN = p : BC. MN = x(p x) ( x+(p x) ) p p p 4. 등호는 x = p x = p 일때. 각의이등분선정리에의해 c AI : IA = AB : BA = c : a b+c =3:.. 서로만나는두원k 과 k 가있고, 중심이 O인원k 3 이 k, k 와각각점A와B에서안쪽으로접한다. 한직선이 k 과 C에서, k 와는 D에서접한다. O에서 CD에내린수선의발을 E라하자. 세직선AC, BD, OE가한점에서만남을보여라. ( 몰도바 998 최종-y9-8)

239 3. 기하중급문제 39 증명 AC \ OE = P 라하자. \OPA =90 ± \ACD =90 ± \AO C = \OAP. 따라서 P k 3. 마찬가지로, BD \ OE k 3.. 한원k가각각의중심이 O, O 인원k, k 와각각점A, B에서외접한다. 한직선이 k, k 와각각점 C, D에서접하고, 선분 CD와 AB는직선O O 에대해서로반대편에있다. 네점A, B, C, D가한원위에있음을증명하여라. ( 몰도바 998 최종-y0-8) 증명 \AO C =a, \BO D =b, \AOB =c라하자 (O는원k의중심). 볼록오각형 OO CDO 의내각의합에서 a + b + c =80 ±. 접현각에의해 \BAC = a + c 이고 \BDC = b. 따라서, 대각의합이 80 ± 이므로 cyclic. 3. 삼각형 의세변의길이는 a, b, c 이고, 삼각형 a 는 a 0, b, c, 삼각형 b 는 a, b 0, c, 삼각형 c 는 a, b, c 0 을각각세변의길이로갖는다. 그리고이들삼각형은모두넓이가 이고, a 6= a 0, b 6= b 0, c 6= c 0 이다. () 세변의길이가a 0, b 0, c 0 인삼각형 0 이존재함을보여라. () 삼각형 0 의넓이를구하여라. ( 호주 Senior Contest 998-) 풀이인하자. 두변의길이가주어지고넓이가주어졌을때그런삼각형은기껏해야두종류밖에없음을확 한길이를밑변 XY 로할때넓이가일정하면높이가일정하므로, 그높이에 XY 의평행선 ` 을그리고 X 를중심으로다른주어진길이의변 XZ 를반지름으로갖는원을그리면교점은많아야둘이기때문이다. 그리고이미하나의삼각형이가능할때그것이직각삼각형이아니면교점은정확히둘이생긴다. 이제 BC, CA, AB 가각각 a, b, c 라하자. 그럼 ABC 는직각삼각형이아니다. ABCD 가평행사변형이되도록점 D 를잡고, AC 의연장선에 AC = AE 인점 E 를잡자. 4ABE, 4ABD, 4ADE 의변의길이를알아보면각각 (EB;b;c), (a; BD; c), (a; b; DE) 이고이들은모두 ABC 와합동이아니므로, 그럼앞에서확인한바에따라이들이 a, b, c 일수밖에없다. 즉 EB = a 0, BD = b 0, DE = c 0 이다. 따라서, 세변의길이가각각 a 0, b 0, c 0 인삼각형 0 = 4DEB 가존재하고, 그넓이는 0 = a + b + c =3 이다. } 4. 볼록사각형 ABCD 의두대각선 AC 와 BD 의교점을 P 라하자. AB 의중점을 M 이라하고, MP 의연장선이 CD 와만나는점을 Q 라두자. 삼각형 BCP 와 ADP 의넓이비가 CQ 와 DQ 의길이비와같음을보여라. ( 폴란드 998/999 차 -6) 증명 BD 로부터의높이를반으로줄이는것을생각하면 j4adp j : j4bcpj = j4mdpj : j4mcpj = DQ : CQ.

240 40 기하 5. P 는삼각형 ABC 내부의점으로, \PAB = \PBC = \PCA 를만족하는점이다. AB =3,BC = 4, CA =5 일때,tanPAB 의값을구하여라. (AIME 999-4) 풀이 P is a Brocard point. We have cot PAB = cot A + cot B + cot C (*). Using cosine formula we find cos A = 33/65, so sin A = 56/65 and cot A = 33/65. Similarly, cos C = 3/5. We recognize 3,4,5 triangle, so cot C = 3/4. Finally cos B = 5/3. We recognize 5,,3 triangle, so cot B = 5/. Hence cot PAB = 95/68. [ 그림 ] To prove (*) take D so that BDA is similar to ABC. Then P lies on CD. So cot PCA = CK/DK = CA/DK + AK/DK. If the altitude from B is BE, then CA/DK = CE/BE + EA/BE = cot C + cot A, whilst AK/DK = cot B. 답 68=95 } 6. P 는삼각형 ABC 내부의점으로 \PAB = \PCA, \PAC = \PBA 를만족한다. O 를이삼각형의외심이라하면, O 6= P 일때 \AP O 가직각임을보여라. ( 폴란드 999 차 -4) 증명 \PAB = \PCA =, \PAC = \PBA = 라할때 + <80 ± 이므로 \A <90 ±. 따라서, O 는직선 BC 에대해 P 와같은쪽에있고, \BPC = + = \BOC 이므로 BCOP cyclic. WLOG O 가 4AP C 내부에있다고하고 \AP O = 360 ± \AP B \BPC \CPO = 360 ± \AP B \BPC \CBO =360 ± (80 ± ) ( + ) (80± ) =90 ±. 증명 AP, AO의연장선이외접원과만나는점을각각 Q, R이라하자. \AQR =90 ± 이고 O가 AR의중점이므로, 문제는 P 가 AQ의중점임을보이는것과동치. 4BPA»4AP C 이고 4PBQ» 4ABC»4PQC 이므로 AP = BP PC = PQ. 끝. 7. 삼각형 4 의내접원의반지름이 r 이다. 4 의수선의길이의합이적어도 9r 이됨을증명하여라. ( 호주 999-4) 증명 삼각형 4 의넓이를 S 라하고, 세변 a, b, c 에대응되는수선의길이를 x, y, z 라하자. 그럼 S =(a + b + c)r = ax = by = cz 산술 - 조화평균부등식 ( 또는코시부등식으로도가능 ) 에의해 a + b + c 9 a + b + c 가성립하므로, x S + y S + z S 9r S 즉, x + y + z 9r 의원하는부등식을얻을수있다. 8. \ACB =\ABC 를만족하는예각삼각형 ABC 의변 BC 위에 \BAD = \ABC 인점 D 가있다. 다음을증명하여라. ( 폴란드 999/000 차 -) BD = AB + AC 증명 BC 위에 \EAB = \B 인점 E 를잡으면 AC = AE = EB 가됨 ( 각크기확인 ). AB : AC = AB : AE = BD : DE = BD : AE BD 의비례식을전개하여정리하면끝. 9. \C =90 ± 인삼각형 ABC 의내부또는둘레위에점 X 가존재한다. 점 X 에서 BC, CA, AB 로내린수선의발을각각 P, Q, R 이라하자. AR RB = BP PC + AQ QC 가성립할필요충분조건은 X 가 AB 위에존재하는것임을증명하여라. ( 폴란드 999/000 차 -6)

241 3. 기하중급문제 4 증명피타고라스정리 AR + RX = AX = AQ + PC, BR + RX = BX = BP + QC 을 (AR + RB) =(BP + PC) +(AQ + QC) 에대입하면, AR RB + AX = BP PC + AQ QC 로끝. 0. AB k CD 인사다리꼴 ABCD에서두대각선 AC와 BD는 O에서만난다. 변 AB와 CD를각각한변으로하는정육각형을사다리꼴의바깥으로그려그중심을각각 M, N이라고하자. M, O, N이한직선위에있음을증명하여라. ( 몰도바 000 최종-y9-7) 증명 OB : OD = AB : CD = MB : ND 이므로 4OBM»4ODN (SAS). 그럼 \MOB = \NOD 로일직선.. BC = AC 인이등변삼각형 ABC의내심과외심을각각 I, O라하자. I를지나고AC에평행한직선이 BC와 D에서만난다. 두직선DO와 BI가수직임을증명하여라. ( 몰도바 000 최종-y0-7) 증명 OIC는일직선. 4ICD»4ACO (AA) 이므로 AC : CO = IC : CD 이고또 4AIC» 4ODC (SAS). 그럼 \ODB + B = C+A + B =90±.. 원에내접하는사각형 ABCD의네변의길이를a, b, c, d라하고넓이를q, 외접원의반지름을 R이라하자. R (ab + cd)(ac + bd)(ad + bc) = 6Q 임을증명하여라. 또, 이로부터 R (abcd)3=4 Q p 임을보이고, 등호조건이 ABCD가정사각형일때임도 보여라. ( 아일랜드 000-7) 증명 Q = (ab + cd)sin = x y (ad + bc)sin 이고 sin sin = R R = ac+bd 4R 이므로, Q = (ab+cd)(ac+bd)(ad+bc) 6R. 여기에바로 AM-GM 적용하면끝. 이등식을제공하지않고바로부등식을증명하라고문제가나온다면으악 AB 6= AC 인삼각형 ABC 에서내심을 I 라하고, 직선 BI 와 CI 가대변과만나는점을각각 D, E 라하자. DI = EI 가되도록하는각 A 의크기를모두구하여라. ( 폴란드 000 차 -4) 풀이 \IEB = \IDC 또는 \IEB + \IDC =80 ±. 전자는 AB = AC 인경우라서곤란하고, 후자의경우 \A =60 ± 가됨. } 4. 삼각형 ABC의변BC, AC 위에각각점 D, E가존재하고, AD와 BE는점P에서만난다. CLPK가평행사변형이되도록 BC와 AC 위에각각점 K와 L을잡자. AE EL = BD 임을증명하여라. DK ( 폴란드 000/00 차-) 증명 AL CL = AP DP = KC DK, EL CL = EP BP = KC BK 임에서 AL EL = BK DK. 5. 두원이두점 A, B 에서만난다. AC 를첫번째원의지름이라하고, AD 를두번째원의지름이라하자. 세점 B, C, D 가한직선위에있음을증명하여라. ( 오클랜드 00-3) 증명지름에대한원주각은직각이므로 \ABC = \R, \ABD = \R 이다. 따라서, C와 D는모두 B를지나는AB의수선위에있다.

242 4 기하 주 \ABC + \ABD = \R + \R =80 ± 이므로한직선위에있다고쓰기쉽다. 그러나, 이것은 C와 D가직선AB에대해서로반대쪽에있을때만고려한것이고, 사실은둘다 AB에대해같은쪽에있는경우도있으므로조금감점될수있으니주의가필요하다. 앞의증명과같이쓰면간단하지만, 두경우를각각나누어서증명해도되겠다. 6. ABC 는 AC > AB 인삼각형이다. BC 의수직이등분선과 \A 의이등분선의교점을 P 라하자. 그리고, P 에서 AB, AC 에내린수선의발을각각 X, Y 라하자. XY 와 BC 의교점을 Z 라할때, BZ=ZC 의값을구하여라. ( 캐나다 00-3) 풀이 \PMB = \PXB = \PMC = \PYC =90 ± 이므로 PMBX와 PMYC는원에내접하는사각형들이다. 또, PBC 는 PB = PC 인이등변삼각형이므로, 원주각에의해 \PXM = \PBM = \PCM = \PYM 그런데 PX = PY 이므로, 만일 M 이 XY 위의점이아니라면 \MXY = \MY X 로 4MXY 도이등변삼각형이되어 M 이 XY 의수직이등분선 AP 위에있어야한다. 이때, 각의이등분선의성질에따라 BA=AC = BM=MC = 이고, 이것은문제의조건에어긋난다. 따라서, XMY 는일직선이고 Z = M, 즉 BZ=ZC = 이다. } 별해 4ABC의외접원을생각하자. 호 BC의중점은선분 BC의수직이등분선과 \A의이등분선이모두지나는점이므로이점이 P 가된다. 심슨정리에의해외접원위의점 P 에서세변 ( 의연장선 ) 에내린수선의발 X, Y, M은일직선 ( 심슨직선 ) 을이룬다. 따라서, Z = M 이고, BZ=ZC =이다. } 7. 삼각형 ABC 에서내심을 I 라하고직선 AI 와 BC 의교점을 D 라하자. \B =60 ± + 3 \C 일때, 또그때만, AI + CD = AC 임을증명하여라. ( 폴란드 00 차 -5)

243 3. 기하중급문제 43 증명 D를 IC에대해대칭시킨점을E. AI + CD = AC, AI = AE, \AEI = \AIE, \ADB =80 ± \CID (= \B) 인데, \ADB =90 ± \B + \C 이므로풀면됨. 8. \B >\C 인삼각형 ABC가있다. 변 BC 위의점 D가등식 \DAC = (\B \C) 를만족한다. 직선 AC와점A에서접하고점 D를지나는원이직선 AB와점P에서다시만날때, 다음을증명하여라. ( 폴란드 00/00 차-) BP AC = BD DC 증명 \BPD = \DAC ( 접현각 ) 이고 \ADB = (\B + \C) =\PDB 임. 4BPD 와 4ACD 에서 sine 정리로비교하면금방확인됨. 별증! CB 와원이 D 말고다시만나는점을 Q 라하면 4CAD»4CQA, 4BDP»4ADQ 임에서금방확인됨. 9. AB k CD 이고 AD < CD 인사다리꼴 ABCD 가원 c 에내접해있다. DP 는 AC 에평행한현이다. D 에서접하는 c 의접선이직선 AB 와 E 에서만나고, 두직선 PB 와 DC 가 Q 에서만난다. EQ = AC 임을증명하여라. ( 북유럽 00-) 증명 4QBC 4ADE (ASA). 따라서, AE 와 QC 는평행하고길이도같음. 30. \BAC = 90 ± 인삼각형 ABC 를밑면으로하는삼각뿔 ABCD 에서, AD = BD, AB = CD 이면 \ACD 30 ± 임을증명하여라. ( 폴란드 00 차 -5) 증명 D에서면 ABC에내린수선의발을 H라하자. CD =(H 에서 AC에이르는거리 ) (D에서 AC에이르는거리 )= CD sin \ACD 이므로 sin \ACD, 즉 \ACD 30±. 3. M, N 은사면체 ABCD 의모서리 AB, CD 의중점이다. 점 P 는선분 MN 위에서 MP = CN, NP = AM 을만족하는점이다. O 를사면체 ABCD 의외심이라할때, O 6= P 이면 OP? MN 임을보여라. ( 폴란드 00/003 차 -8) 증명 ( 경기도여주군홍지현 ) AM = a, CN = b, 그리고사면체의외접원의반지름을 r 로두자. 4OAB은이등변삼각형이고 M은중점이므로 OM? AB, 마찬가지로 ON? CD 이다. 그럼 OM = r a ; ON = r b 4OMN을생각하자. OP = p, \OPM = 라하면코싸인제법칙에의해 r a = p + b pb cos r b = p + a +pa cos 두식을빼어정리하면 p(a + b)cos =0, 즉 cos =0을얻는다. 즉, 는직각.

244 44 기하 3. 4ABC 의외접원위의점 P 에서 BC, CA 에내린수선의발을각각 D, E 라하자. AD, BE 의중점을각각 L, M 이라하면, DE, LM 은서로수직임을보여라. ( 셈본중등고급도전문제 4..) 증명 DE 의중점을 M 0 이라하자. A L E B D M M P C 4LMM 0 4EDP 임을보이자. MM 0 ==BC, LM 0 ==AC 이므로 \LM 0 M = \ACD = \EPD LM 0 = AE, MM0 = BD이므로 LM 0 : MM 0 = AE : BD 4AP E, 4BPD 에서 \PBD = \PBC = \PAC = \PAE 4BDP = \AEP =90 ± 이므로 4AP E 4BPD ) 4LM 0 : MM 0 = AE : BD = PE : PD ) 4LMM 0 4EDP \DM 0 M + \MM 0 L + \M 0 LM = \EDC + \DP E + \DEP = \EPC + \DPE + \DCP =90 ± 이므로 LM 과 DE 는수직이다. 33. 원 O 에내접하고, 다음의성질을만족시키는육각형 ABCDEF 를생각하자. \ABC > 90 ± ; 0 ± < j\acf \CADj < 90 ± 점 B 에서두직선 CA, CE 에내린수선의발을각각 X, Y 라하고, 점 B 에서두직선 DF, DE 에내린수선의발을각각 Z, W 라하자. 두직선 XY, ZW 가이루는예각을 µ 라할때, 다음을증명하여라. ( 한국 003-S4) µ = j\acf \CADj 증명 ( 고양백신중조상영 )

245 3. 기하중급문제 45 직선 ZW와직선XY 의교점 : K 직선 FD와직선AC의교점 : L 이라가정. \CAD = \CFD (* 원주각 ) 으로부터 j\acf \CADj = j\acf \CFDj 그런데, j\acf \CADj = \CLD. (* F 를지나며 FG k AC 인직선 FG 을잡으면, \ACF = \CFG j\cfd \CFGj = \GF D = \CLD ), 로인해 j\acf \CFDj = \CLD 임이증명된다.) 따라서 \CLD = µ 이면증명된다. 여기서 KF 와 LC 의교점을 M 이라가정하고 4KMZ 와 4LMZ 가닮음임을보일수있다. \KZL = \WZD ( 엇각 ) 이고, BZWD 는원에내접하므로 (* \BZD = \BWD 로원주각이같음 ) \WZD = \WBD ( 원주각 ). 그런데 \BDE = \BCE ( 원주각 ), BY? CE, BW? ED 이므로 4BDW»4BY C (AA 닮음 ). 그럼 \WBD = \YBC. 그런데 \YBC = \YXC ( 원주각 ) 이고, \YXC= \KXM ( 엇각 ) 이다. 결국 \KZL = \WZD = \WBD = \YBC = \YXC = \KXM =) \KZL = \KXM =) 4KMZ와 4LMZ가 AA닮음 (\LMZ = \KMX, 엇각 ). 따라서 \CLF = \XKZ = µ =) \CLF = µ. 따라서증명끝. 34. 네점 A, B, C, D 가고정된원의원주위에시계방향으로주어져있고사각형 ABCD 가원의중심을포함한다고한다. 네개의삼각형 ABC, BCD, CDA, DAB 의내심을각각 P, Q, R, S 라고할때, 사각형 PQRS 는직사각형임을증명하여라. ( 대학생수학경시 003-) 증명 ( 부산과학고 학년이태희 ) 라고하자.( ABCD 가원에내접 ) \DCA = = \DBA \ACB = = \ADB \DAC = = \DBC A γ γ δ δ S δ α R D B P α α α γ γ α γ β Q α αα β α β C 9 + \QCB = = + = ; \ACP = = \PCB ) \PBC = + \QBC = =) \QCP = =) \PBQ = = \QCP 그러므로, P, Q, C, B 한원위에존재하고, 따라서 \PQB = ; \QP C = 같은방식으로해서, 더해주면 \SPQ =90 ± 나머지도모두같은방식으로해주면모두 90 ± 가되므로직사각형이된다.

246 46 기하 35. 삼각형 ABC 에서변 AB 위에 4AD = AB 인점 D 를잡자. AB 에대해 C 와같은쪽의영역에, D 에서출발하여 DA 와 µ = \ACB 의각을이루는반직선 ` 을그린다. ABC 의외접원이반직선 ` 과 P 에서만날때, PB =PD 임을보여라. ( 영국 003 차 -) 증명 가정에의해, \ADP = \ACB = \AP B. ) 4ADP v 4AP B 따라서, AD : AP = AP : AB 이고, AP = AD AB =4AD AD =4AD 즉, AP =AD. 한편, AD : AP = PD : PB =: 그러므로, PB =PD. 36. 지름이 AB 이고중심이 O 인반원이있고, 선분 OB 위에서임의의점 C 를택했다. C 에서 AB 에수직한선을그어반원과만나는점을 D 라하자. 호 BD, 선분 CD, CB 와각각점 F, E, G 에서접하는원을생각하자. ADG 가이등변삼각형임을보여라. ( 이탈리아 003-3) 증명 F 로부터의확대 - 축소를생각하면수평지름위의점 E 와 A 는서로대응되므로 FEA 는일직선. 그럼 EFBC cyclic. AG = AE AF = AC AB = AD. 37. 현 AB 를공유하는세원이있다. 첫번째원위의임의의점을 X(6= B) 라하고, AX 가다른두원과만나는점을각각 Y, Z 라하자 (Y 가 X 와 Z 의사이에있다 ). A 를지나는임의의직선 AX(6= AB) 에대해, 비 XY : YZ 는언제나일정함을보여라. ( 캐나다 003-4) 증명직선 AX가어떻게그려지건상관없이, 동일현에대한원주각에의해 \AXB, \AY B, \AZB는언제나일정하다. 이로부터 4BXY 와, 4BY Z의내각들이일정하므로, 닮은꼴을유지하게된다. 따라서, XY : YB 와 YB: YZ 는언제나일정하고, XY : YZ 도언제나일정하다. 38. 점 O 을중심으로하는원 o 밖의점 A 에서원 o 에그은접선의접점을 B, C 라하자. 원 o 에접하는또다른접선이선분 AB, AC 와각각점 E, F 에서만난다. 직선 OE, OF 가선분 BC 와각각점 P, Q 에서만날때, 선분 BP, PQ, QC 는삼각형 AEF 와닮음인삼각형의세변의길이를이룸을증명하여라. ( 폴란드 003 차 -5)

247 3. 기하중급문제 47 증명 EF 와원 o 의접점을 D 라하면두접선의대칭성에서 BP = PD 이고 CQ = QD. 즉, 4AEF»4DQP 임을보이면됨. 이것은 BODE 와 ABOC 등이 cyclic 임으로부터금방확인됨. 즉, \PQD =\QCD = \BOD = \DEA. 39. 원 O 위에순서대로놓인서로다른네점 A, B, C, D 가있다. 삼각형 ABC 의변 BC 에접하는방접원의중심을 O, 삼각형 ACD 의변 CD 에접하는방접원의중심을 O 라하자. () 이때세점A, B, O 을지나는원의중심은원 O의원주위에놓임을보여라. () 점 C가호BD 위를움직일때, 세점C, O, O 를지나는원은항상원 O 위의한고정점을지남을보여라. ( 한국 004-S5) 증명 ( 서울신종현 ) () C 나 D 대신, 일반적으로 X 를원 O 위를움직이는점이라하자 ( 즉, X 는 C 나 D 일수있다 ). 이때 4ABX 의변 BX 에접하는방접원의중심 P 의자취가한원위에있음을보이면되겠다. 원 O 에서현 AB 의원주각을 w 라하자. \AP B = ¼ \PAB \PBA = ¼ \XAB µ ¼ + \XBA ¼ \XAB \XBA = = \AXB = w 따라서, \AO B = \AO B = w 로, ABO O 는원에내접한다. 그리고, 그중심은원주각-중심각의성질에따라원O 위에있는데, 특히 AB의수직이등분선이 (C, D를포함하는쪽의 ) 원주와만나는점이그중심이다. () 4ABO 의외심을 M, 4ADO 의외심을 N이라고하자.() 에서푼결과에서처럼, M은 (C와같은쪽의 ) 호 AB의중점이고, N은 (C와같은쪽의) 호 AD의중점이다. \BO C = \AO B + \AO C = \BO M =90 ± \BMO \ACB + \ABC =90 ± \BAO 4 =90 ± \BAC =90 ± \BAC 따라서 O, C, M은한직선위에있다. 마찬가지로 O, C, N도한직선위에있다. C, O, O 를지나는원이O와만나는 C가아닌점을P라고하면, 원주각에의해 \PO N = \PO C = \PO C = \PO M \PNO = \PNC = \PMC = \PMO 따라서, 4PNO 와 4PMO 은닮았고, 닮음비는 MP : NP = MO : NO = MA : NA 따라서 P 는 MP : NP = MA : NA 인점의자취인아폴로니우스의원과원 O 와의교점중에 A 가아닌점이므로고정된점이다.

248 48 기하 40. 정삼각형 ABC 의변 BC 위에점 D 가있다. D 에서 BC 와접하는원이변 AB 와 M, N 에서만나고변 AC 와 P, Q 에서만난다. BC + AM + AN = CD + AP + AQ 임을보여라. ( 영국 004 차 -) 증명 ( 인천연수고 학년임지웅 ) 원의할선, 접선의비례성질에따라 BD = BN BM =(AB AN)(AB AM) = AB (AM + AN)AB + AN AM CD = CQ CP =(AC AQ)(AC AP ) = AC (AP + AQ)AC + AQ AP ABC 는정삼각형이므로 AB = AC = a, 즉 AB = AC 이고, 역시할선의비례성질에의해 AM AN = AP AQ 이다. 이제앞의두식을비교하면 BD +(AM + AN)a = CD +(AP + AQ)a BD CD =(BD + CD)(BD CD)=BC(BD CD)=(BD CD)a 이므로, 이에의해양변을정리하고 a 로나누어주면 BD + AM + AN = CD + AP + AQ 의식을확인할수있다. 4. 삼각형 ABC 의변 AB 위에점 D 가있다. D 를지나고 AC 와 A 에서접하는원과, D 를지나고 BC 와 B 에서접하는원이점 E 에서다시만난다. 점 C 를 AB 의수직이등분선에대해대칭시킨점을 F 라할때, 세점 D, E, F 는한직선위에있음을증명하여라. ( 폴란드 차 -) 증명 \AEB +\C =(\AED + \BED)+\ACB = 접현각 \A +\B +\C =80 ± 이므로 AEBCF cyclic. \AED = \A = \ABF = \AEF 이므로 DEF 는일직선. 4. 예각삼각형 ABC 에서, D 는점 C 에서 AB 에내린수선의발이고, E 는점 D 에서 BC 에내린수선의발이다. 선분 DE 위에 EF : FD = AD : DB 인점 F 를잡으면, 직선 CF 와 AE 가서로수직임을증명하여라. ( 폴란드 004/005 차 -3) 증명 A에서 BC에내린수선의발을 G. 4ABG»4CDE. 그럼 4CFE»4AEG. 회전관계로 수직임. 43. 세구면은둘씩서로외접하며, 한평면과각각점 A, B, C 에서접한다. BC = a, CA = b, AB = c 일때, 이구면들의반지름을모두구하여라. ( 폴란드 004/005 차 -7) 풀이 a =(r + r 3 ) (r r 3 ) =4r r 3 등. 이로부터 (abc) =4 3 (r r r 3 ), 즉 abc =8r r r 3. r = abc = bc 8r r 3 a 등. } 44. \A =90 ± 인직각삼각형 ABC와변AB 위의세점 X, X, X 3 에대하여 \ACX = \X CX = \X CX 3 = \X 3 CB 이고, BX 3 =AX 이다. 삼각형 ABC의가장작은각의크기를a ± 라할때,7a의값을구하여라. ( 한국 005 차-J9) 풀이일반성을잃지않고 AX = 로두자. 그리고, X X = x, X X 3 = y 라하고, CA, CX, CX, CX 3, CB 를차례로 h, p, q, r, s 라고하자. 이제아래그림처럼 AC 에대해대칭시킨직각삼각형을함께고려해보자.

249 3. 기하중급문제 49 각의이등분선정리에서 x : y = p : r =:x + y y :=q : s = x +:x + y + 이두식에서 x, y에대한연립방정식 y = x(x + y), (x +)=y(x + y +) 를얻을수있다. 첫번째식에서구한 y = x () x 을두번째식에대입하여정리하면다음을얻는다. x 3 4x 4x +8=0 () 다시각의이등분선정리에서 :x = h : q, 즉여기다피타고라스의정리를적용하여 (hx) = q = h +(+x). 이로부터 h = +x (3) x 를얻는다. CX = X B ( ) 즉 p = x + y + 임을확인해보자. 양변을제곱하면, 피타고라스의정리에서 p = h + 이므로 h +=(x + y +) x x = µx + x x + x( x) =8( x) = (x x + x +4 x) ( x) = 6 ( x) 와같이동치변형되고, 이것을정리하면 () 의식과같아성립하는식임을알수있다. 따라서,( ) 이증명되었고, 그럼 4X BC가이등변삼각형이되어 \B = \X CB = 3 \C 4 가된다. 그럼작은각은 \B = a ± 이고, \B + \C =90 ± 이므로 a a = 7 3 a =90. 따라서, a = = 답 } 원 C 가원 C 에점 A 에서내접한다. A 를지나는직선이원 C, C 와각각 B, C 에서다시만난다. B 에서접하는 C 의접선이 C 와두점 D, E 에서만난다. C 에 C 에그은두접선이 C 과각각점 F, G 에서접한다. D, E, F, G 가한원위에있음을증명하여라. ( 북유럽 005-4) 증명 \CBD = \ABE = () \AF B = () \CDA. () 은접현각,() 는 AC가 AB의확대임을관찰하여. 이로부터 4CBD»4CDA. 그럼 CD = CA CB = CF 등등으로 C가중심인원위의네점이됨. 46. 삼각형 ABC 의변 BC, CA, AB 위에각각꼭지점이아닌점 D, E, F 가있다. AD, BE, CF 는한점 G 에서만난다. 세삼각형 AGE, CGD, BGF 의넓이가서로같으면 G 는 4ABC 의무게중심임을증명하여라. ( 아일랜드 005-) 증명 BD : DC, CE : EA, AF : FB 를각각:a, :b, :c 로두고열심히넓이비구하여연립방정식풀어보면 a = b = c =이됨. 잘하면a, b, c에대한대칭식관계로 을삼중근으로갖는 3차방정식을얻을수도있을것임. 47. 예각삼각형 ABC 의외심을 O 라하자. C 에서 AB 에내린수선의발을 D 라하고, D 에서 AC 와 BC 에내린수선의발을각각 E 와 F 라하자. 사각형 EOFC 의넓이는삼각형 ABC 의넓이의절반임을증명하여라. ( 폴란드 005/006 차 -3) 증명 O 에서 AC, BC 에내린수선의발을 P, Q. 그럼등적변형으로 j4dop j = j4eopj, j4doqj = j4foqj. 이로부터 j EOFCj = j DPCQj = j4abcj.

250 50 기하 별증 \ACD =, \BCD = 로하고 O에서 BC, AC에내린수선의발을 K, L이라하자. OK =sin, OL =sin. j EOFCj = (FC OK + EC OL) = R DC(cos sin +cos sin ) = R DC sin( + ). j4abcj = DC AB = R DC sin C. 48. B, C 는한점 A 에서출발하는두반직선위를각각움직이는점으로 AB + AC 는항상일정하다. B, C 가어떤위치에있어도삼각형 ABC 의외접원이항상지나는점 D 6= A 가존재함을보여라. ( 북유럽 006-) 증명 AB = AC 일때AD가지름이되는점 D가바로찾는그점. 이것을증명하자. 다른 B 0, C 0 을잡았을때, AB 0 C 0 D가 cyclic임을보이면됨. 4B 0 BD 4C 0 CD (SAS). 따라서, \BB 0 D = \CC 0 D. 내대각이므로증명끝. 49. AB = CD 이고평행사변형은아닌볼록사각형 ABCD 가있다. 점 M, N 은각각대각선 AC, BD 의중점이다. 선분 AB 와 CD 를직선 MN 위로정사영시킨그림자의길이는둘다 MN 의길이와같음을증명하여라. ( 폴란드 006/007 차 -3) 증명 AD, BC의중점을각각 P, Q. PMQN은마름모. 즉, PQ? MN. 그럼 AB의사영의길이는 PN의사영의길이의두배, 즉 MN이다. 50. 평면에한정육각형이주어져있다. 이평면의임의의점 P 에대해, P 에서이정육각형의각변을연장한직선에이르는거리들의합을 l(p ) 라하고, P 에서이정육각형의각꼭지점까지의거리의합을 v(p ) 라하자. (a) l(p ) 가최소가되는 P 는어떤점인가? (b) v(p ) 가최소가되는 P 는어떤점인가? ( 이탈리아 007-) 풀이 (a) 육각형내부의임의의점. 대칭되는두직선쌍의내부에있어야하므로그교집합.(b) 육각형의중심. 대칭점쌍들에대한삼각부등식. } 3. 기하고급문제. 평면위에주어진 n개의고정점으로부터의거리의제곱의합이일정한점들의자취는무엇일까? 이자취가공집합이아닐조건은, 기하학적관점에서말한다면, 무엇일까? (Putnam 95-A) 풀이 ( 강성경 ) n개의점을각각 P ;:::;P n 이라고하고 G를 P i 들의무게중심 ( 좌표평균점 ) 이라하자. 그리고, 좌표축을새로 G가원점이되도록잡자. 그럼 P i 의좌표가 (a i ;b i ) 일때 P a i = P b i =0이된다. 임의의점을 P (x; y) 라하고일정한합을 k라하면, k = X PP i = X ((x a i ) +(y b i ) ) = X x + y xa i yb i + a i + b i =( X x + y ) x( X a i ) y( X b i )+( X a i + b i ) = n(x + y ) 0 0+ X GP i = n(x + y )+ X GP i 따라서, 구하는자취는 8 < k< P GPi : 해없음 : q P : k GP otherwise : 반지름 = i ; n 중심G인원 이된다. }

251 3. 기하고급문제 5. O 와 O 0 은각각예각삼각형 ABC 와 A 0 B 0 C 0 내부의점이다. O 에서 BC, CA, AB 에내린수선의발을각각 D, E, F 라하고 O 0 에서 B 0 C 0, C 0 A 0, A 0 B 0 에내린수선의발을각각 D 0, E 0, F 0 이라하자. OD k O 0 A 0, OE k O 0 B 0, OF k O 0 C 0 이고, OD O 0 A 0 = OE O 0 B 0 = OF O 0 C 0 이다. O 0 D 0 k OA, O 0 E 0 k OB, O 0 F 0 k OC 이고, O 0 D 0 OA = O 0 E 0 OB = O 0 F 0 OC 임을보여라. ( 소련 968-8) 증명 4OEF»4O 0 C 0 B 0 (SAS). sin법칙에서 EF = OA sin A. 4O 0 B 0 C 0 의넓이를두가지방법으로생각해보면 (OD O 0 A 0 = k), O 0 B 0 O 0 C 0 sin A = B 0 C 0 O 0 D 0. 양변을각각변형하면 k OE k OF EF OA = k EF OE OF O0 D 0. 이로부터 k = O 0 D 0 OA 들도일정하다는것이확인된다. 그담에평행임을이로부터꿍짝꿍짝하면... 됨 ( 쓰기귀찮음 ;). 3. 네변의길이가각각 a, b, c, d이고, 원에외접하는사각형 ABCD의넓이가 p abcd이면이사각형은원에내접함을증명하여라. (Putnam 970-B6) 증명 ( 동주여중권수현 ) 원에외접하므로 a + c = b + d ( ) \ABC = ; \CDA = 라하자. A d S ABCD = S 4ABC + S 4CDA = ab sin + cd sin = p abcd a β D c B α b C 양변을제곱한후 sin = cos ; sin = cos 를대입하면 abcd = 4 (a b + c d +abcd sin sin a b cos c d cos ) 4ABC에코사인제법칙을적용하면 4CDA 에코사인제 법칙을적용하면 AC = a + b ab cos AC = c + d cd cos 3, 3에서 a + b c d = ab cos cd cos 제곱하면 (a + b c d ) = a b cos + c d cos abcd cos cos 4 4+4을하 4 면 (a + b c d ) +4abcd a b c d = abcd cos( + ) 5 4 d = a b + c 이다. (* ( )) 6 a + b c d =(a + c)(a c)+(b + d)(b d) =(a + c)(a + b c d) =(a + c)(b c) (* ( ); 6) (a + b c d ) a b c d 4 = (a + c) (b c) a b c (a b + c) = a b +ab c + b c a bc 4abc bc 3 +a c +ac 3 + c 4 a b a c b c c 4 +abc ac 3 +bc 3 = abc(b a c) = abcd 7

252 5 기하 7 을 5 에대입하면 (a + b c d ) +4abcd a b c d = abcd cos( + ) 4 abcd = abcd cos( + ) a; b; c; d 6= 0이므로 cos( + ) = ; 0 < + <360 ± 이므로 + =80 ± 이다. 두대각의합이 80 ± 이므로사각형 ABCD는원에내접한다. 4. 구면위의주어진두정점 A, B를촛점으로하는 \ 구면타원 " 은 \ PA+ _ PB _ = 상수 " 인구면위의모든점 _ P 들의집합이다. 단, PA는 P 에서 A까지구면을따라이르는가장짧은거리이다. 구면타원이원이되는경우를모두찾아라. (Putnam 97-B4) 풀이답은 A, B가같은점이거나지름의양끝점일때생기는모든구면타원과그렇지않을때는대원뿐이다. Answer: S(A, B, k) is a circle iff k = π and A and B are not antipodal, in which case S(A, B, k) is the great circle perpendicular to the great circle through A and B and so that A and B are on the same side of and equidistant from the plane containing it. Wlog we may take the sphere to have radius. Let the great-circle distance between A and B be d. So 0 d π. If d = 0, then clearly any k (in the range 0 < k < π) gives a circle. If d = π, then any point P on the sphere has PA + PB = π, so S is either empty or the whole sphere. So let us assume that 0 < d < π. Let C be the great circle through A and B. Let O be the centre. Let QR be the diameter with angle AOQ = angle BOR. Let B' be the reflection of B in QR. Then AB' is a diameter. Let C' be the great circle through Q and R perpendicular to C. For any point P on the sphere we have AP + PB' = π. If P lies on C', then by symmetry BP = B'P, so AP + PB = π. If P lies in the open hemisphere containing B, then BP < B'P, so AP + BP < π. Similarly, if P lies in the open hemisphere containing B', then BP > B'P, so AP + BP > π. Thus S(A, B, π) is the circle C'. Let us now assume 0 < k < π. If k < d, then S(A, B, k) is empty, since AP + BP cannot be less than AB. If k = d, then S(A, B, k) is the arc of C between A and B. So suppose d < k < π. Then there are two points on C' in S(A, B, k): Q' on the arc AQ and R' on the arc BR, with AQ' = BR'. Now S(A, B, k) must be symmetrical about the plane containing C', so if it is a circle C'', then it must be the circle diameter Q'R' and perpendicular to C'. It suffices to find a single point X on this circle with AX + BX > k. Let Y be the midpoint of the arc AB, and take X so that the great circle through X and Y is perpendicular to C'. Since we are assuming C'' is a circle we have YQ' = YX = YR'. Since the great circle through X and Y is perpendicular to C'', we must have AX > YX and BX > YX, so AX + BX > YX = YQ' = AQ' + BQ' = k, so Y is not in S(A, B, k) after all. Finally consider k > π. Let A' be the antipodal point to B. Now for any point P on the sphere, AP = π - B'P and BP = π - A'P, so S(A, B, k) = S(A', B', π-k). } 5. 볼록오각형 ABCDE 에서 5 개의삼각형 ABC, BCD, CDE, DEA, EAB 의넓이가각각 이다. 이런볼록오각형은모두같은넓이를가짐을보이고그넓이를구하여라. 또한, 이런성질을갖는합동이아닌오각형이무한히많이있음을보여라. ( 미국 97-5) 6. 3차원공간에꼬인사각형 ( 네꼭지점이한평면위에있지않은사각형 ) 이있다. 이사각형의마주보는대각끼리서로크기가같다면, 마주보는변끼리도서로길이가같음을보여라. (Putnam 97-B5)

253 3. 기하고급문제 53 증명 It is worth looking first at the coplanar case. If ABCD is convex, then opposite angles are equal, so it is a parallelogram and opposite sides are equal. In this case the result is true. But if ABDC is convex, then ABC = ADC implies that A, B, D, C lie on a circle and hence that BAD = BCD. In this case it is certainly not true that AB = CD or that BC = AD. Applying the cosine formula to angles ABC and ADC gives: (AB + BC - AC)/( AB BC) = (AD + CD - AC)/( AD CD). Hence (AB CD - AD BC)(AB AD - BC CD) + AC(AB.BC - AD.CD) = 0 (*). Similarly, applying it to the other two angles gives: (BC + CD - BD)/( BC CD) = (AB + AD - BD)/( AB AD) and hence (AD BC - AB CD)(AB BC - CD AD) + BD(BC CD - AB AD) = 0 (**). Put x = (AB CD - AD BC), y = (AB AD - BC CD), z = (AB BC - AD CD). Then (*) and (**) become xy + ACz = 0, xz + BDy = 0. So if either of y or z is 0, then the other is also. But if y and z are 0, then AB/CD = BC/AD = AD/BC, so BC = AD and hence also AB = CD. On the other hand, if neither y nor z is 0, then we can deduce that ACBD = x, and hence AC BD = ±x = ±(AB CD - AD BC). Thus either AC BD + AD BC = AB CD or AC BD + AB CD = AD BC. In either case we can use Ptolemy's theorem which tells us that A, B, C, D must be () coplanar, and () concyclic (or collinear). But we are told that they are not coplanar. Hence BC = AD and AB = CD as required. 7. 한변의길이가 x 인정삼각형 DEF 의내부에점 M 이있고, MD = a = BC, ME = b = CA, MF = c = AB 를세변의길이로하는삼각형 ABC 의내부에 AD = u, BD = v, CD = w 이고 \ADB = \BDC = \CDA =0 ± 인점 D 를잡았다. x = u + v + w 임을증명하여라. ( 미국 974-5) 8. 두반직선 OA 와 OB 사이에점 P 가놓여있다. 반직선 OA 위의점중에서다음을만족하는점 X 를찾아라 : XP 를연장하여반직선 OB 와만나는점을 Y 라할때, XP PY 의값이최소이다. (Putnam 976-A) 증명 (Kalva) Answer: take OX = OY. Let M be the foot of the perpendicular from P to OA, and N be the foot of the perpendicular from P to OB. Obviously X must lie further from O than M - otherwise moving it closer to M would reduce both PX and PY. Similarly Y must lie further from O than N. Let φ be the angle MPX and let θ be the angle MON. Then the angle NPY is θ-φ. Hence PX PY = PM PN/(cos φ cos θ-φ). So we minimise PX.PY by maximising (cos φ cos θ-φ). But (cos φ cos θ-φ) = / (cos(φ+θ-φ) + cos(φ-θ) ) = / cos θ + / cos(φ-θ). This is obviously maximised by taking φ = θ-φ. But that corresponds to angle PXM = angle PYN and hence OX = OY. Alternatively, PX PY may remind us of the elementary result that if XX'YY' is cyclic with diagonals XY, X'Y' meeting at P, then PX PY = PX' PY'. The problem is to find a suitable circle. With hindsight, it is fairly obvious. Take the circle which touches OA at X and OB at Y. Then any other line through P intersects the circle at X' and Y' before it intersects OA and OB (at X'' and Y'' say). Hence PX'' > PX', PY'' > PY' and PX' PY' = PX PY. However, I failed to find the right circle until after I had solved the problem trigonometrically.

254 54 기하 9. \BAD =60 ±, \BAC =40 ±, \ABD =80 ±, \ABC =70 ± 인사각형 ABCD 가있다. 직선 AB, CD 는서로수직임을보여라. ( 폴란드 차 -) 증명삼각형 ABD의내부에 C가있고, 각 D를 30 ± :0 ± 로나누는직선을그리면각-체바 ( 역정리 ) 에의해 sin 30 ± sin 40 ± sin 0 ± =sin0 ± sin 0 ± sin 70 ± 와동치이고, 이것은 sin 40 ± =sin0 ± cos 0 ± 의 배각공식과동치. 별증 ABE가정삼각형이되는 E AD 를잡고, E의 BC에대한대칭점 F 를잡으면CEF가정삼각형이됨. 0. 볼록사각형 ABCD 의두변 AD 와 BC 의연장선이 E 에서만난다. H 와 G 를각각 BD 와 AC 의중점이라고하자. 삼각형 EHG 와사각형 ABCD 의넓이의비를구하여라. ( 캐나다 978-4) 풀이다음과같이풀수있다. 4j4EHGj =4j4GHBj +4j4GBEj 4j4HBEj =j4gdbj +j4abej j4dbej =j ABGDj =j4agbj +j4agdj = j4acbj + j4acdj = j ABCDj 혹은, j4ehgj = j4dhej + j4dhgj j4dgej 로비슷하게풀어도된다. } 별해 E 를원점 (0; 0) 이라하고 BC 가 x- 축위에있다고하자. 직선 ED 의식을 y = hx 라하고 A, B, C, D 의 x- 좌표를각각 a, b, c, d 라하면 B(b; 0); C(c; 0); A(a; ah); D(d; dh); 그리고 H(b + d; dh); G(a + c; ah) 사각형 ABCD의넓이는abh cdh 이며, 삼각형 EGH의넓이는 b + d a+ c dh ah = ((b + d)ah (a + c)dh) = abh cdh 따라서, j4eghj : j ABCDj =:4. } 별해 E 를원점이라하고, A, D 방향의단위벡터를 ~v =(v x ;v y ), B, C 방향의단위벡터를 ~w = (w x;w y), 그리고 A, B, C, D 에대한벡터를각각 a~v, b~w, c~w, d~v 라하자. 두넓이를각각구하면 j ABCDj = j4eabj j4ecdj = av x bw x av y bw y dv x cw x dv y cw y = (ab cd)jv xw y w x v y j j4eghj = (av x + cw x ) (bw x + dv x ) (avy + cwy) (bwy + dvy) = 8 v x w x v y w y a d c b = 8 (ab cd)jv xw y w x v y j 따라서, j4eghj : j ABCDj =:4임을알수있다. }

255 3. 기하고급문제 55. 각 A와그내부의점 P 가주어져있다. P 를지나는직선이두반직선과만나는점을각각 B, C라할때, BP + 이최대가되도록작도하려면어떻게하면될까? ( 미국 979-4) PC 풀이 P 에서 AB에평행선을그어 AC와만나는점을C 0 이라하자. 다시 C 0 에서 BC에평행선을그어 AP 와만나는점을 P 0 이라하자. 그럼 BP + CP = P 0 C 0 이성립한다. } 풀이 P 를지나는임의의현을 B 0 C 0 이라하고, P 를지나 AB 0 에평행한직선이 AC 와만나는점을 D, D 를지나 B 0 C 0 에평행한직선이 AP 와만나는점을 P 0 이라하자. 그러면 D C C P A P B B 4AP 0 D 4AP C 0 4PDP 0 4AB 0 P 이므로 P 0 D PC 0 = AP 0 AP ; ) P 0 D B 0 P + P 0 D P 0 D B 0 P = P 0 P AP PC 0 = AP 0 + P 0 P AP = AP AP = ) B 0 P + PC 0 = P 0 D 이식의좌변이최대가되려면 P 0 D가최소가되어야한다. 그런데, DP 0?AP 일때 DP 0 이최소이므로, BC가 AP 에수직이되도록현 BPC를작도하면된다. }. 삼각형 A A A 3 내부에점 P 가있다. i =; ; 3 에대해직선 PA i 가 A i 의대변과점 B i 에서만난다. A i B i, PB i 의중점을각각 C i, D i 라하자. 삼각형 C C C 3 와 D D D 3 의넓이가같음을증명하여라. ( 베트남 980-3) 증명 ( 양준호 ) µ A B 3 = p B 3 A = p µ A B = q B A 3 = q µ A3 B = r B A = r A A ;A A 3 ;A 3 A 의중점을각각 T ;T ;T 3 라하자. 그러면중점연결정리의역에의해 C T T ; C 3 T T 3 ; C T 3 T ( 자명 )

256 56 기하 한편 4A A A 3 4T T 3 T 이고닮은비 : [ 그림 ] [ 그림] 또한닮음에의해 0 T C : C T = r : T c 3 : C 3 T 3 = p : p A : T 3 C : C T = q : p 다시 A P; A P; A 3 P 의중점을 S ;S ;S 3 라하면D 3 S S ;D S S 3 ;D S 3 S 한편 4A A A 3 4S S S 3 닮은비 : 또한닮음에의해 0 S D 3 : D 3 S = p : S D : D D 3 = q : q S 3 D : D S = r : r [ 그림 ] 에서 jt T T 3 j =이라고하면 jc C C 3 j = jt T T 3 j jt C C j jt C C 3 j jt 3 C 3 C j = r q r p p q r + r q + q r + r p + p p + p q + q ½ ¾ (p + p )q r + p (q + q )r + p q (r + r ) = (p + p )(q + q )(r + r ) p q r + p q r = (p + p )(q + q )(r + r ) = A ) jc C C 3 j = jt T T 3 j A [ 그림 ] 에서 js S S 3 j =이라고하면 jd D D 3 j = js S S 3 j js D D 3 j js D 3 D j js 3 D D j ½ p = r + p q + q r ¾ p + p r + r p + p q + q q + q r + r 한편, 에의해 = p (q + q )r + p q (r + r )+(p + p )q r (p + p )(q + q )(r + r ) p q r + p q r = (p + p )(q + q )(r + r ) = A ) jd D D 3 j = jt T T 3 j p q r + p q r (p + p )(q + q )(r + r ) = jt T T 3 j A 4T T 3 T 4S S S 3 ) jt T T 3 j = js S S 3 j ) jc C C 3 j = jt T T 3 j A = js S S 3 j A = jd D D 3 j ) jc C C 3 j = jd D D 3 j ) 증명끝

257 3. 기하고급문제 원의지름 AB의양끝점에서그은접선 l, m이있다. 직선 l 위의점 C (C 6= A) 를지나는두직선 q, q 에대해, q i 가원과만나는두점을점C에가까운순서로 D i, E i (i =; ) 라한다. 직선 AD, AD, AE, AE 가직선m과만나는점을각각 M, M, N, N 라고할때, M M = N N 임을증명하여라. ( 오폴 980-9) 증명 ( 신한솔 ) \AE D = \ABD = \AM B = \AM N, \A는공통이므로삼각형AD E 와 AN M 은닮음이다. M N 의중점을 P, D E 의중점을 G라하면삼각형 AN P 와삼각형 AD G도닮음이다. G가 D E 의중점이므로 \CGO = 90 = \CAO. 따라서 C, A, O, G는한원위에있다. 그럼 \AOC = \AGD = \AP N 이고 AB와 m이직교하므로 OC와 AP도직교한다. A에서 CO에내린수선과 m의교점은하나밖에없으므로마찬가지로 M N 의중점도M N 의중점P와일치한다. 그러므로 M M = N N 가당연히성립. 4. m 과 n 은 m<n 인두선분의길이이다. 다음조건들을만족하는삼각형 ABC 를작도하여라 : (i) 각 B 의이등분선이변 AC 와 E 에서만나고각 A 의이등분선과 M 에서만난다고할때, 삼각형 AEM 의외접원은직선 AB 와접한다. (ii) AE = m 이고 EC = n 이다. ( 불가리아 98 3 차 -3) 풀이 ( 임준혁 ) 일단선분 AC 를그리자. E 는선분 AC 위에있으므로 E 를잡을수있다. AB : AC = AE : EC = m : n 이므로 B 는 AE : EC 를 m : n 으로내분하는점과외분하는점사이선분을지름으로하는아폴로니우스의원위에있다. 그리고조건 i 에서 4AEM 과 AB 가접하고, 내대각에의해 \AEM = \BAM 이고, AM 은각 A 를이등분하므로, \AEM = \EAM 이므로 AM = ME 이고, M 은 AE 의수직이등분선위에있다는것을알수있다. AM 이 \A 를이등분하므로, AB : AE = BM : ME B;M 에서직선 AC 에내린수선의발을각각 X; Y 라하자. 이때 Y 는 AE 의중점이다. BM : ME = XY : YE= XY : m 그런데 AE = m 이므로, AB =XY 임을알수있다. A 를 X 에대해점대칭이동한점을 Z 라하자. ( 즉, Z = AX = XZ 를만족하는직선 AC 위에있고, A 와다른점 ) AE = m 이므로, ZE =XY 이고, AB = ZB 이므로 ZB = ZE 이다. 그러면 Z 는아까작도한아폴로니우스원의중심일수밖에없다.( 그렇지않다고가정하자. ZE 가반지름이고, Z 가중심인원을생각할때, B 는그원위에있다. 그런데아폴로니우스원의중심은직선 AC 위에있으므로, 두원은접하게되고, B 는두원위에동시에있어야하기때문에모순 ) 그것을생각하면 B 를작도할수있다. } 5. 주어진사면체에내접하는구면이이사면체의어느세면과각각그수심에서접하고있다. 이사면체가정사면체임을증명하여라. ( 불가리아 98 3 차 -6)

258 58 기하 증명 ( 윤형석 ) 사면체 ABCD는 4ABC; 4ACD; 4ABD의수심에서접한다고하자. 사면체 ABCD( 위의그림참고 ) 에서사면체 ABCD에내접하는구는삼각형 ABC와삼각형 ACD와각각수심I;J에서만난다고하자.(AE; CG; AF; CH는각각꼭지점에서대변에내린수선이다.) 4AIC 4AJC(SSS합동 ) 구밖의한점A에서구에그은두접선 AI = AJ이고, 마찬가지로 CI = CJ이고또한 AC는공통이다. 4AEC 4AFC(RHA합동 ) \AEC = \AF C =90 ± ; \CAI = \CAJ(* ), AC는빗변으로서공통 3 4AGC 4AHC(RHA합동 ) \AGC = \AHC =90 ± ; \ACI = \ACJ(* ), AC는빗변으로서공통 3에의해서 \CAB = \CAD임이증명되었다. 또한마찬가지로하면 \CAB = \BAD임도증명할수있고, \CAB = \CAD = \BAD이다. 4 4ABE 4ADF (SAS합동) \BAE = \DAF(* \CAB = \CAD이고, \CAI = \CAJ이므로, \BAE = \CAB \CAI = \CAD \CAJ = \DAF) \AEB = \AF D =90 ± ;AE = AF (* 4AEC 4AF C) 4에의해서 AB = AD가증명되었다. 마찬가지방법으로하면 AB = AC임을증명할수있고, AB = AC = AD이다. 5 4ABC 4ACD 4ABD(AB = AC = AD이고, \CAB = \CAD = \BAD이므로 SAS합동 ) 6 4BCD는정삼각형 (* 5) 7 구는 4BCD와역시수심에서접한다. 4BCD와는 K에서접한다고하자. BI = BK;CI = CK;DJ = DK인데 4ABC와 4ACD는합동이고 AB = AC = AD인이등변삼각형이므로, BI = CI = DJ가되고, 따라서 BK = CK = DK이다. 따라서, K는 4BCD의외심이된다. 그런데, 4BCD는정삼각형이므로, K는수심이기도하다. 7에의해서주어진구는모든면과수심에서접한다. 따라서,»6와같은방법으로하면, 모든면이정삼각형임을증명할수있다. 따라서, 사면체ABCD는정사면체이다. 6. 한직선이한삼각형을똑같은넓이와똑같은둘레를갖는두영역으로나눈다면그직선은그삼각형의내심을지남을보여라. ( 유고슬라비아 98 고 4-) 증명 WLOG, D는 AB 위의점, E는 BC 위의점이라하자 (D = A 인경우도포함하는것으로함 ). 각 B의이등분선과 DE의교점을 I라하자. I에서 AB, BC에이르는거리를 ( 같으므로 ) r, CA에이르는거리를q라하자. A로부터 D의 배점을 F 라하고, C로부터 E의 배점을 G라하자. 그럼둘레관계에서 AC = BF + BG, 넓이관계에서 AC q =(BF + BG)r. 즉, q = r 이되므로끝. 7. 사면체 ABCD 의높이들을외부로연장하여각각 AA 0 = k h a ; BB 0 = k h b ; CC 0 = k h c ; DD 0 = k h d 가되도록점 A 0, B 0, C 0, D 0 을잡는다. 여기서 k 는상수이고, h a 는 A 에서의높이이며나머지도비슷하게정의된다. 사면체 A 0 B 0 C 0 D 0 의중심과 ABCD 의중심이일치함을보여라. ( 캐나다 98-5)

259 3. 기하고급문제 59 증명 사면체 ABCD 의부피를 V 라하고, k = k 0 V 라두자. 그럼 AA 0 = k 0 j4bcdj; BB 0 = k 0 j4cdaj; CC 0 = k 0 j4dabj; DD 0 = k 0 j4abcj 가된다. 면 BCD 의바깥쪽으로이면에수직하고이면의넓이를길이로갖는벡터를 ~a 라하자. 비슷하게면 CDA, DAB, ABC 에대해서도벡터 ~ b, ~c, ~ d 를정의하자. 그럼! AA 0 = k 0 ~a;! BB 0 = k 0 ~ b;! CC 0 = k 0 ~c;! DD 0 = k 0 ~ d 이된다. 이때, 사면체 A 0 B 0 C 0 D 0 의중심이 ABCD 의중심과일치한다는것은 A 0 + B 0 + C 0 + D 0 = A + B + C + D 라는것이므로 ~a + ~ b + ~c + ~ d =0 과동치가된다. 이것을증명하기위해, 임의의점 X 에대해 ~X (~a + ~ b + ~c + d)=0 ~ 임을보이자. A에서 BCD에내린수선의발을 H라하면! AX ~a = V 가된다. 실은, AH에의사영을생각하면, 평면 BCD 위의임의의점 P 에대해! AP ~a = V 이다. 이를이용하면,( 사면체내부의 ) 임의의점X에대해! AX ~a = V jxbcdj 가된다. 여기서 jxbcdj는사면체 X-BCD의부피이다. X-BCD와같은작은사면체4개의부피를모두더해도V가되므로, 임의의점 X에대해! AX ~a +! BX ~ b +! CX ~c +! DX d ~ =3V 로일정한값을갖는다. 그럼 X를무게중심 G로택해도! AG ~a +! BG ~ b +! CG ~c +! DG d ~ =3V 가되고, 두식을변변빼면! GX (~a + ~ b + ~c + d)=0 ~ 가된다. 여기서! GX의방향은임의이므로 ~a + ~ b + ~c + d ~ =0일수밖에없다. 8. 각각의실수 r 에대해, T r 은점 (x; y) 를점 ( r x; r r x + r y) 로옮기는평면위의변환이라하자. F 를그러한모든변환의집합이라하자. F 내의어떤변환으로도그그래프가변하지않는곡선 y = f(x) 를모두찾아라. ( 캐나다 983-) 풀이 (x; y) 가 y = f(x) 위의점이면 T r (x; y) =( r x; r r x + r y) 도그그래프위의점이다. 즉, f() = a 라두고 x =, r = t 로치환하면 f( r x)=r r x + r f(x) 8x; r R ( ) f(t) =t log t + at (t >0) 비슷하게 f( ) = b 라두고 x =, r = t 로치환하면 f( t) = t log t + bt (t >0) 또 ( ) 에 x =0 을대입하면 f(0) = 0 도얻을수있다. 즉, 8 >< x log x + ax (x >0) f(x) = 0 (x =0) >: x log ( x) bx (x <0) 가되고, 이것은 ( ) 에대입하면모두잘성립함을확인할수있다. }

260 60 기하 9. 4ABC의외심O가삼각형내부에있다. O에서각변에내린수선을연장했을때외접원과만나는점을각각 K, M, P 라하자.!!!! OK + OM + OP = OI 임을증명하여라. 단, I 는 4ABC 의내심이다. (Towns 983 가을 S3) 증명잘알려진MA = MI = MC 에의해MP는 AI의수직이등분선이므로 MP? KI. 비슷하게하여 I가 KMP의수심임을알수있다. 그럼또잘알려진외심-무게중심-수심의관계에의해, KMP의무게중심을 X라하면3! OX =! OI. 0. 원에내접하는사각형 ABCD 에서네삼각형 BCD, CDA, DAB, ABC 의수심을각각 H A, H B, H C, H D 라하자. 사각형 ABCD 와사각형 H A H B H C H D 가합동임을보여라. ( 발칸 984-) 증명그림을잘그려보면두사각형이점대칭관계에있을것으로예측됨. AHA, BH B 의중점을각각 X, Y 라할때X = Y 임을보이면충분 ( 그럼순환적으로 CH C, DH D 의중점들도모두일치하게되고, 이일치하는중점이점대칭의중심이됨 ). 이것은 ABH A H B 가평행사변형임과동치. 그리고, 이것은 CD의중점을 M이라할때잘알려진사실! BH A = OM! =! AH B 임에의해성립.. 원 C에내접하는정칠각형 A A A 3 A 4 A 5 A 6 A 7 의작은호A 7 A 위에점 P 가있다. 다음식을증명하여라. ( 오폴 984-4) PA + PA 3 + PA 5 + PA 7 = PA + PA 4 + PA 6 증명 A A = x, A A 3 = y, A A 4 = z 라하자. PA A A 3, PA 7 A 6 A 5 등에서톨레미정리에의해 PA x + PA 3 x = PA y, PA 7 x + PA 5 x = PA 6 y 가되어변변더하면Lx =(R PA 4 )y ( 단, L, R은증명할식의좌변과우변 ). 또, PA A 4 A 6 에서톨레미로 (R PA 4 )y = PA 4 z. 이두 Ry Lx 식에서 PA 4 = = Ry y y + z, 즉 Lx(y + z) =Ryz. A A 3 A 5 A 7 에서톨레미로 xy + xz = yz 가나오므로 L = R 이성립.. 반지름이 인원에내접하는한볼록오각형의어느두대각선이오각형안에서서로직교한다고한다. 이오각형의넓이는최대얼마인가? (Putnam 984-A4) 증명 Answer: + (3 3)/4. Let the pentagon be ABCDE with AC and BD the perpendicular diagonals. It is clear that E should be taken as the midpoint AD (since this maximises the area of ADE without affecting the area of ABCD). There are evidently two independent variables, so in principle all we have to do is to choose the variables, express the area as a function of the two variables and set its partial derivatives to zero. The difficult part is choosing the variables. If, for example, we take the two lengths AC, BD, then we get a simple expression for the area of ABCD, but a messy expression for the area of ADE. The best choice seems to be the angles subtended by the sides at O, the center of the circle. Note that if AB subtends α and CD subtends β, then angle ACB = α/ and angle CBD = β/. But if AC and BD are perpendicular, then these two angles must sum to π/. So α and β sum to π. Thus we may take as our two independent variables α and θ, the angle AOE. Then x area ABCDE = area AOB + area BOC + area COD + area DOE + area EOA = sin α + sin(θ) + sin α + sin θ cos θ. We maximise sin α at by taking α = π/, and we maximise sin θ + sin θ cos θ at (3 3)/4 by taking θ = π/3 (for example, write k = sin θ, so ( - k) = cos θ and differentiate).

261 3. 기하고급문제 6 3. 삼각형 ABC 의내심 I 을지나는한직선이내접원과 D, E 에서만나고, 외접원과는 F, G 에서만난다. DF EG r 임을보여라. 단, D 는두점 I 와 F 사이에있고, r 은내접원의반지름이다. 그리고, 등호는언제성립하는가? ( 발칸 986-) 증명 FI IG =(R+OI)(R OI) =R OI =Rr (Euler). 또한 FI IG =(r +DF)(r +EG) = r +(DF +EG)r+DF EG r +(R r)r+df EG. ( 위의부등식은 DF +EG = FG r R r 로부터얻음.) 이두식을비교하면 DF EG r 이바로나옴. 등호는 FG =R 일때, 즉지름일때. 4. 세변의길이가 a, b, c 인예각삼각형 T 에대해, T 를밑면으로하고옆면도모두같은넓이의예각삼각형인사면체를모두구하여라. ( 이탈리아 986-5) 풀이 BC; CA; AB; DA; DB; DC의길이를각각 a; b; c; d; e; f라하자. 밑면과부피가일정하므로각꼭지점에서내린높이도모두일정하다. 높이를 h라하고, 각면의넓이를 K 라하자. D에서 ABC에내린수선의발을 H라하고, H에서 BC, CA, AB에내린수선의발을 P, Q, R이라하자. 삼수선의정리에의해 DP? BC 등이성립하고, DBC의넓이를생각할때 K = a DP. 피타고라스정리에의해 HP =( K a ) h. HQ, HR도마찬가지로나오고, 세삼각형HBC, HCA, HAB의넓이의합을생각하면 K = a HP + b HQ + c HR = p K (ah) + p K (bh) + p K (ch). 다른면을밑면으로보았을때도각각마찬가지의식을얻을수있으므로, K = q K (ah) + q K (bh) + q K (ch) K = q K (ah) + q K (eh) + q K (fh) K = q K (dh) + q K (bh) + q K (fh) K = q K (dh) + q K (eh) + q K (ch) 처음두식의합과뒤의두식의합을비교하면 a = d. 마찬가지로 b = e, c = f. 즉, 대변끼리길이가같은, 네면이모두합동인사면체. } 5. 삼각형 ABC 의내접원이변 BC, CA, AB 와각각점 D, E, F 에서만난다. X, Y, Z 를각각 EF, FD, DE 의중점이라하자. ABC 의내접원의중심, ABC 의외접원의중심, 그리고 XY Z 의외접원의중심이한직선위에있음을증명하여라. (IMO-LL 986, 이스라엘출제 ) 풀이 ( 대전과학고조주형 ) O 0 을삼각형 ABC 의외심, O 을삼각형 ABC 의내심, O 00 을삼각형 XY Z 의외심이라하자. 4AF X 4AEX(AF = AE, AX 는공통, \FAX = \FAX = ) X 는 FE 와 AO 의교점이고, \EXO = \R ) \OEX = \ZEO = (4)

262 6 기하 A β β F α β X β γ E O Y β γ O O β α Z B α α α + β - γ α γ P D M N γ γ C 같은방법으로 사각형 XY ZE 는평행사변형이므로 (XY==ZE, XY = ZE) \YFO= \YDO= (5) \ZDO = \ZEO = (6) \XY Z = \XEZ = + ((), (3) 에서 ) 또, XO? YZ (XO? ZE==Y Z) ) \OXY = ¼ ³ ( + ) = + + = ¼ 위와같은방법으로하면, \OXY = \OZY = \OY X = \OZX = \OY Z = \OXZ = 한편, \O 0 BA = Á라하면, O 0 은삼각형ABC의외심이므로, \O 0 BA = \O 0 AB = Á \O 0 AC = \O 0 CA = Á \O 0 CB = \O 0 BC 그런데, \O 0 CB = + Á \O 0 BC = Á ) + Á = Á ) Á = + + : 같은방법으로하면, \O 00 ZX = \O 00 XY = \O 00 XY = \O 00 YX = \O 00 YZ = \O 00 ZY = (4)

263 3. 기하고급문제 63 XO 00 의연장선과 BC 와의교점을 M, XZ 의연장선과 BC 와의교점을 N 이라하면, \MXN = ((4) 에서 ) \MNX = \NCZ + \NZC = + ) \XMN = ¼ ( + + ) = ¼ DO = EO = FO = a 라하면삼각형 XOZ 에서 XQ sin = XZ sin \XOZ = a cos ³ ¼ = a sin + ³ ³ ¼ XZ = YD= a cos ; \XOZ = ¼ ( + ) =¼ = ¼ + ) XO = a sin (5) 또 XO 0 = XZ cos = a cos cos = a (6) O 에서 XM 에내린수선의발을 H 라고하면, OH = XO sin \OXO 00 = XO sin j j = a sin sin j j (7) XH = XO cos( ) =a sin cos( ) ((5) 에서 ) (8) ) O00 H OH = = XH XO00 OH = sin cos( ) sin sin j j a sin cos( ) a a sin sin j j (6), (7), (8) 에서 ) (9) O 0 에서 BC 에내린수선의발을 P 라고하면, BD = a tan ; DC = a tan a BD + DC tan + a tan a(tan +tan ) BP = = = tan tan ) PD = jbd BPj µ = a a tan tan + a tan = a tan tan (0) O 0 P = BP tan \O 0 BP = tan( + ) = a(tan +tan ) tan tan a(tan +tan ) tan tan tan ³ + + = ¼ ()

264 64 기하 (0), () 에서 OD O 0 P PD = = = = = a(tan +tan ) a tan tan tan a tan tan tan tan tan tan tan (tan +tan ) tan j tan tan j sin sin sin cos (sin cos +cos sin ) sin j sin cos cos sin j 4sin cos sin sin cos sin( + ) sin cos sin j j 4sin µsin sin + sin sin sin j j () () 에서, 4sin µsin sin + sin = 4sin ½ (cos( + ) cos( )) + sin ¾ = 4sin cos( ) ³ ³ ¼ cos( + ) =cos sin = sin cos( ) ) OD OP PD = ) O0 H OH = OD O0 P PD sin cos( ) sin sin j j O 0 O 와 OO 00 의 BC 에대한기울기가같으므로, O 0, O, O 00 은동일직선상에있다. } 6. ABC 는삼각형이고 X, Y, Z 는각각 BC, CA, AB 위의점으로선분 AX, BY, CZ 는 ABC 내부의한점 D 에서만난다. 사각형 DY AZ, DZBX, DXCY 중두개가원에외접하면나머지하나도또한원에외접함을보여라. (IMO-LL 986, 터키출제 ) 풀이 A B c Z b a Y c b X a C BC = a; CA = b; AB = c; AD = a 0 BD = b 0 ; CD = c 0 : [ 정리 ] 4 각형 DY AZ 이외접한다.() b 0 c 0 = c b

265 3. 기하고급문제 65 위의정리에의해사각형 DCBX와사각형 DXCY 가원에외접하면 a 0 b 0 = b a c 0 a 0 = a c 두식을더하면 c 0 b 0 = b c 그러므로사각형 DY AZ이원에외접한다. } 정리의증명 DY AZ 이외접한다고가정하자. DY, YA, AZ, ZD 가원과만나는점을 P, Q, R, S 라놓으면 b 0 c 0 = BP CS = BR CQ = c b. 역으로 DY AZ 가내접원을가지지않는다고가정하자. ABY 의내접원을생각하자. 점 C 에서그내접원에접선을그린다. 그것은 D 가아닌 E 점에서 BY 와만난다. 위에서했던논리로 명백히 EC = c 0 +(EC CD) () 에대입하면 c b = b 0 c 0 +(ede EC + CD) c b = BE EC () BE = b 0 + ede; (e =; ) ede EC + CD 6= 0(CDE 는삼각형이므로 ) 이리하여증명되었다. 7. 육각형 ABCDEF 내부에여섯각 AOB, BOC, COD, DOE, EOF, FOA 가모두 60 ± 가되도록점 O 를잡았다. OA > OC > OE 이고 OB > OD > OF 라면, AB + CD + EF < BC + DE + FA 임을증명하여라. ( 레닌그라드 987-4) 증명각꼭지점들을적당히회전하여 C, E를반직선 OA 위로, D, F 를반직선 OB 위로이동한후생각하자. AF 가 CD와만나는점을 X라고하면, ABXC와 XEFD에각각삼각부등식을적용하여합하면 AB + CX < AX + BC, XD + EF < XF + DE 로끝. 정n각형에서라거나, 두반직선혹은두평행선위의 n쌍의점에대해서라거나하는문제로확장하는것도금방되겠음. 8. 구면 S 와그내부의점 P 가주어져있다. P 를지나고서로직교하는세개의현을그리면, 세현의길이의제곱의합은이세현의방향을어떻게잡았느냐에관계없이일정함을증명하여라. ( 오폴 987-) 증명세현을AA 0, BB 0, CC 0 이라하고, 중심 O에서세현에이르는거리를각각 a, b, c라하자. 그럼 AA 0 +4a =4R 등이고, 이들을모두더하면 AA 0 + BB 0 + CC 0 +4(a + b + c )=R. 이제 a +b +c 이일정하다는것만확인하면된다. a, b, c는 OP를주대각선으로하는직육면체의세면대각선들이므로, 이직육면체의가로, 세로, 높이를각각 x; y; z라하면a +b +c =(x +y +z )=OP. 따라서, 일정 ( 그값은R 8OP ). 주평면버전의문제를먼저생각해보는것이힌트가될수있다. AA 0 + BB 0 +4OP =8R. 9. ABCD 는평행한변을갖지않는볼록사각형이다. 대변끼리연장하여이루는두각의이등분선이사각형 ABCD 의변과만나는점들을 P, Q, R, S 라하자. 단, PQRS 도볼록사각형이다. 사각형 ABCD 가원에내접할때, 또그때만, PQRS 가마름모임을증명하여라. ( 오폴 988-3) 증명 WLOG, AB와 CD가 A, D쪽연장의점K에서만나고, AD와 BC는 A, B쪽연장의점 L에서만난다고하자. 그리고, \K =a의이등분선이 AD, BC와 P, R에서만나고, \L =b의이등분선이 AB, CD와 Q, S에서만난다고하자. 또, KR \ LS = M 이라하고 \C = c 라하자. 그럼 4KBC, 4LCD의외각에서 \KBL =a+c, \KDL =b+c 이고, KMLC의외각에서 \KML = a+b+c. 따라서, ABCD가 cyclic () (a + b + c) =\KBL+ \KDL =80 ± () \KML = a + b + c =90 ± () 4KQS와 4LP R이이등변삼각형 () PQRS가마름모. 30. 한평면위에있지않은네점이주어져있다. 이네점을한평면에사영시킨점들이평행사변형을이루는, 그런평면이항상존재하는가? 이런평면은일반적으로몇개가있겠는가? 단, 평행한것은하나로본다. ( 이탈리아 988-5)

266 66 기하 풀이사영했을때대각선의중점이일치해야하므로사영전의두선분의중점을이은직선이평면과수직이되어야함. 즉, 네점을A, B, C, D라했을때,(AB; CD), (AC; BD), (AD; BC) 로짝을지어중점을연결한직선의방향에따라평면이하나씩결정됨. 그런데, 주의가요구되는것이두가지가있음. () 이런중점연결직선에수직인평면에사영시켰을때, 평행사변형이나오지않고퇴화된평행사변형이나온다면? 그건사영된네점이한직선위에있다는것이므로, 이직선을지나고사영평면에수직인평면에원래의네점이모두있어이경우는조건에맞지않으므로 OK. () 이런세짝에의해각각얻어진평면들중에평행한것이있다면? 그럼그법선도평행하므로,WLOGAB, CD, AC, BD의중점을각각M, N, K, L이라할때 MN과 KL이평행하다고해보자. 벡터또는좌표적으로 MN의방향은 (A+B) (C +D) 이고 KL의방향은 (A+C) (B+D). 평행하다면 A+B C D = t(a+c B D). 즉,( t)(a D) =(+t)(c B). 즉, AD와 BC가평행하다는뜻이므로역시조건에맞지않음. } 3. 삼각형 ABC 의내심을 I 라하고, 삼각형 IBC, ICA, IAB 의외심을각각 A 0, B 0, C 0 이라하자. 삼각형 ABC 와 A 0 B 0 C 0 의외심이일치함을증명하여라. ( 미국 988-4) 3. 예각삼각형 ABC의내부또는경계위의각각의점P에대하여, P 에서변 BC, CA, AB로내린수선의발을각각 P a, P b, P c 라하자. f(p )= AP c + BP a + CP b PP a + PP b + PP c 로정의하면, f(p ) 가일정할때, 또그때만, 4ABC 가정삼각형임을보여라. ( 오폴 989-8) 증명 f(p ) 가일정하면 f(i) = s 3r, f(o) = s = s d +d +d 3 R+r 임에서 3r = R + r, 즉 R =r. 이경우는정삼각형. 또, 정삼각형이라면 f(p ) 의분모는일정한값이므로 ( 넓이를생각하면됨 ), 분자도일정함을보이면되는데, P 를변에수직인방향으로이동시킬때항상분자가불변임을금방확인할수있어서이런이동으로모든점에다도달할수있으므로 ok. 33. 삼각형 ABC의변BC, CA, AB 위에각각점 X, Y, Z를잡는데, X, Y, Z에서의각이각각 A, B, C에서의각과같아삼각형 ABC와 XY Z가닮도록한다. 삼각형 XY Z의넓이가최소가되는 X, Y, Z를찾아라. ( 호주 989-8) 증명 ( 신한솔 ) 삼각형 AZY; XZY 에서ZY 가공통이고 ZY 와마주보는각의크기가같으므로 AZY; XZY 의외접원의반지름은같다. YZ가최소일때넓이가최소이므로 ZY 가최소일때를구하면된다. 삼각형 AY Z; BZX의외접원의교점을 P 라하면각YPX = 각A + 각B 이므로 CY PX는내접사각형이다.(p가외부에있을때에도비슷하게됨 ) 따라서 P 는각삼각형의외접원의교점이다. 같은크기의원에서의동일현에대한원주각으로두각이같으니까 PAB는이등변삼각형이다. 즉, PA = PB, 마찬가지로 PB = PC 이므로 P 는삼각형 ABC외심이다. P 가외심이고 AZP Y 가내접사각형이므로 PZY는언제나닮음이므로 PY가최소일때즉 Y 가 P 에서 AC에내린수선의발일때ZY 가최소이므로 Y 는 AC의중점이고대칭적으로 X; Z도각변의중점이여야하고이때 XY Z와 ABC는닮음이므로 X; Y; Z는각변의중점이다. 넓이가최소일려면닮음비가최소여야하므로 z가최소일때넓이가최소이다. 34. 반지름 R 의원내부에점 A 가있다. A 를지나는서로직교하는두직선을그리자. 그다음, 이두직선을 A 를중심으로각각 V 의각으로회전시키자. 이로부터원의내부에생긴그림 회전하는동안직선이지나간영역 은 A 를중심으로하는십자가꼴로생겼다. 이영역의넓이를구하여라. (Towns 989 가을 SA3) 35. 삼각형 ABC의각B와각C의이등분선이각각대변과만나는점을 P 와 Q라하자. 직선 PQ는삼각형 ABC의내접원과만남을보여라. ( 통신강좌 990-E6) 풀이 ( 제주오현고고봉균 ) 내심을 O 라하고, \OBC =, \OCB =, \OPQ = 0, \OQP = 0 이라하자.80 ± > \B +\C = ( + ) 이고 + = \COP = \ 이므로 = + <90 ± ()

267 3. 기하고급문제 67 그리고 \OPC =80 ± ( + ) =80 ± \OQB =80 ± ( + ) =80 ± 선분 PQ 가내접원 O 와만나지않는다고가정하자. 그럼 \OPQ > \OPC, \OQP > \OQB 이므로 0 > 80 ± ; 0 > 80 ± 두식을합하면 > 360 ± 3( + ), 즉 () 에의해 + >90 ± 로모순이다. 따라서귀류법의가정이틀렸고 PQ 는내접원과만난다. } 36. 삼각형 ABC 에서두점 A 와 C 를지나는원 k 이변 AB, BC 와각각서로다른점 K, N 에서새로만난다. k 의중심을 O 라하자. 삼각형 KBN 의외접원을 k 라하고, 삼각형 ABC 의외접원이 B 가아닌점 M 에서 k 와또만난다. OM 과 MB 가수직임을증명하여라. ( 호주 990-3) 증명 (BC > AB > AC 꼴의그림일때 ) MA\ k =: X 라하고 Y 를! KM 의연장선위의점이라하자. \BMC = \BAC = \KNB = \BMY =: a 라하자. 그럼,b := \KMC =(80 ± \KNB) \BMC = 80 ± a =80 ± \KOC. 즉, KMOC cyclic. KO = OC 이므로동일현에대한원주각으로 \KMO = \CMO = b. 즉, \BMO = a + b =90 ±. 37. 삼각형 ABC 의방접원을 P, Q, O 라고할때각꼭지점에서그대변과방접원들의접점을잇는선분들은한점에서만남을보여라. ( 통신강좌 99--) 풀이 G K P A Q F E H B X D C L I J O 이문제는 Ceva 의정리를사용하면굉장히쉽게풀린다. AD; BE 와 CF 가한점에서만날조건은다음과같다. FB AF CD BD AE = () CE

268 68 기하 그런데여기서두원의내접선을생각해보자. 두원의반지름을각각 r ;r 라고하면, A O₁ R₁ B r₂ O₂ C 삼각형 O AB와 O BC는닮음꼴이므로 AB : BC = r : r 이다. 이것을원문제에적용하자. 원 P; Q; O의반지름을각각 r p;r q;r o 라하면, AF = GA이므로 AF : AE = GA : AE = r p : r q ; AE AF = rq r p () FB = BH 이므로 CD = CJ 이므로 BF : BD = BH : BD = r p : r o; CD : CE = CJ : CE = r o : r q ; BF BD = r p r o (3) CD CE = ro r q (4) 를얻을수있다. 여기서 (), (3), (4) 를모두곱하면 r q r p r o = AE r p r o r q AF BF BD CD CE = FB AF CD BD AE CE (5) 를얻을수있고또 (5) 는 () 과같으므로세선분들은한점에서만난다. } 38. 직각삼각형 ABC 에서 P 와 Q 는각각 BC 와 AC 위의점이며 CP = CQ = a 이다. AP 와 BQ 의교점을 R 이라하고직선 RC 와 AB 의교점을 S 라하고직선 PQ 와 AB 가만나는점을 T 라고한다. AC = b, AB = c 일때 TS 의길이는? ( 통신강좌 ) 풀이 ( 서울과학고 3년박종원 ) CP = CQ = a, AC = b, AB = c, BC = `, BS = x, BT = y라하자. 체바의정리에의해 PC QA SB = PB QC SA =) a (b a) = (` a) a (c x) =) (b a) = (` a)(c x); 정리하면 b + ` a 6= 0 (* b>a; `>a) 에서 x = C (` a) c b + ` a. Q P R A S B T

269 3. 기하고급문제 69 메네라우스정리에의해 정리하면 b 6= ` (* Ã! PQ 와 Ã! AB 가만남 ) 에서 y = PB TA QC = PC QA TB =) (` a)(c + y) a = a (b a) y =) (` a)(c + y) =(b a)y; (` a) c. b ` µ ST = x + y =(` a) c b + ` a + b ` (` a) = (` a) c (b `)(b + ` a) = c(b a)(` a) (b `)(b + ` a) ; 이과정은 A, B, C 중어느각이 90 ± 이더라도상관없이성립한다. ) TS = c (b a)(` a) (b `)(b + ` a) 8 >< C =90 ± 일때 ` = p c b 단, B =90 >: ± 일때 ` = p b c A =90 ± 일때 ` = p b + c } 39. 주어진삼각형 ABC 의변 AB 위에서잡은임의의점을 D 라하고, 삼각형 ACD 와 BCD 의내접원들의 (AB 가아닌 ) 공통외접선과 CD 가만나는교점을 E 라하자. D 가변 AB 위를움직임에따른점 E 의자취는한원호임을증명하여라. ( 미국 99-5) 증명접선의길이를이리저리옮기는계산을해보면그냥길이계산만으로 CE = (AC +BC AB) 임을얻어낼수있음. 40. 원 S 과 S 가두점A, A 4 에서만나고, 원 S, S 3 가두점A, A 5 에서만나고, 원 S 3, S 이두점A 3, A 6 에서만난다. 선분 M k M k+ 가 A k 를지나고점 M k, M k+ 이 A k 에서만나는원들위에놓여있도록꺾은선 M M M 7 이그려져있다. M 과 M 7 이일치함을보여라. ( 러시아 99 4차-y-6) 증명 M 8 까지한번더보낸다고생각하자. 그럼직선 M M 가계속한방향으로회전하게되는데, M 7 M 8 에이를때까지다음의각만큼씩회전하게됨 : A 3 A 4, A 6 A 의 S 에서의원주각, A A, A 4 A 5 의 S 에서의원주각, A A 3, A 5 A 6 의 S 3 에서의원주각. 즉, 이들의합이 80 ± 임을보이면될듯. 그것은원주각을교점 A i 들중안쪽에있는세점으로된삼각형의내각으로쪼개어옮기면확인됨. 4. 볼록사각형 ABCD와같은평면위에네삼각형ABP, BCP, CDP, DAP가같은넓이가되는점P가존재한다. 이런 ABCD는어떤사각형인가? 또, 그런사각형이하나주어졌을때이런점 P 는최대몇개인가? ( 중국 99-) 풀이 AC, BD의중점을각각 M, N 이라하자. () P 가사각형의내부에있을때 : j4abp j = j4adp j 이므로 AP 는 BD를이등분, 즉 P 는 AN 위의점. 마찬가지로, P 는 CN 위의점. 즉, ANC가일직선이거나아니면 P = N. 블라블라생각해보면두대각선 AC와 BD 둘중하나가다른것을이등분해야하고 P 는이등분하는대각선의중점임 ( 유일함 ). () P 가사각형의외부에있을때 : 꼭지점뒷편영역에있을때는역시P가AN 위의점이라는식으로되어곤란해지기때문에 P 는변바깥영역의점. AD의바깥쪽에있다고하면등적변형에의해 AP 는 BD와평행해야하고, DP도 AC와평행해야함. 그럼 AP DX는평행사변형 (X는두대각선의교점). 또, j4pabj + j4pbcj + j4pcdj j4pdaj = j ABCDj 임에서 j4pdaj = j ABCDj. 즉, 4PDA 4XAD 임에서 j4xadj = j ABCDj. 즉, 대각선들로분할된네영역중어느하나가전체넓이의 일때라야하고, 그때그영역바깥쪽에평행사변형이되는점이 P 임 ( 이때도유일함 ). }

270 70 기하 4. 삼각형 ABC 의중선 AD 위에한점 E 를잡고, E 에서 BC 에내린수선의발을 F 라한다. EF 위의한점 M 에서 AC, AB 에내린수선의발을각각 N, P 라하고 E, N, P 가한직선위에있다면 M 은 \BAC 의이등분선위에있음을보여라. ( 통신강좌 ) 풀이 ( 공주사대부고 년장근호 ) A P E N M B F D C! AB =! b;! AC =! AE c;! EF =! f 라하자. 이때! AD = (! b +! c ), EM EF = t; AN AC = n; AD = e; AP AB = p;! PE = AP! AB + AE! AD = p! b + e AB AD (! b +! c );! AE! EN = AD + AN! AC = e AD AC (! b +! c )+n! c 이다. E, N, P 가일직선상에있으므로! PE = k EN! 이라하면 ³ e p e!b e ³ +! c = k e ³!b + k n e!c ; k= p e = 가된다. 여기서 e = e +(n + p)e 4np. 즉 e = np n + p 이고,! PE = p! b + np n + p (! b +! c )= p n + p (p! b n! c );! np EN = n + p (! b +! c )+n! c = n n + p (p! b n! c ); e n e 이다. 또! p AB?MP에서 b n + p (p! b n! c )+t! f =0; (6) AC?MN에서! n c n + p (p! b n! c ) t! f =0; (7)!! BC?EF에서 f ( b! c )=0 (8) 이므로, ()+() 하면 n + p (p! b + n! c ) (p! b n! c )+t! f (! b! c )=0, 다시 (3) 에의해 (p! b + n! c ) (p! b n! c )=0, 즉 pj! b j = nj! c j이다. 따라서 pj! b j = AP, nj! c j = AN이므로 AP = AN이된다. 4AP M과 4ACM에서 \AP M = \ACM = \R; AP = AN; AM은공통으로 RHS-합동이된다. 따라서 \BAM = \CAM이다. }

271 3. 기하고급문제 삼각형 ABC 의변 BC, CA, AB 위에한점씩잡아각각 M, N, P 라하자. 직선 AM, BN, CP 와삼각형 ABC 의외접원과의교점을각각 Q, R, S 라할때 AM MQ + BN NR + CP PS 9 임을보여라. ( 통신강좌 ) 풀이 BC와평행하고 A의맞은편에서외접원에접하는직선을 `이라하고, 그접점을Q 0, AQ 0 과 BC의교점을 M 0 이라하자. 직선 AQ와 `의교점을Q 00 이라하면, 삼각형 AMM 0 과 AQ 00 Q 0 은닮은꼴이므로 AM MQ AM MQ 00 = AM 0 AM M 0 이성립한다. 따라서 Q0 MQ 이최소이려면 M = M 0 ;Q = Q 0 이어야함을알수있다. 한편, Q 0 은호BC의중점이므로 BQ 0 = CQ 0 이고같은현에대한원주각으로서 \BAQ 0 = \CAQ 0, 즉 AQ 0 은각A를이등분한다. 이제 AM, BN, CP를각각각A, B, C의이등분선이라하자. BC, CA, AB의길이를각각 a, b, c라하고, s = (a + b + c) 라하자. 삼각형 MBQ와 MAC의닮음에서 BQ AM = b BM이며, 삼각형 MAB와 MCQ의닮음에서 c MQ = CQ BM이므로 AM MQ = bc BQ CQ = bc BQ 이다. 또 a =BQ cos A 이고 cos A r s(s a) = ( 통신강좌제5권제4호7쪽참조 ) 이므로다시정리 bc 하면 이된다. 같이하여 AM MQ = 4bc a cos A 4s(s a) = a = (b + c) a a = b + c a + bc a AM MQ + BN NR + CP PS = b + c a + c + a b + a + b c + bc a + ca b + ab c 3 을얻게되는데 µ b a + a b bc a + ca b + ab µ bc ca ab c 3 a b c b + c a + c + a b + a + b c = µ c + =3 =3 b + b c µ a + c + c a ++=6; 의부등식을이용하면 AM MQ + BN NR + CP PS =9 가증명된다. 등호는 AM, BN, CP 가각각각 A, B, C 의이등분선이고삼각형 ABC 가정삼각형일때성립한다. } 44. \C =90 ± 인직각삼각형 ABC에대하여반직선! AC 상에 \CPB = \CBA 가되도록점 P 를잡고, 반직선! CB 상에 \CQB = \CAB 가되도록점 Q를잡는다. 또한, 반직선! AC,! BC 상에각각점 R, S를 \ABR = \BAS =90 ± CP CQ 가되도록잡는다. 가최대가되는직각삼각형 ABC의모양을결정하 AR BS 고그값을구하여라. ( 한국 99-5)

272 7 기하 풀이 BC = a, CA = b, AB = c, CP = p, CQ = q, AR = r, BS = s 라하자. 위그림에서 이므로 따라서, \CPB = \BRA = \ABS = \CAQ = \BAC 4CPB _ 4CAQ _ 4BRA _ 4ABS _ 4CBA CB CP = CA CB 에서 a p = b a ) p = a b CQ CA = CA CB 에서 q b = b a ) q = b a AR AB = AB AC 에서 r c = c b ) r = c b BS AB = BA CB 에서 s c = c a ) s = c a CP CQ AR BS = pq rs = p r q s 이다. p r + q s = a c + b c = c c = 이므로 p r q s = p r 에서 p r = 일때최대값 4 을갖는다. p r = 일때 q s = 때최대가된다. ) 4ABC 가직각이등변삼각형일때최대이고최대값 4 ³ p = µ p r 4 r 이므로 a c = b c =. 즉 a = b = c p 일 } 45. 중심이 O 이고반지름이 r 인원 C 안에중심이각각 O, O 이고반지름이 r, r 인두개의원 C, C 를원 C 와각각점 A, A 에서내접하고, 원 C 과 C 는점 A 에서서로외접하도록그린다. 직선 OA, O A, O A 은한점에서만남을증명하여라. (APMO 99-) 풀이 (O±cial) O A = O A = r ;O A = O A = r 이다. AB jjo A 되게 O A 위에 B 를잡고, AB jjo A 되게 O A 위에 B 을잡는다. AB =AO = O A =O A 이므로 AB = r r =(r + r ). 같이하여 AB = r r =(r + r ) 이므로 AB = AB = r r =(r + r ). OA의연장이 A O ;A O 와만나는점을각각 M ;M 라하면OM =AM = OA =AB = r(r + r )=r r ;OM =AM = OA =AB = r(r +r )=r r 이므로 OM =AM = OM =AM 즉 M = M 를얻는다. 따라서세직선OA; O A ;O A 은한점에서만난다. }

273 3. 기하고급문제 73 풀이 (O±cial) 4OO O 에서 OA A O O A AO O A A O = r r r r r r = 이므로체바의정리의역정리에의하여세직선 OA; O A ;O A 은한점에서만난다. } 46. 사각형 ABCD에서, 직선 AB와 CD가 P 에서, AD와 BC는 Q에서만난다. AC와 BD는 PQ와 X, Y 에서각각만난다. PX XQ = PY 임을증명하여라. ( 통신강좌 ) YQ 풀이 4PQC 에서 Ceva 의정리에의해 PX XQ QB BC CD DP = 4PQC 와직선 YBD 에서 Menelaus 의정리에의해 PY YQ QB BC CD DP = ) PX XQ = PY YQ } 47. 예각삼각형 ABC 안에점 P 가있다. P 에서각변에내린세수선의길이의합이최소가되는점 P 의위치를구하여라. ( 통신강좌 ) 풀이 P 에서 BC; CA; AB 에내린수선의길이를각각 x; y; z 라하자. 삼각형의넓이를 S 라하고, 일반성을잃지않고 a b c 라할수있다. S = ax + by + cz = a(x + y + z) f(a b)y +(a c)zg a(x + y + z) (* a b 0; a c 0; y;z 0) ) x + y + z S a i) a>b a b>0, a c>0이므로, y = z =0일때그리고이때에만위식의등호가성립하므로이때최소값을갖는다. 이런경우는점 P 가 A에있을때이다. ii) a b>c a b =0,a c>0이므로, z =0일때그리고이때에만위식의등호가성립하고그경우는점 P 가 AB위에있을때이다. iii) a = b = c 위식에서 P 가어디있든등호가성립한다. 즉 P 가임의의점이어도최소값 S a 를갖는다. 결론 ) 4ABC에서가장긴변이 개있을때 : 그변에대응하는꼭지점에 P 가있을때. 개있을때 : 그나머지변위에P가있을때. 3개있을때 : 임의의점이라도상관없다. } 48. C 0 ;C ;C ;::: 들을다음과같이정의된평면상의원들의수열이라하자. (i) C 0 은원x + y =이다. (ii) n =0; ; ;::: 에대하여원 C n+ 은 y 0 인평면위에있고, 원 C n 과외접하고쌍곡선 x y = 과두점에서접한다. r n 을원C n 의반지름이라할때, r n 은정수임을보이고, r n 을구하여라. ( 한국 993-0)

274 74 기하 풀이 y 0인평면에서원 C 0 과쌍곡선은 y축에대칭이므로원 C n 의중심은y축위에있다. 원 C n 의중심을 (0; a n ) 라하면C n 의방정식은다음과같다. 원 C n 과 C n 이외접하므로 원 C n 과쌍곡선 x y = 이접하므로 () 에의하여 x +(y a n) = r n (n =0; ; ; ) () a n a n = r n + r n () y ++(y a n) = r n 이등근을갖는다.y a n y + a n r n +=0 에서판별식 4 = a n (a n r n +)=0 (3) 에서 a n =r n 이므로 a n a n =(r n r n ) ) (a n a n )(a n + a n )=(r n + r n )(r n r n ) () 를대입하면 (), (4) 에서 ) a n =r n (3) a n + a n =(r n r n ) (4) a n =3r n r n (5) (5) 에서 a n = r n 3r n (6) (6) 에의하여 a n =3r n r n r n =6r n r n (n =; 3; ) (7) 을얻는다. 한편, a 0 =0; r 0 =이므로 () 에서 n =일때a = r + (5) 에서 n =일때a =3r 두식에서a 을소거하면 r =3을얻는다. r 0 =;r =3이므로 (7) 에의하여 r =7;r 3 =99; 와같이귀납적정의에의하여모든 r n 은정수이다. 다음에는 r n 을구한다. r n r n = (r n r n ) 라하면 (8) 과비교하여 r n =6r n r n (n =; 3; ) (8) + =6; = ; 는 차방정식 t 6t +=0 의두근 3 p 이다. fr n r n g 은초항 r r 3, 공비 인등비수열이므로 r n r n =(r r 0 ) n (9)

275 3. 기하고급문제 75 =3+ p ; =3 p 라하고, ; 는서로바꿔도되므로 r n r n =(r r 0 ) n (0) (0) (9) 라하면 ( )r n = (r r 0 ) n (r r 0 ) n ) r n = r r 0 n r r 0 n =4 p r r 0 =3 (3 p ) = p r r 0 =3 (3 + p ) = p 답 r n = f(3 + p ) n +(3 p ) n g 참고 : 점화공식 r n 6r n + r n =0에서 t 6t +=0을특성방정식이라한다. 두근을 ; 라하면일반적으로 r n = c n + c n 이성립한다. 여기서 r 0 = c + c r = c + c 인조건에의하여 c = r r 0 ; c = r 0 r 을얻고 r n = r r 0 n + r o r n 이구하는해이다. 일반적으로 r n + ar n + br n =0일때t + at + b =0가특성방정식이다. 두개의서로다른실근을가질때위와같이하면된다. = ( 등근 ), 또는 ; 가허근일때는다른조사가필요하다. } 49. 직선 l 이원 S 와점 A 에서접한다. B 와 C 는점 A 에대해서로반대쪽에있는직선 l 위의점들이다. B 와 C 에서 S 에그은또다른두접선이점 P 에서만난다. jabj jacj 의값이일정하게유지되도록하면서 B 와 C 를직선 l 을따라움직일때, 점 P 의자취를구하여라. ( 아일랜드 993-3) 풀이 l에이르는거리 (h) 가일정함. 즉, l과평행한직선이됨. 증명은아래와같이하면됨. xy와 r이일정하게주어진상태. S = r(x + y + z) = x+y+z z (x + y)h 이므로 x+y 가일정함을보이면, 즉 x+y 가일정함을보이면됨. 헤론의공식에서 r (x + y + z) =xyz. 이로부터 x+y+z z 가일정함을, 즉 x+y z 가일정함을알수있으므로끝. 참고로, 직접 h를식으로구해보면 h = rxy xy r. } 50. 임의의복소수 z = x + iy 에대응되는좌표 (x; y) 를 P (z) 로나타내기로하자. z, z, z 3, z 4, z 5, 가다음을만족하는 0 아닌복소수들이라고한다. (i) P (z ), P (z ), P (z 3 ), P (z 4 ), P (z 5 ) 는원점 O 를내부에포함하는어떤볼록오각형 Q 의꼭지점들이다. (ii) P ( z ), P ( z ), P ( z 3 ), P ( z 4 ), P ( z 5 ) 는모두 Q 의내부에있다. P ( ) =(p; q) 라할때, p + q 와 p + q tan ¼ 5 가성립함을보여라. ( 아일랜드 993-5) 증명 ( 장도한 )(a)p i Q( z i );Q i P (z i )(i =; ; 3; 4; 5) 이라두면 Q 가볼록 (convex) 이므로, Q 의내부의두점 P i ;P j (i 6= j) 을잇는선분 P i P j 는 Q 의내부에있음을알수있다. 그러므로 P i (i = ; ; 3; 4; 5) 를꼭지점으로하는오각형 P 라할때 P 는 Q 의내부에있게된다. 즉, P 의면적은 Q 의면적보다작다. 또한, OP i = jj jj OQ i 이고 arg z i =argz i +arg 이므로 P 는 Q 를 arg 만큼회전하여 P 와 Q 의닮음비가 jj jj : 인 Q 와닮은도형임을알수있다.

276 76 기하 만일, jj jj = p + q > 이라가정하면 area(p )=jj jj area(q) > area(q) 즉, P 의면적이 Q 의면적보다크게되어모순이다. 따라서 p + q 이다. (b) Q의내각의합이 540 ± 이므로 0개의 \Q i Q j O(Q i 와 Q j 는도형Q의인접한서로다른두꼭지점이다.) 들중54 ± 이하인각이존재한다. 이제일반성을잃지않고 \Q Q O 54 ± 이라하자. 먼저, p + q 이므로 jpj; jqj 이다. 또한,0< tan ¼ 5 < 이다. ) p 0이면 p + q tan ¼ 5 q tan ¼ 5 jqjtan ¼ 5 < jqj 이고, q 0이면 p + q tan ¼ p jpj 5 이므로 p>0이고, q>0일때, 부등식 p + q tan ¼ 5 이성립함을보이면충분하다. 이제, p>0;q >0이라하면, P 이직선OQ 에대하여Q 쪽에있게되고또한, P 에서직선 OQ 에내린수선의발을 H 이라하면 H 은선분OQ 상의점이된다. 그리고 \P Q O \Q Q O 54 ± 이므로, \Q P H 36 ± (= ¼ (rad)) 5 이다. )\H P I = ¼ 5 이되도록H Q 상에점 I 을잡을수있다. 즉, joi j joq j이다. 한편, \P OQ =arg ( µ) 이고, jop j = jj z jj이므로, joh j = jj z jj cos µ; jp H j = jj z jjj sin µ 이고 jh I j = jp H j tan ¼ 5 = jj z jj sin µ tan ¼ 5 이다. ) joi j = joh j + jh I j = jj z jj cos µ + jj z jj sin µ tan ¼ 5 = jjz jj(jj jj cos µ)+(jj jj sin µ)tan ¼ 5 )=jjz jj(p + q tan ¼ 5 ) = joq j(p + q tan ¼ 5 ) ) p + q tan ¼ 5 = joi j joq j 5. 삼각형 ABC의두변AB, AC로내린중선이서로직교한다.cotB +cotc 3 임을증명하여라. ( 캐나다 993-3) 증명그림과같이점과각의크기, 길이등에이름을붙이자.

277 3. 기하고급문제 77 G 는무게중심이므로 BG =x, CG =y 가된다. tangent 합차공식에의해 cot B =cot(a + b) = cot C =cot(c + d) = tan a +tanb = y x y x y x + y x tan c +tand = x y x y x y + x y tan a tan b = tan(a + b) tan c tan d = tan(c + d) = 4x y 6xy = 4y x 6xy 따라서, 이것을합하면 cot B +cotc = x +y 6xy = x + y 3xy xy 3xy 3 로문제가증명된다. 5. 삼각형 ABC 의변 BC, CA, AB 위에각각점 D, E, F 가있다. 또한삼각형 AEF, BFD, CDE 의내접원의반지름은모두 r 이다. 삼각형 DEF 의내접원의반지름을 r, ABC 의내접원의반지름을 r 이라할때, r + r = r 임을보여라. ( 폴란드 993 차 -5) 증명 DEF 의내심은ABC의내심과일치. 작은삼각형들의내심에서꼭지점, 접점을이어만든직각삼각형들을접어보면두삼각형 DEF와 O O O 3 는넓이및둘레의길이가같음. 그럼내접원의반지름도같음. 53. ABCD 는 BC 와 AD 가평행인사다리꼴이다. M 은 CD 의중점, P 는 MA 의중점, Q 는 MB 의중점이다. 직선 DP 와 CQ 가만나는점을 N 이라할때, N 은 ABM 의외부에있지않음을보여라. ( 통신강좌 ) 증명 L을 AB의중점이라고하자. AD = BC인경우N = L이므로 N은 ABM의경계에있다. 그러므로, AD > BC라고가정해도된다. 평행사변형 MDAX와 MCBY 를만들자. 그러면 Y 는 L과 M사이에있고 L은 X와 M사이에놓여있다. 그런데, N은 DX와 CY 위에있다. DX와 CY 가 AB와각각H; K에서만난다고하자. 그러면, 4HAD s 4HLX이므로 HL XL = HA AD 이다. 비슷한방법으로 KL LY = KB BC 이다. 결국 HL LX = HA AD < LA AD < KB AD < KB BC = KL 이므로 N은 4ABM의내부에있다. LY 54. P 를 4ABC의구점원의중심, AP, BP, CP가각각BC, CA, AB와만나는점을 D, E, F 라했을때, BD DC, CE EA, AF 를삼각형 ABC의세변의길이로표시하여라. ( 통신강좌 ) FB

278 78 기하 풀이난다. O 를 4ABC 의외접원의중심, 반직선 AO; BO; CO 는변 BC; CA; AB 와각각 G; H; I 에서만 \BAG =90 ± \C; \GAC =90 ± \B이므로 BG GC = c sin(90± \C) b sin(90 ± \B) = c cos C 이다. 코사인제 b cos B 법칙 c = a + b ab cos C; b = a + c ac cos B를사용하면 BG GC = L K () 비슷하게 CH HA = J L () AI IB = K J (3) 을얻는다. 단, J = a (b + c a );K = b (a + c b );L = c (a + b c ) 이다. 그림에서 BG a BG = BG GC = L K 이므로 BG = al K + L 이다. 4AGC와직선BOH에메넬라우스정리를적용하면 CH HA AO OG GB BC BC BG 를얻는다. () 와 (4) 에의해 AO OG = K + L J (4) = 또는 AO OG = AH HC (5) 이다. 4A 0 B 0 C 0 을 4ABC의중점삼각형이라하면구점원의중심 P 는 4A 0 B 0 C 0 의외접원의중심과일치하므로점 P 와점O는4A 0 B 0 C 0 과 4ABC에서비슷한위치에있다 (homologous points). A 0 P 를 B 0 C 0 와 G 0 에서만나게연장하면 C0 G 0 G 0 B 0 과 A0 P PG 0 는 CG GB ; AO OG 와각각같아야한다. AG 0 를 BC와 B에서만나게연장한다면 BM MC = C0 G 0 G 0 B 0 = CG GB = K L 이다. BM a BM = BM MC = K L 에서 BM = ak K + L (6) 이다. MA 0 = BA 0 BM이고 BA 0 대신 a 로바꾸어 (6) 을사용하면 MA 0 = a(l K) (K + L) (7)

279 3. 기하고급문제 79 를얻는다. 4MA 0 G 0 과직선 AP D에메넬라우스정리를적용하면 MD DA 0 A0 P PG 0 G0 A AM MD DA 0 = G0 P PA 0 AM AG 0 이다. (5) 에서 PA0 G 0 P = AO OG = K + L 이고 AH MD =이므로 J AG DA 0 = J K + L 이고 J 와 (7) 에서 K + L MD = aj(l K) (K + L)(J + K + L) 이다. 이제 BD = BM + MD 이고 (6) 과 (8) 를사용하면 = 또는 MD MA 0 MD = MD DA 0 = (8) BD = a(j + K) J + K + L (9) 이다. DC = a BD 이므로 (9) 를사용하면 DC = a(j + L) J + K + L 이다.(9) 과 (0) 에서 BD DC = J + K J + L = a (b + c a )+b (a + c b ) a (b + c a )+c (a + b c ) 이다. 비슷하게 CE EA = b (a + c b )+c (a + b c ) b (a + c b )+a (b + c a ) ; AF FB = c (a + b c )+a (b + c a ) c (a + b c )+b (a + c b ) 를얻는다. (0) } 55. A, B, C, D는평면위의네점으로, C, D는직선AB를기준으로같은쪽에있고, AC BD = AD BC 와 \ADB =90 ± AB CD + \ACB 를만족한다. 를구하고, 두원ACD와 BCD는서로수직임을보 AC BD 여라. ( 통신강좌 ) 증명 DE를 DB에수직이며같은길이로그린다. 그러면 \ADE = \ACB이다. 또, AD AC = BD BC = DE 이다. 그러므로 4ADE와 4ACB는닮음꼴이다. 즉, \CAB = \DAE이고 AB=AE = AC=AD이 BC 다. 또, \CAD = \CAB \DAB = \DAE \DAB = \BAE이다. 따라서 4CAD와 4BAE는닮음꼴이다. 그러므로 AC AB = CD BE = p CD 이다. BD (* 4BDE가직각이등변삼각형이므로 ) AB CD 즉, AC BD = p 이다.

280 80 기하 CT;CU를각각원 ACD; BCD의 C를지나는접선들이라고하자. 그러면 \DCT \DCU = \DBC이다. 즉, \ADE + \DAB + \DBA =80 ± 90 ± =90 ± 이다. 그리고닮은꼴삼각형으로부터 = \DAC 이고 \ACB + \CAB \CAD + \ABC \DBC =90 ± 이므로 \CAD + \DBC =80 ± 90 ± =90 ±. 즉, \TCU =90 ± 이다 ABC 에서 4A 0 B 0 C 0 을중점삼각형, I 0 을 4A 0 B 0 C 0 의내심, AI 0, BI 0, CI 0 과 BC, CA, AB 가만나는점을각각 D, E, F 라했을때 AF=F B, BD=DC, CE=EA 를구하여라. ( 통신강좌 ) 풀이 c>b를가정하자. 4A 0 B 0 C 0 와 4ABC의닮음비를 :이므로꼭지점 A 0 로부터 4A 0 B 0 C 0 의내심I 0 까지의거리는 AI이다. A; B; C로부터의각이등분선이대변과각각 M; N; Q에서만난다고하면 AQ=QB = b=a; AN=NC = c=a이므로체바의정리의따름정리에의해 AI=IM = AQ=QB + AN=NC = b=a + c=a =(b + c)=a 이고 AI=AM = AI=(AI + IM)= (b + c)=a IM ((b + c)=a +) IM = b + c a + b + c 이다. 사각형 AB 0 A 0 C 0 은평행사변형이므로 I 0 A 0 과 AM 은평행이다. 그러면 DA 0 =DM = I 0 A 0 =AM = (AI=AM) =(b + c)=(a + b + c) 이고 DA0 =A 0 M = DA 0 =(DM DA 0 )= µ b + c (a + b + c) DM b + c (a + b + c) DM = b + c a + b + c 이다. c=b = BM=MC =(BA 0 + A 0 M)=(A 0 C A 0 M)=(a= +A 0 M)=(a= A 0 M) 에서 A 0 M = a(c b)=(c + b) 이다. 그러므로 DA 0 b + c a(c b) BD = a + b + c A0 M = 이고 (a + b + c) DC = BA0 PA 0 A 0 C + DA 0 = a= a(c b)=(a + b + c) a=+a(c b)=(a + b + c) = a + b a + c 이다. 대칭을생각하면 AF=F B =(c + a)=(c + b);cf=fa=(b + c)=(b + a) 이다. } 57. 4ABC의중선AD가내접원과두점 X, Y 에서만난다. AC = AB + AD 일때 \XIY 의크기를구하여라. 단, I는내심이다. (Towns 994가을 SA3) 풀이 AC 위에 AE = AB 인점E를잡자. 4ABD의넓이가 배되어4ABC가되면서, 4IAB! ABIE 로, 4IBD!4IBC 로 배되므로남은건 4IAD!4IEC 이므로 e = r. (e 는 I에서 AD까지의거리 ) }

281 3. 기하고급문제 삼각형 ABC 에서 K, L, M, N, R, F 를각각 BC, AC, AB, LM, MK, KL 위의점이라고하자. 또한 E, E, E 3, E 4, E 5, E 6, E 는각각삼각형 AMR, CKR, BKF, ALF, BNM, CLN, ABC 의넓이이다. 다음을보여라 ( 통신강좌 ) 증명 ( 언주중공유식 ) AM AB 라하자. = a, BK BC E 8(E E E 3 E 4 E 5 E 6 ) 6 CL MN = b, = c, CA ML = p, KR LF = q, KM Lk = r Α Μ Ν Λ Β Ρ Κ Φ Χ E = 4AMR = MR MK 4AMK AM AB 4ABK AM AB = ( q)abe = MR MK = MR MK BK BC 4ABC 마찬가지로 E = ( a)( b)qe E 3 = c b ( r)e E 4 = ( c)( b)re E 5 = c a ( p)e E 6 = ( c)( a)pe 8(E E E 6 ) 6 = 8f( a) a ( b) b ( c) c ( p) p( q)q( r)rg 6 E µ E(* AM GM) = E 등호조건은산술기하평균부등식에의하여 a = b = c = p = q = r = 즉, M, R, K, F, L, N 이각선분의중점일때이다. 59. 삼각형 ABC 에서 AB 의중점을 D, 내접원과 AB 가만나는점을 E, CE 의중점을 F, 내심을 I 라하면 D, F, I 가한직선위에있음을보여라. ( 통신강좌 )

282 8 기하 증명 ( 대전대덕중임재한 ) 삼각형 ABC 에서 AB, BC, CA 를각각 c, a, b 라하자. DE = AD AE = c a + b + c DK = AD AK = c c IC IK = AC AK = FE FC = ) b b c a + b DK DE IC IK FE FC = = a b b c(a b) = a + b (a + b) = a + b (* AI는 \CAK의이등분선) c c(a b) (a + b) a + b = a b c 따라서메네라우스의정리에의해 D, I, F 는일직선상에존재한다. 60. 삼각형 ABC 에서 \C =30 ± 이고, D 는 AC 위의점, E 는 BC 위의점으로 AD = BE = AB 를만족한다고한다. OI = DE, OI? DE 임을보여라. 단, O, I 는각각삼각형 ABC 의외심과내심이다. ( 통신강좌 ) 증명 ( 경북과학고류홍규 ) 삼각형 AB, BC, CA의길이를각각 c, a, b라하고s = a + b + c, 삼각 형 ABC의외접원과내접원의반지름을각각 R, r이라하자. 4ABC의넓이 = ab ab sin C = 4 4ABC의넓이 = rs 4ABC의넓이 = abc 3 4R,, 3 에의해 R = c; r = ab 4s 오일러의공식은 R OI =Rr이다. OI = p r R Rr = c abc s a + b + c = c abc(a + b c) (a + b) c s = c abc(a + b c) ab + p 3ab (* cos 제법칙 c = a + b p s 3ab) = c c(a + b c) + p q 3 = c ( p 3)c(a + b c) q = (3 p 3)c ( p 3)(a + b)c

283 3. 기하고급문제 83 p 또한 DE = DC + EC p q 3DC EC (* 4CED에서 cos 제 법칙 ) = (b c) +(a c) p 3(b c)(a c) p = a + b +c ac bc p 3ab + p 3ac + p 3bc p q 3c = c + p 3ab +c ( p 3)(a + b)c p 3ab p 3c q = (3 p 3)c ( p 3)(a + b)c ) OI = DE 그림과같이 I, O, D 에서 BC 위에내린수선의발을 H, H, H 3 라고 O 에서 IH 에내린수선의발을 H 4 라하자. 따라서 OH 4 = DH 3 이다. 삼각형 OIH 4 와삼각형 DEH 3 에서 OH 4 = BH BH = a (a + c b) = (b c) DH 3 = DC sin C = DC = (CA DA) = (b c) OI = DE; OH 4 = DH 3 ; \OH 4 I = \DH 3 E =90 ± 이므로삼각형 OIH 4 와삼각형 DEH 3 는합동이다. 그런데 OH 4?DH 3 이므로 OI?DE 이다. 6. 세변의길이가각각 a, b, c (a b c) 인삼각형 ABC의내접원의중심이 I이다. 다음을증명하여라. ( 통신강좌 ) s r abc c IA + IB + IC 3 a + b + c a + a c 증명 ( 상문고이경용 ) IA + IB + IC 3 IA + IB + IC 3 IA = s IA + IB + IC 이므로 3 abc a + b + c ( a c + c ) 을증명하면된다. a r r sin A = cos A = r a (b c) bc = r bc (s b)(s c) = = (s a)(s b)(s c) s (s a)bc s bc (s b)(s c)

284 84 기하 마찬가지로 IB (s b)ca =, IC = s 증명하고자하는식은다음과같다. (s c)ab s IA + IB + IC (s a)bc +(s b)ca +(s c)ab = 3 3s abc ³ c s a + a c ³ c, (b + c)bc +(c + a)ca +(a + b)ab 3abc a + a c = 3(bc + a b) bc + a b = bc + a b b c + a c bc + a b(* a (c b) bc(c b)) + ab + c a bc + a b(* ab(b a) c (b a)) (b + c)bc +(c + a)ca +(a + b)ab 3(bc + a b) 즉, IA + IB + IC 3 abc ³ c a + b + c a + a c 증명되었다. 6. 삼각형 ABC 의내접원의반지름을 r, 외접원의반지름을 R 이라한다. 다음을증명하여라. ( 통신강좌 ) sin A sin B +sinb sin C +sinc sin A r 4R

285 3. 기하고급문제 85 증명 sin A +sin B +sin C = cos A cos A B = cos A + B = sin C cos A B = sin C µ cos A B = sin C sin A sin B +sin C +sin C cos A + B + cos B +sin C = sin A sin B sin C 4cos A cos B cos C sin A sin B sin C = µ cos A + B +cos A B cos C sin A sin B sin C = cos A + B + C +cos A + B C sin A sin B sin C = sin A +sinb +sinc = +cos A B + C R sin A sin B sin C r(r sin A + R sin B +sinb + R sin C) r R = 4ABC 4ABC = r R r R sin A sin B +sinb sin C +sinc sin A µ sin A +sinb +sinc = µ 3sin A + B + C ³ r 6 R = r 4R µ sin A B C +sin +sin (by Jensen 부등식 ) +cos A B C 등호는중간에사용한 Jensen 부등식에의하여 A = B = C 즉, 정삼각형일때만성립한다. 63. 삼각형 ABC 의내심을 I, 내접원의반지름을 r 이라고하자. 다음을증명하고등호조건을밝혀라. ( 통신강좌 ) IA IB IC 8r 3

286 86 기하 증명 ( 언주중공유식 ) AI sin A = r 에서 AI = r. 마찬가지로, BI = r, CI = r. sin A sin B sin C AI BI CI = r 3 sin A sin B sin C r 3 Ã sin A +sin B +sin C! 3 3 r 3 µ sin A + B + C 3 = µ r3 3 =8r 3 6 등호조건은 Jensen 부등식에의하여 A = B = C 즉정삼각형일때이다. 64. 한변의길이가인정n각형이있다. n개의꼭지점을반시계방향으로 A ;A ;:::;A n 이라하고, 선분 A i A i+ (i =; ;:::;n) 의중점을 M i 라하자. 단, A n+ = A 이다. M n+ = M 이라고할때, 선분 A i M i+ (i =; ;:::;n) 들로둘러싸인작은정n각형과본래의정n각형사이의넓이를 S n 이라하면 S n < 6¼ (n 4) 9 임을증명하여라. ( 한국 995-7) 증명서 ( 과기원 95 학번허충길 ) A i M i 와 A i M i+ 의교점을 C i 라하자. 두삼각형 A i M i C i 와 A i M i+ A i+ 에 \M i A i C i = \M i+ A i A i+ = 공통 \A i M i C i = \A i M i A i = \A i M i+ A i+ (* 대칭에의해 ) 이므로두삼각형은닮음이다. A i A i+ = ; A i+ M i+ =, A i M i =, \A i A i+ M i+ =¼ n ¼이고두삼각형 A im i C i 와 A i M i+ A i+ 이닮음이므로코사인제법칙과닮음의성질을이용하여두삼각형의나머지길이를구하면 s µ ¼ A i M i+ = 5+4cos ; M i+ A i+ =; A i+ A + i = n 이다. A i M i =; M i C i = q5+4cos( ¼n ) ; C i A i = q 5+4cos( ¼ n ) C i C i+ = A i M i+ A i C i C i+ M i+ = A i M i+ A i C i C i M i s 0 0 µ ¼ B C B C = q q A n 5+4cos( ¼ ¼ ) 5+4cos( ¼ ¼ ) = +4cos( ¼ n ) q 5+4cos( ¼ n ) 이다. 원래의정n각형의한변의길이는 이고작은정n각형의한변의길이는 C i C i+ 이므로두다각형의 +cos( ¼ n 닮음비는 : q ) 이고면적비는 : +4cos( ¼ n )+4cos ( ¼ n ) 5+4cos( ¼ n ) 5+4cos( ¼ n ) 이다. 따라서, 원래의정n각형의면적과 S n 의비율은 : +4cos(¼ n )+4cos ( ¼ n ) 4sin ( ¼ n 5+4cos( ¼ n ) =: ) 5+4cos( ¼ n )

287 3. 기하고급문제 87 이다. 원래의정 n 각형의면적을구하는과정은간단하므로생략하겠다. 그값을구하면 원래의정 n 각형의면적 = n tan( ¼ n ) 이다. 그러므로이제우리가원하는면적 S n 은 S n = n tan( ¼ n ) 4sin ( ¼ n ) (5 + 4 cos( ¼ ¼ )) = cos ( ¼ n ) (8 cos ( ¼ n )+) 6n sin ( ¼ n ) tan( ¼ n ) ³ 이다. 여기서 cos ¼ ³ < 이므로 9cos ¼ ³ < 8cos ¼ +이고 cos ( ¼ n ) n n n 8cos ( ¼ n )+ < 9 이다. 또한 ¼ < ¼ ³ ¼ n 이므로 sin < ¼ ³ ¼ ³ ¼ < tan 이다. 따라서 sin n n n n < ¼ n 이고 tan( ¼ n ) < ¼ 이다. 그러므로 n S n = cos ( ¼ n ) (8 cos ( ¼ n )+) 6n sin ( ¼ n ) tan( ¼ n ) < 9 6n( ¼ n ) ¼ = 6¼ 9 n 이다. 65. 반지름의길이가각각 r, R (r <R) 인두개의구 S, S 가외접하고두구는하나의원뿔에내접하고있다. S, S 의중심을각각 O, O 라할때다음물음에답하여라. () 두구S, S 에외접하고원뿔에내접하는구 S의반지름의길이 x와 S의중심O에서직선 O O 에그은수선의길이 y를 r, R의식으로나타내어라. () () 에서구한 S와같은구n개를이웃끼리외접하면서두구 S, S 와원뿔사이에꼭끼워넣을수있는 n의값을모두구하여라. ( 한국 995-8) 풀이 ( 과기원 95학번허충길 ) () 원뿔의단면에서꼭끼는원은실제로꼭끼는원이된다. 따라서원뿔의단면에대해생각을하자 ( 그림 I). O ;O 에서원뿔에수선을내려그발을 H ;H 라하자. O 에서 O H 에내린수선의발을 A라하고, O에서 H H 에평행한선을그어 O H 과 O H 와만나는점을각각 B; C라하자. 그러면 O A = O O O A =(R + r) (R r) =4Rr OB = O O O B =(r + x) (r x) =4rx OC = O O O C =(R + x) (R x) =4Rx 이다. O A = OC + OB 이므로 p Rr + p rx 이다. 따라서 Rr x = ( p R + p r) 이다. 삼각형 OO O 의세변의길이는 r + R; r + x; R + x 이다. 헤론의공식을이용해서삼각형의넓이를구한후이를이용해서 y 를구하자. 4OO O = p (R + r + x)rrx = (R + r)y 이다. 따라서 y = p (R + r + x)rrx R + r 이다. 위의 x 자리에위에서구한 x 값을대입하면 r; R 에관한식이된다.

288 88 기하 () 구S를이웃끼리외접하면서끼워넣은단면을생각해보자.( 그림 II) 처럼구S가반지름 y인원위에중심을두고서로외접하는형태가된다. 큰원의중심과구S들끼리의접점을연결해보자. 그러면하나의구S가큰원에대해차지하는각 µ 를생각할수있다. µ µ = x y sin µ x µ =sin y n개의구s를꼭끼워넣기위해서는 µ 의 n배가 ¼가되어야한다. n = ¼ µ = ¼ ³ ¼ = ³ sin x sin y x y 이제 R r ¼ ³ 의범위를구해이범위내의정수값을구하면이것이우리가구하려는 n값이다. sin x y = t 로놓으면 R>r 이므로 t> 인모든값을취할수있다. x = Rr ( p R + p r) = y = p (R + r + x)rrx R + r x y x y = = = = R r r µq R r + = = rt ( p t +) r ³ R r ++ x Rx r R r + = (t +)x (t +)q x r q t = q ++ x t = r Rx ++ x R µ r r s t ++ q t ++ x r Rx t + (t +) q t ( p t+) t + t + q (t +)( p = t +) + t p t +t p t +3t + p t + t ( p t+) t t + q(t + p = t + t +) (t + p t +) Ã t + (t + p t +) = p! 0 t t + p p A t + t + pt + p t> 이므로 p t + pt 은증가함수이다. 따라서 x 는증가함수이다. 그러므로 y x = y t= 3 < x y < lim x t! y < lim t + t! (t + p t +) = x y 가연속함수이고 t>인모든값을취하므로 x µ y 도 3 ; 구간의모든값을취한다.sin x는 0 <x<에서연속인증가함수이므로 ¼ sin < ¼ sin ³ x y ¼ < sin 3 이다. x µ y 가 3 ; ¼ 구간의모든값을취하므로 ³ sin x y! ¼ ³ ¼ sin 구간의모든값을취한다.sin = ³ ¼ 6,sin 9 3 Ã ¼ 도 sin ; =0:34,sin ³ ¼ 0 =0:309 이

289 3. 기하고급문제 89 다. 따라서 9= ¼ ¼ 9 = ¼ 이다. sin =6 이고 ¼ sin (0:34 ) < ¼ sin 3 ¼ sin = ¼ ¼ 6 ¼ sin 3 중정수인것은 7, 8, 9 뿐이다. 따라서, n =7; 8; 9 이다. < =6 ¼ sin (0:309 ) = ¼ ¼ 0 는 9 와 0 사이의실수이다. 그러므로 =0 ¼ ³ 가취하는값 sin x y O B H x y O 3 q O A C H ( 그림 I) ( 그림 II) } 66. 원 S 의중심O 이 O 를중심으로하는원 S 위에있다. S 의반지름이 S 의반지름보다크다. S 과선분 O O 의교점을A라하자. S 위의임의의점 X를중심으로하고 A를지나는원 S를생각하자. 이원 S가 S 과다시만나는점을 Y 라하자. 직선 XY 는 X의위치와상관없이모두고정된한점을지남을증명하여라. ( 벨로루시 995-A5) 증명 ( 김규완 ) 전체를적절히확대, 축소하여원 S 의반지름의길이를 이라고할수있다. O 을 (0, 0), O 를 (, 0), A 를 (a; 0) 이라하자. 각 PXO 는원주각이므로 90 도. A; Y 는 XO 에대해대칭이므로각 ARO =90 도. ) PX;AY 는평행하다. 삼각형 PXQ 와삼각형 AY Q 는닮음. PX : AY =: a = PQ : AQ 을만족하는 Q 는 (=a; 0). a 는상수이므로 Q 는고정된한점. XY 는항상고정된한점 Q 를지난다. 67. 중심이 O, 반지름의길이가 R 인원 C 의내부에고정점 S 가주어져있다. 점 S 를지나고서로직교하는현 AA 0, BB 0 을긋고, 직사각형 SAMB, SBN 0 A 0, SA 0 M 0 B 0, SB 0 NA 를만든다. 점 A 가원 C 위를움직일때점 M, N 0, M 0, N 들이움직이는점의집합의합집합을구하여라. (APMO 995-4)

290 90 기하 풀이사각형 SAMB, SANB 0, SBN 0 A 0, SA 0 MB 0 이직사각형이므로, MNM 0 N 0 도직사각형이다. 그리고, BB 0 과 AA 0 의수직이등분선의교점은 O 이고, 이것은 MNM 0 N 0 의중심이된다. 여기서 OM = OM 0 = ON = ON 0 임을알수있다. 직선 MS 와 AB, A 0 B 0 와의교점을 I, H 라하자. \HSA 0 = \ASI = \SAI = \A 0 AB = \A 0 B 0 S =90 ± \HA 0 S 이므로 MS? A 0 B 0 이다. MN 0 과원과의 B 이외의교점을 D 라하면 \DA 0 A = \BAA 0 = \ASM 에서 A 0 D k MS 즉 \B 0 A 0 D =90 ± 가된다. 따라서 B 0 D 는원의지름. 방멱의정리에의해, SA 0 SA = SB 0 SB 0 SB = R OS 이므로다음식이성립한다. 4 OM = MN + MN 0 =(SB + SB 0 ) +(SA + SA 0 ) = SB + SB 0 +SB SB 0 + SA + SA 0 +SA SA 0 =(SA + SB )+(SA 0 + SB 0 )+4(R OS ) = AB + A 0 B 0 +4(R OS ) = A 0 D + A 0 B 0 +4(R OS ) = B 0 D +4(R OS )=(R) +4(R OS ) =4(R OS ) ) OM =R OS 이고, OM = p R OS 이때, R, OS 는일정한값이므로, MNM 0 N 0 의꼭지점은중심이 O 이고, 반지름이 p R OS 인원주상에있게된다. 구하는점의집합은중심 O, 반지름이 p R OS 인원이다. } 68. P 는볼록사각형 ABCD 내부의점이다. \AP B, \BPC, \CPD, \DPA 의각의이등분선들이 AB, BC, CD, DA 와각각점 K, L, M, N 에서만난다. KLMN 이평행사변형이되는점 P 를모두찾아라. (Towns 995 가을 SA) 풀이 BP = PD 이면각의이등분선정리로 AK : KB = AP : PB = AP : PD = AN : ND 이므로 KN k BD k LM. BP > PD 이면비슷하게 AK : KB = AP : PB > AP : PD = AN : ND 이므로 KN과 LM 모두 BD와 B쪽연장에서만남 ( 즉, K, L이 N, M보다 BD에더가까움 ). 그러므로 BD, AC의두수직이등분선의교점. } 69. 한원주위에네점 P, P, P 3, P 4 가주어져있다. 삼각형 P P 3 P 4, P P 3 P 4, P P P 4, P P P 3 의내심을각각 I, I, I 3, I 4 라한다. 사각형 I I I 3 I 4 는직사각형임을증명하여라. (APMO 996-3) 증명 ( 과기원수학과 94 학번이정훈 ) Π Π4 Ι 3 Ι Ι 4 Ι Π Π3

291 3. 기하고급문제 9 선분 P 3 I 은 \P P 3 P 4 의이등분선이고, 선분 P 3 I 은 \P P 3 P 4 의이등분선이므로 \I P 3 I = \P 4 P 3 P \P 4 P 3 I = \P P 3 P 4 \P P 3 P 4 = (\P P 3 P 4 \P P 3 P 4 )= \P P 3 P 마찬가지로하면 \I P 4 I = \P P 4 P 그런데 \P P 4 P 와 \P P 3 P 는원주각으로같다. ) \I P 3 I = \I P 4 I 따라서사각형 I P 3 P 4 I 는원에내접하는사각형이다. 마찬가지로하면사각형 I P 3 P I 4 도원에내접하는사각형이된다. 따라서 \I 4 I I = 360 ± \I 4 I P 3 \I I P 3 = 360 ± (80 ± \I 4 P P 3 ) (80 ± \I P 4 P 3 ) = \I 4 P P 3 + \I P 4 P 3 = \P P P 3 + \P P 4 P 3 = (\P P P 3 + \P P 4 P 3 )= 80± =90 ± 마찬가지로하면 \I 3 I 4 I, \I I 3 I 4, \I I I 3 도 90 ± 가됨을보일수있다. 따라서사각형 I I I 3 I 4 는직사각형이다. 70. 삼각형의 ABC 의세각 A, B, C 의이등분선들이마주보는변과각각점 D, E, F 에서만나고, 삼각형의 ABC 의외접원과는점 K, L, M 에서각각만난다. 다음을증명하여라. ( 폴란드 996/997 차 -5) AD DK + BE EL + CF FN 9 증명 AD DK = j4abcj j4bkcj = bc BK =( b+c a ). (4ABC와 4BCK에서 cos 제법칙으로비교하면나옴.) 이식들다합하여AM-GM 으로처리하면끝. 7. 삼각형 ABC 의방심 I A, I B, I C 가있다 (I X 는꼭지점 X 의맞은편에있는방심이다 ). 4I A BC, 4I B CA, 4I C AB 의수심을각각 H A, H B, H C 라하자. AH A, BH B, CH C 가한점에서만나고, 삼각형 ABC 의내심 I 와무게중심 G 를연결하는직선 IG 위에있음을보여라. ( 통신강좌 ) 증명 Ι Χ Α Ι Β Η Χ Β Ι Η Β Χ Η Α Ι Α AI A 는각A의이등분선 BI B 는각B의이등분선 CI C 는각C의이등분선 ) AI A ;BI B ;CI C 는 4ABC의내심에서만난다. IB?I A I C ;CH A?I A I C! IB==CH A IC?I A I B ;BH A?I A I B! IC==BH A

292 9 기하 ) IAH C B는 평행사변형이다. 마찬가지로, IBH A C는평행사변형 ICH B A는평행사변형이된다. I 는원점이라하자. AH A 의중점 T A ;BH B 의중점 T B ;CH C 의중점 T C 를생각하자. T A = H A + A T B = H B + B T C = H C + C = A + B + C = A + B + C = A + B + C G = A + B + C 이므로 3 ) T A ;T B ;T C 는한점이고, 그점을 T 라하면 AH A ;BH B ;CH C 는 T 에서만난다. ) I;T;G 는한직선위에있고, IG 를 3: 로외분하는점이 T 이다. 7. 한평면에다음을만족하는볼록각형이존재함을보여라. () 변의길이를 a ;a ;:::;a 라하면fa ;a ;:::;a g = f; ;:::;g 이다. () 이다각형에내접하는원이존재한다. ( 통신강좌 ) 증명이웃하는변순서대로길이를재어각각을 a ;a ; ;a 이라하자. x + x = a ;x + x 3 = a ; ;x n + x = a n 내접원을그린후그중심각을생각하자. f(r) =tan x x r +tan r + x +tan n r = ¼가된다. n¼ lim f(r) = r!+0 lim f(r) =0 C r! A! f(r) =¼를만족하는 r이 (0, ) 에서존재한다. f(r) 은연속 ξι ξι ρ (a ;a ; ;a )=(; 4; 3; 6; 5; 8; 7; 0; 9; ; ; ) 으로잡으면 8 i (i는홀수) >< 3 x i = (i (mod4)) 가된다. >: (i 0(mod4)) ) 과같이순서를정하면적당한 r이존재해서볼록 각형을만들수있다. 73. 삼각형 ABC 의외접원 O 가있다. C 를지나고원 O 와 Q 에서다시만나는 O 은직선 AB, BC 와각각 A, B 에서만난다. 직선 AB 와 A B 은 P 에서만난다. AB 와 A B 의중점을각각 M 과 M 이라하자. 직선 CM 과원 O 가 N 에서, 직선 CM 과원 O 은 N 에서각각만난다. P, Q, N, N, M, M 이한원위에있음을보여라. ( 통신강좌 )

293 3. 기하고급문제 93 증명다음을보이면문제가풀린다. PQB;B와 PQA;A은각각한원에내접한다. 점 P; Q를지나는임의의원 O 는 AB와 A; B을같은비율로분할한다.( 즉원O 은 AB; A; B과만나는점을각각 T;T이라할때 TA TB = T A ) T B 3 원 O 와원O 이만나는점을 S라할때점C; S; T 가한직선위에있다. Α Ο Ο Ν Β Μ Ν Β Μ Χ Π Θ Ο i) \PBQ = \ACQ = \PB Q! PQB;B는한원에내접 \PAQ = \BAQ = \BCQ = \PA Q! PQA A는한원에내접 ii) \TAQ = \BAQ = \BCQ = \B CQ = \B A Q = \T A Q \AT Q =80 ± \PMQ =80 ± \PM Q = \A T Q ) 4AT Q 4M B Q! QT QT = TB TB \BMQ = \PMQ = \PM Q = \B M Q \MBQ =80 ± \PBQ =80 ± \PB Q = \M B Q ) 4MBQ 4M B Q! QT QT = TB TB ) T A TA = T B TB! TA TB = T A T B iii) \TSC = \TSQ+ \QSC = \TSQ+ \QA C = \TSQ+ \AP Q (* PQA A는한원위에있다.) = 80 ± (* PQTT 은한원위에있다.) ) 점 C; S; T 는한직선위에있다. 74. 원 O, O 가 I 에서서로외접하고, 원 O 는원 O, O 에대해각각 R, S 에서외접한다. 원 O, O 의공통외접선이원 O, O 에각각 T, U 에서접하고 A, B 에서원 O 와만난다. I 를지나는원 O, O 의공통내접선이원 O 와 C, D 에서만난다. 단, C 는직선 AB 에대해 I 와같은쪽에있는점이다. 다음을보여라. () R, T, D가한직선위에있음을보여라. () I가 4ABC의내심임을보여라. ( 통신강좌 ) 증명 Ο Α Ρ Χ ν Σ Τ Ε Υ Β Ο (or Μ) Ο 직선 AB, 호 ADB, 호 ACB 의중점을각각 E;M;N 이라고하자. MN 은 O 의지름이고, E; M; N 은한직선위에있다. D 에서의접선은 AB 와평행하다. ) R; T; M 은한직선위에있다. 4MRN» 4MET (AAA 닮음, 한각은직각, 한각은공통 ) MT MR = ME MN 마찬가지로 MU MS = ME MN;MT MR = MU MS (O 에대한 M 의방멱 = O 에대한 M 의방멱 ). ) M 은 CD 위에있다.! M = D: ) R; T; D 는한직선위에있다.

294 94 기하 AD = BD! \DAB = \DBA \DAB = \DCB! \DCB = \DCA! CD는 \ACB의이등분선 \DBA = \DCA (O 에대한D의방멱 )=(DI) = DE DN: 4ADN과 4AED는직각삼각형이므로 DE DN =(DA) ) DI = DA \DAI = \DIA \DAI = c + \IAB C \DIA = c A! \IAB = \CAI + \CAI ) AI 는 \CAB 의이등분선이다. ) I 는 4ABC 의내심이다 ABC 에서변 BC 위의점 D 가다음조건을만족한다 : 4ABD 의내접원과 \A 안의 4ADC 의방접원은반지름이 x 로같다. 같은방법으로 y, z 를정의하였을때 x + y + z 7 6 r 임을보여라. 단, r 은 4ABC 의내접원의반지름이다. ( 통신강좌 ) 증명 ( 과기원수학과 96학번허석문 ) E를 4ADC의방심이라하고, P 를 E에서 AC의연장선에내린수선의발이라하자. I, J를각각4ABC, 4ABD의내심이라하자. I에서 AB, AC에내린수선의발을각각 R, Q라하고J에서 AB에내린수선의발을T라하자. Α Χ Π Θ Ε Ι ϑ Β Ρ Τ AB = c, BC = a, CA = b, s = (a + b + c) 라하면 PE = TJ = x; IR = IQ = r I, J 는내심이므로 BR = s b; BT = (c + BD AD) () E 는방심이므로 AP = (b + AD + CD) () 4IRB 와 4JTB 가닮음꼴이므로 () 에의해 x r = BT c + BD AD = BR (s b) ) (s b) c + BD AD x = r (3) I는내심이므로 CQ = s c \QCI = \DCI, \PCE = \DCE이므로 \QCI + \PCE =90 ± 가되어 \PEC와 \QCI는닮음꼴이다. 따라서 x CP = QC IQ = s c r

295 3. 기하고급문제 95 () 에의해CP = AP b = (AD + CD b) 이므로위식에서 (3) 과 (4) 를변변더하면 µ s b 이므로 r rx s c = CP = (AD + CD b) (4) + r x = s c (c b + BD + CD)= (a b + c) =s b x = r(s b)(s c) r +(s b)(s c) 헤론의공식에의해 r s = s(s a)(s b)(s c) 이므로 이다. 같은방법으로 y = x = 이다. 코시 - 슈바르츠부등식에의해 r(s b)(s c) (s a)(s b)(s c) +(s b)(s c) s rs s b, z = rs 이다. 따라서 s c x + y + z = 4 r = rs s a 3(x + y + z ) (x + y + z) (5) 산술 { 조화평균부등식에의해 x + y + z 3 µ 3 x + y + = 3 4 r (6) z µ 3 (5), (6) 에의해x + y + z 3 4 r = 7 6 r 이다. 76. 한원에내접하는삼각형 ABC 에대하여 l a = ma M a ; l b = m b M b ; l c = mc M c 라하자. 여기서 m a, m b, m c 는내각의이등분선의길이 ( 삼각형내부의 ) 이고, M a, M b, M c 는내각의이등분선이원과만날때까지연장했을때의길이이다. 이때, l a sin A + l b sin B + l c sin C 3 임을보이고, 등호가성립할필요충분조건은삼각형 ABC 가정삼각형일때임을보여라. (APMO 997-3) 풀이 (O±cial)

296 96 기하 4ABC 의세변의길이를 BC = a; CA = b; AB = c 라하고, 외접원의반지름을 R 이라하자. \A 의이동분선이변 BC 와만나는점을 D 외접원과만나는점을 L 라하자. \BAL = \DAC = n \ ^ \BLA = \DCA = \C; \ADC = \ABL 이므로 4ABL 4ADC ) AB AL = AD AC bc AB = m a;al= M a 이므로 m a = µ M a 4ABL에서, AL =Rsin B + A 같이하며 ) l a sin A + l b sin B = ca b cos C A ) l a sin A + l b sin B + µ bc 3 a cos B C =R sin ) l a = m a M a = µ bc M a bc ) AD = 90 ± + B C = AB AC AL bc 4R cos B C 4R sin A cos B C l c ; sin C = ab c cos A B l c sin C = bc a cos B C ca b cos C A µ =3 sec B C sec C A sec A B 3(sec B C sec C A sec A B =R cos B C = bc a cos B C + ca b cos C A ab c cos A B ) 3 등호는 bc B C a sec = ca C A b sec = ab A B c sec sec B C =sec C A =sec A B + ab c cos A B 산술평균 기하평균 즉, A = B = C(a = b = c) 일때성립한다. 즉등호는 4ABC 가정삼각형일때성립한다. } 77. 삼각형 ABC 에내접하는원의반지름을 R 이라하고, 이내접원과외접하면서변 AB, AC 에접하는원의반지름을 R A 라하자. 비슷한방법으로 R B, R C 를정할때다음을증명하여라. ( 통신강좌 ) R A R B R C R3 7 증명 ( 동주여중권수현 ) 4ABC 의내접원이각변과접하는점을 D; E; F 라하자. A = F I E B D C

297 3. 기하고급문제 97 BDIF 에대해생각해보자. F F I B E I G B D 내심은각의이등분선의교점이므로반지름이 R B 인원과원 I 의중심은모두 \B 의이등분선위에있다. 반지름이 R B 인원을 I B, 각접점을차례로 D 0 ;E 0 ;F 0 라하자. DI k D 0 I B (*\I B D 0 D+\DD 0 =80 ± ) 점 I B 를지나고 DD 0 에평행한직선이 DI 와만나는점을 G 라하면, D \II B G = \B; II B = R + R B ; IG = R R B 이다. 따라서 sin B = R R B R + R B, 즉 R B = sin B sin B +R 마찬가지로 R A = sin A +sin A R; R C = sin C +sin C R 이다. 그러므로, R A R B R C R3 7 을증명하려면 을보이면된다. 분모와분자에 sin A +sin A sin B +sin B sin C +sin C 7 ³ ³ ³ +sin A +sin B +sin C 를곱하면좌변은 8 >< cos A cos B cos C 9 >= >: ( + sin A )( + sin B )( + sin C ) >; 이된다.0< A ; B ; C < 90± 이므로 cos µ; sin µ(µ = A ; B ; C ) 는모두양이다. 따라서, ³ +sin A cos A cos B cos C ³ +sin B ³ +sin C 5 3 p 3 을보이면된다. +sin A =sina + B + C +sin A = sin A + B + C + A 4 µ = sin 90 B + C 4 = cos B + C 4 cos B + C 4 cos B + C 4 그러므로 µ +sin A µ +sin B µ +sin C µ =8 cos A + B cos B + C cos C + A 4 4 4

298 98 기하 cos A + B 4 cos B + C 4 = µ cos A + B + C + B +cos A C 4 4 = µ cos(90 A + C )+cos A C 4 4 = µ sin A + C +cos A C 4 4 cos A + B 4 cos B + C 4 cos C + A 4 = = µ = 4 µ sin A + C 4 sin A + C +cos A C cos C + A cos A + cos C µ cos A +cosb +cosc µ * sin A + C =sin 80 B =cos B 따라서 µ +sin A µ +sin B µ +sin C = µ cos A +cosb +cosc cos A cos B cos C = = = µ cos A + B µ cos A + B +cos A B cos C +cos A B µ * cos C =cos80 A B sin A + B =sin A + B µ sin(a + B)+ sin A + sin B = (sin A +sinb +sinc) 4 (* sin(a + B) = sin(80 C) =sinc) µ +sin A cos A cos B cos C µ +sin B µ +sin C = cos A cos B cos C µ cos A +cosb +cosc 5 cos A cos B cos C r Ã3 3 cos A cos B cos C! (* 산술기하평균등호조건 A = B = C) = r 3 cos A 9 cos B cos C = r 3 (sin A +sinb +sinc) 9 4 s 5 µ 3 A + B + C sin 3 (* 젠센부등식 0 µ 80 ± 일때 sin µ 는위로볼록등호조건 A = B = C =80 ± ) s = p () A + B + C =80 ± )

299 3. 기하고급문제 99 따라서, ³ +sin A cos A cos B cos C ³ +sin B ³ +sin C 5 3 p 3 임이증명되었다. 등호조건은 A = B = C 일때이므로, 4ABC 가정삼각형일때이다. 78. 삼각형 ABC에서점 C에서 AB로내린중선과점B에서 AC로내린중선이서로수직일때,cotB + cot C 3 임을보여라. ( 통신강좌 ) 증명 ( 부산과학고 학년성충엽 ) AB; AC 의중점을각각 E; D 라하고, BD 와 EC 의교점을 T 라하자. \BTC =90 ± A E D T B C cot B + cotc = tan B + tan C = tan(\ebt + \CBT) + tan(\dct + \BCT) = = (tan \EBT) (tan \CBT) (tan \EBT)+(tan\CBT) ET DT BT CT BT ET BT + CT BT + CT BT CT DT CT + BT CT = BT ET CT BT(ET + CT) + CT DT BT CT(DT + BT) + (tan \DCT ) (tan \BCT) (tan \DCT)+(tan\BCT) = = ( 3 BD) 3 3 EC ( + 3 CE) 3 3 BD 3 BD CE CE BD 3 (* T 가 4ABC의무게중심 ) 9 (BD + EC ) = 3 BD CE 3 BD + CE BD CE = 3 = 3 ³ * BD + CE BD CE = $ (BD CE) = 볼록오각형 ABCDE 에서 4ABC, 4BCD, 4CDE, 4DEA, 4EAB 의넓이가모두같다고할때다음을증명하여라. 4 S(ABCDE) <S(ABC) < 3 S(ABCDE) 단, S(X) 는도형 X 의넓이를나타낸다. ( 통신강좌 , 미국 97-5 응용 ) 증명 ( 경성중 3 학년김윤태 )

300 300 기하 먼저, area(abcde) >area(abc) 를증명하자. 3 A 오른쪽그림에서 area(abcde) = area(abc)+ area(acd)+ area(ade) 한편, 오각형 ABCDE 는볼록오각형이므로 area(acd) >area(cde) ) area(abcde) >area(abc)+area(cde)+area(ade) =3area(ABC) B E C D 따라서, area(abcde) >area(abc) 3 다음으로, area(abc) > area(abcde) 를증명하자. 4 area(abc) 5 area(abcde) 라고가정하자. 4 그러면, area(acd) =area(abcde) area(abc) area(ade) = area(abc) =area(bcd) 4ACD, 4BCD, 4ABE 의높이를각각 h, h, h 3 라하면 area(bcd) =area(cde) 에서 BE==CD이므로 h = h + h 3 에의해 h CD = h CD = h CD ) h = h + h 3 = h h 3 = h area(abe) = area(bce) area(abe) = area(abcde) area(abe) area(cde) area(abc) = 3 area(abcde) 이는앞에서증명한것과모순이다. ) area(abc) > 4 area(abcde) 따라서, 4 area(abcde) <area(abc) < 3 area(abcde) 80. 4ABC 에서 D 는 A 에서변 BC 에내린수선의발이다. 점 D 를지나는한직선위에 D 가아닌두점 E, F 를잡아 AE? BE, AF? CF 가되게한다. M, N 이각각선분 BC, EF 의중점일때, AN? NM 임을보여라. (APMO 998-4) 증명 (O±cial) ADMP 가직사각형이되게점 P 를잡고, 직선 AP 위에점 Q; R을 QBDA와 ADCR이직사각형이되게잡는다. Α Θ Π Ρ Φ Β Μ Ν Χ Ε

301 3. 기하고급문제 30 이때, AE? BE 이므로 ADEQ 는 AB 를지름으로하는원의내접사각형이고, 같이하여 ADF R 은 AC 를지름으로하는원의내접사각형이다. \AQE =80 ± \ADE = \ADF =80 ± \ARF 이므로 EQ=RF 즉, EQRF 는사다리꼴이다. 한편, 사각형 QBCR 에서 M 은 BC 의중점이고 PM=QB 이므로 P 는 QR 의중점이다. 따라서사다리꼴 QEF R 에서 P, N 이각각 QR, EF 의중점이므로 NP =QE. \QED(= \QBD) =90 ± 이고 NP =QE 이므로 \PND =90 ± = \PAD. 따라서 ANDP 는 PD 를지름으로하는원의내접사각이다. 또 PD 는직사각형 ADMP 의대각선이므로점 M 은이원주상에있다. 즉 ANDM 은원에내접하는사각형이다. 따라서, \ANM = \ADM =90 ± 증명 (O±cial) \ADB = \AEB =90 ± 이므로사각형 ADEB는 AB를지름으로하는원의내접사각형이다. 또한, \ADC = \AF C =90 ± 이므로사각형 ADCF 는 AC를지름으로하는원의내접사각형이다. Α Β Ε Μ Ν Χ Φ 따라서, 같은호에대한원주각이므로 \ABD = \AED, \ACD = \AF D이고따라서 4ABC 4AEF AM; AN은각각4ABC와 4AEF 의중선이므로 4AMC 4ANF. 따라서, \AMD = \AMC = \ANF =80 ± \AND 그러므로 A, M, D, N은동일원주상에있다. 따라서, \ANM = \ADM =90 ± 이다. 주 ) AB; AC의크기와EF의위치에의하여여러가지형태가가능하나어느경우나위의증명과정은동일하다. ) \AMN \ACF 를구하여 \ANM = \AF C =90 ± 라하여도된다. 8. 네점A, B, C, D가한직선위에차례로위치해있다고하자. 두점B와 C를지나는 ( 움직이는 ) 원을 k라할때, A와 D에서원 k에그은접선의접점을각각m, N; K, L이라하자. P 를 MN과 BC의교점, Q를 KL과 BC의교점이라할때, P 와 Q의위치는원 k에상관없이일정함을보여라. ( 통신강좌 ) 8. 볼록육각형 ABCDEF 에서 \A+\C+\E =360 ± 와 AB BC CD DE EF AB =이성립하면 FA BF FD DE EC CA = 임을증명하여라. ( 폴란드 999 3차-6) 증명 4BAF를 4BCX로회전닮음시키면앞조건에의해 \DCX = \E 이고그럼또뒷조건에의해 4DCX도 4DEF 와회전닮음. 그럼 4ABC»4FBX 와 4CDE»4XDF 도회전닮음이고이로부터끝.

302 30 기하 83. 삼각형 ABC 의외부에 4ABD, 4BCE, 4CAF 가정삼각형이되도록하는점 D, E, F 를잡자. 이정삼각형들의변을연장하여다음과같은교점을만들자 : BE 와 AF 는 K 에서만나고, DB 와 FC 는 L 에서, DA 와 EC 는 M 에서만난다. 세직선 DK, EL, FM 이모두평행함을증명하여라. ( 호주 999-6) 증명 \KAB =80 ± \FAC \A = 0 ± \A 이다. 비슷하게 \KBA =0 ± \B 이고, \A + \B + \C = 80 ± 이므로 4KAB 의내각의합에서 \AKB = 0 ± \C 이다. 4LBC, 4MCA의내각들도마찬가지로구할수있고, 그럼 4ABK»4LBC»4AMC 의닮은삼각형들이된다. 이닮음비를 l ::k 라하고 CB = x; BL = y; LC = z 라하면 가된다. 4KBD 와 4EBL 에서 KB = xl; BA = yl; AK = zl; CM = xk; MA = yk; AC = zk KB : BD = KB : BA = xl : yl = x : y = EB : BL 로맞꼭지각과함께서로닮음이고, 따라서 DK 와 EL 은평행. 같은방법으로 4KAD 와 4FAM 이닮음이고 KD 와 FM 도평행. 따라서, 세직선 DK, EL, FM 은모두평행하다. 주복소수를이용할수도있다. 84. 두원 과 가두점 P 와 Q 에서교차한다. P 에더가까운이두원의공통접선이, 와각각점 A, B 에서접한다. 점 P 에서의 의접선이 와점 C 에서다시만나고, AP 의연장선이 BC 와 R 에서만난다. 삼각형 PQR 의외접원이 BP 와 BR 에모두접함을보여라. (APMO 999-3) 증명 A α γ P β B γ R Γ Γ Q C

303 3. 기하고급문제 303 \PAB =, \ABP =, \QAP = 라하자. PC 는 의접선이므로 \QBC = \QP C = \QAP = 이다. 따라서네점 A, B, R, Q 는동일원주상에있다. AB 는 과 의공통접선이므로 A; B; R; Q 가동일원주상에있으므로 이고 \BQR = \BAR = 이다. 따라서, \AQP = \PAB = ; \PQB = \PCB = \ABP = \ARB = \AQB = \AQP + \PQB = + \PQR = \BQR + \PQB = + \BPR 은삼각형 ABP 의꼭지점 P 에서의외각이므로 \BPR = + 이다. 따라서 \PQR = \BPR = \BRP 그러므로 BP 와 BR 은삼각형 PQR 의외접원에접한다. 85. 삼각형 ABC 에서각 A 의이등분선과외접원이점 D 에서만난다. 단, D 와 A 와다른점이다. 점 B, C 에서직선 AD 에내린수선의발을각각 K, L 이라할때, AD BK + CL 임을증명하여라. ( 폴란드 000 차 -) (AB+AC)BD 증명톨레미에의해 AD =, BC 또 4ABK»4ACL»4BDM 임에서 (M은 BC의중점 ) BK + CL = DM BD (AB + AC). 이것을준식에대입하면 BD BC DM =BD DM, 이것은 (BD DM) 0 이므로성립. 86. AC = BC 인삼각형 ABC 의내부에 \PAB = \PBC 를만족하는점 P 가있다. AB 의중점을 M 이라할때, 다음을증명하여라. ( 폴란드 차 -) \AP M + \BPC =80 ± 증명 CP \ AB = Q, PM이 PAB의외접원과만나는점을 R, P 에서 AB에내린수선의발을 H라하자. AP B의외접원에AC, BC는접선이된다. ACBO cyclic 임에서 \AP B =90 ± + C 임과 PM MR = AM MB = CM MO 임을알수있다. 여기서 CPOR cyclic. \HPQ = \OCQ = \ORP = \OPR 임에서 \OPA = \HPB 를빼면 \AP M = \BPQ. 87. 삼각형 ABC 의꼭지점 A 에서그은중선과각의이등분선이 BC 와만나는점을각각 M, N 이라하자. N 에서 NA 에수직하도록그린직선이 MA, BA 와만나는점을각각 Q, P 라하자. 그리고, P 에서 BA 에수직하도록그린직선이 AN 과만나는점을 O 라하자. OQ 와 BC 가수직임을보여라. (APMO 000-3) 증명 ( 과기원전기및전자공학과 99 학번박치연 ) AN 을 y 축, N 을원점으로하여좌표축을잡는다. A B P Q N M O C 일반성을잃지않고 P ( ; 0), A(0; a) 라고하면 PO?AP 에서 O(0; a ) 이다. AN 은 \A 의이등분선이므로

304 304 기하 ½ AP : y = ax + a AC : y = ax + a () BC 를 y = mx 라고하면 () 에의해 B; C 의 x 좌표는각각 a m a, a m + a 이다. M 의좌표 : ma m a ; m a m a O 를지나는 BC 에수직인직선과 PN 의교점을 Q 0 라하면 Q 0 ( m a ; 0) (* OQ0?BC) 따라서 AQ 0 : y = a x + a가되고이것은m ma m m a ; m a m a 를지난다. ) Q 0 는 AM 과 PN 의교점, 즉 Q 가되고 ) OQ 0?BC 이다. 88. 반지름 인원의원주위에세점 P 0, P, P 가있는데, P P = t< 이다. 각각의 i 3 에대해, 4P i P i P i 3 의외심을 P i 라하자. () 점 P, P 5, P 9, P 3, ::: 들이모두한직선위에있음을증명하여라. 500 () P 에서 P 00 까지의거리를 x라하고, P 00 에서 P 00 까지의거리를 y라하자. p x=y 이정수가되게하는t의값을모두구하여라. ( 캐나다 00-5) 풀이 P 0 은잊어버리고 P 부터생각하기로하자. P 3 은주어진원의중심이고, P 3 은 P P 의수직이등분선위의점이므로4P 3 P P 은 P 3 을꼭지각의점으로하는이등변삼각형이다. 또한 P 4 는이이등변삼각형의외심으로역시 P P 의수직이등분선위의점이므로결국 P 3 P 4 가그수직이등분선이다. 즉, 이수직관계에의해 µ = \P 3 P P =90 ± \P 4 P 3 P = \P 5 P 4 P 3 이성립함을그림에서발견할수있다. 따라서, 4P 3 P P 과 4P 5 P 4 P 3 은닮은이등변삼각형이고, 그닮음비를 r 이라하면 r = P P P 3 P 4 = P P P P 3 P P 3 P 3 P 4 =cosµ sinµ 이된다. 또한, 그림에서 \P P 3 P 5 =90 ± 임을알수있다. 지금까지의이야기는 P ;:::;P 5 대신에 P i ;:::;P i+4 로해도다마찬가지이므로, P! P 3! P 5! P 7! P 9! 는일정한방향으로 90 ± 씩꺽이며, 일정한비 r 만큼거리가달라지는점열이된다.

305 3. 기하고급문제 305 따라서, 그림에서직각삼각형의닮음에따라 P, P 5, P 9 가한직선위에있고, 마찬가지로 P ;P 5 ;P 9 ;P 3 ;::: 들은모두한직선위에있어 () 이증명되었다. 이제 () 를풀어보자. P 5 P 9 = r P P 5 가되는것처럼, 마찬가지의닮음에의해 P 00 P 00 = r 500 P P 00 이되므로, r 500 x y = r =sinµ 이값은양수이고또 이하이므로이것이정수가되는경우는 sin µ = 또는 일때뿐이고,0± <µ< 90 ± 에서구해주면 µ =5 ± ; 45 ± ; 75 ± p p 가해가된다. t =cosµ 이므로, t = 6 ; p p p ; 6+ 가답이다. } 89. 볼록육각형 A A A 3 A 4 A 5 A 6 이주어져있다. 선분 A A, A A 3, :::; A 5 A 6, A 6 A 의중점을각각 M, M, :::;M 6 으로나타내자. M M 3 M 5 와 M M 4 M 6 이둘다정삼각형이라고할때, A A 3 A 5 와 A A 4 A 6 둘다정삼각형임을증명하여라. (00 여름학교모의고사 ) 증명 중점연결정리에의해 M M 6 = A 6 A M M 3 = A A 4 M 4 M 5 = A 4 A 6 이제 M M 6 = M M 3 = M 4 M 5 만보이면증명끝. 두정삼각형 M M 3 M 5 와 M M 4 M 6 은같은중심을갖는다 ( 좌표를생각해보라 ). 그중심을 O 라고하자. 그러면 M OM 6 M 3 OM M 5 OM 4 (SAS 합동 ) 이므로 M M 6 = M M 3 = M 4 M 두개의합동인정 n 각형 S 와 T 가평면에있는데이들의교집합이 n 각형을이루고있다 (n 3). S 의변들을빨강으로, T 의변들을파랑으로색칠했다. 다각형 S \ T 에서파랑변들의길이의합이빨강변들의길이의합과같음을증명하여라. (APMO 00-3) 증명 (Kalva) Let one regular n-gon have vertices P ;P ;:::;P n and the other have vertices Q ;Q ;:::;Q n. Each side of the n-gon forms a triangle with one of the P i or Q i. Note that P i and Q j must alternate as we go around the n-gon. For convenience assume that the order is P ;Q ;P ;Q ;:::;P n;q n. Let the length of the side which forms a triangle with P i be p i,and the length of the side which forms a triangle with Q i be q i. Each of these triangles has one angle (80 ± 360 ± =n). Adjacent triangles have one of their other angles equal (alternate angles), so all the triangles are similar. If the sides of the triangle vertex P i have lengths a i, b i, p i, then the side P i P i+ is b i + q i + a i+,anda i =a i+ = b i =b i+ = p i =p i+. Similarly, if the sides of the triangle vertex Q i have lengths c i ;d i ;q i, then the side Q i Q i+ is d i + p i+ + c i+ and c i =c i+ = d i =d i+ = q i =q i+. But a i =b i = d i =c i (not c i =d i ), because the triangles alternate in orientation. Put a i =p i = h, b i =p i = k. Notethata i +b i >p i,soh+k > 0. We have also c i =q i = k, d i =q i = h. Adding the expressions for P i P i+ we get perimeter P i = P (b i +q i +a i+ )=k P p i + P q i +h P p i. Similarly, perimeter Q i =(h + k) P q i + P p i. The two n-gons are equal, so (h + k ) P p i = (h + k ) P q i. Hence P p i = P q i, which is the required result. 9. 삼각형 ABC 의세변 BC, CA, AB 가각각 a, b, c 이고 b<c 이다. \A 의이등분선이 BC 와만나는점을 D 라하자. (i) 변 AB, AC 위에꼭지점아닌점 E, F 가있어BE = CF, \BDE = \CDF 를만족할필요충분조건을구하여라.( 구하는조건을 A, B, C의각에대해서술하여라.) (ii) 위조건하에서BE의길이를 a, b, c에대한식으로나타내어라. ( 중국 00-)

306 306 기하 풀이 \A =, \B =, \C = 라하고BE = CF = x, \EDB = \FDC = µ 라하자. 그리고, BA : AC = BD : DC 이므로 BD AB = CD AC = k 라두자. 4ABC 와 4BED, 4CFD 에서 sine 정리에의해 가된다. 여기서 c sin = b sin x sin µ = x sin µ = kc sin( + µ) = kb sin( + µ) 이고, kc sin(80 ± ( + µ)) = kb sin(80 ± (µ + )) = kc sin( + µ) kb sin( + µ) sin sin( + µ) =sin sin( + µ) 가된다. 그런데 sin A sin B = (cos(a B) cos(a + B)) 이므로 이고,0< + µ; + µ<¼, 6= 이므로 가된다. 즉 + + µ = ¼ = + + 이므로, cos µ cos( + µ) =cosµ cos( + µ) cos( + µ) =cos( + µ) + +µ =¼ µ = 이다. F 가 AC 위에있기위해서는 \ADC > µ 여야하므로, + >, 즉 \A <\B ( ) 이다. \B <\C 이므로 \A <\C 는얘기하지않아도되겠다. 역으로 ( ) 를가정하자. 그러면우리는 \BDE = \CDF = \A 를만족하는 E, F 를각각 AB, AC 위에서찾을수있다. 닮은삼각형이생기므로 BD : BE = AB : BC; CD : CF = AC : BC 즉 BE BC = BD AB = k; CF BC = CD AC = k 로 BE = CF 임을알수있다. 따라서,( ) 은구하는필요충분조건이다. a = kb + kc 이므로 k = a 이고, 위의닮음비에따라 b + c BE = k BC = a b + c 가된다. }

307 3. 기하고급문제 임의의삼각형의세변의길이 a, b, c 에대하여다음의부등식 a 3 + b 3 + b 3 + c 3 + c 3 + a 3 < abc 이성립함을증명하여라. ( 한국 00-) 증명 보조정리 a; b; c 가삼각형의세변이면 a b + c + 보조정리증명 ( 방법 ) 삼각부등식에의해 a b + c + b c + a + b c + a + a b + c ; c a + b < ( 방법 ) 일반성을잃지않고 a b c 라하자. 그럼 a b + c + b c + a + b c + a ; c a + b < a + a b + c + a + c a + b < + c 모두 보다작으므로 a + b b + b c + a + b + b c + b + c + c a + b + c = c c + b = 이제원래의문제를증명하자. a ab + b ab 이므로 따라서, 대칭적으로, abc a 3 + b 3 = abc a 3 + b 3 + abc (a + b)(a ab + b ) abc b 3 + c 3 + abc c 3 + a 3 c a + b + abc (a + b)ab = c a + b a b + c + b c + a < 양변을 abc 로나누면원문제의부등식이다. 93. 삼각형 ABC에서꼭지점 A, B, C를마주보는변의길이를각각a, b, c라하자. 삼각형 ABC의내심을 I, 내접원이변 BC와접하는점을 P, 두직선IC와 AP 의교점을 X라할때, IX 를 a, b, c에대한식으 IC 로나타내어라. 단, \C >\B 이다. ( 한국 00-8) 풀이 AI의연장선이BC와만나는점을 D라두자. 그러면 4IDC 와직선 AP 에대한메넬라우스의정리에의해 IA AD DP PC CX IX = 즉 CX IX = AD IA PC DP ( )

308 308 기하 임을알수있다. 그리고 BI가 \B의이등분선이므로 AI : DI = AB : BD. AD도 \A의이등분선이므로 ac AI : DI = AB : BD = c : b + c 그리고 이를 ( ) 에대입하면 DP = BP BD = a b + c PC = a + b c CX IX (a + b c)(a + b + c) = ac + ab b + c IX IC = + CX IX ac b + c = ac + ab b + c a(a +3b c) 따라서구하려는값은 ac + ab b + c a(a +3b c) 이된다. } 94. 사면체 ABCD의무게중심을 S라하자. S를지나는어떤직선이점 K와 L에서사면체 ABCD의표면과만난다. 3 KS LS 3 임을증명하여라. ( 오폴 00-3) 증명 H I B M A S J C L K D F E 위그림에서의점 K, L 은문제의점 K, L 과관련이없는것으로하자. 위그림에서 (A; S; J; E), (B;S;K; F), (C; S; L; H), (D; S; M; I) 는각각일직선상에있는점들이다. 먼저사면체 ABCD 의표면의임의의점 X 에대해, \XS : SY =:3 이고, 세점 X, S, Y 는이순서로한직선위에있다." 를만족하는점 Y 의자취는점 S 에대한대칭확대변형이므로사면체 EFHI 의표면이된다. 비슷하게, \XS : SZ =3: 이고, 세점 X, S, Z 는이순서로한직선위에있다."

309 3. 기하고급문제 309 를만족하는점 Z의자취는사면체 JKLM의표면이된다. 사면체 ABCD의표면은 JKLM의표면과 EFHI의표면사이에끼어있으므로 ( 즉, XW가문제의선분 KL이라하면 W 는 Y 와 Z 사이에끼어있으므로 ), 3 = ZS XS WS µ = KS YS XS LS XS =3 이성립한다. 등호는직선 KL 이사면체 ABCD 의한중선일때성립한다. 95. 볼록사각형 ABCD 의두대각선이 E 에서만난다. U 와 H 를각각삼각형 ABE 의외심과수심이라하자. 비슷하게, V 와 K 를각각삼각형 CDE 의외심과수심이라하자. E 가직선 UK 위에있다는것과 E 가직선 VH 위에있다는것이동치임을증명하여라. ( 오폴 00-6) 증명 \BAC = ², 그여각을 90 ± \BAC = ± 이라하면, 외심과수심의성질로부터다음이얻어지는것은잘알려져있다. \UEB =90 ± \BUE =90± ²= ± \HEA =90 ± ²= ± 즉, \UEB = \HEA = ±; 마찬가지로 \VEC = \KED = 가된다. UEK 가일직선이된다는것과 HEV 가일직선이된다는것은각각 \UEB = \KED 그리고 \VEC = \HEA 에동치가된다. 둘다 ± = 로 4 개의각이모두같다는것을의미하므로이둘은서로동치이다. 96. 구면 s와이구면과만나지않는평면 ¼가주어져있다. ¼에서한직선위에있지않은세점A, B, C를고르자. 이세점각각에대해, 그점을지나고s에접하는접선을하나씩만들어접점을각각 K, L, M이라하자. 그리고 s 위의점 P 는 AK AP = BL BP = CM CP 를만족한다. 삼각뿔 ABCP 에외접하는구면은 s 에접함을보여라. ( 폴란드 00 3 차 -5) 증명 ( 미국코넬대학교수학과김현규 ) 먼저다음사실을확인해두자 : 점 T 를지나는두직선이한구면과만나는점들을각각 X; Y ; Z; W 라하면TX TY = TZ TW 를만족한다. 이것은두직선에의해결정되는평면이구면과만나원을이루고, 위의모든점이이원위에서이야기되는것임을생각하면알수있다. 한직선이접선일때에도평면에서와마찬가지로 TX TY = TZ 의식이성립한다. 직선 AP, BP, CP가구면s와다시만나는점을각각 D, E, F 라하자. 그럼 AK = AP AD 이고, 다시쓰면 AK AP = AD 가된다. 즉, 문제의조건에의해 AP AD AP = BE BP = CF CP

310 30 기하 이된다. 이식은, 여기에 을더하거나혹은빼어 PD PA = PE PB = PF PC 와동치임을알수있다. 즉, D, E, F 는 P 를중심으로 A, B, C 를비례변환 ( 점대칭이동과확대 / 축소변환을포함한개념 ) 한점들. 따라서, 삼각뿔 P -ABC 와 P -DEF 는 P 를닮음의중심으로하여닮음의위치에있다. 그럼 P -ABC 의외접구면 t 와 P -DEF 의외접구면 s 도역시 P 를닮음의중심으로하여닮음의위치에있다. 두원은모두 P 를지나므로, 두원의중심과 P 가한직선위에있고, 이로부터 t 와 s 는 P 에서접함을알수있다. 97. 삼각형 ABC 의외부에두직사각형 ACP Q 와 BKLC 를그렸다. 이두직사각형의넓이가같다면, PL 의중점과점 C, 그리고삼각형 ABC 의외심이한직선위에있음을증명하여라. ( 폴란드 00 3 차 -) 증명 (KAIST 과학영재센터연구원고봉균 ) P 를지나CL과평행한직선을그리고, 이것이직선 OC와만나는점을 U라하자. 또직선OC가 PL과만나는점을 T 라하고, O에서변 AC, BC에내린수선의발을각각Y, Z라하자. A Q B K O Z Y C L T P U \TCL+ \OCZ = \TCP + \OCY = \R 이므로, \PUC = \TCL = \COZ = \COB = \A \PCU = \TCP = \COY = \COA = \B 따라서, 4PUC»4CAB 로닮았음을알수있다. 또, 두직사각형의넓이가같으므로 CA CP = CB CL. 그럼 PU : PC = CA : CB = CL : CP 따라서, PU = CL 이다. 즉, PU 와 CL 은서로평행하고길이도같으므로 PULC 는평행사변형이고, 그두대각선의교점 T 는대각선 PL 의중점이다. 98. 삼각형 ABC 가다음을만족한다고하자. µ cot A µ + cot B µ + 3cot C = µ 6s 7r 여기서 s 와 r 은각각둘레의절반과내접원의반지름이다. 이때, 삼각형 ABC 는세변의길이가모두서로소이면서자연수인어떤삼각형 T 와닮음임을증명하여라. 그리고, T 의세변의길이도결정하여라. ( 미국 00-)

311 3. 기하고급문제 3 풀이내접원이변 BC, CA, AB 와접하는점을각각 D, E, F 라하고, AE = AF = x, BF = BD = y, CD = CE = z 라하자. 그럼 s = x + y + z 이고 cot A = x r ; cot B = y r ; cot C = z r 이다. 이를준식에대입하고양변에 r 을곱하면다음의식이된다. x +4y +9z = 36 (x + y + z) 49 다시정리하면, 즉 3x + 60y +405z 7(xy + yz + zx) =0; (3x y) +(4y 9z) +(8z x) =0 이된다. x =4y =9z 이므로 x : y : z =36:9:4. 따라서, 삼각형 ABC 의세변의길이의비는 a : b : c = y + z : z + x : x + y =3:40:45 로, 세변의길이가각각 3, 40, 45 인삼각형 T 와닮았다. } 주 3x + 60y +405z =7(xy + yz + zx) 의식에서 x : y : z 는 Cauchy-Schwarz 부등식 ( )(x +(y) +(3z) ) (6 x +3 y + 3z) 에서의등호조건으로부터얻을수도있다. 99. 중심이 O 인원 의한지름을 BC 라하자. 위의한점 A 가 0 ± < \AOB < 0 ± 를만족한다. C 를포함하지않는쪽의호 AB 의중점을 D 라하자. O 를지나고 DA 에평행한직선이직선 AC 와 J 에서만난다. OA 의수직이등분선이원 와 E, F 에서만난다. FJ 의연장선이원 와 G 에서만난다고하면 4EJG 는정삼각형임을보여라. (ML 프로포절 8-, IMO 00 변형 ) 증명 ( 서울백운중 3 학년고휘석 ) 먼저다음의보조정리를확인하자. 보조정리 4ABC 의내심을 I 라고하고 AI 의연장선과 4ABC 의외접원 O 의교점을 K 라할때 BK = KI = KC 가성립한다.

312 3 기하 증명 A I B C K \A = 라고하고 \B = 라고하면, \BIK = \ABI + \BAI = + \IBK = \IBC + \KBC = \B + \KAC = + 이므로 \BIK = \IBK. 따라서 BK = IK. 마찬가지로 IK = KC. 이제본문제를증명해보자. E D A F J B O C G \EOF =\EGF =\EGJ =0 ± 이므로 \EGJ =60 ± 임을알수있다. 원 O 와 4CEF, 내심 J 에대해서만보면위의보조정리에의해서 EG = JG = CG 가성립한다. 4EGJ 에서 \EGJ =60 ± 이고 EG = GJ 이므로 4EGJ 는정삼각형이다. 00. 세점 A, B, C 와 \PQR 이주어져있다. 두점 A, B 를지나는원중에서, 점 C 에서이원에그은두접선이이루는각의크기가 \PQR 이되는원을작도하여라. ( 셈본중등고급도전문제 4..5) 관찰두점은한원을결정하지못하므로, 주어진두점 A, B를지나는원은많이작도할수있다. 그러면, 주어진두점을지나고반지름이 R인원은작도할수있을까? 풀이 () \PQR의이등분선을그리고이등분선위의임의의점 X에서반직선 QR 위로수선을내린다. 그리고반직선과의교점을 D라고할때, 선분 QX와선분XD의길이의비를알아낸다. () 위에서알아낸길이의비율로점 C와점A에대한아폴로니우스의원을그리고, 점 C와점B에대한아폴로니우스의원을그려서두원의교점 O를찾는다. (3) 이교점O가바로구하려는원의중심이다. O를중심으로하고A를지나는원을그리자. C에서이원에접선을그리고접점을 T 라하면, XD QX = OT CO = OA CO = OB CO 가성립하므로, 두아폴로니우스원의교점이바로구하려는원의중심이다. }

313 3. 기하고급문제 주어진원 O 의내부에두점 A, B 가있다. A 와 B 를지나고이원에내접하는원을작도하여라. (003 KAIST Cyber 영재교육겨울캠프평가시험 ) 풀이내접원과원 O의접점을 R이라하자. R만찾아주면 ABR의외접원을그리는것으로원하는작도가끝난다. R에서의접선과 AB의연장선 l이만나는교점을 C라하고, l과원o가만나는교점을 C에서가까운것부터 P, Q라하자. R 을찾기에앞서먼저 C 를찾자. A, B, P, Q 는문제에주어진점이므로, CP CQ = CA CB(= CR ) 의관계로부터 C 의위치를알아낼수있겠다. CP = x 로두면 x(x + PQ)=(x + PA)(x + PB) (PQ PA PB)x = PA PB 의식이성립하게된다. PA, PB, PQ 는모두알고있는길이들이므로이로부터 x 를구해내면된다. 즉, ZX = PA, ZY = PB 가되도록한직선위에세점 X; Z; Y 가이순서로일직선위에있게잡고, 이직선밖에 ZF = PQ PA PB 가되게점 F 를잡자. 그래서 FXY 의외접원을그린후 ZF 의연장선이이원과만나는점을 E 라하면 ZE = x = CP 가된다. 이길이를 l 에 CP 가되도록옮겨오면 C 를알게된다. 이제 CO 를지름으로하는원이원 O 와만나는교점이 R 이되고, 이것으로필요한작도과정을모두알아내었다. R 에서의접선과 l 이평행하여교점 C 가생기지않는예외적인경우는 OA = OB 인경우로, 이때에는 AB 의수직이등분선이원 O 와만나는교점으로 R 을쉽게찾을수있다. } 0. 내각이모두 0 ± 보다작은삼각형 ABC의내부의점 P 가 AP BC = BP CA = CP AB 를만족한다고하자. \BAC =50 ± 일때, \BPC 의크기는얼마인가? ( 한국 003 차 -J0) 풀이

314 34 기하 4AP C 를닮은도형으로 4AP 0 B 가되게 P 0 을잡고, 4AP B 를닮은도형으로 4AP 00 C 가되게 P 00 을잡자. BP 0 : AB = CP : AC 이므로 BP 0 = AB CP AC = BP 이때, \P 0 AP = \BAC =50 ± 이고 AP 0 : AP = AB : AC 이므로, 4AP 0 P 는 4ABC와닮음. 따라서, PP 0 AP BC = = BP AC 즉, 4BP 0 P 는정삼각형이다. 같은방법으로 4PCP 00 도정삼각형. 따라서, \BPC =360 ± \P 0 PB \P 00 PC (\AP P 0 + \AP P 00 ) =360 ± 60 ± 60 ± (80 ± 50 ± )=0 ± 답은 0 ± 이다. } 별해 BC, CA, AB, AP, BP, CP 를각각 a, b, c, p, q, r 이라하자. 그럼 ap = bq = cr. 4ABP» 4C 0 BC 되게 C 0 을 \ABC 안에잡자. 그럼 CC 0 = ap q = b. 또 \ABC0 = \PBC 이고 c : q = x : a 이므로 4ABC 0»4PBC. 그럼 AC 0 = cr q = b. 따라서, 4ACC0 은정삼각형이다. 따라서, \BPC = \BAC 0 =50 ± +60 ± =0 ±. } 주 C = A 0 = B 00 로하고A 0 BC 00 이정삼각형이되도록 C 00 을잡자. 그리고, 4ABC 4A 0 B 0 C 0 4A 00 B 00 C 00 이되도록하자. 그럼육각형 ABC 00 A 00 B 0 C 0 가생기고 4A 00 B 0 C도정삼각형이된다. 그리고, 4AC 00 B 0 과 4A 00 C 0 B은모두한변의길이가 abc 인정삼각형이된다. 단, M = ap = bq = cr. M 03. \A <90 ± 인마름모 ABCD의두대각선 AC, BD의교점을M이라고하고, 선분 MC 위의점 O를 OB < OC 가되도록잡아 MA = t 라하자. 단, O 6= M 이다. 점 O를중심으로하고두점B, D를지 MO 나는원이직선 AB와만나는점을 B, X ( 직선 AB가이원과접할경우에는X = B 임 ), 직선 BC와만나는점을 B, Y 라하자. 두직선DX, DY 가선분AC와만나는점을각각 P, Q라할때, OQ OP 를 t의식으로나타내어라. ( 한국 003 최종-) 풀이

315 3. 기하고급문제 35 OB = r 이라두면할선의비례관계에의해 AX AB = AE AF =(AO + r)(ao r) =AO r 또 \EOB = \DOB = \DY B 이므로 따라서사각형 PXBO 는원에내접한다. \DXB + \EOB = \DXB + \DY B =80 ± AX AB = AP AO, 에서 AO r = AP AO AO(AO AP )=r AO OP = r 마찬가지로 CO OQ = r. 그럼 AO OP = CO OQ. 여기서 OQ OP = AO AM + MO = CO CM MO = t + t 임을구할수있다. } 04. 삼각형 ABC 의내접원이변 AB, BC, CA 와접하는점을각각 P, Q, R 이라할때, 다음부등식이성립함을보여라. ( 한국 003 최종 -4) 증명 BC PQ + CA QR + AB RP 6 먼저각각의변들은모두양수이므로산술-기하평균에의해다음과같이나타낼수있다. s BC PQ + AC QR + AB RP 3 BC CA AB 3 PQ QR RP 여기서 4ABC 의외접원의반지름을 R, 내접원의반지름을 r 이라하면, BC CA AB 4ABC = 4R PQ QR RP 4PQR = 4r () BC CA AB =4R 4ABC () PQ QR RP =4r 4PQR 이된다 (* 4PQR 의외접원은 4ABC 의내접원이다 ). 따라서, BC CA AB PQ QR RP = 4ABC 4PQR R r

316 36 기하 이성립. 한편, 4PQR = 4IPQ + 4IQR + 4IRP = r (sin A +sinb +sinc) = r ( a R + b R + = r (a + b + c) 4R 4ABC = r(a + b + c) c R ) 이므로 4ABC 4PQR R µ R r = r 이제, R 인것을증명하면된다. 이것은외심 O, 내심 I라하고, OI = d 라할때, 오일러의정리 r d = R rr 에의해R>r 이어야함에서확인된다. 따라서, BC CA AB PQ QR RP BC PQ + CA QR + AB RP = µ R 8 r 3 3p 8=6 이확인된다. 증명 \AIP, \BIQ, \CIR 을각각,, 라하자. 그럼 + + =¼ 이므로 + + = ¼ 이고 ; 0 < ; ; < ¼ 이다. 내접원의반지름을 r 이라하면 BC PQ + CA QR + AB r tan + r tan r tan + r tan r tan + r tan = + + RP r sin r sin r sin = µ tan +tan tan +tan tan +tan + + sin sin sin = µ cos + cos + cos + tan sin + tan sin + tan sin à s s! 3 3 cos cos cos + tan tan tan 3 sin sin sin à s s! = 3 3 cos cos cos + 3 cos cos cos s =3 3 cos cos cos = k 산술 - 기하평균부등식에서위와같이성립한다. 다시산술 - 기하평균과젠센부등식에의해 p 3 cos +cos +cos cos cos cos cos + + = 3 3 따라서, k 6 이되고 BC PQ + CA QR + AB RP 6 도성립한다.

317 3. 기하고급문제 중심 O 를가진원 K 가있고 K 를지나지않는직선 l 이있다. O 에서 l 에내린수선의발을 H 라하고, l 상에 H 가아닌임의의점하나를잡아 P 라하자. P 에서 K 에그은두접선의접점을각각 A, B 라하자. H 에서 PA, PB 에내린수선의발을각각 C, D 라하자. 직선 CD 와직선 HO 의교점을 M 이라하면, M 의위치는 P 의위치에관계없이일정함을보여라. (ML 프로포절 3-3) 증명 HO와평행한접선이직선 l과만나는점을 P 0 이라하고, P 0 을지나는또다른K의접선과 HO와의교점을그러한 M이라추측할수있다. l 상의 H가아닌임의의점을P라하고, 위의문제에서처럼 C, D를정하자. 그러면이제보여야할것은C, D, M이한직선위에있다는것이다! 원의중심을원점으로놓고좌표계산을통해 C M = v, D M = w, 적당한상수 a, b, k에대해서 av + bw = k(v w) 를보임으로써 v k w 를증명하면된다. 06. PAB 와 PDC 는그점의순서대로닮은삼각형이고, 꼭지점 P 에서맞꼭지각으로맞닿아있다. A 와 C 에서올린 AC 와수직인두직선이 B 와 D 에서올린 BD 에수직인두직선과각각만나는점을 X, Y 라하자. 그럼세직선 AD, BC, XY 는한점에서만남을증명하여라. (ML 프로포절 36-3) 증명그점의순서대로닮았고P가맞꼭지각이라고했으니까 \ABD = \ACD이고, 따라서 ABCD는원에내접한다. 이제 X; Y 에서직선 AD에내린수선의발을각각 F; G라하고, 직선 BC에내린수선의발을각각 I;H라하자. \ABD = \ACD = \BAC = \BDC = \XBA = y \XAB = x \CAD = \CBD = a \BCA = \BDA = b 라하자. y + = x + = ¼ ) 이므로 \YCD= ¼ \ACD = y; \YDC = ¼ \BDC = x 이다. 따라서, AX : BX =siny :sinx = DY : CY 각도를잘따져보면 \BXI = \AXF = a; \CY H = \DY G = b 임을알수있다. IX : FX = BX cos a : AX cos a = BX : AX; HY : GY = CY cos b : DY cos b = CY : DY: 따라서 IX : FX = HY : GY ( )

318 38 기하 이쯤되면거의증명이다되었다는느낌이올것이다. AD, BC 가 XY 와만나는점을각각 E, E 라하자. 직각삼각형의닮음에서 E X : E Y = FX : GY E X : E Y = IX : HY 인데, 이두비는 ( ) 로부터같음을알수있다. 따라서, E 과 E 는같은점이고, 세직선 AD, BC, XY 는한점 E = E = E 에서만난다. 07. 주어진삼각형 ABC의변BC, CA, AB 위에각각점 D, E, F 를임의로잡는다. R이삼각형ABC의외접원의반지름일때, 다음을보여라. (ML프로포절 39-3) µ (DE + EF + FD) AD + BE + AB + BC + CA CF R DE + EF + FD 증명 (KAIST 수학과 03학번김린기 ) D를고정시키고, 가최소가되는값을구하 AD 자. AD는고정되어있으므로 DE + EF + FD 가최소가되게 E, F 를잡아주면된다. 여기서다음의유명한보조정리를이용하자 ( 최단거리부등식을이용하면쉽게확인된다 ). 보조정리 A가예각이면 D를 AC, AB에대칭이동시킨점 D, D 를이어AC와 AB와만나는점을각각 E, F 라할때DE + EF + FD 가최소가된다. 이를이용하면 DE + EF + FD AD sin A 라는것을알수있다. A가둔각이라면 E = F = A 일때가최소이므로 DE + EF + FD 는 AD 이상이된다. 그러므로 A가예각이든둔각이든 가된다. 마찬가지로 DE + EF + FD BE DE + EF + FD AD sinb; sina DE + EF + FD CF sinc 가되므로, 이세식을합하면싸인법칙에의한 AB + BC + CA =R(sin A +sinb +sinc) 로부터문제가성립함을알수있다. 등식은 D, E, F 가각각 A, B, C 에서내린수선의발일때성립한다. 08. 평면위의볼록다각형 P 를그내부의대각선을모두그려작은볼록다각형들로분할했다. 다각형 P 의각변과각대각선의길이는모두유리수이다. 분할된작은다각형들의각변의길이도모두유리수임을증명하여라. ( 미국 003-) 증명볼록사변형 ABCD의경우를먼저풀자. 코사인법칙에의해유리수변을가진삼각형은코사인이유리수인각을가짐을알수있다. 그래서 cos x, cosy, 그리고 cos(x + y) 는유리수 ( 단, x, y는한꼭지점에서대각선으로나뉘어진두각 ). 코사인합의공식 cos(x + y) =cosx cos y sin x sin y 를쓰면 sin x sin y 도유리수임을알수있다.sin y = cos y 도유리수이다. 그래서,sinx= sin y 는유리수이다. 이제 따라서, AP D의넓이 = AD PDsin x CPD의넓이 = CD PDsin y AP AP D의넓이 = PC CPD의넓이 = ³ sin x sin y ³ AD = 유리수 CD 그러나 AP + PC 는유리수, 그래서 AP 는유리수. 다른길이들도마찬가지로성립한다. 이제일반적인경우를풀자. 대각선 A i A j 를생각하자. 이대각선들의교점을순서대로 P 0 = A i ; P ; P ; :::; P m = A j 라하자. P k 가 A i A j 와 A ra s 의교점이라면 4 변형 A i A ra j A s 에서 P 0 P k (= A i P k ) 가유리수임을유추할수있다. 이에의하여 P h P k = P 0 P k P 0 P h 로모두유리수이다. 따라서작은다각형들의각변의길이는모두유리수이다.

319 3. 기하고급문제 판지를잘라만든한변의길이가 a 인정사각형 ABCD 가있다. 역시거리 a 만큼떨어진두평행한직선 ` 과 ` 가한평면위에있다. 정사각형 ABCD 를이평면에놓는데, 변 AB 와 AD 가 ` 과각각 E 와 F 에서만난다. 또한, 변 CB 와 CD 가 ` 와각각 G 와 H 에서만난다. 4AEF 와 4CGH 의둘레의길이를각각 m 과 m 라하자. 정사각형에어떻게놓이든상관없이, m + m 의값은일정함을증명하여라. (APMO 003-) 증명 ( 제주과학고 학년윤혜원 ) ABCD 와 l, l 를다음과같이좌표축에두자. AF = n, AE = m 이라하면 (0 n; m a), DF = a n 임을알수있다. 따라서직선 l 의방정식은 y = n x +(a n) m 이다. l k l 이므로 4AEF 와 4CHG 는닮음이고, CH = kn, CG = km 이라할수있다. 점 G(a; kn) 와 l 의거리 = n a +(a n) kn m r = a n m + 이므로, 이된다. 이제 +k = a mn (n + m p n + m ) m + m = n + m + p n + m + k(n + m + p n + m ) =(+k)(n + m + p n + m ) = a mn [(n + m) (n + m )] = a 이므로 m + m 는 l, l 의위치에관계없이 a 로일정하다. 0. 삼각형의세변의길이를 a, b, c 라하고외접원의반지름을 R 이라할때, 다음을증명하여라. ( 북유럽 004-4) ab + bc + ca R 증명 ab = c a+b+c 4RS 등을이용해정리하면문제의식은 4S R 과동치. 더정리하면 (a+b+c)r 4S =r(a + b + c), 즉 R r 의오일러의삼각형공식과동치.. D, E는각각삼각형 ABC의변BC, CA 위의점이고 BD = AE 를만족한다. 선분 AD와 BE의교점을 P 라하자. 각 C의이등분선이선분 AD, BE와각각점Q, R에서만날때, PQ AD = PR 임을증명하 BE 여라. ( 폴란드 004 차-5)

320 30 기하 증명 ADKE가평행사변형이되는점 K를잡자. AE = DK = DB 이고 \QP R = \BEK, \PQR = C + \DAE = C + \DKE = \BKE. 고로 4PQR»4EBK.. 한원에외접하고모든내각과외각의크기가 60 ± 이상인사각형 ABCD 가있다. 3 jab3 AD 3 j jbc 3 CD 3 j 3jAB 3 AD 3 j 가성립함을증명하고등호가성립할조건을밝혀라. ( 미국 004-) 증명양쪽부등식은서로대칭적인관계이므로오른쪽부등식만보이겠다. jab ADj = jbc CDj 이므로 3(AB + AB AD + AD ) BC + BC CD + CD ( ) 만보이면충분하다 ( 양변의식이항상양수이므로절대값도벗겨내었다 ). cosine 제 법칙에서 AB + AD AB AD cos \A = BC + CD BC CD cos \C (= BD ) 그럼, 문제의조건에서 60 ± \A; \C 0 ±, 즉 j cos \Aj; j cos \Cj 이므로, AB + AD + AB AD BC + CD BC CD 를얻는다. 이제 BC + CD BC CD ( 산술 - 기하 ) 를함께이용하면 3(AB + AB AD + AD ) 3BC +3CD 3BC CD BC + CD +4BC CD 3BC CD 로 ( ) 이성립함을확인할수있다. 3. 예각삼각형 ABC 의외심과수심을각각 O 와 H 라하자. 세삼각형 AOH, BOH, COH 중어느하나의넓이는나머지둘의넓이의합과같음을증명하여라. (APMO 004-) 증명 ( 명지외국어고 학년이민희 ( 수정됨 )) 일반성을잃지않고, 직선 OH가변AB, AC와만난다고하자. B, M, C에서직선 OH에내린수선의발을각각 D, E, F 라하자. 그럼 EM은 DB와 FC의평균길이가되므로 j4ohbj + j4ohcj =j4ohmj A 에서직선 OH 에내린수선의발을 H 0 이라하자. OM 과 AH 는모두 BC 에수직이므로서로평행하고, 따라서 4EOM»4H 0 HA. 이제평행사변형 HBXC 를그리면 M 이대각선 BC 의중점이므로대각선 HX 의중점이기도하고, BX k HC? AB 이므로 AX 는외접원의중심 O 를지난다. 따라서, 닯음비 OM : HA =:, 즉 EM : H 0 A =: 가된다. 따라서, j4ohaj =j4ohmj = j4ohbj + j4ohcj 가되고, 일반적으로는셋중가장큰삼각형이뒤바뀔수있다.

321 3. 기하고급문제 3 4. 삼각형 ABC의외접원은중심 O와반지름R을갖고, 내접원은중심 I와반지름 r을갖는다. O 6= I 이고, 이삼각형의무게중심을 G라하자. IG? BC 일때, 또그때만, b = c 또는 b + c =3a 임을증명하여라. ( 루마니아 005 지역예선 y9-) 증명 (Mathlinks.ro 의 yptsoi, 홍콩 ) 일반적으로확인할수있는다음의사실을이용하자. XY? ZW () XZ + YW = XW + YZ 문제는 GI? BC 의필요충분조건을구하는것이므로, GB + IC = GC + IB 와같다. GB와 GC는 Pappus의중선정리에의해 a, b, c로부터계산할수있고, 꼭지점에서내접원의접점까지의거리를 x, y, z로할때, 직각삼각형을관찰하여 IC = r + z 으로계산할수있으므로, a +c b µ a + b c + r + = a +b c µ a + c b + r 와같다. 이식을간단히하면 (b c)(b + c 3a) =0 이된다. 따라서,`GI? BC' 는 `b = c 또는 b + c =3a' 와필요충분이다. 5. () 어느내각에대해서도그이등분선이항상대각선을하나포함하여지나는볼록오각형이존재하는가? () 어느내각에대해서도그이등분선이항상대각선을하나포함하여지나는볼록칠각형이존재하는가? ( 벨로루시 005 최종 -A8/B8) 풀이 ( 배정우 )() 위그림에서직선 DP 가꼭지점 D 의각의이등분선이라면오각형 AQP CD 는모든내각에서각의이등분선이대각선과일치하는볼록오각형이다.( 나머지점에서는원과접선사이의관계에의해각의이등분선이대각선이된다. 여기서각 A 가각 C 보다크거나같다고가정한다.)

322 3 기하 C 를중심으로하는원이고정되어있고점 A 의위치도고정되어있는상태에서점 A 를중심으로하는원의반지름의크기만변한다고하자. 이렇게변하는원을 R 이라하면점 D 와 B 가두원의외접선을지나게그릴수있다 ( 증명은두원의중심간의거리와한쪽원의반지름이일정하다는걸이용해서할수있다 ). 이때원 R 이밑에원보다작다면꼭지점 D 에서의각의이등분선인선분 DP 에서 P 점은위의첫번째그림과같이외접선안쪽에서존재하게된다. 만약 R 이더커져서밑에원과반지름이같게되면위의두번째그림과같이 P 는외접선 L 의바깥에서점 B 와일치하게된다. 위의첫번째그림에서 R 이조금더커지면 P 와 B 가외접선 L 을중심으로서로반대의위치에존재하게할수있다. 그런데 R 의반지름이연속적으로변하므로 AD 의길이와각 D 의크기도연속적으로변한다. 따라서점 P 도선분 CB 위를연속적으로움직인다. 따라서젤위에있는그림과같이점 B 가외접선 L 의바깥에있으면서점 P 가외접선 L 위에놓이도록만들수있다. 따라서모든꼭지점에서각의이등분선이대각선과일치하는볼록오각형이존재한다.( 단, 각 A 가 0 도보다작으면두원의크기가같을때점 B 가외접선 L 의안에위치하므로존재하지않는다. 따라서각 A 가 0 도보다는커야한다.) () 볼록칠각형의경우는위의볼록오각형의경우와비슷한데우선점 P 가외접선 L 의바깥에있을때직선 DP 와외접선 L 의교점 G 를잡는다. 그리고점 G 로부터직선 CB 방향으로점 A 를중심으로하는원의외접선을긋고이직선과직선 CB 와의교점을 H 라한다. 마지막으로점 G 의위쪽방향으로외접선 L 위에점 F 를적당히잡고이점에서직선 AB 방향으로점 C 를중심으로하는원에외접선을그으면교점 E 가나타난다 ( 점 E 와 F 를잡는방법은무수히많다 ). 그러면칠각형 AEF GHCD 는모든내각에서각의이등분선이대각선이되는볼록칠각형이된다. 아래그림은점 H 부근의확대장면이다. 결론이와같은방식으로하면 5 이상의홀수다각형에대해서항상만들수있다. 짝수다각형은정 N 각형일때항상존재하므로 ( 반드시정 N 각형일필요는없음 )4 각형이상의모든다각형에대해서모든내각의이등분선이대각선인볼록다각형을만들수있다. } 6. ABC 는예각삼각형이고, P 와 Q 는변 BC 위의두점이다. 다음조건을만족하도록점 C 을작도하자 : AP BC 은원에내접하는볼록사각형이고, QC k CA 이며, C 과 Q 는직선 AB 에대해서로반대쪽영역에있다. 또다음조건을만족하도록점 B 을작도하자 : AP CB 은원에내접하는사각형이고, QB k BA 이며, B 과 Q 는직선 AC 에대해서로반대쪽영역에있다. B, C, P, Q 가한원위의네점임을증명하여라. ( 미국 005-3)

323 3. 기하고급문제 33 증명 ( 대전대덕중 3학년고기혁 ) 조건에맞게그림을그리되, C A의연장선과 4AP C의외접원이만나는교점 B 를 B 대신잡자. 여기서 B Q k AB 임을증명하면, B = B 이된다. 원주각과동위각으로부터 \C QP = \ACP = \AB P (=\C B P ) 이므로 C PQB 는원에내접하는사각형이다. 그럼내대각과원주각으로부터 \B QC = \AC P = \ABP 이므로동위각으로 B Q k AB 임을알수있다. 따라서, B 는 B 과동일한점이고, PQB C 은원에내접하는사각형이다. 7. 삼각형 ABC 의변 AB 와 AC 에각각점 M 과 N 이있고 MB = BC = CN 을만족한다. 삼각형 ABC 의외접원과내접원의반지름을각각 R 과 r 이라하자. 비 MN=BC 를 R 과 r 에대한식으로나타내어라. (APMO 005-5) 풀이 ( 서울영일고 학년김글빛, 수정됨 ) 외심을 O, 내심을 I, 각 B 의이등분선이원과다시만나는점을 E 라하고, P = BE\CM, Q = OE\CA 라하자. 이때, 잘알려져있는대로 EA = EC = EI ( ) 가성립한다. 이등변삼각형의꼭지각의이등분선은밑변과직교하므로 OE? AC, BE? MC 이고, 그럼 CPQE 는원에내접하는사각형이다. 여기서원주각으로 \PCQ = \PEQ, 즉 \MCN = \IEO () 가된다. 한편, \EOC = \AOC = \ABC 이므로 4MBC 와 4OCE 는닮은이등변삼각형이고, 따라서, MC : CN = MC : CB = CE : EO = IE : EO ()

324 34 기하 가성립한다.(),() 에서 4MCN»4IEO 임을알수있고, 그럼 MN BC = MN NC = OI p r R OE = Rr = r R R 이다. 여기서쓰인 OI = R Rr 은오일러의공식으로잘알려진것이다. } 주 ( ) 의증명은다음과같이할수있다 : \B =, \C = 라하면우선 \ABE = \EBC = 로원주각이같으므로 EA = EC 이다. 또, \ACE = \ABE = 임에서 \EIC = + = \ECI 이므로 EI = EC 이다. 8. 예각삼각형 ABC의수심H를지나는한직선이변 AC, BC와각각점D, E에서만난다. H를지나 DE에수직인직선이직선 AB와점F에서만날때, DH HE = AF 임을증명하여라. FB ( 폴란드 005/006 차-6) 증명 CK를높이라하고, A를지나DE에평행인직선이 CK, BC와만나는점을각각 I, G라하자. 그럼 I는 4AHF 의수심. 수심의성질과내대각에서 \AF I = \AHK = \B. 즉 IF k BG 이고 DH=HE = AI=IG = AF=F B. 9. A 와 B 는평면위의두점이고, 직선 AB 로나뉘어진한쪽영역에반직선 a, a, a 3 ; b, b, b 3 들을잡는다. 앞의셋은 A 에서출발하며, AB 와이루는각이작은것부터쓴것이고, 뒤의셋은 B 에서출발하며, 역시같은순서로쓴것이다. i 6= j 에대해 a i 와 b j 가만나는모든교점들을 P ;P ;:::;P 6 이라하고이여섯점이모두서로다른점이라고하자. P i 들이한원위에있으면 ABP i 들이모두닮은삼각형인가? a i 들과 b i 들이 AB 의수직이등분선에대해대칭이라면어떤가? (ML 프로포절 74-6) 풀이 ( 양준호 ) 명제 P i 들이한원위에있을때 4ABP i 가모두닮음일필요충분조건은 a i 와 b i 들이 AB의수직이등분선에대해대칭이다. 증명 i) 4P AB 4AP 3 B 이므로 4P 3 AB 4BAP 4 이므로, 연립시 AB = BP 3 BP AB = AP 4 AP 3 BP 3 BP = AP 4 AP 3 3 점 A; B 에서원에그은한선의길이를각각 a; b 라고하면 3 에의해 a = b ) AO r = BO r ( 단, O 는원의중심, r 은원의반지름 )

325 3. 기하고급문제 35 ) AO = BO ) AB 의수직이등분선은원의중심을지난다. ) AB 의수직이등분선은원의대칭축이다. 이때원위의한점고정점 X 에대하여 4XAB 4YAB 인원위의점 Y 는유일하게하나 ) AB 의수직이등분선에대한대칭점이다. 따라서 P 과 P P 6 와 P 3 P 5 와 P 4 는 AB 의수직이등분에대해대칭이고이에따라 a i 와 b i 들도대칭임은자명하다. ii) P AB; P 5 AB; P 3 AB가닮음임을보이면충분. \P 4 AB = \P 5 BA = \P 4 P 5 B = \AP B ) 4P AB 4BAP 5 (AA) \AP 3 B = \P 4 P 5 P = \P 5 AB = \P 4 BA )4P 3 AB 4BAP 4 4P 3 AB 4BAP 5 ) 6 개모두닮음! 이제문제로돌아가자! (i) a i 들과 b i 들이수직이등분선에대칭 )4ABP i 들은닮음. (ii) a i 들과 b i 들이수직이등분선에대칭이아님 ) 모두닮음인삼각형이아니다. ) 증명끝! } 0. 예각삼각형 ABC에서 O는외심이고, CD는높이, E는변AB 위의한점, M은 CE의중점이다. M에서 LM 올린 OM의수선이직선 AC, BC와각각점K, L에서만난다. MK = AD 임을증명하여라. DB ( 폴란드 007 3차-) 증명 AC, BC의중점을각각 X, Y. OMKX, OMY L cyclic. 4OMK»4ADC, 4OML» 4BDC. 고로, LM : MK = LM OM : MK OM = CD DB : CD AD = DB : AD.

326 36 기하

327 제 4 장 조합 4. 조합중급문제. 양수 a; a;:::;(n )a 들중에서, 적당한정수와의차이가많아야 =n 인것이하나있음을증명하여라. ( 헝가리 98-) 증명구간 [0; ) 을길이 n 의소구간들로 n등분했을때맨앞구간이나맨뒤구간에속하는수가있으면끝. 그렇지않은경우에는남은구간이 n 개인데위의수들은 n 개로더많으므로같은구간에속하는두수가있다 ( 비둘기집 ). 그럼그두수의차 ( 이것도역시 ka꼴, k n ) 가적당한정수와의차이가 n 이하가된다.. () 7개의정수가임의로주어지면그중 4개를잘골라그합이4의배수가되도록할수있음을증명하여라. () `임의의 n 개의정수중에는합이 n의배수가되는 n개의정수가항상존재한다 ' 는명제를 P (n) 이라하자. P (a) 가참이고P (b) 가참이면 P (ab) 도참임을보여라. (ErdÄos-Ginzburg-Ziv 정리의쉬운단계 ) 증명 () 3개의수만있으면그중에홀짝이같은두수가반드시존재함을이용하자.7개의수중에홀짝이같은두수 (a; b) 를골라내자. 나머지 5개의수중에홀짝이같은두수 (c; d) 를골라내자. 나머지 3개의수중에홀짝이같은두수 (e; f) 를골라내자. 그럼 a + b; c + d; e + f는모두짝수이고, 세정수 a+b ; c+d ; e+f 중에홀짝이같은두수가있다.WLOG앞의두수가그렇다면 a+b + c+d 가짝수, 즉 a + b + c + d 는 4의배수. () P (b) 가참이므로 b 개의정수중에는합이 b의배수가되는 b개의정수가항상존재한다. 그럼 ab 개의정수중에그런 b개의정수의집합 X 을골라내고, 나머지 (a )b 개의정수중에서그런 b개의정수의집합 X 를골라내고,... 이렇게계속하여마지막으로 b 개의정수중에서그런 b개의정수의집합 X a 을골라내자. 집합 X i 의원소의합을 x i 라하면, x b ;:::; x a b 는 a 개의정수들이므로, P (a) 에의해이중에합이 a의배수가되는 a개의수가있다. 즉, x i + + x ia 가 a의 b 배수이므로 x i + + x ia 가 ab의배수. 따라서, ab의배수가되는 ab개의원소가존재한다. 주 P (n) 이항상참이라는것을 ErdÄos-Ginzburg-Ziv 정리라고한다.() 에의하여 n이소수일때만증명하면되는데, 이것은어렵다. 그때의증명은 MathLetter 006년 월호에실려있으니참고하기바란다. 3. 자연수들의임의의수열 a ;a ;a 3 ;::: 이주어져있다. 그럼여기서감소하지않는부분수열 a i ;a i ;a i3 ;::: ( i <i <i 3 < ) 이항상존재함을증명하여라. 풀이수열이 unbounded면 strictly increasing 수열을잡을수있고,bounded이면무한번나타나는원소가존재함. }

328 38 조합 4. 임의의자연수 n에대해 ³ n ³ n ³ n ³ n 의값을간단히하여라. 풀이 Â(n) = +( )n 이라하면 fâ(n)g n 0 은 와같이의배수이면, 아니면 0인함수이다. 이것을이용하면문제에서구하는식은 ³ n ³ n ³ n ³ n ³ n A = Â(n) 0 3 n nx ³ n = Â(n) r r=0 nx ³ n +( ) r = r r=0 = nx ³ n + r nx ³ n ( ) r r r=0 r=0 = ( + )n + ( + ( ))n = n 이된다. 답 n } 5. (a) 위원회가 40회모였는데매회 0명이참석하였다. 어떤두사람도함께모임에나간경우는최대한번뿐이다. 위원회에는적어도 60명의사람이있음을증명하여라. (b) 5명이소속된위원회에서 5명이상의소위원회를구성한다. 임의의두소위원회에공통적으로참석한사람이최대한명이라면소위원회를 30개넘게구성할수없음을증명하여라. ( 소련 965-) 증명 ( 홍문기 ) 일단다음과같은 Lemma를증명하자. Lemma p명의사람이 q명씩모인다고하자. 이때, 임의의두사람은최대한한번겹쳐서모인다고하면, 모든사람은최대 (p ) (q ) 번의모임에참석할수있다. 증명사람들을 ; ;:::;p라고하자. 일반성을잃지않고 이가장모임에많이참석했다고할수있다. 그러면 은» p 까지 p 명의사람들과함께한번의모임당 q 명을더모아서모임에참석할수있다. 이런식으로순서대로 이모임에참여한다고하자. ( 즉, 다음번모임에는 q+;q+;::: ;(q )+과함께모임에참여하고, r번째모임에는 (r )(q )+;::: ;r(q )+과함께참여한다.) 이렇게 이총t번의모임에참석하면,은; 3;::: :t(q ) + 과함께모임에참여했으므로 t(q ) + p 즉, t (p ) (q ) 이된다. (a) 위원회에총 n명이있다고가정하고, n명의사람을 ; ;:::;n이라고하자. 이때, k가나간모임에나간횟수를d(k) 라고하자. 그러면, d() + d() + :::d(n) n(n ) (0 ) 그런데 0명씩모이는모임이총 40회의모임이있었으므로, d() + d() + :::d(n) =400이다. 그러므로 3600 n(n ) ) 60 n. (b) 한사람이참여할수있는소위원회의수 (5 ) =6. ) (5 ) 각각의사람들이참여할수있는소위원회의수를모두합치면 6 5 = 50 이하가된다. 그런데, 소위원회에참석하는인원은최소 5명이므로, 최소 5번씩중복된다는것이다. 따라서소위원회 50 =30. 5 즉,30개이하의소위원회만만들어진다. 6. n 개의하얀색공과 n 개의검은색공을일렬로나열하는정렬 X 에대해, 한쪽끝에서다른쪽끝까지이동하며관찰할때색이변하는횟수를 f(x) 라하자.0<k<n 인임의의정수 k 에대해, f(x) =n k 를만족하는 X 의개수는 f(x) =n + k 를만족하는 X 의개수와같음을보여라. ( 헝가리 968-3)

329 4. 조합중급문제 39 증명같은색의덩어리들로분할. f(x) =m 이면덩어리가 m개. i번째덩어리의크기를 ai. 그 럼흰공과검은공의개수에서 a + a 3 + = n = a + a 4 +. 이런부정방정식의해의개수는 n dm=e n bm=c. m = n k 일때이값이같음을확인하면끝 과 만으로된유한수열이다음의성질을만족한다 :() 연속한다섯항으로된부분수열들은모두서로다르다.() 수열의맨처음이나맨마지막에하나의수를더적으면 () 의성질은더이상성립하지않는다. 이수열에서, 처음 4 개의항이마지막 4 개의항과일치함을보여라. ( 소련 969-3) 증명이수열을abcd xyzw 라할때, xyzw0과 xyzw은이미등장한부분열이어야하므로, xyzw가맨앞과같지않다면 ( 귀류법 ) xyzw꼴이이수열에벌써세번등장한것이되어비둘기집의원리에의해똑같은연속다섯항이존재함. 그러므로 xyzw는 abcd와같아야함. 8. 어느나라에몇개의도시가있다. 각도시는항공편으로많아야세개의다른도시와연결이되고, 어떤두도시사이에서도기껏해야한번만갈아타면항공편으로오갈수있다고한다. 이나라의도시는최대몇개인가? ( 소련 969-9) 풀이 A에연결된도시를 B, C, D( 이중에몇개없을수도 ). 이외의도시는 A에 B, C, D 중하나를통해서만간접연결됨.B,C,D각각에연결되는도시는최대 개씩뿐이므로,A에직접연결되지않은도시는최대 6개뿐임. 즉, 도시는 0개이하.0개가가능한경우가있음. 오각형 ABCDE 안에별꼴오각형을그리고, 외부의오각형과내부의오각형의꼭지점을가장가까운것끼리둘씩연결한그림. } 9. 주어진방정식 x 3 + ax + bx + c =0 에대해서, 갑과을두사람이게임을한다. 갑이먼저 a, b, c 중하나의값을결정하고, 다음을이남은둘중하나의값을결정한후, 갑이다시마지막으로남은한문자의값을결정한다. 단, 이들은모두정수라야한다. 갑의목표는이방정식이세개의 ( 서로다를필요는없는 ) 정수해를갖게하는것이고, 을의목표는이것을방해하는것이다. 누가이기겠는가? ( 소련 969-) 풀이갑이 c =0 으로하면쉬움. a + b +=0 이되도록하면승리. } 0. 주어진정 (n +) 각형에서임의로서로다른세꼭지점을택했다. 모든선택이균일한확률을갖는다면, 이렇게택한세점에의해결정되는삼각형의내부에원래의정다각형의중심이놓일확률은얼마인가? ( 미국 973-3) 풀이예각삼각형이될확률을물었는데,에서둔각삼각형이될확률을빼어주면된다 ( 홀수인정다각형이라서지름이되는경우가없으므로직각삼각형이되는경우는없다 ). 각선택이균일한확률을가지므로경우의수만세면된다. 한꼭지점이해당삼각형의둔각에서시계방향쪽의예각꼭지점이되는경우를세어보자. 그점을남극 (S) 으로보면오른쪽에둔각이있으므로나머지두꼭지점은모두오른쪽 n개의점들중에서택하는상황이된다. 즉, n 가지. S로한꼭지점을택하는방법이 n +가지있으므로, 둔각삼각형은총 (n +) n 개있다. 이것을전체의경우에서빼주면되므로구하는확률은 (n +) n = n + 4n 답 } n+ 3. 도서관에한개의입구와한개의출구가있고, 입구와출구에칠판이하나씩있다. 그리고 명이상의사람이동시에출입한적은없다고한다. 각각의사람은도서관에들어올때현재도서관에있는사람의수를입구쪽에있는칠판에적고, 나갈때도현재도서관에남아있는사람의수를출구쪽에있는칠판에적는다. 도서관이문을닫아현재도서관에아무도없다고할때, 입구쪽에있는칠판과출구쪽에있는칠판에적힌수들은배열순서를무시하면똑같아야함을증명하여라. ( 헝가리 974-) 증명도서관안의인원을사람이들어올때또는나갈때의차례에대한그래프로나타내면, 항상 씩올라가거나내려와야하고 0에서시작하여 0에서끝나므로, 그래프가올라갈때나열된수 ( 입구쪽수 ) 는내려올때나열된수 ( 출구쪽수 ) 와일대일대응하게됨.. 3 장의에이스를포함하는 n 장의카드를임의로섞어한곳에쌓는다 ( 모든가능한카드의순서가균일한확률로나타난다고가정 ). 두번째에이스가나타날때까지위에서부터한장씩차례로뒤짚는다. 뒤짚은카드의개수의기대값이 (n +)= 임을증명하여라. ( 미국 975-5)

330 330 조합 증명위에서부터한장씩뒤짚는것과밑에서부터한장씩뒤짚는것은완전히대칭적인문제이므로기대값이같고, 카드가쌓여진각각의상황마다위에서부터뒤짚은카드와아래에서부터뒤짚은카드의수의합은 n +이므로기대값은그평균인 (n +)=이다 (E = P p i n i = P p i (n +)=n +). 3. 딱정벌레가 n 각뿔의변을따라서기어가고있다. 밑면중한변의중점 A 에서출발하여다시 A 로돌아오는동안각꼭지점을많아야 번지났다. 이렇게할수있는방법은모두몇가지인가? 그리고각각의경우딱정벌레가지나간변의수를모두합하면 n 이됨을보여라. ( 독일 BW 977 차 -) 증명뿔의꼭지점에언제오르는지에따라그냥열심히세면됨. 아마도한쪽방향으로도는것만계산한듯.( 실제로는마지막제시한값의 배가나옴 ) 4. 가로 m 블럭, 세로 n 블럭으로이루어진직사각형꼴의도시가있다. 이도시의남서끝점에사는어떤여자가북동끝점에있는직장으로일하러다닌다. 그녀는매일걸어다니지만, 어떤날에도출근하는동안같은지점을두번지나지않는다.( 최단거리로갈필요는없다.) 이런서로다른출근길의가지수를 f(m; n) 이라하자. f(m; n) mn 이성립함을보여라. ( 캐나다 977-7) 증명이도시에는모두 m n블럭이있다. 출근길이하나정해지면도시는이출근길에의해좌우두영역으로분리되고, 서로다른두출근길은이런영역의분리가다른형태가된다. 즉, 출근길의가지수는도시의각블럭을좌, 우두집합으로분류하는총경우의수 mn 을넘지않는다. 5. 검은말과흰말로 A 와 B 가 3 3 체스판에서게임을한다. 규칙은다음과같다 : 가. 두사람은번갈아게임을한다. 나. 자기차례에서는말하나를체스판의빈칸중한곳에놓는다. 다. 말을놓을때는검은색이나흰색말중어느것을선택해도되고, 항상같은색의말을놓지않아도된다. 라. 판이가득찼을때,A 는각행, 각열, 각대각선마다검은말이짝수개있으면 점씩얻는다.B 는각행, 각열, 각대각선마다검은말이홀수개있으면 점씩얻는다. 마. 전체 8 점중 5 점이상을얻은사람이이긴다. 처음하는사람이반드시이기는방법을서술하여라. ( 캐나다 978-5) 풀이먼저하는사람이 A라면처음에검은말을중앙에놓는다. 그후에는항상 B가놓는곳의대칭에되는위치에B가놓은말과다른색의말을놓는다. 그럼게임이끝났을때 A는중앙을지나는 4개의선에는모두짝수개의검은말이있어서우선4점을얻고, 좌변과우변이서로색이대칭으로뒤집혀있으므로둘중한곳에서점, 또상변과하변중한곳에서 점을얻어총 6점을얻게된다. 먼저하는사람이 B라면처음에흰말을중앙에놓고같은방식으로하면된다. } 6. 자연수 n 을하나이상의자연수들의합으로나타내는방법의수를 p(n) 으로정의하자. 예를들어,4 는 +++; ++; +3; +; 4 등의다섯가지방법으로표현할수있으므로 p(4) = 5 이다. 모든 n> 에대하여 p(n +) p(n) + p(n ) 0 이성립함을증명하여라. ( 영국 979-5) 증명 q(n) =p(n) p(n ) 이라하면, q(n) 은 n을 이상의자연수들로분할하는방법의수 (을포함하는분할은그것을제외하면 n 의분할이되니까 ). q(n) 의한분할에서가장큰수를증가시키면 q(n +) 의한분할 ( 특히가장큰항이유일한분할과일대일대응함 ) 이되므로q(n +) q(n).

331 4. 조합중급문제 n 개의독립적인테스트로이루어진실험이있다. i 번째테스트가성공할확률을 p i 라하자. 정확히 k 개의테스트가성공할확률을 r k 라할때, 다음을증명하여라. nx nx p i = kr k ( 폴란드 979 3차-3) i= k=0 증명점화식. 테스트가 n +개일때k개의테스트가성공할확률을 r 0 k 라하면, rk 0 =( p n+)r k + p n+ r k.( 단, r = r n+ =0으로간주 ) P r k =임에유의하여 P krk 0 를잘풀면됨. 물론테스트가 개뿐을때는자명하게성립 명의사람들이모여있는데이들중어떻게 3 명을골라도그셋중서로모르는둘이항상존재한다고한다. 이사람들중에서로아무도모르는네명이있음을보여라. ( 영국 980-5) 증명네명이상을아는사람 A가있으면 A가아는사람들끼리는서로다모르므로 A가아는사람넷을고르면됨. 이제어떤사람도 3명이하의사람만알때만풀면됨.A가모르는6명이있고, 그 6명중에서로모르는 3명을찾으면 A와함께원하는넷이되므로그런 3명을찾아보자.6명중한명B가나머지 5명중3명을알때는그 3명이서로모르므로찾았고,B가나머지 5명중3명이상을모를때에는그3명중서로모르는둘이있으므로, 그둘과B가원하는3명이됨. 9. 한도시에 980 개의교차로가있고, 각교차로는삼거리이다. 이도시의모든교차로를딱한번씩들르는순환형버스노선이있다. 각거리마다소나무, 동백나무, 향나무중한가지를택해그거리를따라서쭉심는데, 모든교차로마다세종류의나무가모두만나도록할수있음을보여라. ( 유고슬라비아 980 고 -4) 증명버스가노선을따라지나는거리를 e ;e ;:::;e m 이라하자 (m = 980). 그리고, e i 들에는소나무를, e i+ 들에는동백나무를심자. 이노선은모든교차로를딱한번씩들렀으므로, 각교차로마다소나무와동백나무거리가하나씩만나고, 따라서남은모든거리에향나무를심으면된다. 0. 자연수 a <a < <a 9 00 과 b <b < <b 00 이있다. 이중에서 a j a i = b q b p > 0 이되는네수 a i, a j, b p, b q 를고를수있음을보여라. ( 유고슬라비아 980 고 -4) 증명 c i;j := a i + b j 로쓰자. c j;p = c i;q 인 (i; j; p; q) 가있음을보이는문제이다 (i = j와 p = q 둘중하나가성립하면둘다성립 ). 같은게없다면 c i;j 는 9 = 399가지인데, c i;j 400 의범위에도 399개의수뿐이므로모두한번씩나타나야함. 그럼 a = b =,a 9 = b =00이어야하는데, c 9; = c ; 이되어그때도모순.. 길이가 x ;x ;:::;x 8 인 8 개의선분이주어져있다. 단, x x x 이다. 이들중세선분을골라서그것으로삼각형을만들수있음을보여라. ( 유고슬라비아 980 고 3-) 증명귀류법으로풀자. 그럼 xi x i +x i. x = f, x = f, x i x i +x i f i +f i = f i 로항상 x n f n 이성립. 그런데 x 8 f 8 =584 이므로모순.. 에서 50 까지쓰여진카드가있다. 카드를잘섞은후각행에 0 장씩 5 행을배열한다. 카드를각행마다왼쪽에서오른쪽으로증가되도록재배열한다. 다시각열마다위에서아래로증가하도록재배열한다. 이렇게얻은배열에서각행마다여전히카드의수가왼쪽에서오른쪽으로증가하는가? ( 캐나다 980-) 풀이여전히왼쪽에서오른쪽으로증가하도록배열된다. 일반적으로 m행 n열일때에도이문제는항상성립한다. 이것을증명하자. 열이 개일때만증명하면된다. 그럼 열보다 열이크고,열보다 3열이크고, :::; 이렇게모두크기순서가잘되어있음이확인되기때문이다. a b a b.. a m b m

332 33 조합 재배열이후의왼쪽열의수를차례로 a ;a ;:::;a m, 오른쪽열의수를차례로 b ;b ;:::;b m 이라하자. 이제임의의 k 에대해 a k <b k 임을보이면된다. 오른쪽열에서 b k 보다작거나같은수는 b ;:::;b k 의 k 개가있다. 이들은원래재배열되기전에그보다왼쪽에더작은수가있었으므로, a i 들중에 b k 보다작은수가적어도 k 개있다. 즉, 왼쪽열에서가장작은 k 개의수 a ;:::;a k 는 b k 보다작다. 따라서, a k <b k. 이로써원하던명제가증명되었다. } 3. r 과 s 를자연수라하자. 다음식 3 r 7 s =lcm(a; b; c) =lcm(a; b; d) =lcm(a; c; d) =lcm(b; c; d) 을만족시키는자연수해순서쌍 (a; b; c; d) 의개수를 r 과 s 에대한식으로나타내어라. (Putnam 980-A) 풀이먼저 a, b, c, d의소인수는 3과 7뿐임을알수있다. 그리고, 서로다른소인수 3과 7은각각독립적으로생각할수있다. a, b, c, d의 3의지수를각각A, B, C, D라하자. 이때다음이성립하여야한다. max(a; B; C) =max(a; B; D) =max(a; C; D) =max(b; C; D) =r 이식에서 A, B, C, D 중적어도둘은 r 이어야하며, 둘만 r 이면항상이식이성립한다. 순서쌍의개수를몇가지경우를나누어서셀수도있으나, 잘생각해보면이런순서쌍의개수는 (r +) 4 의전개식에서이차이하의식만을빼내면된다는것을알수있다. 즉, f0; ;rg 의 r + 식의수를 f0; ;r g 의 r 개와 frg 의 개로나누었을때 r 개의집합에서둘이하를뽑는경우가되는것이다. 따라서 (r +) 4 mod r 3 =6r +4r + 개의순서쌍이있다. 7 의경우도마찬가지로 6s +4s + 의경우가있고, 따라서순서쌍 (a; b; c; d) 의개수는 (6r +4r + )(6s +4s +) 이다. } 4. 공간에어느네점도한평면위에있지않은n개의점이주어져있다. 이점들을네꼭지점으로갖는모든사면체들의집합을 S라하자. 주어진점들을하나도지나지않는한평면이 S에포함되는사면체중에사각형의단면으로자르고지나가는것은 n (n ) 개보다많을수없음을보여라. ( 영국 98-4) 64 증명사각형의단면으로자르고지나간다는것은꼭지점을그평면의양쪽에 :로둔다는것이다 ( 삼각형단면은 3:). n개의점이이평면의양쪽에 k : n k로나뉘어놓여있다고하자. 그럼양쪽에서 개씩골라만든사각형의개수를원하는것이므로 k n k k(k )(n k)(n k ) = ( k+(n k) ) ( (k )+(n k ) ) 4 4 이된다 (GM-AM). ³ n 과 만으로이루어진 n항으로된수열들중에서정확히 m개의 0을갖는것은개있음을보 m + 여라. ( 영국 98-6) 증명 0은 0에서 로바뀌는지점을뜻한다. 이웃한 0과 0 사이에는반드시 에서 0으로바뀌는지점이딱한번있다. 수열의맨앞과맨뒤에각각 과 0을더추가하여길이 n +의수열로생각하여 0지점과 0지점들을찾아보면, 반드시 0지점으로시작하여 0지점으로끝난다. 즉, m +곳의 0지점과 m곳의 0지점을정하는것과같으므로 (m +곳의지점을택하면그순서는저절로결정되고수열도저절로결정되므로 ) n+ m+ 이다. 6. p 를집합 S n = f; ;:::;ng 의순열이라하자. p(j) =j 인경우 j S n 를고정점이라한다. 고정점이없는순열의개수를 f(n) 이라하고, 고정점이정확히하나있는순열의개수를 g(n) 이라하자. jf(n) g(n)j = 임을보여라. ( 캐나다 98-4) 증명난순열이다음과같은점화식을갖는것은잘알려진사실이다. f(n) =(n )f(n ) + (n )f(n ) (n 3) ( ) 그리고, 고정점이하나있는순열은그고정점을하나택하고나머지는난순열을이루면되므로 g(n) =nf(n )

333 4. 조합중급문제 333 이된다. 이제 jf(n) g(n)j = 을수학적귀납법으로보이자. 먼저 n =, 일때는 jf() g()j = j0 j =,jf() g()j = j 0j = 로성립. n = k 일때성립한다고가정하자. 그럼 jf(k +) g(k +)j = jkf(k)+kf(k ) (k +)f(k)j = jg(k) f(k)j = 로 n = k + 일때도성립한다. 따라서, 문제가증명되었다. 특히 f(k +) g(k +)= (f(k) g(k)) 임에서 f(n) g(n) =( ) n 임도확인할수있다. 주 ( ) 의개략적인증명을소개하면다음과같다. p(n) =i 라하자. i를택하는n 가지가모두대칭적이므로앞으로구한결과에 n 을곱하면된다.()p(i) =n 이면나머지 n 개의수가난순열을이루어야하므로 f(n ) 가지.()p(i) 6= n 인경우는이 n을 i로간주하고모두 n 개의수가난순열을이루는경우와같으므로 f(n ) 가지. 따라서, 모두 f(n) =(n )(f(n ) + f(n )) 가지가된다. 7. 자연수집합 N 이두집합 A 와 B 로분할되어있다. 모든 n N 에대해, fa; b; a + bg 가 A 에포함되거나 B 에포함되는, n 보다큰서로다른정수 a, b 가존재함을보여라. ( 오폴 983-4) 증명 우선 n =0으로할때문제가성립함을보조정리로보인다. 그후n보다큰적당한 m을하나택한후 N m = fm; m; 3m; 4m; :::g 을생각하면이것은구조가 N과똑같으므로역시보조정리에의해성립함을알수있다. 증명 귀류법.() 둘다무한집합이다 ( 유한집합이있으면반대쪽집합에충분히큰모든자연수가다포함되므로 ). () 임의의 n<a A 에대해, a의다음항은a +또는 a +이다 ( 다음항이 b a +3 이면 a +;b B 라서 a + b 는어느쪽집합에서도처치곤란 ). 이것은 B도마찬가지이므로,(3) n보다큰어떤적절한시점부터그이후의수들만남기면 A, B는홀수 / 짝수들로분할된집합. 그럼짝수들을가진집합쪽에서모순발생. 증명3 귀류법. n보다큰수를한집합이다갖고있을수없다. 즉, a A, b B 인 a, b를찾을수있다.wlog,a + b A 라하자. 그럼 a + b B, a +b A, 3a +b B, 3a +3b A 가되고, a + b; a +b; 3a +3b A 이므로모순. 증명 4 귀류법으로풀자. n 보다큰서로다른세정수 x, y, z 를갖는집합이존재. 그것을 A 라하자. 그럼귀류법에의해 x + y; y + z;z + x B. 또 x +y + z A. 이것을 x A 와함께생각하면, x+y+z B. 이것을 x+y B 와함께생각하면 x+y+z A. 그런데이제, y; x+y+z;x+y+z A 가되므로모순. 8. k 개의자연수 a ;a ;:::;a k 들의기하평균은이들의곱에 k 제곱근을한것으로정의한다. 예를들어,3, 4, 8 의기하평균은 6 이다. n 개의자연수들의집합 S 의기하평균은공집합이아닌 S 의모든부분집합들에서기하평균을낸것들의기하평균과같음을증명하여라. ( 캐나다 983-5) 증명 집합 X 의기하평균을 G(X) 라쓰자. jxj = a, jy j = b 이고 G(X) =G(Y )=g 로같으면 G(X [ Y )=(x x a y y b ) =(a+b) =(g a g b ) =(a+b) = g 로합집합을해도기하평균은유지된다 ( ). 이사실을이용하기로하자. S 의부분집합중에서 k 개의원소를갖는것들의모임을 X k 라하자. X k 의각각의집합에서기하평균을낸것들의기하평균을생각하면, 모든원소가같은지수로균일하게들어가므로 S 의기하평균과같음이자명하다. 즉, G(X )= = G(X n )=G(S) 이다. 그럼 ( ) 에의해 G(X [ [X n )=G(S) 임을알수있다. 별해 S = fa ;a ;a 3 ;:::;a ng 이라하자. a 을포함하는크기가 k인부분집합은모두 n k 개이고, 거기서기하평균을낼때 a 의차수는 k 이다. 공집합이아닌 S의부분집합은 n 개이므로, 문제에서구하는전체기하평균에서의 a 의차수는 nx ³ n nx ³ n k k k n k= k= n = n = n n n = n 로집합 S 의기하평균을낼때의차수와같다. a 이아닌것들도마찬가지이므로, S 의모든부분집합들에서기하평균을낸것들의기하평균은 S 의기하평균과같음이증명되었다.

334 334 조합 9. 7개의자연수가임의로주어졌을때그중에서항상다음의성질을갖는 5개의수를찾을수있음을증명하여라 : 네개의수가모두나머지하나의수로나누어떨어지거나, 어떤수도다른수로나누어떨어지지않는다. (Towns 983봄 JA5) 증명 ( 육형빈 ) 다음과같이조건을구분하자. () 네개의수가모두나머지하나의수로나누어떨어진다. () 어떤수도다른수로나누어떨어지지않는다. 7개수를작은수부터나열한수열을 fa n g (n =; ; 3;:::;7) 이라고하자. 이수열을 A = fa g, B = fa ;a 3 ;a 4 ;a 5 g, C = fa 6 ;a 7 ;a 8 ;a 9 g, D = fa 0 ;a ;a ;a 3 g, E = fa 4 ;a 5 ;a 6 ;a 7 g 로분류한다. D의한원소x에대해, E의원소 ( 총 4개 ) 가모두x의배수이면조건 () 을만족하는 5개의수를찾을수있다. 그렇지않다면, x의배수가아닌수가적어도하나존재한다. 그수와x로순서쌍을만든다. 그럼총 4개의순서쌍 (a d ;a e)(a d D; a e E) 가만들어진다. C의한원소y에대해, 위순서쌍의수 ( 총 8개 ) 들중y의배수가4개이상이라면, 조건 () 을만족하는 5개의수를찾을수있다. 그렇지않다면, 배수가아닌수가 3개이하이므로, 배수가아닌수로만이뤄진순서쌍이적어도하나존재한다. 이순서쌍을이루는 개의수와 y로새로운순서쌍을만든다. 그럼이번에도총 4개의순서쌍 (a c;a d ;a e)(a c C; a d D; a e E) 이만들어진다. 이순서쌍의수는어떤것도다른수로나누어떨어지지않는다. 마찬가지방법으로 A, B의원소를포함한순서쌍 (a ;a b ;a c;a d ;a e)(a b B; a c C; a d D; a e E) 를만들수있고, 이순서쌍은조건 () 혹은조건 () 를만족한다. 위의과정으로조건을만족하는 5개의수를항상찾을수있다. 별증 ( 고봉균 ) 보다일반화한다음의문제를증명하자 : 문제 mn +개의자연수가임의로주어지면항상다음을만족함을증명하여라 : () 하나가다른 m개의수의약수인 m +개의수가있거나 () 어떤두수도서로약수배수관계가아닌n +개의수가있다. 원래의문제는 m = n =4인경우이다. n에대한수학적귀납법으로증명하자. 먼저 n =일때는서로약수배수가아닌두수가있으면 () 가만족되므로그렇지않으면모든쌍이서로약수배수관계에있으므로가장작은수가다른모든수의약수가되어 () 이만족된다. n 일때성립한다고가정하고 n일때성립함을보이자. mn +개의수중가장작은것을a라하자. 나머지 mn개의수중에a의배수가m개이상있으면전자의조건으로성립하므로, a의배수가 (a를포함하여 ) m개이하있는경우만확인하면된다. 이 a의배수를모두제외한집합을 A라하면, jaj m(n ) + 이다. 그럼귀납법의가정에의해 A에는 (i) 하나가다른 m개의수의약수인 m +개의수가있거나 (ii) 어떤두수도서로약수배수관계가아닌 (n ) + = n개의수가있다. (i) 의경우는바로 () 이만족되고,(ii) 의경우에는이n개의수중어떤것도 a의배수가아니므로 a를포함하여 n +개의수가 () 를만족하게된다. 따라서, n일때도성립하고, 일반화된문제가수학적귀납법으로증명되었다. 주이것은 mn +개의항으로된수열에는길이 m +의증가하는부분수열이있거나길이 n +의감소하는부분수열이있다는유명한 ErdÄos-Szekeres 정리의응용에해당한다. 30. 무도회장에 n 명의여자와 n 명의남자들이각각한줄로서서서로마주보고있다 ( 즉,n 명의사람들이 n 으로서있다 ). 모든사람들이자신의왼손을각각왼쪽, 맞은편, 또는맞은편왼쪽에있는사람과잡고있다. 같은방식의규칙이오른손에도적용된다. 두손을같은사람과잡고있는사람은아무도없다고할때, 이런식으로손을잡을수있는방법은몇가지일까? ( 오폴 984-6) 풀이여자들을차례로 G ;G ;:::;G n, 남자들을차례로 B ;B ;:::;B n 이라하고, 첨자가낮은쪽을동쪽, 높은쪽을서쪽이라하자. G, B 은동쪽손을잡을사람이상대방뿐이므로서로그손을맞잡아야한다. n쌍일때의방법의수를 a n 이라하자. G 이서쪽손을G 의동쪽손과잡았다면B 은서쪽손을 B 의동쪽손과잡을수밖에없다 (G, G 와는잡을수없으므로 ). 또, G 이서쪽손을B 의동쪽손과잡았다면 B 은서쪽손을G 의동쪽손과잡을수밖에없다. 즉,(i) G G B B 이거나 (ii) B G B G 이고, 이두상황은아직결정되지않은손들의가능성이똑같으므로한가지만살펴보고두배를하면된다. 우선 a =0,a =이고 n 3 일때를보자.()

335 4. 조합중급문제 335 G B 인경우 : G ;G ;B ;B 는모든손을사용했으므로나머지 n 쌍이처음부터다시시작하는것과같다. 즉, a n 가지. () G = B 인경우 : G ;B 을없다생각하고 G ;B 과잡았던손을 G B 로바꿔잡게하고생각하면 G ;B 을제외한 n 쌍이손을잡는경우들에일대일대응한다. 즉, a n 가지. 따라서, a n =(a n + a n 의점화식이성립한다. 그럼일반항을선형동차점화식으로구하면끝. } 3. 음이아닌두정수 n m 에대하여, 다음을증명하여라. ³ n ³ n ³ n + +3 m m m ³ m + +(n + m) = m ³ n + m + ³ r r(r )(r ) (r s +) 단, 는이항계수 s s! 을나타낸다. ( 영국 985-3) 증명 항등식 m m + + n m = n+ m+ 을여러번 (triangular한꼴이차원적으로) 더해서얻은식 임. 3. 파티에 n 명의사람 P ;P ;:::;P n 이참가하였다. P ;P ;:::;P n 6 은각각 4; 5;:::;n 3 명씩의사람을알고있고, P n 5 ;P n 4 ;P n 3 은각각 n 명씩을, P n ;P n ;P n 은각각 n 명씩을알고있다. 단, ` 안다 ' 는것은 ` 서로알고있다 ' 는것이고, 자기자신을 ` 안다 ' 고말하지는않는다. 이것이가능한 n 8 을모두찾아라. ( 오폴 985-) 풀이 P n ;P n ;P n 은모든사람을다아니까이들을고려에서제외하자. 그럼 (P ;P ;:::;P n 6 ;X;Y;Z) 은각각 (; ;:::;n 6;n 5;n 5;n 5) 명씩을안다 (X; Y; Z는 P n 5 ;P n 4 ;P n 3 ). P 은 X; Y; Z 중에적어도둘을모르고 P 는 X; Y; Z 중에적어도한명를모른다. 그런데, X; Y; Z가모르는사람은각각한명씩모두세명이므로이들이모르는사람은 P, P 말고는없다.WLOGP 과 X가모르고, P 는 Y, Z를모른다고하자. 그럼 P 3 은 X, Y, Z를모두아니까벌써세명이차서 P n 6 은모른다. 그럼 P n 6 은 P ;P ;P 3 을모르니까많아야n 7명밖에알수없고이것은 n 6명을안다는것에모순. 즉, n 0 이면 P 3 과 P n 6 등이틀림없이별개의인물로존재하므로안된다. n =8; 9 일때는가능한경우의그림이있다. 답 8, 9 } 33. ; ;:::;n 은임의로 n 개씩두그룹으로나눌수있다. 첫번째그룹을 이라하고, 두번째그룹을 a <a <a 3 < <a n b >b >b 3 > >b n 이라하자. 이때, 다음을증명하여라. ( 소련 985) ja b j + ja b j + + ja n b n j = n 증명각각의 i = ; ;:::;n 에대해, ai 와 b i 중하나는 f; ;:::;ng 에속해있고, 다른하나는 fn+;n+;:::;ng 에속해있다. 왜냐하면, 만약 a i n, b i n 이라면 (a i, b i 가동시에 f; ;:::;ng에속해있다면 ) 문제의조건에의해 a <a < <a i n 이고 b i <b i+ < <b n n 이므로, f; ;:::;ng 이서로다른 n + 개의원소를가져야해서모순이기때문이다. 또, a i n + 이고, b i n + 일때도같은방법으로불가능하다는것을알수있다. 따라서, ja b j + ja b j + + ja n b n j =[(n +)+(n +)+ +n] [ n] =n 임이증명되었다. 34. 모든 r n 에대하여 a r <a r+ 이고모든 r n 3 에대하여 a r <a r+3 인,; ;:::;n 의순열 a ;a ;:::;a n 의개수를구하여라. ( 영국 986-5)

336 336 조합 풀이피보나치수열이됨. x =,x =는그냥자명하고점화식 x n+ = x n + x n 이성립함을보이면됨.3번째이후의항은더작은항이있으므로최소의항이될수없고, 따라서 이될수있는것은 a 과 a 뿐. a =일때는이것을던져버리고남은항을모두 씩빼어주면 n이하나작을때의경우와일대일대응함. a =일때는a =일수밖에없고 ( 나머지항들중제일작으니까 ) 이두항을던져버리고남은항을모두 씩빼어주면 n이둘작을때의경우와일대일대응함. } 35. 한변의길이가 n 인정삼각형 ABC 를변에평행한격자선을그려서한변의길이가 인 n 개의정삼각형으로분할하였다. 각각의격자점마다실수를하나씩배정하였고, 특히세꼭지점 A, B, C 에배정된수를 a, b, c 라하자. 한변을공유하는두작은정삼각형으로이루어진각각의마름모에대해, 마주보는두꼭지점의수의합이같다고한다. () 가장작은수가배정된점과가장큰수가배정된점사이의거리의가능한최소값을구하여라. () 모든격자점에배정된수의합을 a, b, c에대한식으로구하여라. ( 중국 987-) 풀이 () 임의의연속한세점을 X, Y, Z, 그위사이의두점을A, B라하면 ( 이런두점은항상있다 ), 마름모 XY BA, ZY AB에대해A + Y = X + B, Z + A = Y + B, 여기서 A, B를소거하면 Y = X + Z 가된다. 즉, 임의의연속한세점은등차수열. 그럼 3개이상의항을가진라인은항상등차수열이되고, 항이 개이하인수열은당연히등차수열이므로, 모든라인이다등차수열이다. 따라서, a, b, c가모두똑같은경우 ( 그때는거리가) 를제외하면최대값과최소값은꼭지점에있어서그거리는항상n 답 () 등차수열과등차수열을합하면역시등차수열이므로, 삼각형 ABC를 60 ± 회전시켜가며원래삼각형에더해주면세꼭지점에배정된수가모두 a + b + c 인모든라인이등차수열인삼각형이나온다. 즉, 이삼각형은모든항 ( 점 ) 이 a + b + c이다. 점의개수는 ++ +(n +)= (n+)(n+) 이고이것은원래삼각형 3장을합한것이므로 S = (a+b+c)(n+)(n+) 답 } (a) 4개의연속한자연수로, 각각이 p 인소수중하나로나누어떨어지는것은존재하는가? (b) 개의연속한자연수로, 각각이 p 3 인소수중하나로나누어떨어지는것은존재하는가? ( 미국 986-) 주 (a) 존재하지않는다. 만일존재한다고가정하면 ( 귀류법 ), 7개의홀수가있고 ( 이집합을A), 그중에 3의배수인것은 ( 연속한세홀수중항상딱하나가 3의배수이므로 )3개이하이고,5의배수인것은최대개이며,7의배수와 의배수는각각많아야하나씩뿐이다. 따라서, 유일한가능성은최대 3+++개인경우가모두적용되면서서로중복되지않는상황이고, 그럼 3의배수가 3개여야하는데, 그경우는A의양끝의홀수가 3의배수인경우라서그둘을제외한남은 5개의연속한홀수중에 5의배수인것은하나뿐이다. 즉, 모두최대이면서서로중복되지않는상황은없다. (b) 존재. 예 : 9440{ n 명의선수가풀리그 ( 모든선수와한번씩경기를함 ) 로씨름경기를하였다. 비기는경우는없다고할때다음물음에답하여라. () 경기가모두끝난후앞사람이바로뒷사람을이긴순서로한줄로정렬할수있음을밝혀라. nx n () k번째선수가이긴경기의수를 w k, 진경기의수를 l k 라하면 wk = X lk 임을증명하여라. k= k= ( 소련 987) 증명 () 귀납법으로증명하자. n =일때는자명. n명일때가능하다고가정하고, n +명일때를보자. 그중n명만생각하면그들사이에도풀리그를벌였으므로이 n명을한줄로잘정렬할수있고, 그순서를 a! a!!a n 이라하자. 나머지한명 b가 a 을이겼으면 b! a! 로앞에붙이면되고, b가 a n 에게졌으면! a n! b 로뒤에붙이면된다. b가 a 에게는지고 a n 에게는이겼다면, 앞에서부터차근차근살피면 a i 에게는지고 a i+ 에게는이긴 i가틀림없이존재한다. 그럼!a i! b! a i+! 로끼워넣으면된다. () P w k 와 P l k 는둘다전체경기의수와같으므로같고, w k + l k 는한사람이경기한횟수이므로늘 n 이다. 따라서, nx n wk X lk = X (w k + l k )(w k l k )=(n ) X ³X (w k l k )=(n ) wk X l k =0 k= k=

337 4. 조합중급문제 337 로 P wk = P lk 임이증명된다. 38. a, b, c 세문자만을사용하여단어를만들려고한다. 문자 a 와문자 b 는같은것끼리인접하여배열할수없다고할때, 문자의개수가 n 인모든단어의개수를구하여라. ( 한국 988-) 풀이 구하는수를 f(n) 라하고, f(0) = 로약속한다. n =일때, a; b; c의세가지 ) f() = 3 n =일때, ab; ba ; ca; cb; cc ; ac; bc의 7가지 ) f() = 7 다음과같이분류하여생각한다. (i) 첫자리에 c가있을때 () c {z } f(n ) 가지 n 개 (ii) k( k n) 번째자리에처음으로 c가나타날때 aba (k) c bab c {z } f(n k) 가지 (n k) 개 ¾ (iii) c가하나도없을때 aba bab 의 가지 따라서 n = 3일때 f(n) = f(n ) + f(n ) + +f(0) + 또 f(n ) = f(n ) + f(n ) + +f(0) + 이므로 f(n) f(n ) = f(n ) + f(n ) ) f(n) =f(n ) + f(n ) n =; 3; f(n +)=f(n)+f(n ) (n = ) 로놓고 ½ + = f(n +) f(n) = (f(n) f(n )) 라하면 = ) f(n +) f(n) = n(f() f(0)) = n(3 ) = n( + ) ; 를바꾸면 f(n +) f(n) = n (3 ) = n ( + ) ) ( )f(n) = n( + ) n ( + ) = p ; =+ p ½ p + = 라하면 + = p ) f(n) = n( + ) n ( + ) = ( + p ) n+ +( p ) n+ 답 f(n) = ( + p ) n+ +( p ) n+ } 39. 한변의길이가 인정오각형이주어져있다. 이도형을반지름 r 인원 5 개로덮을수있다고할때, r 의최소값을구하고그것을증명하여라. ( 이탈리아 988-3) 증명정오각형의다섯꼭지점 A, B, C, D, E와중심을 O를생각하자. 이여섯점중에어떤두점은한원에의해덮여야한다. AB > OA 이므로이원은지름이최소 OA. 실제로 r = OA 일때OA, OB, OC, OD, OE를지름으로하는다섯원을생각하면잘덮인다 ( 인접한두원은지름의원주각이 90 ± 라는사실을생각하면각변의중점에서만난다는것을금방알수있다 ). 따라서, 구하는최소값은 r = OA = 4 csc 36± = p (5 p 5) 답 40. 정육면체의각꼭지점마다 또는 의수를배정하고, 각면에는그면의네꼭지점의곱을배정한다. 점과면에배정된4개의수의합이 0일수있는가? (Towns 988가을 JA) 풀이불가능하다. 꼭지점 개의부호를바꾸면 4개항중 4개항의부호가바뀜. 따라서 인항은항상짝수개. } 4. 평면위의각각의점들에두가지색중에서골라한가지씩색을칠했다. 같은색의세꼭지점을갖는정삼각형이존재함을보여라. ( 오폴 989-)

338 338 조합 증명같은색의두점이있고그두점의색을a라하면차근차근고려하여다음과같은색배열을얻을수있음.( 대문자 A가시작위치 )... a a..... b b b... a A A a... b b b.. 그럼그림에서보듯 a 로된정삼각형이금방나타남 마리의원숭이가 5 개의사다리가있고, 각사다리의맨위에는바나나가있다. 몇개의밧줄로사다리들을연결하는데, 각밧줄은 개의사다리를연결한다. 같은사다리의같은단에두개의밧줄이묶일수없다. 각원숭이는각각다른사다리의맨아래에서출발한다. 원숭이는사다리를오르다가밧줄을만나면그밧줄을따라다른사다리로넘어가사다리오르기를계속한다. 밧줄이얼마나많고어떻게연결되어있는지에상관없이, 모든원숭이는바나나를갖게됨을증명하여라. ( 캐나다 989-4) 증명하나의밧줄은양방향의통행로가되고같은사다리의같은단에두개의밧줄이연결되는일이없으므로, 이밧줄을다음과같이둘로나누어생각해도동치가된다. 원숭이들이지나는길은항상 n(= 5) 개의일방통행로와몇개의원 ( 순환로 ) 으로구성됨을증명하자. 우선밧줄이하나도없을때에는자명하다. 이제밧줄이 k 개일때성립함을가정하고, 밧줄이하나더늘어도성립함을보이자. 새로운밧줄은두개의통행로를연결할수도있고, 같은통행로의두지점을연결할수도있다. 그럼위와같이네가지경우로나눌수있고, 모든경우에서원의개수가늘어나거나감소할뿐, 일방통행로의개수가잘유지된다는것을확인할수있다. 따라서, 수학적귀납법에의해증명이되었다. 모든원숭이는각각이일방통행로의아래쪽끝점에서출발하므로, 각각서로다른위쪽끝점에도착하게된다. 별증모든사다리는아래에서위로올라가는방향성이있다. 모든밧줄은양쪽으로통행가능한방향성이있다. 모든밧줄의양방향성을다음과같이각각둘로분리하여생각하자. 그럼이렇게얻어진방향그래프에는나가는길만하나있는 n(= 5) 개의출발점 ( 사다리의아래끝 ) 과들어오는길만하나있는 n 개의도착점 ( 사다리의위끝 ) 이있고, 그밖의점 ( 사다리의각단을 개의점으로분리한것 ) 들은항상들어오는길과나가는길이하나여서선택의여지가없는중간경로가될뿐이다. 점의개수가유한하므로, 한출발점에서출발한원숭이가어떤도착점에이르지못한다면한번지났던점을다시지나야하는데, 그럼그점의차수가 보다커서모순, 따라서출발점에서출발한원숭이는반드시어떤도착점에도착하게된다. 또한, 서로다른출발점에서출발한두원숭이가같은도착점에이른다면, 두원숭이가지나는길이어느점에선가만나야하고, 그럼역시그점의차수가 보다커서모순, 따라서서로다른출발점에서출발한원숭이는서로다른도착점에이르게된다 직육면체블록 77 개를쌓아서 7 9 직육면체를만들수있는가? ( 소련 989-y8-)

339 4. 조합중급문제 339 풀이불가능하다. 만들수있다면빈틈없이꽉채우는수밖에없는데,7 단면을생각하면그단면은각각의블록들과 3개혹은9개의칸에서만난다. 그런데그단면은칸의 3의배수개가아니므로모순. } 44. 체스판위에 8개의칸에졸이하나씩놓여있다. 체스판의각행과열마다딱하나씩의졸이있다. 검은칸에놓인졸의개수는짝수임을보여라. ( 소련 989-y8-5) 증명짝수번째행들과짝수번째열들을모두합하면검은칸에놓은졸의개수와홀짝이같음 ( 검은칸은한번씩, 흰칸은0번혹은번씩세어졌으니까 ). 여기에졸이총 4+4개세어지므로문제는틀림없이성립. 45. 여러가지크기의정사각형모양의카드들을바닥에늘어놓았다. 이카드들로덮인부분의전체넓이를 S 라하자. 이들카드중에서제일큰카드 A 을고르고 A 과서로겹쳐있지않은카드중에서가장큰카드 A 를고른다. 다음에 A, A 와겹쳐있지않은카드중에서제일큰카드 A 3 을고른다. 이와같은방법으로끝까지계속하여 n 개의카드 A ;A ;:::;A n 을골랐다면이들카드들의면적의합은 3 S 보다큼을밝혀라. ( 한국 990-3) 증명카드 A ;A ; ;A n 의한변의길이를각각 a ;a ; ;a n 라한다. A 은최대크기의카드이므로 A 과겹쳐있는카드는모두A 으로부터 A 의대각선의길이 p a 이내에있다.( 그림참조 ) 위그림의전체영역의넓이는 a +4( p a )+¼( p a ) =(+4 p +¼)a () 이다. 카드로접힌전체영역의넓이S는위의넓이 () 들의합보다크지않으므로 nx S ( + 4 p nx +¼)a i =(+4p +¼) a i i= i= nx a i 은 A ;A ; ;A n 의넓이의합이고 i= +4 p +¼<3 이므로 +4 p +¼ > 3 ) nx a i = S +4 p +¼ > S 3 i= 46. 평면위에 989 개의점이있고, 이중어느셋도한직선을이루지않는다. 이점들을서로다른개수의 30 개그룹으로분할하려고하는데, 세꼭지점이서로다른그룹에속하는삼각형의개수를최대로하고싶다면어떻게분할하면되는가? ( 중국 989-5)

340 340 조합 풀이 0 <a <a < <a 30, a + a + + a 30 =989일때max P i<j<k a ia j a k 를묻는문제. a i <a j 를 a i, a j + 로각각바꾸면a i a j a k 꼴들은감소하고 a i a k a l + a j a k a l 꼴들은불변이고나머지도모두불변, 즉종합하면이합은감소함. 따라서, 가능한한빽빽한수열이라야함 ( 좀더정확히말하면, a i 들사이에빈칸은많아야하나뿐이라야함 ). 연속한 3개의수를 n 5;:::;n+5 로두고그중 n + a 가빠진다고하자 ( 5 a<5). 그럼이들의합은 30n a = 989. 즉 n =66,a = 9. 5;:::;8 중에 57만빼고몽땅임. } 과 만으로이루어진 k 개의항으로된모든수열들의집합을 T 라하자. S 는 T 의부분집합이고 k 개의원소를갖는다. 그리고, 임의의 x T 에대해 x 와최대 3 개의항을제외하고나머지항은모두같은 S 의원소가유일하게존재한다. k>5 라면, k = 임을보여라. ( 아일랜드 990-0) 증명 임의의하나의수열 x T 에대해그것과최대3개의항이다른수열은 ³ k ³ k ³ k ³ k a = = k(k 3k +4) 개씩있다. 즉, S 의임의의원소 y 에대해 T 의원소가꼭 a 개씩대응되므로 jt j = ajsj 가되어야한다. a = jt j=jsj = k 이므로, k(k 3k +4)=3 k 의부정방정식이된다. 우변을관찰하면 k = r 혹은 3 r 꼴이다. 두경우각각열심히풀어주면 k = r 꼴일때는 ( 진법과연관시킨부등식적부정방정식으로 ) 해가없음을확인할수있고, k =3 r 꼴일때도열심히해주면아마 r = 만답으로나오겠지 n진법으로썼을때, 모든자릿수가서로다르고, 가장왼쪽자릿수를제외한나머지자릿수는그보다왼쪽에있는자릿수중에 만큼차이나는것이항상있는자연수는모두몇개인지구하고그것을증명하여라. ( 미국 990-4) 풀이조건을만족하는 t 자리수의개수는다음과같이구할수있다 (t =;:::;n). 먼저이수의자릿수를모두모으면연속한 t개의숫자들이된다. 즉, 먼저연속한 n개의숫자 (n진법의자릿수들 ) 열에서연속한t개의덩어리를떼어내는방법을생각하면 n t +가지 (= A). 다음, 이 t개의숫자를조건에맞게배열하는방법을생각하면되는데, 맨뒷자리부터생각하면그숫자는이덩어리중에가장작거나가장큰수이므로 가지, 그다음맨뒤바로앞자리를생각하면남은 t 개덩어리중역시가장작거나가장큰수이므로 가지,... 이런식으로계속생각하면뒤에서부터계속 가지씩곱해져서결국 t 가지 (= B). 따라서, 이런 t자리의수는맨앞자리가 0인것 (0 (t ) 로유일 ) 을제외하면 AB =(n t +) t 개있다. 이것을 t =;:::;n 에대해모두합하면 n+ n 개. } 49. 두아이가다음과같은게임을한다 : 첫번째아이가세개의정수를마음대로택하면, 두번째아이는그세정수를적당한순서의세계수로하는이차방정식을하나만든다. 이이차방정식이서로다른두유리수를근으로가지면첫번째아이가이긴다. 첫번째아이가이길수있음을증명하여라. ( 소련 990-y9-6) 증명 a + b + c =0인서로다른세정수를택하면된다. 그럼 은반드시한근이고, 나머지한근은 이아닌유리수이다 크기의격자판의각칸을검은색혹은흰색으로칠하는데, 각열과각행은두가지색을절반씩가져야하고, 격자판의중심에대해서대칭의위치에있는두칸의색은서로달라야한다. 이렇게색칠하는것이가능한가? ( 소련 990-y0-) 풀이불가능하다 크기의네영역으로분할하여 AB/CD라하면,A와 B는두색의개수가서로뒤집혀져있고 B와 C도그렇다. 그럼 A와 C는두색의개수가서로같아야하는데, 또한중심에대해대칭이면반대색이라는조건에의해 A와 C 역시서로뒤집혀져있어야한다. 즉,A의검은칸과흰칸의개수가같아야한다. 그런데 A는홀수개의칸을가지므로이것은모순. } 5. 각각,, 3,..., 990 개의돌이있는돌더미가 990 곳있다. 여기서몇개 ( 한개만택해도된다 ) 의돌더미를택해같은개수씩의돌을제거할수있다. 모든돌을없애려면이러한작업을최소몇번해야하는가? ( 소련 990-y0-)

341 4. 조합중급문제 34 풀이서로다른개수의돌을갖는돌더미를서로다른종류라고말하자. 돌더미의종류수는최대 =로줄어들수있다 ( 그보다더줄어들수는없다. 절반미만을택하면선택되지않은돌더미들때문에, 또절반이상을택하면선택한돌더미들때문에 ). 따라서, 처음 990에서 995, 498, 49, 5, 63, 3, 6,8,4,,종류로감소시키는게최선이고, 실제로이렇게하는것이가능하다. 답 번 } 5. ³ 99 ³ 임을증명하여라. ³ ( )m 99 m ³ 99 m + ³ 996 = m ( 한국 99-5b) [ n ] X 증명 S(n) = ( ) m³ n m ; n =; ; 라하자. m m=0 n X S(k) = k=0 = = [ n ] n X X ( ) m³ k m m k=0 m=0 [ n ] X n X ³ k m ( ) m m m=0 k=m [ n ] X m=0 = S(n) ( ) m³ n m m + (() 에의하면 ) n X ) S(n) = S(k) k=0 이로부터다음점화공식을얻는다. S(n +)=S(n) S(n ) () S(0) = S() = 이므로 S() = 0; S(3) = ; S(4) = ; S(5) = 0; S(6) = ; S(7) = 따라서점화공식 () 로부터 m = n (mod 6) 이면 S(m) =S(n) 이다. n ³ n m ³ n m ³ n m = + n m m m m 이므로½ ( ) m m 99 m m ¾ = S(99) S(989) = 0 ( ) = X995 ( ) m ³ 99 m ) = 99 m m 99 m=0 53. 양수 x가두자식x +과 x 를가진다.의자손을모두구하여라. ( 이탈리아 99-6) x + 풀이 m>n일때 n m n A 이면 n n m A. m<n일때 m m A 이면 n A. 이로써 m + n m 에대한귀납법으로모든양의유리수가 A에속함. } 체스판의 64 개의칸에자연수를하나씩써넣었는데, 각칸의수는그칸과변으로이웃한칸의수들의평균이된다고한다.64 칸의모든수가똑같아야함을보여라. ( 인도지역예선 99-8)

342 34 조합 증명최소원의원리. 64칸의모든수가똑같지는않다면, 전체가연결되어있으므로최소값을갖는칸중에서최소값이아닌칸과맞붙은경우가존재한다. 그런데최소값인칸은이웃한칸의수들의평균이되려면이웃한칸들도모두그칸과같은최소값이어야한다. 이것은모순. 따라서, 모든칸의수는같다. 55. 하나이상의수를모은임의의집합 S에대해, 원소들의합과곱을각각 ¾(S) 와 ¼(S) 로나타내자. X µ ¾(S) ¼(S) =(n +n) (n +) n 임을증명하여라. 단, P 은 f; ; 3;:::;ng 의공집합을제외한모든부분집합 S 에대한합이다. ( 미국 99-) 증명귀납법. 56. 다각형모양으로생긴도시가직선길들에의해몇개의구역으로나뉘어져있다. 다각형의꼭지점마다광장이있다. 각각의길은일방통행이며, 두개의광장을잇고, 중간에다른광장을거치지않으며, 서로가로지르지도않는다. 도시외곽의경계선을이루는길을따라서도시를한바퀴돌수있다면, 이도시에는둘레를따라한바퀴돌수있는구역이있음을보여라. ( 러시아 99 4차-y0-3) 증명만일도시외곽의경계선외에길이없으면그냥외곽을따라돌면되므로성립. 대각선이하나추가될때마다 ( 추가된영역이둘레를따라한비퀴돌수있는영역이었다면 ) 그대각선으로나뉜두영역중하나가다시조건을만족하는영역이됨. 이렇게귀납적으로성립. 57. 크기의직사각형이주어져있다. 다음을보여라. (a) 이직사각형은 6 과 7 크기의타일 0 개로잘깔수있다. (b) 위와같은타일 9 개로는잘깔수없다. ( 러시아 99 4 차 -y0-6) 증명 (a) b =3이므로 7크기의타일은 b =개,6크기는 8개. 7 와 4 를조립하면실제로가능.(b) 한코너로부터대각선방향으로한줄을색칠하고, 그로부터거리 7에있는대각선을칠하면색칠된칸은모두 0개. 각타일은색칠된칸을많아야하나만깔수있으니까타일은적어도 0개는필요. [(b) 별해 ]6짜리 개와 7짜리 8개로깔아야하는상황. 모든행을 으로똑같이칠한다 ( 색이 7종류 ). 그럼 7 로는가로로깔거나세로로깔거나각각의색의사용횟수에 mod 7로똑같이기여한다. 6 은어떤특정한한가지색만다른색보다만큼부족하게기여한다. 즉, 6 이 개이므로, 모든색이사용된횟수가 mod 7로다똑같은데한가지색만 만큼부족해야한다. 그런데, 색칠된상황을보면 03이다똑같고 456도다똑같고두그룹은서로다르니까적절한상황이아님. 58. 칠판에 n개의수가적혀있다. 우리는이수들중두수를택해, 그두수를a, b라하면, 그두수를지우고그대신 a+b 4 를적을수있다. 처음에칠판에있었던수가 n개의 이었다면, 이런과정 n 번을통해마지막으로남는수는 n 보다작지않음을보여라. ( 소련 99-y9-) 증명 a + b 4 a+b 의부등식 (AM-HM 혹은코시나완전제곱 ) 에의해칠판에있는수들의역수의합이감소함 ( 혹은유지됨 ) 을알수있다. 따라서, 처음에역수의합이 n이었으므로마지막수의역수의합 ( 즉, 그냥그수의역수 ) 은 n 이하이다. 59. 다각형을한직선으로잘라두조각으로만든후한조각을뒤집어서절단선을따라두조각을다시붙여새로운다각형으로만들수있다. 단, 두조각이겹치게해서는안된다. 정사각형을이런과정을반복하여삼각형으로만드는것이가능한가? ( 소련 99-y0-6) 풀이불가능하다. 이런과정으로는넓이와둘레의길이가둘다불변인데, 둘레의길이가일정한삼각형중에서가장넓이가넓은정삼각형일때도정사각형의넓이보다작다. }

343 4. 조합중급문제 [n] =f; ;:::;ng 의순열 ¼ =(¼ ;:::;¼ n ) 의반전수 inv(¼) 를다음과같이정의하자. inv(¼) =jf(i; j) j j<i; ¼ j >¼ i gj inv(¼) =inv(¼ ) 임을증명하여라. ( 통신강좌 ) 풀이 I(¼) =f (i; j) j j<i;¼ j >¼ i g라하자. 즉 inv(¼) =ji(¼)j이다.(i; j) I(¼) 와동치인명제들을구해보자. (i; j) I(¼) () i>j; ¼ i <¼ j () ¼ ¼ i >¼ ¼ j ; ¼ i <¼ j () (¼ j ;¼ i ) I(¼ ) 따라서 I(¼) 의원소 (i; j) 와 I(¼ ) 의원소 (¼ j ;¼ i ) 가일대일대응한다. 그러므로 ji(¼)j = ji(¼ )j이성립한다. } 6. 조커한장과 에서 n 까지중의한숫자가적힌카드를각각두장씩포함한총 n + 장의카드가있다. 이 n + 장의카드들을가로로펼쳐늘어놓았는데, 가운데에는조커가놓여있다. k n 인각각의 k 에대해, k 가적힌두장의카드사이에는정확히 k 장의카드가놓여있다. 이렇게늘어놓는것이가능한 n 0 과불가능한 n 0 을모두구하여라. ( 캐나다 99-5) 풀이조커 (J) 가놓이는위치의좌표를0으로하고, n부터 n까지의정수좌표에카드를놓는것으로하자. i 카드를놓는두위치를 a i, b i 라고하면문제의조건은 b i a i = i 라야한다는것이된다. 그럼 n(n +) nx nx = (b i a i ) (b i + a i )=0 (mod) i= i= 라야하므로, n 0; 3 (mod 4) 라야한다. 즉, n =; ; 5; 6; 9; 0 일때는불가능하다. n =3; 4; 7; 8 일때는다음과같이가능한경우들이있다.(n =7; 8 일때는다른해도많다.) 3J3 34J J J 답 3, 4, 7, 8은가능.,,5,6,9,0은불가능. } 6. 어떤나라에몇개의도시가있다. 이도시들중에몇몇은일방통행항공노선으로이어져있다. 어떤도시에서는그도시에서출발하면도착할수없는다른도시가존재한다. 몇개의도시들그룹을잘고르면, 이그룹에속하지않는어떤도시에서도이그룹에속하는도시로올수없음을보여라. ( 러시아 99 4 차 -y-3) 증명모든도시에도달할수는없는그도시를 A라하고,A에서도달할수없는도시들을모두모아그그룹으로택하면됨. 63. 다음의성질을만족하는공간상의직선들의집합 M 이존재하는가? (i) 공간상의모든점은M에포함되는정확히두직선위에놓여있다. (ii) 공간상의모든두점은M에포함되는직선들을따라꺾은선으로연결할수있다. ( 러시아 99 4차-y-6) 풀이 () z-축에평행한모든직선,()y-축에평행한모든직선중에 z 6= 0인것들,(3)z =0평면위의 x-축에평행한모든직선들로하면된다. 모든점은 () 에서하나, 또 () 나 (3) 에서하나의직선이지나가므로 (i) 을만족. (ii) 는다음과같이확인하면된다. 임의의두점이있으면둘다 () 에의해z =0 인평면으로옮겨오고, 다시 (3) 에의해x-값을똑같이맞춰주고, 다시 () 에의해z =인평면으로올라오고, 마지막으로 () 에의해y-값을만나게하면된다. }

344 344 조합 64. 한변의길이가 0 인정삼각형을변에평행한직선들을그려서한변의길이가 인 00 개의작은정삼각형들로분할하였다. 위의그림의왼쪽과같은평행사변형타일 5 m 개와오른쪽의정삼각형타일 m 개가주어져있다고하자 ( 이타일들을뒤집거나회전할수도있다 ). (a) m =0일때위의타일들을잘깔아서큰삼각형을덮을수있는가? (b) 큰삼각형을덮을수있는m의값을모두구하여라. ( 소련 99-y-5) 풀이 (a) 이웃한칸이서로다른색이되도록흑백으로색칠하면, 코너가흰색이라고할때, 흰칸이검은칸보다 0개더많다. 즉흰칸55개, 검은칸 45개. 평행사변형타일은흰색과검은색을 ( 똑같은개수로 ) 짝수개씩깔고, 정삼각형타일은홀수개씩깐다. 즉, 정삼각형타일이홀수개라야하는데 0개이므로모순. (b) 흰칸이0개더많으니까정삼각형타일은 5개이상이라야하고,(a) 에서봤듯이홀수개라야함.5이상의홀수일때는항상잘됨. 큰삼각형을한변의길이가 인마름모0개와한변의길이가 인정삼각형5개로분할해놓고생각하면쉬움. } 65. 각항이 0 아닌정수로이루어진수열이있다. 임의의연속된 7 개의항의합은양수이고, 임의의연속된 개항의합은음수이다. 이러한성질을갖는최대항수의수열을하나구하여라. (APMO 99-5) 풀이 (O±cial) 항수가 7 인이러한수열은없음을밝힌다. = 이므로항수가 인부분수열의첫 4 항의합은음수이다.4=3+ 이므로항수가 4 인부분수열의첫 3 항의합은양수이다.5=4+ 이므로항수가 5 인부분수열의모든항의합은음수이고 5=+7+7 이므로항수가 5 인부분수열의첫항은음수이다. 에서 5 항의첫항은제 항, 에서 6 항의첫항은제 항,3 에서 7 항의첫항은제 3 항이므로항수가 7 인수열에서는처음세항은모두음수이고이들의합은양수이므로모순이다. 따라서항수가 7 인이러한수열은없다. 항수가 6 인수열의보기는다음과같다. 5; 5; 3; 5; 5; 5; 3; 5; 5; 3; 5; 5; 5; 3; 5; 5 연구음수항끼리는모두같은수 a (a >0), 양수항끼리도모두같은수 b (b >0) 인경우를생각하자. 처음두항은음수이고, 셋째항은양수이라야한다. 위의논의에서첫 4항,3항,항의논의는끝 4항,3항,항도마찬가지이므로수열은 이라야하고위의조건들에서 이들을종합하면 a; a; b; a; a; a; b; a; a; b; a; a; a; b; a; a b a >0;b 3a <0; b 5a >0; 3b 8a <0 5a <6b <6a 를만족시키는 a; b 를택하면된다. a; b 가정수인경우에 a =5;b=3 이위에서든보기이고 a =7;b= 8; a =8;b=; 와같은보기도들수있다. 66. 공간상의 n 개의점들을생각하자 (n >). 이들이최소한 n + 개의선분으로서로서로연결되어있다고하자. 최소한한개이상의삼각형이생김을보여라. 또한, 이 n + 이최선의답임을예를통해보여라. ( 통신강좌 ) 증명우선 n 개의선분일때는,n개의점을 n개 :n개로두집합으로분할하여서로다른집합에속하는두점끼리만선분을그릴때를생각하면반례가있다 ( 이분그래프 ). 이제 n +개의선분일때는반드시삼각형이생김을귀납법으로보이자. 우선 n =일때5개의선분을그린다는것은완전그래프에서한변만뺀다는것이므로삼각형이 ( 둘이나 ) 남아있음이자명. 이제 n일때성립함을가정하고 n +일때를보자. 서로선분으로연결된두점 XY 를하나택하면, 나머지 n개의점들중에 X, Y 와둘다연결된점이있으면삼각형이생기므로끝. 그렇지않다면 X, Y 에서연결된선분은 (XY 를포함해 ) 최대 n +개뿐이므로,n개의점들안에서나머지 (n +) + (n +)=n +개이상의선분이있다. 그럼귀납법의가정에따라삼각형이있으므로역시끝. }

345 4. 조합중급문제 S = fx ;x ;:::;x n g 는평면위의점들의집합인데, S 의임의의서로다른두점사이의거리는 이상이다. 거리가 인점들의쌍은기껏해야 3n 개임을보여라. ( 통신강좌 ) 증명정육각형분포에서힌트를얻으면, 점하나당이웃 ( 거리 떨어진점 ) 은최대6개씩. 한변 ( 이웃관계 ) 은두점에의해세어지므로사실상점하나당평균 3개이하. 68. n개의원소를가진집합 A가있다. 집합 A의모든부분집합을각각한번씩, 다음의조건을만족하며일렬로나열할수있음을보여라 : 임의의연속한두부분집합에서앞의부분집합에한개의원소를추가하거나제거한것이다음의부분집합이되어야한다. ( 통신강좌 , Gray code) 증명 ( 경기과학고 학년이필규 ) 일반성을잃지않고 n개의원소를가진집합을 f; ; ;ng이라할수있다. 수학적귀납법을사용하자. n =일때 ;, fg이므로성립한다. n = k일때성립한다고하자. f; ; ;kg의부분집합 k 개를위의조건을만족하며일렬로나열한것이 A! A!! A k 이라하자. 그러면 f; ; ;k+g의 k+ 개의부분집합은 A ;A ; ;A k, A [fk +g;a [fk + g; ;A k [fk +g이다. 이것을다음과같이나열하면문제의조건을만족시킨다. A! A!!A k! A k [fk +g!a k [fk +g!!a [fk +g 69. 이항계수 n r 은 n 개사물에서 r 를택하는경우의수를나타내고, n 0 =, 그리고 n<r 이면 n r =0 으로정의한다. r n 인임의의정수 r, n 에대해다음항등식을증명하여라. ( 아일랜드 993-8) X ³ n r + ³ r d d d= ³ n = r 증명 d 을 r d 로만바꿔주면조합적논의로 m 9 에대해, mn 자리의자연수들중에자릿수에 이 n 개, 가 n 개,...,m 이 n 개있는수들을모두모아놓은집합을 Z(m; n) 이라하자. Z(m; n) 에속하는자연수 N 에대해서 d(n) 을서로이웃하는자릿수의차들의합으로두자. 예를들어서 d(43) = j 4j + j4 j + j 3j =6 이다. Z(m; n) 에속하는모든 N 에대해서 d(n) 의평균값을구하여라. ( 영국 993 차 -4) 풀이 ( 구상준 ) 우선배열을생각하지말고, 숫자의차이를생각해보자. 즉, 예를들어 d(34) 에대해서이수열을하나로생각하지말고, j j, j 3j, j3 4j 이렇게 3개가있다고생각하는것이다. 그러면이제 Z(m; n) 에대해서생각해보자. 이 Z(m; n) 에는다음과같은원소들이있다. j j; j j; j 3j;:::;jm mj 여기서 jp; pj(p는 이상 m이하의임의의자연수 ) 꼴은그차가 0이기때문에배제하자. 이집합속에jk ij(k >j) 가몇개있는지세어보자. 우선원소 jk jj가들어있는배열은다음과같이되어있을것이다. kj kj kj ( 중간생략 ) kj kj가올수있는자리는위에서보듯이mn 가지경우가있다. 그리고남은숫자들은 k와 j는 n 개, 나머지숫자들은 n개로되어있다. 그럼배열의가짓수는 (mn )(mn )! (n!) m (n )! 개이다. 그배열당kj로인한 차이는 k j이다. jj kj배열도이와마찬가지이기때문에이역시고려해주면이차이로인해서발생 하는총합은 (k j) (mn )(mn )! (n!) m (n )! 이다. 이를모든경우로확장시키면총합은다음과같다. mx k X k= j= 이를총배열의수 (mn )(mn )! (k j) (n!) m (n )! = (m )m(m +) (mn )(mn )! 3 (n!) m (n )! mn! n(m )(m+) (n!) m 으로나누게되면평균 3 을얻을수있다. }

346 346 조합 7. 0 개의정수가한행으로배열되어있다. 각각의수 A 의밑에는그보다오른쪽에있는수들중에서 A 보다큰수의개수를적는다. 그렇게두번째행을완성했으면그로부터같은방식으로세번째행을만들고, 이런식으로계속한다. (a) 언젠가영행 (0 으로만된행 ) 이나타나게됨을증명하여라. (b) 영행이아닌행은최대몇개나나타날수있는가? (Towns 993 가을 JA) 풀이 (a) 첫행빼면음수는없고, 행최우측의 0이하나씩늘어남. (b) 0이딱하나씩만늘어나는예가존재함 : } 7. 칠판에식 x 3 + x + x + =0이적혀있다. 두명의학생이번갈아가면서사각형을실수로바꾼다. 첫번째학생은이방정식이단한개의실근만을가지게하려고한다. 두번째학생이첫번째학생의목표를달성하지못하게방해할수있는가? ( 러시아 993 4차-y-5) 풀이 갑이먼저 b =로하고a = c = k 로하면 (k는을이결정), 준식은 (x +)(x + k) =0으로 x = k 가유일함. } 풀이 계수를차례로 a, b, c. 첫번째학생이a =0으로하면이기게됨 ( 두번째학생이방해하지못함 ). 두번째학생이c를결정한다면첫번째학생은 b =0으로하면되고, b를결정한다면 c를 b와반대의부호로충분히절대값이크게정하면됨. } 73. M 은 f; ;:::;5g 의부분집합으로 M 의임의의세원소의곱이제곱수가아니다. M 의원소의개수의최대값을구하여라. ( 통신강좌 ) 풀이 f; 4; 9g; f; 6; g; f3; 5; 5g; f7; 8; 4g는모두세원소의곱이제곱수이다. 즉모든M의부분집합이될수없다. 그리고서로소이므로 j M j 이된다. 만일 0 6 M이면 j M j 0이다. 우선 0 M이라고가정하자. 그러면 f; 5g; f6; 5g; f; 4; 9g; f7; 8; 4g는 M의부분집합이될수없다. 만일 f3; g 6½ M이면역시 j M j 0이다. f3; g ½M이라고가정하면 fg; f4g; f9g; f; 6g; f5; 5g; f7; 8; 4g는 M의부분집합이될수없다. 즉, j M j 9를얻을수있다. 결국어느경우에도 j M j 0이성립한다. 그런데 M = f; 4; 5; 6; 7; 0; ; ; 3; 4g는원하는성질을만족시킨다. 즉최댓값은 0이다. } 74. 정수좌표의점들의유한집합 S 가, 임의의점 (p; q) S 에대해 (p ;q), (p; q ) 중에딱 개가 S 에다시속할때, S 를두 - 이웃집합이라고한다. 두 - 이웃집합의가능한원소의개수를모두구하여라. ( 북유럽 994-) 풀이모든점이두개의이웃을갖는집합임. 폐곡선이생김.,6 을제외한모든짝수 답 } 75. 한정사각형을 n 개의볼록다각형으로분할하였을때, 그결과도에나타나는변의최대개수를구하여라. 주다음의오일러의공식을사용하는것이도움이될수있다 : 한다각형을 n개의다각형으로분할했을때결과도에나타나는점과변의개수를각각v와e라하면, v e + n =이된다. ( 아일랜드 994-0) 풀이각점의차수를살펴보면정사각형의꼭지점만 이상이고, 나머지점 ( 변위의점, 내부의점 ) 들은차수가 3 이상이어야함을알수있다. 차수의합은변의개수의 배이므로,e 4+3 (v 4) = 3v 4. 이것을 ( 외부영역을제외한 ) 오일러의공식 v e + n =에대입하면,3=3v 3e +3n e +4+3n, 즉 e 3n +. 등호는세로선만 n 개를그렸을때를생각하면실제성립. 답 3n +. } 명의사내가원형의탁자에둘러앉았다. 매 시간마다투표를하고, 각자는 ` 예 ' 나 ` 아니오 ' 를답해야한다. 각사내는다음과같이행동한다 : n 번째투표에서, 만일그의답이그의양옆의두사내중적어도한명과일치했다면, 그는 n + 번째투표에서도 n 번째투표에서와똑같이답한다. 그러나, n 번째투표에서양옆의두사내의답이모두자신의답과달랐다면 n + 번째투표에서는 n 번째투표와다른답을한다. 첫번째투표에서모두가어떻게투표를했더라도, 언젠가는아무도답을바꾸지않게되는때가옴을증명하여라. ( 캐나다 994-3)

347 4. 조합중급문제 347 증명 5명대신일반적인홀수 m에대해증명하겠다. 자신의답이양옆의두사내의답과모두다른사람을 `고립된사람 ' 이라부르기로하자. 고립된사람의수가항상줄어든다는것을보이겠다. m이홀수이고원형으로둘러앉았으므로, 처음투표에서모든이웃한사람의답이다다를수없고, 따라서고립되지않은사람이반드시있다. 고립되지않은사람은다음번투표에서같은답을했던이웃한사람과함께계속같은답을하므로계속고립되지않은사람으로남는다. 고립된사람중에는어떤고립되지않은사람A와이웃한사람 B가있다. 그럼 A와 B는다음투표에서같은답을하게되므로B도고립되지않은사람이된다. 따라서, 고립된사람의수는항상줄어들어, 언젠가 (m번이내에) 고립된사람이아무도남지않을것이다. 그럼그때부터는아무도답을바꾸지않게된다. 주 n이짝수이면 `예 ' 와 `아니오 ' 가번갈아놓인경우모든사람이항상끝없이답을바꾸게되어문제가성립하지않는다 명의학생들이있다. 어떤 0 명을택하더라도그중어느 3 명은같은동네에산다. 같은동네에사는 5 명이있음을증명하여라. (Towns 994 봄 JO) 증명귀류법으로한동네에는최대 4 명산다고가정. 명이상사는동네의수를 N 이라하면 (N 5 이면다섯동네에서둘씩뽑아모으면문제의조건에위배되므로 ) N 4. 만일 N 3 이면이세동네에 4 명이하가사니까 명만사는동네가 8 곳이상. 그럼역시이 8 명과다른 명을택하면문제의조건에모순. 즉, N =4 일때만남았음. 그때는 명만사는동네가 4 곳이상이고, 거기에둘둘둘모아보면역시문제의조건에모순. 78. 한의회에450명의의원이있다. 각의원이각각다른의원한명의따귀를때렸다.50명을잘택하면그안에서는다른사람에게따귀를맞은경우가없도록할수있음을증명하여라. (Towns 994봄 JA3) 증명누구에게도따귀를맞지않은사람이있다면, 그사람을택하고그사람에게맞은사람은집합에서제거함. 이과정을더이상할수없을때까지반복. 그럼그때까지고려한인원들중절반은택하고절반은버렸으며, 남은인원은모두누군가에게맞았음. 따귀때린사람의꼬리를무는 cycle을생각하면이런 cycle 몇개로남은전체인원이분할되고, 길이가 k 인 cycle에서 b k c k 3 명씩택할수있으니까. 79. 두사람이 9 6 크기의초콜렛을가지고게임을한다. 한번씩번갈아서, 자기차례가되면열또는행을따라폭이 이되도록꺾어자른후그것을먹는다. 만일한변의길이가인초콜렛만남으면다음사람은그것을자르지않고통째로먹는다. 먼저시작하는사람은 54개의조각중에서 30개이상을먹을수있음을보여라. (Towns 994봄 SO3) 증명먼저하는사람이 6개를먹어간후짝수 짝수크기가되니까, 다음사람이하는걸그대로따라하기전략을사용하면됨. 80. a 는실수이고 n 은자연수이다. 다음과같은조건을만족하는정수 q 와 r 이존재함을증명하여라. ( 통신강좌 ) jqa rj < n ( 단, 여기서 0 <q n) 증명 ( 대전과학고김인태 ) n x < n +일때 [x] =n(n은정수 ) 로정의한다. 0 xa [xa] < 에다가 x 에 0,,, n을대입하여수직선상에찍어보면비둘기집원리에의하여어느두점사이의거리는 보다작다. n 즉, j a [ a] ( a [ a])j < n [ a] [ a] 로두면 (0 < < n 이고, 는정수 ) 이다. 이때 q =, r = jqa rj < n : 0 <q n

348 348 조합 8. 어떤원판의둘레가 6 등분되어있다고한다. 우리는이각부분에 0 또는 의수를매겨 진원순열을만들고자하는데, 단이원순열의연이은부분순열중 4 자리짜리들은모두다르게하려고한다. (a) 이문제를해결할그래프모델을세우고, 그그래프를그려라. (b) 이그래프의오일러회로와우리가원하는순열과의관계를서술하여라. (c) 조건을만족하는원순열을구하여라. ( 통신강좌 ) 풀이 (a)(b) 아래의그래프를생각해보자. 그리고각변에다음과같은방법으로 4 자리 진수를주자 즉,00! 00이면 00, 00! 0이면 00, 0! 0이면 00과같이변의양끝점에주어진 3자리 진수의가운데공통의두자리수를 4자리 진수의중간 자리로주고각앞뒤자리수를 4자리 진수의앞뒤자리수로주는것이다. 그렇다면, 이그래프의모든변은 0000» 의모든 4자리 진수와대응된다. 그러므로이그래프의오일러회로를구하면원순열상의가능한 6개부분수열을모두정확히 번씩포함시키는순서를구할수있으므로원하는원순열이얻어진다. (c) 위의그래프에서, 000! 000! 00! 00! 0! 0! 0! 0! 00! 00! 00! 0!!! 0! 00! 000 인오일러회로를잡으면, 원하는원순열 을얻을수있다. } 연결된평면그래프 G 가 8 개의정점과 3 개의변을가지고있다. 이 G 의정점들을서로이웃한것끼리는다른색이되도록색칠하려면 가지색으로는부족함을보여라. ( 통신강좌 ) 증명 ( 경남과학고송윤성 ) G 의면의개수 f 는 v e + f = 에서 f =7, 각면의변수를 f, f, f 3, f 4,, f 7 이라하자. 만일 f n 4(n =; ; ; 7) 이라면 e 4 7=8 e 4 이므로모순이다. 따라서어떤 i(i =; ; ; 7) 에대해 f i =3 즉 G 는삼각형을포함한다. 그런데삼각형의세꼭지점을두가지색으로칠하면같은색이꼭이웃하게되므로, G 는 - 채색일수없다. 83. G 가연결평면그래프이고 G 의모든회로는적어도 k 개의변으로구성되어있다고할때, e k k (v ) 임을보여라. ( 통신강좌 ) 증명 ( 상문고이경용 ) G에의해생기는영역을 R, R,, R r 이라하고 R i 는 f i 개의변에의해경계가생긴다고하면, f i k(i =; ; ;r) 이므로 f + f + + f r kr

349 4. 조합중급문제 349 각변은두번씩세어지므로 r = v + e 이므로 f + f + + f r =e kr e k( v)+ke k(v ) (k )e e k k (v ) 84. 정의구역이 000 부터 000 까지의정수인함수 f 와 g 가있다. m 은 f(x) =g(y) 인 (x; y) 의개수이고, n 은 f(x) =f(y) 인 (x; y) 의개수이고, k 는 g(x) =g(y) 인 (x; y) 의개수이다.m n + k 임을증명하여라. ( 통신강좌 ) 증명정의역이유한집합이므로 f, g의치역도유한집합이다. 두치역을합집합한집합을 Y 라고하자. a Y 일때f(x) =a를만족하는x의개수를f (a), g(x) =a를만족하는 x의개수를 G(a) 라고하자. m = X F (a)g(a) ay n = X ff (a)g ay k = X fg(a)g ay 산술기하평균부등식에의하여 ff(a)g + fg(a)g F (a)g(a) ( 어느항이 0이더라도성립함을주의한다 ) 이므로 X ff (a)g + X fg(a)g X F (a)g(a) ay ay ay n + k m 85. 둥근원형탁자주위에 995 개의의자가놓여져있다. 이의자들는차례로 ; ; 3;:::; 995 로번호가붙여져있다. 각의자에한사람씩앉는다. 이제학생들이다음과같은게임을한다. 처음학생부터 ` 예 ', ` 아니오 ' 를번갈아말하면서게임이진행되는데,` 아니오 ' 라고말한학생은게임에서진것으로간주되어그학생의자리는없는것으로생각하자. 게임의마지막승자는몇번째의자에앉은사람일까? ( 통신강좌 ) 풀이총 n개의의자가있다고하자. N번째의자가최후의승자가앉게되는의자라고가정하자. N이홀수라는것은당연하고, N =과 n = k 가서로필요충분조건이라는것은쉽게알수있다. N>이라고가정하자. 우승자가처음 `예 ' 를말하기전에게임을끝낸사람수는 N 명이다. 지금부 터는학생수가 n N 명이고, 우승자가 번자리에앉아있는경우와같다. 그러므로 n N = m 이다. 즉 N =n m+ + 이다. 분명히 N 995 이면서 m 이제일작은것이구하는답이된다. n +=399 이고 = 976, 399 = 943. 따라서 n = 995 일때 N = 943 이다. } 86. n 개의원소를가지는집합 A 가있다. 이들중임의로두개의부분집합을골랐을때 ( 같아도됨 ) 한쪽이다른쪽의부분집합이될확률을구하여라. ( 통신강좌 )

350 350 조합 풀이 ( 서울과학고신석우 ) A의임의의부분집합 B를생각하자. b = n(b) 라하자.()B ½ C이고 B 6= C인 C의갯수는, A B의 Á 아닌부분집합의수이므로 n(a B) = n b () B ¾ C 이고 B 6= C 인 C 의갯수는, B 의진부분집합의수이므로 (3) B = C 인 C 는하나따라서, n(b) =b 인 B 에대해가능한 C 의갯수는 n(b) = b ( n b ) + ( b ) + = b + n b 또한, n(b) =b 인 B 의갯수는 n b 개따라서, B ½ C 또는 B ¾ C 인 (B; C) 쌍의갯수를구해보자. nx b=0 ³ n b ( n b + b ) = 한편,(B; C) 를임의로뽑는경우의수는 ( n ) = n 따라서, B ½ C 또는 B ¾ C 일확률은 3 n n n = nx b=0 nx + ³ n n b n b ³ n b b nx ³ n b b=0 b=0 = (+) n +(+) n n = 3 n n µ 3 n 4 µ n } 87. 평면상에서로다른 64 개의점이주어져있다. 이점들을이어서만들수있는서로다른직선의개수가정확히 995 개였다. 어떤 4 점은같은직선상에있음을증명하여라. ( 통신강좌 ) 증명 ( 강원과학고 학년정동욱 ) 어떤 4점도같은직선상에있지않다고가정하자. (i) 어떤 3점도같은직선상에없는경우 ³ 64 =06 (ii) 3점이같은직선상에있는것이 n쌍일경우 쌍생길때마다직선 개씩줄어든다. ) 직선의개수 N =06 n 이다. 즉 N 은짝수, 그러므로 N =995 라는것은모순이된다. ) 어떤 4 점은같은직선상에있어야한다. 88. S는 [0; ] 의서로소인유한개의부분구간들을모은집합으로, S의어떤두원소도차이가 0 이아니라고한다. S를구성하는구간들의길이의합이 이하임을보여라. ( 독일BW 995 차-)

351 4. 조합중급문제 35 증명길이가 5 인구간 [a; a + 5 ) 안의수들은차이가 0 인것끼리둘씩짝지을수있어서, 항상둘중적어도하나는뽑을수없고, 그럼이구간에서 S에포함되는부분의길이의합은 0 이하임. S \ [0; 5 ), S \ [ 5 ; 5 ), :::; S\ [ 4 ; ) 5 에서많아야 0 씩이므로합하면 이하. 89. () 정삼각형의각변을 6등분한점에서다른변에평행한직선들을그어같은크기의작은정삼각형들로분할하였다. 이렇게만들어진격자의모든꼭지점마다딱정벌레가한마리씩있다. 모든딱정벌레가격자선을따라같은속력으로기어가기시작했다. 딱정벌레는다음격자점을만나면왼쪽혹은오른쪽으로 60 ± 혹은 0 ± 를꺾어기어간다. 언젠가두딱정벌레가한점에서만나게됨을증명하여라. () 각변을5등분한경우로해도위의문제의결론이여전히참인가? ( 벨로루시 995-B8) 풀이 ( 장도한 )() 빨간점에위치한딱정벌레가이웃한빨간점으로이동하기위해서는최소한 3회이상을움직여야한다. 그러므로초기에빨간점에위치한딱정벌레 0마리는 회이동할때, 검은점으로이동하게된다. 또한, 위의빨간점의딱정벌레 0 마리가 회이동할때, 검은점에위치한 0 마리의딱정벌레가빨간점을차지하게된다. 이제빨간점을차지한딱정벌레는다음시행때, 다시 0 개의검은점으로이동하게된다. 따라서모든딱정벌레들이 회이동할때빨간점에서출발하거나빨간점을거쳐서검은점으로이동하는 0 마리의딱정벌레는 8 개의검은점에대응되어야하므로 ` 비둘기집의원리 ' 에의하여최소한두마리의딱정벌레가만나게됨을알수있다. () 위그림에서빗금친영역의경계에있는모든딱정벌레들은자신의영역을따라시계 ( 또는반시계 ) 방향으로돈다면어떤딱정벌레들도서로만나지않는다. } 90. 합동인네직각삼각형이주어져있다. 한직각삼각형을골라빗변에내리는수선을따라두조각으로자를수있다. 이런작업을몇번반복해도항상합동인두직각삼각형이있게됨을증명하여라. (Towns 995 봄 JA5) 증명원래직각삼각형의넓이를 로하자. 넷모두자르면도루묵이므로셋만자른다고하자. 그럼 a 셋과 b 셋. a 중적어도둘을, b 중에서도적어도둘을잘라야함. 그럼 ab가네조각생겨서역시도루묵. 9. 네마리의메뚜기가정사각형의각꼭지점에있다. 매초마다한마리가다른한마리를뛰어넘어점대칭점으로옮겨간다.(a) 어떤세마리도원래사각형의변에평행한한직선위에앉게되는경우는없음을보여라.(b) 어떤세마리도한직선위에앉게되는경우는없음을보여라. (Towns 995봄 SO) 증명 (a) 처음에 (0; 0), (0; ), (; 0), (; ) 의네점에서출발했다면, 각좌표의홀짝이모두항상불변. 따라서, 세마리의한좌표의홀짝이모두같을수없다. (b) 한직선에앉은세마리의좌표를 (x ;y ), (x ;y ), (x 3 ;y 3 ) 라하자. x ;x ;x 3 중에어느둘은홀짝이같고나머지하나는다르다.WLOG,x 만홀짝이다르다고하자. 그럼 x 와 x 3 가홀짝이같으므로

352 35 조합 y 와 y 3 는서로홀짝이달라서둘중하나만 y 과홀짝이같다.WLOG,y 만 y 과홀짝이같다고하자. 일직선이므로기울기가 y y = y 3 y x x x 3 x 로같다. 이것은짝 = 홀 = 홀 = 홀꼴이므로홀짝이맞지않 아모순. 9. n 각기둥의각각의모서리를 3 가지색에서골라칠하는데, 모든꼭지점마다세가지색의모서리가다만나고, 모든면마다세가지색의모서리가다있게하려고한다. 가능한 n 을모두구하여라. (Towns 995 봄 SA) 풀이옆면을생각하면어느한색만둘있어야하므로어느대변이서로같은색이고나머지두변은나머지두색.(i) 같은색의두대변이세로변일때 : 쭉쭉생각해보면윗면은두가지색의변만번갈아나타나게되어곤란. (ii) 같은색의두대변이윗밑면의변일때 : 쭉쭉생각해보면윗면에세가지색이차례로번갈아나타나야하므로 n은 3의배수. } 93. 크로스컨트리스키트랙을따라 000개의좌석이일렬로놓여있고 부터 000까지의좌석번호가차례로붙어있다. n장의티켓이팔렸는데,00<n<000 이고, 주최측의실수로각티켓의번호가모두 ;:::;00 중의하나라고한다. 또한,에서 00까지의각번호마다적어도한장씩은팔렸다고한다. n명의관중은한번에한명씩도착한다. 각자는자기번호의좌석을찾아가서비어있다면거기에앉고, 만일이미누군가가먼저앉아있다면 `어 ' 라고말한후다음번호의좌석으로이동한다. 그리고, 빈좌석을발견할때까지같은상황을계속반복한다. 모든관중이앉을때까지 `어 ' 라는탄식의총횟수는관중의도착순서에는무관하고, 발행된티켓의번호분포에만관계됨을보여라. (Towns 995가을 SA4) 증명탄식의횟수는늘 ( n) ( 티켓번호합 ) 이됨. 94. 그래프 G 에대하여 G c 는꼭지점은 G 와같으나, G 에서변이없는곳에만변을가진그래프라한다. G 가연결되지않은그래프이면, G c 는연결된그래프임을보여라. ( 통신강좌 996--) 증명 ( 서울과학고 학년이지운 ) G c 의임의의두꼭지점을잡아 a, b라하자. 만약 G에서 a와 b사이에변이없다면g c 에서는 a와 b사이에변이있게된다. G에서 a와 b사이에변이있다고하자. G는연결되어있지않은그래프이므로, 연결되어있는부분들로나눌수있다. 이들을각각 G, G,, G k 이라하자. a와 b는연결되어있으므로같은부분에있게된다. 편의상 a와 b가속해있는부분을 G 이라하자. G 의임의의점 c를잡으면, 그래프 G에서 a와 c, b와 c사이에는변이없다. 따라서 G c 에서는 a와 c, b와 c사이에변이존재하고, 따라서 a와 b는연결되어있다. 따라서 G c 의임의의두점이연결되어있으므로 G c 는연결된그래프이다. 95. 정수 ;:::;n 의임의의순열을 a ;:::;a n 이라하자. 이런순열중에서다음을만족하는것들의개수를 f(n) 으로한다. (i) a =. (ii) ja i a i+ j, i =;:::;n. f(996) 이 3 의배수인지아닌지를알아내어라. ( 캐나다 996-) 풀이 좌변을관찰하면 x, y, z 는모두음이아닌수임을알수있다. 산술 - 기하평균관계에의해 y = 4x +4x 4x 4x = x 마찬가지로하여 y x z y 이고, 따라서 x = y = z 일수밖에없다. 이제세식은모두같고, 4x +4x = x 의식을풀어주면 4x = x +4x 3, 즉 x(x ) =0이므로, 실근은 (x; y; z) =(0; 0; 0) 과 ( ; ; ) 뿐이다. } 96. 모든 i 에대해 f(i) i 인일대일대응 f : f; ;:::;ng!f; ;:::;ng 들을생각하자. 이런 f 가모든 i 에대해 f(i) i + 을만족할확률을 p n 이라고할때, p n > 3 인자연수 n 을모두구하여라. ( 폴란드 차 -6)

353 4. 조합중급문제 353 풀이 f(i) i 인 f의개수를 q n 이라하면, f(n) 은 n과 n 둘중하나, f(n ) 은 n 와앞에서선택되지않은것중하나,... 이런식으로 f() 까지항상 가지의선택을갖고, f() 은자동으로결정되므로 q n = n. i f(i) i + 인 f의개수를 r n 이라하면, f(n) =n 일때와f(n) =n, f(n ) = n 일때로구분되어 r n = r n + r n 의피보나치수열. p n = rn q n 은감소하므로앞에서부터계산해보면 n 6 일때만. } 97. n 은자연수이다. a + b, :::; a k + b k 가모두 n 미만의서로다른값이되는 k 개의서로다른자연수 a ;b ;:::;a k ;b k 가존재할때, 자연수 k 가조건 C n 을만족한다고한다. k 가 C n 을만족하면 k (n 3)=5 임을보이고, 특히우변이정수인 n 에대해서는항상등호가성립할수있음을보여라. ( 프랑스 996-5) 증명 ++ +k a + b + + a k + b k =(a + b )+ +(a k + b k ) (n ) + +(n k) n 3 이므로 k(k +) k(n k ). 정리하면 5k n 3. 5 가정수일때는 n =5m 꼴일때이고 k max = m. 이경우, (; 4m ); (3; 4m 3);:::;(m ; 3m ) 을짝짓고, 또 (; 3m ); (4; 3m 3);:::;(m ; m) 을짝지으면됨 크기의체스판에서갑과을두사람이번갈아게임을한다. 갑은자기차례에빈칸에 X를표시하고, 을은 O를표시한다. 그리고, 갑이먼저시작한다. 모든칸에표시가되면,X-오목의수를갑의점수로하고,O-오목의수를을의점수로하며, 점수가더많은사람이이기고점수가같으면비긴다. 단,A-오목이란행, 열, 또는대각선방향으로연속한다섯칸에모두 A가표시된것을말한다. 연속한여섯칸에 A가표시되어있으면 A-오목은두번, 일곱칸이면세번등으로센다.(a) 갑에게는필승의전략이있는가?(b) 갑에게는지지않을전략이있는가? (Towns 996봄 JO5) 풀이대칭성... (a) 선대칭또는점대칭으로플레이하면비김. (b) 이미대칭적이면 ( 을이그렇게만들어주면 ) 아무데나놓고, 아니면을이방금놓은위치와대칭적인위치에두면됨. } 99. m n 크기의체스판의한귀퉁이에룩 ( 차 ) 이하나있다. 갑과을두사람이, 갑이먼저시작하여, 이룩을번갈아움직인다. 단, 룩은상하또는좌우로한번에몇칸이든이동할수있다. 룩을움직일때마다방문한칸들에 ( 도착한칸뿐만아니라지나친칸들에도 ) 색칠을한다. 이미색칠된칸은다시지날수없고, 더이상움직일수없는사람이지게된다. 갑과을둘중누구에게필승의전략이있는가? 그리고, 그전략은무엇인가? (Towns 996봄 JA4) 풀이여야함. 먼저하는사람이무조건 m방향으로최대한움직이면승리. 물론계속 mgeqn 이유지됨을보 } 00. 한변의길이가n인정삼각형을네변의길이가,,, 인사다리꼴타일들로잘깔수있는n을모두구하여라. (Towns 996가을 JO) 풀이타일넓이가 3이니까정삼각형의넓이 n 이 3의배수라야겠고그럼 n도 3의배수라야함. 또그때다잘됨. 한변의길이가 3인정삼각형을이타일 3장으로맞붙여만들수있으므로, 큰정삼각형을먼저한변의길이가 3인정삼각형들로분할하기만하면끝. } 0. n +명의소녀와 n명의소년이있다. 각소녀가아는소년의수는모두서로다른데, 각소년이아는소녀의수는모두같다고한다. 가능한 n>을모두구하여라. (Towns 996가을 SO4) 풀이각소녀의차수는0부터 n까지각각한사람씩. 소녀의차수의합 = 소년의차수의합이므로 n = nk, 즉 k = n+ 가각소년의차수. 따라서 n은홀수라야하고, 그때항상실제예가있음 ( 소녀둘씩묶어아는소년들의집합이전체소년들의집합을둘로분할한것이되도록하면됨 ). } 0. n 명의사람들이있다. 이사람들이어떤기간동안세사람씩함께저녁을먹곤했는데, 그동안이들중어느두사람도함께저녁을먹은적이정확히한번씩이라고한다. n 을 6 으로나눈나머지는 또는 3 임을증명하여라. ( 폴란드 996/997 차 -) 증명 횟수는 우선한사람의식사참가수를고려하면 : n이홀수임을알수있다. 두사람의쌍을세면 : 회식 n 이되므로 3 j n(n ). 3

354 354 조합 03. 곱하기에대하여, n n (n >) 크기를가지고각각의숫자가 ; ;:::;n 인마방진이존재할수있는가? 즉, 어느행의수들을곱하든, 어느열의수들을곱하든그곱이일정하게되도록 부터 n 까지의자연수를 n n 크기의정사각행렬에배열시킬수있는 n 이존재하는가? ( 통신강좌 ) 풀이답은 \ 불가능 " 이다. 만약가능하다고가정하고, 각각의행의곱이 P 와같다고하자. 그러면,, :::, n 의곱은A는 P n 과같다. 이제 m을 A를소인수분해했을때,의지수라고하자. 우리는 m이 n으로나눠지지않음을보이면된다. 자연수 k를 k n < k+ 를만족하도록선택하면, n m = + n n k 이다. 특별히, n =,3,4,5,6일때, m =+=3,m =4++=7, m =8+4++=5, m =+6+3+=, m = =34이다. 그리고, 각각의경우에 m은 n으로나눠떨어지지않는다. 자, 이제우리는 n>6일때, n(n ) <m<n 임을보이고자한다. m n + n n k <n µ k + = n 따라서, m<n 이다. 또,[x] >x 이므로, µ n µ n µ n µ m> k = n µ k k = n k k 이것이 n(n ) 보다크다는것을보이기위해서는 n n + k<n임을보이기만하면되고, 또한 k k < 이고, k가정수이므로, k<n 를보이기만해도된다. k n 에서 k log n이다. 미분을이용하면 log n<n (n>6) 임을쉽게알수있다. 따라서 k<n 이다. } 주 n 이하의가장큰소수 p를잡으면 p는 n! 에딱한번만등장하므로바로풀이가끝난다. 그러나, 이렇게푸는것은 n < p 임을말해줄수있는베르트랑의공준을가정해야하므로많이감점될수있다. 한편, 이문제에서는 p가 n번은나타나야하므로 p; p;:::;np n, 즉 p n 이되고, 이것은 n + 부터 n 까지의수가모두합성수임을뜻한다. 즉, 베르트랑의공준대신에 n<p n 인소수p가항상존재한다는것만알아도충분하다. 이정도라면과연쉽게증명할수있을까? 04. p 는 3 이상의소수, n 은자연수, T = f; ; 3;:::;ng 이라하자. 만약, 다음을만족하는 T 의공집합이아닌부분집합 T ;T ;:::;T p 들이존재할때, n 을 p- 분할가능하다고하자. (i) T = T [ T [ [T p (ii) 임의의서로다른 i, j ( i; j p) 에대해T i \ T j = ; (iii) T i 의원소들의합은i에상관없이모두같다 (i =; ;:::;p). [ 예를들어 5 는 3- 분할가능하다. T = f; 4g, T = f; 3g, T 3 = f5g 로잡아주면조건을만족한다.] 이때다음을증명하여라. () n이 p-분할가능하면 p는 n 또는 n +을나눈다. () n이 p의배수이면 n은 p-분할가능하다. ( 아일랜드 997-0) 증명 ( 과기원 97학번이학성 ) n(n +) (a)» n까지의합은 p n(n +) (b) n이 n =p k라하자. f; ng; f; n g; 는 p k개이고이중k개씩묶으면된다.

355 4. 조합중급문제 닫힌구간 A =[0; 50] 을길이 인유한개의닫힌구간의합집합으로나타냈다. 이단위구간들중몇개를제거하여남은것들이모두서로소이고총길이가 5 가되도록할수있음을보여라. 단, a b 일때닫힌구간 [a; b] :=fx R : a x bg 이고길이는 b a 이다. 그리고두구간이서로소라는것은교집합이없다는것이다. ( 캐나다 997-) 증명이구간들중 ; 3; 5;:::;49 를포함하는것을각각하나씩만남기고나머지는모두지운다. 단, 이수들중어느하나를오른쪽끝점으로하는구간이있으면그것은꼭지운다. 그래도그것말고도그수를포함하는구간이반드시있다. 왜냐하면, 어떤구간이든그왼쪽과오른쪽에서각각겹치는다른구간이있어서,0 과 50 이아닌어떤점이어떤구간의끝점이면그점을포함하는구간은하나가아니기때문이다. 이제지우지않고남긴구간은모두서로소임이분명하고, 모두 5 개이므로총길이가 5 이다. 06. 갑과을두사람이게임을한다. 갑이먼저평면의한점을빨갛게칠하고, 다음을이다른 0개의점을골라파랗게칠한다. 다시갑이아직색칠되지않은한점을골라빨갛게칠하고, 또을이아직색칠되지않은 0개의점을골라파랗게칠한다. 이렇게계속반복한다. 세꼭지점이빨간정삼각형을만들면갑이이긴다. 을이지지않을수있는가? (Towns 997봄 SO5) 풀이먼저한직선위에n개의빨간점을칠하면 ( 그중두점을택할때마다정삼각형의나머지한꼭지점이될점이양쪽에하나씩두개가존재하고, 이런점들은모두서로다르므로 ) 을은 n = n(n ) 개의점을방어해야하는데, 그동안 0n개밖에못칠하니까 n이충분히크면 (n 이면 ) 곤란함. 갑의승리. } 07. n 체스판의각칸마다돌이하나씩있다. 첫수는돌하나를이웃한칸으로옮겨두돌이같은칸에있도록하는것이고, 다음수부터는매번돌이있는칸하나를택해그칸의돌의개수만큼의칸만큼그칸의돌전체를어느한방향으로옮긴다 ( 예를들어, 돌이 3 개면오른쪽이나왼쪽으로 3 칸을이동 ). 첫수를선생님이아무렇게나움직여도, 그이후모든말을첫수포함 n 수만에한칸에모두모을수있음을증명하여라. (Towns 997 가을 JA3 확장 ) 증명우선, 임의의 k 에대해그냥중앙에서부터차례로 칸,칸,3칸,... 왔다갔다움직이기만하면항상한끝에 k 번만에다모이도록할수있음. 중앙이아닌임의의위치에서시작했으면한쪽의돌이다청소될때까지왔다갔다움직이면... k 꼴이됨. 그럼반대쪽 (... ) 을그중앙에서부터적당한방향으로움직여서아까멈췄던 k와마지막에합쳐지도록하면됨. 08. 자연수 k 가있다. 다음조건을만족하는자연수 n 은 3 k 개존재함을보여라. (a) n은십진법으로 k자리수이다. (b) n을이루는모든자릿수는홀수이다. (c) n은 5로나누어떨어진다. (d) m = n 일때m을이루는모든자릿수중홀수인것은 k개존재한다. ( 통신강좌 ) 5 증명 ( 서울과학고조규붕 ) n은 k자리수이므로 n =0 k a k +0 k a k + +0 a +a 0 이라하면 (b), (d) 에의해a 0 =5 한편 (d) 에의해n; m도 k자리수이므로 a k 5 ) a k =5or 7 or 9 만약, a k =5 이면, a k =5or7or9 이다. (* a k 가홀수여야하고, m 의모든자리수도홀수여야하므로 ) 또마찬가지로, a k =7 이면, a k =5or7or9 a k =9 이면 a k =5or7or9 이다. 귀납적으로, i k 인 i 에대해위의성질이적용되므로, a i 에올수있는숫자는 5, 7, 9 뿐이고, 세개의수를임의로배열해도성질 (a), (b), (c), (d) 는만족한다. 따라서, 구하는자연수 n 의개수는 3 k 개이다. 09. G 는꼭지점이 0 개이고변이 6 개인단순그래프이다.( 단순그래프란두꼭지점을잇는변이많아야한개인그래프이다.) G 는적어도 5 개의삼각형을가짐을보여라. 또한정확히 5 개의삼각형을갖는 G 가존재하겠는가? ( 통신강좌 )

356 356 조합 풀이 < 그림 > < 그림 > < 그림 3> < 그림 4> (i) 꼭지점들의 degree 중최대가9일때, 나머지 9개의꼭지점에 7개의변이연결되는데변한개가연결될때마다. 삼각형의하나씩생기므로적어도 7개는된다! (ii) 꼭지점들의 degre중최대가8일때연결이안된나머지꼭지점하나의 degree가최대8이라해도 6 8=0이므로 8개의꼭지점에 0개의변이연결되면변한개가연결될때마다삼각형이하나씩생기므로적어도 0개는된다! (iii) 꼭지점들의 degree중최대가7일때마찬가지로생각하면 = 5이므로삼각형이적어도 5개는된다! (iv) 꼭지점들의 degree중최대가 6일때연결이안된나머지세꼭지점의 degree가모두5이하라면마찬가지방법으로 =5이므로문제성립 ) 연결이안된나머지세곡지점들의 degree로가능한쌍은 (6,6,6),(6,6,5),(6,6,4),(6,5, 5) 이다.( 순서무시 ) 네경우모두degree의합은6이상 8이하이다. (i) 연결이안된나머지세꼭지점들간에변으로하나도연결되지않았을때 (6,6,6),(6,6,5),(6,6,4) 의경우6 6 8 = 이므로변이 4개남는데변한개를그을때마다삼각형이적어도 3개는만들어지므로문제성립 (6,5,5) 의경우는 6 6 ( ) = 4이므로변이 4개남는데변한개를그을때마다삼각형이적어도두개는만들어지므로문제성립 (ii) 연결이안된나머지세꼭지점들간에변이하나있을경우 (6,6,6) 일때A; B모두 C부터 H까지중에연결되는게 5개씩있으므로겹치는게 4개는있다. 따라서삼각형이 4개만들어지고 = 이므로두개더만들어진다. ) 문제성립 (6,6,5) 일때도A; B에서 C부터 H까지중에연결되는것중겹치는게적어도 3개는되고 = 3이므로삼각형이적어도 6개는만들어진다. ) 문제성립 (6,6,4) 와 (6,5,5) 일때마찬가지로 A; B에서 C부터 H까지중에겹치는게적오도하나는되고 = 4이므로문제성립 (iii) 연결이안돈나머지세꼭지점들간에변이두개있을경우 (6,6,6) 일때A; B가 D부터 I까지중에연결되는게각각 5개,4개이므로겹치는게적어도 3개는되고 = 이므로문제성립 (6,6,5) 일때A; B가 D부터 I까지중에연결되는것중겹치는게두개는되고 = 3이므로문제성립 (6,6,4) 와 (6, 5, 5) 일때A; B가 D부터 I까지중에연결되는것중겹치는게한개는되고 = 4이므로문제성립 (iv) 연결이안된나머지세꼭지점들간에모두변으로연결됐을때우선삼각형이하나생겼다. (6,6,6) 일때A; B에서 D부터 I까지중에연결되는게각각 4개씩이므로겹치는게두개는있고 = 이므로삼각형이적어도 (++)=5개는있다. (6,6,5) 일때A; B에서 D부터 I까지중에연결되는것중겹치는한개는되고 =

357 4. 조합중급문제 이므로삼각형이적어도 (++3)=5 개는있다. (6,6,4) 와 (6,5,5) 일때 = 4 이므로삼각형이적어도 +4=5 개는있다.( 증명끝 ) 정확히 5 개의삼각형만갖는경우있다. complete bipartite graph( 완전이분그래프 ) 에 edge 가하나추가된꼴이다. } 0. 자연수들로이루어진집합 A 에대해, A 의원소가 a <a < <a r 일때, f(a) = P a k i k 로정의하자 (i = p ). 또, f; ;:::;ng 의부분집합인모든 A 에대해 f(a) 를합한것을 S n = P f(a) 라하자. S 8 = 76 64i 임을알고있다고할때, S 9 를구하여라. (AIME 998-3) 풀이 There are 8Ck subsets A of {,,..., 8} with k elements. If we adjoin 9 to one of these we add 9ik+ to f(a). So adjoining 9 to each of the subsets of {,,..., 8} increases the sum by 8Ck 9ik+ = 9i (+i)8. The subsets of {,,..., 9} are the subsets of {,,..., 8} and the subsets of {,,..., 8} with 9 adjoined to each. Hence S9 = S8 + 9i(+i)8. Note that (+i) = i, so (+i)8 = 6. Hence S9 = -35-8i + 44i = i. 답 i }. 8 8 체스판의모든칸을색칠하는데, 어떤칸에대해서도그상하좌우로이웃한칸들중에서그칸과같은색의칸이둘이상있도록하려고한다. 최대몇개의색을사용할수있는가? (Towns 998봄 SO3) 풀이있음. 6색인듯. 구역 6곳으로분할하면. 각색마다적어도 4칸씩칠해야함을금방보일수 }. 타와섬의모든주민이어느축제날자신에게친구가한명, 두명, 세명일때각각스스로를빨강, 노랑, 파랑의색으로칠하였다. 그랬더니색칠하지않은주민은없었고, 서로친구인두사람은항상서로다른색이되었고, 어떤파란주민에게도노란친구가없었다. 다음날,400명의파란주민들과 33명의노란주민들이스스로를빨강으로다시칠한반면,5명의빨간주민들이자신의색을파랑으로바꿨다. 그랬더니모든친구끼리같은색이되었다. 타와섬에는몇명의주민이있는가? ( 몰도바 999 최종-y89-3) 풀이 이틀동안계속빨강, 계속노랑, 계속파랑이었던사람을각각 a명, b명, c명이라하자. 첫날의 색은각자의친구수를의미하므로, 둘째날의색깔과친구수에따라주민들의인원분포상황은다음과같 다. 친구명 친구명 친구3명 빨강 a 노랑. b. 파랑 5. c 문제의조건에서서로차수가다른사람끼리만친구일수있으므로, b =0. 같은이유로, c는 5명들과만연결되어야하고그럼 3c = 5, 즉 c = 75. 문제의또다른조건에서차수인사람과차수3인사람은친구인경우가없다고하였으므로 33명과 400명은모두 a명과만연결되어야하고그럼 a = = 66. 따라서, 주민은모두 a + b + c = 999명 답 } 3. 처음에수 99 9(9가n개, n ) 가칠판에쓰여져있다. 매분마다, 칠판에쓰여진수하나를골라서그것을두자연수의곱으로분해한후, 각각의인수를 씩증가혹은감소시킨다 ( 하나는증가시키고다른하나는감소시킬수도있다. 단, 음수를만들어서는안된다 ). 그렇게얻어진두인수를원래의수를지우고그대신에칠판에쓴다. 이런작업을유한번거쳤더니칠판의수가모두 9가되도록할수있었다. 가능한 n을모두구하여라. ( 몰도바 999 최종-y-6) 풀이칠판에쓰여진수는계속홀수뿐. 홀수 c = ab 를 a, b 로바꿔쓰면 (a )(b ) = ab +( a b) 4 ab = c (mod 4) 이므로칠판에쓰여진수들전체의곱은 mod 4 로불변. 따라서, 마지막에칠판의수가모두 9 뿐이라면 n (mod4) 라야함. 이것은 (mod4) 와같고, 이것이성립하는경우는 n = 일때뿐. } 4. a, b 는주어진음아닌정수이다. 무한격자평면 Z Z 를한병사가다음과같이걷는다 : 현재위치가 (x; y) 이면다음걸음은반드시 (x a; y b), (x b; y a) 중의한위치로간다. 이런무한걸음으로모든격자점을다들를수있는 a, b 를모두구하여라. ( 북유럽 999-3)

358 358 조합 풀이출발지를원점으로하자. 일단 a, b는서로소라야하고, 또홀짝이달라야함 ( 홀짝이같으면 ( 홀, 홀 ) 과 ( 짝, 짝 ) 점만들를수있음 ). 이때, 항상된다는것을증명하자.( a; 0), ( n; 0) 방문가능. 그럼 ((ax + by); 0) 꼴은다들를수있으므로 (; 0) 도들를수있음. 즉,( 짝, 짝 ) 은모두가능. 그럼 ( 홀, 짝 ), ( 짝, 홀 ) 도모두가능하고또 ( 홀, 홀 ) 도모두가능. } 5. a ;a ;:::;a 8 은 f; ;:::;6; 7g 에서고른여덟개의서로다른정수이다. a i a j = k 가적어도세개의서로다른해를갖는정수 k>0 가존재함을보여라. 또한, f; ;:::;6; 7g 에서일곱개의수를골라, a i a j = k 가어떤 k>0 에대해서도서로다른세개의해를갖지않도록하여라. ( 캐나다 999-4) 풀이 a >a > >a 8 이라하고, 세개의해를갖는 k 가없다고가정하자. (a a )+(a a 3 )+ +(a 7 a 8 ) 이므로, 정리하면 a a 8 6. 이것을만족하는경우는 a = 7, a 8 = 뿐이고, 이때등호가성립하므로 fa a ;:::;a 7 a 8 g = f; ; ; ; 3; 3; 4g 여야한다. 그럼 (a a 3 )+(a a 4 )+ +(a 6 a 8 ) 정리하면 a + a a 7 a 8 33, 즉 a a 7 7 인데이것은불가능하므로모순. 따라서우리의가정이틀렸고, 그런 k 가존재한다. 일곱개의수를고르는경우의예는 f; ; 4; 8; 4; 6; 7g 등이있다. } 6. a <a < <a 0 은 5050 이하의자연수들이다. a k + a l a m a n 이 5050 의배수가되는서로다른네항 a k, a l, a m, a n 이존재함을증명하여라. ( 폴란드 차 -) 증명 a k + a l a m + a n (mod 5050) 인네항을찾는것. a i + a j 꼴은모두 0 =5050개가있는데이들이모두서로달라야하므로 ( 모두 5050 이하의자연수임에서 a k = a m 이면 a l = a n 이므로합이 mod 5050으로같은데두항중한쌍만같은경우는없다!) 5050으로나눈모든나머지가다꼭한번씩나타나야한다. 그럼 00(a + + a 0 )= P (a i + a j ) (mod 5050) 인데이것은짝 = 홀이므로모순. 7. 한칠판에자연수들 a 0 ;a ;:::;a n 이적혀있다. i =0; ;::: 에대해, 이칠판의수들중i보다큰수의개수를 b i 라하고, 두번째칠판에b 0 ;b ;::: 를 0이나타나기직전까지적었다. 이두번째칠판의수들에대해같은방식으로조사하여얻은 c 0 ;c ;::: 를세번째칠판에적었다. 첫번째칠판과세번째칠판에적힌수들이완전히똑같음을증명하여라. (Towns 999봄 JO3) 증명 Young diagram 으로끝 개의수가한줄로늘어서있다. 양끝수를제외하면각수는이웃한두항의합이다. 첫항이일때마지막항은얼마인가? (Towns 999봄 SO) 풀이 처음두항이 a, b 라면이수열은 a; b; b a; a; b; a b;a;b;::: 의주기 6 인주기수열. 즉 999 번째항은 번째항과같다. 답 } 9. M = f; ;:::;000g 의부분집합 X에대해, X의최대원소와최소원소의합을 a X 로나타내자. M의공집합이아닌모든부분집합 X에대해, a X 들의산술평균을구하여라. ( 몰도바 000 최종-y0-3) 풀이 X = fx ;:::;x k g 를 X 0 = f00 x ;:::;00 x k g 와짝짓자. X 6= X 0 이면이게일대일대응이므로 a X 와 a X 0의평균이 00. X = X 0 이면그냥 a X = 00. 그래서어쨌든 00. } 0. 한변의길이가 4 인정육면체상자에지름이 인공 65 개를넣을수있는지증명또는반증하여라. ( 폴란드 000/00 차 -4) 풀이 4 4 구조로 3 개층을만든사이에 3 3 구조로 개층을끼워넣으면그냥가능 ; }

359 4. 조합중급문제 359. n 개의자연수들로이루어진순서쌍 (c ;c ;:::;c n ) 이적합하다는것은,(c + c + + c n ) 을넘지않는자연수 k 를항상다음과같은꼴로나타낼수있음을뜻한다. nx k = a i c i ; i= a i f ; ; 0; ; g 각각의 n 에대하여, 적합한순서쌍 (c ;:::;c n ) 의각항의합 c + + c n 의최대값을구하여라. ( 폴란드 000/00 차 -) 풀이 가능한표현은 ai 의선택에따라총 5 n 가지인데, a i 들의부호를모두바꾼경우와, a i 들이모두 0인경우를고려하면, 표현가능한자연수는많아야 5n 가지. 따라서,(c + c + + c n ) 5n, 즉 c + + c n 5n 4 이고, 실제로 c i =5 i 로하면등호성립하고표현가능함이 ( 진법관련하여 ) 확인됨. }. 00 마리의수탉이서로싸우는동안에각수탉은다른수탉의깃털하나를뽑아냈고각수탉은깃털한개씩을잃었다. 어떤세마리의수탉을골라도그중어느수탉에게도깃털을뺏기지않은한마리가있다. 다음성질이항상만족되는가장작은 k 를찾아라 : 수탉 k 마리를잘골라팔아버리면, 같은닭장안의수탉끼리깃털을뽑아낸일이없도록두개의닭장안에수탉들을잘나눠넣을수있다. ( 몰도바 00 최종 -y0-3) 풀이수탉들이물고물린 cycle들로구성됨.cycle의길이는 3일수없음.일수도없음. 홀수 cycle마다 마리씩팔아버려야함. 홀수 cycle은크기가 5 이상. 즉, 홀수 cycle의최대개수인b n c 5 만큼팔아버리면충분함. n ; 3(mod5) 일땐좀특이해서 ( 남게되는 cycle의크기가애매해서 ) 한마리덜팔아도됨. 답 n =5k; 5k +; 5k +4; 5k +6; 5k +8일때k마리. 특히 n =00이면 5k +6꼴에해당하고 399마리. } 3. A 는좌표평면에서정수좌표의꼭지점을갖는단위정사각형들의유한집합이다. A 의원소의 =4 이상을갖는부분집합 B 를만드는데, B 의서로다른어떤두정사각형도서로공유하는꼭지점이없도록할수있음을보여라. ( 북유럽 00-) 증명격자점을좌표의홀짝에따라 4종류로구분하고, 각칸의좌하꼭지점의색이같은것들만모으면서로만나지않음. 4. 아래와같이 0에서 0까지적혀있는숫자판이있다. 각칸을빨강이나하양으로칠하는데, 빨강칸의수의합이n이다.Maureen은 0이적힌칸에말을놓고동전을열번던진다. 앞면이나올때마다오른쪽으로한칸, 뒷면이나올때마다왼쪽으로한칸말을움직인다. 열번의시행이모두끝난후, 말이빨강칸에놓여있을확률은 a b 꼴의유리수이다. a + b =00일때, n이될수있는최대값을구하여라. ( 캐나다 00-) 풀이 0 번의동전던지기중에앞면이 k 번나올경우의수는 0 k 이고, 이때말의위치는 k 0 이된다. 홀수칸에말이놓여있을확률은모두 0 이다. 0 k 들을구하면 이다. 모든칸의확률이 ; 0; 45; 0; 0; 5; 0; 0; 45; 0; ( ) x 0 꼴이므로 a b 도이런수몇개를더한것이라 a b = x 0 이다. 양변을기약분수로옮겨그분모를생각하면, b= gcd(a; b) j 0, 즉 b = j gcd(a; b) (0 j 0) ( )

360 360 조합 꼴이어야한다는것을알수있다. a + b =00=3 3 9 에서 gcd(a; b) 는 ; 3; 3; 9; 3 3; 3 9; 3 9; 00 등이가능하다. 확률은 0과 사이이므로 0 a, b 이로부터 b a + b b, 즉 00 b 00 이어야한다. 각각의 gcd(a; b) 에대해 ( ) 의꼴로써이범위에드는것을각각찾으면, a b 는다음과같은것들이가능한후보가된다 ; ; ; ; ; ; ; 0 00 이로부터 x = 0 a=b 는다음과같은것들이가능하다. 977; 30; 368; 80; 83; 448; 5; 0 x는 ( ) 의수들중일부를합한것이므로이로부터가능한경우를모두찾아보자. () x =977일때 : 0 x =47도 ( ) 의수들의합이다. 가능한경우는 47=45++ 뿐임을금방알수있다. () x = 30 일때 : 일의자리를따지면 과 5는쓰일수없고45는항상둘이묶여90으로만쓰일수있음을알수있다. 따라서, 합에포함될후보수들은 ( ) 에서다음과같이축약된다 : 0, 0, 90, 0, 0, 0, 0. 0으로나눠생각하면,, 9,,,, 이된다.3으로나눈나머지를따지면 은반드시하나가쓰여야하고, 나머지중에서합이 30이되는것은 + 9 뿐임을금방알수있다. (3) x =368일때 : 일의자리를따지면 45와 은반드시하나씩쓰여야하고 5도꼭쓰여야한다 = 98 을제외하면 70을더만들어야하는데이것은불가능함을금방알수있다. (4) x =80일때 : 해당되는해가없다. (5) x =83일때 : 0 x =9에대해따지면해당되는해가없다. (6) x =448일때 :(3) 에서와비슷하게하면해당되는해가없다. (7) x =5일때 : 일의자리를따지면 5가쓰이거나혹은 이둘다쓰이거나둘중하나라야한다. 그리고,45는항상둘이묶여서만쓰일수있다. (7a) 5가쓰일때는나머지60을 0, 0, 90, 0, 0, 0에서골라합해야한다.0으로나눠생각하면 6을,, 9,,, 에서만드는것이된다.3으로나눈나머지를따지면 은둘다쓰여야하고, 나머지에서 4를만드는경우는 + 뿐이다.(7b) 두개의이쓰일때는나머지50을 0, 0, 90, 0, 0, 0, 0에서만들어야한다.30으로나눈나머지를따지면 0은쓸수없다.30으로나눠생각하면 7을 3, 4, 4, 7, 7로만드는것이되고,3+7+7 뿐임을금방찾을수있다. (8) x =0일때 : 자명하게아무것도합하지않은것. (){(8) 을종합하면x로가능한것은다음이전부이다. 977 = = = = = 공합 ( ) 의수들중같은두수는항상서로부호가다른칸이배정되므로, x에같은수가번포함되거나전 혀포함되지않는경우는 n의합에서는항상서로상쇄되고, 또,5는 0의칸이므로역시 n의합에영 향의없어서, x에오직둘중한번만포함되는수 ( 이들을기여수라고하자 ) 들만이의미를갖는다. 즉, n의최대값은기여수들의양의칸과그리고양의홀수들의칸 ( =5) 을빨강으로칠했 을때발생한다. c 기여수 해당칸 max(n) ; 0 ; 없음 없음 5 5 없음 없음 5 0 없음 없음 5 따라서, 최대의 n 은 35 이다. }

361 4. 조합중급문제 볼록다각형을서로교차하지않는대각선들을그려삼각형들로분할하였다. 그리고각꼭지점마다그꼭지점을포함하는삼각형영역의개수를적고나서앞서그렸던대각선은모두지워버렸다. 꼭지점에적힌수들만으로대각선들을복구할수있는가? (Towns 00봄 JA4) 풀이늘 0 인꼭지점이있으므로, 그런꼭지점을하나씩제거해나가면귀납적으로... OK } 6. 네개의동전이있는데누군가그중에가짜동전이두개섞여있다고주장하였다. 진짜끼리는무게가같고가짜끼리도무게가같은데진짜동전이가짜동전보다무겁다. 천칭을 번만사용하여저주장이맞는지검증할수있을까? (Towns 00가을 JO3) 풀이 :로잰후기울어지면하나씩바꿔서또 :로재면되고, 평형이되면한쪽의 개를나눠 :로재면됨. } 7. n 3 명의사람들로이루어진단체에서각각의회원은짝수명의다른사람들을알고있다 ( 아무도모르는경우도가능하다 ). 같은수의사람을알고있는세회원이반드시존재함을증명하여라. 참고로자기자신은아는사람에포함하지않으며, A 가 B 를알면 B 도 A 를안다. ( 폴란드 00/00 차 -7) 증명그런세회원이없다고하고귀류법. n =m 일때각자가아는사람의수는 0; ; 4;:::;m 가가능한데, 각수마다최대 명씩만배정될수있어서, 어느한수를제외하면나머지 m 개의수에 m 명을배정할수없음. 즉, 각수마다다사람이있음. 그럼아무도모르는사람도있고모두아는사람도있어서모순. n =m 일때도비슷하게보면모든수마다 명씩꼭맞게배정되어야하고, 그럼아무도모르는사람이 명생기는데딱명만모르는사람도있으므로모순. 8. () 한그룹의사람들이어떤파티에참석했다. 각사람은그그룹에서최대 3명의다른사람을알고, 서로모르는두사람은그그룹에서동시에아는사람이반드시있다. 이사람들은최대몇명인가? () 추가적으로이그룹에서로아는세사람이있음을안다면, 이사람들은최대몇명인가? ( 아일랜드 00-) 풀이 () 답은 0명.0명일때의예는정오각형 ABCDE 내부에별 abcde를그리고대응되는대소문자끼리연결한그림.() 답은 8명.8명일때의예는삼각형 ABC의각꼭지점마다대응되는한점씩 a, b, c를연결하고, 남은두점 X, Y 에는 a, b, c를모두연결한그림. () 에서 명이상은불가능함을보이자.A가아는사람을B, C, D라하고 ( 셋보다적어도상관없음 ), 그럼 (A와모르는) 남은사람이 7명이상인데, 그런사람은 A와동시에아는사람이있어야하고그건 B, C, D 중한명일수밖에없으므로, B, C, D 중에는이 7명중3명이상을아는사람이있고 ( 비둘기집 ) 그럼 A까지 4명이상을아니까모순. () 에서 9명이상은불가능함을보이자.A,B,C가서로아는세사람이면이들은한명씩만더알수있고 ( 각각 a, b, c라함. 이들중몇은없어도상관없음 ), 그럼이외에 (A,B,C중누구도모르는 )3명이상이더있는데, 이들각각 A, B, C와동시에아는사람이있어야하고그것은각각 a, b, c일수밖에없어서 a, b, c가 4명씩알게되니까모순. } 9. 자연수 n 이주어져있다. n 명으로이루어진한단체가 6 개의업무를맡고있다. 각각의업무에는 n=4 명이상의사람들이참여하고있다. n=30 명이상의사람을공유하고있는두업무가존재함을증명하여라. ( 폴란드 00 차 -3) 증명업무 i를맡은사람수를ai, 업무 i와 j를동시에맡은사람수를 b ij 로나타내자. 딱하나의업무만맡은사람수를x라하면, 귀류법으로, n x P a i P b ij > 6 n 4 6 n 30 = 3 n n = n 으로모순. 30. 모든항이 f; ;:::;mg 의원소이고, 길이가 k 인모든수열들의집합을생각하자. 각각의수열에서가장작은항을뽑고, 그값들을더하자. 그러면그합은 k + k + + m k 임을보여라. ( 폴란드 00/003 차 -4)

362 36 조합 증명 ( 부산데레사여고 학년방윤희 ) 모든수열은 m k 개이고, 가장작은항은 이상이다. 을제외한 m 개의원소들로만만들수있는길이 k 인수열은 (m ) k 개이고, 가장작은항은 이상이다., 를제외한 m 개의원소들로길이 k 인수열을만들면 (m ) 개, 가장작은수는적어도 3.,, :::; m 을제외한 개의원소로길이 k 인수열을만드는경우의수는 k 개, 가장작은수는 m. 모든 m k 개의수열에서최소 씩더해지고그중 (m ) k 개에서는최소항이 이상이므로 이추가로더해진다. 또, 그중 (m ) k 개에서는최소항이 3 이상이므로 이다시추가로더해지고,... 이렇게각각의경우의수만큼 을더한것이구하는합이므로문제에서처럼 k + k + +(m ) k + m k 이된다. 3. 셈타우르라는별은 3 개의구멍이뚫린도넛꼴이라고한다. 이별의표면에평면그래프를그리면 v e + f 가어떤값들을가질수있는지모두찾아라. ( 셈본중등중급도전문제 5..) 풀이입체 ( 별 ) 에서한개의고리를자르되그단면이꼭지점을지나지않도록하자. 그리고단면과원래변과의교점을추가된꼭지점으로하고단면과면과의교선을추가된변으로하며 개의단면을추가된면으로하면이입체는구멍이하나줄어든모양이된다. 이때단면이k각형이라면새로만들어진그래프의점, 선, 면의개수는 v 0 e 0 + f 0 =(v +k) (e +3k)+(f + k +)=(v e + f)+ 로오일러식의값이 만큼늘어난다. 즉, 구멍이 n개라면이구멍을차례로모두제거했을때 v e + f의값이 n 증가하여일반다면체에서의값인 가되므로, 원래의그래프는 v e + f = n 이다. 이문제에서는 n =3일때이므로 답 4 } 3. 다음은파스칼의삼각형을 3차원으로확장시켜파스칼의사면체를만들고그것의단면을차례로나열한것이다 파스칼의사면체의원리를설명하고, 각단면의수의합을구하여라. 또, 파스칼의삼각형의수들을 nc r = n! r!(n r)! 과같이나타내듯이, 파스칼의사면체의수들을표현하는식을찾아라. ( 셈본중등고급도전문제 5..) 풀이꼭대기부터 h 번째층, n 번째행, r 번째열의값을나타내는함수는 f(h; n; r) 이라하자. (h,h,0) (h,h,h) r f(h,n,r) n (h,0,0)

363 4. 조합중급문제 363 f (0,0,0) f (,,0) f (,,) f (,0,0) f (,,) f (,0,0) f(h; n; r) 의값은꼭대기층부터 h, n, r 점까지모서리를다라내려가는최단거리의경우의수이다. (h, n,r) b c (h+, n+, r) a (h+, n+, r+) (h+, n, r) 임의의한층에서다음층까지내려가는경우의수는 ~a, ~ b, ~c 의세경로분이고임의의한점 (h; n; r) 이 ~a 를따라한번가면 (0,0,0) 부터시작해서 (h; n; r) 점에도착하려면 ~c 경로 r번, ~ b경로 n r번, ~a 경로 h n번을움직여야한다. 즉 aaaa abb bcc c {z } {z } {z } h n n r r 을배열하는경우의수다. h! (h n)!(h r)!r! } 33. 합하는순서를무시하고 0 을 의지수형태의자연수의합으로표시하는가지수는얼마인가? ( 예를들어, = + = = 이므로 4 를위의형태의합으로표시하는가지수는 4 가된다.) ( 한국 003 차 -S) 풀이 0 을하나이상포함하는경우와그렇지않은경우로나누면 f(n +)=f(n) =f(n ) + f(n) 의점화식을얻을수있다. 이것으로좀계산해주면 f(0) = 60 이된다. } 명의선수가참가하는체스토너먼트가있다. 어떤두선수도서로많아야한번경기하고, 어떤다섯선수도서로모두경기를하지는않는다. 이토너먼트는최대 4 경기로이루어져있음을보여라. ( 아일랜드 003-4) 증명 ( 장영실과학고 학년김윤섭 ) 우선 8명의선수를 ABCDEF GH라하겠다. 8 7 =8 개의가능한쌍이있다. 먼저 4경기가가능함을보이자. AE, BF, CG, DH만을제외해보자.5명을고르면 4보다크므로비둘기집의원리에의해그것은 fa; Eg; fb; Fg; fc;gg; fd; Hg 중에적어도하나를포함해야한다. 이에의해서어떤 5명의선수도서로모두경기를하지않는다. 따라서 4경기는성립한다. 5경기이상은불가능함을보이자.3개의쌍을제외하는경우는 5가지경우가있다. () 6명의선수가다르다. ex) AB, CD, EF () 5명의선수가다르다. ex) AB, AC, DE (3) 4명의선수가다르다. ex) AB, AC, AD

364 364 조합 (4) 4명의선수가다르다. ex) AB, AC, BD (5) 단 3명의선수가다르다. ex) AB, BC, CA 이경우들모두다음의 5 명이있어각각성립하지않는다. () fg, H, A, C, Eg () ff, G, H, B, Cg (3) fe, F, G, H, Ag (4) fe, F, G, H, Ag (5) fd, E, F, G, Hg 35. n n 크기의박물관이한변의길이가 인정사각형꼴의 n 개의방을가지고있다 (n >). 모든인접한두방 ( 벽을공유하는두방 ) 사이에는문이있다. 경비원이박물관의각방을순찰하려고한다. 처음에아무방에서나출발할수있고, 각방에서정확히 분동안살펴본후에문을통해다른방으로간다. 전에들렀던방을다시들를수있고, 순찰이끝났을때각방을정확히 k 분씩살펴본상태여야한다. 이렇게순찰할수있는 n 과 k 를모두찾아라. ( 이탈리아 003-) 풀이 n이짝수일때는순환경로가있으므로 k번순환하면언제나그런순찰이가능하다. n이홀수일때, 일단 k =일때는가능함을쉽게확인할수있고, k 일때는불가능함을보이자. 체스판처럼흑백으로칸을칠하면 ( 코너는흑색 ) 흑칸이백칸보다하나더많아서 m개라하면, 항상흑백을번갈아다니므로, 흑칸에서백칸으로넘어가는횟수는흑칸의입장에서는 km 또는 km 번 ( 후자는흑칸에서끝날경우 ), 백칸의입장에서는 k(m ) 또는 k(m ) 번 ( 후자는백칸에서출발한경우 ) 인데, 그럼이둘이같아야하므로 jkm k(m )j, 즉 k. } 36. 세개의연속한정수를포함하지않는집합을못마땅한집합이라말하기로하자. 공집합도못마땅한집합으로여긴다. f; ; 3; 4; 5; 6; 7; 8; 9; 0g 이집합의못마땅한부분집합의개수를구하여라. ( 영국 003/004 차 -4) 풀이 (KAIST 과학영재센터연구원고봉균 ) 집합 f; ;:::;ng의못마땅한부분집합의개수를 a n 으로나타내자. a n 의못마땅한부분집합은 n이없거나 (a n 개 ), n은있다면 n 이없거나 (a n 개 ), n과 n 이다있다면 n 가없어야한다 (a n 3 개 ). 따라서, 공집합도못마땅한집합으로여긴다고했으므로, 다음의점화식이성립한다. a 0 =; a =; a =4; a n = a n + a n + a n 3 (n 3) 이점화식에의해쭈욱계산해주면 n a n 와같이된다. 즉, a 0 =504 개 답 } 37. 어떤볼록다면체의각면이삼각형, 사각형, 오각형, 또는육각형이라하자. 이다면체에삼각형면이두개, 사각형면이한개있을때, 다음중옳은것은어느것인가? () 오각형면은세개, 육각형면은두개있을수있다. () 오각형면은두개, 육각형면은세개있을수있다. (3) 오각형면은네개, 육각형면은한개있을수있다. (4) 오각형면은두개, 육각형면은두개있을수있다. (5) 오각형면은한개, 육각형면은네개있을수있다. ( 한국 004 차-J8) 풀이다면체의꼭지점, 모서리, 면의개수를각각 v, e, f라하자. 그럼다음의사실들이성립한다. (a) v e + f =( 오일러의공식 ) (b) b := P ( 각면의모서리의개수 )=e (c) P ( 각꼭지점에연결된모서리의개수 )=e

365 4. 조합중급문제 365 특히 (c) 에서, 각꼭지점에는최소 3 개의모서리가연결되어있어야하므로 e 3v 의부등식이성립한다. 보기 (){(5) 의값들을모두표로나타내어보면다음과같다. n 3 n 4 n 5 n 6 f b e = b= v = e f + e 3v () = () 땡 (3) 성립 (4) 땡 (5) = 따라서, 가능한경우는 (3) 뿐이다. 실제로, 정육면체의한면에서마주보는두꼭지점을조금잘라내면 (3) 의조건을만족하는다면체가됨을확인할수있다. } 별해오각형이 m개, 육각형이 n개라하자. 그럼다음이성립. f =3+m + n e =0+5m +6n 3v 0 + 5m +6n 이식들을 v e + f = 에대입하면 m 4. } 38. 동전을 0 번던졌을때, 뒷면이 6 번나오고앞면이연속해서 5 번이상나오지않는경우의수는얼마인가? ( 한국 004 차 -J6) 풀이 0번중에서뒷면이 6번나오는모든경우들의집합을 U라하자. 그리고, 뒷면과뒷면사이에는앞면이연속하여나오는구간이모두 7개있으므로, 이들의길이를각각 b ;b ;:::;b 7 이라하자. 예를들어 OXOOOOXXOOOOOXOOOXOX =) (b ;b ;:::;b 7 )=(; 4; 0; 5; 3; ; 0) 와같이대응시키는것이다. U 에서 b i 5 인경우들의집합을 A i 라하자 (i =; ;:::;7). 그럼우리가구하는것은다음과같다 ( 포함과배제의원리 ). jxj = ju (A \ A \ \A 7 )j = juj X ja i j + X ja i \ A j j X ja i \ A j \ A k j + 먼저 juj를구해보자. 이것은 0번의시기중에서뒷면이되는 6번을고르는경우의수이므로 juj = 0 6 이다. 다음으로 ja i j, 즉 b i 5 인경우의수를구해보자. 이것은 5번의동전시기를먼저제외해놓고뒷면의위치를정한후 b i 의자리에앞면 5번을추가로끼워넣는것과같으므로, 그경우의수는 ja i j = 5 6 이다. 비슷하게, ja i \ A j j = 0 6 이고, jai \ A j \ A k j = 5 6 =0으로이후는더계산할필요가없다. 따라서, jxj = juj X ja i j + X ja i \ A j j 0 ³ 0 ³ 7 ³ 5 ³ 7 ³ 0 = 로, 이것을계산하면답은 835 이다. } 39.,, 3, 4를사용하여만든 9자리의자연수중에서다음조건을만족시키는것의수는얼마인가? ( 가 ) 최고자리수는 이다. ( 나 ) 같은숫자가연속하여나타나지않는다. ( 다 ) 일의자리수는 이아니다. ( 한국 004 차-S8)

366 366 조합 풀이위와같은성질을갖는 n자리자연수의개수를 an (n ) 이라하자.( 가 ) 와 ( 나 ) 를만족시키는 n자리자연수는최고자리다음부터그앞자리와다른숫자를고르면되므로 3 n 개가있다. 이중에서 ( 다 ) 를만족하지않는것, 즉마지막자리수가 인것만빼주면된다. 마지막자리수가 인것은그앞자리가 이아닌것의개수만큼있다. 그리고이것은 a n 가지가된다. 이제 a 9 를구하면, 점화식을반복적으로풀어 a n =3 n a n (n 3) a 9 = a = 이된다. 이것을계산하면 490. } 40. m 은자연수이고 n = m 일때, 집합 M(n) =f; ;:::;ng 에대해다음을증명하여라 : M(n) 의원소들을적당한순서 a ;a ;:::;a n 으로잘늘어놓으면, 어떤 i<j<k n 에대해서도 a j a i 6= a k a j 가성립한다. ( 오폴 004-6) 증명 m =일때, 3,, 4로하면됨. 이후엔귀납적으로, n = m 일때a ;a ;:::;a n 으로잘늘어놓았다면,n = m+ 일때는a ; a ;:::;a n ;a ; a ;:::;a n 로하면됨. 등차삼항이앞쪽그룹과뒷쪽그룹에나뉘어있다면홀짝성에서모순이생기고, 한쪽그룹에몰려있다면 n = m 일때의귀납법의가정에의해모순이됨. 4. 칠판에수,, 3, 4, 5, 6, 7, 8, 9 가적혀있다. 두사람 A 와 B 가번갈아게임을한다. 각사람은자기차례에서칠판의수중하나를골라지운후, 그것의배수들도함께지운다. A 가먼저시작하고, 마지막수를지우는사람이지는것으로한다. 두사람중에누가필승의전략을갖는지결정하고, 그전략을서술하여라. ( 중미 004-) 풀이 A 가 4 를먼저지우고, f6; 9g, f; 3g, f5; 7g 를짝으로보고게임하면이김. } 4. ; ; 3;:::;004 를재배열한수열 a ;a ;a 3 ;:::;a 004 중에, ja j = ja j = = ja j > 0 을만족하는것의개수를구하여라. ( 호주 004-3) 풀이수열대신함수의방식으로 ai = a(i) 로쓰기로하자. 그리고, a 의역함수를 a 0 으로쓴다. d = ja() j = = ja(004) 004j (> 0) 로두자. 그럼 d = ja 0 () j = = ja 0 (004) 004j > 0 이기도하다. 우선 a() > 0 이므로 a() = d+ 이다. 마찬가지로 a 0 () > 0 이므로 a 0 () = d+ 이다. 즉, 과 +d 는서로짝을이뤄대응한다. 비슷하게, Ã! d + Ã! d +. d Ã! d + d 들이각각서로짝을이뤄대응하게된다. 이렇게 부터 d 까지모두그안에서대응이완료되므로,d+ 이상의수들도마찬가지로생각하면 d +Ã! 3d + 과같이대응하게된다. 즉, a = a 0 이고 (k)d + r Ã! (k +)d + r ( r d) 와같이짝을이뤄대응한다. 이런대응은 d 개씩묶여닫힌대응을하므로,d j 004, 즉 d 는 00 = 3 67 의양의약수들만이가능하고, 또그럴때 a(n) 이항상유일하게결정된다. 따라서, 구하는개수는 ( + )( + )( + ) = 8 개. }

367 4. 조합중급문제 번부터 005번까지번호가매겨진 005개의카드를 번카드가맨위, 그아래번카드, 그아래3번카드, :::; 005번카드가맨아래에오도록쌓은카드더미가있다. 이제,005번카드부터시작하여카드더미의맨아래의카드는버리고, 다음카드는카드더미의맨위로보내고, 다시맨아래의카드는버리고, 다음카드는카드더미맨위로보내는식의작업을계속하기로하자. 마지막에남는카드의번호를구하여라. ( 한국 005 차-S9) 풀이 (KAIST 과학영재센터연구원고봉균 ) 카드의번호를 씩줄여0번부터 004번까지의카드로생각하고, 이때카드가 0번부터 n번까지일때마지막에남는번호를 h(n) 이라하자. ² n =m + 일때 ;0 m m + 에서 m + 번시행하면 0 m m 이된다. 다시 m부터버리기시작하므로이것은 0번부터 m번까지의카드로시작하는것과같고그럼 h(m) 번카드가마지막에남는다. i번카드는 i이므로, h(m +)=h(m). ² n =m 일때 ;0 m m에서 m번시행하면 3 m 3m 0이된다. 0은버려질카드이므로 m + 이대신있는것으로생각해도되고, 그럼 0번부터 m번까지의카드로시작하는것과같으므로 h(m) 번카드가마지막에남는다. 여기서 i번카드는i + 이므로 h(m) =h(m)+. 따라서, h(n) 은 h(0) = 0, h(m) =h(m)+,h(m +)=h(m) 의점화식을갖는다.진법으로생각하면, n 일때h(n) 은 n을이진법으로전개한후각자리의 은모두0으로,0은모두로바꿔놓는함수가된다. 따라서, h(004) = h(0000 () ) = 00 () =43이고, 원래의문제로다시옮기면마지막에남는카드의번호는 44가된다. 답 44 } 44. (a) 다음을만족하는평면위의 5개의점을찾을수있음을증명하여라 : 이중에서세점을골라만들수있는직각삼각형이 8개이상있다. (b) 다음을만족하는평면위의 64개의점을찾을수있음을증명하여라 : 이중에서세점을골라만들수있는직각삼각형이 005개이상있다. ( 주니어발칸 005-3) 증명 (a) 정사각형의중심에점하나를찍으면됨. (b) 8 8 로배치하면됨. 여기서임의의한점 A 를택했을때 A 를직각으로하고격자에평행한옆변을갖는직각삼각형 ABC 를세어보면, A 를포함하는가로줄과세로줄에서각각점 B 와 C 를택하면되므로, A 를택하는경우의수 8 에 B 와 C 를택하는경우의수 7 을곱하면,8 7 > 50 = 500. 격자에평행하지않은옆변을갖는직각삼각형도많을테니 문제가출제되는어떤수학경시대회가있다. 각각의문제마다학생들은 6, 5,, 0 점중한가지점수를받게된다. 어떤두참가자 A, B 를골라도두사람이받은점수가서로다른문제가적어도둘있다고한다. 이것이가능한최대의참가인원을구하여라. ( 폴란드 005/006 차 -4) 풀이참가인원은 N명.6문제의점수를차례로나열한것을점수열이라고하자. 모든참가자들의점수열의집합을 A, 그렇지않은점수열의집합을 B라하자. A 각각의점수열과, 그것과딱한과목만점수가다른 B의점수열을변으로잇기로하고, 이변의개수를두가지관점에서세자. A의입장에서는 6과목각각의점수가다른 3종류로바뀔수있으므로 8N. B의입장에서는 6과목각각에대해그과목만점수가다른 A의점수열이많아야하나씩뿐이므로 6(4 6 N). 즉 8N 6(4 6 N), 3N 4 6 N, N 4 5. 실제로등호가성립하는경우가다음과같이있다 : 네가지점수를6, 5,, 0점대신3,,, 0점으로하고, 총점이 4의배수인점수열들을모두모아 A로하면된다. } 46. 세수가주어지면그중한수는그대로두고, 나머지두수를그두수의합과곱으로각각대체하는조작을할수있다.(3; 4; 5) 에서시작하여이런조작을유한번거쳐서, 직각삼각형의세변의길이를이루는또다른세수를얻을수있을까? ( 폴란드 차 -4) 풀이 ( 홀, 홀, 짝 ) 을 A 타입,( 홀, 짝, 짝 ) 을 B 타입이라하자.A 타입은다시 A 타입이되거나아니면 B 타입이됨.B 타입은항상다시 B 타입이됨.B 타입은피타고라스의정리에서홀짝이맞을수없으므로직각삼각형이생길수없다. 따라서, 항상 A 타입만계속반복하여만들어야함. 이때,` 짝 ' 이계속하여연산에참여하게되는데, 한번작업한이후에는이 ` 짝 ' 이항상다른두수의합이상이되어삼각형자체가구성될수없음. } 47. 어떤써커스단에 n 명의광대가있는데, 이들은 가지색에서적절히골라옷을입고분장을칠한다. 각각의광대는최소한다섯가지색을사용해야한다. 어느날이써커스단의단장이지시하기를, 두명의광대가정확히같은색을사용하면안되고한가지색마다최대 0 명의광대만그색을쓸수있다고하였다. 단장의지시가성립할수있는광대의수 n 의최대값을구하여라. (APMO 006-5)

368 368 조합 풀이 ( 서울중앙중 학년이수홍 ) 각광대가사용한색의총수를두가지방법으로셀것이다. 각각최소 5 가지색을쓰므로그수는 5n 이상이다. 그런데한가지색마다최대 0 명이쓸수있으므로그수는 0 = 40 이하이다. 따라서, 5n 40 =) n 48 이제 n =48 일때가능함을보이겠다.5 개씩사용하고그림과같이한칸씩밀리게돌리면 개가되고이것을 4 가지종류로하면된다 따라서 n 의최대값은 48. } 48. 칠판에수 이쓰여져있다. 두사람 A 와 B 가번갈아다음중한가지조작을하는게임을한다. (i) 칠판의한수 x를택해, ab = x 인 보다큰두정수a, b로대체한다. (ii) 칠판에같은두수가있을때그중하나혹은둘다를지운다. 더이상위의조작중어떤것도실행할수없는사람이지게된다.A 가먼저시작하고둘다최선의경기를한다고할때누가이기겠는가? ( 북유럽 007-3) 풀이 A 가먼저 을 007 과 으로분리하고나서,B 를따라대칭플레이. 대칭성의게임. } 49. 한변의길이가 인정삼각형타일 n 개가모여한변의길이가 n 인정삼각형을이루고있다. 각각의타일의한면은흰색이고반대쪽면은검은색이다. 이제다음의조건을만족하는타일 P 를하나골라뒤집는시행이허용된다 : P 와변으로접하는타일들중에윗면이 P 와다른색인타일이둘이상이다. 각각의 n 에대해, 이런시행이무한히계속될수있는초기상태가존재하는지알아내어라. ( 폴란드 007 차 -3) 풀이가능. 서로다른색의두타일이만나는쌍의개수가계속줄어든다. 따라서, 이런시행은늘유한번만 } 의카드판에서두사람이놀이를한다. 카드판은현재비어있고, 그옆에부터 9까지의서로다른숫자가적혀있는9개의카드가, 숫자가적힌면을위로하여선수들앞에놓여있다. 각선수는번갈아서자기차례에카드를하나골라판위에놓는데, 모든카드가없어질때까지계속한다. 첫번째선수는첫번째행에놓인카드의합과세번째행에놓인카드의합이각각두번째행에놓인카드의합보다크면이기고, 작거나같은것이있으면진다. 어떤선수가어떤방법으로이기겠는가? ( 모의MO J4) 풀이첫번째선수가이긴다. 카드판의세행을 AaD/Bbx/Ccd 로할때, 먼저 x 에 을넣고, ( 9), (3 8), (4 7), (5 6) 을짝으로볼때나중에하는선수의플레이에따라서짝이되는칸 (A 와 a, B 와 b, C 와 c, 그리고 D 와 d 가서로짝이되는칸임 ) 에짝이되는수가들어가도록하면된다. } 4. 조합고급문제. 자연수의임의의무한수열이 3 개주어져있다. 각각의수열에서 m 번째수가 n 번째수보다작지않은서로다른자연수 m, n 을찾을수있음을증명하여라. ( 소련 96-) 증명세개의수열을ai, b i, c i 라하자. a i 에서감소하지않는부분수열 a ji 를먼저잡고, 이인덱스를그대로가져온수열 b ji 에서다시감소하지않는부분수열 b ki 를잡고, 다시수열 c ki 에서감소하지않는부분수열 c mi 를잡으면, m i 를인덱스로하는세부분수열이모두감소하지않는수열이므로...

369 4. 조합고급문제 369. 평면위에 n ( 3) 개의점이주어져있다. 어느세점도한직선위에있지않을때, 이점들을꼭지점으로하는단순다각형이존재함을증명하여라. 단, 단순다각형이란어떤변도서로가로지르거나접하지않는 ( 보통의 ) 다각형을말한다. (Putnam 966-B5) 증명 Take arbitrary x, y axes. Take P to be the point with the smallest x-coordinate, or if there are two such, the one with the smaller y-coordinate. Take Q to be the point with the largest x-coordinate, or if there are two such the one with the smaller y-coordinate. Join P to Q by a path p along the lower part of the convex hull of the points (so that all other points are above the path). Now order the remaining points not in the path according to the size of their x-coordinate (with the largest first). Continue the path from Q back to P by taking the remaining points in this order. We can never intersect the existing path p without going outside the convex hull and we cannot intersect the later part of the path because it has smaller x-coordinate. Comment. It is tempting to argue by induction. Take a path through the first n. Then given another point P, we can always find two earlier points Q and R, so that PQ and PR do not intersect the n-path. Then replace the segment QR by QP and PR. Unfortunately, this is false. It is not too hard to find cases where we cannot find such Q, R. 3. 두사람이번갈아가며 ; ;:::;0 의수들중에서하나를택해그수의부호를결정한다. 한번부호가결정된수는다시바꿀수없다. 첫번째사람은최종결정된수들의총합의절대값을작게만들려고하고, 두번째사람은크게만들려고한다. 두사람이모두최선을다했다고했을때결과는어떻게되는가? 예를들어, 두사람이, 0, 9, 8, 7, 6, 5, 4, 3,,, 0, 9, 8, 7, 6, 5, 4, 3, 와같이경기할수있다. 그러나이예는두번째사람이최선을다하지않은것이다! ( 소련 966-) 풀이결과는 30이된다 (0 대신 n으로일반화해도결과는 3n이된다 ). 첫번째사람 ( 갑 ) 에게항상 30 이하를만들수있는전략이있고, 두번째사람 ( 을 ) 에게항상 30 이상을만들수있는전략이있음을보이면된다. 을의전략은다음과같다 :(; ), (3; 4), (5; 6), :::; (9; 0) 으로짝을짓고,9과 0은같은부호가되도록하고, 나머지짝들은모두둘이서로다른부호가되도록한다 ( 갑이부호하나를결정하면을은그것과짝이되는수의부호를결정한다 ). 그럼최소 (9 + 0) 9 =30이상을만들수있음이보장된다. 갑의전략은다음과같다 :() 먼저 을 + 로결정하고,(; 3), (4; 5), :::; (8; 9) 와같이짝을짓고0은홑수라고명명한다.() 을이어느짝에속하는수의부호를결정하면, 갑은그와짝이되는수를반대의부호로결정한다.(3) 을이홑수의부호를결정하면, 남아있는수중에서가장큰수 (a라하자) 를그홑수의짝으로삼아반대부호로결정하고, a와짝이었던수 ( 당연히 a ) 를새로이홑수로명명한다. 이런전략에의하면항상 30 이하가됨을확인해보자. 먼저 0이맨마지막에결정된다면결과는최대 = 30 이됨이자명하다.0이도중에결정되었고,0의짝을a, 또 (a ) 의짝을a,... 이렇게가다가 a k 이맨마지막에결정된홑수라고하자. 즉,(0;a ), (a ;a ), :::; (a k ;a k ) 들이서로짝을이루었고, 이게 k개의짝이므로처음의짝그대로결정된경우는그럼 9 k개이다. 각각의 a i 들은그때까지남아있던가장큰수였으므로각짝에서 a i >a i+ 임은당연하다. 그럼결과는최대 +(9 k) +f(0 a )+((a ) a )+ +((a k ) a k )g+(a k ) = +(9 k)+0 k =30 k 이므로역시 30 이하이다. } 4. 학생들이몇명있는데그중영어를하는학생은 50 명, 프랑스어를하는학생은 50 명, 스페인어를하는학생도 50 명이다. 그리고몇몇학생은둘이상의언어를사용한다. 이학생들을 ( 꼭서로크기가같을필요는없는 )5 개의그룹으로분할하는데, 각각의그룹마다영어를하는학생이 0 명, 프랑스어를하는학생도 0 명, 스페인어를하는학생도 0 명이되도록할수있음을보여라. ( 소련 968-4) 증명영어, 프랑스어, 스페인어중하나만하는학생의수를각각 E, F, S라하고, 하나만못하는학생의수를 e, f, s, 셋모두하는학생의수를 A 등으로나타내자. 영어와프랑스어가둘다 50명씩임에서, E + f = F + e, 즉 E e = F f = S s = a 로상수임을알수있다. E와 e에서한명씩뽑아짝지어한집합을만들자. 그럼이집합은영어, 프랑스어, 스페인어를하는학생이꼭한명씩있는집합이된다. 이것을 `홑세트 ' 라하자. F 와 f, 그리고 S와 s에서도마찬가지로이런세트를구성한다. 이렇게

370 370 조합 더이상짝지을수없을때까지작업하고남은인원을 E 0 ;F 0 ;S 0 ;e 0 ;f 0 ;s 0 이라하자. 그럼 (E 0 ;F 0 ;G 0 ) 과 (e 0 ;f 0 ;s 0 ) 은둘중한쪽은 (0,0,0) 이고나머지한쪽은 (a; a; a) 꼴이된다. E 0 = F 0 = G 0 = a 이면여기서각각한명씩뽑아짝지어홑세트를구성할수있다. e 0 = f 0 = s 0 = a 이면여기서각각한명씩뽑아한집합을만들면각언어마다 명씩있는집합이된다. 이런집합을구성하여 ` 겹세트 ' 라고부르기로하자. 그리고,A 에서는한명씩으로각각홑세트를구성할수있다. 즉, 모든학생들을홑세트와겹세트로분할하였다. 그럼겹세트를적당히먼저나누어담고, 남는인원만큼홑세트를나누어담으면, 각각의그룹 G i 마다각언어를하는학생이적당한짝수 b i 명씩으로똑같은상황은늘잘구성된다. 5. (a) 4 4 표에각칸에모두 + 부호가적혀있는데, 귀퉁이가아닌칸하나에만 부호가적혀있다. 한행이나한열, 또는한대각선을택해서그한줄의부호를모두바꿀수있다. 대각선은주대각선만이아니라길이 4짜리에서 짜리까지모두를말한다. 이런과정을몇번반복해도모든칸의부호가 + 가되게할수는없는처음 부호의위치를모두구하여라. (b) 8 8 표로바꾸어생각하면어떻게되는가? ( 소련 968-) 풀이 (a) 둘레에서귀퉁이들만제외한여덟위치 ( 이런여덟칸의집합을좋은 8-위치라부르자 ) 만을생각하면, 이안에들어있는 부호의개수의홀짝은항상불변이다. 즉, 이중에딱하나만 였다면 ( 가홀수개 ) 이여덟위치가모두 + 가될수 ( 가짝수개 ) 는없다. 즉, 귀퉁이가아닌둘레위의칸은모두불가능한위치. 나머지내부의네위치는모두가능하다 : 그중한칸을생각하면, 그칸을지나는네방향의부호바꾸기를모두작업하고, 그동안한번도바뀌지않은네칸도적절한대각선으로그것들만딱한번씩부호가바뀌도록작업하면, 원래주목했던칸은 4번, 나머지모든칸은딱 번씩부호가바뀌었다. 즉, 이제다시모든칸의부호를뒤집어주면원래주목했던칸만 5번, 나머지모든칸은부호가바뀌지않았다.(b) 좋은 8-위치를안에서얼마든지움직여줄수있으므로귀퉁이를제외한어떤칸이든그칸을포함하는좋은 8-위치가있어서 부호가어떤칸에있어도불가능하다. } 6. `n 개의변을갖는어떤다각형은정사각 n n 배열의 n 개의격자점을꼭지점으로가질수있다 ' 는명제를 P (n) 이라하자. P (4) 와 P (5) 의참거짓을밝히고증명하여라. n 6 일때의P (n) 의참거짓에관해예측하고, 약간의해설을주어라. [ 주 ] 다각형은상식적으로다음과같은성질을만족해야한다 :(a) 두변이서로겹칠수없다.(b) 이웃하지않은변은서로만날수없다.(c) 이웃한변은방향이같지않다. ( 영국 969-9) 풀이 P (4) 는최외곽을도는정사각형의각변의중간에서안쪽점을한번씩방문하도록경로를수정하면금방됨. P (5) 는중앙의 3 3 아홉점 (B) 와외곽의정사각형 6점 (A) 를생각할때,A끼리연결되는경우는각변마다 번씩최대 8번이고, 그럼 A에서 B로는적어도 6번 ( 왕복으로는 8번 ) 오고가야하는데,B의점중에서도중심의점에연결하는경우는바로다시 A로복귀할수없어서어쩌구저쩌구따져보면 B 내부끼리의연결도적어도 번, 그래서 A와 B를왕복8번연결할수가없어짐.( 혹은폐곡선의일부 ( 연속한몇개의선분열 ) 가전체영역을공집합이아닌두동강으로내면곤란하다는것을이용해서도중앙에연결되는두개의선이모두 A로부터연결된것이아님을말할수있음 ) P (6) 이상의짝수에서는 P (4) 로부터귀납적으로외곽을확장할수있음. P (7), P (9) 도가능하고, P (n) 으로부터 P (n +4) 로외곽을확장시킬수있음 ( 그냥 P (7) 말하고, P (n)! P (n +) 로말할순없을까?). } 7. 어떤체스대회에 p명이참가했다. 이들은서로많아야한번씩경기를한다. n번의게임이끝난뒤에관찰해보니어떻게세명을골라도그중에두명은서로경기한적이없었다. n p 임을증명하여라. 4 ( 독일BW 97 차-4) 증명 ( 신한솔 ) 각각의사람을점으로생각하자. 두사람이경기를하면이두사람에해당하는점을연결하자. 그럼변의개수가 n개이고 k3이존재하지않을때 n p 4 임을증명하는문제와같다. 임의의변 AB를생각하자. A와 B가인접하므로 A에인접한점들중에서 B에인접한점들중일치하는점이있으면삼각형이생기므로모순이다. 따라서 A에인접한점들과 B에인접한점들중겹치는게없다. 그러므로 d(a)+d(b) p. 모든변에대해이부등식을쓰고다더하자. A를포함한변은총 d(a) 개이고각각의변마다 d(a) 개씩세어지므로 d(a) 이된다. 다른꼭지점들도마찬가지이므로좌변은 P AV d(a) 이고우변은pn이므로 X d(a) pn AV 코시부등식에의해 ( + +) X ³X d(a) d(a) =4n Av

371 4. 조합고급문제 37 따라서 4n p pn, n p 두형제가 n 개의황금조각을상속받았고황금의총무게는 n 이다. 각각의황금조각의무게는자연수이며가장무거운조각의무게는남은모든조각의무게합보다무겁지않다. n 이짝수이면두형제는황금조각을부수지않고도유산을공평히둘로나눠가질수있음을보여라. ( 독일 BW 975 차 -4) 증명각조각의무게를 a a a n n 이라하고 S i = a + + a i 라하자. 비둘기집에의해a a n;s ;S ;:::;S n 중에 mod n으로같은두수가있다.(i)s i S j (mod n) 인 i<j 가있을때 : 0 <S j S i < n 이므로 S j S i = n. 따라서, 한명이a i+ + + a j 를가지면됨. (ii) a a n S i (mod n) 인 i 가있을때 : i =이면 a n = n 이므로 ok. i = n 이면 a a n 0 (mod n), 즉 a = = a n =이므로 ok. <i<n일때는 (a + a a i )+a n = n 이므로 ok. 9. 아버지, 어머니, 아들세사람이비기는경우가없는 인용보드게임토너먼트를한다. 토너먼트의규칙은다음과같다 : (i) 게임에가장약한사람이맨처음경기를할두사람을고른다. (ii) 한경기에이긴사람이바로다음경기를나머지한사람과갖는다. (iii) 가장먼저두경기를이긴사람이토너먼트의우승자가된다. 아버지가가장약하고, 아들이게임에가장강하며, 각두사람간에한쪽이이길확률은토너먼트가벌어지는동안변하지않는것으로간주한다. 토너먼트에서우승하기위한아버지의최선의전략은맨처음경기를어머니랑둘이하는것임을증명하여라. ( 미국 974-4) 0. 각각 40 개씩의원소를갖는 978 개의집합이있다. 임의의두집합은항상딱한개의공통원소를갖는다. 978 개의집합전체가하나의공통원소를가짐을증명하여라. ( 오폴 978-5) 증명 ( 이수홍 ) 각집합이 A; ;A978 이라하자. 또 A \ \A978 이공집합이아니라고가정하고모순을보이자. A; ;A978 은 A 과딱하나의공통원소를가지므로 A =fa; ;a40g 일때그교집합이 a 인것,, a40 인걸로분할된다. 따라서 [977=40] = 49:43 이므로어떤 ai 가존재해서그것을교집합으로가지는집합이 50 개이상이된다. 일반성을잃지않고이것을 a 이라하자. a 을포함하는집합을일반성을잃지않고 A;A; ;A5 이라하자. 가정에의해 a 을포함하지않는집합이있다. WLOGA5 라하자. A5 는 A; ;A5 과하나씩의공통원소를가지는데 A5 의원소의개수는 40 개뿐이므로 A5 의원소 x 가존재해서 A 5 중 x 를포함하는것이두개이상이된다.( 자명히 x 는 a 이아니다.) 이것을 Ai; Aj 라하면 Ai 와 Aj 의교집합이 a;x 를포함해서공통원소가하나라는가정에모순. 따라서집합전체가하나의공통원소를갖는다.. 아홉명의수학자가어느국제회의에서만났는데, 그들중어느세명을택해도그중같은언어로말할수있는두명이항상있음을발견하였다. 각수학자는최대세가지언어로말할수있다고할때, 같은언어로말할수있는세명의수학자가있음을증명하여라. ( 미국 978-5) 증명 a와 b가서로말이통하지않는다면다른 x는 a나 b 중적어도한명과말이통해야함. 그럼 a와 b 중에 d 7 e =4 명이상과말이통하는사람있음 (a라하자). a는 3개이하의언어를구사하므로, a의언어중어느하나는 a와말이통하는사람들중 d 4 e = 3 명이사용함. 그럼 a를포함하여 3명이되므로끝. 만일서로말이통하지않는 a, b를잡을수없다면모든사람이서로말이통해야하고, 그럼 a와말이통하는사람이위에서보다더많으므로역시금방끝남.. 어떤조직에는 n 명의회원이있고, 세명씩으로이루어진 n + 개의위원회가있다. 어떤두위원회도회원구성이일치하지않는다. 정확히한명의회원을공유하는두위원회가있음을증명하여라. ( 미국 979-5) 3. = x <x <x 3 < 의순서를가지는정수들이모든 n 에대해 x n+ n 의관계식을만족할때, 모든정수 k 는서로다른 i, j 에대해 x j x i 로나타내어질수있음을보여라. ( 오폴 980-)

372 37 조합 증명 ( 이영욱 ) 귀류법. 어떤자연수 k 가임의의 i, j 에대해 x j x i 로나타내질수없다고가정. n =; ;:::;k 이도록잡으면 x 에서 x k+ 까지모두 k 이하임을알수있다. 귀류범의가정에서 (;k+ ); (;k+); (3;k+3);:::;(k; k) 의순서쌍에서어떤순서쌍에서도순서쌍의두수모두수열 x i 에포함될수없다. 즉각순서쌍에서최대 개씩이수열 x i 에포함되고,k 이하의자연수중최대 k 개까지만수열 x i 에포함된다. 이는 x ;::: ;x k+ 이모두 k 이하임에모순. 증명끝. 4. 집합 f; ;:::;00g 을 7 개의부분집합으로분할하였다. 다음을만족하는네수 a, b, c, d 를갖는부분집합이존재함을보여라 : a, b, c, d 중적어도세수가서로다르고, a + b = c + d 를만족한다. ( 유고슬라비아 98 고 -4) 증명비둘기집의원리에의해 5개이상의원소를갖는집합존재. 그집합에서두수를골라뺀차는 5 =05가지이상인데, 두수의차는최소에서최대 99까지만가능하므로중복되는것이있음. 즉, a d = c b 인경우가있음. a = c 나 b = d 일순없으므로여기서 OK 명이참석한파티에서, 네명의그룹을어떻게택해도그중한명이나머지 3 명을모두안다고한다. 이파티에참석한다른모든사람을아는사람은최소몇명인가? ( 미국 98-) 6. 마르티안시에서는지하철을건설할계획을하고있다. 이지하철노선은도중에스스로와교차할수있는하나의폐곡선으로이루어져있다. 단, 각교차점에서는한번만교차해야한다 ( 즉, 같은지점을세번지나서는안된다 ). 지하철노선이교차할때는한터널이다른터널위를지나도록해야하는데, 한지하철이이노선을따라달리면서각교차점을지날때마다위쪽터널, 그다음엔아래쪽터널, 그다음엔위쪽터널, 그다음엔아래쪽터널,... 이렇게위와아래를번갈아지나도록터널을건설할수있음을보여라. (Towns 98 가을 S5) 증명각교차점의차수가짝수이므로이웃한두영역이서로다른색이되도록흑백으로칠할수있다. 좌후방과우전방이흑색영역이되는지점을통과할때는아랫쪽터널, 그반대는윗쪽터널을이용하도록건설하면된다. 7. 한정사각형을 K 개의같은크기의작은정사각형들로분할하였다. 이작은정사각형의중심을모두지나도록꺾은선 ( 여러개의선분이이어져만들어진선 ) 을그렸다. 이꺾은선은스스로와도중에만나거나가로지를수있다. 이꺾은선의꺾인횟수의최소값을구하고그것이최소임을증명하여라. (Towns 98 봄 S) 풀이 K =일때 :0번자명. K =일때 :번. 한선분이최대 개의점을지날수있으므로적어도 개의선분은필요한데, 딱 개의선분이각각두점씩지나는경우두선분이서로연결되는경우가없다는것금방확인됨.3개의선분으로는가능. K 3 일때 : 필요한꺾인횟수의최소값이 K 3 임을보이자. 즉선분이최소 K 개필요함을보이자. 우선 K 개의선분이면충분하다는것은, 먼저 K =3일때 (3; 3)! (0; 0)! (3; 0)! (0; 3)! (0; ) 로움직이면되고, K가커질때마다! (3; )! (3; 3) [K =4완성 ]! ( ; 3)! ( ; ) [K =5완성 ]! 이렇게둘레둘레둘러가면확인된다. 이제항상 K 개이상의선분이필요하다는것만보이면된다. 사용한선분중에수평선이 a개, 수직선이 b개라고하자. a K 이면이수평선을둘씩연결할선분이 K 개이상필요하므로 K +(K ) K 로 ok. a = K 이면이수평선들로덮이지않은행이하나남아있고, 그행의K개의점각각을지나는선분이독립적으로하나씩더필요하므로 (K ) + K K 로역시ok. 즉, a K 일때만보면된다. 마찬가지로 b도 b K 라가정할수있다. 즉, 수평선으로도수직선으로도덮이지않은점들이적어도 (K a) (K b) 영역만큼있다. 이영역의내부는무시하고외곽선 s만을생각하면, 나머지선분각각은 s의점을최대개씩만지난다. s의점은(k a)+(k b) 4개이므로추가로필요한선분은적어도 (K a)+(k b) 개. 즉, a + b +(K a)+(k b) =K 이므로이때도역시ok. } 8. 자연수 n 에대해 C(n) 을 의거듭제곱수의합으로 n 을표현할수있는방법의수라하자. 단, 어떤 의거듭제곱수도많아야세번까지중복하여쓰일수있고, 더하는순서는무시하기로한다. 예 C(8)=5: 8=4+4=4++=4+++=++++ 임의의 n 에대해 C(n) =[P (n)] 인다항식 P (x) 가존재하는가? 단,[t] 는 t 를넘지않는최대의정수를나타낸다. (Putnam 983)

373 4. 조합고급문제 373 풀이 n = a k k + + a +a 0 (0 a i 3) 로표현할때 a 0 = 0; 인것과a 0 = ; 3 인것사이에일대일대응이있다. 즉, C(n +) = C(n) 이다 (). n을이런표현으로나타내어보면 a 0 =0or인데, a 0 =0일때n = a k k + + a 꼴과일대일대응하므로 C(n) 가지, a 0 =일때 n =a k k + + a 꼴과일대일대응하므로 C(n ) 가지이다. 즉, C(n) =C(n)+C(n ) 이다 (). (), () 에의해수학적귀납법으로 C(n) =[ n +] 임을증명할수있다. 답 x + } 별해생성함수로풀자. X C(n)x n =(+x + x + x 3 )( + x + x 4 + x 6 )( + x 4 + x 8 + x )( + x 8 + x 6 + x 4 ) = x4 x 8 x 6 x 3 x x x 4 x 8 = x x =(+x + x + x 3 + )( + x + x 4 + x 6 + ) = X x a+b 즉, a +b = n 인 (a; b) 의개수가x n 의계수C(n) 이다. 이것은 0 b n [ n +]. 인 b의개수이므로 C(n) = } 9. 무한평면에가로세로평행선들을그려정사각형칸들로분할하였다. 두사람이다음과같은게임을한다 : 먼저하는사람이한칸을골라빨갛게칠하고, 다음사람이색칠되지않은한칸을골라파랗게칠한다. 다시먼저했던사람이색칠되지않은한칸을골라빨갛게칠하고, 다음사람이또한칸을골라파랗게칠한다. 이런식으로계속한다. 먼저하는사람의목표는빨간색꼭지네모 ( 어떤직사각형의네코너를이루는같은색의네칸 ) 를만드는것이고, 두번째하는사람의목표는이것이불가능하도록하는것이다. 먼저하는사람이이길수있을까? 또, 두번째사람이한번에두칸을칠할수있는것으로한다면결과는어떻게되겠는가? (Towns 983봄 JA4/SA4) 풀이 ( 김대성 ) 두번째사람이n개의색을칠한다고하자. 그래도항상처음하는사람이이긴다. 이유는 () 처음시작하는사람은한행 A에자신이원하는개수만큼붉은색을칠할수있다 ( 물론연속하지않아도된다 ). n +개의색을한행에붉은색으로칠했다고하자. () 무한개의행중에서 번째사람이 n +개의색을칠한 A행과 번째사람이색칠하지않은한행을제외한한 B행을고른다. 그 B행에서 번째사람이 A행에색칠해둔것과같이열에있는한곳에붉은색을칠한다. 두번째사람은최대 n개의색을칠하므로,b행에서 A행에색칠된것과같은열에있는곳중최대 n곳을칠할수있고그러면,곳이남아서거기에붉은색을칠하면된다. } 풀이 ( 김대성 ) 두번째사람이n개를색칠한다고하자. 첫번째사람을철수, 두번째를영희라고하자. 다음과같은알고리즘을따른다. () 철수가연속한 n +개의행을고른다. 철수는그연속한 n +개의행에서만색칠을하려한다. () 철수가그 n +개의행들중아무곳이나한곳을골라서색칠을한다.A이라하자. 영희는철수가고른n +개의행중A열에나머지 n +칸을모두칠할수는없다. 그러면, 철수는 A열에다른한곳도칠할수있다 ( 즉, 두곳을칠할수있다 ). (3) 영희의다음작업이끝난뒤철수는영희가색칠하지않은열을고른다. 그열을A라고하자.() 와비슷하게철수는두곳을붉은색으로칠할수있다. (4)... 계속반복... 철수가이런작업을 n+ +번반복하면원하는사각형이생긴다. } ³ n 0. 이항계수 (0 r n) 중에서 3으로나눈나머지가 인것의개수를a n 이라하고나머지가 인것의 r 개수를 b n 이라하자. a n >b n 임을보여라. ( 영국 984-)

374 374 조합 증명 0만으로이루어진 mod 3 파스칼삼각형을그려보자.은 3단계후 00이됨. 그동안부등식잘성립. 다시 두개로시작하는데, 그럼 00둘이만나 0000이되기직전까지는앞에서했던것을두군데에서반복할뿐이므로 a n > b n 인상태가되어역시부등식잘성립. 다시두개의 과하나의 로시작하는데,로시작하는것은 mod 3으로 과 를바꿔쓴꼴이라서 로시작된것한쪽과개수가서로상쇄됨 (를곱하면mod 3으로 과 는서로뒤바뀜 ). 그래서 00과00와 00과만나게되고, 그럼 이되어지금까지의것을또두곳에서반복하는것이됨. 비슷한과정의반복. mx. ja ij j 이고모든 j에대해서 a ij =0인실수들의행렬 i= 0 a a n (a ij )=@ A a m a mn 이있다. 이행렬의항들을같은열안에순서를적당히잘바꾸어주면그결과로얻은행렬 (b ij ) 가모든 nx i에대해 b ij < 를만족하도록할수있음을보여라. ( 오폴 984-7) j= nx mx 증명행합 Si := b ij 라하고S = js i j 가최소인경우가바로문제에서원하는경우임을증명 j= i= 하자 ( 열의순서를바꾸는방법의수는유한하므로 S가최소인경우는틀림없이존재한다 ). 이경우에문제의결과를만족하지않는다고하자 ( 귀류법 ). 그럼 js k j 인 k가존재한다. 모든항의부호를바꿔도 P 문제는마찬가지이므로 WLOG S k 일때만보면된다. 열합이 0이라했으므로모든항의합도 0, 즉 Si =0이므로, S l < 0 인 l이존재한다. S k 와 S l 을같은열의두항끼리비교하면 a kj >a lj 인 l이존재한다. 그둘을교환하면S k 는 또는 가감소, S l 은 또는 가증가하므로 jsk 0 j < js kj, jsl 0j js lj 이되어 S 가감소하게된다. 이것은 S 의최소성에모순이므로귀류법성공.. 어떤어려운수학경시대회가 부시험과 부시험으로나뉘어져치러졌는데, 둘을합해모두 8 문제가출제되었다. 각참가자는이중 7 문제씩을풀었다. 각각의두문제의쌍에대해서, 두문제를모두푼참가자는정확히 명씩이었다. 그럼 부시험에서한문제도못풀거나 4 문제이상을푼참가자가있음을증명하여라. ( 미국 984-4) 3. a ;a ;:::;a 00 은 ; ;:::;00 을재배열한수열이다. b = a b = a + a b 3 = a + a + a 3. b 00 = a + a + + a 00 로정의하자. b ;b ;:::;b 00 을 00으로나눈나머지를살피면그중서로다른것 개를찾을수있음을증명하여라. (Towns 984가을 SA) 증명 0개뿐이라면, a i = b i b i 이므로 a i mod 00 들은그 0개의나머지의차들로표현되어야하는데이차들은많아야 0 9=90종류뿐 표의각칸에 0; ;:::;9 의숫자를하나씩써넣는데, 각숫자는 0 번씩사용할수있다. (a) 모든행과모든열이각각 4 종류이하의숫자만갖도록할수있는가? (b) 4 종류이상의숫자를가진행이나열이반드시있음을보여라. (Towns 985 봄 JA5)

375 4. 조합고급문제 375 증명 ( 백진언, 수정됨 ) (a) 할수있다. 예 : (b) x 가나타나는행의개수와열의개수를각각 a, b 라하면 x 는이들행과열의교집합영역 (a b 크기 ) 에만나타날수있으므로 0 ab ( a+b ) (AM-GM). 따라서,(a + b) 40 > 6 6, 즉 a + b 7. 따라서, 한숫자 x 가나타나는줄 ( 행또는열 ) 의개수는항상 7 이상이다. 이제, 숫자 x 가줄 L 에나타나는쌍 (x; L) 의개수 T 를 double counting. x 의입장에서적어도 7 개씩은세어지므로, T 7 0 = 70. 그런데, L 은 0 개가있으므로평균 3.5 개씩에해당하여비둘기집원리로 4 개이상의 x 를갖는 L 이반드시존재한다. 5. 정수 n 의분할 ¼ 란, n 을한개이상의자연수의합으로감소하지않는순서로나열하여나타내는것을말한다.( 예를들어 n =4 라면, 분할 ¼ 는 +++, ++, +3, +, 4 등일수있다.) 임의의분할 ¼ 에대해, ¼ 에나타나는 의개수를 A(¼) 라하고, ¼ 에나타나는서로다른정수의개수를 B(¼) 라정의하자.( 예를들어 n =3 이고 ¼ 가분할 이면, A(¼) = 이고 B(¼) =3 이다.) 임의의고정된 n 에대해, n 의모든분할 ¼ 에대한 A(¼) 의합은 B(¼) 의합과같음을증명하여라. ( 미국 986-5) 증명 ( 부산과학고 학년성충엽 ) 자연수 n의분할의개수를 a(n) 으로, A(¼) 들의합을 g(n) 으로, B(¼) 들의합을 r(n) 으로나타내자. 8n N,g(n) =r(n) 을보이기위해서는다음셋을보이면충분하다. () g() = r() () g(n +)=g(n)+a(n) (n N) (3) r(n +)=r(n)+a(n) (n N) 이제 (), (), (3) 을차례로보이자. () : 의분할은, 한가지인데 g() = r() = 이된다. () : (n +) 분할중 이나타나지않는분할은g(n +) 의계산과는무관하므로제외한다. 또 (n +) 의분할중 이나타나는각분할에서 을하나씩없애면, 꼭 n의분할과같아진다. 그러면이중에서나타나는 의수가g(n) 이고, 맨처음 을하나씩없앤분할의개수는 a(n) 이된다. 따라서 (n+) 의모든분할중각분할에나타나는 의개수의합, 즉 g(n +)=g(n)+a(n) 이된다. (3) : k i (n) 이 n의분할중i가들어있는분할의개수라하자.( 단,5i5n) nx 그러면 r(n) = k i (n) 이된다. i= 그런데 5 i 5 n 에대하여 k i (n) =a(n i) 이된다. (* i가들어있는 n의각분할에서 i를하나씩없애면, 꼭 (n ) 의분할과같아지기때문이다.) 그리고 k n(n) =이므로 n X n X n X r(n) = k i (n)+= a(n i)+= a(i)+ i= i= i= nx n X ) r(n +) r(n) = a(i)+ a(i)+=a(n) i= i= ) r(n +)=r(n)+a(n) (), (), (3) 에의해문제의뜻은증명되었다. 6. 평면위의모든점을각각빨강혹은파랑으로색칠하였다. 세꼭지점이모두같은색인한변의길이가 또는 p 3 인정삼각형이존재함을증명하여라. ( 중국 986-6)

376 376 조합 p 증명먼저평면위에임의의점 A를잡자. 그리고길이가 3인원을그리자. 그원위의점이모두 A와같은색이면성립한다. 만약다른색이존재한다면그점을 B 라하고변의길이가,, p 3 인이등변삼각형 ABC 를그리자. 그러면 C 의색은 A 또는 B 와같다. 일반성을잃지않고 A 와 C 가색이다르다고하자. AC 의중점을 D 라하면마찬가지로 D 도 A 또는 C 와색이같다. A 와같다고하자. 그러면 AD 를한변으로하는정삼각형두개를잡을수있는데그두점을각각 E, F 라하자. 만약 E, F 중하나라도 A 와색이같다면성립. 둘다 A 와색이다르다면 E, F 는같은색깔이고또 C 도 A 와다른그색깔이다. EF = CE = CF = p 3 이므로성립한다. 따라서항상존재한다. 7. A,B,C 세사람이다음과같은게임을한다. 정해진정수 k (k 986) 가있을때집합 f;:::;986g 의 k 개의원소를가진부분집합을뽑는다 ( 각각이뽑힐확률은같다 ). A, B, C 중승리자는이뽑힌집합의원소들의합을 3 으로나누었을때나머지 0,, 에따라결정된다. 어떤값 k 에대하여이게임이공정한가? 단, 게임이공정하다는것은세가지결과의나올확률이같다는것을의미한다. (IMO-LL 986, 헝가리출제 ) 8. 무한평면으로나타내어지는마을이있다. 이마을에는가로세로직선들로길이나있고, 이길들에의해마을이정사각형블럭들로분할되어있다. 어떤하나의길을쭉따라가다보면 00 번째교차로마다경찰관이한명씩서있다. 마을어딘가에강도가한명있는데그위치나속력은모른다. 단, 그는길을따라서만움직인다. 경찰의목표는이강도가어디있는지찾는것이다. 경찰이목표를달성할수있는알고리즘이있는가? 단, 경찰관들의시력은무한히멀리까지볼수있을만큼좋은것으로가정한다. (Towns 987 가을 S7) 풀이있다. 각경찰관은좌표 (00a; 0) 마다있다고하고, 길은가로또는세로의격자선들이라하자. 경찰관들은모두같은속도로움직인다고하자. 우선 a 가짝수인곳에있는경찰관은모두세로선을따라 (00b; 00b) 까지움직인후영원히계속그자리에있도록한다. 그럼이경찰관들의 x- 좌표는불변이므로, 강도는폭이 00 인적당한한세로띠 00b <x<00b +00 안에서벗어나지못하고갇힌다. 나머지경찰관들은우선모두원점을향해움직이다가아직아무도감시하고있지않은원점에서가장가까운세로선 (x = c 꼴의직선 ) 에이르면멈춘다 ( 즉,(00; 0) 에있던경찰은 (; 0) 에서멈추고,(300; 0) 에있던경찰은 (; 0) 에서멈추고,...). 그럼어떤세로선이든유한시간안에감시가시작되고, 특히어떤세로띠든유한시간안에그띠안의모든세로선의감시가시작된다. 그럼강도는그때부터폭이 인한가로선분내에갇히게되므로더이상 y- 좌표가달라지지않는다. 마지막으로, 아까세로선 x = c 에이르러멈췄던경찰관들은각각멈추자마자세로선을따라 (c; c) 의위치로움직인다. 그럼어떤가로선이든유한시간안에감시가시작되고, 따라서강도가어디에있든발견될수밖에없다. } 9. 탁자위에두더미의성냥개비가있고, 각각 00개와 5개의성냥개비가있다. 두사람이다음과같은게임을한다. 각사람은자기차례에, 어느한더미에있는성냥개비개수의약수만큼을반대쪽더미로부터가져간다. 마지막성냥개비를가지는사람이이기는것으로하고두명다최선의경기를한다고할때, 누가이기겠는가? ( 레닌그라드 988-y8-E6) 풀이 n의소인수분해에서의 의지수를 e(n) 으로쓰기로할때, e(a) =e(b) 가되도록만드는것이필승의전략 (a, b는두더미의성냥개수 ). e(a) =e(b) 이면한쪽에서 e 짝수만큼가져가야 e가 ( 반대쪽과같게 ) 유지되는데이것은반대쪽의약수가될수없음 ( 즉, 이꼴은반드시깨짐 ). 한편 e(a) >e(b) 이면 a쪽에서 e(b) 개를가져가면 e(a) =e(b) 가되도록항상만들수있음. }

377 4. 조합고급문제 과 로만된유한수열을 `낱말 ' 이라고하자. 또, 한낱말이연속적으로 3번반복된것을`트리플 ' 이라고하자. 예를들어, 000은 0이 3번반복된트리플이다. 낱말내의어느곳에든 ( 맨앞이나맨뒤에도가능 ) 트리플을끼워넣거나, 낱말내의어느곳에든트리플을지움으로써낱말을변형할수있다. 낱말 0을이런변형을몇번한후에낱말0이되도록할수있는가? ( 레닌그라드 988-y8-E7) 풀이일의자리부터차례로 a, b, c 세가지색으로색칠. 트리플의길이는항상 3의배수이므로삽입이나제거되는녀석이아닌자리들은색깔이그대로유지되고, 삽입이나제거되는트리플은한세트를이루는세개의자릿수마다모두같은색깔이거나모두다른색. 즉, 세가지색의개수가mod 3으로똑같은양만큼늘어나거나똑같은양만큼감소함 ( 불변량 ). 그런데 0과 0은양이다름. } 3. 자연수 n 이주어져있다. k n 인자연수 k 에대해 (i) i <i < <i k n (ii) i s+ i s (mod) (s =; ;:::;k ) 를모두만족하는자연수순서쌍 (i ;i ;:::;i k ) 의개수를 a kn 이라하자. 다음의값을계산하여라. ( 루마니아 TST 988-B3) a(n) = nx a kn k= 풀이 ( 구상준 ) 우선 a n 을다음과같이나눌수있다. a n = b n + c n.( 단, b n 은 a n 중맨앞의수가홀수인순서쌍의개수. c n 는 a n 중맨앞수가짝수인순서쌍의개수 ) 이제다음의사실을알수있다. c n = b n () b n 에해당하는순서쌍에 씩더한다고생각해보자. 예를들어 (,3,4) 는 (,,3) 을바꾼것이다. 한번실제로해보면 b 3 에해당하는순서쌍 (),(3),(,)(,,3) 은각각 (),(4),(,3)(,3,4) 가되고이는 c 4 에해당한다. b n =+c n + b n =+b n + c n = a n + () 이것은 b n 에해당하는순서쌍의원소에 씩더하고, c n 에해당하는순서쌍앞에 을붙인다고생각하며, 이때,() 은위의경우어디에도해당되지않으므로따로세주어야한다. 그러면 a n = b n + c n =+(b n + c n )+c n =+a n + c n =+a n + b n =+a n + a n + =+a n + a n 임을보일수있다. 이때, 이식을다음과같이변형할수있다. (a n +)=(a n +)+(a n +) 임의의 (a i +) = d i 로놓으면 d n = d n +d n 가되어피보나치수열형식으로변형할수있다. 실제로 p + [( 5 ) n+3 ( p 5 ) n+3 ] d n 을구해보면피보나치수열의일부가된다.3,5,8,3,,:::. 따라서 a n = p 5 가된다. } 3. 다음과같은버스노선을가진도시가있다 : (a) 각각의노선마다정확히 개의버스정류소가있다. (b) 임의의두버스정류소사이를노선을갈아타지않고갈수있다. (c) 임의의두버스노선은공통으로지나는정류소가정확히하나있다. 이도시에는몇개의버스노선이있는가? ( 아일랜드 988-B3, 호주 988-4)

378 378 조합 풀이 ( 옥성민 ) 버스정류장의집합을 X라하고, 각노선에해당하는버스정류장의집합을생각하면 X의원소개짜리부분집합들이서로다른두개의집합은하나의공통원소를갖고, 어떤두원소를골라도그두원소를포함하는부분집합을찾을수있는경우를찾으면된다. 버스노선의수를 n이라하자. 우선 n =일때는m =로가능. n > 일때는적어도두개의 X ;X 가있어서 jx j = jx j = ; jx \ X j = 을만족한다. X = fa; b 0 ;b ; ;b 9 g;x = fa; c 0 ;c ; ;c 9 g라하자. b 0 과 c 0 을포함하는부분집합을 X 00 이라하면, b 0 X \ X 00 ;c 0 X \ X 00 이므로 X 00 은이미찾은 개의원소외에 9개의원소를더포함해야한다. 이를 d 0 ;d 0 ; ;d 09 라하자. b 0 과 c 을포함하는부분집합을 X 0 라하면, b 0 X \ X 0 ;c X \ X 0 ;b 0 X 00 \ X 0 이므로역시 X 0 도다른9개의원소 d ;d ; ;d 9 를가져야한다. 같은식으로 X의부분집합 D = fd ij j0 i; j 9;j >0g를찾을수있다. b 와 c j 를포함하는집합X ij 가 X [ X [ D의부분집합임을보이자. i>0인경우만보이면된다. X ij 는 j를제외한모든 0 k 9에대해X 0k 와정확히하나의공통원소를갖고있어야하고, 이는 X 또는 X 의원소가될수없다. 따라서 j를제외한모든 0 k 9에대해d k ;d k ; ;d k9 중하나를갖고, 총 9개의원소가정해진다. b i 와 c j 를포함하면총 개의원소가되고, 이는 X ij 를빠짐없이구성한다. 따라서 X ij ½ X [ X [ D이다. a와 d 0i 를포함하는집합을 T i 라하자. T i 는 X ;X 와 a 외의교점을가질수없으므로, j>0에대해 X 0j 와가져야하는공통원소는 D의원소이다. X 0j 들은 D에서공통원소를갖지않고, j =; ; ; 9에대해 T i 와 X 0j 는각각하나의공통원소를가져야하므로 T i 의 개원소가 X [ X [ D 내부에서결정된다. T i 들은서로 a만을공통원소로가지므로 d 9 i= T i의원소는총 9개이고, 이는 D의각원소가정확히하나의 T i 에속함을나타낸다. X [ X [ D에서임의로두점을잡아그두점을포함하며문제의조건을만족하는집합은위의 개집합중하나임을보이자. 주어진집합이 a를포함하는경우는당연하다. a를포함하지않는다고하자. X 과의공통원소를 b i ;X 와의공통원소를 c j 라하면이집합은X ij 이다. 따라서 X [ X [ D와그부분집합들 X ;X ;T ;T ; ;T 9 ;X ij ; 0 i; j 9은 m = n =로문제의조건을만족한다. 만약 m>에서문제의조건을만족하는경우가생긴다면임의의두집합에서시작하여위와같은결과를얻어낼수있고, 이 개를제외한다른점을포함하는집합은 X 과교점을가져야한다. 이점이a가아닐경우X 와도교점을가져야하고, X ij 중의하나를포함해야하므로모순. a일경우 X 0j ; 0 j 9에대해각각하나의교점을가져야하므로원소의개수가 개를넘어모순. 즉 m =이다. m =에대해서는항상위와같이 개의집합을찾아낼수있고, 임의의두점을포함하는집합은모두위의 개중의하나이다. 답, } 33. 정팔각형의각변을파랑또는노랑으로색칠하였다. 매단계마다각변에대해, 그변에이웃하는양옆변의색이서로다를경우에는그변을파랑으로, 서로같을경우에는노랑으로, 이렇게모든변의색을동시에다시칠한다. 언젠가유한번의단계가지난후에모든변이노랗게됨을보여라. 그리고, 초기상태에관계없이모든변이노랗게됨을확신할수있으려면최소몇단계를거치면되는가? ( 오폴 988-7) 풀이노랑을 로, 파랑을 로보면문제의과정은양옆변의수의곱으로대체하는것. 즉, 맨처음수의배치를 a ;a ;a 3 ;a 4 ;a 5 ;a 6 ;a 7 ;a 8 이라하면, 한단계가지나면 a 8 a ;a a 3 ;a a 4 ;a 3 a 5 ;a 4 a 6 ;a 5 a 7 ;a 6 a 8 ;a 7 a 이되고, 다시한단계가더지나면 (a i =이므로 ) a 7a 3 ;a 8 a 4 ;a a 5 ;a a 6 ;a 3 a 7 ;a 4 a 8 ;a 5 a ;a 6 a 가되며, 다시한단계가더지나면 a a 4 a 6 a 8 ;a a 3 a 5 a 7 이번갈아반복되는모양이되고, 그럼한단계만더지나면모두이됨. } 이하의자연수가 n 3 개있다. 이자연수들의최대공약수를 d 라하자. 이 n 개의자연수중최대공약수가 d 인세자연수가존재함을보여라. ( 이탈리아 988-7) 증명모두 d 로나눠서생각하면 d = 로봐도무방함. 그럼홀수 a 가적어도하나존재하고,3 5 7 > 00 이므로 a 의소인수는 종류이하. a 의소인수를 p, q 라할때, p 의배수가아닌수, q 의배수가아닌수를택하면됨.( 귀류법으로논리를쓰면, 그런세자연수가없다면 p 의배수가아닌수는 q 의배수, q 의배수가아닌수는 p 의배수라서이세수는서로다른수임이분명함 ) 35. S = fa ;a ;:::;a rg 은정수들의수열이다. S 의비어있지않은임의의부분수열 A 에대해서, A 의모든정수들의곱을 p(a) 로정의하자. 또 m(s) 는 S 의모든비어있지않은부분수열 A 들에대한 p(a) 값들의

379 4. 조합고급문제 379 산술평균이라하자. 만일 m(s) =3 이고어떤자연수 a r+ 에대해서 m(s [fa r+ g)=49 라고할때, a ;a ;:::;a r 과 a r+ 의값을구하여라. ( 캐나다 988-5) 풀이 S의임의의부분집합 A들에대한 p(a) 값들의합을 T (S) 라하고, A가공집합일때의 p(a) 는 0으로약속하자. 그럼 m(s) = T (S) r 이고, 즉 T (S) =(+a )( + a ) ( + a r )=+m(s)( r ) 이다. S 0 = S [fa r+ g 이라하면, 마찬가지로 T (S 0 )=(+a ) ( + a r )( + a r+ )=T (S)( + a r+ )=+m(s 0 )( r+ ) 이된다. a r+ = A, r = B 라두자. m(s) =3,m(S 0 )=49임을이용해위의식을정리하면 ( + 3(B ))( + A) = + 49(B ) 3AB A 85B = 36 (3A 85)(3B ) = 55 = 이된다. 이부정방정식은 A, B 는모두정수이고 B 이므로 을두양수의곱으로분해해야한다. 특히 mod 3 으로 6 꼴로분해해야한다. 두인수중어느하나는소인수 3 을갖지않으므로, 3 3 의약수,, 4, 8, 3, 6,, 4 중에서 mod 3 으로 이나 6 인것을찾으면 과 6 뿐이다. 그런데, 3B > 이므로 은부적절하고, 결국유일한가능성은 (3A 85; 3B ) = (6; 9) 가된다. 그럼 (A; B) =(7; 8), 즉 (a r+ ;r)=(7; 3) 이다. 이제 T (S) =(+a )( + a )( + a 3 )=+3 7=9= 3 이고, 이것의정수해 fa ;a ;a 3 g을모두찾으면다음과같다 : (i) 꼴일때 ; f; ; g, f0; 0; 9g, f0; ; 45g, f0; 3; g. (ii) + 꼴일때 ;+ 인약수가,, 4, 3, 46, 9일때로경우를나눠보면 f0; ; 93g, f0; 3; 47g, f0; 5; 4g, f; ; 47g, f; 3; 4g, f3; ; 4g, f; ; 5g, f; 3; 3g, f45; ; 3g, f9; ; g. 등이가능하다. } 36. 무한체스판에서가로와세로의좌표가모두 4의배수인칸에졸을하나씩배열하였다. 기사가졸이없는칸들만을꼭한번씩지나도록하는무한경로는만들수없음을보여라. (Towns 988봄 SO4) 증명 4n 4n 크기의영역을생각하면그안에는 n 개의졸이배열되어있고, 체스판처럼색칠했을때졸들은모두같은색칸에있음. 그럼이영역안의흑칸과백칸의개수차이가 n 개이므로, 전체경로중에서이영역을거치는최초의순간부터최후의순간까지를생각했을때, 그구간경로에서이영역에서밖으로빠져나갔다가돌아오는걸최소 n 번은반복해야함. 그런데그렇게나가거나돌아오는관문의역할을할수있는이영역안의칸은외곽으로부터두께 만큼안쪽에있는띠에속하는칸들뿐으로, 모두기껏해야 3n개미만임. n을충분히크게잡으면 (n ) > 3n 이므로불가능하게됨. 37. 볼록 n 각형을서로교차하지않는대각선들을그려삼각형들로분할하였다. 공통변을갖는두삼각형 ABD 와 BCD 가있을때이것을 ABC 와 ACD 로대체하는시행을 ` 재구성 ' 이라고말한다. 임의의분할을또다른임의의분할로재구성할때필요한최소시행횟수의최대값을 P (n) 이라하자. 다음을증명하여라. (a) P (n) n 3 (b) P (n) n 7 (c) P (n) n 0 (n 3 일때 ) (Towns 988 가을 JA5)

380 380 조합 보조정리 A에연결된대각선이하나도없을때모든대각선이 A에연결되도록재구성하려면 n 3번이면된다. 증명은귀납법으로하면됨. 우선 n =3일때는자명하고, n 4 일때A를포함하는삼각형 ABC를잡으면 BC는대각선. 그럼 BC를변으로갖는삼각형 BCD가또있고, ABC와 BCD를 ABD와 ACD로재구성하면 n각형이대각선 AD를기준으로둘로분할된꼴로생각할수있음. B쪽과 C쪽이각각 b각형, c각형이라하면 n = b + c 이고, 귀납법의가정에의해 (b 3) + (c 3) = n 4번더진행하면양쪽모두해결됨.[ 따름정리 ] A에연결되지않은대각선이 m개있을때모든대각선이 A에연결되도록재구성하려면 m번이면된다. A에연결된대각선들에대해 r개의도형으로분할된꼴로보고위의보조정리를각각의분할영역에대해적용하면됨. 즉,(b 3)+ +(b r 3) = n+(r ) 3r = (n 3) (r ) = m 의식으로확인됨.(a) 한번의재구성으로새로운대각선은딱하나씩만생기므로, 모든대각선이 A에연결된경우를모든대각선이 B에연결된경우로재구성하려면새로 n 3개의대각선을얻어야해서최소 n 3번은기본으로필요함.(b) 분할 alpha에서분할 로만든다고하자. 우선 의한대각선의한끝점 A를잡아모든대각선이A에연결된분할» 로재구성하면 n 4번이면됨. 를» 로재구성하는것도 n 3번이면되는데이과정을역으로하여»! 로재구성할수있음. 즉,!»! 는 (n 4) + (n 3) 번이면충분함.(c) 역시분할 alpha에서분할 로만든다고하자. 와 를통틀어서,4개이상의대각선이연결된꼭지점이있다는것만보이면충분하다. 대각선은둘을통틀어서 (n 3) 개이므로, 대각선에연결된점의수 ( 각점의대각선차수의합 ) 는 4(n 3) 이고, n 3 일때는 4(n 3) 3n +이라서, 비둘기집의원리에의해대각선차수가 4 이상인꼭지점이반드시존재. 증명끝. 38. 어느학생이하루용돈으로,000 원또는,000 원을쓰는데 0 일동안쓴총금액이 30,000 원을넘지않았다. 그렇다면연속한며칠동안에쓴용돈의합이꼭 7,000 원이되는기간이적어도세번존재함을보여라. ( 한국 995-, 전국수학과학경시 989 차변형 ) 증명 ( 과기원수학과 94학번엄상일 ) i번째날까지쓴돈이 a i,000원이라고하자. a ;a ;a 3 ; ;a n 은단조증가수열이고, a 0 30 이다. a ;a ; ;a 0 ;a +7;a +7; ;a 0 +7의총40개수를생각해보자. a 이고, fa n g은단조증가수열이므로위수들의각항은 이상 37이하의자연수만가능하다. 한편 fa n g은단조증가수열이므로위수들중에어떤두개가같으려면 a i 꼴과 a j +7꼴이같은경우밖에없다. 그러므로 3개의항이같은값이될수는없다. 한편 a i = a j +7인경우, j번째날부터 i 번째날까지쓴용돈의합이 7,000원이된다.이상 37까지자연수중 40개를택하였으므로, 적어도 3개는값이같아야하므로연속한며칠동안에쓴용돈의합이꼭 7,000원이되는기간이적어도 3번존재함을알수있다. 39. 점 P 를평면위의한점이라고한다. 평면위에유한개의선분이아무렇게나그려져있으며, 그선분들의길이의합이 989 이다. 이때, 어떠한선분과도만나지않으며, 점 P 로부터의거리가 704 보다작은직선이존재함을증명하여라. ( 한국 989-7) 증명 (i) 평면위의길이 a인선분의 x축및 y축위에의정사영의길이를각각 a ;a 라하면 a + a = a(cos µ +sinµ) = p ³ a sin µ + ¼ 4 ) a + a 5 p a (ii) P 를원점으로잡고 x축, y축을적당히정한다. 모든선분의두좌표축위에의정사영의길이의합은 (i) 에의하여 989 p 보다작다. 따라서한좌표축위에의정사영의길이의합이 989p 이내인좌표축이있다. 이좌표축에서양의부분과음의부분으로나누어그중한부분의정사영의길이의합은 989 p 이내이다. 989 p =703: < 이므로원점 P 에서이좌표축위에거리 704인점 Q를잡으면선분 PQ위에는주어진선분들의정사영의점이아닌것이존재한다. 이점을 R이라하고, R을지나이좌표축에수선을그으면이직선은모든주어진선분과만나지않는다.

381 4. 조합고급문제 n 개의수 ; ; ;:::; n 이주어져있다. 이수들의순서를바꾼어떤순열 ¾ = X ;X ;:::;X n 에대해, S (¾) =X, S (¾) =X + X, S 3 (¾) =X + X + X 3, :::, 그리고 Q(¾) =S (¾)S (¾) S n(¾) 로정의한다. 모든가능한순열 ¾ 에대한합 P =Q(¾) 를구하여라. ( 캐나다 989-5) 풀이 A를임의의 k개의양수들의집합이라하자. 그리고, A의원소들의순서를임의로정한모든가능한순열들의집합을 P (A) 라하자. 다음식을증명하기로하자. X Q(¾) = Y a aa ¾P (A) k =일때는순열이¾ = a 으로유일하므로 Q(¾) = a 이되어자명하다. k = n 일때성립한다고가정하자. 그럼 A = fa ;:::;a n g 를임의의 n개의양수들의집합이라하면, 위의식이임의의 A fa i g 에대해성립하므로, X ¾P (A) Q(¾) = n X X Q(¾) i= ¾(n)=a i nx X = a i= + + a n ¾P (A fa i g) nx Y = a i= + + a n a = aa fa i g Q(¾) a + + a n = = Y (a + + a n )a a n a aa n X i= a + + a n a i a a n 로 A 에대해서도, 즉 k = n 에대해서도성립한다. 따라서, 수학적귀납법에의해위의식이임의의양의정수 k 에대해항상성립함이확인되었다. 원래의문제는 A = f; ;:::; n g 일때이므로, X ¾P (A) Q(¾) = n = n(n )= 답 이된다. } 4. 어느지역테니스클럽의 0 명의회원들이단식전을갖기로했다. 경기는모두 4 번이고, 각회원은최소한한경기에는참가한다. 그럼이경기들중에서로다른 명이참가하는 6 경기가있음을증명하여라. ( 미국 989-) 4. 체스판의좌하구석의칸 A 에룩이있다. 룩은수평또는수직으로이웃한칸으로움직일수있다. 룩이체스판을여행하는데, 어떤한칸은한번들르고, 또다른한칸은두번들르고,..., 한칸은 64 번들르려고한다.( 처음에시작하는칸인 A 도한번들른것으로센다.) () 이여행이 A로돌아오는것으로끝날수있는가? () 이여행이 A에서끝날수있는가? ( 러시아 989 4차-y0-6) 풀이 () 불가능하다. 처음에시작해서돌아올때까지총들른횟수는상하이동과좌우이동이모두짝수번씩이어야해서처음에시작하는칸을포함하므로홀수가되어야하는데, 각칸에적힌정보로는총들르는횟수가 = 080 으로짝수가되어모순. () 가능하다. 다음과같이하면된다 순서대로!! 4, 3! 4! 6, 5! 6!!6, 6! 6! 63, 64!! 64 로하면된다.`,' 는그사이를다소모될때까지왕복한다는뜻이다. }

382 38 조합 43. S 는 a<b n 을만족하는 m 개의양의정수쌍 (a; b) 로구성된집합이다.(a; b), (a; c), (b; c) 가모두 S 에포함되는삼원쌍 (a; b; c) 가적어도 4m (m n 4 ) 3n 개있음을보여라. (APMO 989-4) 증명 n개의꼭지점과 m개의변을갖는그래프가 m(4m n ) 3n 개이상의삼각형을가짐을보이면된다. 점 i의차수를d i 라하자. 변 (i; j) 가있다면, 두점i와j에서다른 n 개의점에연결되는변은 d i + d j 개이고, 그럼변 (i; j) 를포함하는삼각형이최소d i + d j n개있다. 각삼각형은 3개의변으로부터세어지므로, 3T X 의순서로원하는부등식이확인된다.() 은 d i 를더하고있으므로 (d i + d j n) = X (d i + d j ) mn (i;j)e (i;j)e =(d + + d n) mn () n (d + + d n ) mn () = n 4m mn (3) X X (i;j)e (i;j)e(d i + d j )=X i (d i + d j ) 에는점 i 에연결된 d i 개의변각각에대해 이됨으로부터얻은것이고,() 는코시부등식,(3) 은각변이두점으로부터세어지므로 d + + d n = m 임을이용한것이다. d i 44. 명의클럽에위원회가하나있다. 위원회회의가열리면그때마다다음번위원회를지금의위원회에서한명추가하거나혹은한명을제외시켜조직한다. 위원회는언제나 3명이상이어야하고, 클럽의규칙에의하면같은멤버로된위원회가두번만들어질수없다. 모든가능한구성의위원회가모두한번씩만들어졌을때가언젠가올수있겠는가? (Towns 989봄 SA4) 45. m 행 n 열의직사각배열이있다 (m <n). 어떤칸에는별이있는데, 각열마다적어도하나씩의별이있다. 별 A 를포함하는행이 A 를포함하는열보다별을더많이갖는그런별 A 가존재함을증명하여라. (Towns 989 가을 SA6) 명의소년이있다. 각소년은한아이스크림가게에세번씩들렀다. 또한어떤두소년도이가게에서한번이상만났다고한다. 어떤세명의소년이동시에이가게에있었던순간이있었음을보여라. ( 소련 989-y8-) 증명귀류법으로그런순간이없었다고하자. 누군가새로운소년이가게를방문할때가게에는기껏해야한명의소년이있었다. 또한가장처음가게에간소년은방문순간누구와도만나지않았으므로, 두소년이만나는횟수는기껏해야 3 7 =0번이다. 그런데 7 =로소년의쌍은더많으므로어떤두소년은서로가게에서만난적이없다. 이것은모순 개의구슬로된목걸이 A와 9개의구슬로된목걸이 B가있다. n은임의의홀수자연수이다. 이 33개의구슬에다음의집합 fn; n +;n+;:::;n+3g 에서하나씩대응시켜번호를붙이는데, 이웃한두구슬의번호는항상서로소가되도록할수있음을보여라. ( 미국 990-3)

383 4. 조합고급문제 383 증명 n +3와 n을이웃하게연결시켜서커다란하나의고리로생각했을때, 이웃한두수끼리는항상서로소임에먼저주목해두자. 이고리에서 n + a +부터 n + a +4까지의 4개의구슬을떼어낸후양끝을이어목걸이 A를만들고, 남은것들의양끝 (n + a와 n + a +5) 를역시이어목걸이 B를만드는것을생각하자. 그럼 gcd(n + a +;n+ a +4) = gcd(n + a +; 3) = or 3 이고, gcd(n + a; n + a +5)=gcd(n + a; 5) = or 3이나 5의배수이다. 만일 ( 귀류법 ) 어떤 a를택해도이런두목걸이를만들수없다고한다면, 어떤 a에대해서도 n + a +이 3의배수이거나 n + a 가 3 또는 5의배수라야한다. 즉, a =; ; 3; 4; 5 를택한다섯가지경우를살피면그중에3 j n + a + 인경우는많아야한번이고,3jn + a 인경우는많아야두번이며,5jn + a 인경우는딱한번이다. 따라서, 이런 a 중에어느하나는셋모두성립하지않아서좋은목걸이가된다 명의사람들로이루어진집합을다음과같은방법으로서로소인몇개의부분집합으로나누었다. (a) 어떤사람도자신이속한부분집합의사람들을모두다알지는못한다. (b) 같은부분집합에속하는세사람을임의로뽑아도그들중적어도둘은서로알지못한다. (c) 같은부분집합에속하는서로모르는두사람을임의로뽑아도그둘모두가아는사람이그부분집합에꼭한명있다. (i) 각부분집합내에서, 각자가아는사람의수는모두같음을증명하여라. (ii) 부분집합의수가몇개까지가능한지그최대의수를구하여라. [ 주 ] A 가 B 를안다면 B 도 A 를아는것으로한다. 또한각자는자기자신을아는것으로한다. (APMO 990-4) 관찰그래프로생각했을때 (b) 는삼각형이없음을의미하고,(c) 에서사각형이없다는것도알수있다. 즉, 다각형구조로는오각형이최소이다. (i) 의증명 (Modi ed O±cial) 임의의한부분집합을 S 라하자. x S 를 S 에속하는사람을가장많이아는사람이라고하고, x ;x ;:::;x n S 를 x 가아는 (x 이외의 ) 사람들이라고하자. 먼저 (b) 에의하여 x i 들끼리는서로모르는사이이다. 각각의 i 에대하여 N i 를 S 에속하는사람들중 x i 는알고 x 를모르는모든사람들의집합이라하자.(c) 에의해 x i, x j 를동시에아는사람은 x 한명뿐이므로 N i \ N j, 즉 N i 들은모두서로소이다.(a) 와 (c) 에의해 N 6=? 임을가정할수있다. y N 이라고하자.(c) 에의하면, 각 i> 에대하여 N i 에 y 을아는사람이꼭한사람씩있으며그들을 y i 라하자 (i =;:::;n). 따라서, y 은 x ;y ;:::;y n 을안다. y 이아는사람의수가 n 보다클수없으므로 y 이아는사람의수는정확히 n 이다. 같은방법으로 N i 에속하는각자가아는사람의수는모두 n 이다. y ;y ;::: 등을 x 와역할을바꿈으로써 x i 들도꼭 n 명의사람을안다.(c) 에의하여 x; x ;:::;x n 이외의사람은각각어떤 N j 에속해야하므로 S 에있는모든사람이 S 안에서 ( 자신을제외하고 ) 꼭 n 명을안다. (ii) 의풀이부분집합의수가가장많으려면각부분집합이최소한의인원으로구성되면된다. 그러기위해서는각자가두사람씩만알아야하므로 5명이서로아는사람과만이웃하도록둥글게배열하는경우가최소이다. 따라서, 가능한부분집합의개수는최대 990=5 =398개이다. } 궁금증모든부분집합의크기는항상 5 의배수일까? 49. 단순그래프 G가 n개의정점과 m개의변을가지고 m (n )(n ) + 이면 G는 Hamilton 회로 를가짐을증명하여라. 또한 n개의정점과 (n )(n ) + 개의변을가지는그래프로서 Hamilton 회로가존재하지않는예를찾아라. ( 통신강좌 99--0) 풀이 ) n =3일때, m (3 )(3 ) + = 3이므로당연히 Hamilton회로를갖는다. ) n일때성립함을가정하고 n +개의정점을생각해보자. n개의정점과 m (n )(n ) + 개

384 384 조합 의변은 Hamilton 회로를가지며, n + 번째점에서한점을제외한나머지 n 개의점에각각연결하면 n + 개의점은 Hamilton 회로를갖는다. 이때 m (n )(n ) + + (n ) = (n )n + 이므로 n개의정점과 m (n )(n ) + 개의변을갖는단순그래프 G는항상Hamilton회로를 갖는다. } 서양장기판을 8 개의 타일로겹치지않게덮었을때, 타일들사이의금으로이루어진, 장기판의양끝변을잇는직선이있음을보여라. ( 통신강좌 ) 풀이장기판의금을따라관통하는임의의한직선 s를생각하자. 이때, s위를횡단하여덮는타일은짝수개있어야한다. 만일홀수개있다면, s의한쪽에이미홀수개의칸이덮여있으므로나머지홀수개의칸을 타일로가득덮을수없어모순이다. s s 이제 타일들사이의금을따라관통하는직선이없으려면, 임의의 s에대해적어도두개의 s위를횡단하여덮는타일이있고, s는가로, 세로모두 0개이므로적어도 0개의타일이필요하다. 그러나, 이장기판에는 8개의타일이면가득차므로 pigeon-hole principle에의해, 타일로덮이지않는 s가존재한다. } 5. n 명의소년과 n 명의소녀가참석한모임에서, 어떤자연수 p ( p n) 에대하여각소년들은 p 명의소녀들을알고, 또각소녀들은 p 명의소년들을안다고한다. 이때서로아는소년소녀끼리모두짝을지어줄수있음을보여라.( 단,A 가 B 를알면 B 도 A 를안다고한다.) ( 통신강좌 ) 풀이소년들의집합 X, 소녀들의집합 Y, 그리고서로아는소년과소녀 x, y에대해fx; yg E인집합 E를생각하자. 이때그래프G =(X [ Y;E) 는이분그래프이다. 이그래프가 Hall의조건 : 8A ½ X; jj(a)j jaj 을만족함을보이자. 단, J(A) ½ Y 는 A 의치역, 즉 a A 에대해 fa; yg E 인 y 들을모두모아놓은집합이다. 임의의 A ½ X 에대해 A 에서 J(A) 로연결된모든변 fx; yg 의수 a 는정확히 pjaj 이다. 그런데 J(A) 에서 A 로연결된모든변의수 b 는많아야 pjj(a)j 이고, a 와 b 는동일한변들을센것이므로 a = b, 따라서 pjaj pjj(a)j 이다. 따라서 G 는 Hall 의조건 jj(a)j jaj 을만족시킨다. 아래의보조정리에의해 Hall 의조건을만족시키는이분그래프 G 는완전짝짓기가가능하다. } [ 보조정리 ] 이분그래프 G =(X [ Y;E) 에서 Hall 의조건은완전짝짓기가가능할필요충분조건이다. 단완전짝짓기란, E 의부분집합이면서 X [ Y 의분할이기도한집합을만드는것을말한다. 보조정리의증명 ((): 완전짝짓기가가능하다고하자. 임의의 A ½ X에대해그짝짓기에의해 A의원소들과짝이된 jaj개의 Y 의원소들은 J(A) 의부분집합을이룬다. 따라서 jj(a)j jaj이다. ()): 거꾸로 Hall 의조건이만족된다고하자. 완전하지않은임의의짝짓기 ( 짝짓지못한소년과소녀들이남아있는경우 ) M ½ E 에대해더좋은짝짓기 M 0, 즉 jm 0 j = jmj + 인짝짓기를만들어낼수있음을보이려고한다. x 0 X 를 M 에의해짝짓지못한원소라하자. jj(fx 0 g)j jfx 0 gj = 이므로 x 0 과알고있는 y 이최소한하나있다. 만일 y 이 M 에의해짝지어져있지않다면 M 에변 x 0 y 을추가하여 M 0 을얻을수있다. 만일 y 의짝이있고, 그를 x 이라하면, jj(fx 0 ;x g)j jfx 0 ;x gj =

385 4. 조합고급문제 385 이므로 y 과다른, x 0 이나 x 과알고있는 y 가있다. y 의짝이없다면여기서멈추고, M 에의한짝 x 가있다면다시같은방식으로 x 0, x, x 중어느하나와알고있는 y 3 를찾을수있다. 이런방식으로계속하다보면 G 가유한하므로언젠가는짝이없는 y r 을발견하게되고, 여기서과정을멈춘다. 각각의 y i ( y r) 들은 x 0, x,, x i 중어느하나와알고있다. 앞의과정을거꾸로되새겨보면다음과같은통로 y r ;x s ;y s ;x t ;y t ; ;y w ;x 0 가존재함을알수있다. 여기서 x i y i 꼴의변들은 M 에있고, 이들과엇갈려있는 y i x j 꼴의변들은 M 에없다. M 에서이통로에있는 x i y i 꼴의변들을빼고, y i x j 꼴의변들을대신넣으면, 새로운짝짓기 M 0 이이루어진다. 위통로의양끝변이 M 0 에포함되므로 jm 0 j = jmj + 이된다. 따라서두조건은필요충분이다. } 5. 임의로주어진자연수 k 에대하여집합 N k = f (x ;x ;:::;x k ) j x ;x ;:::;x k N g 을생각하자.( 단, N 은자연수전체의집합이다.) N k 의원소 x =(x ;x ;:::;x k ), y =(y ;y ;:::;y k ) 에대하여 x i y i (i = ; ;:::;k) 일때 x y 라고정의한다. X 를 N k 의임의의부분집합이라고하고, x X 에대하여 m(x) 를 X 의원소로서 x 보다작거나같은원소의개수라고할때집합 M(X) =f x X j m(x) =g 이유한집합임을보여라. ( 한국 99-4) 증명 M(x) =fx X j m(x) =g을 X의극소부분집합 (minimal subset) 라한다. 즉 X의임의의원소 x에대해서m(x) 의원소a가존재하여 x = a이고 M(x) 의원소들사이에는대소관계가존재하지않는다. N k 에서 k에관한귀납법으로증명한다. (i) k =일때 X = fx j x Ng ½ N 라하고minfxg = a 라하면X의임의의원소 x는 x = a이다. 즉 m(a) =이고 M(X) =fag이므로 M(X) 는원소개인유한집합이다. (ii) k =일때 X = f(x ;x ) j x ;x Ng ½N 라하고minfx i g = a () 라하자. X의원소중에서첫째좌표가 a () 인원소전체의집합을 A = f(a () ;x ) X j x Ng ½X라하자. 여기서 minfx g = a () 라하고 a () =(a () ;a() ) 라하자. X = f(x ;x ) X j x i a () ;x a () g = f(x ;x ) X j (x ;x ) a () g 이라하면 X X 의원소 (x ;x ) 는 x >a () ; x <a () 이므로 a() 과대소비교불가능하다. 즉, a () 는하나의극소원소이다. X X = f(x ;x ) j x >a () ;x <a () 의원소들중에서 minfx g = a () 이라하고 A = f(a () ;x ) X X j x Ng라하고A 의원소들중에서 minfx g = a () ; a() =(a () ;a() ) 라하면 X X [ X 의원소 (x ;x ) 는 x >a () ; x <a () (<a() ) 이므로 m(a() )=이다. 즉 a () 도하나의극소원소이다. 다음에는 X X [ X 에서이와같은방법으로원소 a (3) =(a (3) ;a(3) ); a(4) =(a (4) ;a(4) ); 와집합 X 3 ;X 4 ; 를만들어나가면원소 a (i) 의둘째좌표 a (i) 는순감소하고유한번 ( 이것을 n 번이라한다.) 계속하면 이된다. 즉 a (n) = 이된다. 여기서 a () ;a () ; ;a (n) 는극소원소이고, 극소원소는이것뿐이다. 즉 M(X) =fa () ;a () ; ;a (n) g 는 n 개의원소를갖는유한집합이다. (iii) N k 즉 k 차원에대해서 M(X) 가유한집합이라가정하고, N k+ 에대해서 M(X) 가유한집합임을증명한다. X = f(x ;x ; ;x k ;x k+ ) j x ;x ; ;x k+ Ng ½N k+ 이라하자. X 의원소의 N k 에의정사영 (x ;x ; ;x k ; 0) 을 (x ;x,, x k ) N k 로나타내고이들전체를 X ½ N k 라하자. 가정에의하여 M(X) 는유한집합이므로 M(X) =fa ;a ; ;a n g; a i = fa i ;a i ; ;a i kg (i =; ; ;n)

386 386 조합 라하자. 앞으로 (a i ;ai ; ;ai k ;x k+) 을 (a i ;x k+ ) 로나타내기로한다. A i = f(a i ;x k+ ) X j x k+ Ng; (i =; ; ;n) 라하고A i 에서 minfx k+ g = a i k+ 이라하자. 여기서 i =; ; ;n을고정시켜놓고이론을전개한다. n[ X i = fx X j x a i g 라하면 X i = X이다. i= X i = f(x; x k+ ) X j x X i ;x k+ Ng 라하면X i 는 X i 의 N k 에의정사영이라생각할수있 n[ 고, i 6= j이면원소 (a i ;x k+ ) 과 (a i ;y k+ ) 은서로비교불가능하며, X i = X이다. X i = f(x i ;x k+ ) X i j x i a i ; x k+ a i k+ g h i 이라하면 X i X i 의원소 (x i ;x k+ ) 은 x i >a i 이고 x k+ <a i k+ 이므로 m (a i ;a i k+ ) =이다. 따라서 (a i ;a i k+ ) 은극소원소이다. 집합 X i X i ½ Nk+ 에대하여위의방법을반복하면극소원소의 k + 번째좌표는순감소하여유한번의조작으로 이된다. i =; ; ;n 개의유한개의부분집합 X i 에대하여얻는모든극소원소는유한개이므로 M(X) 는유한집합이다. ( 여기서 i 6= j 이경우에 X i 에서얻은극소원소와 X j 에서얻은극소원소가서로비교가능하는경우가생길수도있으나이들까지합한것이유한개이므로비교가능한것이있을때큰것을제외하여도유한개라는사실에는변동이없다.) 53. 정육면체의각꼭지점에파리가한마리씩앉아있다. 이파리들이동시에다른꼭지점으로이동했는데, 다시한꼭지점에한마리씩앉았다. 처음에앉았던꼭지점들이이루는삼각형과나중에앉은꼭지점들이이룬삼각형이합동이되는세마리의파리가있음을보여라. 단, 세마리가앉아있는순서는무시하는것으로한다. ( 러시아 99 4차-y0-5) 증명이웃한점이서로다른색이되도록흑백으로번갈아칠하자. 같은색네점은정사면체를이룬다. 그럼, 같은색네점중에어떤세점이옮겨간이후에도같은색이되는경우가있다면그세점이합동인정삼각형임이유지되므로 ok. 따라서, 검은네점도옮겨간후에는같은색끼리 :로, 흰네점도옮겨간후에는 :로나뉘어진경우만생각하면된다.P색이었다가 Q색으로옮겨간파리를 PQ로나타내기로하자. 즉, BB, BW, WB, WW가모두마리씩이다. 육면체를적당히돌려이동전과이동후의 BB-BB의위치가똑같도록대응시키면, 이동전의 BW-BW의위치는결정되어있고, 보존삼각형이생기지않으려면 () 이동후 BW들이모두윗면 (BB-BB와같은면) 에있다면 WW는둘다밑면에가게되고그럼 BB-BB-WW 삼각형이없으려면이동전 WW는둘다윗면에있었고등등... 하면 BB-WW-WB 삼각형이생김. 나머지두경우도좀따져보면다그럴듯 ;;; 아너무막노동아닌가이거. 드리블을좀더잘해야할듯.[ 주의!] 세마리가앉아있는순서를무시하지않으면다음과같은반례가있다 : ACac-DBdb에서 ABab-dcDC로이동 ( 면대각선일부만보존되고정삼각형도없음 ). 54. 각각의꼭지점마다 3 개씩의모서리가연결된어떤다면체가있다. 각모서리들을 3 가지색중에서골라색칠하는데, 각꼭지점마다세가지색의모서리가모두만나도록할수있었다고한다. 각각의꼭지점마다 이아닌복소수를하나씩부여하여, 모든면마다그면의꼭지점의값들을모두곱한값이동시에 이되도록할수있음을증명하여라. ( 중국 99-6) 관찰한가지색에만주목하면, 모든점을둘씩짝짓고있음. 즉, 점은짝수개이고, 각색은같은횟수 ( 점의개수의 ) 로사용됨. 각각의꼭지점은 3개씩의면에속하므로 =( 모든면의수의곱 )=( 모든꼭지점의수의곱 ) 3. 즉, 모든꼭지점의수의곱은 ;w;w 만가능. 그러니까, 꼭지점의배정하는수로는 w와 w = w 만잘사용하여할수있다면행복하지않을까하는생각을해볼수있음. 증명세가지색에,, 3의이름을붙이자. 꼭지점은다면체의바깥쪽에서보았을때세가지색의모서리가시계반대방향으로,, 3의순서로된것과그반대의순서로된것두가지가있음.,,3의순서로된점에는 w를, 그반대의순서로된점에는 w 을배정하기로하면, 각각의면의입장에서보면시계방향으로모서리의수가 만큼증가하는 (3에서 로가는것도mod 3으로생각하여증가하는것으로봄 ) 꼭지점에는 w가, 감소하는꼭지점에는 w 이붙어있는상황이됨. 한바퀴돌았을때원래의색으로돌아와야하므로이런증가 / 감소의총합 s는 3의배수라야함. 그럼각면의꼭지점에부여한수는 w의지수에색번호의증가 / 감소량을달아놓은것이므로꼭지점의수의곱은 w s = w 3t =이됨. i=

387 4. 조합고급문제 유한개의구간의합집합이구간 [0,] 을덮는다. 이구간들중에서각각서로소이고전체의길이의합이 / 이상인구간들을골라낼수있음을보여라. ( 통신강좌 ) 풀이 i) 먼저구간들의합집합이여전히 [0, ] 을덮고있게하면서구간들을하나씩제거할수있는데까지제거한다. ii) 이제임의의두구간을비교하면항상어느한구간이그구간이갖는수의최대값과최소값모두다른한구간보다크므로 ( 포함하지않으려면 ) 구간들사이의크기순서를정할수있다. 이순서에서임의의구간은이웃한두구간을제외한다른구간과 i) 에의해서로소가된다. 따라서이구간들을홀수번째, 짝수번째의위치에따라두집합으로나누면, 두집합각각이서로소인구간들의합집합이된다. 두집합의구간들의길이의총합이 이상이므로, 둘중한집합은구간들의길이의합이/이상이된다. 이집합이구하는구간들의집합이다. } 56. (a) 좌표평면의제사분면에일부무한히많은격자점들위에발광체가놓여있다. 이발광체는그점을중심으로특정각만큼빛을발하는데그특정각을이루는두반직선은두좌표축에평행하며방향도같다 ( 그두반직선이이루는각내부의모든방향으로빛을쏜다 ). 이발광체를유한개만남기고모두꺼버려도원래빛을받던점들은여전히빛을받고있게할수있음을보여라. (b) 좌표공간의제팔분영역에서발광체가세반직선 ( 모두세좌표축에평행하고방향도같음 ) 의내부공간각에모두빛을쏠때로하여위의문제를다시풀어라. ( 러시아 99 4차-y0-3) 증명 (a) 발광체중에왼쪽이나아랫쪽에다른발광체가없는경우만남겨놓고다끄면된다. 남은발광체가유한개임을보이면충분하다. 같은 x-좌표에여러개의발광체가남아있을수없으므로가장왼쪽에남은것이유일하게결정된다 (x-좌표의최소원의원리에의해존재성도틀림없음 ). 그럼그것의 y-좌표보다작은 y-좌표들에대해발광체가각각많아야하나만남아있으므로다른발광체는유한개뿐임이증명되었다.(b) 역시 x, y, z 중어느하나의값이더작은위치에다른발광체가없는경우만남겨놓고다끄면된다. 역시남은발광체가유한개임을보이자.[ 보조정리 ] z = c 꼴의평면에남은발광체는유한개뿐이다. 그런평면에서는 x-좌표와 y-좌표만비교하며생각하면되므로 (a) 와똑같은상황이기때문이다. x = a 꼴이나 y = b 꼴의평면에대해서도마찬가지. 이제 z-좌표가최소인발광체를하나잡자 ( 유일할필요는없다 ). 그럼나머지발광체는이점보다 x-좌표가작거나아니면 y-좌표가작다. 즉, 이발광체의좌표를 (a; b; c) 라하면, 나머지발광체들은 x =;:::;a 인평면들과 y =;:::;b 인평면들에만있고, 이런유한개의각각의평면에는보조정리에의해발광체가유한개씩만있으므로, 모두합해도유한개뿐이다. 57. 그램으로 을넘지않는서로다른정수무게의 n 개의추들의집합을생각해보자. 이런가능한모든집합은무게의합이같은두쌍의추를항상고를수있다. n 의가능한최소값을구하여라. ( 러시아 99 4 차 -y0-7) 풀이두추의무게차들을모두모은집합 X를생각하자. a>b이면 a b 0 이므로이집합은 0개이하의원소를갖고, a b = b c 인경우를제외하면중복되지않음. a b = b c 인경우는각각의 b에대해 (a; c) 가기껏해야한세트뿐 ( 두세트있으면a a 0 = c 0 c 가되어버림 ), 따라서, X에서중복은 b를택할수있는n 가지뿐. 즉, n 0 + (n ). 풀면 n 7. n =7일때실제,, 3, 5, 8, 3, 의예가있음. } 58. S 는원소의개수가 n 인집합이고, A = fa ;:::;A n g 은 S 의서로다른 n 개의부분집합들의집합이다. 이때 S 의어떤원소 x 가있어서 A [fxg, A [fxg, :::, A n [fxg 들이모두서로다른집합이되도록할수있음을보여라. ( 통신강좌 ) 증명귀류법. 임의의 x S 에대해 x 의포함여부만다른두집합늘존재 ( 그래서 Ai [fxg = A j [fxg) 한다고가정. 각 x 마다그런한쌍의집합을서로변으로연결 ( 여러쌍이있어도한쌍만연결 ). 그럼점 (= 집합 ) 이 n 개, 변이 n 개이므로 tree 보다변이많아서 cycle 이생김. 그 cycle 을한바퀴돌아오려면 x 의포함여부가두번바뀌어야하므로모순. 59. 자연수 n 의자릿수의합을 T (n) 으로나타내기로하자. () T (kn) 이모든k =; ;:::;99 에대해짝수이고k =993일때는홀수인 N을하나구하여라. () 모든자연수 k에대해t (kn) 이항상짝수가되는자연수 N은없음을보여라. ( 북유럽 993-4)

388 388 조합 풀이 () k 99 일때, 항상똑같은수가두번나열된형태이면좋겠음. k =993일때는자리올림이발생해서헝클어지고. 즉,99x<0 n < 993x, 다시쓰면 993 < x 0 n < 994 인좋은정수 x를찾는것이관건. k =993을곱했을때자리올림이발생하면그것을받아헝클어지기위해선일의자리가 9였어야하므로또 x의일이자리는3인것으로찾아야함. =0: 임에서 99 x = , n = 로하면됨. 즉, N = 실제로확인해보면 ok. () 그런수를 ` 재수 ' 라부르기로하자. N 이재수이면 N 의배수도모두재수.; 0; 0 ; 0 3 ;::: 을 mod N 으로관찰하면 0 m 0 n N 인 m>n 이만드시존재. 즉,0 m 0 n 꼴만살펴보면충분. 자릿수중맨마지막 0 들은무시해도상관없으므로 0 n 꼴의수들만살펴보면충분.( 0 n +)(0 n ) = 0 n 9 0 n = 로 9 의개수가짝수개이므로자릿수의합이홀수. } 60. 몇개의학교가어떤테니스대회에참가하였다. 같은학교에서온선수들끼리는시합하지않는다. 서로다른학교에서온두선수들끼리는각각정확히한번씩경기를갖는다. 두명의소년이나두명의소녀가벌이는경기를단식이라하고한명의소년과한명의소녀가벌이는경기를혼합단식이라한다. 소년의총인원과소녀의총인원은많아야 명밖에차이나지않는다. 단식경기의총횟수는혼합단식경기의총횟수와많아야 경기밖에차이나지않는다. 홀수명의선수를데리고참가한학교는최대몇학교인가? ( 캐나다 993-4) 풀이참가한학교의수를 n이라하고, k번째학교가보낸남학생수와여학생수를각각 bk, g k 라하자. 또, 남학생과여학생의전체인원을각각 B, G라하고, 단식경기와혼합단식의총경기수를각각 S, M이라하자. 문제의조건에의해 jb Gj, js Mj 이다. S = nx b i (B b i )+ nx g i (G g i ) i= i= = B + G nx b i nx gi i= i= nx nx M = b i (G g i )=GB b i g i i= i= 와같이계산할수있다. js Mj 의부등식에이것을대입하면 nx (B G) (b i g i ) i= jb Gj 이므로 nx (b i g i ) 3 i= 이된다. 홀수명의선수를데리고참가한학교는남학생수와여학생수가같을수없으므로 jb i g i j 이고, 따라서, 위식에의해그런학교는 3 개교를넘을수없다. 한편, n =3 일때, 두학교는각각남학생 명씩만, 나머지한학교는여학생 명만대회에참가시켰다면문제의조건이모두만족되므로, 홀수명의선수를참가시킨학교의최대수는 3 이된다. } 6. 칠판에숫자 0이적혀있다. 첫번째사람이그오른쪽에 + 나 중에하나를적으면두번째사람은그오른쪽에,,..., 993 중에하나를적고, 이렇게서로번갈아서계속한다. 한번적은숫자는다시적을수없고, 각각 993번의차례가지나면게임이끝난다. 게임이끝나면칠판에적힌계산을해서그절대값을구한다. 첫번째사람은이값을낮추기위해, 두번째사람은값을높이기위해최선을다한다면, 결과는어떻게되는가? ( 러시아 993 4차-y9-8) 풀이결과는 993이된다.() 갑 ( 첫번째사람 ) 은항상993 이상으로만들수있음을,() 을 ( 두번째사람 ) 은항상993 이하로만들수있음을보이자.() 갑의전략은처음에 + 만계속적다가적절한시점부터 만계속적는것으로한다. 를적기직전까지의수의합을 A, 나머지수의합을 B라하고,부터 993까지의합을 S라하자. 한번에택하는수의크기는최대 993이므로, A가반드시길이993의구간 [ S 993 ; S+993 ] 에걸리는순간이있다. 그때까지를A로하면, ja Bj = ja Sj 993 으로목표달성! () 을은 + 다음에는 X = f; 3; 5;:::;995g [f998; 000;:::;99g 의원소중에서아무거나쓰고, 다음에는 Y = f; 4; 6;:::;996g [f997; 999;:::;99g 의원소중에서아무거나쓰기

389 4. 조합고급문제 389 로한다. 만일한쪽부호가훨씬많아서한쪽집합을다소모했다면일단 993 을쓰고, 그이후에는아무렇게나쓰기로한다. 그럼두부호의개수가맞다면 ( 즉, 딱 개차이라면 ) X 의원소의합과 Y 의원소의합이같으므로딱 993 차이가나게되고, 개보다더차이가크다면그쪽부호에반대쪽집합에서수를몇개더양보하게되므로 ( 또그게 993 의부호와같으므로 ) 차이가 993 보다더벌어지게된다. } 6. (a) 어떤앨범에사진이 0장들어있다. 각사진마다 3명의사람이있는데, 가운데에는남자가한명있고왼쪽에는그의아들이, 오른쪽에는그의형제가있다. 모든사진에서가운데찍힌사람은각각다르다고할때, 이앨범의사진에있는사람은최소몇명인가?(b) 사진이 0장이아니라 n장있으면어떻게되는가? ( 러시아 993 최종-y9-4) 풀이사진이 n장일때등장인물의최소수가 T (n) =d 3n+ e 임을보이겠다. 등장인물끼리의가계도를사진에드러난정보만으로 ( 즉, 실제로는두사람이가계관계가있다고하더라도사진으로얽혀있지않다면서로관계가없다고보는것이다 ) 그린다고하자. 그때그림은몇개의연결성분 ( 어느한사람으로부터연결된모든사람들의모임을한연결성분이라함 ) 의합집합이되는데, 각각의연결성분에대해 T (n) 이위와같음을증명하면 ( 일차식에양의상수가추가된꼴이므로 ) 연결성분의개수가늘어날때필요한등장인물의수가더늘어난다는것은금방확인이가능하고, 따라서연결성분이오직하나뿐일때에대해서만증명하면충분하다. 각사진의중앙의인물을그사진의주인공이라고하고, 그사진을그사람의사진이라고말하기로하자. 이제등장인물들은어느세대에속하느냐에따라분류할수있다. 사진의주인공을기준으로, 가장윗세대의사진들부터순서대로늘어놓았다고하자. 특히, 가계도가모두연결되어있으므로, 같은형제들중에서도이미앞의사진들에서등장한사람이들어있는사진을먼저골라서늘어놓는것이가능하다. 즉, 이렇게늘어놓으면다음사진의세등장인물이모두새로운인물일수없으므로, 사진한장이늘어날때총인원은 명또는명이늘어난다는것을알수있다. 또한, 최소의경우를구하는것이므로,명만늘어나는경우가많을수록좋고, 따라서 명만늘어나는사진들은다른영향이우려되지않는다면무조건포함하는경우가최선이다. 이제형제가있는사람은무조건주인공으로등장하는게최선임을알수있다 (). 왜냐하면, 그사람이주인공으로등장하지않는다면다른사람의형제로등장했는데, 그때그사람을주인공으로하는사진을추가하면 ( 그리고, 사진의수를유지시키기위해사진열의마지막사진을제거하면 ) 아들만새로추가되므로총인원이 명만늘어나는경우이기때문이다. 단, 맨마지막에형제로등장한사람 명만은 ( 사진개수의제한때문에마지막사진을다시지우면도루묵이므로 ) 주인공으로등장하지않을수있다 (() 의예외상황 ). 최선인 T 명으로구성된 n장의사진이있고, 그중에독자가a명있다고하자. 각사진마다주인공이다르고주인공은모두아버지 ( 아들이있는사람 ) 이고아버지는모두주인공으로등장했으므로아버지는 n명이다. 독자는 ( 오른쪽에찍힐형제가없으므로 ) 주인공으로등장하지않으며따라서아들이없는사람이다.() 의예외상황으로언급된마지막형제도아들이없다. 또한아들이없는사람은 () 에의해형제가있으면예외상황으로마지막에등장했어야한다. 즉, 최선의경우에서는모든인물은아버지와독자와예외상황의마지막형제로구분된다. 즉, T = n + a ( 또는예외상황인경우에는 T = n + a +). 서로형제관계인사람들을모두모아놓은집합을형제그룹이라고부르자. 아버지가따로명시되어있지않은맨윗세대 ( 이들은서로형제관계로만얽혀있으므로한형제그룹이다 ) 를제외하면아버지가모두명시되어있으므로, 형제그룹의수는 n + 이다. 또, 형제그룹은독자인경우와그렇지않은경우로구분할수있고, 독자가아닌경우는최소 명이상으로구성되므로형제그룹의수는 ( 독자의수 )+ ( 독자가아닌사람의수 )/ 이하이다. 즉, n + a + n. 즉, a n +. ( 또는예외상황인경우에는 n + a + n+ 로하여a n+.) 따라서, T = n + a 3n+ ( 또는예외상황인경우에는 T = n + a + 3n+3 으로더커지므로예외상황이없는경우가최선임 ), 즉 T (n) d 3n+ e 임이확인되었고, 등호가성립하는경우는각세대마다 명으로된형제그룹이딱하나씩있는그림으로쉽게예를찾을수있으므로원하던증명이완료되었다. } 63. k개의자연수무게의추들의모임으로, 어떠한 m개의추를제외하더라도남은추들로 에서 55까지의자연수무게를모두만들어낼수있는집합을찾으려고한다.(a)k =0,m =로하여위의문제를풀어라.(b)k =,m =로하여다시풀어라. 구한집합이그런성질을틀림없이갖는다는것도증명해야한다. (Towns 993가을 JA6) 풀이 (a) f = f =,f n+ = f n + f n 의피보나치수열로충분함. f ;:::;f n 까지로 ( 임의로하나를제외하더라도 )부터 f n+ 까지잘만들수있음을귀납법으로증명. (b) g = g = g 3 =, g n+ = g n + g n 의수열로충분함을역시귀납법으로. }

390 390 조합 64. 볼록 993각형이볼록 7각형들로분할되어있다. 단, 원래 993각형의각변은통째로어느한 7각형의변이되고, 두 7각형은서로만나지않거나한꼭지점을공유하거나한변을공유한다. 같은 7각형에속하는,993각형의연속하는세변이있음을증명하여라. (Towns 993가을 SA4) 증명내부의변의수, 꼭지점의수,7각형의수를각각 e, v, n이라하고, 모든점에서의내각의합, 모든면에서의변의개수의합, 모든꼭지점에서의차수의합 ( 이건부등식 ) 등을두관점에서세기. e: 내부에있는변의수, v: 내부에있는꼭지점의수, n: 7각형의수라두자.() 모든점에서의내각의합을 double counting 하면 : 5n = 99 + v. () 모든면에서의변의개수의합을 double counting 하면 : 7n =993+e. 위의두식에서 n을소거하면4v = 0e ( ). (3) 993각형의원래꼭지점중에서내부로향하는변을갖는것의개수를 x라하면, 모든꼭지점에서의차수의합을 double counting 해보면 :e 3v + x. 이것을 ( ) 과함께풀어보면397 = 0e 4v v +5x>5x. 65. ; ; 3; 4;::: 의자연수전체를차례대로두집합 A, B 에나누어담는데, 다음의절차를따른다 : 먼저 을 A 에넣고,;:::;n 의모든수를 A 혹은 B 로나누어담았으면, n 의약수가 A 에몇개있는지검사한다. 만일홀수개의약수가 A 에있으면 n 을 A 에넣고, 짝수개가있으면 n 을 B 에넣는다. A 에들어가는처음몇개항을써보면 ; ; 3; 5; 6; 7;::: 이다. 이때 A 의원소들을공통적으로어떤좋은구조를갖게된다. 이좋은구조란무엇을의미하는가? ( 통신강좌 ) 풀이 square-free 수. 추측에좀시간이걸릴뿐증명은귀납법으로간단. } 66. 각각주기가 m, n인두수열이있다 (m, n은서로소이다 ). 이두수열의첫부분은최대몇항까지일치할수있는가? (Towns 994가을 SA5) 풀이답은 m + n. m>n이라하고, 귀류법으로 m + n 항이상일치시킬수있는m + n 이최소인경우를택했다고하자 ( 최소원의원리 ). n =일때는직접따져보면곤란함을금방확인할수있으므로 n. m을 n으로나눈나머지를 r이라할때 ( 서로소이므로 r ), 주기 m인수열의순환부마지막 r항과그뒤로 m 항을합한 r + m 항이 (n; r)-문제가되는상황이므로최소성에모순 (n + r<m+ n). 실제처음 m + n 항이일치하는수열을만드는건 0과 만으로된수열만생각하더라도금방만들수있을듯... } 67. 어떤학교의 64 명의학생이 5 개의올림피아드에출전한다. 각각의올림피아드에는적어도 9 명씩의학생들이참가하고, 네개이상의올림피아드에참가하는학생은없다. 어느세올림피아드에도공통으로참가하는학생이있다면, 공통으로참가하는학생이다섯명이상인두올림피아드가존재함을증명하여라. ( 폴란드 994/995 차 -4) 증명임의의세올림피아드 A, B, C를택했을때이세올림피아드에공통으로참가하는학생은항상적어도한명있고, 다른세올림피아드쌍에공통으로참가하는학생과중복될수없음. 그럼각세올림피아드에참가하는학생들이독립적으로한명씩은있으므로그 5 3 =0명을제외하고생각하면, 문제는다음과같이바뀜.54명의학생이 5개대회에출전하는데각대회에는 3명이상씩참가하고, 한학생은많아야 3개대회에참가할때, 두명이상의참가자를공유하는두대회가있음을보이면됨. 이것은 > 5 임에서... 좀더알기쉽고조심스럽게잘말하면되는건데 이상 5 이하의정수들 a, b, c 에대해 a 3 b 5 c 꼴의수들을생각하자. 이런꼴의수들중임의로 3 개의원소를뽑으면그중에는반드시하나가다른하나를나누는두원소가있음을증명하여라. ( 통신강좌 ) 증명우선 a 3 b 5 c 과 a 3 b 5 c 에대해만약a = a, b = b 라고하면항상둘은반드시한쪽이다른한쪽을나누는관계에놓이게된다. a 3 b 5 c 에대해a, b가고정되어있으면c는많아야 개만존재한다 ( 귀류법 ).! 가능한것 :6 =36가지 x ; x ; x 3; x 6; x ; 3 5 x ; 3 5 x 3; x 6; x ;3 3 5 x ;3 3 5 x 3; x 6; x ; x ; x 3; x 6;6

391 4. 조합고급문제 39 여기서 x ; >x ; > >x ;6 >x ;6 >x 3;6 > >x 5;6 >x 6;6 임은당연하다 ( ㄴ자열 ). 이 개항에사용할수있는수는많아야 6 개이므로적어도 5 개의항은빠져야한다. 마찬가지로다음ㄴ자열 x ; >x ; > >x ;5 >x 3;5 > >x 6;5 의 9 개의항중에서도 6 개의수만사용할수있어서적어도 3 개의항은빠져야한다. 그다음의ㄴ자열에서도 7 6= 개항은빠져야한다. 즉, 이것만으로도 36 (5+3+)=7 항밖에남지않았다. 69. 크기가 n n (n ) 인체스판이있는데, 각칸에 부터 n 까지의수가적혀있다고한다. 그러면, 숫자의차이가적어도 n 이되는이웃하는두칸이있음을증명하여라. ( 통신강좌 ) 증명모든이웃하는두칸의숫자의차이가 n 이하라고가정하자. k =; ; ;n n 일때 A k 는 ; ; ;k 가적혀있는칸들의집합이라고 B k 는 k + n; k + n +; ;n 이적혀있는칸들의집합이고 C k 는 A k ;B k 의원소가아닌칸들의집합이다. 가정에의해, A k 와 B k 의원소들은서로이웃하지않는다. 그리고 C k 의원소의갯수는 n 개이다. 따라서임의의 k 에대하여 C k 의어떤원소도포함하지않는행과열이존재하고, 따라서어떤행과열은 A k 또는 B k 의원소로만이루어져있다. A k 의행과 B k 의열이동시에있을수도없다 ( 그행과열이만나는곳주변에서문제가생김 ). A 이나 B n n 은그렇게될수없다. 그러므로 부터 n n 까지숫자중에서어떤 k 가존재하여 B ;B ;:::;B k 는어떤행과열을모두채우게되고, B k+ 은그렇지않게된다. 따라서 A k+ 은어떤행과열을모두채우게되고, 따라서이는 B k 와겹친다. A k+ \ B k =? 이므로 A k+ 과 B k 의열이만날수없어서모순이므로주어진명제가증명되었다. 70. N = f; ;:::;ng (n ) 인 N 에대해서, N 의부분집합 A i 들로이루어진집합 F = fa ;A ;:::;A tg ( i t) 를생각하자. 만약모든 fx; yg ½N 에대해 ja i \fx; ygj = 인 A i 가 F 에존재한다면, F 는분리되었다고한다. 또한만약 N 의모든원소가적어도하나의 A i 에속해있다면 F 는덮였다고한다. F 가분리되어있는동시에덮여있을최소의 t 값인 f(n) 을구하여라. ( 통신강좌 ) 풀이 k n< k 라가정한다. 다음과같이 k개의원소를가지고분리되었으며덮인집합을만들어보자. N의원소를 k자리 진수로나타내자. A i 를 i번째자리가 인 N에속한숫자들의집합이라고하자. 이러한집합들의집합은명백하게분리되어있으면서덮여있다. 따라서 f(n) k이다. 역으로 A ;A ; ;A t 들의집합이분리되어있으면서덮여있다고가정하자. 모든 N의원소에대하여, 그원소가 A i 에속해있다면 i번째자리에 을그렇지않은경우는 0을대응시킨 0과 의수열을생각한다. A i 의집합이분리되어있으므로, 이러한수열은모두서로다르다. 또한덮여있으므로,00 0의수열은존재하지않는다. 따라서 n< t 이고 t k이다. 즉 f(n) k, 따라서 f(n) =k = [log n]. } 7. 어떤연결그래프 G 에서몇개의변을제거하면이 G 가 개의서로떨어진부분으로나뉜다고할때, 이변들의집합을 S 라고하자. 그리고이중에서 S 의모든진부분집합들은위의조건을만족하지않는다면, 특별히이 S 를 cutset 이라고하자. 또 G 의 ` 확장수형도 ' 란 G 의모든정점을포함하고 G 의변들만을일부취하여이루어진수형도를뜻한다고할때, 다음을보여라. (a) G 의임의의 cutset 은임의의 G 의확장수형도와적어도한변을공유한다. (b) G 의변들의집합이어떤확장수형도 T 의꼭한변만을포함하고, G 의임의의회로와짝수개의변을공통으로가진다면이집합은 cutset 이다. ( 통신강좌 ) 증명 (a) 만약임의의 cutset( 이것을 C라하자 ) 과 G의어떤확장수형도T와아무변도공유하지않는다고가정하자. 그렇다면 E(G) C ¾ T 이고, T 로인해G는연결되어있다. 그러므로, 이것은가정에위배되어 C는 cutset이아니다. (b) 가정한 G 의변들의부분집합을 A 라하자. 그러면가정에의해 A \ E(T )=v 라고하자. 그렇다면 v 를제거하여나눠지는 V (T ) 로구성되는두부분그래프 G, G 를생각해보자. 또한 G 의오일러회로를생각해보면, 이회로의시작점 (= 끝점 ) 이 G 에있다가정하면 G 과 G 를연결하는선들은항상짝수개존재할것이다. 이것을각각 v, v, 라하자. 그렇다면가정에의해이중하나는 v 일것이다.( 일반성을잃지않고 v 이라하자.) 이때이 G 에서 v 과다른홀수개의 v i 들을제거하

392 39 조합 면분명히 G 과 G 는서로분리된그래프이다. Γ Γ ϖ Γ ϖ ϖ ϖ3 ϖ4 Γ 7. 여러종류의입자들로이루어진한집단이시간에따라다음규칙을만족하며변해간다 : 각각의입자들은어떤하나의세대에속하며, 각각의 n 에대하여 n + 세대는 n 세대의후손들로이루어진다. i (i =0; ; ;:::) 류의입자는정확히 0 류, 류, 류, :::; i+ 류를하나씩, 총 i + 개후손들을생산한다. 제 세대가 k 류의입자하나로이루어졌을경우 n 세대의입자의개수를 N(n; k) 라고할때, N(n; k) 의공식을하나구하여라. ( 통신강좌 ) k + 풀이수학적귀납법으로 N(n; k) = n+k n + k n 임을보인다. n =인경우는명백히성립한다. 이제, 위식이어떤n에대하여, 모든 k에서성립한다고가정하자. N(n +;k) = = = k+ X N(n; i) i=0 ³ n + i n k+ X i + n + i i=0 k+ X ½ ³n + i n i=0 ³ n + i ¾ n 한편, bx j=a ³ j m = bx j=a ½ ³ j + ³ j ¾ = m + m + ³ b + ³ a m + m + 을이용하면 N(n +;k) = = = ³ n + k + ³ n ³ n + k + ³ n + n n n n ³ n + k + ³ n + k + n n k + ³ (n +)+k (n +)+k (n +) 따라서, 수학적귀납법에의하여 N(n; k) = k + n+k n + k n 임이증명되었다. } 73. X 는정수집합의부분집합이다. 000 이상 000 이하의모든자연수를 X 의공집합아닌어떤부분집합의원소들의합으로나타낼수있다고한다. X 의원소의개수의최소값을구하여라. ( 통신강좌 ) 풀이 jxj = 인경우에는가능하다.( 예컨데 X = f; ; 4; ; 04g 라두면된다.) jxj =0 일때는불가능함을보이자. X = fx ;x ; ;x 0 g 이라두자.

393 4. 조합고급문제 393 x i 중어떤둘이상의 500 보다큰경우 ( 편의상 x 9, x 0 이라하자.) 임의의 A ½fx ;x ; ;x 8 g 에대하여 S A = X xa x 라두자.( 편의상 S Á =0 으로약속하자.) S A ; S A + x 9 ; S A + x 0 ; S A + x 9 + x 0 중많아야 3 개만 000 이상 000 이하이다. 왜냐하면 x 9 + x 0 00 이기때문이다. 가능한 A 의갯수는 3 8 = 768 < 00 이므로,000 이상 000 이하의수를모두나타낼수는없다. 많아야 개만 500 이상인경우 ( 편의상 x 0 이라하자.) x ;x ; ;x 9 중 개만뽑은합은 000 이될수없다. ³ 9 ( 0 ) =987< 00 이므로,000 이상 000 이하의수를모두나타낼수는없다., 에서 jxj =0 인경우는불가능. 즉최소값은 이다.(jAj 는 A 의원소의개수를뜻한다.) } 74. 모든유리수들을다음과같이두집합 A, B 로분할하는것이가능한가? () 같은집합에서고른두수 ( 두수는같을수있다 ) 의합은항상A에속한다. () 같은집합에서고른서로다른두수의합은항상 A에속한다. ( 벨로루시 995-A8) 증명 ( 한석원 ) () 가능하다고가정하자. 어떤임의의유리수 a 에대해, a= 가 A; B 중어디에속하던관계없이 a=+a= =a 는 A 에속한다. 그러면 A = Q 가되어모순. 따라서불가능하다. () 가능하다고가정하자. 일단 0 B 라가정하면 0 이아닌임의의 B 의원소 k 를잡으면 0+k = k A 로모순. ) 0 A 이제임의의 0 이아닌유리수 a 를생각하자. i) a A a A 라면, a +a =3a A, a +3a =4a A, a +4a =5a A, a +5a =6a A 이다. a 가 A 에속하지않는다면, 이아닌임의의유리수 k 에대해 ka 와 ( k)a 모두가같은집합에속할수없다. 여기서일반성을잃지않고 ka B, ( k)a A 라할수있겠다. a+( k)a =(3 k)a A 이므로위와마찬가지로 (k )a B 이고, ka+(k )a =(k )a A, (k )a+( k)a =(k+)a A, (k +)a +( k)a =3a A 이다. () 그리고 3a + a =4a A, 4a + a =5a A, 5a + a =6a A 이된다. ) 6a A ii) a B a 가 A 에속하지않고 a=3 +a=3 =a 이므로 a=3 과 a=3 은같은집합에속할수없다. a=3 A 라면 () 에의해 a A 로모순. 따라서 a=3 A 이고이때 () 을두번적용하면 3 3 a=3 =6a A i), ii) 에서어느경우나 6a A 가되어 A = Q 가되므로모순. 따라서불가능하다. 75. 하나의내각이 80 ± 보다큰사각형을부메랑이라고부르자. C 를 s 개의변을갖는볼록다각형이라하자. 그리고, C 의내부영역을서로교차하지않는 q 개의사각형의합집합으로나타내었다고하자. 이사각형들중 b 개가부메랑이라면, q b + s 임을보여라. ( 캐나다 995-3)

394 394 조합 증명부메랑의오목각에해당하는꼭지점을오목점이라하자. 각각의부메랑마다오목점이하나씩대응되므로오목점은모두 b개이고, 이오목점은모두 C의내부에있다. q개의사각형의내각의합은 ¼q이고, 이것은오목점에모이는각의합과 C의모든꼭지점에서의각의합을포함하므로 q¼ b¼ +(s )¼ 이성립한다. 양변을 ¼ 로나누면문제에서원하는식이된다. 별증이그림의점, 선, 면 ( 무한면포함 ) 의개수를각각v, e, f라하자. 그리고, 어떤면의꼭지점이다른면의변에서만날수도있으므로 ( 이때이변은개이상의선으로이루어진다 ), 이그림의점들중에서어떤영역의변의내부에놓이는점의개수를 n이라하자. n의각각의점은어떤면이갖는점과선의개수를각각 씩증가시킨다. 각면의가지는선의개수를모두세면 4q + s + n 이되고, 각각의선은두개의면으로부터세어지므로 e = 4q + s + n =q + s + n 이다. f = q + 임은당연하고, v는부메랑의오목각을갖는점이 b개, 평각을갖는점이 n개, 그리고 C의꼭지점이 s개등이므로이것만으로도 v b + s + n 이다. 오일러의공식에의해 µ =v e + f (b + s + n) q + s + n +(q +) q + b + s + 따라서, q b + s 이성립한다. 76. 한원의둘레위에 4n 개 (n 은자연수 ) 의점을택해, 시계방향으로차례로 ; ; 3;:::;4n 의번호를매겼다. 짝수번호를가지고있는 n 개의점들을 n 개의쌍으로나누어, 각쌍의두점을녹색현으로연결하였다. 마찬가지로홀수번호를가지고있는 n 개의점들도 n 개의쌍으로나누어, 각쌍의두점을금색현으로연결하였다. 어떠한세현 ( 색은상관없이 ) 을택해도한점에서만나는경우가없다면, 녹색현과금색현의교점이 n 개이상있음을증명하여라. ( 호주 995-) 증명귀납법. n =일때는자명. n 일때성립함을가정하고 n일때를보자. 녹색현과금색현의교점을호점이라말하자. 모든금색현마다호점이있다면자명하므로, 그렇지않을때만보자 ( 드리블 ). 즉, 어떤녹색현과도만나지않는금색현을 l이라하자. 원이 l에의해두영역a와 B로분할된다. A의녹색현의개수를a개 ( 즉, 녹색점이 a개 ), B의녹색현의개수를b개 ( 녹색점이 b개 ) 라하자. a + b = n 이므로, A, B 영역에각각 a, b개이상의호점이있음을보이면된다. 대칭적이므로, A 영역에 a개이상의호점이있음만보이면충분하다. l의양끝점 ( 금색점 ) 을제거하고, 그두점바로밑 (B쪽) 의점 ( 녹색점 ) 도하나씩제거하자. 그리고, 남은 B 영역의녹색점들은원래의연결을잊어버리고이웃한것끼리새로연결하자. 그럼 B 영역에는 b 개의녹색현이있고, 각각의현마다정확히 개씩의호점이있다. 그리고, A 영역은원래그대로불변. 귀납법의가정에의해현재 n 의상황이므로 n 개이상의호점이있어야하는데, B 영역에는 b 개뿐이므로, A영역에 a개이상의호점이있음. 끝 크기의판에서갑과을두사람이게임을한다. 판옆의상자에 n 개의칩이들어있다. 갑이먼저하는데,7 개이하의칩을집어서 ( 이미판위에놓인칩을집을수도있다 ) 비어있는칸에놓는다. 단, 한칸에는한개의칩만놓일수있다. 그럼을은판위에서연속한칸을차지하는칩한무리를집어서다시상자에넣는다. 갑의목표는 n 개의칩을모두판위에연속하게놓는것이다. (a) n =98이면먼저하는사람이이김을증명하여라. (b) 먼저하는사람이이기는 n의최대값은얼마인가? (Towns 995 가을 JA6) 풀이우선 (b) 를먼저보임. 먼저하는사람이틀림없이이길수있다면그가한수만더진행하면이기는 ( 승리직전의 ) 순간이틀림없이있고, 그때의연속하게놓여진각덩어리의개수를 b ;a ;a ;:::;a k ;b 라하자. 우선 b i 들은 (a i 들도그렇지만 a i 들은더작게될거라서여기선언급안함 )7보다클수없고( 그걸밀어버리면다음차례에 n개를다못놓음 ), 따라서, a + + a k n 34. 그럼 max a i n 34. k 두번째사람이max a i 를하나지워버릴때를생각하면첫번째사람은마지막수에서그것을복구하고또각덩어리사이사이에하나이상을두어덩어리들을모두연결시켜야하므로 max a i +(k +) 7. 따라서,7 n 34 + k + p n k 풀면 n 끝. 위의풀이로부터 을만들어야 n =98일때이긴다는것을알수있고, 그것은먼저 을만들고나면고담턴에바로만들수있음. }

395 4. 조합고급문제 한변의길이가5인정육각형을, 변과평행한직선들에의해, 한변의길이가인정삼각형들로분할하였다. 이작은정삼각형의꼭지점들을매듭이라고부르자. 전체의절반이상의매듭에표시를했다면, 표시된매듭들중에한원위에있는5개의점이있음을보여라. ( 러시아 995 4차-y9-7) 증명점은총 9개. 매듭은 46개이상. 중심 O를제외하면매듭은 45개이상. O를중심으로하고한점이라도지나는원은꼼꼼히따져보면 개.45> 4 이므로비둘기집의원리에의해이중에서매듭을 5개이상갖는원이존재. 79. jx yj f5; 7; g 일때는항상 f(x) 6= f(y) 이고, 모든정수의집합 Z 에서정의되고치역이 f; ;:::;kg 인함수 f 가존재하는최소의양의정수 k 를구하여라. (APMO 995-5) 풀이 (a) f(x), f(x 5), f(x +7) 에서, 세수는모두달라야한다. 이때 f(x +) 를택해서살펴보자. jx + (x 5)j =7에서 f(x +)6= f(x 5) 이고, jx +7 (x +)j =5에서 f(x +)6= f(x +7) 이다. 이때 k =3이라면, f(x) =f(x +) 가되어야한다. 비슷한방법으로 f(x) =f(x +) = f(x +4) = = f(x +) 를유도할수있다. 여기서 jx + xj =에서 f(x) 6= f(x + ) 이므로, k 4이어야함을알수있다. (b) f : z!f; ; 3; 4g인함수의존재를밝힌다. f(0) = 이라하고, i =; ; 3; 의순으로 f(i), f( i) 를다음과같이정의한다. 임의의양의정수 i에대하여 에속하지않는최소의자연수를 f(i) 라고하고, A i = ff(j) j i j f5; 7; g; i <j<ig B i = ff(j) j j i f5; 7; g; i <j ig 에속하지않는최소의자연수를 f( i) 로정의한다. 임의의 x, y 에대하여 jx yj f5; 7; g 이면, 정의에의해 f(x) 6= f(y) 가된다. 또한 ja i j 3, jb i j 3 이므로 f 는 Z 에서 f; ; 3; 4g 로가는함수가된다. (a), (b) 에의하여 k 의최소값은 4 이다. } 80. 에서 n 까지의번호가매겨진 n 권의책이책장에꽂혀있다. 우리는이중몇권의책을뽑아제일뒤쪽에다시꽂는조작을할수있다.( 단, 한번의조작에여러권을뒤로보낼수있으나, 뒤로보내어지는책들의순서는바뀔수없다.) 최소한몇번의조작을하면처음에책이어떻게꽂혀있더라도번호순서대로다시배열할수있겠는가? ( 통신강좌 ) ( 조작의예 ) 조작! 534 조작! 345 풀이 ( 서울과학고 학년임성진 ) 일렬로배열되어있는책의상태에서사이클을다음과같이정의하자. 책의배열상태에서 k +이 k의뒷쪽에나타나면 k와 k +은같은사이클에들어가고, k +이 k의앞쪽에나타나면 k와 k +은다른사이클에들어간다. 또한한사이클은연속한수로만이루어져있다. 위와같은방식으로하면 부터시작해서차례대로몇개의사이클을만들수가있다. 예를들면다음과같다. h책의배열상태i h사이클i (; ) (3) (4; 5; 6) (7; 8) 우리가마지막으로원하는배열상태는사이클이 개인상태이다. 따라서우리는이사이클의개수를줄여나가야한다. 이제어떻게조작을하면사이클의개수를최대로줄일수있는지알아보자. 조작을해서뒤로보내어지는수들도연속한수끼리묶어보자. 예를들면 357 을뒤로보낸다면다음과같이묶는다. (3) (5) (7) 어떤묶음이 i 부터 i + k 까지의수로이루어져있다고하자. 이묶음이뒤로보내어질때 i 부터 i + k 까지의상대적배열상태는바뀌지않으므로사이클의상태에변화를줄수있는부분은 i 과 i 사이, i + k 와

396 396 조합 i + k +사이이다. 이묶음이뒤로보내어지고나면반드시 i는 i 의뒤로가고, i + k는 i + k +의뒤로간다. 따라서 i 과 i가다른사이클에들어있었다면조작후두사이클은연결되어사이클의개수가하나줄고, 그렇지않았다면사이클의개수는변함이없다. 또한 i + k와 i + k +이같은사이클에들어있었다면조작후그사이클은두개의사이클로갈라져사이클의개수가하나늘고, 그렇지않았다면사이클의개수는변함이없다. 그러므로한묶음이뒤로이동하면서줄일수있는최대의사이클의개수는 개이다. 그리고이렇게되기위해서는묶음의첫수가어떤사이클의첫수이고, 묶음의끝수가어떤사이클의끝수가되어야한다.( 단, 묶음의첫수가 인경우는안된다.) 이제사이클의개수를최대로줄이는방법을생각해보자. 사이클의개수는위의조건을만족시키는묶음의개수만큼줄므로, 위의조건을만족시키는묶음을최대로많이만드는방법이사이클을최대로줄이는방법이다. 위의조건을만족시키는묶음을가장많이만드는방법은당연히짝수번째사이클을모두묶음으로만드는방법이다. 예를들면주어진배열의사이클이 (,)(3)(4,5,6)(7,8) 이라면 번째와 4번째사이클인 (3) (7, 8) 을묶음으로만들어뒤로보내는것이다. 그러면사이클은 (,, 3) (4, 5, 6, 7, 8) 이되어개가줄어든다. k + 따라서사이클이 k개인배열이주어지면조작을하여사이클이개인배열상태로만들수있다. 또한이조작이사이클을가장많이줄이는방법이다. 따라서이런조작을계속하는것이가장빨리책을정돈하는방법이다. 이제 n권의책이임의로배열되어주어졌다고하자. 처음주어진상태의사이클의개수가많을수록정돈하는데걸리는조작의최소회수가커진다. 처음주어진상태에서사이클의개수가가장많은경우는 n; n ; ; 로주어지는경우이며이때사이클의개수는 n개이다. 이상태에우리가앞에서만든조작을계속시행하자. 그리고 i번실행한후의사이클의개수를 a i 라하자. 그러면다음과같이된다. ai + a i+ = ; a 0 = n(i 0) 이제몇번을시행하면 a i = 이되는지알아보자. i) n = 일때조작할필요가없다. 즉 0 번의조작이면충분하다. ii) k + n k+ 일때 (k =0; ; ; ) 따라서 k + 번의조작이필요하다. k + a 0 = n k+ k + a k k + a k. 0 += a k = a k+ = 따라서임의로배열된 n권의책을정돈하는데필요한조작의최소회수는 Ã n =일때 0번 n>일때 [log (n )] + 번 이다.( 단,[ ] 은가우스기호이다.) } 8. 자연수 n에대해,`' 과기호 +,, (,) 들만을여러번이용하여 n을표현할때 ( 그러나 을,과같이사용할수는없다 ), 사용해야하는 `' 의최소개수를f(n) 이라고하자. 예를들어,80=(++++ ) (+++) (+++) 이므로 f(80) 3 이다. n> 일때다음부등식이성립함을보여라. ( 브라질 996-3) 3log 3 n f(n) < 5log 3 n 증명 ( 임준혁 ) 좌변을강한수학적귀납법으로보이자. f() = > 3log 3, f(3) = 3 > log 3 3 이다. ( 참고로 f() = 이므로성립하지않는다.) 이제 ; 3;::: ;n 에좌변이성립한다고가정하자. n 일때도성립함을보이겠다. n 을 x + x + + x p 로표현할때 ( 예를들면 3=(++)(+++++)+(++++) 에서는 로표현한것이다.) 필요한 의개수는 f(x )+f(x )+ + f(x p ) 이상이다.

397 4. 조합고급문제 397 i) p ) x i 중 이없을때이때는 x i n 이기때문에귀납가정을쓸수있다. f(x )+ + f(x p ) px 3log 3 x i =3log 3 (x x x p ) i= 따라서 x x x p x + + x p = n 임을보이면되는데, 이는성립한다.(x i 기때문에 ) 따라서 f(x )+ + f(x p ) 3log 3 n 이다. ) x i = 인 i 존재이러한 i 가 k 개존재한다고하자. 일반성을잃지않고 x = x = = x k = 이라할수있다. k 일때는 f(x )+ + f(x p) f(x + + x k )+f(x k+ )+ + f(p) 이므로 ) 의경우에서보일수있다. 따라서 k = 일때만고려해도된다. 같은식으로 f(x )+ + f(x p)=+f(x )+ + f(x p) +3log 3 (x x p) 역시마찬가지로 x x p x + + x p = n. ) f(x )+ + f(x p) +3log 3 (n ). n 4일때는3(log 3 n log 3 (n )) 이므로 +3log 3 (n ) 3log 3 n이다. 따라서증명된다. (* n 3이므로 log 3 (n ) = y라고두면 y 이라서 3 y+ 3 3 y +이성립하기때문에 ( 세제곱하면간단히보일수있다.)) 따라서 n 3일때만고려해주면되는데 f(n) =n으로그냥성립한다. 종합하면 p 일때는성립한다. p =일때n = x x k 로표현하자.( 예를들면면 4 = 7 3 로표현한것이다.) 이때필요한의개수는 f(x )+ +f(x k ) 개이다. f(x )+ +f(x k ) 3(log 3 x +log 3 x + +log 3 x k )=3log 3 n이므로역시성립하게된다. 따라서좌변이강한수학적귀납법으로성립하게된다. 우변을역시수학적귀납법으로보이자. n =; 3; 4; 5일때f(n) 이각각; 3; 4; 5가되어성립하게된다. n이성립할때 3n; 3n +; 3n +일때성립함을보이자. N을 n을최소의 의개수로표현하는방법이라고하자.3n =(++)N이면 f(n)+3개로표현할수있다. ) f(3n) f(n)+3 3n +=(++)N + 이면 f(n)+4 개로표현할수있다. ) f(3n +) f(n)+4 3n +=(++)N ++ 이면 f(n)+5 개로표현할수있다. f(3n +) f(n)+5 그런데 f(n) +5< 5log 3 n +5=5(log 3 n +)=5log 3 3n 이므로 f(n) +3;f(n) +4;f(n) +5 모두 5log 3 3n 보다작다. 따라서 f(3n), f(3n +), f(3n +)< 5log 3 3n 이므로 3n; 3n +; 3n + 일때도성립함을알수있다. 따라서모든 n 에대하여우변이성립한다. 종합하면 3log 3 n f(n) < 5log 3 n 이성립한다. 8. 국가결혼상담소는 n 쌍의남녀를초청하여, 다음조건들을만족시키는 7 개의토론조를만들고자한다. () 각조의구성원은같은성이어야한다. 즉모두남성이거나모두여성이다. () 임의의두조의구성인원의차는 0 또는 이다. (3) 모든조는적어도한사람의구성원을갖는다. (4) 각자는반드시한조에만속한다. 가능한 n (n 996) 의값을모두구하고, 그이유를밝혀라. (APMO 996-4) 풀이 ( 과기원수학과 95학번허충길 ) 임의의두조의구성인원의차가 0 또는 이므로각조가취할수있는구성인원의값은 k명이나 k +명두가지뿐이다. 또한남성조의개수를 x개, 여성조의개수를 y개라하면, 일반성을잃지않고 x y라할수있다. 이제남성조에있는 k +명짜리조의개수를 x 0 개, 여성조에있는 k +명짜리조의개수를 y 0 개라하자. 그러면남성조의사람수의합과여성조의사람수의합이n명으로같아야하므로 k(x x 0 )+(k +)x 0 = k(y y 0 )+(k +)y 0 ()

398 398 조합 이다. 즉, k(x y) =y 0 x 0 () 이다. 이제 x; y; x 0 ;y 0 ;k 의조건을명확히해보자. 각조가적어도한사람의구성원을가지므로 k (3) 이다. 전체조의개수가 7 개이므로 x + y =7 (4) 이다. x 0 과 y 0 은각각남성조와여성조의개수보다작아야하고, x y 이므로 () 식에서 y 0 x 0 이다. 따라서 0 x 0 y 0 y x (5) 이다. 여성조의인원수만세면 n 명이므로 n = k(y y 0 )+(k +)y 0 = ky + y 0 (6) 이다.() 식과 (5) 식에서 k(x y) k(x y)+x 0 = y 0 y (7) 이다. 따라서 k(x y) y 0 y (8) 이다. 위 (8) 식에서 y 0 이존재하기위해서는 0 k(x y) y (9) 이어야한다. 이제위식들을정리해보면 이다. 이식에 x =7 y 를대입하면 이다. (3) 식과 (3) 식에서 (y; k) 쌍을구해보면, ky + k(x y) n = ky + y 0 ky + y (0) 0 k(x y) y () k(7 y) n (k +)y () 0 k(7 y) y (3) (y; k) = (6; ) (7; ); (7; ) (8; ); (8; ); (8; 3); (8; 4); (8; 5); (8; 6); (8; 7); (8; 8) 이다. 따라서이 (y; k) 쌍을 () 식에대입하여 n 의값을구하면 9 n 6 8 n 4 7 n 3 36 n n n n 64 n =7 이다. }

399 4. 조합고급문제 수학로또복권에는 부터 36 까지의번호가적혀있고, 복권을살때이중에 6 개의번호를선택하여표시한다. 부터 36 까지중에서 6 개의수를추첨으로뽑고, 추첨된번호가하나도포함되지않은복권이당첨된다. (a) 9장의복권을산다면그중반드시당첨권이한장이상나오도록번호를선택할수있음을증명하여라. (b) 8장의복권으로는위와같이하는것이불가능함을증명하여라. (Towns 996가을 JA6) 증명 (a) 뭐잘만들면됨... 예 : (...6), (..3,7..9), (4...9); (0...5), (0..,6..8), (3...8); (9...4), (5...30), (3...36). (b) 귀류법. 복권 3 장에동시에나타나는수있으면안됨 ( 그수와나머지복권에서수하나씩뽑으면끝 ). 어떤두장에동시에나타나는수있고또다른두장에동시에나타나는수가있어도안됨 ( 그두수와나머지복권에서수하나씩뽑으면끝 ). 복권에쓰인수는총 48 개이므로적어도 번은중복. 어떤두장 A, B 는공통된수를갖고있고, 그외에두장이상에서나타나는수는모두 A 나 B 에나타나야하는데, 그런수는 개가못되므로모순. 84. X 는 k 개의원소를가진집합이다. 는원소의개수가 k 개인 X 의부분집합들중의몇개를모아놓은것이고원소의개수가 k 개인 X 의부분집합은각각 에속하는단하나의집합에만포함된다 ( 부분집합이라는뜻 ) 고한다. 이를만족하려면 k + 이소수여야함을증명하여라. ( 통신강좌 ) 증명 i는 i (k ) 인정수이고 A는원소의개수가 i개인 X의한부분집합이라고하자. 이제 k 개의원소를가진 X의부분집합중에i개의고정된원소를가진것들의개수를두가지방법으로세어보자. µ k i C i (A) 를집합A를포함하고 k 개의원소를가지는 X의부분집합의모임이라면 j C i (A) j= k i 된다. 또한 D i (A) 를 A를포함하는부분집합중 에속하므로원소의개수가 k개이다. D i (A) 의한집합 B의원소중에A에속하지않는 k i개의원소중하나를빼면 C i (A) 의한집합이된다. 원소의개수가 k 개인 X의부분집합은 에속하는정확히한개의집합에만포함되므로 jc i (A)j =(k ) jd i (A)j가성립하게된다. µ 즉각각의i ( i (k )) 에대해 (k i) k i 여야한다. 만약 k +이소수가아니라면 k i µ 한소인수 p(< k+) 를잡을수있고이때 i = k p(< k ) 로잡으면 p k i 가되고 k i µ p k + p 이된다. p µ k + p (k + p)(k + p ) (k +) = 인데 p j (k +) 에서 p는 k +;k+3; ;k+ p를모두나 p (p )! 누지못하므로모순이생긴다. 그러므로 k + 은소수이다. 가 85. n n n 인체스판을지배하려면루크가최소한몇개있어야하는가? 여기서루크는항상직선으로이동하며, 루크들이체스판을지배한다는말은이체스판안의각각의칸에대해항상가로, 세로, 높이방향중적어도한방향에는루크가있다는것이다. ( 통신강좌 ) 풀이 ( 과기원 97 학번이학성 ) n n 인직육면체를 ` 한층 ' 이라고부르자. Η Γ Ε Φ Χ Α Β 루크의개수가 m 개로최소인층 L 을택하자.(L 을수평에있다고가정한다.) L 상의루크들은 m 개의행과 m 개의열을지배한다고하자.( 여기서행은 AB 에평행이고열은 AD 에평행하다고하자.) L 안의루크들에의해지배되지않는작은정육면체가층 L 안에 (n m )(n m ) 개있고이들은수직방향에있는루크들의지배를받아야한다. 이제 ABF E 에평행한모든층위에있는루크들의분포를살펴보자.

400 400 조합 유형 : L 상에있는루크들이없는층 ; 유형 의층은 n m 개가있다. 그들각각은적어도 n m 개의루크를포함해야한다.( 이층들과 L 과공통부분에는속하는 n 개의작은정육면체중정확히 n m 개는수직방향의루크들로부터지배를받아야한다.) 따라서유형 의모든층들은적어도 (n m )(n m ) 개의루크들을포함하고있다. 유형 : ABF E 에평행한나머지 m 개의층 ; m 이최소개수이므로각층에있는루크의개수는적어도 m 개이다. 따라서유형 의모든층은적어도 m m 개의루크를포함한다. 위를정리하면루크의갯수를 r 이라하면, r (n m )(n m )+m m m m 라하면, n m n m, m m 이므로 r (n m ) + m m (n m ) + m = n + (n m ) 이다. r은정수이므로 n이짝수이면, r의최소값은 n 보다크며 n이홀수이면, r의최소값은 n + 보 다크다. 이제이런경우가존재함을보이자. 아래의적힌숫자는몇층에있음을나타내는것이다. n이짝수 ( 예 : n =8) n이홀수 ( 예 : n =7) 위와같이 n이짝수인경우는반으로나누어채우고, n이홀수인경우는 n 과 n + 로나누어서채 우면된다. 따라서최소값은 n이짝수인경우는 n, n이홀수인경우는 n + 이다. } 86. p 는홀수인소수이다. f; ;:::;pg 의부분집합들중에 p 개의원소를갖고원소의총합이 p 로나눠지는것의개수를구하여라. ( 통신강좌 ) 풀이 ( 과기원기계과 96학번이원철 ) 집합 A의원소들의총합을 S(A) 로나타내자. 일단원소의개수 ³ p 가 p개인부분집합의갯수는개이다. p B = f; ;:::;pg; C = fp +;:::;pg 라하면, S(B) =S(C) 0(modp) 이다. 이제 B, C 를제외한 ³ p p 개의부분집합을다음의규칙 대로 p개씩나누기로한다. 즉두부분집합 A와 A 0 이다음두성질을만족할때같은그룹이라고한다. ² A 0 \ C = A \ C이고, ² A 0 \ B = fx + m j x A \ B, x + m pg[fx + m p j x A \ B, p<x+ mg (0 <m<p인적당한 m에대해 ) n = ja \ Bj라하면, S(A 0 ) S(A) mn (mod p) 이고, mn은 p와서로소이므로, 하나의그룹에는반드시 S(A) 0(modp) 를만족하는집합하나가있다. 즉, 답은 µ ³p +이다. } p p 87. n 4 은짝수이고 A 는 f; ;:::;ng 의한부분집합이다. x ;x ;x 3 A 이고 e ;e ;e f ; 0; g 인 e x + e x + e 3 x 3 꼴의수들을생각하자. 단, e = e = e 3 = 일수없고, x i = x j 인두항이있다면 e i e j 6= 인경우만생각한다. 이런어떠한합도 n 의배수가될수없을때, A 를 free 하다고말하자. (a) 원소의개수가 b n c 4 인 free한집합을하나찾아라. (b) 더많은원소를갖는 free 한집합은없음을증명하여라. ( 불가리아 997 봄 y9-3)

401 4. 조합고급문제 40 풀이 (a) A = f; 3; 5;:::;b n 4 c g 이한예이다. e i 중에 0인것이없거나개인경우에는 e x + e x +e 3 x 3 이홀수가되므로n의배수일수없고, 따라서 x x 의경우만보면되는데,0<x +x <n 임과 jx x j < n, x x 6=0임에서역시불가능. (b) A가 free한집합이라하자. 우선 n과 n (=: m) 은 A의원소가아니다. 또한, k와 n k는 A에동시에포함될수없고, k A 이면 k 앞에적용될 e i 를늘 e i 로바꾸는것을생각하면 k 대신 n k A 로해도 A가 free임에변함이없다. 따라서, A ½f; ; 3;:::; m g =: B 일때만보면충분하다. u := b m c 이라하고, A가 u +개의원소 x <x < <x u+ 을갖는다고하자. x <x < <x u+ 과 x + x <x + x < <x + x u+ 등총u +개의수를생각하면, 이들은구간 [x ;x + x u+ ] 에속하고이구간에포함되는자연수의개수는 x u+ + m u + 이므로앞의수들중에같은것이있다. 크기관계에따라 x j = x + x i 일수밖에없고, 따라서, A는 free가아니다. } (b) 별해위풀이에서처럼 A ½f; ; 3;:::; m g =: B 일때만보기로하자. A 이면 = A 이고, f3; 4g; f5; 6g; 6½ A 이므로 jaj b m c 성립. 따라서,= A 일때만보면된다. m A 이면 = A 이고, f3;m g; f4;m 3g; 6½ A 이므로역시 jaj b m c 성립. 이에착안하여, 이제 a 또는 m a가 A의원소가되는최소의 a를잡자 (a < m ). (i) a A 일때 : A의원소들을 a로나눈나머지에따라묶자. 한묶음a+r; a+r; 3a+r;:::( m a) 에서연속한두항이동시에 A에속할수는없으므로이묶음에서A에속하는것의개수는최대 b (m a) (a+r) c+ = b m r a a c bm a c. 묶음은 r =0;:::;a 의 a개있으므로 jaj ab m a c m c 이다. (ii) a= A 이고 m a A 일때 : 역시 a = A 이고, fa+;m (a+)g; fa+;m (a+)g; 6½ A 이다. 즉,a+;:::;m a 의 m 3a 개의수중에서최대 b m 3a c 개, 그리고 a+;:::;a 과 m a 등 a개를추가하면 jaj b m a c b m c 임을역시알수있다. 따라서, 어느경우에나성립함이증명되었다. } 88. X 는 n+ 개의원소를갖는집합이다 (n ). X 의서로다른 n 개의원소들로된순서쌍 (a ;a ;:::;a n ) 과그런또다른순서쌍 (b ;b ;:::;b n ) 에대해, a i = b j 인첨자 i 6= j 가존재하면이두순서쌍이분리되었다고말한다. 서로서로분리된순서쌍들을최대몇개까지택할수있는가? ( 불가리아 차 -6) 풀이구하는개수를 an+ 이라하자. 한원소xX 에대해x를첫항으로갖는순서쌍들은그첫항을제외한나머지 n 개의항들중에서서로위치가다른항들이있어야하므로 ( 또그n 개의항은 x를제외한 n개의원소중에서택한것들이므로 ) n 일때의상황과같다. 즉, 각각의 x에대해많아야 a n 개씩만가능하므로 a n+ (n +)a n. 따라서, a n =n! a 3 =3! =. 실제로 a n = n! 이가능하다는것은, 순서쌍에나타나지않는마지막원소를순서쌍의맨끝에추가로덧붙였을때짝순열이되는순서쌍만모두모으면조건을만족함을확인하면된다. 이런두순서쌍이분리되지않았다면, 두순서쌍이완전히같지는않으므로 a i 6= b i 인 i가존재하고, 분리되지않았으므로 a i = b j 인 b j 가있을수없어서a i 는새로추가된항b n+ 과같다. 마찬가지로 b i = a n+ 이다. 따라서, a i 6= b i 인 i는딱하나뿐이고, 그럼두순서쌍은 i번째항과 n +번째항이서로뒤바뀌었을뿐이므로순열의홀짝성이다르다. } 89. 좌표평면의제 사분면을격자칸들로분할하였다. 이칸들을검은색과흰색으로칠하는데, 다음을만족하도록할수있는가? (i) 원점을한꼭지점으로하고좌표축에접하는임의의격자정사각형내부에는항상검은칸이더많다. (ii) y = x 에평행한모든대각선은각각검은칸을유한개씩만지난다. ( 이탈리아 997-3) 풀이다. 가능하다. 직선 y = x a 마다 x-축또는y-축으로부터처음 a +개의칸을검은색으로칠한 } 90. 평면위에열세개의점이있는데, 이중에서어느다섯점을택해도그중어느네점이한원위에있다고한다. (a) 이열세개의점중어느여섯점은한원위에있음을증명하여라. (b) 더강력한결과가성립하는것은아닐까? ( 플란더즈 997-4)

402 40 조합 풀이 ( 신한솔 )3개의점중임의의 9개의점 a ;a ;:::;a 9 을택하자. 이아홉개의점중 5점을지나는원이존재함을보이겠다. 이런원이존재하지않는다고하면임의의 5개의점을택하면 4개를지나는원이딱하나만있다.4점을지나는원을 n개라하고 x ;x ;:::;x n 으로나타내자. x 이 a ;a ;a 3 ;a 4 를지난다고하면 9 5 를계산했을때 (a5 ;a ;a ;a 3 ;a 4 ), (a 6 ;a ;a ;a 3 ;a 4 ),..., (a 9 ;a ;a ;a 3 ;a 4 ) 을택했을때x 이총5번세어지므로 9 5 =5n 9 그런데 5 는 5의배수가아니므로모순. 따라서임의의 9개의점을택해도 5개를지나는원이존재하므로 3개의점중다섯개를지나는원이존재한다. 그점을A; B; C; D; E라하자. 이원위에있지않는점이개이상존재한다고가정하고그두점을 X; Y 라하면다섯개의점 A; B; C; X; Y 를택하면 4개의점이한원위에있는데 X; Y 는 A; B; C를지나는원위에있지않으므로A; B; C중 개와 X; Y 는한원위에있다. 일반성을잃지않고 A; B; X; Y 가한원위에있다고하자. 마찬가지로 B; C; D;X; Y 를택하면 B;C; D 중 개와 X; Y 는한원위에있는데 C; D는 B;X; Y 를지나는원위에있지않으므로 (A; B; X; Y 를지나는원과 A; B; C; D; E를지나는원의교점은 A; B이므로 ) B; C; X;Y 나 B; D;X; Y 는한원위에있을수없다. 따라서 C; D; X; Y 는한원위에있다. 마찬가지로 D; E; X; Y 도한원위에있다. 따라서 X; Y; C; D; E는한원위에있다. 그럼 X; Y 는 C; D; E를지나는원, 즉 A; B; C; D; E를지나는원위에있으므로가정과모순이다. 따라서 A; B; C; D; E를지나는원위에있지않은점은많아야한개이므로 개의점이한원위에있고나머지한점은이원위에있지않아도조건을만족하므로 개의점은한원위에있다. } 9. 어떤체스대회에 n명이참가하고있다. 각경기마다이기면점, 비기면 0.5점을받는다. 한라운드동안각참가자는다른모든참가자와정확히한번씩경기한다. 두라운드가지난후, 모든참가자마다제 라운드에서의점수합계와제 라운드에서의점수합계가다르고그차이가늘 n 이상이었다고한다. 그럼그두합계의차이는모든참가자에대해서다똑같이 n점씩임을증명하여라. (Towns 997봄 JA3) 증명 제 라운드의점수가더잘나온사람들의집합을 A, 덜나온사람들의집합을 B라하고, A, B의제i라운드의점수합계를각각 A i, B i 라하자 (i =; ). 문제의조건에의해 A A njaj, B B njbj ( ). 두식을합하면 (A +B ) (A +B ) n. A +B = A +B = n =n n, 즉 (A + B )+(A + B )=4n n 이므로, 둘을연립하면 A + B n n. 한편, A jaj, B jbj 이므로 A + B jaj +jbj jaj+jbj n n. 따라서, 등호가성립할때이고, 그렇다면 ( ) 도등호가성립해야하므로다똑같이두라운드의합계차이가 n점씩임. 증명 위와같이 A, B를정의하자. B와의대결은무시하고, A 안의사람들끼리의대결만생각했을때, 그점수합계는두라운드에서같으므로,라운드의점수가 라운드에서보다좋지않은사람 a A가존재한다. 그럼그 a는 라운드에서 n 이상더좋아진총점을 B와의대결에서벌어들였어야하므로, jbj n. 대칭적으로 jaj n. 그럼 jaj = jbj = n 일수밖에. 이제등호가성립할때임에서등호조건을좀따져보면원하는결론이금방나옴. 9. 다음성질을만족하는 f; ;:::; 996 g 의부분집합 A 가존재함을보여라. (a) A 이고 996 A. (b) 을제외한 A의나머지원소들은 A의두원소 ( 같을수도있음 ) 의합으로표시된다. (c) A의원소의개수는 0를넘지않는다. ( 통신강좌 ) 증명 f(n) 을 A ½f; ; ; ng이면서조건 a), b) 를만족하는 A의원소의개수의최소값이라하자. f( 996 ) 0를보이면된다. f(n) 은다음두가지성질을갖는다. ) f( n+ ) f( n ) + 만약 A ½f; ;:::; n g이 a), b) 를만족하고 f( n ) 개의원소를가지면 B = A [f n+ Ã ; n+ g은 f; ;:::; n+ g의부분집합이고 a), b) 를만족한다!. * n+ =( n ) + ( n ) 이고 (* n A ½ B) n+ =+( n+ ) 이므로 ) f( n+ ) jbj = f( n ) + ) f( n ) f( n ) + (n +) 만약 A ½f; ; ; n g이 a), b) 를만족하고 f( n ) 개의원소를가지면 B = A [f( n ); ( n ); ; n ( n ); n g은 f; ; 3; ; n g의부분집 µ 합이고 a), b) 를만족한다. * j+ ( n ) = j ( n ) + j ( n ) (j =0; ; ;n ) n = n ( n ) + ( n ) (* n A ½ B) ) f( n ) jbj = f( n ) + (n +)

403 4. 조합고급문제 403 ), ) 에의해 f( 996 ) f( 998 ) f( 998 ) f( 499 ) f( 499 ) f( 498 ) + f( 498 ) f( 49 ) + 50 f( 49 ) f( 48 ) + f( 48 ) f( 4 ) + 5 f( 4 ) f( 6 ) + 63 f( 6 ) f( 3 ) + 3 f( 3 ) f( 30 ) + f( 30 ) f( 5 ) + 6 f( 5 ) f( 4 ) + f( 4 ) f( 7 ) + 8 f( 7 ) f( 6 ) + f( 6 ) f( 3 ) + 4 f( 3 ) 5 ) 모두더하면 f( 996 ) 0: ( 증명끝 ) 별증 ( 경남과학고박현정 )3j( n ) 를이용한다. 996 = 3 (996 ) + 3 (996 ) 3 (996 ) = 3 (998 )( 998 +) = 3 (998 )f 499 +( 449 +)g = 3 (998 ) (998 )( 449 +) f 996 ; 3 (996 ); 3 (996 ); 3 (998 )( 499 +); 3 (998 ) 449 ; 3 (998 )g 라하면, 3 (998 ) 499 과 3 (998 ) 을제외하고는모두조건을만족한다. 3 (998 ) 499 를두원소씩의합으로표현하기위해 3 (3998 ) 499 ; 3 (998 ) 498 ; ; 3 (998 ) ; 3 (998 ) 을생각해보면, 인접한두항은앞의항이뒤의항의 배이므로같은두원소의합으로생각할수있다. 이제 3 (998 ) 을생각하자.5 A 라해보면 ( 994 -) ( 994 -) ( 994 -) 귀납적으로 998 4k 로줄일수있다 =, = 이므로 = 까지줄인다고하고 = 5 + 6, 6 = 8, A = f; ; 4; 5; 8; 6g[ ` f 3 (4k+ )jk =; ; ; 48;`=0; ; ; 3; 4g[ f k 998 jk =; ; ; 499g[ 3 f( 499 +) 998 ; 3 3 (998 )( 998 +); 3 (996 ); 996 g 라하면 n(a) = = 749 이므로문제의조건에맞는다.

404 404 조합 93. 수열 (a n ) 은 n 에대하여 a n = a n + a [n=] 로정의되고 a 은임의로주어진정수이다. 이수열은 7 의배수인항을무한히많이포함함을증명하여라. ( 폴란드 차 -4) 증명 mod 7 로 0 인항이있으면 (a N 0 이면 ) 그후 a; a; a 꼴의연속한세항이나타나고 (a N a N a N+ 이되고 ), mod 7 로같은연속한세항이있으면 (a M a M+ a M+ 이면 ) 그후 mod 7 로등차수열인연속한일곱항 (a M ;:::;a M+5 ) 이나타나므로그중 0 이반드시있다. 94. 다음과같은등차수열 x 0 ;x ;:::;x M 이존재하는실수수열 a <a < <a M 을길이 M 의약한등차수열이라고부른다. x 0 a <x a <x a M <x M () a <a <a 3 이면 (a ;a ;a 3 ) 은항상약한등차수열임을증명하여라. () f0; ; ;:::;999g에서 730개이상의원소를골라만든임의의부분집합에는길이 0의약한등차수열이반드시포함됨을증명하여라. ( 아일랜드 000-4) 증명길이 0 의약한등차수열이없는부분집합을생각하자.(x0 ;x ;x ;:::;x 00 )=(0; 00; 00;:::;000) 으로한다면, 길이 00 짜리구간중적어도하나는통째로없어야한다. 남은 9 개의길이 00 짜리구간들을각각 0 등분해서생각하면또각각길이 0 짜리구간이적어도하나씩통째로없어야한다. 계속하면, 남은 9 개의길이 0 짜리구간들각각에서또각각수하나씩은없어야하고, 그럼총 = 8 개의원소는제거되어야한다. 별증연속한 개의정수중에서는적어도 개는지워져야한다. 000 = 이므로, 이것만으로 90 + = 8개의원소가제거된다. 즉, 이방법을더개선하면 (8개중5개는지워져야한다거나 ) 또다른풀이가가능할지도모른다. 95. 자연수 n 을 의거듭제곱들의합으로나타내는데지수중에같은것은최대두번씩만있을때그것을 n 의멋진표현이라고말하기로하자. 단, 합하는순서만다른것은같은표현으로본다. () 0의다섯가지멋진표현을적어라. () 짝수개의멋진표현을갖는자연수를모두구하여라. ( 중미 000-6) 풀이 () 은 8+=8++=4+4+=4+4++=4++++ 이고,() 만풀도록하자. 구하는자연수들은, 5, 8,,... 등 3n 꼴들임을추측할수있다. 귀납법으로증명하자. 3n 이홀수이면, 즉 6m 꼴이면 0 은항상딱하나이고, 그것을제거하면나머지항들을모두 로나눠줄수있어서 ((6m ) )= =3m 의멋진표현과일대일대응하므로,6m 의멋진표현도 3m 의멋진표현과마찬가지로짝수개이다.3n 이짝수이면, 즉 6m +꼴이면 0 은 0개혹은 개이다.0개인경우는남은항들을로나눠주면3m +의멋진표현들과대응되므로홀수개. 개인경우는그것을제거하고남은항들을 로나눠주면 3m의멋진표현들과대응되므로역시홀수개. 둘을합하면짝수개이므로역시성립한다.3n, 3n +의멋진표현이항상홀수개라는것도비슷하게말할수있다. 즉,6m은3m 및 3m 과대응되므로홀 + 짝 = 홀수개.6m +3은 3m +과대응되므로역시홀수개. 또,6m +은 3m과대응되므로홀수개. 6m 는 3m 및 3m 와대응되므로역시짝 + 홀 = 홀수개. } 96. 수열 0; ;:::;n 의순열 (a 0 ;a ;:::;a n) 이주어져있다. a i =0(i>0) 이고 a i +=a j 일때 a i 와 a j 를교환하는것을준법적이라고한다. 그리고, 준법적인교환만을반복하여 (; ;:::;n;0) 이되는순열 (a 0 ;a ;:::;a n) 을정규적이라고한다. 어떤 n 에대해순열 (;n;n ;:::;3; ; 0) 이정규적인가? (APMO 000-5) 풀이 ( 과기원 00학번최수영 ) 먼저 n =일때는0번의법정호환에의해서정규순열이된다. 이제 n = 3일때를살펴보자. 법정호환을계속반복했을때더이상법정호환을하지못하는경우는 0앞에 n이왔을때이다. 처음의법정호환을시작했을때 0과 3이바뀌고, 다시 0은 5와바뀐다. 이런식으로반복해나가다보면 0은모든홀수와호환이되면서결국 n 또는 n 둘중의하나와호환이되어야함을알수있다. 만약 n 과호환이된다면 0앞에 n이있으므로더이상의법정호환은불가능하다. 역시이는정규순열이되지못하기때문에 n은반드시홀수임을알수있다. 이제이순열의변한모습을살펴보면 (, 0, n ; ; 4, 5,, 3) 형태임을알수있다. 맨아의과 0을 b 0 로두고두개씩연속으로묶여있는값들을각각b ; b ; ; bn 로두자.

405 4. 조합고급문제 405 이다음번의법정호환에서 0은 와바뀌어야한다. 그리고는 b n 5 의맨앞에있는수와호환이되고차 례대로 b n k에있는수와호환이됨을알수있다. 따라서이역시b ; b 둘중의한수와호환이되어야하는데, 만일 b 의맨앞에있는수와호환이된다면그앞의수는역시n이되어서더이상의법정호환은불가능하고이역시정규순열이되지못하므로 n 는홀수이어야한다. 역시반복해서법정호 환을해주면결국제일처음의호환이되었던 b n k의모든수는다호환이되어서bn k+에차 례대로연결되어나간다. 같은방식으로계속법정호환을계속하면언제나이순열은, (,, ; k ; 0; (n k +); (n k +); ; n; ; k ; k +; ; k + k ) 꼴로변한다는것을알수있다. 이는귀납적으로증명이가능하다. 따라서이순열이정규순열이되기위해서는 n = k + k = k+ 이어야한다. ) n =; k (k N) } 97. n 은자연수이다.Nancy 에게는각칸마다자연수가하나씩적힌직사각형판이주어져있다. 그녀는다음의두가지움직임중한가지를취할수있다. () 한가로행을골라그각칸에 n 을곱한다. () 한세로열을골라그각칸에서 n 을뺀다. 다음진술이참이되게하는모든 n 의값을찾아라 : ( 캐나다 00-4) 임의로주어지는어떤직사각형판에대해서도,Nancy 가위와같은시행을유한번하여판의모든칸의수를 0 이되도록만들수있다. 풀이구하는 n은 뿐이다. n =일때는 () 이의미가없어지므로 () 만생각하면되고, 그럼어느세로열에서로다른두수가있으면그둘이같아질수가없어불가능함을알수있다. n 3 일때는모든칸의수가mod n 로 0이될수없음을대신보이자. na a (mod n ) 이므로 mod n 로생각하면 () 은아무작용도하지않는것이된다. 그리고,() 는각칸에서 을빼는것과같다. 즉 n = 일때와거의똑같은상황이되고, 그때처럼어느세로열에 mod n 로서로다른두수가있으면그둘이같아질수없어불가능함을알수있다. 이제 n =일때는가능함을보이자. 모든수가양의짝수라고가정해도된다 ( 아니면모든가로행에 를곱해그렇게만든다 ). 먼저첫번째세로열 C 을택한다. 이제이세로열에대해다음과같은작업 P (C ) 을반복하여행한다 : P (C): 세로열 C에서 가있는칸들을찾아 ( 그런칸이있다면 ) 그칸이있는가로행들에 를곱해준다. 그리고나서 C의모든칸에서를뺀다. P (C ) 을한번행하면C에서 였던칸은그대로 를유지하고,보다컸던칸은 씩감소한다. 그리고, C 이외의세로열의수는어쨌든계속양의짝수들이다. 따라서, P (C ) 를반복하면 C 의모든수들이 가될때까지감소하여언젠가 로만이루어진세로열이된다. 그때C 전체에서 를빼어C 전체를 0으로만든다. 다음두번째세로열 C 에대해또P (C ) 를반복하여 C 도모두로만든후또모두 0으로만든다. 그동안 C 에는영향이가지않는다. 이렇게 C 3 ;C 4 ;::: 에도계속같은식으로하면모든세로열을모두 0으로만들수있다. } 98. 원주위에 n 개의점이주어져있고, 이점들을잇는 n(n ) 개의현이그어져있다. 어느세현도원의내부의한점에서만나지않을때, 이현들에의하여나누어진원의내부의영역의개수를구하여라. ( 한국 00 차 -S5, 원래잘알려진문제 ) 풀이 다. 주장직선 l 개가교점 p 개를가지고있을때, 평면이분할되는개수는 l + p + 개 증거직선에개수에대하여수학적귀납법을적용하겠다. 직선에개수가없다면교점도없을것이고당연히평면의개수는 개이다. 이제직선의개수가정해진수 l까지위의법칙이성립한다라고하자. 그리고 l개의직선과다른한직선이새로이그어졌고, 그직선으로부터 s개의교점이더생겼다고하면, 직선은 s + l개로나누어진것이고, 평면은 s + l개만큼더생긴것이된다. 이는주어진공식을잘만족시켜준다. 즉 l +개의직선이있더라도, 주어진식은성립한다. }

406 406 조합 n(n ) 그렇다면문제를풀기위해서필요한것은개의직선중에서교점이몇개나있는가를구하는 일만이남았다. 그런데, 원주상에 4개의점을지나는 6개의현들중에서의교점은단하나가있게된다. 그리고, 어떠한세현도한직선위에있지않으므로, 전체교점의수는 nc 4 가된다. 결국문제의답은 nc + n C 4 + 이다. } 주이문제는 Problem-Solving Strategies/Athtur Engel/Springer에나오는것이다. 위의풀이는그문제에대한해답을편역한것으로, 조용의수학페이지 에서옮겨온것이다. 풀이 평면에서의오일러식 v e + f = 을이용하자. () 우선, 점의개수부터구하도록하자. v =( 원주위의점의수 ) +( 원내부의대각선의교점으로이루어진점의수 ) ³ n = n + 4 설명하자면, 원주위의점의수가 n 개인것은자명한사실이고, 원내부의대각선교점수는원주위의 4 개의점을선택할때마다하나의교점이만들어진다. 즉원주위의점중 4 개를뽑는경우의수와원내부의대각선의교점으로만들어진점수는일대일대응이다. () 이제선의개수를구하도록하자. e =(n각형의현의수 )+( 원호의수 ) +( 대각선의교점으로만들어진점들로인해늘어난선의수 ) ³ n ³ n = + n + 4 왜냐하면, n 각형의현의수는 n 임이자명하고원호의수도 n 개인것이자명하다. 결국대각선의교점으로만들어진점의수로결정되는데이는대각선의교점으로이루어진점이하나결정될때 개의직선이더생기게되므로 n 4 가되게된다. 그러므로구하고자하는면의수, 즉 f 는 + n + n 4 가된다. } 99. P P :::P n 은짝수개의변을갖는볼록다각형이다. 이다각형의어떤변과도평행하지않은대각선 P i P j 가존재함을증명하여라. ( 오폴 00-) 증명변의개수는 n개. 각변과평행할수있는대각선의수는많아야 n 개이므로, 평행한변이있는대각선들의최대수는 n(n ) = n 4n 개이다. 그런데, 대각선의개수는 (n)(n 3) =n 3n 개로이최대수보다더많다. 따라서, 어떤변과도평행하지않은대각선이반드시존재한다 명의사람들의집합 P 가주어져있다. P 에서임의의 00 명의사람들을모아서로아는두사람의쌍을모두세면그수는항상일정한상수 ( 단,0 은아닌 ) 가된다고한다.( 임의의두사람은서로알거나서로모른다.) 집합 P 에서의서로아는두사람의쌍의최소개수를찾아라. ( 오폴 00-9) 풀이 () 한명이00명이상을모를수는없음을귀류법으로보이자. 어떤사람 A와서로모르는사람이적어도 00명이있어서그들을 A ;A ;:::;A 00 이라하자. A ;A ;:::;A 000 의 000명중서로아는두사람의쌍이x개있다면, A; A ;A ;:::;A 000 의 00명중서로아는두사람의쌍의개수역시 x개이다 (A와 000명모두는서로모르므로 ). 따라서, A ;A ;:::;A 00 의 00명중에서서로아는두사람의쌍의개수역시 x개로일정해야하고, 즉 A 00 은 A ;:::;A 000 을모두몰라야한다. 같은방법으로, A ;A ;:::;A 000 ;A 00 의 00명모두가서로알지못한다는것을보일수있다. 즉, 00명의사람중아는두사람의쌍을세었을때의일정한상수가 0이되어버리고, 집합 P 의모든 00명의사람들은서로알지못한다. 이것은문제의조건에모순.

407 4. 조합고급문제 407 () 위의 () 에의해, 한사람이아는사람의수는 00 명이상이어야한다. 그런데 () 과같은방법으로한명이 00 명이상의사람을알때그 00 명은모두서로알고있음을보일수있다.( 임의의두사람은서로알거나모르므로 00 명의사람중서로아는두사람의쌍의개수가항상일정하면마찬가지로모르는두사람의쌍의개수역시항상일정하다.) 즉임의의 00 명의사람들은모두서로를알고있으며집합 P 의모든사람들이서로를알고있다. 구하는개수는 = 00300( 쌍 ). } 0. 칠판에 3 개의음이아닌정수가적혀있다. 이정수들중에서두수 k, m 을선택하여 k + m 과 jk mj 으로치환된다. 세번째정수는바뀌지않고남아있게된다. 이렇게구한새로운세개의정수로부터같은방법으로계속시행한다. 목표는세정수중어느둘을 0 으로만드는것이다. 이것이가능한지어떤지설명하여라. ( 폴란드 00 3 차 -3) 풀이 (KAIST 과학영재센터연구원고봉균 ) 처음세정수가어떤것이든항상 개를 0으로만드는것이가능하다. 이것을증명하겠다. 우선세정수가 (da; db; dc) 꼴이면, 두수의합과차로바꿔넣어도d가일정하게쫓아다니므로 (a; b; c) 꼴로간주해도된다. 이것을 (da; db; dc)» (a; b; c) 와같이나타내기로하자. 또, (x; y; z)! (x; y + z; y z)! (x; y; z) ( ) 이므로 ( 단, y z), 세수중어느두수를 배로만드는것이가능하다. 또,( ) 에의해 (x; y; z)! (x; y; z)! (x; y; z)» (x; y; z) ( ) 이므로, 세수중어느짝수를절반으로만드는것이가능하다. 이제다음사실들을확인하자. (i) 세정수중0이아닌짝수가있으면세정수의합을줄일수있다. (ii) 세정수중홀수가둘있으면세정수의합을줄일수있다. (i) 는 ( ) 에 x >0 이면 x + y + z>x+ y + z 이므로바로확인된다. 또, 두홀수의합과차는모두짝수이므로 (ii) 도 ( ) 을이용하면다음과같이확인된다. µ (x; y; z)! (x; y + z; y z)! x; y + z ;y z µ! x; y + z ; y z 여기서 x + y + z>x+ y+z + y z. 음이아닌세정수중 0인것이많아야개뿐이라면, 거기에는짝수가있거나아니면 개의홀수가있다. 따라서, (i), (ii) 에의해세정수의합이줄어든다. 음이아닌세정수의합도음이아닌정수이고, 음이아닌정수는무한번줄어들수없다. 따라서, 언젠가는음이아닌세정수중 0인것이개이상나타나게된다. } 0. m, n 은 보다큰정수들이다. 가로 m 세로 n 개의직사각형꼴격자점배열을생각하자. 이들중 k 개의점을빨강으로칠하는데, 빨강점들중어느세점도두변이격자방향과평행한직각삼각형의꼭지점을이루지않도록한다. k 의가능한최대값을구하여라. ( 영국 00/003 차 -4) 풀이 m 혹은 n이 일때에는모든점을빨강으로칠할수있음이자명하다. m; n 일때k의최대값이 m + n 임을 m + n 에대한수학적귀납법으로보이자. 어느한점 X를잡고, X를포함하는가로줄과세로줄의 X를제외한모든격자점을빨강으로칠하면직각삼각형이안생기므로 k m + n 임을알수있고, 이제반대방향인 k m + n 만보이면되겠다. 먼저 m + n =4, 즉 m = n =일때에는 점이최대임을쉽게알수있다. 이제 m + n<r(r 5) 일때모두성립함을가정하고, m + n = r 일때를증명해보자. 임의의한가로줄 L을택하고, 그 m개의격자점중빨강점이 l개라고하자. 만일 l = m 이면더이상다른곳에빨강점을찍을수없으므로됐고, l = m 이면 L에서유일하게빨강점이아닌점 X를포함하는세로줄에만빨강점이더있을수있으므로역시 k m + n 를만족한다. l m 일때는 L에서빨강점이아닌 m l 개의점들을각각포함하는세로줄 H ;:::;H m l 을보자. 빨강점은이제이들세로줄에만더있을수있다. 그럼나머지세로줄과 L을제외한이들부분만보면 ( 일반성을잃지않고 m n 이라하면 n 3 이되므로 ) m 0 ;n 0 ; m 0 + n 0 =(m l)+(n ) <r 의 m 0 n 0 격자점배열이되므로, 수학적귀납법의가정에의해이영역에는 (m l)+(n ) 개이하의빨강점이있다. 즉, 처음 L 위의 l 개의빨강점과합하면모두 m + n 3 개이하의빨강점이되어역시성립함을알수있다. 따라서, 수학적귀납법에의해증명이되었다. }

408 408 조합 장의카드에각각 ; ;:::;5 라고라벨을붙인다. 이카드들의순열 ¼ : f; ;:::;5g!f; ;:::;5g 가다음조건을만족시킬때, 이를 shu²e 이라고부른다. 조건 : 적당한 m 5 이있어서, i<5 가 m 일때를제외하면항상 ¼(i) <¼(i +) 이성립한다. 다음을증명또는반증하여라 : 카드가어떤적당한순서로정해져있으면, 이로부터 5 번의 shu²e 을통해모든다른카드순열들을얻을수있다. ( 폴란드 00/003 차 -0) 풀이 (KAIST 과학영재센터연구원고봉균 ) 셔플이어떤것인지의미를파악해보자. 순서가정해진 5 장의카드가있으면그것을 A 와 B 두그룹으로분리하는데같은그룹에들어가는카드는분리하기전의순서를유지한다. 그후 A 그룹의카드들뒤에 B 그룹의카드들을통째로이어붙이는것이셔플이다. 이때, m 이 A 그룹의카드개수가된다. (a ;a ;a 3 ;:::;a 5 ). 순서를유지하면서둘로분리 & (a j ;a j ;:::;a jm ):A 그룹 B 그룹 :(a k ;a k ;:::;a k5 m ) & 그대로연결. (a j ;a j ;:::;a jm ;a k ;a k ;:::;a k5 m ) 위와같이 ¼(i) <¼(i +) 이아닐수있는경우는 a jm ;a k 의곳뿐이다. 5 장의카드 a ;a ;:::;a 5 중에서 a i >a i+ 이되는위치 i 를 descent 라고부르기로하자. 그리고, 주어진카드의배열에 n 곳의 descent 가있다고하자. 한번의셔플을행하면,A 그룹과 B 그룹각각의 descent 는최대 n 곳이다 ( 원래카드의순서를이어받기때문 ). 그리고 A 그룹과 B 그룹을연결하는지점에서추가로 descent 가발생할수있으므로, 셔플이완료된후의카드배열에는최대 n+ 곳의 descent 가있을수있다. 카드가어떤적당한순서로정해져있어도그카드들에순서대로이름을다시부여하는것을생각한다면, 카드는원래부터 ; ;:::;5 의순서로정해져있다고해도일반성을잃지않는다. 이때,5 번의셔플을행하면 descent 는최대 0!! 3! 7! 5! 3 곳이생길수있다. 그런데순서가완전히뒤바뀐순열 5; 5;:::; 은 5 곳의 descent 가있으므로이순열은얻을수없다. 따라서,5 번의셔플로모든다른카드순열들을얻을수는없다. } 04. f; ;:::;003g 에서 N 개의수를고르는데, 어느두수도차가 0 이되지않도록한다. 이렇게수를고르는방법이 N = 003 일때는얼마나되는지구하고, N =00 이면 ( ) 0 7 가지가됨을보여라. ( 아일랜드 003-0) 풀이 ( 대원외국어고 3학년강성환 )0으로나눈나머지가같은원소들끼리묶으면, 나머지가,, 3인집합은각각0개의원소를갖고그이외의집합은각각 00개의원소를갖는다. 문제의조건을만족하려면이각집합들안에서연속한두원소를뽑지않으면되므로, 나머지가,, 3인집합에서는각각최대0개, 그이외의집합은각각최대00개를뽑을수있다. 즉, N의최대값은 003. () 이렇게 N =003의최대로뽑는경우의수는, 나머지가,, 3인집합에서는 OXOX XO 와같이뽑아야하므로 (O가뽑는수들의위치 ) 방법이유일하고, 그이외의집합에서는 OXOX XO 에서 X 하나를추가시킬수있는위치가 0곳이므로방법은 0가지, 즉모두0 7 가지가된다. () N =00일때에는소집합들에서각각최대로뽑고한집합만한개부족하게뽑으면된다. (i) 나머지가,, 3인집합중한곳에서한개덜뽑을때 :0개짜리집합에서 00개를조건을만족하며뽑는경우의수는,OXOX XO 에서 X 둘을추가시키면되므로, 다른두곳에 X 하나씩추가시키는경우 0 가지와한곳에 X 둘을한꺼번에추가시키는경우 0가지가있다. 그럼모든경우의수는 ³ µ 3 ³ = (ii) 그외집합중한곳에서한개덜뽑을때 :00개짜리집합에서 99개를조건을만족하며뽑는경우의수는,OXOX XO 에서 X 셋을추가시키면되므로, 서로다른세곳에 X 하나씩추가시키는경우 00 3 가지, 한곳에는X 둘을추가하고다른한곳에는 X 하나를추가하는경

409 4. 조합고급문제 409 우 00 99가지, 한곳에X 셋을모두한꺼번에추가하는경우 00가지등이있다. 따라서, 모든경우의수는 ³ µ 7 ³ = 그러므로모든경우의수는 ( ) 0 7 가지이다. } 05. 원주위에서로다른 n 개의점이놓여있다. 이점들중임의의한점에서시작하여그점과그점으로부터시계반대방향으로 m 번째점을선분으로연결하고, 이 m 번째점과이점으로부터시계반대방향으로 m 번째점을선분으로연결하고, ::: 이러한과정을새로운선분이생기지않을때까지되풀이하자. 이렇게그려진선분들의교점중, 원의내부에있는것들의개수를 I 라할때, 다음에답하여라. 단, m, n 은서로소인자연수로서 6 m <n 을만족한다. (a) 원주위에놓인서로다른 n개의점의위치가변할때, I가취할수있는최대값을m, n의식으로나타내어라. (b) 부등식 I n 이항상성립함을보여라. (c) m =3이고 n이위의조건을만족시키는임의의짝수일때 I = n 인경우가존재함을보여라. ( 한국 003 최종-6) 풀이 (a) m과 n은서로소이므로n개의선분이그려지면서원주를 m번돌고처음시작한점으로돌아온다. 이렇게원안에선분을그릴때, 어느세선분도한점에서만나지않을때 I가최대값을갖는다. 이때각선분에는, 열호 ( 그현에의해잘리는두호중작은쪽의호 ) 안에있는 m 개의점으로부터시작된 개씩의선분에의해 (m ) 개의교점을갖는다. 각교점은두선분에의해세어지므로 I max = n (m ) = n(m ) (b) I n을증명하자. 원위의각점P i 에선분이 개씩연결되는데, 각점에서원의중심을봤을때왼쪽선분위의첫번째교점을Q i 라하자 (i =; ;:::;n). 이때, P i 6= P j 이면 Q i 6= Q j 이다 ( ). 따라서, P i 7! Q i 의대응이일대일 ( 단사 ) 함수가되어 ) I n. ( ) 의증명이필요하면다음과같이할수있다 : 만약 P i 6= P j 일때, Q i = Q j 라고가정하자. 그럼 Q i = Q j 는 P i 와 P j 의왼쪽선분들이만나는교점이다. 이때, P j 의오른쪽선분이 P i 의왼쪽선분과만난다면 Q i 가첫번째교점이라는가정에모순이므로, P j 의오른쪽선분의한쪽끝은P i 와 P j 사이에있다. 그러면 P j 의오른쪽선분에의한열호가P i 의왼쪽선분에의한열호에완전히포함되어열호위의점의개수가일정하다는데에모순이생긴다. (c) n이 n =q의짝수일때 I = n 인경우를만들어보자. 정q각형 A의각변에중점을잡아 A에내접하는정q각형 B을다시만든다. 그리고, A와 B의각변을연장하여가장가까운곳에맺히는 q개의점을택한다. 그럼이 q 개의점을지나는원과그점들이우리가찾는그경우에해당한다. 그리고, 두정 q 각형 A, B 의꼭지점이 I 의점들이다. } 06. 어떤전산실의컴퓨터들이다음과같이네트워크를이루고있다. 각각의컴퓨터는세개의케이블을통하여세대의다른컴퓨터와직접연결되어있고, 임의의두컴퓨터는직접또는 ( 다른컴퓨터들을거쳐 ) 간접적으로연결되어서로데이터를주고받을수있다. 이제이컴퓨터들중 K 대를제거하여서로데이터를주고받을수없는두대의컴퓨터가존재하거나한대의컴퓨터만남도록하는 K 의최소값을 k 라하고, 한편케이블중 L 개를제거하여서로데이터를주고받을수없는두대의컴퓨터가존재하도록하는 L 의최소값을 l 이라고하자. 이때, k = l 임을보여라. ( 한국 003 최종 -)

410 40 조합 증명 먼저 k l 임을확인할수있다. l개의케이블을제거하는대신그케이블에연결된컴퓨터들을하나씩제거하면충분하기때문이다. 그리고, 문제를각컴퓨터에연결된케이블이꼭 3개씩일때가아니라 3개이하일때로확장하여풀자. k의값에대한수학적귀납법으로풀어보자. 이미 k l 이므로 k l 만보이면된다. 조건에서 k, l이모두자연수이므로수학적귀납법을쓸수있다. (i) k =일때 ; k =이므로두컴퓨터집단이오직한컴퓨터로만연결이될수있다. 이형태는다음과같이 가지경우로만될수있고, 대칭적으로는같은꼴이다. 처음경우는좌편의케이블을두번째경우는우편의케이블을끊으면되므로 l, 즉 l =이다. 가운데컴퓨터에연결된케이블이 3개보다작을때에도마찬가지. (ii) k = n일때성립한다고가정. 즉, k = l = n. 이제 k = n +일때를살펴보자. 임의의 k = n + 인네트워크 N에대해, n개의컴퓨터를잘골라제거한네트워크 N 0 은컴퓨터하나만더끊으면전체연결이해체되므로 k =이다. 그럼 (i) 에의해N 0 은 l =이기도하고, 그하나의케이블을 C 0 이라하자. 제거하는순서를거꾸로생각하면, 원래의 N에서 C 0 를제거한네트워크 N 00 은앞으로 n개의컴퓨터를제거하면해체되므로 k = n 이다. 귀납법의가정에의해 N 00 은 l = n 이기도하고, 그럼이 n개의케이블과 C 0 를제거하여 N이해체되므로 N에서 l n +, 즉 l = n +. (i), (ii) 에의해문제의주어진명제는참이다. 증명 앞선풀이에서처럼 k l 임을아는것으로하고, 각컴퓨터에연결된케이블이 3개이하일때로확장하여풀자. 그리고, 어느한컴퓨터에연결된세대의컴퓨터를제거하거나, 그세케이블을제거하는것을생각하면 k; l 3 인것도쉽게알수있다. 이제 k의값에따라경우를나누자. () k =이면 ; 앞선풀이에서처럼 l =임을알수있다. () k =이면 ; l 임을알고있으므로 l 만보이면 l =가확인된다. k(n) =인임의의네트워크 N에서하나의컴퓨터 (B 라하자 ) 를지운N 0 은 k(n 0 )=, 즉 () 에의해l(N 0 )=이다. 즉, N 0 은하나의케이블 (C 이라하자 ) 을제거하면해체된다. N에서케이블 C 을제거한 N 00 은컴퓨터 B 를제거하면해체되므로 k(n 00 )=, 즉 () 에의해l(N 00 )=이다. 즉, N 00 은하나의케이블 C 를제거하면해체된다. 따라서, N은두케이블 C, C 의제거로해체되므로 l(n), 즉 l =. (3) k =3이면 ;3=k l 3 이므로 l =3은당연. (){(3) 에의해항상k = l 임이증명되었다. 주항상 k = l =3 인것으로추측하고이것을풀려고시도할수도있지만, 그렇지않은경우도많다. 예로, 엇갈린대각선을한쌍씩그린 개의오각형의두꼭지점을이은그래프를생각해보아라 개의문자 A, B, C, D, E, F, G, H, I, J 에서중복을허용하여고른길이 n 의문자열을생각하자. 이렇게만든임의의문자열에서각문자를 0; ;:::;9 의숫자로하나씩대응하여적당히바꾸어주면최고자리가 0 이아니고 9 의배수인 n 자리의수를항상얻을수있음을보여라. ( 호주 003-8) 증명 (Kalva 싸이트,ErikPantapaa) 9의배수라는것은자리수의합이9의배수라는것과같다. 그리고, 최고자리가 0일때에는0인문자와 9인문자를바꾸어주면그만이므로그것때문에조심할필요는없다.0개의문자각각의개수를 x ;x ;:::;x 0 이라하자. 우리는 P b i x i 0(mod9) 를만족하는 0; ;:::;8; 0 의순열b ;b ;:::;b 0 을찾으면된다.

411 4. 조합고급문제 4 벡터표현을빌려 X =(x ;x ;:::;x 0 ) 으로두고 B 0 =(0; 0; ; ; 3; 4; 5; 6; 7; 8) B =(0; ; ; 3; 4; 5; 6; 7; 8; 0) B =(0; ; 3; 4; 5; 6; 7; 8; 0; ) B 3 =(0; 3; 4; 5; 6; 7; 8; 0; ; ). B 8 =(0; 8; 0; ; ; 3; 4; 5; 6; 7) 이라하자. 그럼 mod 9 로 B k+ B k =(0; ; ;:::;) 이므로, B k+ X B k X = x + x x 0 (= d 로두자 ) 이된다. 따라서, B k X = B 0 X + kd 만일 d 가 9 와서로소이면, 즉 3 의배수가아니면, d 는 mod 9 로역수를가지고, 그럼 kd B 0 X (mod 9) 인 k 를고를수있다. 이때 B k X 0(mod9) 이므로원하는것을찾았다. 여기까지의이야기가 x 에만특별히적용되는것이아니므로, 모든 i 에대해 ( P x j ) x i 가 3 의배수인경우를제외하면늘위와같은방법으로해를찾을수있다. 그리고, 이예외적인경우에는모든 x i 들이 mod 3 으로같고, 그럼 x i mod 9 의값은 3 가지만있다 ( 예를들면,,4,7). x i 들을이세값에따라세집합으로나누자. 세집합의원소의합이 0 으로짝수이므로홀수개의원소를갖는집합이 0 개혹은 개이다. 만일 개의집합의크기가홀수이면두집합에서각각한원소씩을골라 b i =0 으로두자. 그럼세집합모두짝수개의원소가있는것으로간주해도되겠다. 이제같은집합에속하는두원소마다 f; 8g; f; 7g; f3; 6g; f4; 5g; f0; 0g 들을 b i 로대응시키면, 각쌍이 9 의배수가된다 ( 마지막쌍 f0; 0g 은이미사용했을수도있다 ). 이것으로 P bi x i 0(mod9) 인 b i 들을이때도찾았다. 08. 주어진두양의정수 m 과 n 에대해, 다음을만족하는가장작은양의정수 k 를찾아라 : 임의의 k 명의사람들중에는, 서로아는사람끼리둘씩짝을짓게할수있는 m 명이있거나혹은서로모르는사람끼리둘씩짝을짓게할수있는 n 명이있다. (APMO 003-5) 풀이 (KAIST 과학영재센터연구원고봉균 ) 그런 k 를 K(m; n) 이라하자. K(m; n) =m + n +max(m; n) 임을보이자. 대칭성에의해 K(m; n) =K(n; m) 이므로, 일반성을잃지않고 m n 일때 K(m; n) =m + n 임을보이면되겠다. K(m; n) =m + n 라면다음과같은반례가있다 :m 명과 n 명의두그룹 U, V 로나누어서, U 의사람들끼리는모두서로모르고, V 의사람들끼리는모두서로알며, U 의사람과 V 의사람은늘서로모르는것으로하자. 그럼서로아는 m 쌍이있다면이들은모두 V 에속하는데 jv j < m 이어서모순이고, 서로모르는 n 쌍이있다면각쌍의어느한사람은 U 에속하는데 juj <n 이어서모순이된다. K(m; n) 이더작아도같은얘기를할수있으므로, K(m; n) m + n 이제 m + n 명일때문제의조건이만족된다는것 ( ) 을통해 임을확인하자. K(m; n) m + n

412 4 조합 U m 명 모두서로모른다 Ã! n 명 V 모두서로모른다 모두서로안다 () n =일때 ;m명중에서로모르는쌍이하나라도있으면조건이성립. 또한모두서로안다면서로아는m쌍이생기므로역시조건이성립한다. () m = n 일때 ; m = n 에대한수학적귀납법으로보이자. m = n =일때에는 () 에서성립. m + n 명중에서 A, B가서로알고A, C가서로모른다고하자. 만일이런 A를찾을수없다면, 모든사람은유명인 ( 모든사람과서로아는사람 ) 이거나고립인 ( 모든사람과서로모르는사람 ) 이다. 유명인과고립인은서로알수도서로모를수도없으므로함께존재할수없으므로, 모두가서로알거나모두가서로몰라야하고, 그럼문제의조건이성립된다. 이제 A, B, C 세사람을제외하면 (m ) + (n ) 명이되므로, 수학적귀납법의가정 K(m ;n ) = (m ) + (n ) 에의해이중에는서로아는 m 쌍이있거나서로모르는 n 쌍이있다. 그럼 (A,B) 쌍이나 (A,C) 쌍을추가하여서로아는 m쌍이나서로모르는 n쌍이있음을알수있다. (3) m n 일때 ; m에대한수학적귀납법을쓰자. m = n 일때에는 () 에서성립. m + n 명중에서서로아는한쌍 (A,B) 를잡자. 만일이런쌍이없다면모두서로모르는것이되고, 그럼문제의조건이쉽게성립한다.A,B두사람을제외하면 (m ) + n 명이되므로, 수학적귀납법의가정 K(m ;n)=(m ) + n 에의해이중에는서로아는 m 쌍이있거나서로모르는 n쌍이있다. 그럼 (A,B) 쌍을추가하여서로아는 m쌍이나서로모르는n쌍이있다. 따라서,( ) 이증명되었다. } 09. 길이가 인변과 3 인변만을각각짝수개씩갖는볼록다각형이있다. 이다각형의둘레의길이를이등분하는두꼭지점이존재함을증명하여라. ( 폴란드 003/004 차 -0) 증명각변의길이를a ;a ;:::;a n 이라하고, A i = a i + a i+ + + a i+n 이라하자 ( 즉, a i 로부터절반개수의연속한변들의길이의합, 단첨자는 mod n으로생각한다 ). 그럼 A i + A i+n = p 이므로 ( 둘레의길이 ), A i p 라하면A i+n p. i를 증가시킬때마다 A i의변화량은 이하이므로 (ja i+ A i j = ja i+n a i j ), A i ;A i+ ;:::;A i+n 은 A i 부터 A i+n 까지의모든정수가나타나고, 따라서 p 도나타날수밖에없다 ( 일종의중간값정리 ). 별증귀류법으로, 각꼭지점에대해그대응점 ( 짝을이뤄둘레를이등분하는점 ) 은어떤변의내부에있다. 그런데길이 3인변에서내부의두점이모두대응점일수없으므로각변마다많아야한점씩만대응점이될수있음. 그럼대응점후보의개수는변의개수, 즉꼭지점의개수와같으므로, 각변마다정확히하나씩대응점이있어야하고, 또꼭지점이돌아가는순서와대응점이놓인변이돌아가는순서가정확히맞아떨어짐 ( ). 꼭지점들을 A ;A ;:::;A n 으로쓰자 ( 첨자는 modulo로생각 ). 변 A i A i+ 위의대응점을 M i 라하자. 꼭지점 A i 의대응점이 M i+a 라하자.( ) 에의해a는상수. 그럼 A i+a 의대응점은 A i 또는 A i 의위치에있으므로 M i 일수밖에없다. 즉, i (i + a)+a (mod n) 으로홀짝성에모순. 0. 다음조건을만족시키는자연수 m 의최소값은얼마인가? ( 한국 004 차 -S9) 모든사람이각각 m 명이상을알고있는 30 명의사람중에는모두서로아는 4 명의그룹이항상있다.( 단,A 가 B 를알면 B 도 A 를알고있다.) 풀이최소값은 명이다. 우선 명일때조건이성립함을보이자.30명의사람들을 a ;:::;a 30 이라하고, a 과 a 가알고있다고하자. a 과 a 는각각0명의사람들을더알고있어야하므로, 적어도 (30 ) = 명은 a 과 a 가동시에알고있다. 이중한명을 a 3 이라하면, a 과 a 는 a 3 이외에적어도 명을함께알고있고, a 3 은 a 과 a 이외에 9 명을더알고있다. 이두인원의합 30 명이 a, a, a 3 을제외한인원

413 4. 조합고급문제 43 7명을넘으므로, a, a, a 3 이함께아는사람이또 ( 적어도 3명 ) 있다. 그중한명을a 4 라하면a, a, a 3, a 4 가조건을만족하는 4명의그룹이다. 0명이하일때는조건이성립하지않을수있다.30명을 0명씩 3개의집단 X, X, X 3 으로나누고, 같은집단의 명은서로모두모르고다른집단의 명은서로모두아는것으로하자. 임의로 4명을택하면이중에같은집단에속하는 명이있고그둘은서로모른다. 따라서, 이경우조건이성립하지않음을알수있다. } 별해전체인원을 30명대신n명으로, 모두가서로아는그룹의인원 4명을 k명으로일반화하자. 그리고, 조건을만족하는최소의 m을 m(n; k) 라하자. m명일때조건이성립한다고해보자. 한명A가아는인원은m(n) 명. 서로아는두명 A, B가동시에아는인원은 jx A \ X B j = jx A j + jx B j jx A [ X B j m n( 명 ) 이상이다. 비슷하게, 서로아는세명 A, B, C 가동시에아는인원은 jx A \ X B \ X C j = jx A \ X B j + jx C j j(x A \ X B ) [ X C j 3m n( 명 ) 이상임을알수있다. 일반적으로 k 명이동시에아는인원은 (k )m (k )n 명이상임을말할수있다. 따라서,(k )m (k )n >0 이면, 즉 ¹ º k m k n + 이면, 모두서로아는 k명의그룹이항상있다. 이값이m(n; k) 임을보이자. 전체인원을최대한균등하게 k 개의집단으로나누자. 그리고, 같은집단에속하는사람끼리는항상서로모르고다른집단에속하는사람끼리는항상서로아는것으로하자. 그럼 k명을뽑으면그중에같은집단에속하는 명이있어, k명이모두서로알수는없게된다. 한사람이아는인원은전체에서자기집단의인원을빼면되므로, 최소» ¼ ¹ n n = n + n º ¹ º k = k k k n 로, 이경우원하던 m(n; k) 의값과같음을확인할수있다. }. k 와 N 은 k N 을만족시키는자연수라하자. i k 에대하여다음성질을만족시키는 f; ;:::;Ng 의부분집합 A i 가주어져있다. 성질 : 공집합이아닌임의의부분집합 fi ;i ;:::;i s g½f; ;:::;kg 에대하여다음을만족시킨다. A i 4 A i 4 4A is 6=? 이때, ja j 4 A j 4 4A jt j k 를만족시키는부분집합 fj ;j ;:::;j t g½f; ;:::;kg 가존재함을보여라.( 단, 집합 A, B 에대하여 A 4 B =(A [ B) (A \ B) 이다.) ( 한국 004-4) 증명 ( 서울대수학과 03학번서인석 ) 중등부의문제 번에서사용했던 I 의정의와다음의성질들을그대로이용하기로하자. () 4은교환법칙과결합법칙이성립한다 ( 따라서위와같이쓸수있다 ). () A 4 B =? () A = B. (3) I 는 fa i g ii 들중홀수개에포함되는원소들만을모은집합이다. (4) J 4 K = J4K. 그리고, 이문제에주어진조건에따라다음을또확인해두자. (5) 각각의 I ½f; ;:::;Ng = U 에대해 I 는모두서로다르다.

414 44 조합 (5) 의증명임의의 I 6= J (I;J ½ U) 에대해,() 에의해K = I 4 J 6=? 이고, 그럼 (4) 와문제의성질에의해 I 4 J = K 6=? 따라서 () 의의해 I 6= J 이다. I(6=?) ½ U 에대해문제의조건에의해 I (6=?) ½ U 이고 (5) 에의해이것은일대일로대응하므로, (6) 임의의 J(6=?) ½ U 에대해 I = J 인 I(6=?) ½ U 가존재한다. 즉다음과같이정의하면 j k j = k 이다. k = f J j J ½f; ;:::;kgg 이제 k 를상수로보고 N 에대한수학적귀납법으로원명제를증명하자. (i) N = k 이면 (6) 에의해 U k 이므로성립. (ii) N = m 일때성립함을가정하고 N = m + 일때를보자. fg; fg;:::;fng k 이라고가정하면, U = fg4 4fNg k 이므로성립.( 사실은이때 k = U f?g 이되므로이런경우는생기지않는다.) 따라서, 일반성을잃지않고, fng = k 인경우만보면된다. 이때, 각각의 A ;:::;A N 에서 N 를제거하여얻은 A 0 ;:::;A 0 N (A 0 i = A i fng) 를생각하자. A 0 ;:::;A0 N ½f;:::;N g 이고, A0 ;:::;A0 k 는문제의성질을만족하므로, 귀납법의가정에의해N = m 일때이A 0 i 들에의해서도명제가성립한다. 즉, j 0 Ij k 인 I ½f;:::;kg 가존재한다. 그럼 I = 0 I or 0 I [fng 이므로 j Ij k 로 N = m +일때도성립한다. (i), (ii) 의수학적귀납법에의해모든 N k 에대해서문제가성립한다.. 여러가지색의구슬로목걸이를만들려고한다. 같은패턴의구슬들의주기적인사슬로구성되지않은목걸이를특별하다고말한다. n과 q는자연수이다. 각각이 q n 가지색중하나인 n개의구슬로된특별한목걸이의개수는, 각각이 q가지색중하나인 n 개의구슬로된특별한목걸이의개수의 n배와같음을증명하여라. [ 주 ] 적당히회전시키거나뒤집었을때똑같은모양이되는것은같은목걸이로본다. ( 중미 004-6) 증명회전하거나뒤집어서같은것을같은것으로보지않기로했을때의 a가지색에서고른 b개의구슬로된주기적이지않은목걸이의개수를 f a(b) 라하자. 회전해서같은 b개씩묶이므로, 문제에서요구하는것은 f q n(n)=n = n f q(n )=n, 즉 f q n(n) =f q(n ) 임을보이라는것이다. 각각의색을 0;:::;q n 의정수로보고, 이것을 q진법으로생각하자. 그럼 f q n(n) 은 q진법으로 n자리의수가 n개있는것이고, f q(n ) 은 q진법으로한자리의수가 n 개있는것이므로자릿수의개수가일치하여같은수를나타내는것으로볼수있다 ( 일대일대응 ). 주기가있느냐없느냐하는것도일치한다는것만확인하면된다. 길이 n 의순환열에주기 T 가있다면그주기는 n 의순약수. 그럼그주기에자연수배를하여여전히 n 보다작은약수이지만 n의배수가되도록할수있다 (T 와 n의각소인수의지수를비교하여둘중큰값을새주기t 0 의지수로하면됨 ). 별증뫼비우스공식을이용한김규완 xmo 풀이참조 의모든양의약수들의집합을 T 라하자. S 는 T 의부분집합인데, S 의원소중어떤것도 S 의다른원소의정수배가아니다. S 의원소의개수의최대값은얼마인가? ( 캐나다 004-5) 풀이 004 = 3 67 이다.,3,67대신 p, q, r로하자. 일반적으로 p n q n r n 의모든양의약수들의집합을 T n 이라할때, 문제의조건을만족하는부분집합 S의원소의개수의최대값이 t n =(n +) 임을보이자. 우선 S = f p a q b r c T n j a + b + c =ng 으로두어보자. 그럼 S의서로다른두원소는한쪽지수가크면다른쪽지수가작은것이있게되어서로약수배수가될수없음이분명하다. 또한, 이 S의원소의개수는, b; c를 0 b; c n 의범위에서선택하면 a =n (b + c) 로저절로선택되고 T 의원소임에도차질이없으므로 (n +) 개이다. 따라서, t n (n +). 한편, 만일 S의원소의개수가 (n +) 개보다많다면,0 b; c n 인서로다른 (b; c) 쌍은 (n +) 개뿐이므로, 비둘기집의원리에의해 S의원소들중 (b; c) 가같은것이적어도한쌍있다. 그럼이두수중a가더큰것이a가작은것의정수배가되어모순. 따라서, t n (n +), 즉구하는최대값은정확히 (n +) 답 }

415 4. 조합고급문제 어떤대회에 n 명의선수가참가한다 (n 3). 모든두선수는서로정확히한번의경기를치르며, 무승부는없는것으로한다. 서로물고물린세명의선수를비긴삼인조라고부른다. 이대회에서비긴삼인조는최대몇조나구성될수있는가? ( 폴란드 차 -3) 풀이안비긴삼인조에는 승거둔사람이꼭명존재. 즉안비긴삼인조의개수 x는 A가 B와 C를이긴짝 (A,B,C) 의개수가같음. 이것을최소로. i번째선수가이긴횟수를 w i 라하면 P w i = n 이고, x = P w i 임. 이것은 wi 들이모두비슷할때최소. 그담은홀짝나눠서계산노동. } 격자판에서경미와남훈이가어떤게임을한다. 각선수는자기차례가되면아직격자판에나타나지않은유리수를하나골라격자판의남은빈칸중하나에그수를쓴다. 모든칸에수가채워졌으면각각의행에서가장큰수가쓰여진칸을검게칠한다. 그래서판의꼭대기에서밑바닥까지검은칸만을지나는 ( 꼭지점은지나도된다 ) 곡선을그릴수있으면경미가이기고, 그렇지못하면남훈이가이긴다. 누구에게필승의전략이있는지찾고, 그것을증명하여라. ( 미국 004-4) 풀이 ( 대전과학고 학년김두리 ) 격자판맨위의두줄만생각하자. A B C D E F d e f a b c 위와같이윗줄과아랫줄의칸을세로가 3 차이가나는같은칸끼리쌍으로보자. 쌍이되는두칸을같은알파벳의대문자와소문자로표기하였다. 경미가이중어느칸에수를쓰면, 남훈이는그와쌍이되는칸에수를쓰는데, 윗줄의알파벳들의크기순서와아랫줄의알파벳들의크기순서가계속일치하도록할수있다. 그럼예를들어결과적으로윗줄에서 B 가제일커서검은칸이된다면아랫줄에서는 b 가검은칸이되고, 이렇게검은칸의위치가 3 만큼떨어져있으므로곡선으로넘어갈수없다. 따라서, 남훈이가항상이긴다. } 별해 (O±cial Solution) 다음과같이기호를부여하고같은알파벳의대문자하나와소문자하나씩을짝지어보자. A A A a a a B B B b b b C C c c c C D d d d D D e e e E E E f f f F F F 경미가어느칸에수를쓰면, 남훈이는그와짝이되는칸에수를쓰는데, 짝이되는두수중에서대문자인칸이항상더크도록한다. 그럼검은칸은위의배열중에서대문자인칸에만있을수있고, 좌상의대문자영역에서우하의대문자영역까지곡선으로옮겨갈방법이없으므로, 남훈이가항상이긴다. } 6. 평면위에어느세점도한직선위에있지않은 004 개의점들의집합 S 가주어져있다. 이집합의임의의두점에의해결정되는직선들의집합을 L 이라하자. 개이하의색으로 S 의점들을칠하는데, S 의임의의점 p 와 q 에대해, p 와 q 가같은색이라는것과 p 와 q 를가로지르는 L 의직선이홀수개라는것이동치가되도록칠할수있음을보여라. 주어떤직선 `이두점p와 q를가로지른다는것은이두점이 ` 밖에있으며 `에의해나뉘어지는서로반대의영역에놓여있음을말한다. (APMO 004-3) 증명 ( 김병수 ( 수정됨 )) S의점의개수는004 대신임의의짝수로해도된다. 다음의보조정리를먼저확인하자. 보조정리 S의임의의세점 A, B, C에대해, BC, CA, AB를가로지르는 L의직선의개수를각각 X, Y, Z라하자. 그럼 X + Y + Z 는홀수이다.

416 46 조합 보조정리의증명세점에의해주어지는직선들로나뉘는영역을다음그림과같이표시해보자. 세변중한변이상을가로지르는직선은, ² 세꼭지점중한점을지나고그대변을가로지르거나 (L 직선 ), ² 어느두변을가로지르거나 (L 직선 ) 의둘중하나이다. L 직선은 X + Y + Z 에 만큼기여하므로 X + Y + Z 의홀짝에영향을미치지않는다. L 직선은 X + Y + Z 에 만큼기여하므로 X + Y + Z 의홀짝은 L 직선의개수로판단하면된다. S 에서 A, B, C 세점을제외하면홀수개의점이있고, 이들은위그림의영역 a, b, c, x 중한곳에있다. 영역 a 에있는점들은 A 와이어 BC 를가로지르는 L 직선을하나만들고, 영역 b 와영역 c 의점들도마찬가지이다. 영역 x 에있는점들은 A, B, C 등과함께 L 직선을 3 개씩만든다. 따라서, L 직선도홀수개이고, 즉 X + Y + Z 는홀수이다. 두점 p 와 q 를가로지르는직선이홀수개이면 p» q 로나타내기로하자. 위의보조정리에의해 A» B, B» C 이면 A» C 이므로 (X, Y, Z 중어느둘이홀수이면나머지하나도홀수 ),» 은동치관계로볼수있다. 또한, 위의보조정리의의해 A B, A C 이면 B» C 이므로 (X, Y, Z 중어느둘이짝수이면나머지하나는홀수 ),» 에의한동치류는많아야 개뿐이다. 따라서, 같은동치류에속하는점들은같은색으로칠하면된다. 7. 어느학교에서 6 명의학생들이다음과같이원칙에따라 ` 공부모임 ' 과 ` 봉사활동모임 ' 을만들고자한다. 각모임은반드시 3 명으로구성되며,6 명의학생중임의의두학생에대하여이두학생이함께속한공부모임의수와봉사활동모임의수는같다. 최소한한개의모임은있다고가정하고, 구성원 3명이모두같은공부모임과봉사활동모임은없다고할때, 공부모임의수와봉사활동모임의수의합의최소값을구하여라. ( 한국 005-J8/S8) 풀이 6 명의학생을 a, b, c, d, e, f 라하고, 공부모임들의집합과봉사활동모임들의집합을각각 P, Q 라하자. 같은공부모임에속한두학생의쌍을 ( 중복을허용하여 ) 모두모은집합을 P 0 이라하고, 마찬가지로 Q 0 도정의한다. 하나의모임에서두학생의쌍을택하는방법은항상 3 가지이므로, jp 0 j =3jP j, jq 0 j =3jQj 가된다. 그리고, 문제의조건으로부터 P 0 = Q 0 이고, 따라서 jp j = jqj 임을알수있다. 모임이최소한하나는있으므로, n = jp j = jqj 이다. 일반성을잃지않고 (abc) P 라하자. ab; bc; ca P 0 = Q 0 이므로, 일반성을잃지않고 abd; bcx; cay Q (x; y fd; e; fg) 인 x, y 가존재한다. 마찬가지로 abc; bdz; adw P (z;w fc; e; fg) 이다. 여기서, n 3 이다. n =3 이라가정해보자. 그럼위의모임이전부이다. bd 는 P 0 에한번밖에나타나지않으므로 Q 0 에서도그래야하고, 따라서 x 6= d 이다. 그럼 Q 0 에나타나는 cx 가 P 0 에는나타날수없으므로모순이다. 따라서, n 4 이고, n =4 일때는 P = fabc; bde; adf; cefg; Q = fabd; bce; caf; defg 이면문제의조건을잘만족한다. 따라서, 전체모임의수는 m =n 8 이고최솟값은 8 이다. }

417 4. 조합고급문제 합성수인자연수 n 에대해, n 의 보다큰모든약수들을한원의원주위에늘어놓는데, 이웃한두수끼리서로소가되는경우가없도록하려고한다. 이것이가능한 n 을모두구하여라. ( 미국 005-) 풀이 ( 대전과학고 학년김진현 ) 우선소인수가하나뿐인 p q 꼴의수는다된다. 서로다른두소수의곱 pq꼴의수는다불가능하다. 이제 pq꼴의수를빼고는다가능함을보이자. 우선소인수가 3개이상일때를보자. p a q b r c m꼴로쓸수있다 (p; q; r은소수, a; b; c; m ). 그림에서 A 구간에는남아있는 p 의배수를모두써넣자. 그리고 B 구간에는남아있는 q 의배수를모두써넣고, C 구간에는남은수 ( 모두 r 의배수이다 ) 를모두써넣으면된다. 마지막으로소인수가 개인경우에대해생각하자. p a q b 꼴이고 b 라할수있다. A 구간에는남은 p 의배수를모두써넣고, B 구간에는남은 q 의배수를모두써넣으면된다. } 9. 철희가어떤모임에나갔는데, 이모임에모두 63명이참석하고있다고한다. 임의의세참석자중에는, 서로모르는두명이반드시있다고한다. 만약임의의참석자 A와 B가서로모르면, A와 B를모두아는참석자가정확히두명이있다고한다. 이문제가성립할수없음을보여라. ( 한국 006 차-J6) 증명철희가아는사람의집합을 A, 모르는사람의집합을 B라하자. 삼각형 ( 서로아는세명 ) 이생기면안되므로 A의사람끼리는서로모른다. 서로모르는두명에게는그둘을아는사람이항상딱 명씩있으므로, A에서임의의두명 a, a 0 을택하면그둘을아는사람이철희외에B에딱한명이있다. 그사람을b라하면, 철희와 b는서로모르므로, 이둘을아는사람이A에딱두명있고그둘이처음에택했던 a와 a 0 이다. 따라서, A에서두명을고를때마다B의한명과일대일로대응하므로, jaj = jbj 이다. jaj = K, 즉철희가아는사람의수를 K명이라하면 ³ K =630 K 이다. 이것을풀면, K(K ) = 60 K, K + K 60 = 0, (K +36)(K 35) = 0. 따라서, K =35 이다. 그런데,63 명중에누구라도철희가될수있으므로, 각자가아는사람이항상 K =35 명씩있다는것이된다. 각자가아는사람의수를모두더하면, 변 ( 서로아는두명 ) 의개수의 배가되므로 63K =e 라야한다. 그러나, K 가홀수이므로이것은짝수가될수없어모순. 따라서, 문제의조건은성립하지않는다 이상의각각의정수 n 에대하여, 모든항이 S = f0; ; ;:::;9g 의원소인수열 (c ;c ;:::;c n) 중에서다음성질을만족하는것의개수를구하여라 : 연속한세항을어떻게골라도그중서로같은두항이있다. ( 폴란드 006/007 차 -4) 풀이이런수열중에서, 마지막두항이서로같은것의개수를an, 마지막두항이서로다른것의개수를 b n 이라하자. 그럼 a n+ = a n + b n, b n+ =(n )a n +b n 의복점화식이성립. b n = a n+ a n 으로 b n 들을소거해주면 a n+ =a n+ +(n )a n. 이제열심히풀면될듯 ; }

418 48 조합

419 제 5 장 종합문제 5. 종합중급문제. 어떤직각삼각형의세변의길이가등차수열을이룬다고한다. 이세변의길이의비가 3:4:5 임을증명하여라. 증명세변의길이를n a, n, n + a라하자. 피타고라스정리에서 (n a) + n =(n + a). 정리하면 n =4an, 즉 n =4a. 따라서, 세변의길이는3a, 4a, 5a.. 임의로주어진실수 q 에대해 s n =+q + q + + q n S n =+ +q µ +q µ +q n 로정의할때, 다음을증명하여라. ( 헝가리 93-) ³ n + ³ n + ³ n + ³ n + + s + s + + s n = n S n 3 n + 증명 ( q)s n = q n+,( +q +q )Sn = ( )n+ 이므로, 증명하려는식의양변에 ( q) 를곱하면다음과같다. ³ m ³ m ³ m ( q)+ ( q )+ + ( q m m )=(+) m ( + q) m 단, m = n +. 이식은우변을이항정리로전개하면좌변과같음. 3. a, b, c 를고정된자연수라하자. 임의의자연수 n 에대해, 세변의길이가각각 a n, b n, c n 인삼각형이존재한다고한다. 이삼각형들은이등변삼각형임을증명하여라. ( 헝가리 94-) 증명부등변삼각형이라고가정하고,WLOGa>b>c라하자. 그럼 a n >b n >c n 이고, 삼각부등식 a n <b n + c n 을만족해야한다. 그런데, n!일때( b a )n! 0, ( c a )n! 0 이므로언젠가우변이좌변보다작아진다. 모순. 4. 공간에서두원이접한다는것은두원이만나는점에서의두원의접선이일치함을뜻한다. 공간상의세원이서로다른세점에서둘씩접하면, 세원이모두한평면위에있거나모두한구면위에있음을증명하여라. ( 헝가리 937-)

420 40 종합문제 증명두원이접할때한평면위에있지않으면중심법선의교점으로부터두원의모든점이같은거리에있음을확인하면끝. 5. m n r 크기의나무토막이있다. 겉표면을모두칠한후이나무토막을 mnr개의단위정육면체들로잘랐더니, 어떤면에도색칠이되지않은정육면체는그들중딱절반이었다. 이런성질을갖는서로다른크기의나무토막은유한개뿐임을증명하여라. (Putnam 95-A6) 증명 WLOG m n r 이라하자. r =; 일때는모든블럭에색이묻고, r =3; 4 일때는최상층과최하층블럭들에다가 ( 이것만벌써절반 ) 옆면에드러난블럭몇개가더색이묻으니조건에맞지않다. 즉, r 5. 문제의조건은 (m )(n )(r ) = mnr 이고, 양변을 mnr로나눠주면 L := ( m )( n )( r )=. r 0 이면 L ( 4 5 )3 > 이므로해가없음. 즉,5 r 9. 이각각의 r에대해m := ( m )( n )=c r 5 6 인적당한상수 c r에대해해를구하는것이되는데, n 4 이면 M ( ) > 5 6 c r 로해가없음. 즉, 각각의 r에대해n도유한개씩이고, n이정해지면 m은바로결정되므로 (m; n) 도유한개. 6. 볼록다각형 P 가단위정사각형의내부에들어있다. P 의각변의길이의제곱의합이 4를넘지않음을보여라. (Putnam 966-B) 증명 Form a right-angled triangle on each side of the polygon (and outside it), by taking the other two sides parallel to the sides of the square. The sum of the squares of the polygon's sides equals the sum of the squares of the non-hypoteneuse sides of the triangles. Because the polygon is convex, these triangle sides form 4 sets, one for each side of the square, and each set having lengths totalling less than (the side of the square). So the sum of the squares in each set is less than ( x < ( x) = ). 7. x + x +=py 를만족하는자연수 x, y 가존재하는소수 p 가무한히많음을보여라. ( 소련 968-4) 증명 p j x + x + 인 x 가존재하는 p 를찾는것이된다. 즉, a(n) = n + n + 이라할때, a();a();a(3);::: 들의소인수를모두모은것이무한집합임을보이라는것이다. 귀류법으로, 유한하다고가정하고그소인수들을 p ;p ;:::;p k 라하자. 그럼 a(p p :::p k )=(p :::p k ) +(p :::p k )+ 은 p ;:::;p k 의어느것으로나누어도 이남으므로어느것의배수도아니다. 즉, 이것은기존에없는소인수들 ( 만 ) 을갖는다. 8. 모든각의크기가서로같고모든변의길이가유리수인팔각형은대칭중심을가짐을증명하여라. ( 폴란드 차 -5) 증명한변에평행한직선에사영을내리면 A + B p =0 꼴의식을얻고무리수의상등에서 A = B =0. 특히 A =0 에서마주보는변의길이가서로같음을얻음. 그럼마주보는두변에의해항상평행사변형이결정되고, 그럼모든주대각선의중점이같은점임이전염적으로확인됨. 9. 삼각형 ABC 의세각 A, B, C 에대응하는변의길이를각각 a, b, c 라하자. a( cosa) +b( cosb)+c( cosc) =0 이면 ABC 는정삼각형임을증명하여라. ( 헝가리 969-) P 증명코싸인제법칙대입하고 abc 곱하여정리하면준식은 a 4 P a b + abc(a + b + c) = abc(a + b + c) 6S =(a + b + c)( P a 3 P a b +3abc) =(a + b + c)( a+b c (a b) + a b+c (c a) + a+b+c (b c) )=0이므로 a = b = c. 0. 정사각형꼴의나무판자를 n 개의똑같은정사각형조각으로잘랐다. 이조각들을다시조립해네개의직사각형으로만들었는데, 한조각은사용하지않고남겨두었다. 이다섯사각형들의아홉가지치수 ( 네직사각형의가로와세로, 그리고남은한조각의한변의길이 ) 가모두다르다고한다. 이런상황이가능한 n 의최소값을구하여라. 그리고, 그때의각사각형들의치수를말하여라. 이런상황이가능한다른 n 의값들을모두말하고, 그것이전부임을입증하여라. ( 영국 969-3)

421 5. 종합중급문제 4 풀이다섯사각형의변의길이가,, 3, 4, 5, 6, 7, 8, 9일때가최소. 그최소값은+( 9) + (3 8) + (4 7) + (5 6) = 0 (.. 뒤에증명 ). n 0 이므로 n = 일때최소이고실제로 +( 9)+(3 8)+(4 7)+(5 6) = 로구성하면됨. n 일때항상가능한데, 다음과같이하면됨 : +( (9+k))+(3 8)+(4 7)+(5 6) = k+0 또는 +( (9+k))+(3 7)+(4 8)+(5 6) = k+0. () 의증명은다음과같음... ( 혹은고중대에있는보조정리문제처럼해도됨...) b c +b c + +b nc n = [(b + c ) +(b + c ) + +(b n + c n) (a + + a n )] 이므로 (b + c ) + +(b n + c n) 의최소값을찾는것과같다. 코시부등식에서 n[(b + c ) + +(b n + c n ) ] (a + + a n ) 이고, 등호는 b + c = = b n + c n 일때성립하므로 (a ;a n ), (a ;a n ), :::; (a n ;a n+ ) 로짝지어야한다. }. 정사각형운동장의한가운데늑대가한마리있고, 네귀퉁이에개가한마리씩있다. 늑대는자유로이움직일수있고, 개는운동장둘레를따라서만움직일수있다. 개의속도는늑대의.5 배이다. 개한마리로는늑대를이길수없고, 개두마리가모이면늑대를죽일수있다. 개들이늑대가도망가지못하게할수있음을보여라. ( 소련 969-) 증명정사각형을 ABCD라하고, 늑대의위치를 P 라할때, P 에서두대각선에평행한직선을그려변과만나는점들을 A 0, B 0, C 0, D 0 이라하면개들은이 A 0, B 0, C 0, D 0 의위치에계속있도록하며움직이면됨. 속도는늑대의 p 배이상이면충분함.. 둘레가 0 km 인원형도로와 km 의곧은도로로이루어진어떤버스노선이있다. 곧은도로의한쪽끝은종점이고반대쪽끝 Q 가원형도로와만나연결된다. 이노선에는두버스가달리는데, 한번돌아올동안 0 분이걸린다고한다. 번버스는종점에서출발하여곧은도로를달린후원형도로를시계방향으로한바퀴돌고다시곧은도로를통해종점으로들어온다. 번버스는 번버스가출발한 0 분후에종점을떠나고 번버스와같은방식으로노선을달리는데, 단원형도로를시계반대방향으로달린다고한다. 두대의버스는손님들을싣고내리는무시할수있는짧은시간을제외하고는이노선을쉬지않고계속달린다. 한남자가 번버스의노선을따라종점으로부터 xkm (0 x<) 떨어진위치 P 에서기다리다가두대의버스중한대를이용하여종점까지가려고한다. 그는자신을목적지에가장빠른시각에데려다줄수있는버스를탄다고할때, 그의여정 ( 기다리는것과버스로이동하는것을합한 ) 에걸릴수있는최대시간을 w(x) 라하자. () w() 와 w(4) 의값을구하여라. () x가어떤값일때시간w(x) 가가장길어질까? (3) 0 x< 에대해y = w(x) 의그래프를그려라. ( 캐나다 974-7) 풀이 (3) 번을풀면 () 과 () 의답도자연스럽게구할수있을테니 (3) 번만풀자. 두버스의배차간격이같고두노선이대칭적이므로 w(x) =w( x) 가될것이다. 따라서,0 x 6 에서만풀어보자. 기다리는시간을포함해걸릴수있는최대시간을찾는것이므로, 막버스를놓친시점에대해서만조사하면된다. 그렇지않고버스가지나간지좀된뒤라면그경우보다더오래기다린경우도있을테니최대시간이될수없다. 우선곧은도로위인 0 x 일때는, 번버스를구분할필요없이 0 분에한대씩들어온다. 따라서, 막버스를놓친후 0 분을기다려서다음버스를타고 xkm 를달리면되므로 w(x) =0+ 0 x =0+5 x (0 <x ) 3 이다. 이제 <x 6 일때를생각하는데, 두종류의버스를구분해야하므로, x 지점에서막 m 번버스를놓친후다음에도착하는 n 번버스를이용할때걸리는시간 w mn(x) 를구해보자. w (x) 를먼저생각해보자. 번버스를막놓쳤으면다음 번버스가올때까지 0 분을기다린후그버스를타고 xkm 를달려야한다. w (x) = ( x) 비슷하게, w (x) = x

422 4 종합문제 다음 w (x) 를생각해보자.번버스를 x지점을통과할때 번버스는그번버스가 0분후지나는위치인 x +6지점과대칭이되는위치, 즉 6 x지점을지난다. 따라서, x<3 이면 번버스가지나간후 번버스가바로도착하고, x 3 이면종점에들어갔다가다시나오는 번버스를기다려야한다.번버스를놓친직후번버스를기다려목적지까지가는시간은 번버스를놓친직후부터 번버스가목적지까지걸리는시간이라고생각하면된다. 즉, >< (6 x) (x<3) >< (6 x) (x<3) w (x) = 3 >: 또, w (x) = 3 3 (6 x) (x 3) 5 >: (6 x) (x 3) 3 막버스를놓친시점은 번버스를놓친시점과 번버스를놓친시점의 가지가있고, 두경우중더오래걸리는경우를구하면된다. 각버스를놓친시점에서는다음에도착하는 번버스를탈수도있고 번버스를탈수도있는데, 가장빠른시각에데려다줄수있는버스를타므로둘중빨리도착하는경우를구하면된다. 따라서, w(x) = max(min(w (x);w (x)); min(w (x);w (x))) 가되고, 앞에서구한 w mn (x) 들로부터이것을구해보면 min(w (x);w (x)) = w (x); min(w (x);w (x)) = x<3 에서는 w <w, x 3 에서는 w w 이므로, 종합하면 x (0 <x ) >< 3 w(x) = w (x) = x ( <x<3) 이다. 그래프는다음과같다. >: w (x) =0+ 5 (6 x) (3 x 6) 3 ( w (x) (x<3) w (x) (x 3) } 3. 임의의양의실수 r에대하여, 중심이원점이고반지름이 r인원에서원주에가장가까운격자점까지의거리를 d(r) 로나타내자. lim d(r) =0임을보여라. ( 헝가리 973-) r! 증명 x +y = r 의원이직선y = brc 과만나는 사분면의점을 A(a; q brc), 그리고그오른쪽의가장가까운격자점을 B(dae; brc), OB와원의교점을C라하자. d(r) BC brc +( p r brc +) r = p r brc + p p r+! 0( brc + +r r 분자유리화 ) 4. S n 은한변의길이가 n 인정사각형이다. S ;S ;::: 들을겹치지않게한변의길이가 k인정사각형안에놓으려고한다. 가능한최소의 k를구하여라. ( 헝가리 974-) 풀이답은.5. k<:5 일때는S 과 S 를함께놓을수없으므로곤란하고, 그럼 k =:5 일때가능하다는것만확인하면됨 < 이런식으로 k m< k+ 이면 S m 대신 S k 을적당히잘넣을수있음. }

423 5. 종합중급문제 x 0 =5,x n+ = x n + x n 로정의된수열에서 45 <x 000 < 45: 임을증명하여라. ( 헝가리 975-3) 증명 x n 은증가수열. 따라서, 모든항이양수. x n+ = x n ++ x >x n +. 따라서, x n > n x n +> >x 0 +n, 즉, x n > p n +5. 고로, x 000 > p = 45. 한편, n x n =n +5+ X x i=0 i 임에서 x 000 =05+( x + )+( 0 x + )+( 5 x + )+( 00 x x ) < =09< 034:0 = (45:) 6. x 3 + xy + y 3 =3 의그래프를그려라. ( 캐나다 978-6) 풀이 x + y = t, x y 변수분리하여정리하면 = u 로치환해보자. x = t + u, y = t u 이므로 (t + u) 3 +(t u )+(t u) 3 =3 t 3 + t +(6t )u =3 u = t3 t +3 6t 이된다. u 0, t +3t +3> 0 이므로 t 6t 0, 즉 = t 6t (t +3t +3) 6 <t 혹은 3 <x+ y 의범위에그래프가존재한다. 이범위에서 t 가증가함에따라분자 t 3 t +3 는감소, 분모 6t 은증가하므로 u 은 t 에대해감소함수이다. 그리고 juj! 이면 t! 6 + 이다. ( 3p 3; 0), (0; 3p 3), (; ) 등의절편을참고하면그래프는위와같다. } 7. 하나의단위원안에반지름의합이 979 이고서로만나지않는여러개의원을배치하는것이가능한가? ( 유고슬라비아 979 고 -3) 풀이한변의길이가 인정사각형이포함되므로, 반지름 4n 인원을정사각형꼴로 n 개배치할수있다. 그럼반지름의합은 n. n = 로하면끝. } 8. f;:::;ng 의공집합이아닌모든부분집합 fi ;:::;i k g 에대해서다음식을증명하여라. ( 오폴 980-4) X i i i k = n 증명양변에 더하고인수분해 ;

424 44 종합문제 9. x ;x ;:::;x n 과 a ;a ;:::;a n (a < <a n ) 이다음두식을만족하는자연수들일때, n 의가능한최대값을구하여라. ( 오폴 980-7) x x x n = 980; x i = a i (i =; ;:::;n ) x i 풀이 ( 박경택 ) a i 들이서로다르므로 x i 들도서로다르다 (i n ). 980 = 3 5 로소수,, 3, 3, 5, 이곱해져있다. x i =의경우가추가될수있고 와 3이각각중복되므로, 여기서 n의최대값은,, 3, 5, 6, 일때의6까지가능함을알수있다. 그럼최대인 n =6일때를살펴보자. x i (i =;:::;6) = ; ; 3; 5; 6;. 첫번째조건식은성립하므로두번째식만확인하면된다.(x i ;a i ) =(, 98 ), (, 99 ), ( 3, 663 ), ( 5, 40 ), ( 6, 336 ), (, 9 ) 에서, x =,x =6,x 3 =5,x 4 =3,x 5 =,x 6 =이되어야함을알수있다.(* a < <a 6 ) ( 두번째조건식성립 ) 따라서, n의최대값은 6이됨을알수있다. } 0. 삼각형 ABC 의높이 CH 와두변 CA, CB 는각각어떤정칠각형의한변과서로다른두대각선과같은길이이다. \ACB < 0 ± 임을증명하여라. ( 불가리아 98 4 차 -) 증명 ( 양준호 ) Lemma 한변의길이가 a 이고서로다른두대각선의길이가 b; c 인정칠각형에서 b + c = a 가성립한다. 증명 ACF G 에서톨레미의정리 ac + ab = bc =) a = bc c+b. ) a = b+c bc = b + c. \C = 라하자. S 4ABC = bc sin = CH AB = ap b + c bc cos b c sin = a (b + c bc cos ) 한편 Lemma에의해bc = a(b + c) 이므로 b c = a (b + c) 이다. (b + c) sin = b + c bc cos (b + c )(sin ) + bc(sin +cos ) =0,(sin )b +c(sin +cos )b +(sin )c =0 b가실수이므로 D 4 0 )c (sin +cos ) c (sin ) 0, c f(sin +cos ) (sin ) g 0

31. 을전개한식에서 의계수는? 를전개한식이 일 때, 의값은? 을전개했을때, 의계수와상수항의합을구하면? 을전개했을때, 의 계수는? 를전개했을때, 상수항을 구하여라. 37

31. 을전개한식에서 의계수는? 를전개한식이 일 때, 의값은? 을전개했을때, 의계수와상수항의합을구하면? 을전개했을때, 의 계수는? 를전개했을때, 상수항을 구하여라. 37 21. 다음식의값이유리수가되도록유리수 의값을 정하면? 1 4 2 5 3 26. 을전개하면상수항을 제외한각항의계수의총합이 이다. 이때, 의값은? 1 2 3 4 5 22. 일때, 의값은? 1 2 3 4 5 27. 를전개하여간단히 하였을때, 의계수는? 1 2 3 4 5 23. 를전개하여 간단히하였을때, 상수항은? 1 2 3 4 5 28. 두자연수 와 를 로나누면나머지가각각

More information

체의원소를계수로가지는다항식환 Theorem 0.1. ( 나눗셈알고리듬 (Division Algorithm)) F 가체일때 F [x] 의두다항식 f(x) = a 0 + a 1 x + + a n x n, a n 0 F 와 g(x) = b 0 + b 1 x + + b m x

체의원소를계수로가지는다항식환 Theorem 0.1. ( 나눗셈알고리듬 (Division Algorithm)) F 가체일때 F [x] 의두다항식 f(x) = a 0 + a 1 x + + a n x n, a n 0 F 와 g(x) = b 0 + b 1 x + + b m x 체의원소를계수로가지는다항식환 Theorem 0.1. ( 나눗셈알고리듬 (Division Algorithm)) F 가체일때 F [x] 의두다항식 f(x) = a 0 + a 1 x + + a n x n, a n 0 F 와 g(x) = b 0 + b 1 x + + b m x m, b m 0 F, m > 0 에대해 f(x) = g(x)q(x) + r(x) 을만족하는

More information

제 3강 역함수의 미분과 로피탈의 정리

제 3강 역함수의 미분과 로피탈의 정리 제 3 강역함수의미분과로피탈의정리 역함수의미분 : 두실수 a b 와폐구갂 [ ab, ] 에서 -이고연속인함수 f 가 ( a, b) 미분가능하다고가정하자. 만일 f '( ) 0 이면역함수 f 은실수 f( ) 에서미분가능하고 ( f )'( f ( )) 이다. f '( ) 에서 증명 : 폐구갂 [ ab, ] 에서 -이고연속인함수 f 는증가함수이거나감소함수이다 (

More information

3.2 함수의정의 Theorem 6 함수 f : X Y 와 Y W 인집합 W 에대하여 f : X W 는함수이다. Proof. f : X Y 가함수이므로 f X Y 이고, Y W 이므로 f X W 이므로 F0이만족된다. 함수의정의 F1, F2은 f : X Y 가함수이므로

3.2 함수의정의 Theorem 6 함수 f : X Y 와 Y W 인집합 W 에대하여 f : X W 는함수이다. Proof. f : X Y 가함수이므로 f X Y 이고, Y W 이므로 f X W 이므로 F0이만족된다. 함수의정의 F1, F2은 f : X Y 가함수이므로 3.2 함수의정의 Theorem 6 함수 f : X Y 와 Y W 인집합 W 에대하여 f : X W 는함수이다. Proof. f : X Y 가함수이므로 f X Y 이고, Y W 이므로 f X W 이므로 F0이만족된다. 함수의정의 F1, F2은 f : X Y 가함수이므로성립한다. Theorem 7 두함수 f : X Y 와 g : X Y 에대하여, f = g f(x)

More information

<B4EBC7D0BCF6C7D02DBBEFB0A2C7D4BCF62E687770>

<B4EBC7D0BCF6C7D02DBBEFB0A2C7D4BCF62E687770> 삼각함수. 삼각함수의덧셈정리 삼각함수의덧셈정리 삼각함수 sin (α + β ), cos (α + β ), tan (α + β ) 등을 α 또는 β 의삼각함수로나 타낼수있다. 각 α 와각 β 에대하여 α >0, β >0이고 0 α - β < β 를만족한다고가정하 자. 다른경우에도같은방법으로증명할수있다. 각 α 와각 β 에대하여 θ = α - β 라고놓자. 위의그림에서원점에서거리가

More information

1 11 111 111-1 p, q, r A, B, C (1 p

More information

FGB-P 학번수학과권혁준 2008 년 5 월 19 일 Lemma 1 p 를 C([0, 1]) 에속하는음수가되지않는함수라하자. 이때 y C 2 (0, 1) C([0, 1]) 가미분방정식 y (t) + p(t)y(t) = 0, t (0, 1), y(0)

FGB-P 학번수학과권혁준 2008 년 5 월 19 일 Lemma 1 p 를 C([0, 1]) 에속하는음수가되지않는함수라하자. 이때 y C 2 (0, 1) C([0, 1]) 가미분방정식 y (t) + p(t)y(t) = 0, t (0, 1), y(0) FGB-P8-3 8 학번수학과권혁준 8 년 5 월 9 일 Lemma p 를 C[, ] 에속하는음수가되지않는함수라하자. 이때 y C, C[, ] 가미분방정식 y t + ptyt, t,, y y 을만족하는해라고하면, y 는, 에서연속적인이계도함수를가지게확 장될수있다. Proof y 은 y 의도함수이므로미적분학의기본정리에의하여, y 은 y 의어떤원시 함수와적분상수의합으로표시될수있다.

More information

완벽한개념정립 _ 행렬의참, 거짓 수학전문가 NAMU 선생 1. 행렬의참, 거짓개념정리 1. 교환법칙과관련한내용, 는항상성립하지만 는항상성립하지는않는다. < 참인명제 > (1),, (2) ( ) 인경우에는 가성립한다.,,, (3) 다음과같은관계식을만족하는두행렬 A,B에

완벽한개념정립 _ 행렬의참, 거짓 수학전문가 NAMU 선생 1. 행렬의참, 거짓개념정리 1. 교환법칙과관련한내용, 는항상성립하지만 는항상성립하지는않는다. < 참인명제 > (1),, (2) ( ) 인경우에는 가성립한다.,,, (3) 다음과같은관계식을만족하는두행렬 A,B에 1. 행렬의참, 거짓개념정리 1. 교환법칙과관련한내용, 는항상성립하지만 는항상성립하지는않는다. < 참인명제 > (1),, (2) ( ) 인경우에는 가성립한다.,,, (3) 다음과같은관계식을만족하는두행렬 A,B에대하여 AB=BA 1 가성립한다 2 3 (4) 이면 1 곱셈공식및변형공식성립 ± ± ( 복호동순 ), 2 지수법칙성립 (은자연수 ) < 거짓인명제 >

More information

<3235B0AD20BCF6BFADC0C720B1D8C7D120C2FC20B0C5C1FE20322E687770>

<3235B0AD20BCF6BFADC0C720B1D8C7D120C2FC20B0C5C1FE20322E687770> 25 강. 수열의극한참거짓 2 두수열 { }, {b n } 의극한에대한 < 보기 > 의설명중옳은것을모두고르면? Ⅰ. < b n 이고 lim = 이면 lim b n =이다. Ⅱ. 두수열 { }, {b n } 이수렴할때 < b n 이면 lim < lim b n 이다. Ⅲ. lim b n =0이면 lim =0또는 lim b n =0이다. Ⅰ 2Ⅱ 3Ⅲ 4Ⅰ,Ⅱ 5Ⅰ,Ⅲ

More information

일반각과호도법 l 삼각함수와미분 1. 일반각 시초선 OX 로부터원점 O 를중심으로 만큼회전이동한위치에동경 OP 가있을때, XOP 의크기를나타내는각들을 ( 은정수 ) 로나타내고 OP 의일반각이라한다. 2. 라디안 rad 반지름과같은길이의호에대한중심각의 크기를 라디안이라한

일반각과호도법 l 삼각함수와미분 1. 일반각 시초선 OX 로부터원점 O 를중심으로 만큼회전이동한위치에동경 OP 가있을때, XOP 의크기를나타내는각들을 ( 은정수 ) 로나타내고 OP 의일반각이라한다. 2. 라디안 rad 반지름과같은길이의호에대한중심각의 크기를 라디안이라한 일반각과호도법 l 1. 일반각 시초선 OX 로부터원점 O 를중심으로 만큼회전이동한위치에동경 OP 가있을때, XOP 의크기를나타내는각들을 ( 은정수 ) 로나타내고 OP 의일반각이라한다. 2. 라디안 rad 반지름과같은길이의호에대한중심각의 크기를 라디안이라한다. 3. 호도법과육십분법 라디안 라디안 4. 부채꼴의호의길이와넓이 반지를의길이가 인원에서중심각이 인 부채꼴의호의길이를

More information

1 1,.,

1 1,., ,.,. 7 86 0 70 7 7 7 74 75 76 77 78 79 70 7 7 7 75 74 7 7 7 70 79 78 77 76 75 74 7.,. x, x A(x ), B(x ) x x AB =x -x A{x } B{x } x >x AB =x -x B{x } A{x } x =[ -x(xæ0) -x (x

More information

Microsoft PowerPoint - 26.pptx

Microsoft PowerPoint - 26.pptx 이산수학 () 관계와그특성 (Relations and Its Properties) 2011년봄학기 강원대학교컴퓨터과학전공문양세 Binary Relations ( 이진관계 ) Let A, B be any two sets. A binary relation R from A to B, written R:A B, is a subset of A B. (A 에서 B 로의이진관계

More information

1 1 장. 함수와극한 1.1 함수를표현하는네가지방법 1.2 수학적모형 : 필수함수의목록 1.3 기존함수로부터새로운함수구하기 1.4 접선문제와속도문제 1.5 함수의극한 1.6 극한법칙을이용한극한계산 1.7 극한의엄밀한정의 1.8 연속

1 1 장. 함수와극한 1.1 함수를표현하는네가지방법 1.2 수학적모형 : 필수함수의목록 1.3 기존함수로부터새로운함수구하기 1.4 접선문제와속도문제 1.5 함수의극한 1.6 극한법칙을이용한극한계산 1.7 극한의엄밀한정의 1.8 연속 1 1 장. 함수와극한 1.1 함수를표현하는네가지방법 1.2 수학적모형 : 필수함수의목록 1.3 기존함수로부터새로운함수구하기 1.4 접선문제와속도문제 1.5 함수의극한 1.6 극한법칙을이용한극한계산 1.7 극한의엄밀한정의 1.8 연속 2 1.1 함수를표현하는네가지방법 함수 f : D E 는집합 D 의각원소 x 에집합 E 에속하는단하나의원소 f(x) 를 대응시키는규칙이다.

More information

2 KAIST 1988,,KAIST MathLetter, 3,,, 3,, 3, 3,

2 KAIST 1988,,KAIST MathLetter, 3,,, 3,, 3, 3, (M 2 ) 2 KAIST 1988,,KAIST MathLetter, 3,,, 3,, 3, 3, 3,,, 2003 8, 4 1 7 11 8 12 26 2 39 21 40 22 54 23 67 24 80 3 93 31 n! 94 32 101 33 115 4 131 41 132 6 42 146 5 163 51 164 52 180 1 8 11 4 4?!,? 2??,?

More information

2

2 rev 2004/1/12 KAIST 2 6 7 1 13 11 13 111 13 112 18 113 19 114 21 12 24 121 24 122 26 13 28 131 28 132 30 133 (recurrence) 34 134 35 4 2 39 21 39 211 39 212 40 22 42 221, 42 222 43 223, 45 224 46 225, 48

More information

제 12강 함수수열의 평등수렴

제 12강 함수수열의 평등수렴 제 강함수수열의평등수렴 함수의수열과극한 정의 ( 점별수렴 ): 주어진집합 과각각의자연수 에대하여함수 f : 이있다고가정하자. 이때 을집합 에서로가는함수의수열이라고한다. 모든 x 에대하여 f 수열 f ( x) lim f ( x) 가성립할때함수수열 { f } 이집합 에서함수 f 로수렴한다고한다. 또 함수 f 을집합 에서의함수수열 { f } 의극한 ( 함수 ) 이라고한다.

More information

2 머리말 KAIST 수학문제연구회는한국수학올림피아드가처음탄생할무렵인 1988년부터지금까지다양한영재교육프로그램을운영하거나적극적으로기여해왔습니다. 한국수학올림피아드통신강좌와계절학교등의교육프로그램, 대전 충남지역중학생영재수학교실, KAIST Cyber영재교육등이그예들입니다

2 머리말 KAIST 수학문제연구회는한국수학올림피아드가처음탄생할무렵인 1988년부터지금까지다양한영재교육프로그램을운영하거나적극적으로기여해왔습니다. 한국수학올림피아드통신강좌와계절학교등의교육프로그램, 대전 충남지역중학생영재수학교실, KAIST Cyber영재교육등이그예들입니다 hkmo 2 차시험대비 i 엠제곱 (M 2 ) 실전수학올림피아드 1400 중학생중 / 고급풀이편 고봉균 http://cafe.naver.com/xmo Rev: 2009 년 4 월 2 일 KAIST 수학문제연구회 2 머리말 KAIST 수학문제연구회는한국수학올림피아드가처음탄생할무렵인 1988년부터지금까지다양한영재교육프로그램을운영하거나적극적으로기여해왔습니다. 한국수학올림피아드통신강좌와계절학교등의교육프로그램,

More information

Microsoft PowerPoint Relations.pptx

Microsoft PowerPoint Relations.pptx 이산수학 () 관계와그특성 (Relations and Its Properties) 2010년봄학기강원대학교컴퓨터과학전공문양세 Binary Relations ( 이진관계 ) Let A, B be any two sets. A binary relation R from A to B, written R:A B, is a subset of A B. (A 에서 B 로의이진관계

More information

제 5강 리만적분

제 5강 리만적분 제 5 강리만적분 리만적분 정의 : 두실수, 가 을만족핚다고가정하자.. 만일 P [, ] 이고 P 가두끝점, 을모두포함하는유핚집합일때, P 을 [, ] 의분핛 (prtitio) 이라고핚다. 주로 P { x x x } 로나타낸다.. 분핛 P { x x x } 의노름을다음과같이정의핚다. P x x x. 3. [, ] 의두분핛 P 와 Q 에대하여만일 P Q이면 Q

More information

1 경영학을 위한 수학 Final Exam 2015/12/12(토) 13:00-15:00 풀이과정을 모두 명시하시오. 정리를 사용할 경우 명시하시오. 1. (각 6점) 다음 적분을 구하시오 Z 1 4 Z 1 (x + 1) dx (a) 1 (x 1)4 dx 1 Solut

1 경영학을 위한 수학 Final Exam 2015/12/12(토) 13:00-15:00 풀이과정을 모두 명시하시오. 정리를 사용할 경우 명시하시오. 1. (각 6점) 다음 적분을 구하시오 Z 1 4 Z 1 (x + 1) dx (a) 1 (x 1)4 dx 1 Solut 경영학을 위한 수학 Fial Eam 5//(토) :-5: 풀이과정을 모두 명시하시오. 정리를 사용할 경우 명시하시오.. (각 6점) 다음 적분을 구하시오 4 ( ) (a) ( )4 8 8 (b) d이 성립한다. d C C log log (c) 이다. 양변에 적분을 취하면 log C (d) 라 하자. 그러면 d 4이다. 9 9 4 / si (e) cos si

More information

0 cm (++x)=0 x= R QR Q =R =Q = cm =Q =-=(cm) =R =x cm (x+) = +(x+) x= x= (cm) =+=0 (cm) =+=8 (cm) + =0+_8= (cm) cm + = + = _= (cm) 7+x= x= +y= y=8,, Q

0 cm (++x)=0 x= R QR Q =R =Q = cm =Q =-=(cm) =R =x cm (x+) = +(x+) x= x= (cm) =+=0 (cm) =+=8 (cm) + =0+_8= (cm) cm + = + = _= (cm) 7+x= x= +y= y=8,, Q . 09~ cm 7 0 8 9 8'-p 0 cm x=, y=8 cm 0' 7 cm 8 cm 9 'åcm 90 'åcm T T=90 T T =" 8 - =' (cm) T= T= _T _T _'_ T=8' (cm ) 7 = == =80 -_ =0 = = _=(cm) M = = _0= (cm) M M =" - = (cm) r cm rcm (r-)cm H 8cm cm

More information

01

01 2019 학년도대학수학능력시험 9 월모의평가문제및정답 2019 학년도대학수학능력시험 9 월모의평가문제지 1 제 2 교시 5 지선다형 1. 두벡터, 모든성분의합은? [2 점 ] 에대하여벡터 의 3. 좌표공간의두점 A, B 에대하여선분 AB 를 로외분하는점의좌표가 일때, 의값은? [2점] 1 2 3 4 5 1 2 3 4 5 2. lim 의값은? [2점] 4. 두사건,

More information

ÀÎÅͳÝ-°ø°£µµÇüÇØ

ÀÎÅͳÝ-°ø°£µµÇüÇØ .. Q.... M M : M Q : Q M : //Q.,.. I FG FE F FG, HG EH H HG F G FG ;!;_F _FG ;!;_G _F ;!;_'_;!; F F... 5. 5. 6. 5 7. 0 8. 7 9. ' FG, HG H G, H F E G H '. FG HG F, H. FH ' FH ' ' {} +{} -(') cos h -;!;

More information

제 2 교시 2019 학년도 3 월고 1 전국연합학력평가문제지수학영역 1 5 지선다형 1. 의값은? [2점] 일차방정식 의해는? [2 점 ] 두수, 의최대공약수는? [2 점 ] 일차함수 의그래프에서

제 2 교시 2019 학년도 3 월고 1 전국연합학력평가문제지수학영역 1 5 지선다형 1. 의값은? [2점] 일차방정식 의해는? [2 점 ] 두수, 의최대공약수는? [2 점 ] 일차함수 의그래프에서 제 2 교시 2019 학년도 3 월고 1 전국연합학력평가문제지 1 5 지선다형 1. 의값은? [2점] 1 2 3 4 5 3. 일차방정식 의해는? [2 점 ] 1 2 3 4 5 2. 두수, 의최대공약수는? [2 점 ] 1 2 3 4 5 4. 일차함수 의그래프에서 절편과 절편의합은? [3 점 ] 1 2 3 4 5 1 12 2 5. 함수 의그래프가두점, 를지날때,

More information

정수론 - (Number Theory)

정수론 - (Number Theory) 정수론 (Number Theory) 정주희 (Jeong, Joohee) Kyungpook National University 2017 년 9 월 4 일. 자연대 101 정주희 (Jeong, Joohee) (K.N.U.) 정수론 2017 년 9 월 4 일 1 / 36 목차 1 최대공약수 2 부정방정식과합동식 3 페르마의정리와오일러의정리 4 원시근, 이산로그,

More information

Python과 함께 배우는 신호 해석 제 5 강. 복소수 연산 및 Python을 이용한 복소수 연산 (제 2 장. 복소수 기초)

Python과 함께 배우는 신호 해석 제 5 강. 복소수 연산 및 Python을 이용한 복소수 연산      (제 2 장. 복소수 기초) 제 5 강. 복소수연산및 을이용한복소수연산 ( 제 2 장. 복소수기초 ) 한림대학교전자공학과 한림대학교 제 5 강. 복소수연산및 을이용한복소수연산 1 배울내용 복소수의기본개념복소수의표현오일러 (Euler) 공식복소수의대수연산 1의 N 승근 한림대학교 제 5 강. 복소수연산및 을이용한복소수연산 2 복소수의 4 칙연산 복소수의덧셈과뺄셈에는직각좌표계표현을사용하고,

More information

-주의- 본 교재는 최 상위권을 위한 고난이도 모의고사로 임산부 및 노약자의 건강에 해로울 수 있습니다.

-주의- 본 교재는 최 상위권을 위한 고난이도 모의고사로 임산부 및 노약자의 건강에 해로울 수 있습니다. Intensive Math 극악 모의고사 - 인문계 등급 6점, 등급 점으로 난이도를 조절하여 상위권 학생들도 불필요한 문제에 대한 시간 낭비 없이 보다 많은 문제에서 배움을 얻을 수 있도록 구성하였습니다. 단순히 어렵기만 한 문제들의 나열이 아니라 수능에 필요한 대표 유형을 분류 하고 일반적인 수험환경에서 흔하게 배울 수 있는 내용들은 과감하게 삭제 수능시험장

More information

LTUR Q X 01 LTUR LTUR K 6 5, 6, 6, 7, 8, 9, 9, 9, 10, b= =: :=8.5 a+b= cm , = =: 7 := a+b+c 0 =1 a+b+

LTUR Q X 01 LTUR LTUR K 6 5, 6, 6, 7, 8, 9, 9, 9, 10, b= =: :=8.5 a+b= cm , = =: 7 := a+b+c 0 =1 a+b+ 우공비 중등 수학 (하) 특강편 SLUTIN LTUR K WRK K 0 LTUR Q X 01 LTUR LTUR K 6 5, 6, 6, 7, 8, 9, 9, 9, 10, 10 8+9 b= =: :=8.5 a+b=17.5 17.5 1 159 cm 6 9 58 6, 7..5 01 = +0+1++++ 7 =: 7 := a+b+c 0 =1 a+b+c=6 6+8+1 =:

More information

PowerPoint Presentation

PowerPoint Presentation 5 불대수 IT CookBook, 디지털논리회로 - 2 - 학습목표 기본논리식의표현방법을알아본다. 불대수의법칙을알아본다. 논리회로를논리식으로논리식을논리회로로표현하는방법을알아본다. 곱의합 (SOP) 과합의곱 (POS), 최소항 (minterm) 과최대항 (mxterm) 에대해알아본다. 01. 기본논리식의표현 02. 불대수법칙 03. 논리회로의논리식변환 04.

More information

( )EBS문제집-수리

( )EBS문제집-수리 www.ebsi.co.kr 50 024 www.ebsi.co.kr 025 026 01 a 2 A={ } AB=2B 1 4 B a 03 æ10 yæ10 y 10000 y (log )( log y) Mm M+m 3 5 7 9 11 02 { -2 1} f()=-{;4!;} +{;2!;} +5 Mm Mm -21-18 -15-12 -9 04 a =1a«+a«=3n+1(n=1,

More information

(001~042)개념RPM3-2(정답)

(001~042)개념RPM3-2(정답) - 0 0 0 0 6 0 0 06 66 07 79 08 9 0 000 000 000 000 0 8+++0+7+ = 6 6 = =6 6 6 80+8+9+9+77+86 = 6 6 = =86 86 6 8+0++++6++ = 8 76 = = 8 80 80 90 00 0 + = 90 90 000 7 8 9 6 6 = += 7 +7 =6 6 0006 6 7 9 0 8

More information

#수Ⅱ지도서-4단( )

#수Ⅱ지도서-4단( ) IV 4 3 4 5 5 exponent 3 3 Archimedes B.C. 87~B.C. Diophantos?00~?84 a m _a n =a m+n (mn=0y) Stifel M. 487~567 Arithmetica integra y-3--03y y ;8!; ;4!; ;!; 48y Stevin S. 548~60 xx x ()()(3) x ;!; x ;3!;

More information

Unknown

Unknown 0 THEME!!!_!_!_!_!=_6=8 pp. ~8!!!_!=70 0, P =_=, 0, _=9, _=9,, +9+9=0 6 6!=70, f, l, w, r P _!= =88 70-88= THEME (-)!=!!!_!=6 (-)!=!!!_!= 6 (-)!=! 6_!=6_= 6 (6-)!=!=0 0_=60, 6! 6 = =60 _ e, t l, r 6! =80!!

More information

PSFZWLOTGJYU.hwp

PSFZWLOTGJYU.hwp 학년도대수능 9 월모의평가 ( 수리영역 - 가형 AH AT sin 8. log 9 log. log log 일때, ( 분모 ( 분자 이어야한다. 즉, ( +a-b+a-b a - b - ᄀ +a+b - (-(-b (-( ++ -b + + - b -b 9 ᄂ ᄀ, ᄂ에서 a, b 8 a+ b 5. log log X AB -B ( ( - - ( - ( 5 - -8

More information

(001~006)개념RPM3-2(부속)

(001~006)개념RPM3-2(부속) www.imth.tv - (~9)개념RPM-(본문).. : PM RPM - 대푯값 페이지 다민 PI LPI 알피엠 대푯값과산포도 유형 ⑴ 대푯값 자료 전체의 중심적인 경향이나 특징을 하나의 수로 나타낸 값 ⑵ 평균 (평균)= Ⅰ 통계 (변량)의 총합 (변량의 개수) 개념플러스 대푯값에는 평균, 중앙값, 최 빈값 등이 있다. ⑶ 중앙값 자료를 작은 값부터 크기순으로

More information

<B1B9BEEE412E687770>

<B1B9BEEE412E687770> 201 학년도대학수학능력시험 6 월모의평가문제및정답 2016 학년도대학수학능력시험 6 월모의평가문제지 1 제 2 교시 5 지선다형 1. 두행렬 성분은? [2 점 ] 에대하여행렬 의 3. lim 의값은? [2점] 1 2 3 4 5 1 2 3 4 5 2. 의값은? [2점] 1 2 3 4 5 4. 공차가 인등차수열 에대하여 의값은? [3 점 ] 1 2 3 4 5

More information

함수공간 함수공간, 점열린위상 Definition 0.1. X와 Y 는임의의집합이고 F(X, Y ) 를 X에서 Y 로의모든함수족이라하자. 집합 F(X, Y ) 에위상을정의할때이것을함수공간 (function space) 이라한다. F(X, Y ) 는다음과같이적당한적집합과

함수공간 함수공간, 점열린위상 Definition 0.1. X와 Y 는임의의집합이고 F(X, Y ) 를 X에서 Y 로의모든함수족이라하자. 집합 F(X, Y ) 에위상을정의할때이것을함수공간 (function space) 이라한다. F(X, Y ) 는다음과같이적당한적집합과 함수공간 함수공간, 점열린위상 Definition.1. X와 Y 는임의의집합이고 F(X, Y ) 를 X에서 Y 로의모든함수족이라하자. 집합 F(X, Y ) 에위상을정의할때이것을함수공간 (function spce) 이라한다. F(X, Y ) 는다음과같이적당한적집합과같음을볼수있다. 각 x X에대해 Y x = Y 라하자. 그리고 F := Y x x X 이라하자.

More information

제1장 군 제1절 소개와 예 제2절 이항연산 2.1 보기. 다음은 정수방정식 a + x = b를 푸는 과정이다. (1) 준식에 a를 더하여 ( a) + (a + x) = ( a) + b. (2) 결합법칙을 사용하면 (( a) + a) + x = ( a) + b. (3)

제1장 군 제1절 소개와 예 제2절 이항연산 2.1 보기. 다음은 정수방정식 a + x = b를 푸는 과정이다. (1) 준식에 a를 더하여 ( a) + (a + x) = ( a) + b. (2) 결합법칙을 사용하면 (( a) + a) + x = ( a) + b. (3) 제장 군 제절 소개와 예 제절 이항연산. 보기. 다음은 정수방정식 + x = b를 푸는 과정이다. () 준식에 를 더하여 ( ) + ( + x) = ( ) + b. () 결합법칙을 사용하면 (( ) + ) + x = ( ) + b. () ( ) + = 임을 이용하면 + x = ( ) + b. (4) + x = x 이므로 x = ( ) + b. 이를 유리수방정식

More information

7. 인실수 에대하여 log 의지표를 이라할때, 옳 은것을보기에서모두고르면? ( 단, 는 를넘지않는최대의정수이다.) 7 ) ㄱ. log ㄴ. log 의지표는 이다. ㄷ. log log 이면 은 자리의정수 이다. 10. 다음은어느인터넷사이트의지도상단에있는버튼의기능을설명한

7. 인실수 에대하여 log 의지표를 이라할때, 옳 은것을보기에서모두고르면? ( 단, 는 를넘지않는최대의정수이다.) 7 ) ㄱ. log ㄴ. log 의지표는 이다. ㄷ. log log 이면 은 자리의정수 이다. 10. 다음은어느인터넷사이트의지도상단에있는버튼의기능을설명한 제 2 교시 2008 년 5 월고 3 모의고사문제지 성명수험번호 3 1 먼저수험생이선택한응시유형의문제지인지확인하시오. 문제지에성명과수험번호를정확히기입하시오. 답안지에수험번호, 응시유형및답을표기할때는반드시 수험생이지켜야할일 에따라표기하시오. 단답형답의숫자에 0 이포함된경우, 0 을 OMR 답안지에반드시표기해야합니다. 문항에따라배점이다르니, 각물음의끝에표시된배점을참고하시오.

More information

1 peaieslvfp3 1. 두점사이의거리 수직선위의두점사이의거리를구할수있다. 좌표평면위의두점사이의거리를구할수있다. 수직선위의두점사이의거리 todrkrgo qhqtlek 오른쪽그림은충무로역을중심으로한서울시지하철 3`호선노선도의일부분이다. 충무로역을` 0, 을지로 3`

1 peaieslvfp3 1. 두점사이의거리 수직선위의두점사이의거리를구할수있다. 좌표평면위의두점사이의거리를구할수있다. 수직선위의두점사이의거리 todrkrgo qhqtlek 오른쪽그림은충무로역을중심으로한서울시지하철 3`호선노선도의일부분이다. 충무로역을` 0, 을지로 3` peaieslvfp. 두점사이의거리 수직선위의두점사이의거리를구할수있다. 좌표평면위의두점사이의거리를구할수있다. 수직선위의두점사이의거리 todrkrgo qhqtlek 오른쪽그림은충무로역을중심으로한서울시지하철 `호선노선도의일부분이다. 충무로역을` 0, 을지로 `가역을 ``로나타낼때, 다음물음에답하여라. 독립문 경복궁 안국종로 가을지로 가충무로동대입구약수금호옥수압구정잠원신사

More information

고 학년도 9월고수학 1 전국연합학력평가영역문제지 1 1 제 2 교시 수학영역 5 지선다형 3. 두다항식, 에대하여 는? [ 점 ] 1. 의값은? ( 단, ) [ 점 ] 다항식 이 로인수분해될때, 의값은? ( 단,,

고 학년도 9월고수학 1 전국연합학력평가영역문제지 1 1 제 2 교시 수학영역 5 지선다형 3. 두다항식, 에대하여 는? [ 점 ] 1. 의값은? ( 단, ) [ 점 ] 다항식 이 로인수분해될때, 의값은? ( 단,, 고 208학년도 9월고수학 전국연합학력평가영역문제지 제 2 교시 수학영역 5 지선다형 3. 두다항식, 에대하여 는? [ 점 ]. 의값은? ( 단, ) [ 점 ] 2 3 2 3 4 5 4 5 2. 다항식 이 로인수분해될때, 의값은? ( 단,, 는상수이다.) [ 점 ] 4. 좌표평면위의두점 A, B 사이의거리가 일때, 양수 의값은? [ 점 ] 2 3 4 5 2

More information

수리 영역 가 형 5. 다음 그림과 같이 크기가 같은 정육면체 개가 한 모서리씩을 공유하 면서 각 면이 평행 또는 수직 관계를 유지한 채로 한 평면 위에 놓여있 다. 그림의 세 꼭짓점 A, B, C에 대한 두 벡터 BA 와 BC 가 이루는 각 의 크기를 h라 할 때,

수리 영역 가 형 5. 다음 그림과 같이 크기가 같은 정육면체 개가 한 모서리씩을 공유하 면서 각 면이 평행 또는 수직 관계를 유지한 채로 한 평면 위에 놓여있 다. 그림의 세 꼭짓점 A, B, C에 대한 두 벡터 BA 와 BC 가 이루는 각 의 크기를 h라 할 때, 제``교시 수리 영역( 가 형) 시간:00분 점수:00점 성명 수험 번호 쭚 반드시 본인이 선택한 유형( 가 형 또는 나 형)의 문제인지 확인하시오. 쭚 문제지와 답안지에 성명과 수험 번호를 정확히 기입하시오. 쭚 MR 답안지에 성명, 수험 번호, 응시 유형 및 선택 과목, 답 등을 표기할 때에는 반드시 수험생이 지켜야 할 사항 에 따라 표기하시오. 쭚 문항에

More information

10-2 삼각형의닮음조건 p270 AD BE C ABC DE ABC 중 2 비상 10, 11 단원도형의닮음 (& 활용 ) - 2 -

10-2 삼각형의닮음조건 p270 AD BE C ABC DE ABC 중 2 비상 10, 11 단원도형의닮음 (& 활용 ) - 2 - 10 단원 : 도형의닮음 10-1 닮음도형 p265 ABC DEF ABC DEF EF B ABCD EFGH ABCD EFGH EF A AB GH ADFC CF KL 중 2 비상 10, 11 단원도형의닮음 (& 활용 ) - 1 - 10-2 삼각형의닮음조건 p270 AD BE C ABC DE ABC 중 2 비상 10, 11 단원도형의닮음 (& 활용 ) - 2 -

More information

8. 수직선위에다음수들이대응할때, 원점에서가장멀리 위치한수는? 12. Å + 7 ã Å + 5 ã Å 16 ã + 3 을계산하여라 다음에서그결과가다른하나는? 1 3 보다 5 만큼큰수 9. 두정수 a, b

8. 수직선위에다음수들이대응할때, 원점에서가장멀리 위치한수는? 12. Å + 7 ã Å + 5 ã Å 16 ã + 3 을계산하여라 다음에서그결과가다른하나는? 1 3 보다 5 만큼큰수 9. 두정수 a, b 범위 : 소인수분해 정수와유리수 50 문항 / 중반 : 이름 : 중 1-1 수학중간고사대비 1. 다음중 81 의약수는? 1 2 2 4 3 5 4 6 5 9 6. 다음수들에대한설명으로옳은것은? 1 10, 1.2, 2, 2 5, 0, 4, 10 2 1 양수는 4 개이다. 2. 세수 2 7 2, 2 2 7 11, 5 11 2 의최소공배수는? 1 2 5 7 11 2

More information

3 x =2y x =-16y 1 4 {0 ;4!;} y=-;4!; y x =y 1 5 5'2 2 (0 0) 4 (3-2) 3 3 x=0 y=0 x=2 y=1 :: 1 4 O x 1 1 -:: y=-:: 4 4 {0 -;2!;} y=;2!; l A y 1

3 x =2y x =-16y 1 4 {0 ;4!;} y=-;4!; y x =y 1 5 5'2 2 (0 0) 4 (3-2) 3 3 x=0 y=0 x=2 y=1 :: 1 4 O x 1 1 -:: y=-:: 4 4 {0 -;2!;} y=;2!; l A y 1 = =-6 {0;!;}=-;!; = 5 5' (00) (-) =0=0 == :: -:: =-:: {0-;!;}=;!; 0 l :: -:: =:: F 5 0cm =- = =- (0)=- =6 =0 =- (0)=- - =- {-;#;0}=;#; =- - =- (0)=- = =8 -:: :: =:: - =- 98 R l Q P B S r rb l Q P B lb

More information

프로그래밍개론및실습 2015 년 2 학기프로그래밍개론및실습과목으로본내용은강의교재인생능출판사, 두근두근 C 언어수업, 천인국지음을발췌수정하였음

프로그래밍개론및실습 2015 년 2 학기프로그래밍개론및실습과목으로본내용은강의교재인생능출판사, 두근두근 C 언어수업, 천인국지음을발췌수정하였음 프로그래밍개론및실습 2015 년 2 학기프로그래밍개론및실습과목으로본내용은강의교재인생능출판사, 두근두근 C 언어수업, 천인국지음을발췌수정하였음 CHAPTER 9 둘중하나선택하기 관계연산자 두개의피연산자를비교하는연산자 결과값은참 (1) 아니면거짓 (0) x == y x 와 y 의값이같은지비교한다. 관계연산자 연산자 의미 x == y x와 y가같은가? x!= y

More information

Check 0-9, 9,, - 6, 6, 6, =0.04, (-0.) = , =64 8 8, -8 (-6) =6 (-6) 6, -6 7, , -0. 8, -8 6, '7 ' '

Check 0-9, 9,, - 6, 6, 6, =0.04, (-0.) = , =64 8 8, -8 (-6) =6 (-6) 6, -6 7, , -0. 8, -8 6, '7 ' ' 0 06 0 4 4 9 4 8 5 40 45 5 57 Check 0-9, 9,, - 6, 6, 6, -6 0-0. =0.04, (-0.) =0.04 0.04 0., -0. 8 =64 8 8, -8 (-6) =6 (-6) 6, -6 7, -7 0. 0., -0. 8, -8 6, -6 0-7 7 '7 ' 0.5 0.5 -' 0.5 ;!; ;!; æ;!; '7 '

More information

Vector Differential: 벡터 미분 Yonghee Lee October 17, 벡터미분의 표기 스칼라미분 벡터미분(Vector diffrential) 또는 행렬미분(Matrix differential)은 벡터와 행렬의 미분식에 대 한 표

Vector Differential: 벡터 미분 Yonghee Lee October 17, 벡터미분의 표기 스칼라미분 벡터미분(Vector diffrential) 또는 행렬미분(Matrix differential)은 벡터와 행렬의 미분식에 대 한 표 Vector Differential: 벡터 미분 Yonhee Lee October 7, 08 벡터미분의 표기 스칼라미분 벡터미분(Vector diffrential) 또는 행렬미분(Matrix differential)은 벡터와 행렬의 미분식에 대 한 표기법을 정의하는 방법이다 보통 스칼라(scalar)에 대한 미분은 일분수 함수 f : < < 또는 다변수 함수(function

More information

16중등빨이수학3-2교부(01~23)

16중등빨이수학3-2교부(01~23) 9 a b c a+b+c 0 7 a 79799 79a 0 9 1 0 7 0 99 9 9 0 0 7 7 0 717 9 7 0 1 7-1 - - 7 1 0 0 1 ab b-a 0 997 7 9191 17 S 7 1 0 0 1 0 0 a b c a+b+c 1 7 [0~09] 0 _ 7 7 0 0 0 [~0] [09~] 777 017 0 09 0 0 1 0 1 9

More information

Microsoft PowerPoint - ch02-1.ppt

Microsoft PowerPoint - ch02-1.ppt 2. Coodinte Sstems nd Tnsfomtion 20 20 2.2 Ctesin Coodintes (,, ) () (b) Figue 1.1 () Unit vectos,, nd, (b) components of long,, nd. 직각좌표계에서각변수 (,, ) 들의범위 < < < < < < (2.1) 직각좌표계에서임의의벡터 는,, 가그림 1.1 에서와같이,,

More information

°ø±â¾Ð±â±â

°ø±â¾Ð±â±â 20, 30, 40 20, 30, 40 1 2 3 4 5 6 7 8 9 10 3.1 6.3 9.4 12.6 15.7 18.8 22.0 25.1 28.3 31.4 2.4 4.7 7.1 9.4 11.8 14.1 16.5 18.8 21.2 23.6 7.1 14.1 21.2 28.3 35.3 42.4 49.5 56.5 63.6 70.7 5.9 11.9 17.8 23.7

More information

집합 집합 오른쪽 l 3. (1) 집합 X 의각원소에대응하는집합 Y 의원소가단하나만인대응을 라할때, 이대응 를 X 에서 Y 로의라고하고이것을기호로 X Y 와같이나타낸다. (2) 정의역과공역정의역 : X Y 에서집합 X, 공역 : X Y 에서집합 Y (3) 의개수 X Y

집합 집합 오른쪽 l 3. (1) 집합 X 의각원소에대응하는집합 Y 의원소가단하나만인대응을 라할때, 이대응 를 X 에서 Y 로의라고하고이것을기호로 X Y 와같이나타낸다. (2) 정의역과공역정의역 : X Y 에서집합 X, 공역 : X Y 에서집합 Y (3) 의개수 X Y 어떤 다음 X 대응 1. 대응 (1) 어떤주어진관계에의하여집합 X 의원소에집합 Y 의원소를짝지어주는것을집합 X 에서집합 Y 로의대응이라고한다. l (2) 집합 X 의원소 에집합 Y 의원소 가짝지어지면 에 가대응한다고하며이것을기호로 와같이나타낸다. 2. 일대일대응 (1) 집합 A 의모든원소와집합 B 의모든원소가하나도빠짐없이꼭한개씩서로대응되는것을집합 A 에서집합

More information

[Real Analysis]4.1

[Real Analysis]4.1 정동명해석학 4.1 수열의수렴성 1. 다음의수열 중에서어느것이수렴하는가를조사하여라. 또, 그이유를밝혀라. (1) 수렴한다. 임의의 에대하여 아르키메데스성질에의하여 을만족하는 을택하면 일때, 이성립한다. 여기서 이므로 이성립한다. 따라서 은 1 로수렴한다. (2) 수렴한다. 임의의 에대하여 아르키메데스성질에의하여 을만족하는 을택하면 일때, 이성립한다. 따라서

More information

A y y y y y # 2#

A y y y y y # 2# 0. 9 A 0 0. 0-0.5748 0 0.454545 04 0.4 05 0.5 06 0.4 07-0.555 08 0.9666 09 5@ 5@ 00 0.5 0 5 5 5@ 5 # # 7 0.07 0.5 0.55 4 0.5 5 0.06 6 7 8 \ 9 \ 0 \ 0.^ 40-.4^0^ 4 50.^5^ 5 55.0^5^ 6 0.4^857^4857 7 0.^8^8

More information

121_중등RPM-1상_01해(01~10)ok

121_중등RPM-1상_01해(01~10)ok 1-01 00 11 03 1804 4 05 3506 45 07 5 65 0001 000 0003 0004 0005 01 4 4 6 5 6 9 Í = + =,, Í=Í=Í = = Í Í Í,, 0006 0007 0008 0009 0010 0011 001 7c 5c 3, 3 3, 6, 6 +50 =180 =130 130 +90 +30 =180 =60 60 =60

More information

Microsoft PowerPoint - chap04-연산자.pptx

Microsoft PowerPoint - chap04-연산자.pptx int num; printf( Please enter an integer: "); scanf("%d", &num); if ( num < 0 ) printf("is negative.\n"); printf("num = %d\n", num); } 1 학습목표 수식의 개념과 연산자, 피연산자에 대해서 알아본다. C의 를 알아본다. 연산자의 우선 순위와 결합 방향에

More information

완비거리공간 완비거리공간 Definition 0.1. (X, d) 는거리공간일때 X의점렬 < a n > 이모든 ɛ > 0에대해 n o N such that n, m > n o = d(a n, a m ) < ɛ 을만족하면이점렬을코시열 (Cauchy sequence) 이라

완비거리공간 완비거리공간 Definition 0.1. (X, d) 는거리공간일때 X의점렬 < a n > 이모든 ɛ > 0에대해 n o N such that n, m > n o = d(a n, a m ) < ɛ 을만족하면이점렬을코시열 (Cauchy sequence) 이라 완비거리공간 완비거리공간 Definition 0.1. (X, d) 는거리공간일때 X의점렬 < a n > 이모든 ɛ > 0에대해 n o N such that n, m > n o = d(a n, a m ) < ɛ 을만족하면이점렬을코시열 (Cauchy sequence) 이라한다. Example 0.2. < a n > 이 p에수렴하는점렬이면모든 ɛ > 0에대해 n

More information

1

1 절대수학 검은 대장간 인문 Blacksmith Day 1 최석호 1. 그림과 같이 A B C D E의 다섯 개의 영역에 빨강, 노랑, 파 랑, 초록의 네 가지 색으로 색칠을 하려고 한다. 네 가지 색 중 한 색 은 두 번 사용하고 나머지 세 가지 색은 한 번씩만 사용하여 칠하는 데, 인접한 영역에는 서로 다른 색을 칠하기로 할 때, 색칠하는 방법 의 수를 구하시오.

More information

2 A A Cs A C C A A B A B 15 A C 30 A B A C B. 1m 1m A. 1 C.1m P k A B u k GPS GPS GPS GPS 4 2

2 A A Cs A C C A A B A B 15 A C 30 A B A C B. 1m 1m A. 1 C.1m P k A B u k GPS GPS GPS GPS 4 2 www.ebsi.co.kr 2 A A 1 133 Cs 1 11 1 A C C A A B A B 15 A C 30 A B A C B. 1m 1m A. 1 C.1m P k A B u k GPS GPS GPS GPS 4 2 www.ebsi.co.kr A B t B A ;2!;t v v= = (3_t)+(6_0.5t) v=4 m/s t+0.5t 3 m/s 6 m/s

More information

5. 두함수 log 에대하여옳은것을 < 보기 > 에서모두고르면?5 ) ㄱ. ㄴ. ㄷ. < 보기 > 1 ㄴ 2 ㄷ 3 ㄱ, ㄴ 4 ㄴ, ㄷ 5 ㄱ, ㄴ, ㄷ 7. 인실수 에대하여 log 의지표를 이라할때, 옳 은것을보기에서모두고르면? ( 단, 는 를넘지않는최대의정수이다.

5. 두함수 log 에대하여옳은것을 < 보기 > 에서모두고르면?5 ) ㄱ. ㄴ. ㄷ. < 보기 > 1 ㄴ 2 ㄷ 3 ㄱ, ㄴ 4 ㄴ, ㄷ 5 ㄱ, ㄴ, ㄷ 7. 인실수 에대하여 log 의지표를 이라할때, 옳 은것을보기에서모두고르면? ( 단, 는 를넘지않는최대의정수이다. 제 2 교시 2008 년 5 월고 3 모의고사문제지 성명수험번호 3 1 먼저수험생이선택한응시유형의문제지인지확인하시오. 문제지에성명과수험번호를정확히기입하시오. 답안지에수험번호, 응시유형및답을표기할때는반드시 수험생이지켜야할일 에따라표기하시오. 단답형답의숫자에 0 이포함된경우, 0 을 OMR 답안지에반드시표기해야합니다. 문항에따라배점이다르니, 각물음의끝에표시된배점을참고하시오.

More information

Áß2±âÇØ(01~56)

Áß2±âÇØ(01~56) PRT 0 heck x=7y=0 x=0y=90 9 RH RHS 8 O =8 cmp =6 cm 6 70 7 8 0 0 0 SS 90 0 0 0 06 07 08 09 0 cm 6 7 8 9 0 S 6 7 8 9 0 8cm 6 9cm 7 8 9 cm 0 cm x=0 y=00 0 6 7 9 8 9 0 0 cm 6 7 8 9 60 6 6 6 6 6 6 7 8 7 0

More information

중등수학2팀-지도서7

중등수학2팀-지도서7 3 6~7 8~3 3 ª 33~37 4-38~39 40~45 4 46~53 5 54~58 3 59-60 ~6 6~63 64 VII. 4 9 (Klein F849~95) (rlangen Program) (group of transformation) ' O' =k O ' O k O ' O ' O ' ' ' ' (topology) = = O O' =k O ' '

More information

1. A B C 4. ABC B C A B A B C A C AB BC ABC. ABC C + A + B C A B A B C A B C B A C B C A C A B C B A 5. AB xy pqr x B xy p -y AB. A. A. B. TV B. C. AB

1. A B C 4. ABC B C A B A B C A C AB BC ABC. ABC C + A + B C A B A B C A B C B A C B C A C A B C B A 5. AB xy pqr x B xy p -y AB. A. A. B. TV B. C. AB 018 학년도대학수학능력시험문제및정답 1. A B C 4. ABC B C A B A B C A C AB BC ABC. ABC C + A + B C A B A B C A B C B A C B C A C A B C B A 5. AB xy pqr x B xy p -y AB. A. A. B. TV B. C. ABC lålılç lå

More information

TOPOLOGY-WEEK 6 & 7 KI-HEON YUN 1. Quotient space( 상공간 ) X 가위상공간이고 Y 가집합이며 f : X Y 가전사함수일때, X 의위상을사용하여 Y 에위상을정의할수있는방법은? Definition 1.1. X 가위상공간, f : X

TOPOLOGY-WEEK 6 & 7 KI-HEON YUN 1. Quotient space( 상공간 ) X 가위상공간이고 Y 가집합이며 f : X Y 가전사함수일때, X 의위상을사용하여 Y 에위상을정의할수있는방법은? Definition 1.1. X 가위상공간, f : X TOPOLOGY-WEEK 6 & 7 KI-HEON YUN 1. Quotient space( 상공간 ) X 가위상공간이고 Y 가집합이며 f : X Y 가전사함수일때, X 의위상을사용하여 Y 에위상을정의할수있는방법은? Definition 1.1. X 가위상공간, f : X Y 가전사함수일때, T Y = {U Y f 1 (U) is open set in X} 로정의하면

More information

2018년 수학성취도 측정시험 모범답안/채점기준/채점소감 (2018학년도 수시모집, 정시모집 및 외국인특별전형 합격자 대상) 2018년 2월 13일, 고사시간 90분 2018년 1번 x3 + x2 + x 3 = x 1 x2 1 lim. [풀이] x3 + x2 + x 3

2018년 수학성취도 측정시험 모범답안/채점기준/채점소감 (2018학년도 수시모집, 정시모집 및 외국인특별전형 합격자 대상) 2018년 2월 13일, 고사시간 90분 2018년 1번 x3 + x2 + x 3 = x 1 x2 1 lim. [풀이] x3 + x2 + x 3 8년 수학성취도 측정시험 모범답안/채점기준/채점소감 (8학년도 수시모집, 정시모집 및 외국인특별전형 합격자 대상) 8년 월 일, 고사시간 9분 8년 번 x + x + x x x lim. [풀이] x + x + x (x )(x + x + ) lim x x x (x )(x + ) x + x + lim x x+ limx x + x + limx x + 6 lim 8년

More information

(Hyunoo Shim) 1 / 26 조건부생명확률 (coningen probabiliy) 이란? 사망의순서 ( 조건이됨 ) 를고려한생명확률동시생존자 / 최종생존자생명확률 : 사망이 x이든 y이든가리지않음 ( 대칭적 ) [ 조건부생명확률 : x와 y의사망순서를고려함 ( 비대칭적 ) ➀ 기호 : 예를들어, q 1 xy a) 사망순서 : 숫자 1, 2, 3,...

More information

< D312D3220C0CCB5EEBAAFBBEFB0A2C7FC E485750>

< D312D3220C0CCB5EEBAAFBBEFB0A2C7FC E485750> 다음 1)1) 2)2) 다음 가 3) 3) 4) 4) 나 다 5) 5) 라 6) 6) 다음 7) 7) 8) 8) 다음 1. zb 다음그림과같이 AB = AC인 ABC 에서 BC = BD 이고, BDC = 65 일때, DAB - ABD 의크기는? AB = AD 1 BC = DC 2 ( 다 ) 3 1, 2, 3으로부터대응변의길이가같으므로 ABC ( 라 ) BAC

More information

14일등예감수학2-2교사(001~026)

14일등예감수학2-2교사(001~026) 9 8 8 9 0 0 09 0 9 8~9 / [~] [~] [0~] 0 [~] 0 00 00 [~9] 8 9 8 0 00 00 0~ / [~] [~] 8 8 0 0 8 9 0 - ~ / [~] [0~] 0 0 [~9] [~] 0 8 8 9 0 0 - ~ / [~] 8 9 0 [~] ~ /0 8 0 9 0 0 0 8 0 8~9 /0 0 ;#; 0 0 0 8

More information

OCW_C언어 기초

OCW_C언어 기초 초보프로그래머를위한 C 언어기초 4 장 : 연산자 2012 년 이은주 학습목표 수식의개념과연산자및피연산자에대한학습 C 의알아보기 연산자의우선순위와결합방향에대하여알아보기 2 목차 연산자의기본개념 수식 연산자와피연산자 산술연산자 / 증감연산자 관계연산자 / 논리연산자 비트연산자 / 대입연산자연산자의우선순위와결합방향 조건연산자 / 형변환연산자 연산자의우선순위 연산자의결합방향

More information

<30325FBCF6C7D05FB9AEC7D7C1F62E687770>

<30325FBCF6C7D05FB9AEC7D7C1F62E687770> 고1 2015학년도 9월고수학 1 전국연합학력평가영역문제지 1 1 제 2 교시 수학영역 1. 두복소수, 에대하여 의값은? ( 단, ) [2 점 ] 1 2 3 4 5 3. 좌표평면위의두점 P, Q 사이의거리는? [2 점 ] 1 2 3 4 5 2. 두다항식, 에대하여 를간단히하면? [2점] 4. 에서이차함수 의최댓값을, 최솟값을 이라할때, 의값은? [3점] 1

More information

수열의극한 수열의극한에서활용되는방법은크게다섯가지이다. ] 거미줄도형 ] 유계이론 ] 일반항 ] 부동점( 극한값) 활용 ] 샌드위치이론 ] 거미줄도형 가장첫번째로거미줄도형은대부분의경우수열의극한문제에서엄밀한증명을위해활용되기보다는수열이수렴하는지여부를판단하고수열의극한이존재한다

수열의극한 수열의극한에서활용되는방법은크게다섯가지이다. ] 거미줄도형 ] 유계이론 ] 일반항 ] 부동점( 극한값) 활용 ] 샌드위치이론 ] 거미줄도형 가장첫번째로거미줄도형은대부분의경우수열의극한문제에서엄밀한증명을위해활용되기보다는수열이수렴하는지여부를판단하고수열의극한이존재한다 수열의극한 수열의극한에서활용되는방법은크게다섯가지이다. ] 거미줄도형 ] 유계이론 ] 일반항 ] 부동점( 극한값) 활용 ] 샌드위치이론 ] 거미줄도형 가장첫번째로거미줄도형은대부분의경우수열의극한문제에서엄밀한증명을위해활용되기보다는수열이수렴하는지여부를판단하고수열의극한이존재한다면어디로수렴해야하는지를판단하는데에활용된다. 예를들어보도록하자. 수열이다음과같이정의되어있을때,

More information

1.1) 등비수열 전체집합 제 2 교시 나 형 2016 년 3 월고 3 모의고사문제지 수리영역 성명수험번호 3 1 먼저수험생이선택한응시유형의문제지인지확인하시오. 문제지에성명과수험번호를정확히기입하시오. 답안지에수험번호, 응시유형및답을표기할때는반드시 수험생이지켜야할일 에따

1.1) 등비수열 전체집합 제 2 교시 나 형 2016 년 3 월고 3 모의고사문제지 수리영역 성명수험번호 3 1 먼저수험생이선택한응시유형의문제지인지확인하시오. 문제지에성명과수험번호를정확히기입하시오. 답안지에수험번호, 응시유형및답을표기할때는반드시 수험생이지켜야할일 에따 1.1) 등비수열 전체집합 제 2 교시 2016 년 3 월고 3 모의고사문제지 성명수험번호 3 1 먼저수험생이선택한응시유형의문제지인지확인하시오. 문제지에성명과수험번호를정확히기입하시오. 답안지에수험번호, 응시유형및답을표기할때는반드시 수험생이지켜야할일 에따라표기하시오. 단답형답의숫자에 0 이포함된경우, 0 을 OMR 답안지에반드시표기해야합니다. 문항에따라배점이다르니,

More information

, _ = A _ A _ 0.H =. 00=. -> 0=. 0= =: 0 :=;^!;.0H =.0 000=0. -> 00= 0. 00= =: 0 0 :=;()$; P. 0, 0,, 00, 00, 0, 0, 0, 0 P. 0.HH= = 0.H =0. 0=. -> =0.

, _ = A _ A _ 0.H =. 00=. -> 0=. 0= =: 0 :=;^!;.0H =.0 000=0. -> 00= 0. 00= =: 0 0 :=;()$; P. 0, 0,, 00, 00, 0, 0, 0, 0 P. 0.HH= = 0.H =0. 0=. -> =0. 0 P. 8 -, 0, -, 0. p 0 0., 0., =0. =0.., 0., 0., 0., =. =0. =0. =0. P. 0,.8 0.H 8, 0.H8,.H, 0.HH,.HH, 0.H, 0.HH 0.8 0.. 0. 0, - p k k k 0.=0.H 8 0.888=0.H8.=.H 0.=0.HH.=.HH 0.=0.H 0.=0.HH P., 0.H, 0.HH,

More information

- A 2 -

- A 2 - - A 1 - - A 2 - - A 3 - - A 4 - - A 5 - - A 6 - 번호 정답 번호 정답 1 4 16 1 2 1 17 1 3 1 18 3 4 4 19 4 5 2 20 4 6 2 21 4 7 3 22 2 8 4 23 4 9 2 24 4 10 1 25 2 11 2 26 1 12 1 27 4 13 2 28 3 14 3 29 3 15 2 30 3

More information

(Microsoft PowerPoint - Ch19_NumAnalysis.ppt [\310\243\310\257 \270\360\265\345])

(Microsoft PowerPoint - Ch19_NumAnalysis.ppt [\310\243\310\257 \270\360\265\345]) 수치해석 6009 Ch9. Numerical Itegratio Formulas Part 5. 소개 / 미적분 미분 : 독립변수에대한종속변수의변화율 d vt yt dt yt 임의의물체의시간에따른위치, vt 속도 함수의구배 적분 : 미분의역, 어떤구간내에서시간 / 공간에따라변화하는정보를합하여전체결과를구함. t yt vt dt 0 에서 t 까지의구간에서곡선 vt

More information

슬라이드 1

슬라이드 1 3.7 The Inverse -transfor f ( ) Z F( ) long dvson 2 expanson n partal dvson 3 resdue ethod 3.7. Long-Dvson Method B () F( ) B( ) 를 A( ) A () 로나누어 의 negatve power seres 로표현해계수를구함 Regon of Convergence(ROC)

More information

미통기-3-06~07(052~071)

미통기-3-06~07(052~071) 06 F() f() F'()=f()F() f() : f()d f() f() f() f() F()f() F()+C : f()d=f()+c C F'()=f(): f()d=f()+c C d [: f()d]=f() d : k d=k+c k C : «d= + +C =0C + : k f()d=k: f()d k : { f() g()}d=: f()d : g()d =f()

More information

+ F F P. = = = F = F F = = 0 cm =x cm =(x+)x x=0 =0 cm cm cm x cm = =0(cm) P. 0 x=y= x= cm FF cm 0 x= x= =x(0-x) x= 0 (+)=x x= (+)=y 0 y= x= x= = 0= 0

+ F F P. = = = F = F F = = 0 cm =x cm =(x+)x x=0 =0 cm cm cm x cm = =0(cm) P. 0 x=y= x= cm FF cm 0 x= x= =x(0-x) x= 0 (+)=x x= (+)=y 0 y= x= x= = 0= 0 = = = = = - =-=0 0 F ==0 +=0 +F=0 =F ªF F = F =0 F =F = F = 0= x= x= y= y= z= z= x+y+z=++= x y z x+y+z = = ªSS = y` = = (cm) ª 0% 0% P. ªªªF =. =. =. 0 =. F =. =0 = F =. F = 0 F ªF F = =F = x=, y= x=,

More information

수리영역 5. 서로다른두개의주사위를동시에던져서나온두눈의수의곱 이짝수일때, 나온두눈의수의합이 또는 일확률은? 5) 의전개식에서상수항이존재하도록하는모든자 연수 의값의합은? 7) 다음순서도에서인쇄되는 의값은? 6) 8. 어떤특산

수리영역 5. 서로다른두개의주사위를동시에던져서나온두눈의수의곱 이짝수일때, 나온두눈의수의합이 또는 일확률은? 5) 의전개식에서상수항이존재하도록하는모든자 연수 의값의합은? 7) 다음순서도에서인쇄되는 의값은? 6) 8. 어떤특산 제 2 교시 2008 학년도 10 월고 3 전국연합학력평가문제지 수리영역 성명수험번호 3 1 먼저수험생이선택한응시유형의문제지인지확인하시오. 문제지에성명과수험번호를정확히기입하시오. 답안지에수험번호, 응시유형및답을표기할때는반드시 수험생이지켜야할일 에따라표기하시오. 단답형답의숫자에 0 이포함된경우, 0 을 OMR 답안지에반드시표기해야합니다. 문항에따라배점이다르니,

More information

<C1DF29BCF6C7D020315FB1B3BBE7BFEB20C1F6B5B5BCAD2E706466>

<C1DF29BCF6C7D020315FB1B3BBE7BFEB20C1F6B5B5BCAD2E706466> 84 85 86 87 88 89 1 12 1 1 2 + + + 11=60 9 19 21 + + + 19 17 13 11=60 + 5 7 + 5 + 10 + 8 + 4+ 6 + 3=48 1 2 90 1 13 1 91 2 3 14 1 2 92 4 1 2 15 2 3 4 93 1 5 2 6 1 2 1 16 6 5 94 1 1 22 33 55 1 2 3 4 5 6

More information

math_hsj_kK5LqN33.pdf.hwp

math_hsj_kK5LqN33.pdf.hwp 2016 학년도 3 월고 1 전국연합학력평가정답및해설 수학영역 정답 1 1 2 3 3 4 4 3 5 5 6 3 7 2 8 5 9 1 10 5 11 2 12 2 13 5 14 4 15 2 16 1 17 4 18 2 19 4 20 3 21 1 22 23 24 25 26 27 28 29 30 해설 1. [ 출제의도 ] 거듭제곱의뜻을알고식의값을계산한다. 2. [ 출제의도

More information

최종 고등수학 하.hwp

최종 고등수학 하.hwp 철/벽/수/학 고등수학 (하) 제1부 평면좌표 1 ST 철벽 CONCEPT 01 두점사이의거리 q 수직선위의두점사이의거리 수직선위의두점 A, B 사이의거리는 AB w 좌표평면위의두점사이의거리좌표평면위의두점 A, B 사이의거리는 AB Q❶-1 다음두점사이의거리를구하여라. 풀이 ⑴ A, B ⑵ A, B ⑶ A B ⑷ A B 2 배상면쌤 ^ ^ Q❶-2 다음을만족하는

More information

<B1B9BEEE412E687770>

<B1B9BEEE412E687770> 2015 학년도대학수학능력시험문제및정답 2015 학년도대학수학능력시험문제지 1 제 2 교시 홀수형 5 지선다형 1. 의값은? [2점] 3. lim 의값은? [2점] 1 2 3 4 5 1 2 3 4 5 2. 두행렬 성분의합은? [2 점 ], 에대하여행렬 의모든 4. 다음그래프의각꼭짓점사이의연결관계를나타내는행렬의성분중 의개수는? [3점] 1 2 3 4 5 1 2

More information

13일등예감수학1-1정답(077~120)

13일등예감수학1-1정답(077~120) - ~8 0 00,,, 00,, 8, 9, 0 00 00,,, 9 00,,,, 9 00 00 008 009 0 00 8 0 0 fi 0 0 0 9fi 0 0 fl 08 _ 09 _ 00 _ _ 0 _ _ 0 { } ~ 0 _ _ 0 0 _ _ fi _ 0,, 0,,, 8 08,, 9 09, 00,,,,, 8,, 0,,, 0,, 0 0 0 0 0 0, 0, 08,,

More information

기본서(상)해답Ⅰ(001~016)-OK

기본서(상)해답Ⅰ(001~016)-OK 1 1 01 01 (1) () 5 () _5 (4) _5_7 1 05 (5) { } 1 1 { } (6) _5 0 (1), 4 () 10, () 6, 5 0 (1) 18, 9, 6, 18 1,,, 6, 9, 18 01 () 1,,, 4, 4 1,,, 4, 6, 8, 1, 4 04 (1) () () (4) 1 (5) 05 (1) () () (4) 1 1 1 1

More information

A 001~A 036

A 001~A 036 4 3 2 0 8 91 0 1 2 3 4 5 6 08 09 00 01 02 03 04 18 19 10 29 20 22 23 39 30 31 32 33 48 49 40 41 59 50 69 1 2 3 4 1 2 3 4 1 4 7 10 13 1 2 3 4 5 6 rev. C C r C a f h f h L h h nrpm f h f n L C 3 P L

More information

3. 다음은카르노맵의표이다. 논리식을간략화한것은? < 나 > 4. 다음카르노맵을간략화시킨결과는? < >

3. 다음은카르노맵의표이다. 논리식을간략화한것은? < 나 > 4. 다음카르노맵을간략화시킨결과는? < > . 변수의수 ( 數 ) 가 3 이라면카르노맵에서몇개의칸이요구되는가? 2칸 나 4칸 다 6칸 8칸 < > 2. 다음진리표의카르노맵을작성한것중옳은것은? < 나 > 다 나 입력출력 Y - 2 - 3. 다음은카르노맵의표이다. 논리식을간략화한것은? < 나 > 4. 다음카르노맵을간략화시킨결과는? < > 2 2 2 2 2 2 2-3 - 5. 다음진리표를간략히한결과

More information

< D312D3420BBEFB0A2C7FCC0C720BFDCBDC9B0FA20B3BBBDC E485750>

< D312D3420BBEFB0A2C7FCC0C720BFDCBDC9B0FA20B3BBBDC E485750> 1)1) 2)2) 3) 3) 4) 4) 5) 5) 1. zb 그림에서점 O는중옳은것은? ABC 의외심이다. 3. zb 그림에서점 I 는직각삼각형 ABC 의내심이다. 삼각형의세변의길이가각각 10 cm, 8cm, 6cm 일때, 색칠한부분의넓이는? 1 OD = OE = OF 2 OA = OB = OC 3 AD = AF 4 OCE = OCF 5 OBD OBE 1 (

More information

8. 8) 다음중용어의정의로옳은것은? 1 정사각형 : 네변의길이가같은사각형 2 정삼각형 : 세내각의크기가같은삼각형 3 이등변삼각형 : 두변의길이가같은삼각형 4 평행사변형 : 두쌍의대변의길이가각각같은사각형 5 예각삼각형 : 한내각의크기가 90 보다크고 180 보다작은삼각

8. 8) 다음중용어의정의로옳은것은? 1 정사각형 : 네변의길이가같은사각형 2 정삼각형 : 세내각의크기가같은삼각형 3 이등변삼각형 : 두변의길이가같은삼각형 4 평행사변형 : 두쌍의대변의길이가각각같은사각형 5 예각삼각형 : 한내각의크기가 90 보다크고 180 보다작은삼각 1. 1) 수학익힘책문제풀기 중 2-2: 02. 삼각형의성질 ( 기본부터심화까지 ) 다음명제의역이참인지거짓인지를말하여라. 5. 5), 는자연수이고, 문장,, 가각각다음과같을때, 다음기호를명제로나타낼때, 참인지거짓인지를말하여라. : 는짝수이고 는홀수이다. : 는홀수이다. : 는홀수이다. ⑴ ⑵ ⑶ ⑴ 이면 이다. ⑵ 이면 이다. ⑶ 12의배수는 6의배수이다.

More information

미분기하학 II-16 복소평면의선형분수변환과쌍곡평면의등장사상 김영욱 (ÑñÁ) 강의양성덕 (zû ) 의강의록 Ø 'x! xxñ 2007 년 김영욱 (ÑñÁ) 강의양성덕 (zû ) 의강의록 (Ø 'x!) 미분기하 II 2007 년 1 / 26

미분기하학 II-16 복소평면의선형분수변환과쌍곡평면의등장사상 김영욱 (ÑñÁ) 강의양성덕 (zû ) 의강의록 Ø 'x! xxñ 2007 년 김영욱 (ÑñÁ) 강의양성덕 (zû ) 의강의록 (Ø 'x!) 미분기하 II 2007 년 1 / 26 미분기하학 II-16 복소평면의 김영욱 (ÑñÁ) 강의양성덕 (zû ) 의강의록 Ø 'x! xxñ 2007 년 김영욱 (ÑñÁ) 강의양성덕 (zû ) 의강의록 (Ø 'x!) 미분기하 II 2007 년 1 / 26 자, 이제 H 2 의등장사상에대해좀더자세히알아보자. Definition 선형분수변환이란다음형식의사상을뜻한다. Example f (z) = az +

More information

dnu.pdf

dnu.pdf ISODNU 1 ISO DNU/DNUL DNU DNUL 32 40 50 63 80 Gx G G Gy Gy G M10x1.25 M12x1.25 M16x1.5 M16x1.5 M20x1.5 M20x1.5 [mm] 19 21 23 23 30 30 4 [MPa] 1.2 [ C] 20 80 0.6MPa [N] 482 753 1178 1870 3015 4712 415 633

More information

벡터(0.6)-----.hwp

벡터(0.6)-----.hwp 만점을위한 수학전문가남언우 - 벡터 1강 _ 분점의위치벡터 2강 _ 벡터의일차결합 3강 _ 벡터의연산 4강 _ 내적의도형적의미 5강 _ 좌표를잡아라 6강 _ 내적의활용 7강 _ 공간도형의방정식 8강 _ 구의방정식 9강 _2014년수능최고난도문제 좌표공간에 orbi.kr 1 강 _ 분점의위치벡터 01. 1) 두점 A B 이있다. 평면 에있는점 P 에대하여 PA

More information

0 000., 000 0., 000-0., 000 0.666, 0 0.H6 0 0 0.0H8 0 06 07 08 9 09 6 00 0.H 0.H8 000 0.87, 0006-0.66, 0007 0.8, 0008 0.097, 0009 6, 0.H6 000,.HH 0 00

0 000., 000 0., 000-0., 000 0.666, 0 0.H6 0 0 0.0H8 0 06 07 08 9 09 6 00 0.H 0.H8 000 0.87, 0006-0.66, 0007 0.8, 0008 0.097, 0009 6, 0.H6 000,.HH 0 00 ~9 0~6 0 0 0 7 0 0 0 06 6 07 6 08 69 09 78 0 8 9 0 0 0 000., 000 0., 000-0., 000 0.666, 0 0.H6 0 0 0.0H8 0 06 07 08 9 09 6 00 0.H 0.H8 000 0.87, 0006-0.66, 0007 0.8, 0008 0.097, 0009 6, 0.H6 000,.HH 0

More information

PowerPoint Presentation

PowerPoint Presentation 5 불대수 Http://RAIC.kunsn..kr 2 학습목표 마스터제목스타일편집 기본논리식의표현방법을알아본다. 불대수의법칙을알아본다. 논리회로를논리식으로논리식을논리회로로표현하는방법을알아본다. 곱의합 (SOP) 과합의곱 (POS), 최소항 (minterm) 과최대항 (mxterm) 에대해알아본다. 01. 기본논리식의표현 02. 불대수법칙 03. 논리회로의논리식변환

More information

PowerPoint Presentation

PowerPoint Presentation 논리회로기초요약 IT CookBook, 디지털논리회로 4-6 장, 한빛미디어 Setion 진수 진수표현법 기수가 인수, 사용. () = +. = 3 () () + + () +. () + + + () +. + () + - () +. + - () + -3 + -4 Setion 3 8 진수와 6 진수 8진수표현법 에서 7까지 8개의수로표현 67.36 (8) = 6

More information

(001~007)수능기적(적통)부속

(001~007)수능기적(적통)부속 0 6 06. C : k d=k+c k «+-, : «d= «± +C + =- : d=: ;[!; d=l +C : kf()d=k: f()d k : { f()+g()} d=: f()d+: g()d : { f()-g()} d=: f()d-: g()d : si d=-cos +C : cos d=si+c 008 : sec d=ta +C : cosec d=-cot +C

More information

2013 학년도수학성취도측정시험 (2013학년도수시모집및외국인특별전형합격자대상 ) 2012년 12월 18일, 고사시간 90분 1번부터 11번까지는단답형이고, 12번부터 16번까지는서술형입니다. 답안지는깨끗한글씨로바르게작성하되, 단답형은답만쓰고, 서술형은풀이과정과답을명시

2013 학년도수학성취도측정시험 (2013학년도수시모집및외국인특별전형합격자대상 ) 2012년 12월 18일, 고사시간 90분 1번부터 11번까지는단답형이고, 12번부터 16번까지는서술형입니다. 답안지는깨끗한글씨로바르게작성하되, 단답형은답만쓰고, 서술형은풀이과정과답을명시 학년도수학성취도측정시험 (학년도수시모집및외국인특별전형합격자대상 년 월 8일, 고사시간 9분 번부터 번까지는단답형이고, 번부터 번까지는서술형입니다. 답안지는깨끗한글씨로바르게작성하되, 단답형은답만쓰고, 서술형은풀이과정과답을명시하시오. 총배점은 점이고, 각문항의배점은, 기본문제 (-번 각 점, 발전문제 (7-번 각 7점, 심화문제 (4번-번 각 점입니다. x x

More information

mathna_hsj.hwp

mathna_hsj.hwp 2008 학년도 6 월모의평가 ( 수리영역 - 가형 ) 정답및해설 1. 4 4 4. 2. 로놓으면 ᄀ - ᄂ 양변을제곱하면 3. 5 따라서 방정식ᄀ의근은이다. 일때 ( 분모 ) ( 분자 ) 이어야한다. 따라서 따라서 두식ᄀ ᄂ을동시에만족하는실수의값은구하는합은 ( 준식 ) 5 5. 는최고차항의계수가 1인삼차함수 로놓으면 - 1 - 따라서 ㄷ. 3 < 다른풀이

More information

7. 다음그림과같이한변의길이 가 4 6 인마름모의넓이를구 하여라. 10. 다음그림과같이모선의길이가 6 cm 인원뿔의밑면의 둘레의길이가 6π cm 일때, 원뿔의높이와부피를구한 것은? 1 6 cm, 6 π cm 6 cm, 6π cm 8. 다음과같이한변의길이가 8 인정육 면

7. 다음그림과같이한변의길이 가 4 6 인마름모의넓이를구 하여라. 10. 다음그림과같이모선의길이가 6 cm 인원뿔의밑면의 둘레의길이가 6π cm 일때, 원뿔의높이와부피를구한 것은? 1 6 cm, 6 π cm 6 cm, 6π cm 8. 다음과같이한변의길이가 8 인정육 면 . 단원테스트 범위 : 피타고라스의정리 피타고라스의정리의활용 50 문항 / 저반 : 이름 : 출제자 : 박지연. 1. 다음그림에서 x 의값으로적절한것은? 4. 세변의길이가 6 cm, 5 cm, 10 cm 인삼각형은어떤삼 각형인가? 1 직각삼각형 이등변삼각형 직각이등변삼각형 4 예각삼각형 5 둔각삼각형 1 9 9 9 4 4 9 5 5 9. 삼각형의세변의길이가다음보기와같을때직각삼각

More information